You are on page 1of 397

f3esi Seller

Wore than 10,000 copies sold Hit now


SELF ASSESSMENT & REVIEW

Thoroughly Revised & Updated Edition Including


Latest Exam Pattern Questions and Image Based Questions

New Features
New Layout
All Chapters Thoroughly Revised
and Edited
NEET Pattern Questions With
Explanations
FMGE & DNB Questions With
Explanations
Explanatory Questions &
Answers of AIIMS (2000-2013),
PGI (2000-2013) and ALL India
(2000-2011)
5 th Hot Topic-Snoring & Sleep Apne<
Edition Color Plates with Important
Illustrations & Instruments
Self Assessment and Review

Fifth Edition -

SAKSHi ARORA •

Faculty of Leading PG and FMGE Coachings


MBBS "Gold Medalist" (GSVM, Kanpur)
DGO (MLNMC, Allahabad)
• India •

J A Y P E E

JAYPEE BROTHERS MEDICAL PUBLISHERS (P) LTD


New Delhi • London • Philadelphia • Panama

(JAYPEE) J a y p e e B r o t h e r s Medical P u b l i s h e r s (P) Ltd

Headquarters
Jaypee Brothers Medical Publishers (P) Ltd
4838/24, Ansari Road, Daryaganj
New Delhi 110 002, India
Phone:+91-11-43574357
Fax:+91-11-43574314
Email: jaypee@jaypeebrothers.com

Overseas Offices
J.P. Medical Ltd Jaypee-Highlights Medical Publishers Inc
83 Victoria Street, London City of Knowledge, Bid. 237, Clayton
SW1H OHW (UK) Panama City, Panama
Phone:+44-2031708910 Phone:+1 507-301-0496
Fax: +02-03-0086180 Fax:+1 507-301-0499
Email: info@jpmedpub.com Email: cservice@jphmedical.com
Jaypee Medical Inc Jaypee Brothers Medical Publishers (P) Ltd
The Bourse 17/1 -B Babar Road, Block-B, Shaymali
111 South Independence Mall East Mohammadpur, Dhaka-1207
Suite 835, Philadelphia, PA 19106, USA Bangladesh
Phone:+1 267-519-9789 Mobile: +08801912003485
Email: joe.rusko@jaypeebrothers.com Email: jaypeedhaka@gmail.com
Jaypee Brothers Medical Publishers (P) Ltd
Bhotahity, Kathmandu, Nepal •

Phone: +977-9741283608
Email: Kathmandu@jaypeebrothers.com •

Website: www.jaypeebrothers.com
Website: www.jaypeedigital.com

© 2014, Jaypee Brothers Medical Publishers

The views and opinions expressed in this book are solely those of the original contributor(s)/author(s) and do not necessarily represent those of editor(s)
of the book.
All rights reserved. No part of this publication may be reproduced, stored or transmitted in any form or by any means, electronic, mechanical, photocopying,
recording or otherwise, without the prior permission in writing of the publishers.
All brand names and product names used in this book are trade names, service marks, trademarks or registered trademarks of their respective owners.The
publisher is not associated with any product or vendor mentioned in this book.
Medical knowledge and practice change constantly. This book is designed to provide accurate, authoritative information about the subject matter
in question. However, readers are advised to check the most current information available on procedures included and check information from the
manufacturer of each product to be administered, to verify the recommended dose, formula, method and duration of administration, adverse effects and
contraindications. It is the responsibility of the practitioner to take all appropriate safety precautions. Neither the publisher nor the author(s)/editor(s)
assume any liability for any injury and/or damage to persons or property arising from or related to use of material in this book.
This book is sold on the understanding that the publisher is not engaged in providing professional medical services. If such advice or services are required,
the services of a competent medical professional should be sought.
Every effort has been made where necessary to contact holders of copyright to obtain permission to reproduce copyright material. If any have been
inadvertently overlooked, the publisher will be pleased to make the necessary arrangements at the first opportunity.

Inquiries for bulk sales may be solicited at: jaypee@jaypeebrothers.com

Self Assessment and Review: ENT

First Edition: 2010


Second Edition: 2011
Third Edition: 2012
Fourth Edition: 2013
Fifth Edition: 2014

ISBN: 978-93-5152-318-5
Typeset at JPBMP typesetting unit
Printed at Rajkamal Electric Press, Plot No.2, Phase-IV, Kundli, Haryana.
'Dedicated fo
SAI BABA
Just sitting here reflecting on where I am and where I started I could not have done
it without you Sai baba.. I praise you and love you for all that you have given me...
and thank you for another beautiful day ... to be able to sing and praise
you and glorify you .. you are my amazing god

t
»
Preface
"It can be very difficult to sculpt the idea that you have in mind. If your idea doesn't match the shape of the stone, your idea may have to change
because you have to accept what is available is in the rock.... Sometimes thinking about the carving takes longer than carving itself."
F e v e r e i r o 1 9 9 9 in Arctic Spirit

Dear Students,
I w i s h t o e x t e n d m y t h a n k s t o all o f y o u f o r y o u r o v e r w h e l m i n g response t o all t h e f o u r editions o f m y b o o k . I a m e x t r e m e l y d e l i g h t e d by
t h e w o n d e r f u l response s h o w n by t h e readers for t h e 4 t h e d i t i o n a n d p r o v i n g it again as t h e bestseller b o o k o n t h e subject.Thanks o n c e
again f o r t h e i n n u m e r a b l e e-mails y o u have sent in a p p r e c i a t i o n o f t h e b o o k .
The year 2013 was very significant for m e for n o t o n l y t h e g r o w t h w h i c h I saw in m y books b u t also in professional f r o n t . It was an
e x t r e m e h o n o r for m e t o w o r k a n d h e l p legends like Dr. I.B Singh in revision o f "Text b o o k of N e u r o a n a t o m y (revised r e p r i n t 8 t h e d i t i o n
and 9 t h edition-yet t o be released), Text b o o k o f E m b r y o l o g y (10th e d i t i o n ) a n d Dr. Konar, in D.C. Dutta's in Text books o f Obstetrics a n d
Gynecology.
It n o w gives m e i m m e n s e pleasure t o share w i t h y o u t h e n e w (5th) e d i t i o n o f t h e b o o k .
There is a lot o f apprehension a m o n g students a b o u t t h e change in p a t t e r n o f t h e e x a m i n a t i o n u n d e r 'National Eligibility Cum Entrance
Test (NEET). As I have always said, d o n o t panic r e g a r d i n g this issue because t h e syllabus/the subject is still t h e same. If y o u u n d e r s t a n d
t h e subject, t h e n y o u can answer questions asked in any f o r m a t . To m a k e t h e subject m o r e clear a n d f o r easy u n d e r s t a n d i n g in t h e 5 t h
e d i t i o n , I have i n t r o d u c e d m a n y n e w features.

Salient Features of 5th Edition

Recent solved papers of AIIMS a n d PGI w i t h f u l l y e x p l a i n e d , referenced, a u t h e n t i c a t e d answers are i n c l u d e d at e n d .


For t h e sake o f FMGE students, I have i n c l u d e d FMGE 2013 questions w i t h t h e i r answers.
All references are f r o m Dhingra 6 t h e d i t i o n ,
t h e entire t h e o r y has been concised a n d g i v e n in a n e w layout.
All chapters have b e e n t h o r o u g h l y revised a n d u p d a t e d .
N e w tables a n d f l o w charts have b e e n a d d e d w h e r e v e r necessary.
M a n y n e w d i a g r a m s have been a d d e d , for w h i c h I t h a n k Shri Jitendar Pal Vij ( g r o u p chairman o f Jaypee Brothers Medical Publishers)
f o r a l l o w i n g m e t o use illustrations f r o m e m i n e n t ENT books o f Jaypee Publications.
N e w Questions o f NEET p a t t e r n a n d DNB p a t t e r n have been a d d e d a l o n g w i t h explanations.
In t h e n e w e d i t i o n o f Dhingra, a c h a p t e r has been a d d e d o n s n o r i n g a n d sleep apnea, So f o r t h e convenience o f students, I have a d d e d
s n o r i n g a n d sleep apnea in t h e f r o m o f Hot Topic.
• A section o n color plates has been a d d e d w h i c h is a c o m p i l a t i o n o f i m p o r t a n t figures and all i n s t r u m e n t s used in ENT. This w i l l h e l p
t h e s t u d e n t t o solve any figure based q u e s t i o n o n t h e subject.
I h o p e all o f y o u w i l l appreciate t h e changes a n d accept t h e b o o k in this n e w f o r m a t , like y o u have d o n e f o r t h e previous e d i t i o n s .
R e m e m b e r t h e r e is n o s u b s t i t u t e t o t h e o r y books, b u t h o p e f u l l y y o u w i l l find all relevant t h e o r y in this user-friendly b o o k o f ENT. I m u s t
a d m i t hereby t h a t d e s p i t e keeping an eagle's eye f o r any inaccuracy r e g a r d i n g factual i n f o r m a t i o n or t y p o g r a p h i c a l errors, s o m e mistakes
m u s t have crept in inadvertently. You are requested t o c o m m u n i c a t e these errors a n d send your valuable suggestions f o r t h e i m p r o v e m e n t
o f t h i s b o o k . Your suggestions, a p p r e c i a t i o n a n d criticism are m o s t w e l c o m e .

New Delhi Dr Sakshi Arora H a n s


May 2014 drsakshiarora@gmail.com


Acknowledgements
Everything what we are is the outcome of a series of factors and circumstances, in addition to ourselves.
It w o u l d n o t be fair, t h e r e f o r e , t o i g n o r e t h e p e o p l e w h o have played t h e p a r t in m a k i n g m e k n o w n as"Dr Sakshi A r o r a " a n d t o w h o m
I a m deeply grateful.

My Teachers

> Dr M a n j u V e r m a (Prof & Head, Gynae & Obs, MLN MC, Allahabad) a n d Dr G a u r i G a n g u l i (Prof & Ex- HEAD, Gynae & Obs, MLNMC,
Allahabad) f o r t e a c h i n g m e t o focus o n t h e basic concepts o f any subject.

My F a m i l y

> Dr P a n k a j Hans, m y better-half w h o has always been a m o u n t a i n o f s u p p o r t and w h o is t o a large measure, responsible f o r w h a t
I a m t o d a y . His c a l m , consistent a p p r o a c h t o w a r d s any w o r k , brings s o m e (only some) calmness in m y hasty, hyperactive, a n d
inconsistent nature.
> My F a t h e r : Shri H.C. A r o r a , w h o has o v e r c o m e all o d d s w i t h his discipline, h a r d w o r k , a n d p e r f e c t i o n .
> My M o t h e r : Smt. S u n i t a A r o r a , w h o has always believed in m y abilities a n d s u p p o r t e d m e in all m y ventures - b e it a u t h o r i n g a
b o o k or t e a c h i n g .
> My in laws (Hans family): For h.appily a c c e p t i n g m y m a i d e n s u r n a m e ' A r o r a ' a n d t a k i n g p r i d e in all achievements.
> My B r o t h e r s : Mr B h u p e s h A r o r a a n d S a c h i t A r o r a , w h o e n c o u r a g e d m e t o w r i t e books a n d have always t h o u g h t ( w r o n g
a l t h o u g h ) t h e i r sister is a perfectionist.
> My D a u g h t e r : S h r e y a H a n s (A priceless g i f t o f g o d ) . For a c c e p t i n g m y b o o k s a n d w o r k as her siblings a n d is n o w s h o w i n g signs
o f intense sibling rivalry!!

My C o l l e a g u e s : I a m grateful t o all m y seniors, f r i e n d s a n d colleagues o f past a n d present f o r t h e i r m o r a l s u p p o r t .


o Dr M a n o j Rawal o Dr Pooja A g g r a w a l o Dr Parul A g g r a w a l Jain
o Dr Ruchi Aggrawal o Dr Shalini T r i p a t h i o Dr Kushant G u p t a
o DrParminderSehgal o DrAmitJain o D r S o n i k a Lamba Rawal

D i r e c t o r s o f P G E n t r a n c e c o a c h i n g , w h o h e l p e d m e in realizing m y p o t e n t i a l as an academician (and beared w i t h m y s u d d e n resignation


f r o m teaching).
> Mr R a j e s h S h a r m a : Director PGDIAMS/DIAMS c o a c h i n g i n s t i t u t e
> Dr S u s h a n t a B h a n j a : Director PGEI c o a c h i n g i n s t i t u t e
> Dr M u t h u K u m a r : Director Pulse PG (Kanpur) c o a c h i n g i n s t i t u t e

M y P u b l i s h e r s - J a y p e e B r o t h e r s M e d i c a l P u b l i s h e r s (P) L t d
> Shri J i t e n d a r P Vij (Group Chairman) f o r b e i n g m y role m o d e l . His drive t o reach p e r f e c t i o n a n d never-say d i e a t t i t u d e has always
inspired m e t o give t h e best
> Mr A n k i t V i j ( M a n a g i n g Director) f o r h a v i n g c o n s t a n t f a i t h in m e a n d all m y endeavours
> Mr B h u p e s h Arora (General M a n a g e r Publishing) for never b e i n g a b r o t h e r w h e n it comes t o d e l i v e r i n g o f books.
> Dr Mrinalini B a k s h i , Dr S w a t i S i n h a a n d Ms Nitasha A r o r a f o r t h e i r c o n s t a n t s u p p o r t a n d f o r a c c o m p l i s h i n g t h e herculean task
o f u n d e r s t a n d i n g m y h a n d w r i t i n g a n d e d i t i n g t h e entire b o o k
> The e n t i r e staff o f Jaypee Brothers, especially Preeti Parashar ( A u t h o r Co-ordinator), M r Prabhat Ranjan, a n d M r Phool Kumar, M r
Sachin D h a w a n a n d M r Pradeep Kumar
> M r s S e e m a D o g r a for a c c e p t i n g a n d p u t t i n g o n paper m y w e i r d ideas o f cover designs.

L a s t b u t not t h e l e a s t —
All S t u d e n t s / R e a d e r s for sharing t h e i r invaluable, c o n s t r u c t i v e criticism for i m p r o v e m e n t o f t h e b o o k .
M y sincere t h a n k s t o all FMGE/ UG/PG students, present a n d past, f o r t h e r e t r e m e n d o u s s u p p o r t , w o r d s o f a p p r e c i a t i o n rather I s h o u l d
say emails o f e n c o u r a g e m e n t a n d i n f o r m i n g m e a b o u t t h e corrections, w h i c h has helped m e in t h e b e t t e r m e n t o f t h e b o o k .

Dr Sakshi Arora Hans


drsakshiarora@gmail.com
Contents

SECTION l:NOSE
1. A n a t o m y a n d P h y s i o l o g y o f N o s e 3
2. Diseases o f E x t e r n a l N o s e a n d Nasal S e p t u m 11
3. G r a n u l o m a t o u s D i s o r d e r s o f N o s e , Nasal P o l y p s a n d F o r e i g n B o d y i n N o s e 20
4. I n f l a m m a t o r y D i s o r d e r s o f Nasal C a v i t y 30

5. Epistaxfs 38
6A. Diseases o f Paranasal S i n u s — S i n u s i t i s 47
6B. Diseases o f Paranasal S i n u s — S i n o n a s a l T u m o r 62

7. O r a l C a v i t y 69

SECTION III: PHARYNX


8. A n a t o m y o f P h a r y n x , Tonsils a n d A d e n o i d s 97
9. H e a d a n d N e c k S p a c e I n f l a m m a t i o n 109
10. Lesions o f N a s o p h a r y n x a n d H y p o p h a r y n x i n c l u d i n g T u m o r s o f P h a r y n x 115

11. HotTbpics 127

SECTION IV: LARYNX


12. A n a t o m y o f L a r y n x , C o n g e n i t a l Lesions o f L a r y n x a n d S t r i d o r 133
13. A c u t e a n d Chronic I n f l a m m a t i o n o f Larynx, Voice a n d Speech Disorders '. 148
14. V o c a l C o r d Paralysis 161
15. T u m o r o f L a r y n x 169

16. A n a t o m y o f Ear 187


17. P h y s i o l o g y o f Ear a n d H e a r i n g 213
18. A s s e s s m e n t o f H e a r i n g Loss 224
19. H e a r i n g Loss 241
20. Assessment o f Vestibular Function 250
21. Diseases o f E x t e r n a l Ear 261
22. Diseases o f M i d d l e Ear 271
23. M e n i e r e ' s Disease 300
Viii | Contents

24; Otosclerosis 307


25. Facial N e r v e a n d its Lesions 316
26. Lesion o f Cerebellopalatine A n g l e a n d Acoustic N e u r o m a , 328
27. G l o m u s T u m o r a n d O t h e r T u m o r s o f t h e Ear 334
28. Rehabilitative M e t h o d s 339

29. Miscellaneous 343

SECTION VI: OPERATIVE PROCEDURE


30. I m p o r t a n t Operative Procedures 351

i SECTION VII: RECENT LATEST PAPERS


PGI-Nov 2012 363

AIIMS-May2013 365
PGI-May 2013 367
PGI-May 2012 .'. 370

SECTION VIII: COLOR PLATES


Color Plates iii-xvi

• •

. . . ; . . . . . . .


NOSE AND PARANASAL SINUSES

1. Anatomy and Physiology of Nose

2. Diseases of External Nose and Nasal Septum

3. Granulomatous Disorders of Nose, Nasal Polyps and Foreign Body in Nose


4. Inflammatory Disorders of Nasal Cavity
5. Epistaxis

6. Diseases of Paranasal Sinus


A. Sinusitis
B. Sinonasal Tumor
-

CHAPTER
-

Anatomy and Physiology of Nose

ANATOMY OF NOSE
Nose consists of: External nose | NASAL VESTIBULE
Nasal vestibule
Nasal cavity It is a skin lined entrance t o t h e nasal cavity.
Contains hair follicles, hair (called Vibrissae), sebaceous glands
and sweat glands.
| EXTERNAL NOSE
Furuncle o f nose is due t o staphylococcal infection o f hair follicle.

It is a t r i a n g u l a r p y r a m i d w i t h an osteocartilaginous f r a m e w o r k :
Upper 1 /3rd part is b o n y and Lower 2/3rd p a r t is cartilaginous. | NASAL CAVITY

• Bony part consists o f 2 nasal b o n e s w h i c h u n i t e w i t h each


Each nasal cavity has a lateral wall, medial wall, a roof and floor.
o t h e r in t h e m i d l i n e , w i t h t h e f r o n t a l b o n e superiorly at t h e
nasofrontal suture and laterally w i t h t h e f r o n t a l process o f t h e Lateral Nasal Wall
maxilla at t h e nasomaxillary suture.
• It has 3 b o n y projections called as t u r b i n a t e s or conchae.
• Cartilaginous part consists o f u p p e r lateral cartilages: (Hyaline • From b e l o w u p w a r d t h e y are inferior, m i d d l e a n d s u p e r i o r
cartilage) w h i c h articulate w i t h t h e nasal bones, t h e f r o n t a l turbinates.
processes o f maxilla and t h e lower lateral cartilages. • T h e inferior t u r b i n a t e is a s e p a r a t e b o n e , w h i l e rest of t h e
• Limen nasi or Nasal valve or Limen vestibuli is t h e j u n c t i o n b e - t u r b i n a t e s a r e a part of e t h m o i d a l l a b y r i n t h .
0

• Below and lateral t o each t u r b i n a t e is t h e corresponding meatus.


t w e e n t h e u p p e r a n d l o w e r lateral cartilage a n d is t h e site for
• Sometimes a fourth turbinate is also present just above superior
i n t e r c a r t i l a g i n o u s incision. turbinate.This fourth turbinate is known as supreme turbinate'"™ . 11

• Lower lateral cartilages have t w o parts: Supreme (fourth) t u r b i n e is f o u n d in 3 0 % of population.


Lateral crura (which forms the ala) a n d
Inferior Meatus
Medial crura (which forms the columella).
• M u s c l e s of e x t e r n a l n o s e are procerus a n d nasalis consisting • It is t h e largest meatus.
• Its highest poirit is t h e j u n c t i o n o f anterior a n d m i d d l e 1/3rd.
o f compressor a n d dilator naris. These muscles are supplied by
• Nasolacrimal d u c t opens in t h e inferior meatus just anterior t o its
facial nerve.
highest p o i n t (itisclosedbyamucosalflapcalledHasner'svalve).
• N e r v e s u p p l y o f t h e e x t e r n a l n o s e : Nose receives its sensory The direction o f Nasolacrimal d u c t is d o w n w a r d , backward a n d
s u p p l y f r o m t h e o p h t h a l m i c nerve (which s u p p l y t i p o f nose) laterally f r o m Lacrimal sac t o nose w i t h a l e n g t h o f 1.8 cms.
a n d t h e maxillary division o f t r i g e m i n a l nerve (which supplies
Middle Meatus
side a n d ala o f nose).
L y m p h a t i c s : drain i n t o s u b m a n d i b u l a r a n d p r e a u r i c u l a r g r o u p
o f l y m p h nodes. Lies lateral to the middle turbinate
Structures of importance in middle meatus
- Hiatus semilunaris
- Ethmoidal infundibulum
Nose is made o f 4 cartilages (mainly hyaline)-3 paired (upper
- Anterior/posterior fontanelle
lateral, lower lateral or alar cartilage, sesamoid cartilage) and one
- Uncinate process
unpaired (i.e. septal) cartilage.
- Bulla ethmoidale
SECTION I Nose and Paranasal Sinuses'

• Hiatus s e m i l u n a r i s : It is a semilunar g r o o v e w h i c h leads a n - Contd...


teriorly t o the ethmoidal i n f u n d i b u l u m .
• Ethmoidal infundibulum: Other bony minor contributors are:
It is a short passage at the anterior end of the hiatus. • Crest of nasal bones - nasal spine of frontal bone
Frontal sinus, maxillary sinus and anterior ethmoidal sinuses Rostrum of sphenoid • crest of palatine bone
drain into it in that order f r o m anterior t o posterior. Rostrum of sphenoid - crest of palatine bone
• Anterior/posterior f o n t a n e l l e : Nasal spine of maxilla
Are m e m b r a n o u s areas b e t w e e n t h e inferior t u r b i n a t e
a n d u n c i n a t e process.
Accessory ostia are f o u n d mostly in t h e posteriorfontanelle. Amongst all—the nasal septum is mainly formed vomer,
Uncinate process: perpendicular plate of ethmoid and septal cartilage.
T h i n , b o n y structure.
Runs anterosuperiorly t o posteroinferiorly. B l o o d S u p p l y o f t h e N a s a l S e p t u m ( F l o w c h a r t 1.1)
It articulates with the ethmoidal process of inferior turbinate.
It partly covers the opening o f maxillary sinus.
Little's area is the most vascular area on the anteroinferior part
• Bulla e t h m o i d a l i s :
of nasal septum. Branches of anterior ethmoidal, sphenopalatine
It is a r o u n d p r o m i n e n c e f o r m e d by t h e b u l g i n g o f M i d d l e
(artery of epitaxis), superior labial and greater palatine and
E t h m o i d a l Sinuses w h i c h o p e n o n or a b o v e it.
their corresponding veins anastomose here t o form a vascular
Superior Meatus plexus called "Kiesselbach plexus". Blood vessels at this site lack
cushioning effect and are liable to trauma causing epistaxis.
Posterior e t h m o i d a l sinus o p e n i n t o it.

Sphenoethmoidal Recess F l o w c h a r t 1.1: Blood s u p p l y o f t h e nasal s e p t u m


Lies a b o v e t h e superior t u r b i n a t e a n d receives t h e o p e n i n g o f t h e Internal carotid artery
s p h e n o i d sinus.
X
Ophthalmic artery
Osteomeatal Complex Area (Picadli's Circle)
T
• It is t h a t area o f m i d d l e meatus w h e r e sinus ostia o f anterior
g r o u p o f sinuses ( f r o n t a l / a n t e r i o r e t h m o i d a l / m a x i l l a r y ) are
X
Anterior ethmoidal artery
X
Posterior ethmoidal artery
s u r r o u n d e d b y uncinate process, e t h m o i d a l i n f u n d i b u l u m a n d
bulla e t h m o i d a l i s . External carotid artery
• Structures c o n t r i b u t i n g t o its f o r m a t i o n are:
X
Facial artery
X
Maxillary artery
Uncinate Bulla Ethmoidal Hiatus Frontal

Process Ethmoidalis Infundibulum Semilunaris recess


X
Superior labial A r x
Greater palatine Sphenopalatine
Even a m i n o r p a t h o l o g y in this area can lead t o secondary sinusitis
in major sinuses b y o b s t r u c t i o n t o sinus ostia..;. This is t h e site o f
Floor of Nose
pathogenesis o f sinusitis.
E t h m o i d a l air cells: They give a H o n e y c o m b appearance. Formed b y palatine process o f maxilla a n d h o r i z o n t a l process o f
palatine b o n e .
Anterior Middle Posterior

Nerve S u p p l y of Nasal Cavity


2-8 in n u m b e r 1-8 in n u m b e r J • Nasopalatine/Branches o f sphenopalatine ganglia supply
majority of the septal area.
o O n o d i cells: Are t h e m o s t posterior e t h m o i d a l air cells is s u r g i -
• A n t e r i o r e t h m o i d a l nerve supplies t h e anterosuperiorpart.
cally i m p o r t a n t as it is related t o o p t i c nerve in its lateral w w a l l . • Anterior superior alveolar nerve supplies anteroinferior portion.
• Hallercells: Anterior ethmoidal air cells present in t h e o r b i t a l floor. • General sensory nerves derived f r o m t h e branches o f t r i g e m i n a l
nerve are d i s t r i b u t e d t o t h e w h o l e o f t h e lateral w a l l .
Medial Wall of Nose/Nasal S e p t u m S e c r e t o m o t o r s u p p l y o f nose is t h r o u g h t h e v i d i a n nerve (also
Parts k/a nerve o f p t e r y g o i d ) .
• Vidian nerve is t h e nerve o f p t e r y g o i d canal f o r m e d by t h e
Columellar s e p t u m u n i o n o f greater superficial petrosal nerve a n d d e e p pterosal
Medial crura of the alar cartilage nerve. This is t h e m a i n p a r a s y m p a t h e t i c s u p p l y o f nose. In
v a s o m o t o r rhinitis w h e r e t h e r e is a n i m b a l a n c e b e t w e e n s y m -
Membranous septum pathetic parasympathetic system, o n e o f t h e surgical o p t i o n s
Double layer of skin with no bony/cartilaginous support is Vidian n e u r e c t o m y .

S e p t u m proper Lymphatic Drainage of Nasal Cavity


• Septal cartilage • Lymphatics f r o m external n o s e a n d anteriorpart o f nasal cavity
• Perpendicular plate of ethmoid drain i n t o submandibular lymph nodes w h i l e those f r o m t h e rest
• Vomer of nasal cavity drain i n t o upper jugular nodes either directly or
Contd... t h r o u g h the retropharyngeal node.
CHAPTER 1 Anatomy and Physiology of Nose

IMPORTANT CLINICAL VIGNETTES

| D A N G E R O U S A R E A O F F A C E ( F I G . 1.1) | NASAL CAVITY MUCOSA

Dangerous area of face includes upper lip and anterioinferior part of The mucosa of the nasal cavity is divided as (A) respiratory area
nose including t h e vestibule.This area freely communicates with the (B) olfactory area.
cavernous sinus through a set ofvalveless veins, anterior facial vein
and superiorophthalmic vein. Any infection of this area can thus travel Olfactory Area
intracranially leading to meningitis and cavernous sinus thrombosis.
Includes upper 1/3 rd of septum, cribriform plate, and lateral wall of nose
up to the superior turbinate covering an area of approximately 2-5 cm .
2

It has specialized non-cilated olfactory epithelium.


Olfactory area is called as dangerous area of nose as in olfactory
area, olfactory nerves carry sheaths of dura pia and arachnoid along
w i t h t h e m i n t o t h e nose w h i c h is a patent c o m m u n i c a t i o n w i t h t h e
s u b a r a c h n o i d space. An i n f e c t i o n o f t h e o l f a c t o r y area can travel
intracranially t o cause meaningitis, hence called as d a n g e r o u s area
o f nose.

Rhinorrhea

Respiratory Area

It is covered by pseudostratified c o l u m n a r e p i t h e l i u m (Schnederian


membrane).
Fig. 1.1: Dangerous area o f face Flow chart 1.2 shows clinical diagnosis of rhinorrhea.

F l o w c h a r t 1.2: Clinical diagnosis o f rhinorrhea

• Nasal discharge

r X
Clear Purulent

X X Unilateral
Unilateral Bilateral Bilateral
X
CSF rhinorrhea

X El 1
More than 10 days More than 10 days
4
Recurrent (seasonal) Endoscopic examination
with sneezing, itching Common cold
1
X r
Mass Endoscopic
Allergic rhinitis
examination
Recurrent use
of nesal drops
X
Foreign body
1
Tumors
normal

X
Rhinitis mediacamentosa No mass
Symptoms
X
Perennial rhinitis during stress
Rhinosinusitis

—• Allergic rhinitis Rule out gastroesophageal


reflux disease (GERD)
u Vasomotor rhinitis
Advice CT scan

1
Abnormal treat Normal -
accordingly reassure
SECTION I Nose and Paranasal Sinuses

QUESTIONS

i, F r o n t o n a s a l duct o p e n s into: [PGI 98] 17. Direction of n a s o l a c r i m a l d u c t is: [AI99]


a. Inferior meatus b. Middle meatus a. Downward, backward and medially
c. Superior meatus d. Inferior turbinate b. Downward, backward and laterally
2. Frontal s i n u s drain into: [PGI 97, 98] c. Downward, forward and medially
a. Superior meatus Inferior meatus d. Downward, forward and laterally
c. Middle meatus Ethmoid recess 18. W h i c h of the following b o n e s do not contribute the nasal
3. P a r a n a s a l s i n u s o p e n i n g in m i d d l e m e a t u s : [PGI 03,98] septum: [Al 03]
a. Maxillary b. Anterior e t h m o i d a. Sphenoid b. Lacrimal
c. Posterior e t h m o i d 'd. Frontal
c. Palatine d. Ethmoid
e. Sphenoid
1 9 . Quadrilateral cartilage is a t t a c h e d to all except:[DNB01]
T h e maxillary sinus o p e n s into m i d d l e m e a t u s at the level
a. Ethmoid b. Vomer
of: (DNB 02)
c. Sphenoid d. Maxilla
a. Hiatus semulinaris b. Bulla ethmoidalis
2 0 . All t h e s e structures are f o u n d in t h e lateral n a s a l w a l l
c. I n f u n d i b u l u m d. None of the above
except: [MP 07]
All d r a i n s into m i d d l e m e a t u s except: [DNB 02]
a. Superior turbinate b. Vomer
a. Lacrimal d u c t b. Maxillary sinus
c. Aggernasi d. Hasner'svale
c. Frontal sinus d. Ethmoidal sinus
21. Nasal v a l v e is f o r m e d by: [MP 08]
Hiatus s e m i l u n a r i s is p r e s e n t i n : [CUPGEE02]
a. Upper lateral cartilage
a. Superior meatus b. Middle meatus
c. Inferior meatus d. Spenoethmoidal recess b. Lower lateral cartilage
7. Bulla e t h m o i d a l i s is s e e n in: [AIIMS 92] c. Lower end of upper lateral cartilage
a. Superior meatus b. Inferior meatus d. Upper end of lower lateral cartilage
c. Middle meatus d. Sphenoethmoidal recess 2 2 . O n o d i cells a n d Haller cells are s e e n in relation to
S p h e n o i d a l sinus o p e n s into: [Kerala 98] a. Optic nerve and floor of orbit [AIIMS Nov 09]
a. Inferior meatus b. liddle meatus b. Optic nerve and internal carotid artery
c. Superior meatus d. Sphenoethmoidal recess c. Optic nerve and nasolacrimal d u c t
O p e n i n g of posterior e t h m o i d s i n u s is i n : [Jharkhand 06] d. Orbital floor and nasolacrimal d u c t
a. Middle turbinate b. Superior turbinate 2 3 . O s t e o m e a t a l c o m p l e x (QMC) c o n n e c t s : [MH02]
c. Inferior turbinate d. None a. Nasal cavity w i t h maxillary sinus
10. N a s o l a c r i m a l duct o p e n s into: [MAHE05] b. Nasal cavity w i t h sphenoid sinus
a. Superior meatus Middle meatus c. The t w o nasal cavities
c. Inferior meatus Sphenopalatine recess d. Ethmoidal sinus w i t h ethmoidal bulla
11. Inferior t u r b i n a t e is a: [JIPMER 04] 24. Nasal m u c o s a is s u p p l i e d by: [A192]
a. Part of maxilla Part o f sphenoid a. Only external carotid artery
c. Separate bone Part o f e t h m o i d b. Only internal carotid
12. E t h m o i d b o n e forms A/E: [Bihar 05] c. Mainly external carotid artery
a. Superior turbinate Middle turbinate
d. Mainly internal carotid artery
c. Interior turbinate Uncinate process
25. During inspiration the m a i n current of airflow in a normal
1 3 . W h i c h of t h e following is k n o w n a s f o u r t h t u r b i n a t e
nasal cavity is t h r o u g h : [Al 07]
[UP01]
a. Middle part of t h e cavity in middle meatus in a
a. Posterior e t h m o i d cells
parabolic curve
b. Anterior e t h m o i d cells (Aggernasi)
b. Lower part of the cavity in t h e inferior meatus in a parabolic
c. Medial e t h m o i d cells
curve
d. Lateral e t h m o i d cells
c. Superior part of t h e cavity in the superior meatus
14. Turbinate t h a t articulates w i t h e t h m o i d is: [AP02]
a. Superior b. Middle d. Through olfactory area
c. Inferior d. All of t h e above 26. Function of mucociliary action of u p p e r respiratory tract
1 5 . E x t e r n a l n o s e is f o r m e d f r o m : [AP96] is: [Kerala 94]
a. 3 paired + 3 unpaired cartilages a. Protective
b. 2 paired + 2 unpaired cartilages b. Increase the velocity of inspired air
c. 3 p a i r e d + 4 unpaired c. Traps t h e pathogenic organisms in inspired air
d. 1 paired + 1 unpaired d. Has no physiological role
16. C h o a n a is: [TN03] 2 7 . Ciliary m o v e m e n t rate of nasal m u c o s a is: [UP 01]
a. Anterior nares b. Posterior nares a. 1 - 2 m m / m i n b. 2-5 m m / m i n
c. Tonsils d. Larynx c. 5-10 m m / m i n d. 10-12 m m / m i n
CHAPTER 1 Anatomy and Physiology of Nose

28. P a r o s m i a is: [MAHE01] 3 1 . N a s o l a c r i m a l d u c t o p e n s into w h i c h t u r b i n a t e :


a. Perversion o f smell sensation [RajPG09]
b. Absolute loss of smell sensation a. Superior b. Inferior
c. Decreased smellsensation c. Middle d. Lateral
d. Perception o f bad smell 3 2 . All o p e n into hiatus s e m i l u n a r i s except: [DNB2008]
29. Nasal s e p t u m is f o r m e d by all except: [DNB 2005] a. Posterior e t h m o i d sinus b. Anterior e t h m o i d sinus
a. Turbinate b. Vomer c. Frontal sinus d. Maxillary sinus
c. Palatine b o n e d. Maxilla 3 3 . Nasal cycle is the cyclical a l t e r n a t e n a s a l b l o c k a g e o c c u r -
3 0 . W h a t d r a i n s into inferior meatus:[DNB 2007, Raj PG 2009] ring: [DPG2009]
Nasolacrimal duct b. Maxillary sinus a. Every 6-12 hours b. Every 4-12 hours
Frontal sinus d. Ethmoidal sinus c. Every 6-8 hours d. Every 12-24 hours

EXPLANATIONS AND REFERENCES

1. A n s . is b i.e. Middle m e a t u s Ref. Logan Turner lOXh/edp 379; Dhingra 5th/edp 178,6th/ed 136,137;Mohan Bansalp 34,35
2. A n s . is c i.e. Middle m e a t u s Mohan Bansalp 37
3. A n s is a, b a n d d i.e. Maxillary, A n t e r i o r e t h m o i d ; a n d Frontal
4 . A n s . is a i.e. Hiatus s e m i l u n a r i s
5. A n s . is a i.e. L a c r i m a l d u c t
6. A n s is b i.e. M i d d l e m e a t u s
7. A n s is c i.e. Middle m e a t u s Ref. Dhingra5th/edpp 152,153;Tuli Ist/edpp 135-136;Logan Timer Wth/edp379;Mohan Bansalp34
M i d d l e m e a t u s lies b e t w e e n t h e m i d d l e a n d inferior t u r b i n a t e s a n d is i m p o r t a n t because o f t h e presence o f o s t e o m e a t a l c o m p l e x
in this area.

Part of lateral nasal wall Openings


Inferior meatus Nasolacrimal duct

Middle meatus Frontal sinus, Maxillary sinus, Anterior ethmoidal sinus


Superior meatus Posterior ethmoidal sinus

Sphenoethmoidal recess Sphenoid sinus


8. A n s . is d i.e. s p h e n o e t h m o i d a l recess Ref. Dhingra 5th/ed p 153,6th/edp 138; Mohan Bansal p 38
S p h e n o e t h m o i d recess is situated a b o v e t h e superior t u r b i n a t e a n d receives o p e n i n g o f s p h e n o i d a l sinus.
9. A n s is b i.e. S u p e r i o r t u r b i n a t e Ref. Dhingra 5th/ed 153,6th/edpi38
1 0 . A n s . is c i.e Inferior m e a t u s Ref. Dhingra 5th/edp 150,6th/edpi35
• Nasolacrimal d u c t opens i n t o inferior meatus b e l o w t h e level o f inferior t u r b i n a t e "
Nasolacrimal d u c t is g u a r d e d at its t e m p o r a l e n d by a mucosal valve k/a Hasner's valve
Frontonasal d u c t opens i n t o m i d d l e meatus.
-

Posterior e t h m o i d a l sinus o p e n s in t h e superior meatus w h i c h is b e l o w t h e superior t u r b i n a t e .


1 1 . A n s . is c i.e S e p a r a t e b o n e Ref. Dhingra 5th/ed p 150,6th/ed p 135; Tuli 1st/ed p 135,2nd/ed p 140
1 2 . A n s . is c i.e. Inferior t u r b i n a t e Ref. Dhingra 6th/edp 12; Tuli IsX/edp 135,2nd/edp 140
"The inferior turbinate is a separate bone, while rest of the turbinates are a part of ethmoidal labyrinth."
1 3 . A n s . is b i.e. A n t e r i o r e t h m o i d a l cells (Agger nasi)
F r i e n d s -1 haven't been able t o g e t a reference for this answer - b u t I a m p r e t t y sure a b o u t t h e answer itself.
1 4 . A n s . is c i.e Inferior Ref. Scotts Brown 7th/ed Vol2 p 1329; Dhingra 6th/edp 136
Friends here it is i m p o r t a n t t o read t h e q u e s t i o n - t h e q u e s t i o n is asking a b o u t a r t i c u l a t i o n w i t h e t h m o i d .
Its discussed in previous q u e s t i o n s :
M i d d l e t u r b i n a t e and superior t u r b i n a t e are a p a r t o f t h e e t h m o i d a l b o n e whereas inferior t u r b i n a t e articulates w i t h t h e e t h m o i d
b o n e , c o m p l e t i n g t h e m e d i a l wall o f nasolacrimal d u c t .
1 5 . A n s . b. 2 p a i r e d a n d 2 u n p a i r e d c a r t i l a g e Ref. Dhingra 5th/ed pp 149,150; 6th/ed p 134; Mohan Bansal p 30
E x t e r n a l n o s e is m a d e up of b o n y f r a m w o r k w h i c h f o r m s u p p e r t h i r d p a r t a n d c a r t i l a g i n o u s f o r m s lower two-third p a r t
framwork.
SECTION I Nose and Paranasal Sinuses

C a r t i l a g e s of n o s e :
• Paired u p p e r lateral nasal cartilages • •

• Paired l o w e r nasal cartilages


• Lesser alar (sesamoid) cartilages - 2 or m o r e in n u m b e r
• U n p a i r e d septal cartilage.
For e x a m p l e , t h e r e are either 3 paired a n d 1 u n p a i r e d cartilage or 2 paired a n d 2 u n p a i r e d cartilage because lesser alar (or sesamoid)
cartilages can be 2 or m o r e . In this q u e s t i o n , 2 paired a n d 2 u n p a i r e d cartilage is t h e m o r e close o p t i o n a n d is t h e answer.
16. A n s . is b i.e. Posterior n a r e s Ref. Turner 10th/ed p 4; Dhingra 5th/edp 150.6th/edp 135
Nasal c a v i t y
"Nasal fossae are t w o irregular cavities e x t e n d i n g f r o m t h e m u c o c u t a n e o u s j u n c t i o n w i t h t h e nasal vestibule in f r o n t (the anterior
nares) t o t h e j u n c t i o n w i t h t h e n a s o p h a r y n x b e h i n d (posterior nares or choanae)." Ref. Turner 10th/edp4
"Each nasal cavity communicates with the external through naris or nares and with nasopharynx through posterior nasal
aperture or choanal' — D h i n g r a 5th/ed p 150,6th/ed 135
1 7 . A n s . is b i.e. D o w n w a r d , b a c k w a r d a n d laterally Ref. Dhingra 4th/ed p 63; Mohan Bansal Ist/edp 42
N a s o l a c r i m a l d u c t : It is a m e m b r a n o u s passage w h i c h begins at t h e lower e n d o f t h e lacrimal sac. It runs d o w n w a r d , b a c k w a r d a n d
laterally a n d opens in t h e inferior meatus o f t h e nose. A f o l d o f m u c o u s m e m b r a n e called t h e valve o f Hasner f o r m s an i m p e r f e c t
value at t h e l o w e r e n d o f t h e d u c t .
1 8 . A n s . is b i.e. L a c r i m a l b o n e
Ref. BDC4th/ed Vol3pp 228-229;
Dhingra 5th/ed p 162,6th/edp 147
Nasal spine of
Nasal s e p t u m is t h e osseocartilagenous p a r t i t i o n b e t w e e n t h e
frontal bone
t w o halves o f nasal cavity.
Its c o n s t i t u e n t s are (Fig. 1.2):
Crest of nasal
1. O s s e o u s p a r t bone
• The v o m e r
Membranous
• Rostrum and crest o f s p h e n o i d septum
• Nasal crest of maxillary b o n e
• Perpendicular plate o f e t h m o i d Columellar
• Nasal crest o f nasal b o n e septum
• Nasal spine o f f r o n t a l b o n e
Ant. nasal spine
• Nasal crest of palatine b o n e
of maxilla
• Nasal crest o f maxillary b o n e
Crest of palatine bone
2. C a r t i l a g i n o u s part
Septal (Qudrilateral) cartilage Fig. 1.2: A n a t o m y o f Nasal S e p t u m

1 9 . A n s . is c i.e. S p h e n o i d Ref. Scott Brown 7th/ed Vol 2, p 1326; Dhingra 6th/ed pi 47.
Quadrilateral cartilage f o r m s t h e nasal s e p t u m . It is b o u n d e d f i r m l y by collagenous fibers t o t h e
• Nasal bones
• Ethmoid
• Vomer
• Maxilla
2 0 . A n s . is b i.e. V o m e r Ref. Scott Brown 7th/ed Vol2pp 1329-1330; Dhingra 5th/edpp 150-153,6th/ed 134-138
The lateral nasal wall is c o m p o s e d o f t h r e e t u r b i n a t e s
• Superior t u r b i n a t e
• Middle turbinate
• Inferior t u r b i n a t e
Below each t u r b i n a t e is t h e respective meatus:
• Inferior m e a t u s
• Middle meatus
• Superior m e a t u s
• A b o v e t h e superior t u r b i n a t e lies t h e s p h e n o e t h m o i d recess.
• Just a n t e r i o r t o t h e m i d d l e meatus, is a small c r e s t / m o u n d o n t h e lateral wall called as A g g e r nasi.
- In t h e inferior meatus - o p e n s t h e nasolacrimal d u c t g u a r d e d at its t e r m i n a l end by a mucosal valve k/a Hasner's valve.
2 1 . A n s . is c i.e. L o w e r e n d of u p p e r lateral c a r t i l a g e
Ref. Scotts Brown 7th/ed Vol2, p 1358;Dhingra5th/edp 150;6th/edp 138;Mohan Bansalp287
CHAPTER 1 Anatomy and Physiology of Nose

Anterior N a s a l Valve/Internal Nasal V a l v e (Fig. 1.3)


• This is t h e narrowest part o f nose a n d is less w e l l d e f i n e d
physiologically t h a n anatomically.
• It is f o r m e d by the lower e d g e o f t h e u p p e r lateral cartilages,
t h e a n t e r i o r e n d o f t h e inferior t u r b i n a t e a n d t h e a d j a c e n t
s e p t u m t o g e t h e r w i t h t h e s u r r o u n d i n g soft tissues.

NOTE
External
To check the potency of nasal value Cottle's test is done in conditions
nasal valve
like DNS.

2 2 . A n s . is a i.e. O p t i c n e r v e a n d floor of orbit.

Ref. Graijs40th/edp558;Dhingra5th/edp 153,6th/edp 136; Internal


Mohan Bansal lst/edp38 nasal valve
The O n o d i a n d Haller cells are e t h m o i d a l air cells.

Ethmoidal sinuses
• They very f r o m 8 t o 18 in n u m b e r a n d lie w i t h i n t h e lateral
F i g . 1.3: Nasal valves. (A) External nasal valve area ( b o u n d e d b y
p a r t o f e t h m o i d b o n e ( b e t w e e n nasal c a v i t y a n d o r b i t )
p y r i f o r m n o t c h o f maxilla a n d u p p e r a n d l o w e r lateral cartilages);
called as e t h m o i d a l l a b y r i n t h .
(B) Internal nasal valve (inset) is b o u n d e d by s e p t u m , inferior e d g e
• E t h m o i d a l sinuses are d i v i d e d i n t o 2 g r o u p s :
o f l o w e r lateral cartilage, a n d anterior aspect o f inferior t u r b i n a t e
(Note earlier t h e r e w e r e 3 g r o u p s ) :
Coutesy: Textbook of Diseases of Ear, Nose and
- Anterior Throat, Mohan Bansal. Jaypee Brothers, p 287
- Middle
- Posterior
N o w m i d d l e g r o u p is i n c o r p o r a t e d in anterior g r o u p .

Anterior G r o u p Posterior g r o u p

Anterior e t h m o i d a l air cells Middle e t h m o i d a l air cells

• Open into the middle meatus Form a round elevation k/a ethmoidal bulla • Posterior ethmoidal air cells drain
• 2 cells are important in tnis group: on the lateral wall of nasal cavity into superior meatus and some in
1. Agar cells: Middle ethmoidal sinus drains into middle sphenoethmoidal recess
Related to lacrimal sac and duct meatus above the ethmoidal bulla • Most important cells of this group are:
2. Haller cells: Onodi cells:
Related to orbital floor They are the most posterior ethmoidal cells
These ethmoid cells extend into the roof of which lie in close association with optic nerve,
maxillary sinus ostium. These cells remain in the floor of orbit. Onodi cells must be
a s y m p t o m a t i c or affect maxillary sinus recognised during the endoscopic sinus surgery
ventilation and drainage resulting in recurrent on posterior ethmoid to avoid optic nerve injury.
or chronic maxillary sinusitis. They are present
in 1 0 % of population

2 3 . A n s . is a i.e. Nasal cavity w i t h m a x i l l a r y s i n u s Ref. Scott Brown 7th/ed Vol2p 1345


Osteomeatal c o m p l e x lies in t h e m i d d l e meatus. It is t h e final c o m m o n d r a i n a g e p a t h w a y for t h e maxillary, f r o n t a l a n d a n t e r i o r
e t h m o i d sinuses into t h e nasal cavity (so w i l l o b v i o u s l y c o n n e c t any o f these t o t h e nasal cavity).
2 4 . A n s . is c i.e. M a i n l y e x t e r n a l c a r o t i d a r t e r y Resf. Dhingra 5th/ed pp 189,190
B o t h internal c a r o t i d artery a n d external c a r o t i d artery s u p p l y t h e nose b u t m a i n artery is t h e external c a r o t i d artery.

Internal carotid artery

Ophthalmic artery

r
Anterior ethmoidal artery Posterior ethmoidal artery
Supply: Supply:
• Nasal septum • Nasal septum
• Lateral wall • Lateral wall
SECTION I Nose and Paranasal Sinuses

External carotid artery


x

Facial r
artery
1
Maxillary artery

L
Superior labial artery Branches to the
r
Greater palatine artery
T 1
Infraorbital artery Sphenopalatine artery
T nasal vestibule
T T
Septal branch Supplies: Supplies: Anterior superior
Supplies: Nasal septum dental artery
T Nasal septum Lateral wall
Lateral wall of nose
Supplies:
Nasal septum
T
Supply:
Lateral wall of nose
Nasal septum

25, A n s . is a i.e. Middle p a r t of t h e cavity in m i d d l e m e a t u s in p a r a b o l i c c u r v e Ref. Dhingra 5th/ed p 155;6th/edp 140


Nose is t h e natural p a t h w a y for b r e a t h i n g .
During quiet respiration:
• Inspiratory air c u r r e n t passes t h r o u g h m i d d l e p a r t o f nose b e t w e e n t h e t u r b i n a t e s a n d nasal s e p t u m .
• Very little air passes t h r o u g h inferior meatus or o l f a c t o r y r e g i o n o f nose. Therefore, w e a k o d o r o u s substances
have t o be sniffed before t h e y can reach olfactory, area.
• D u r i n g e x p i r a t o r n , air c u r r e n t f o l l o w s t h e same course as d u r i n g i n s p i r a t i o n , b u t t h e entire air c u r r e n t is n o t expelled directly
t h r o u g h t h e nares.
• Friction offered at l i m e n nasi converts it i n t o eddies u n d e r cover o f inferior a n d m i d d l e t u r b i n a t e s a n d t h u s sinuses are v e n t i l a t e d
during expiration.
26. A n s . is c i.e. Traps t h e p a t h o g e n i c o r g a n i s m s in i n s p i r e d air Ref. Dhingra 5th/edp 156,6th/edp 140
27. A n s . is c i.e. 5 to 10 mm/min
In m u c o c i l i a r y m e c h a n i s m o f nasal m u c o s a — c i l i a beat c o n s t a n t l y at speed o f 5-10 m m / m i n u t e a n d are in c o n t a c t w i t h serous
layer o f m u c u s w h i c h entraps t h e f o r e i g n bodies, allergens a n d carry it t o nasopharynx. Beat f r e q u e n c y = 10 Hz
M o v e m e n t is m e t a c h r o n u s (i.e. all cilia at r i g h t angle t o t h e d i r e c t i o n o f f l o w are in t h e d i r e c t i o n o f f l o w w h i l e t h o s e in t h e d i r e c t i o n
o f f l o w are slightly o u t o f phase).
M u c o c i l i a r y clearance t i m e = 5-20 mins.
The pH o f m u c o u s blanket o f nose is 7.

28. A n s . is a i.e. P e r v e r s i o n of s m e l l s e n s a t i o n Ref. Dhingra 5th/edp 157,6th/edp 142


D i s o r d e r s of smell
Anosmia: Total loss o f sense o f smell
Hyposmia: Partial loss o f sense o f smell
Parosmia: Perversion o f smell (Seen in recovery phase o f p o s t influenzal anosmia; intracranial t u m o r s ) .
29 A n s . is a T u r b i n a t e Ref. Dhingra6th/edp 147;Logan Turner 10th/edp5
For details see Ans 18.
3 0 . A n s . is a i.e. Nasolacrimal d u c t Ref. Dhingra 6th/edp 135
3 1 . A n s . is b i.e. Inferior t u r b i n a t e Dhingra 6th/ed p 135
Expl: Repeat.
3 2 . A n s . is b i.e. Posterior e t h m o i d s i n u s : Ref. Dhingra 6th/edp 136 Fig. 23.4
3 3 . A n s . is c i.e. e v e r y 6-8-hours Ref. Dhingra 6th/ed p 140; Mohan Basal Ist/edp40

Nasal cycle: The alternate o p e n i n g a n d closing o f each side o f nose is called nasal cycle
- Kayser first described nasal cycle in 1895
- There is rhythmic cyclical congestion and decongestion of nasal mucosa
- Nasal cycle varies every 2'/>-4 hrs and is characteristic of an individual.

NOTE

The closest answer here is 6-8 hours, hence it is being marked as the correct answer.
-

CHAPTER

n ^ Diseases of External Nose


Nasal Sept

| SADDLE NOSE Diagnosis

• Presence o f m u c o i d discharge in nose.


• Nasal d o r s u m is depressed (sagging o f t h e b r i d g e o f nose).
• Absence o f air b u b b l e s in nasal discharge.
• Depressed nasal d o r s u m may involve either bony, cartilaginous • Failure t o pass a catheter f r o m nose t o p h a r y n x .
or b o t h b o n y a n d cartilaginous c o m p o n e n t s . • P u t t i n g a f e w drops o f m e t h y l e n e b l u e dye i n t o t h e nose a n d
• Most common e t i o l o g y : Nasal t r a u m a . seeing its passage t h r o u g h t h e p h a r y n x .
• CT scan is diagnostic
C a u s e s of D e p r e s s e d Nose/Saddle Nose
Treatment

^ ^ ri in e m o n i c In B / L c h o a n a l atresia: McGovern's t e c h n i q u e - > Placing a f e e d i n g


n i p p l e w i t h a large hole.
H = Hematoma
D e f i n i t i v e t r e a t m e n t : C o r r e c t i o n o f atresia b y t r a n s n a s a l o r
O = Operative, i.e. excessive removal of septum during
submucous resection transpalatal a p p r o a c h . D o n e at 1 Vi years.
T = Trauma
S = Syphilis
A = Abscess Extra Edge
L = Leprosy Ref. Current Otolaryngology 2nd/edp 243
T = Tuberculosis In utero exposure t o methimazole can lead to choanal atresia
HOT SALT along with other anomalies like esophageal atresia and
developmental delay
Earlier it was said choanal atresia is bony in 9 0 % and membranous
Management in 1 0 % cases. But recent studies reveal that in 2 9 % cases, choanal
atresia consists of purely bony elements and in 7 1 % cases both
A u g m e n t a t i o n r h i n o p l a s t y i.e. f i l l i n g t h e d e f o r m i t y w i t h cartilage,
bony and membranous materials are present.
b o n e or synthetic i m p l a n t .
On CT choanal atresia is diagnosed if posterior choanal orifice is
< 0.34 cm or if posterior vomer measures > 0.55 cm.
| CROOKED/DEVIATED NOSE

| TUMORS OF EXTERNAL NOSE


Crooked n o s e : Midline of d o r s u r n f r o m frontonasal angle t o the
t i p is c u r v e d in a C or S shaped manner.
They can be d i v i d e d i n t o t h r e e categories - C o n g e n i t a l , b e n i g n or
D e v i a t e d n o s e : M i d l i n e is s t r a i g h t b u t d e v i a t e d t o o n e side. m a l i g n a n t (Table 2.1).

Classification of Swellings of E x t e r n a l N o s e a n d V e s t i b u l e
| CHOANAL ATRESIA
T a b l e 2.1 C l a s s i f i c a t i o n o f t u m o r s o f external nose
• It is d u e t o persistence o f bucconassssssal m e m b r a n e "
(Right side atresia is more common than left side). 0 Congenital, Benign Malignant
• Unilateral atresia is m o r e c o m m o n . " Dermoid Rhinophyma or potato Basal cell
• Unilateral atresia remains u n d i a g n o s e d u n t i l a d u l t life. tumor carcinoma
• Bilateral atresia presents w i t h respiration o b s t r u c t i o n in n e w - (rodent ulcer)
Squamous
born.
cell carcinoma
• It is m o r e c o m m o n in females. (epithelioma).

Contd.
12 I SECTION I Nose and Paranasal Sinuses

Contd... E t i o l o g y : Septal d e v i a t i o n can be d u e t o :


• T r a u m a : Birth t r a u m a , accidental t r a u m a a n d f i g h t s .
Congenital Benign Malignant
• D e v e l o p m e n t a l error: Unequal g r o w t h b e t w e e n t h e palate
Encephalocele or Papilloma a n d t h e skull base cause b u c k l i n g o f t h e nasal s e p t u m . It is
meningoencephalocele Hemangioma seen in cleft lip a n d palate a n d in case o f d e n t a l anomalies.
Glioma Pigmented nevus • Racial f a c t o r s : Caucasians > Negroes.
• H e r e d i t a r y factors: It runs in families.
Nasoalveolar cyst Seborrheic .keratosis Melanoma
Neurofibrom
Tumors of sweat glands Types

• A n t e r i o r dislocation i.e. a n t e r i o r e n d o f cartilaginous s e p t u m


Rhinophyma/Potato Tumor m a y p r o j e c t i n t o o n e o f t h e vestibules.

• It is a s l o w - g r o w i n g b e n i g n t u m o r w h i c h occurs d u e t o h y p e r - • C-shaped d e f o r m i t y
t r o p h y o f t h e sebaceous glands o f t h e t i p o f t h e nose. • S-shaped d e f o r m i t y
• Seen in l o n g s t a n d i n g cases o f acne rosacea. • Spurs: Sharp shelf like p r o j e c t i o n at t h e j u n c t i o n o f t h e b o n e
• M o s t l y affects m e n past m i d d l e age. a n d t h e cartilage [ m a y occur at t h e j u n c t i o n o f v o m e r b e l o w
• Presents as a p i n k , l o b u l a t e d mass over t h e nose. (Color is p i n k / a n d septal cartilage and/or e t h m o i d b o n e ]
red because o f vascular e n g o r g e m e n t ) . • Symptoms: See Flow c h a r t 2.1.

Treatment o

With C 0 2 laser- -bulk o f t u m o r is r e m o v e d . • DNS is more common in males:


Cottle test: 0

• Purpose: To confirm whether the obstruction is in the nasal


Basal cell carcinoma of external nose - It is the M/c malignant
valve area, which is the narrowest part of the nasal cavity.
tumor of nose inolung the nasal skin. The M/C sites on nose are
• Method: The patient pulls the cheeks outward and breathes
are, nasal tip and ala.
quietly. If the nasal airway improves on the test side, the test is
2nd M/c malignant tumor of nose is squamous cell carcinoma.
positive and indicates abnormality of the vestibular component
of nasal valve.

| SEPTAL DEVIATIONS - DEVIATED NASAL SEPTUM


TVeataient

DNS is a c o m m o n p r o b l e m in w h i c h nasal s e p t u m is displaced. No t r e a t m e n t is r e q u i r e d if it is n o t causing any s y m p t o m s .


N o r m a l l y , s e p t u m lies in c e n t e r t h e r e f o r e nasal c a v i t i e s Surgical management is the treatment of choice.
are s y m m e t r i c a l . In case o f DNS-the c a r t i l a g i n o u s r i d g e o f t h e S e p t o p l a s t y : conservative surgery as m o s t o f t h e septal
s e p t u m lies e i t h e r t o w a r d r i g h t or left side a n d nasal cavities are f r a m e w o r k is r e t a i n e d . O n l y t h e m o s t d e v i a t e d parts are
asymmetrical. r e m o v e d . Rest o f t h e septal f r a m e w o r k is corrected a n d

Symptoms/Pathophysiology of Septal Deviation


F l o w c h a r t 2.1: S y m p t o m s o f septal d e v i a t i o n

Deviated nasal septum on one side

External Hypertrophy of inferior


deformity of nose turbinate on other side

I i

Enlarged turbinate
Nasal obstruction (U/L or B/L) presses on sinuses
opening
r
Air currents
T
Headache
T
Excessive crusting Mouth breathing and 4
cannot reach and facial and drying of consequent dryness Sinusitis i.e.
the olfactory area neuralgias secretions of mouth, pharynx nasal discharge,
and larynx pain
Epistaxis
Recurrent attacks sore
Secondary infection throat common cold,
ear cause Eustachian tonsillitis and Bronchitis
tube catarrh • ;

T
Middle ear
infections
CHAPTER 2 Diseases of External Nose and Nasal Septum

r e p o s i t e d b y plastic means. It is the preferred operation in • Poisons [cocaine, t o p i c a l steroids a n d d e c o n g e s t a n t s ]


children. • T u m o r s o f s e p t u m e.g. c h o n d r o s a r c o m a , g r a n u l o m a
S u b m u c o u s R e s e c t i o n : It is done only in adults. • Idiopathic
• •
NOTE
NOTE
Septal surgery is usually done after the age of 17 so as not to
Syphilis causes perforation of the bony part while lupus, tuberculosis
interfere with the growth of nasal skeleton.
and leprosy involve the cartilaginous part.
Only if a child has severe septal deviation causing marked
nasal obstruction, septoplasty should be done.
Symptoms

• Small a n t e r i o r p e r f o r a t i o n causes w h i s t l i n g s o u n d d u r i n g
| SEPTAL PERFORATION inspiration or e x p i r a t i o n .
• Larger p e r f o r a t i o n s r e s u l t i n crusts f o r m a t i o n w h i c h c a n
Etiology o b s t r u c t t h e nose a n d lead t o excessive b l e e d i n g w h e n it is
• Trauma"—Surgical (during a n d ; C|W1D\ removed.
- Repeated cautery
Treatment
" N o s e
P i c k i
"9
T i g h t nasal p a c k i n g • If p e r f o r a t i o n is a s y m p t o m a t i c n o t r e a t m e n t is r e q u i r e d .
• Chronic i n f l a m m a t i o n [Wegener's g r a n u l o m a t o s i s , Syphilis,TB • Small a n d m e d i u m sized p e r f o r a t i o n (< 2 c m i n d i a m e t e r ) :
Leprosy, a t r o p h i c rhinitis] Closure is d o n e surgically by raising flaps a n d s t i c h i n g o n t h e
• Nasal myiasis perforation.
• R h i n o l i t h or n e g l e c t e d f o r e i g n b o d y
• Large p e r f o r a t i o n (> 2 c m in d i a m e t e r ) : O b t u r a t o r s or silastic
• As a c o m p l i c a t i o n o f septal abscess or h e m a t o m a , if drainage
b u t t o n s are used t o close p e r f o r a t i o n s .
is delayed.


SECTION I Nose and Paranasal Sinuses

QUESTIONS
l.a. R h i n o p h y m a is a s s o c i a t e d w i t h : [AI07][AP96, UP01] a. Indicated in septal deviation
a. Hypertrophy o f t h e sebaceous glands b. M u c o p e r i c h o n d r i u m is removed
b. Hypertrophy o f sweat glands sss c. Preferably d o n e after 16 years o f age
c. Hyperplasia o f endothelial cells d. Done in some cases o f epistaxis
d. Hyperplasia o f epithelial cells 14. A l t e r n a t i v e for SMR: [DNB 01]
l.b. True a b o u t r h i n o p h y m a : [AI01] a. Tympanoplasty b. Septoplasty
a. Premalignant b. C o m m o n in alcoholics c. Caldwell-Luc operation d. Turboplasty
c. Acne rosacea d. Fungal etiology 1 5 . Killian's incision is u s e d for: [TN04]
e. Treatment is shaving, dermabrasion and skin grafting. a. Submucous resection o f nasal s e p t u m
2. Most c o m m o n p r e s e n t a t i o n o f infant w i t h bilateral b. Intranasal antrostomy
choanal atresia: (AIIMS 96] c. Caldwell-Luc operation
a. Difficulty in breathing b. Dysphagia d. Myringoplasty
c. Smiling d. Difficulty in walking 16. C o m m o n indication of s e p t o p l a s t y : [PGIJune04]
3. C h o a n a l a t r e s i a is a s s o c i a t e d w i t h : [PGI 08] a. DNS w i t h symptoms b. Anosmia
a. Colobamatous blindness b. Heart disorder c. Sluder's neuralgia d . Septal spar
c. Renal anomaly d. Ear disorder 17. W h i c h is not d o n e in s e p t o p l a s t y : [St. Johns 02]
e. CNS lesion a. Elective hypotension
4. All a r e t r u e a b o u t n a s o l a b i a l c y s t s except: [AIIMS Nov 08] b. Throat pack
a. They are B/L c. Nasal preparation w i t h 1 0 % cocaine
b. They present in adults d. None
c. Derived f r o m o d o n t o g e n i c epithelium 18. W h i c h of t h e following s u r g e r y is c o n t r a i n d i c a t e d b e l o w
d. Strong female predilection 12 y e a r s of a g e ? [MH03]
5. D e p r e s s e d b r i d g e of t h e n o s e m a y b e d u e to a n y of t h e a. Rhinoplasty b. Antral puncture
following e x c e p t : [DNB 03] c. SMR d. Septoplasty
a. Leprosy b. Syphilis 19. To p r e v e n t s y n a c h i a e f o r m a t i o n a f t e r n a s a l s u r g e r y ,
c. Thalassemia d . Acromegaly w h i c h o n e of t h e following p a c k i n g s is t h e m o s t u s e f u l :
6 . A c r o o k e d n o s e is d u e to: [PAL 93] [AIIMS Nov 04]
a. Deviated Tip and Septum b. Deviated ala a. Mitomycin
c. Deviated septum d . Deviated dorsum and septum b. Ribbon gauze
7. Percentage of n e w b o r n s w i t h d e v i a t i o n of n a s a l s e p t u m : c. Ribbon gauza w i t h liquid paraffin
[PGI 93] d. Ribbon gauza steroids
a. 2 % b. 1 0 % 20. True a b o u t s e p t a l h e m a t o m a is: [PGI 02]
c. 2 0 % d. 5 0 % a. Occurs due t o trauma
8. F e a t u r e s a s s o c i a t e d with DNS include all of t h e following b. Can lead t o saddle-nose deformity
except: [Al 98] c. Conservative treatment
a. Epistaxis b. Atrophy o f turbinate d. May lead t o abscess formation
c. Hypertrophy o f turbinate d. Recurrent sinusitis 21. B o n y s e p t a l perforation o c c u r s i n : [Karnataka 95]
a. TB b. Leprosy
9. All a r e c o m p l i c a t i o n of DNS, E x c e p t : [AIIMS 93]
c. Syphilis d. Sarcoidosis
a. Maxillary sinusitis
22. S e p t a l perforation is not s e e n i n : [DNB 02]
b. Septal spur
a. Septal abscess b. Leprosy
c. Sphenoiditis
c. Rhinophyma d. Trauma
d. Hypertrophied inferior turbinate
23. N a s a l s e p t u m p e r f o r a t i o n o c c u r s in all t h e f o l l o w i n g
10. For d e v i a t e d n a s a l s e p t u m , s u r g e r y is r e q u i r e d for: [PGI
except: [UP 04]
01]
a. Tuberculosis b. Nasal surgery
a. Septal spur w i t h epistaxis b. Marked septal deviation
c. Syphilis d. Rhinosporidiosis
c. Persistent rhinorrhea d. Recurrent sinusitis
24. T h e e t i o l o g y of a n t e r i o r e t h m o i d a l n e u r a l g i a is:
e. Prolonged DNS
[AIIMS 03]
1 1 . T h u d i c u l u m n a s a l s p e c u l u m is u s e d to v i s u a l i z e : [TN 03]
a. Inferior turbinate pressing on the nasal septum
a. Anterior nasal cavity b. Posterior nares
b. Middle turbinate pressing on t h e nasal s e p t u m
c. Tonsils d. Larynx
c. Superior turbinate pressing on t h e nasal s e p t u m
1 2 . W h i c h is not v i s u a l i z e d o n p o s t e r i o r r h i n o s c o p y : [Al 92]
d. Causing obstruction o f sphenoid o p e n i n g
a. Eustachian t u b e b. Inferior meatus
25. Cottle's test tests t h e p a t e n c y of t h e n a r e s i n : [JIPMER]
c. Middle meatus d. Superior concha
a. Atrophic rhinitis b. Rhinosporidiosis
1 3 . All of t h e following t r u e of s u b m u c o u s r e s e c t i o n o p e r a -
c. Deviated nasal s e p t u m
t i o n for DNS e x c e p t : [UPSC]
d. Hypertrophied inferior turbinate
CHAPTER 2 Diseases of External Nose and Nasal Septum

EXPLANATIONS AND REFERENCES


1. a . A n s . is a i.e. H y p e r t r o p h y of t h e s e b a c e o u s g l a n d s . Mohan Bansal p292
1. b. A n s . is c a n d e i.e. A c n e r o s a c e a ; a n d T r e a t m e n t is s h a v i n g , d e r m a b r a s i o n a n d s k i n grafting
Ref. Dhingra 5th/ed p 160,6th/ed p 144; Mohan Bansal Ist/ed p 292
• R h i n o p h y m a is a s l o w - g r o w i n g b e n i g n t u m o r w h i c h occurs d u e t o h y p e r t r o p h y o f t h e sebaceous g l a n d s o f t h e t i p o f t h e nose.
0

• Seen in l o n g s t a n d i n g cases o f acne rosacea. 0

• M o s t l y affects m e n past m i d d l e age.


• Presents as a p i n k , l o b u l a t e d mass over t h e nose.

Treatment

• Paring d o w n t h e b u l k o f t h e t u m o r w i t h a sharp knife, or c a r b o n d i o x i d e laser or scalpel (dermabraions), a n d t h e area is a l l o w e d


t o re-epithelize.
„ >" . .
• S o m e t i m e s t u m o r is c o m p l e t e l y excised a n d t h e raw area is covered w i t h skin graft.
2. A n s . is a i.e. Difficulty in b r e a t h i n g Ref. Logan/Turner Wth/ed p 379; Dhingra 5th/ed p 178,6th/edp 163; Mohan Bansal p 337
• Choanal atresia is usually U/L.
If it occurs bilaterally t h e n e o n a t e presents w i t h d i f f i c u l t y in b r e a t h i n g as infant is a nose breather a n d does n o t b r e a t h e f r o m
m o u t h . The n e o n a t e m a y have asphyxia a n d bilateral b l o c k a g e o f nose t h a t also makes suckling d i f f i c u l t .
• U/L atresia presents w i t h nasal o b s t r u c t i o n i n c l u d i n g s n o r i n g b u t goes u n i d e n t i f i e d till a d u l t life.
3. A n s . is a, b, d a n d e i.e. C o l o b a m a t o u s b l i n d n e s s ; H e a r t d i s o r d e r ; Ear d i s o r d e r s ; a n d CNS lesion
Ref. Scott Brown 7th/ed Vol 1 p 1071; Dhingra 5th/ed p 1786th/edp 163; OP Ghai 6th and 7th/edpp 336,337

Choanal Atresia

• Choanal atresia is associated w i t h CHARGE s y n d r o m e : C l o b o m a o f eye, Heart defects, Choanal Atresia, Retarded g r o w t h , Genital
defects a n d Ear defects.
4. A n s . is c i.e. D e r i v e d f r o m o d o n t o g e n i c e p i t h e l i u m
Ref. http://www.maxillofacialcentre.com./Bondbook/softissue/nasolabialcyst.html#introduction;
Scott Brown 7th/ed Vol2p 1320; Dhingra 6th/ed p 146; Mohan Bansalp 292

Nasolabial Cysts/Nasoalveolar Cyst/Klestadt's Cyst

• It is a rare non o d o n t o g e n i c cyst w h i c h originates f r o m e p i t h e l i a l e n t r a p m e n t in t h e line o f f u s i o n b e t w e e n maxillary a n d


m e d i a n nasal elevation.
Female > M a l e
Bilateral in a p p r o x i m a t e l y 1 0 % o f all cases.
Usually present in 4 t h a n d 5 t h decades o f life.
It lies o n t h e b o n e a n d causes an excavation. It is closely a t t a c h e d t o f l o o r o f t h e nose.
It presents as a s m o o t h a n d soft b u l g e in t h e lateral wall a n d f l o o r o f vestibule anterior t o inferior t u r b i n a t e .
Large cyst obliterates t h e alar facial f o l d (nasolabial sulcus).
T r e a t m e n t is by surgical excision using sublabial a p p r o a c h .

NOTE

Globulomaxillary cyst arise at the junction of the primitive palate and palatine process in the alveolaolar process between lateral incisor and canine teeth.

5. A n s . is d i.e. A c r o m e g a l y - Ref. Dhingra 5th/edpi58,6th/edp 143


Depressed nasal b r i d g e results f r o m s a g g i n g o f t h e b r i d g e o f nose either d u e t o injury or i n f e c t i o n o f osseus or cartilaginous p a r t
o f t h e b r i d g e o f nose
Causes o f depressed nose/saddle nose are:
H = Hematoma
O = Operative, i.e. excessive removal o f s e p t u m d u r i n g s u b m u c o u s resection
T = Trauma
S = Syphilis
A = Abscess
L = Leprosy
T = Tuberculosis
( M n e m o n i c - H O T SALT)

SECTION I Nose and Paranasal Sinuses

A n s . is d i.e. D e v i a t e d d o r s u m a n d s e p t u m
Ref. Dhingra 5th/ed, p 158; 6th/ed p 143; Mohan Bansal p 291
• In crooked nose, t h e m i d l i n e o f d o r s u m f r o m frontonasal
angle t o t h e t i p is curved in a C- or S-shaped manner (Fig. 2.1).
• In a deviatednosme, t h e m i d l i n e is straight b u t d e v i a t e d
t o o n e side.
• Saddle nose is d e p r e s s e d nasal d o r s u m w h i c h may
involve o n l y cartilaginous or b o t h b o n y a n d cartilaginous
components.
A n s . is c i. e. 2 0 % Ref. Turner 10th/ed p 21
• D u r i n g n o r m a l p a r t u r i t i o n t h e f e t a l h e a d is d i r e c t e d
caudally a n d passes t h r o u g h t h e pelvic b r i m .
• The Caucasian head is w i d e s t at t h e occipitonasal diameter.
Up t o 2 0 % o f babies b o r n in t h i s m a n n e r are f o u n d t o have
squashed noses. The m a j o r i t y s p r i n g back i n t o place b u t
a b o u t 1 - 2 % are left w i t h a p e r m a n e n t l y d e v i a t e d s e p t u m .
This m a y n o t be a p p a r e n t initially b u t s u b s e q u e n t l y gives Crooked nose Deviated nose
rise t o nasal o b s t r u c t i o n a n d snuffles.
Fig. 2.1: Nasal b r i d g e is S-shaped in c r o o k e d nose. It is straight
A n s . is b i.e. A t r o p h y of t u r b i n a t e
b u t deviated t o o n e side in d e v i a t e d nose.
Ref. Dhingra 5th/ed pp 164,165,6th/edp 149;Tuli Ist/edp 153;
Mohan Bansal 1 st/ed p 334,335
A n s . is b i.e. S e p t a l s p u r

Deviated nasal septum on one side


I
£
External deformity of nose Hypertrophy of inferior turbinate
on other side
t
Nasal obstruction (U/L or B/L)
Enlarged turbinate presses
on sinuses opening
T I
Air currents cannot reach Headache and facial Mouth breathing and conse-
Sinusities i.e. nasal
the olfactory area neuralgias quent dryness of mouth,
discharge, pain
plarynx and larynx
T X
Anosmia
Recurrent attacks—sore •

throat, common cold,


tonsillitis and bronchitis •

Secondary infection can Excessive crusting and


cause Eustachian tube catarrhal drying of secretions
X X
Middle ear infections Epistaxis

NOTE

• In deviated nasal septum, the nasal chamber on the concave side of the nasal septum is wide and shows compensatory hypertrophy of turbinates
and not atrophy.
• Septal spur is a type of DNS and not its complication

1 0 . A n s . is a, b, c a n d d i.e. S e p t a l s p u r w i t h e p i s t a x i s ; M a r k e d s e p t a l d e v i a t i o n ; Persistent r h i n o r r h e a ; a n d R e c u r r e n t sinusitis


Ref. Dhingra 5th/edpp423,425,6th/edp413,415;Tuli Ist/edp 507,2nd/edp516

I n d i c a t i o n s for S u r g e r y in DNS

• P e r s i s t e n t unilateral n a s a l o b s t r u c t i o n a n d r e c u r r e n t h e a d a c h e s
• D e v i a t i o n causing recurrent sinusitis or otitis m e d i a •

• Recurrent epistaxis f r o m septal spur


• Access f o r o p e r a t i o n in p o l y p e c t o m y w i t h DNS
• As a p a r t o f s e p t o r h i n o p l a s t y f o r cosmetic c o r r e c t i o n o f external nasal d e f o r m i t i e s .

CHAPTER 2 Diseases of External Nose and Nasal Septum


J 1 7

• As a a p p r o a c h t o h y p o p h y s e c t o m y
- Septoplasty is d o n e in c h i l d r e n , adolescents a n d y o u n g female.
- S u b m u c o u s resection is i n d i c a t e d in adults (after 17-18 yrs).
1 1 . A n s . is a i.e. A n t e r i o r n a s a l c a v i t y Ref. Tuli ist/edp 538,2nd/ed p 503; Mohan Bansal p 281; Maqbool 12th/ed p 340
T h u d i c u l u m nasal s p e c u l u m or Vienna t y p e o f nasal s p e c u l u m is' Adenoids
used f o r d o i n g anterior rhinoscopy, f o r e x a m i n a t i o n o f or o p e r a t i o n Posterior
Superior
o n nasal cavity. free margin
turbinate
1 2 . A n s . is b i.e. Inferior m e a t u s Ref. Maqbool 11 th/edp 164 o f s e
P t u m

Posterior r h i n o s c o p y :
It is m e t h o d o f e x a m i n a t i o n o f t h e posterior aspect o f nose a n d Q

pharynx.
Middle
m e a t u s

Structures seen (Fig. 2.2): turbinate


Structures seen on posterior rhinoscopy:
Inferior
- Bothchoanae - Posterior e n d o f nasal s e p t u m Middle -
turbinate
- O p e n i n g o f Eustachian - Posterior e n d o f superior/ meatus
tube m i d d l e a n d inferior t u r b i n a t e s
Pharyngeal
- Fossa o f Rosepmuller - TorusTubarius
opening of
- Adenoids - Roof a n d posterior wall a n d na- Eustachian eustachian
sopharynx. cushion
cushion tube
Fig. 2.2: Structures seen in p o s t e r i o r rhinoscopy.
NOTE

As is evident from the figure superior and middle meatus are seen on posterior Rhinoscopy but not inferior meatus.

1 3 . A n s . is b i.e. M u c o p e r i c h o n d r i m is r e m o v e d Ref. Dhingra 5th/ed p 423,6th/edp413


Submucous Resection of Nasal Septum

Indications
• Deviated nasal s e p t u m (DNS) causing s y m p t o m s o f nasal o b s t r u c t i o n a n d recurrent headache.
• DNS causing o b s t r u c t i o n t o v e n t i l a t i o n o f paranasal sinuses a n d m i d d l e ear, resulting in recurrent sinusitis a n d otitis m e d i a .
• Recurrent epistaxis f r o m septal spur.
• As a p a r t o f s e p t o r h i n o p l a s t y f o r cosmetic c o r r e c t i o n o f external nasal d e f o r m i t i e s .

• As a p r e l i m i n a r y step in h y p o p h y s e c t o m y (transseptal t r a n s s p h e n o i d a l approach) or v i d i a n n e u r e c t o m y (transseptal approach).

Steps of Operation

• Generally d o n e in adults u n d e r local anesthesia.


• Consists o f elevating t h e mucoperichondrial and mucoperiosteal flaps o n either side of t h e septal framework by a single incision made
o n o n e side o f the s e p t u m , r e m o v i n g t h e deflected parts o f t h e b o n y and cartilaginous s e p t u m , a n d t h e n repositioning t h e flaps.
• In S M R - c o m p l e t e removal o f septal cartilage, results in s u p r a t i p depression o f cartilagenous nasal d o r s u m .
• S u b m u c o u s resection is n o t a d v o c a t e d in c h i l d r e n u p t o 17 years o f age as it m a y interfere w i t h d e v e l o p m e n t o f t h e facial bones.
1 4 . A n s . is b i.e S e p t o p l a s t y . c Ref. Dhingra 5th/edp 165,6th/edp 150
1 5 . A n s . is a i.e S u b m u c o u s r e s e c t i o n of n a s a l s e p t u m

Deviated nasal s e p t u m w i t h s y m p t o m s can be t r e a t e d by:


• S u b m u c o u s resection: d o n e o n l y in adults Incision used = Killian's incision
• Septoplasty - can be d o n e in adults as w e l l as c h i l d r e n Incision used = Freer's incisioin

1 6 . A n s . is a i.e. DNS w i t h s y m p t o m s Ref. Dhingra 5th/edp 425,6th/edp 415;Maqbool 11 th/edp 185,12th/edp 137;
Mohan Bansal p 336
Septoplasty is a conservative a p p r o a c h t o surgery. Here m o s t o f t h e septal f r a m e w o r k is r e t a i n e d .
A m u c o p e r i c h o n d r a l / p e r i o s t e a l f l a p is raised generally o n o n e side. O n l y t h e m o s t d e v i a t e d parts are r e m o v e d . Rest o f t h e septal
f r a m e w o r k is corrected a n d r e p o s i t i o n e d . T h i s o p e r a t i o n has replaced s u b m u c o s a l resection.

Indications of Septoplasty

• S y m p t o m a t i c deviated s e p t u m .
• As a p a r t o f s e p t o r h i n o p l a s t y for cosmetic reasons. - i
• As an a p p r o a c h t o h y p o p h y s e c t o m y .
• Recurrent epistaxis d u e t o septal spur

M o s t c o m m o n i n d i c a t i o n for septoplasty - DNS w i t h s y m p t o m -Maqbool 11th/edp 185


SECTION I Nose and Paranasal Sinuses
»t

NOTE

• Septal spur per se is not an indication for septoplasty, only when it leads to recurrent epistaxis then it should be operated.
• "Neither septal deviation nor septal deformities are by themselves an indication for a septoplasty - Scotts Brown 7th/ed Vol 2 p 1580

1 7 . A n s . is d i.e. N o n e Ref. Lees Synopsis of Anaesthesia 13th/edpp 734,735; Current otolaryngology 2nd/edp 175

Intranasal Operations
• "Intranasal operations are polypectomy, septoplasty, rhinoplasty and functional endoscopic sinus surgery. Either a laryngeal mask
or a cuffed endotracheal tube may be used with a throat pack, depending on the anesthetist's confidence, the surgeon, the amount
of blood loss and the duration of surgery. A flexible laryngeal mask or south-facing preformed tube allows the airway to be secured
away from the nose.
• Topical nasal vasoconstriction is extremely useful and may be applied by the anesthetist or surgeon. Commonly used vasoconstrictors
include 5-10% cocaine, cocaine paste, xylometazoline or ephedrine drops or spray, Moffett's solution, or dental cartridge injection
of local anesthetic with epinephrine (adrenaline) 1:80,000. Vasocontstriction by block of the sphenopalatine ganglion, which carries
the vasodilator fibers to the nasal blood vessels, has also been described.
Surgery is easier with controlled hypotension. Profuse bleeding may cause the operation to be abandoned."
Ref. Lees Synopsis of Anaesthesia 13th/ed pp 734,735

Intraoperative and Postoperative Considerations

"The most importantconsideration of nasal surgery is achieving profound vasoconstriction in the nares to minimize and control bleeding.
This vasoconstriction can be achieved with cocaine packs, local anesthetics, and epinephrine infiltration. Since these drugs have a
profound effect on the cardiovascular system, a careful monitoring of the patients cardiovascular functioning is essential, especially
for older patients or patients with known cardiac disease. A vasoconstrictor can also precipitate dysrhythmias.
A moderate degree of controlled hypotension combined with head elevation decreases bleeding in the surgical site. Blood may
passively enter the stomach. Placing an oropharyngeal pack or suctioning the stomach at the conclusion of surgery may attenuate
postoperative retching and vomiting." Current Otolaryngology 2nd/ed p 175
Thus in any nasal surgery:
Elective h y p o t e n s i o n
T h r o a t pack all can be d o n e
Nasal preparation w i t h 1 0 % cociane
1 8 . A n s is c i.e. SMR Ref. Dhingra 5th/edp 423,6th/edp 413

Submucous Resection - Contraindications

• Patients b e l o w 17 years o f age (in such cases conservative surgery i.e. septoplasty s h o u l d be d o n e )
• A c u t e episodes o f respiratory i n f e c t i o n
• Bleeding diathesis
• U n t r e a t e d diabetes or h y p e r t e n s i o n
19. A n s . is a i.e. M i t o m y c i n Ref. Journal of Laryngology and Otology 06, Vol 120, p 921-923ISN 00222151
• A f t e r Nasal s u r g e r y it has b e e n seen t h a t m i t o m y c i n d r o p s a p p l i e d over nasal mucosa decrease nasal synechiae f o r m a t i o n .
• This is t h e n e w e r a p p r o a c h and several trials are b e i n g d o n e o n i t . . . b u t o u r standard t e x t b o o k s have n o t yet i n c l u d e d it.
• "The nasal cavities are packed with ribbon gauze impregnated with Vaseline or liquid paraffin to prevent its sticking to nasal
mucosa."
• "Ribbon gauze impregnated with petroleum jelly or bismuth iodoform paraffin paste (BIPP) is inserted in the entire length
of the nasal cavity in an attempt to tamponade the bleeding." - Scott Brown 7th edpl 602
20. A n s . is a, b a n d d i.e. O c c u r s d u e to t r a u m a ; C a n l e a d to s a d d l e n o s e d e f o r m i t y ; a n d May l e a d to a b s c e s s f o r m a t i o n
Ref. Dhingra5th/edpp 165,166,6th/edp 150;Mohan Bansalp336
• Septal H e m a t o m a is c o l l e c t i o n o f b l o o d w i t h i n t h e s u b p e r i c h o n d r i a l plane o f s e p t u m .
• Etiology: It results f r o m nasal t r a u m a , septal surgery or b l e e d i n g disorder.
• Clinical features: Bilateral nasal o b s t r u c t i o n is t h e c o m m o n e s t p r e s e n t i n g s y m p t o m . It m a y be associated w i t h f r o n t a l headache
a n d a sense o f pressure over t h e nasal b r i d g e .
• Examination: Reveals s m o o t h r o u n d swelling o f t h e s e p t u m in b o t h t h e nasal fossae.
• On p a l p a t i o n : T h e mass is soft a n d f l u c t u a n t .
• T r e a t m e n t : Small h e m a t o m a s can be aspirated w i t h a w i d e b o r e sterile needle. Large h e m a t o m a s are incised and d r a i n e d .
Reaccumalation is p r e v e n t e d b y intranasal p a c k i n g .
CHAPTER 2 Diseases of External Nose and Nasal Septum

Complications

• Septal h e m a t o m a , if n o t d r a i n e d , m a y organize i n t o f i b r o u s tissue leading t o a p e r m a n e n t l y t h i c k e n e d s e p t u m .


• If secondary i n f e c t i o n supervenes, it results in septal abscess.
• Loss o f structural s u p p o r t can cause depression o f nasal d o r s u m l e a d i n g t o saddle nose d e f o r m a i t y .
• Necrosis o f t h e cartilage can cause p e r f o r a t i o n o f t h e nasal s e p t u m — D h i n g r a Sth/ed, p 166
2 1 . A n s . is c i.e. S y p h i l i s
2 2 . A n s . is c i.e. R h i n o p h y m a
2 3 . A n s . is d i.e. R h i n o s p o r i d i o s i s Ref. Dhingra 5th/ed p 166,6th/edp 151; Scott Brown Otolaryngology 7th/ed Vol 2, Chapter 124 p 1583

Septal Perforation

1 t°Xs/Cf^11
f
1 t°!fr.°^ ^ I ' l !ilt '
c

Surgical trauma a. Septal abscess


b. Nasal myiasis
c. Rhinolith
d. Lupus vulgaris
e. TB
f. Leprosy
g. Syphilis
h. Wegner's granuloma
i. Tumorsofnasalseptum.e.g.Chondrosarcoma

A l s o k n o w : Recreational d r u g s like crack or cocaine s n o r t e d nasally are b e c o m i n g increasingly c o m m o n cause o f septal necrosis.
- Scotts Brown 7th/ed Vol2p 1592
Note: C a u s e of Perforation of:

Bony septum Cartilaginous septum Whole septum

Syphilis • Lupus Wegner's


• Leprosy granuloma
• Tuberculosis

2 4 . A n s . is b i.e. M i d d l e t u r b i n a t e p r e s s i n g on t h e n a s a l s e p t u m
Ref. Turner lOth/edp66;Dhingra", 5th/edp461 point 104,6th/edp449
Sluder's n e u r a l g i a or t h e anterior e t h m o i d a l s y n d r o m e is pain a r o u n d t h e b r i d g e o f t h e nose r a d i a t i n g i n t o t h e f o r e h e a d . It is said
t o o r i g i n a t e f r o m t h e m i d d l e t u r b i n a t e pressing o n t h e s e p t u m .
2 5 . A n s . is c i.e. D e v i a t e d n a s a l s e p t u m Ref. Dhingra 5th/edp 164,6th/edpi49;Mohan Bansalp287
Cottle test: It is used t o test nasal o b s t r u c t i o n d u e t o a b n o r m a l i t y o f nasal valve as in case o f d e v i a t e d nasal s e p t u m .
In t h i s test, cheek is d r a w n laterally w h i l e t h e p a t i e n t breathes quietly. If t h e nasal airway i m p r o v e s o n t h e test side, t h e test is p o s i -
tive, a n d indicates a b n o r m a l i t y o f t h e vestibular c o m p o n e n t o f nasal valve.

Also Know

O t h e r tests f o r c h e c k i n g p a t e n c y o f nasal cavities


• Spatula test
• C o t t o n w o o l test

Various Tests of ENT

Test Condition

• ABLB test of fowler • To test positive recruitment as in Meniere's disease and


presbycusis

• Bing test and Chimani-Moos test • Tuning fork test to detect hearing loss

• Doerfler-Stewart test • To detect malinnerino


• Erhard'stest
• Gault'stest
• Crowe-Beck test • Done in lateral sinus thrombosis
• Tobey-Ayer test (Queckenstedt's) • Done in lateral sinus thrombosis

Granulomatous Disorders of
CHAPTER
Nose, Nasal Polyps and Foreign
Body in Nose

GRANULOMATOUS DISEASES OF THE NOSE


Bacterial Fungal Unspecified/Causes

• Syphilis • Rhinosporidiosis • Wegener's granulomatosis

• Tuberculosis • Aspergillosis

• Lupus • Mucormycosis • Non-healing midline granuloma

• Rhinoscleroma • Candidiasis • Sarcoidosis

• Leprosy • Histoplasmosis

• Blastomycosis

| BACTERIAL INFECTIONS • Characteristic f e a t u r e is t h e presence o f apple-jelly nodules


(Brown g e l a t i n o u s nodules) in skin.
Lupus Vulgaris • Lupuscancauseperforationofcartllaginouspartofnasalseptum
• It is an i n d o l e n t a n d chronic f o r m o f t u b e r c u l o u s i n f e c t i o n . * C o n f i r m a t i o n is b y biopsy.

• Female male r a t i o is 2:1 ^ h i l i s ( F l o w C h a r t 3 1)


• Mostcommon site is t h e m u c o c u t a n e o u s j u n c t i o n o f t h e nasal YP l
s e p t u m , t h e nasal vestibule a n d t h e ala.

F l o w c h a r t 3.1: Types a n d clinical feature o f syphilis

SYPHILIS
T
r 1
Acquired Congenital

Early Late
- Can be seen from - Occurs at
3rd week to 3rd month puberty
after birth - Granulomatous
- Presentation - simple disease of nose
catarrh/snuffles

T 1
Primary Secondary Tertiary
Feature—chancre (Most infectious stage) Feature-1. Gumma;
(hard, nontender Site - Bony Nasal S e p t u m 0
Site-Bony Nasal Sepum
ulcerated n o d u l e ) C/F - Persistent rhinitis Note-Initially i t leads t o b o n y s e p t a l
Site = Ext nose/vestibule and crust formation perforation and later cartilage is also involved
2. Saddling of Nose is present
3. Palatal perforation is present
CHAPTER 3 Granulomatous Disorders of Nose, Nasal Polyps and Foreign Body in Nose
J 2 1

Rhinoscleroma Treatment

It is chronic, progressive g r a n u l o m a t o u s disease c o m m e n c i n g i n Dapsone, Isoniazid and Rifampin.


t h e nose a n d e x t e n d i n g i n t o t h e n a s o p h a r y n x a n d o r o p h a r y n x ,
larynx ( s u b g l o t t i c area), trachea a n d b r o n c h i . | FUNGAL INFECTIONS

Organism
Rhinosporidiosis
Klebsiella rhinoscleromatis (Gram-negative Frisch bacillus).
• It is a f u n g a l g r a n u l o m a :
• C a u s a t i v e o r g a n i s m : Rhinosporidium seeberi
Features
• D i s t r i b u t i o n : India, Pakistan, Sri Lanka
• Scleroma can occur at any age a n d in e i t h e r sex.
• M o s t c o m m o n l y affected s i t e s : Nose a n d n a s o p h a r y n x
• The disease has f o l l o w i n g stages:
• O t h e r s : lip, palate, uvula, maxillary a n t r u m , e p i g l o t t i s , larynx,
Atrophic Stage trachea, b r o n c h i , ear, scalp, penis, vulva, vagina.
Resembles a t r o p h i c rhinitis a n d is characterized by f o u l s m e l l i n g • M o d e o f a f f e c t i o n : d u s t f r o m t h e d u n g o f infected h o r s e s a n d
p u r u l e n t nasal discharge a n d c r u s t i n g . cattle a n d t h r o u g h c o n t a m i n a t e d w a t e r o f p o n d .

Granulomatous Stage Features


• Proliferative stage • Y o u n g males are m o r e affected.
• The stage is characterized b y g r a n u l o m a t o u s reactions a n d • Lesions are p o l y p o i d a n d p a p i l l o m a t o u s friable masses w h i c h
presence of'Mikuliczcells'
bleed easily o n t o u c h .
• Painless n o d u l e s are f o r m e d in nasal mucosa.
• They are s t r a w b e r r y (pink t o purple) c o l o r e d a n d s t u d d e d w i t h
• Subdermal infiltration occurs in lower part of external nose and
w h i t e dot's representing t h e sporangia.
upper lip giving a woody feel.
• Patients c o m p l a i n o f nasal discharge w h i c h is b l o o d t i n g e d ,
Cicatricial Stage s o m e t i m e s frank epistaxis is t h e o n l y p r e s e n t i n g c o m p l a i n t .
It is characterized b y f o r m a t i o n of:
Diagnosis
• A d h e s i o n s fibrosis a n d stenosis o f nose, n a s o p h a r y n x a n d
oropharynx.
It is m a d e by biopsy w h i c h shows several sporangia a n d spores.
• S u b g l o t t i c stenosis w i t h respiratory distress m a y occur.
• Pain is n o t a f e a t u r e o f this stage Treatment
• Endoscopic excision o f t h e mass f o l l o w e d by cauterization o f
NOTE
its base.
M/C symptom of Rhinoscleroma is Nasal obstruction and crusting • Recurrence may occur after surgery.
(94%) > Nasal deformity > Epistaxis. • Medical m a n a g e m e n t w i t h dapsone decrease t h e recurrence
rate
Diagnosis
Aspergillosis
' Biopsy shows s u b m u c o s a l infiltrates o f plasma cells, l y m p h o -
• Aspergillosis is the commonest fungal infection of the nose and
cytes, eosinophils, mikuliczcells and russellbodies.
sinuses.
• Mikulicz Cells: are large f o a m cells w i t h a central nucleus a n d
• C a u s a t i v e organism:/), fumigatus ( 9 0 % ) > A. nigerand A. flavus
v a c u o l a t e d c y t o p l a s m c o n t a i n i n g t h e bacilli).
• S p r e a d : air-borne
• RussellBodies:are h o m o g e n o u s eosinophilic inclusion bodies
f o u n d in plasma cells.
Features
• Both o f t h e m are characteristic features o f Rhinoscleroma.
• It can affect any age g r o u p .
Treatment • Black or grayish m e m b r a n e seen o n nasal mucosa.
• Maxillary sinus shows a f u n g a l ball
• S t r e p t o m y c i n (2 g/day) -(-Tetracycline (2 g/day) f o r a m i n i m u m
o f 4-6 weeks (till 2 consecutive samples are negative).
Treatment
• Surgical d i l a t a t i o n o f t h e cicatricial areas w i t h p o l y t h e n e t u b e s
for 6-8 weeks. Surgical d e b r i d e m e n t a n d a n t i f u n g a l d r u g s .

Leprosy Mucormycosis
• M/C in l e p r o m a t o u s leprosy It is an aggressive o p p o r t u n i s t i c f u n g a l i n f e c t i o n
• M/C affected p a r t s : Nasal s e p t u m (anterior part) a n d inferior
Predisposing Factors
turbinate
• I m m u n o s u p p r e s s e d patients
Feature • U n c o n t r o l l e d diabeties

Lead t o p e r f o r a t i o n o f nasal s e p t u m .
SECTION I Nose and Paranasal Sinuses

Features
NASAL POLYPS
• Mucormycosis differs f r o m o t h e r f u n g i as it has a remarkable
a f f i n i t y f o r b l o o d vessels a n d arteries l e a d i n g t o extensive Polyps are non-nedplastic p e d u n c u l a t e d masses w h i c h are
endothelial damage and thrombosis. sparsely cellular a n d are covered by n o r m a l e p i t h e l i u m i.e.
o The disease begins in t h e nose a n d paranasal sinus a n d spreads c o l u m n a r ciliated e p i t h e l i u m .
F e a t u r e s : They are soft, fleshy, pale, insensitive to pain and do
t o o r b i t , c r i b i f o r m plate, m e n i n g e s a n d brain.
not shrink with the use of vasoconstrictors.
• Typical finding: Black necrotic mass seen f i l l i n g t h e entire nasal
They d o n o t bleed o n t o u c h a n d are insensitive t o p r o b i n g a n d
cavity.
never present w i t h epistaxis or b l e e d i n g f r o m nose.
• Erosion o f t h e nasal s e p t u m a n d t h e hard palate m a y be seen. Types o f nasal p o l y p are described in Table 3.1.

Investigations
• Sinus r a d i o g r a p h s s h o w t h i c k e n e d sinus w a l l s a n d s p o t t y Also know: Samters triad - It is a triad of asthma, aspirin
d e s t r u c t i o n o f t h e b o n y walls. ntolerance and nasal polyps.
• MRI detects early vascular a n d intracranial invasion.
Treatment
| F O R E I G N B O D I E S IN N O S E
• Systemic - A m p h o t e r i c i n B
• Surgical d e b r i d e m e n t o f t h e affected tissues May be organic or inorganic a n d are m o s t l y seen in c h i l d r e n "
• Orbital exenteration is m a n d a t o r y in case o f o p h t h a l m o p l e g i a
a n d loss o f vision. Clinical Features

Unilateral f o u l smelling discharge in a c h i l d is p a t h o g n o m i c o f a


EXTRA EDGE
foreign body."
• Syphilis affects the bone, while tuberculosis affects the
cartilagenous framework of nose.
Treatment
• Rhinoscleroma is caused by Frisch bacillus, i.e. Klebsiella
rhinoscleromatis. Mikulicz cells and Russel bodies are typical of the • Removal u n d e r LA/GA . Q

histopathological examination. • In children use o f oral positive pressure t e c h n i q u e called as'Par-


• Sarcoidosis resembles tuberculosis except for caseation, and ent's Kiss'technique is b e i n g practiced f o r removal o f a n t e r i o r
Kveim test and biopsy are diagnostic. nasal f o r e i g n body. (Scott Brown)

T a b l e 3.1: Types o f nasal p o l y p

Ethmoidal polyps Antrochoanal polyps


Age group = 30-60 years • Seen in children and young adults (male > female)
Sex = Male > Female • Maxillary antrum (floor and medial wall)
M/C Site - Ethmoid sinus (can also arise from middle turbinate and middle
meatus)

Etiology; Allergy (M/C) Etiology = Allergy + Infection


On examination - B/L Multiple, smooth, glistening sessile or pedunculated On examination - U/L, pale, white, translucent
polyps It has 3 parts:
Lining epithelium initially is columnar, later due t o trauma it undergoes • Antral
squamous metaplasia • Choanal
• Nasal
Symptoms U/L Nasal blockage (which can become bilateral when polyp grows
Presenting symptom B/L nasal blockage into nasopharynx and obstructs opposite choana)
• Hyponasal voice
• Nasal discharge
• Conductive deafness due to (blockage of Eustachian tube)

Others
• Partial/complete loss of smell
• Pain over nasal bridge forehead/cheek
• Postnasal drip Broadening of nose (frog face deformity)

Note: Polyps do not present with Epistaxis/bleeding Anterior Rhinoscopy: It is not visualized as they are posterior.
O/E Posterior Rhinoscopy - Smooth, white spherical masses seen in
• Anterior Rhinoscopy—multiple, smooth, bluish gray grape-like masses. choana
• On probing - All polyps are insensitive t o probing and donot bleed.

Contd...
CHAPTER 3 Granulomatous Disorders of Nose, Nasal Polyps and Foreign Body in Nose

Contd...
Ethmoidal polyps Antrochoanal polyps

Investigation- X-ray of PNS Treatment


(Waters view and C T t o exclude* bony erosion)
Treatment
Surgical Medical Surgical
• Effective only in 5 0 % cases 4 treatment
Drug used - Intranasal corticosteroids No role

Medical T/t Surgical Management


• Simple polypectomy: Indicated in case of one/two polyps • Intranasal polypectomy Indicated in - young patients with
• Intransal ethmoidectomy: Done when polyps are multiple and sessile. Since incomplete dentition.
it is a blind procedure it can give rise to orbital complications • Caldwell-Luc operation (i.e. opening the maxillary artrum
• Extranasalethmoidectomy: Indicated when polyps recurr after intranasal through canine fossa by sublabial approach). It is done if there
procedures [Howarth's incision (Incision given medial to the inner canthus is recurrence and age of patient is more than 17 years.
of the eye)] Nowadays Antrochoanal polyp is being treated by FESS
• Horgans Transantrulethmoidectomy: When polypoidal changes are also
seen in the maxillary antrum.
• Endoscopic sinus surgery: It is the latest procedure for removal of small
polyps under good illumination using 0° and 30° sinoscope i.e. Functional
endoscopic sinus surgery (FESS).

Complications Later

• Nasal i n f e c t i o n (vestibulitis) a n d sinusitis M a g g o t s m a y crawl o u t o f nose a n d there is f o u l smell.


• Rhinolith f o r m a t i o n .
• I n h a l a t i o n i n t o t h e t r a c h e o b r o n c h i a l tree Complications
• D e s t r u c t i o n o f nose, sinuses, soft tissues o f face, palate a n d
| RHINOLITH eyeball.
• Fistulae in nose a n d palate
• It is stone f o r m a t i o n in t h e nasal cavity.
• Death occurs d u e t o m e n i n g i t i s
• Rhinolith f o r m s a r o u n d t h e nucleus o f a small exogenous
foreign body or blood clot w h e n calcium, magnesium and
Treatment
p h o s p h a t e d e p o s i t a r o u n d it.
Instillation o f c h l o r o f o r m w a t e r a n d oil in nose a n d p l u g g i n g t h e
Clinical Features nose so t h a t m a g g o t s d o n o t crawl o u t .
• M o r e c o m m o n in adults. • Patient s h o u l d be isolated
Presents as unilateral nasal o b s t r u c t i o n a n d f o u l s m e l l i n g dis-
charge (often b l o o d stained) | ALSO KNOW
U l c e r a t i o n o f t h e s u r r o u n d i n g m u c o s a m a y lead t o f r a n k
epistaxis a n d neuralgic p a i n . For u n d e r g r a d u a t e students:

Treatment Or Causes of unilateral blood stained nasal discharge in a child


Removal u n d e r GA. S o m e hard a n d irregular rhinolitis m a y require Foreign body in nose
lateral r h i n o t o m y Rhinolith
Nasal diphtheria
| N A S A L M Y I A S I S ( M A G G O T S IN N O S E ) Nasal myiasis
Acute/Chronic unilateral sinusitis J
• It results f r o m t h e prescence o f gya o f flies particularly Chrys-
o m y i a species i n t h e nose w h i c h p r o d u c e u l c e r a t i o n a n d
d e s t r u c t i o n o f nasal structure.
Some important points to remember in a case of nasal polyp
• M o s t l y seen i n a t r o p h i c rhinitis w h e n t h e mucosa becomes 1. If a polypus is red and fleshy, friable and has granular surface,
insensitive t o flies laying eggs inside ; : especially in older patients, think of malignancy.
2. Simple nasal polyp may masquerade a malignancy under-
Clinical features neath. Hence all polypi should be subjected to histology.
3. A simple polyp in a child may be a glioma, an encephalocele
Initial
or a meningoencephalocele. It shold always be aspirated
• 3-4 days m a g g o t s p r o d u c e and fluid examined for CSF Careless removal of such polyp
• Intense i r r i t a t i o n would result in CSF rhinorrhoea and meningitis.
Sneezing 4. Multiple nasal polypi in children may be assoicated with
• Lacrimation mucoviscidosis.
• Headache 5. Expistaxis and orbital symptoms associated with a polyp
• T h i n b l o o d stained discharge should always arouse the suspicion of malignancy.
24 T SECTION I Nose and Paranasal Sinuses

QUESTIONS

68-year-old C h a n d u is a d i a b e t i c a n d p r e s e n t e d w i t h 14. All o f t h e following a r e t r u e a b o u t a n t r o c h o n a l p o l y p ,


b l a c k , foul s m e l l i n g d i s c h a r g e f r o m t h e n o s e . E x a m i - except: [TN07]
n a t i o n r e v e a l e d b l a c k i s h d i s c o l o r a t i o n o f t h e inferior a. Single
t u r b i n a t e . T h e d i a g n o s i s is: [AIIMS 99] b. Unilateral
a. Mucormycosis b. Aspergillosis c. Premalignant
c. Infarct o f inferior turbinated. Foreign body d. Arises from maxillary a n t r u m
2. I D D M p a t i e n t p r e s e n t s w i t h s e p t a l p e r f o r a t i o n o f n o s e 1 5 . A n t r o c h o a n a l p o l y p is c h a r a c t e r i z e d by: [PGI Dec 03]
w i t h b r o w n i s h black d i s c h a r g e p r o b a b l e d i a g n o s i s is: a. Usually bilateral
a. Rhinosporidiosis b. Aspergillus [AI97;RJ06] b. It is o f allergic origin
c. Leprosy d . Mucormycosis c. It arises from maxillary antrum
R h i n o s p o r i d i o s i s is c a u s e d by: [PGI 99; UP 00] d. Caldwell-Luc operation is treatment of choice in recurrent
a. Fungus b. Virus cases
c. Bacteria d . Protozoa e. Recurrence is c o m m o n
True s t a t e m e n t a b o u t R h i n o s p o r i d i o s i s is: [Al 99] 16. The most appropriate m a n a g e m e n t for antrochoanal
a. Most c o m m o n organism is klebsiella rhinoscleromatis p o l y p in children is: [AIIMS 02]
b. Seen only in i m m u n o c o m p r o m i s e d patients a. Caldwell-Luc operation b. Intranasal p o l y p e c t o m y
c. Presents as a nasal polyp c. Corticosteroids d. Wait and w a t c h
d. Can be diagnosed by isolation of organism 1 7 . T r e a t m e n t of choice for a n t r o c h o a n a l p o l y p in a 10-year-
In r h i n o s p o r i d i o s i s , t h e following is t r u e : [PGI 99] o l d child is: [PGI 96]
a. Fungal granuloma b. Grayish mass a. Caldwell-Luc operation
c. Surgery is t h e treatment d. Radiotherapy is treatment b. Intranasal p o l y p e c t o m y
Ideal t r e a t m e n t of r h i n o s p o r i d i o s i s is: [AIIMS 97] c. Conservative treatment till 16 years
a. Rifampicin b. Excision w i t h cautery at base d. Exploratory rhinotomy
c. Dapsone d. Laser 18. T r e a t m e n t for recurrent a t r o c h o a n a l p o l y p : [MP 2007]
7. R h i n o s c l e r o m a t i s is c a u s e d by: [PGI 99] a. Caldwell Luc operation b. FESS
a. Klebseilla b. A u t o i m m u n e c. Simple polypectomy d. Both a and b
c. Spirochetes d. Rhinosporidium 1 9 . T h e c u r r e n t t r e a t m e n t of c h o i c e for a large a n t r o c h o a n a l
Mikulicz cell a n d russel b o d i e s a r e c h a r a c t e r i s i t c of: p o l y p in a 30-year-old m a n is: [AIIMS Nov 05]
[JIPMER02;Bihar06] a. Intranasal p o l y p e c t o m y
a. Rhinoscleroma b. Rhinosporidiosis b. Caldwell-Luc operation
c. Plasma cell disorder d. Lethal midline granuloma c. FESS (Functional Endoscopic Sinus Surgery)
Atrophic dry nasal mucosa, extensive encrustations d. Lateral rhinotomy and excision
w i t h w o o d y ' h a r d e x t e r n a l n o s e is s u g g e s t i v e of [MH 05] 2 0 . W h i c h o f t h e f o l l o w i n g s t a t e m e n t s is n o t c o r r e c t f o r
a. Rhinosporidiosis b. Rhinoscleroma Ethmoidal polyp: [AIIMS 02]
c. Atrophic rhinitis d. Carcinoma o f nose a. Allergy is an etiological factor
1 0 . Apple-jelly n o d u l e s o n t h e n a s a l s e p t u m are f o u n d in b. Occur in the first decade of life
c a s e of: [MP 05] c. Are bilateral
a. Tuberculosis b. Syphilis d. Are often associated w i t h bronchial asthma
c. Lupus vulgaris d . Rhinoscleroma 2 1 . R e g a r d i n g e t h m o i d a l p o l y p , w h i c h o n e of the f o l l o w i n g
1 1 . A b o u t n a s a l s y p h i l i s the f o l l o w i n g is t r u e : [PGI 02] is t r u e : [Kolkata 05]
a. Perforation occurs in septujri a. Epistaxis
b. Saddle nose deformity may occur b. Unilateral
c. In n e w b o r n , it presents as snuffles c. <10 years
d. Atrophic rhinitis is a complication d. Associated w i t h bronchial asthma
e. Secondary syphilis is t h e c o m m o n association 22. R e c u r r e n t p o l y p s a r e s e e n in: [UP 07]
12. Killian t e r m is u s e d for w h i c h of t h e following p o l y p a. Antrochoanal polyp b. Ethmoidal polyp
a. Ethmoidal b. Antrochoanal /UP 057 c. Nasal polyp d. Hypertrophic turbinate
c. Tonsillar cyst d . Tonsillolith 23. In a p a t i e n t w i t h m u l t i p l e bilateral n a s a l p o l y p s w i t h
1 3 . All t h e f o l l o w i n g are true of a n t r o c h o a n a l p o l y p e x c e p t : X-ray s h o w i n g opacity in the p a r a n a s a l sinuses. T h e treat-
[AI94] m e n t c o n s i s t s of all of t h e following e x c e p t : [AIIMS 02]
a. C o m m o n in children b. Single and Unilateral a. Epinephrine b. Corticosteroids
c. Bleeds o n t o u c h d. Treatment involves Avulsion c. Amphoterecin B d. Antihistamines


CHAPTER 3 Granulomatous Disorders of Nose, Nasal Polyps and Foreign Body in Nose

24. Patient w i t h e t h m o i d a l p o l y p u n d e r g o e s p o l y p e c t o m y . 3 0 . M o s t c o m m o n c a u s e of U/L m u c o p u r u l e n t r h i n o r r h e a in


P r e s e n t s 6 m o n t h s later w i t h e t h m o i d a l p o l y p . Correct a c h i l d is: [Kolkata01/FMGE2013]
Rx: [AIIMS 95] a. Foreign body
a. Intranasal e t h m o i d e c t o m y b . Extranasal e t h m o i d e c t o m y b. Adenoids which are blocking the airways
c. Caldwell-Luc procedure d. Polypectomy c. Deviated nasal septum
2 5 . "Bernoulli's t h e o r e m " explains: [UP 07] d. Inadequately treated acute frontal sinusitis
a. Nasal p o l y p b. Thyroglossal cyst 3 1 . W h a t is a R h i n o l i t h : [Al 91]
c. Zenker's diverticulum d. Laryngomalacia a. Foreign body in nose
26. S a m t e r ' s t r i a d includes:
b. Stone in nose
a. Nasal polyps
c. Deposition of calcium around foreign b o d y in nose
b. Aspirin sensitivity

d. Misnomer
c. Bronchiectasis
32. M a g g o t s in the n o s e a r e b e s t t r e a t e d by: [Al 98; 96]
d. Bronchial asthma
a. Chloroform diluted w i t h water
e. Immunodeficiency
b. Liquid paraffin
27. In Caldwell-Luc operation the nasoantral w i n d o w is m a d e
c. Systemic antibiotics
through: [TN04]
d. Lignocaine spray
a. Superior meatus b. Inferior meatus
33. Frish bacillus c a u s e s : [FMGE20I3]
c. Middle meatus d. None o f the above
a. Rhinosleroma b. Rhinosporidiosis
28. M o s t c o m m o n c o m p l i c a t i o n o f Caldwell-Luc o p e r a t i o n
c. Rhinophyma d. Lupus vulgaris
is: [APOO]
a. Oroantral fistula b. Infraorbital nerve palsy 3 4 . A R a p i d l y d e s t r u c t i v e infection o f n o s e a n d p a r a n a s a l

c. Hemorrhage d. Orbital cellulitis s i n u s e s in d i a b e t i c s is: [DNB 2010]


2 9 . A b o u t f o r e i g n b o d y in a c h i l d t r u e s t a t e m e n t is: a. Histoplasmosis b. Sporotrichosis
a. Unilateral fetid discharge [PGI June 03] c. Mucormycosis d. Sarcoidosis
b. Presents w i t h unilateral nasal obstruction 3 5 . Multiple nasal p o l y p in c h i l d r e n s h o u l d g u i d e t h e clini-
c. Has torrential epistaxis cian to s e a r c h for u n d e r l y i n g : [AP PG 2012]
d. Inanimate is more c o m m o n t h a n animate a. Mucoviscidosis b. Celiac disease
e. Always removed under GA c. Hirschsprung's disease d. Sturge weber syndrome

EXPLANATIONS AND REFERENCES


1. A n s . is a i.e. M u c o r m y c o s i s
2. A n s . is d i.e. M u c o r m y c o s i s Ref. Dhingra5th/edp 1756th/edp 159;Mohan Bansalp 317
• M u c o r m y c o s i s is f u n g a l i n f e c t i o n o f nose a n d paranasal sinuses w h i c h m a y prove rapidly fatal if u n t r e a t e d .
• It is seen in u n c o n t r o l l e d diabetes or in t h o s e t a k i n g i m m u n o s u p p r e s s i v e drugs.
• It presents as black necrotic mass filling t h e nasal cavity a n d e r o d i n g t h e s e p t u m a n d hard palate.
• Treatment is by a m p h o t e r i c i n B a n d surgical d e b r i d e m e n t .
Q

note WB/tKKBasStKtSBSBMBtti P^^^^9BH9BHiHIHBMHiHHBAHMBHBHBR9MliSSBII^HH9H


Most c o m m o n fungal infection of nose is Aspergillosis. 0

3. A n s . is a i.e. F u n g u s
4 . A n s . is c i.e. P r e s e n t s a s a n a s a l p o l y p
5. A n s . is a a n d c i.e. F u n g a l g r a n u l o m a ; a n d S u r g e r y is t h e t r e a t m e n t
6. A n s . is b i.e. Excision w i t h c a u t e r y a t b a s e Ref. Dhingra Sth/edp 174,6th/edp 158,159; Mohan Bansal 316,317

R h i n o s p o r o d i o s i s is a F u n g a l G r a n u l o m a . 0

Agent : Rhinosporidium seeberi. 0

Source : Contaminated water of ponds. 0

Presents as : Leafy p o l y p o i d a l mass, pink to purple in color s t u d d e d w i t h w h i l e d o t s (Strawberry appearance).


0

It is h i g h l y vascular a n d bleeds o n t o u c h . 0

Patients c o m p l a i n o f b l o o d t i n g e d nasal discharge/epistaxis


Diagnosis is m a d e by b i o p s y
Biopsy shows several sporangia filled w i t h spores.
T r e a t m e n t - C o m p l e t e excision o f mass w i t h d i a t h e r m y knife a n d c a u t e r i z a t i o n o f base. 0

Recurrence is c o m m o n after surgery.


26 | SECTION I Nose and Paranasal Sinuses

7. A n s . is a i.e. Klebsiella
8. A n s . is a i.e. R h i n o s c l e r o m a
9. A n s . is b i.e. R h i n o s c l e r o m a Ref. Dhingra 5th/ed p 172,6th/ed 156; Scott Brown's 7th/ed Vol 2 Chapter 115 pp 1462,1463;
Mohan Bansal p 315

Rhinoscleroma

• It is a c h r o n i c g r a n u l a m a t o u s disease caused by gram-negative Frisch bacteria Klebsiella rhinoscleromatis.


• It is seen mostly in p o o r s o c i o e c o n o m i c c o n d i t i o n s a n d c o m m o n l y occurs in Europe, Pakistan, Indonesia a n d South America.

Pathologically

• M i k u l i c z cells or f o a m cells a n d Russell b o d i e s are its characteristic features.


• Russell b o d i e s are h o m o g e n e u s e o s i n o p h i l i c inclusion bodies f o u n d in plasma cells.
• In a p a t i e n t presenting w i t h a t r o p h i c d r y nasal mucosa, extensive c r u s t i n g a n d w o o d y hard external nose:
- Rhinoscleroma s h o u l d be suspected.
- For more details kindly see the preceding text
1 0 . A n s . is c i.e. L u p u s v u l g a r i s Ref. Dhingra 5th/ed p 173,6th/ed p 157; Scott Brown's Otolaryngology 7th/ed Vol 2,
Chapter 115p 1456; Currentoto rhinology 2nd/edp 261; Mohan Bansalp316
• Lupus vulgaris is t h e chronic a n d m o r e c o m m o n f o r m o f t u b e r c u l a r i n f e c t i o n affecting t h e skin a n d m u c o u s m e m b r a n e o f nose
• Apple-jelly appearances are b r o w n g e l a t i n o u s nodules a n d are t y p i c a l skin lesions o f lupus.
1 1 . A n s . is e i.e. S e c o n d a r y syphilis is t h e c o m m o n a s s o c i a t i o n Ref .Dhingra 5th/ed p 173,6th/edp 157
Nasal syphilis may be:
• Acquired: - Primary, e.g. chancre in vestibule
- Secondary, e.g. simple rhinitis, crusting and fissuring leading to atrophic rhinitis
- Tertiary, e.g. Gumma leads to septal perforation and saddle nose deformity (due to collapse of nasal bridge)
• Congenital: - Early (first 3 m o n t h s ) : Presenting as snuffles, p u r u l e n t nasal discharge, fissuring e x c o r i a t i o n .
- late ( a r o u n d p u b e r t y ) : G u m m a in s e p t u m a n d o t h e r stigmatas.
• Teritary syphilis is a c o m m o n association: p r i m a r y a n d secondary syphilis are rare association in nasal syphilitis.
• Septal p e r f o r a t i o n occurs in b o n y part in case o f syphilis.
1 2 . Ans.is b i.e. A n t r o c h o a n a l Ref. Internet search
Killian's p o l y p is t h e n a m e g i v e n t o a n t r o c h o a n a l p o l y p based o n Gustain Killians
1 3 . A n s . is c i.e. B l e e d s on t o u c h
1 4 . A n s . is c i.e. P r e m a l i g n a n t
1 5 . Ans.is c a n d d i.e. It arises f r o m m a x i l l a r y a n t r u m ; Caldwell-Luc o p e r a t i o n is t r e a t m e n t of c h o i c e in r e c u r r e n t c a s e s
Ref. Dhingra 5th/ed pp 186,187,6th/edp 174,175; Scott Brown 7th/ed Vol 2 Chapter 121 p 1554; Mohan Bansal pp 308,309
• Nasal p o l y p s are non-neoplastic masses o f e d e m a t o u s nasal or sinus mucosa. They d o n o t bleed o n t o u c h a n d are insensitive
0

t o p r o b i n g a n d never present w i t h epistaxis or b l e e d i n g f r o m nose.


• Recurrence is u n c o m m o n in case o f a n t r o c h o a n a l p o l y p .
• A n t r o c h o a n a l polyps arise f r o m maxillary a r t r u m a n d t h e n g r o w i n t o choana and nasal cavity.
[For details of Antrochoanal polyps see text]

16. A n s . b i.e. I n t r a n a s a l p o l y p e c t o m y
1 7 . A n s . b i.e. I n t r a n a s a l p o l y p e c t o m y
Ref. Dhingra 5th/ed p 188,6th/ed p 174,175; Tuli Ist/ed p 175; Maqbool 11 th/ed p 206; Turner 10th/ed p 55

M a n a g e m e n t O p t i o n s for A n t r o c h o a n a l Polyp

Avulsion of Polyp
• The t r e a t m e n t o f antrochoanal p o l y p is its c o m p l e t e removal a l o n g w i t h t h e removal o f t h e lining of t h e sinus (to avoid recurrence).
• Sometimes it is possible t o grasp the stalkand avulse the polyp, b u t most of the t i m e it fails t o remove the p o l y p and its lining completely
• Therefore, it is n o t t h e t r e a t m e n t o f choices

Intranasal Polypectomy
It was t h e t r e a t m e n t o f choice for all age g r o u p s prior t o t h e a d v e n t o f e n d o s c o p i c sinus surgery a n d is still t h e t r e a t m e n t o f choice
in t h o s e set-ups w h e r e endoscopic surgery is n o t practised.

Caldwell-Luc Operation
• It is i n d i c a t e d if t h e r e is a recurrence a n d t h e age o f t h e p a t i e n t is m o r e t h a n 17 years
• Nowadays w i t h FESS available - Caldwell-Luc o p e r a t i o n is a v o i d e d
CHAPTER 3 Granulomatous Disorders of Nose, Nasal Polyps and Foreign Body in Nose

Functional Endoscopic Sinus Surgery (FESS)


• These days-it is t h e t r e a t m e n t o f choice f o r all p o l y p s a n d has superceded o t h e r modes o f p o l y p removal.
• N o w in n o n e o f t h e questions—FESS is g i v e n as an o p t i o n — T h e r e f o r e w e w i l l go w i t h t h e second best for children w h i c h is
intranasal p o l y p e c t o m y .
1 8 . A n s . is d i.e. B o t h a a n d b Ref. Dhingra 5th/ed p 188,6th/ed175,2nd/ep 182,183; Tuli 1 st/ed p 175,2nd/ed pi82, 183;
Turner lOth/edp 55
As e x p l a i n e d in t h e previous q u e s t i o n
For m a n a g e m e n t of recurrent polyps, w e have 2 o p t i o n s , i.e.
[— ^ 1

Caldwell-Luc o p e r a t i o n Functional endoscopic sinus surgery


Can be done only in patients > 17 yrs of age Can be done in all age groups

Many complications Fewer complications
Not preferred nowadays TOC for recurrent polyps

NOTE

The question says "Treatment for Recurrent Antrochoanal polyp - therefore we have selected option d. i.e. both Caldwell-Luc and FESS
If the question would have been - Treatment of choice for Recurrent Antrochoanal polyp, then the answer would be - option'b'i.e. FESS

1 9 . A n s . i s c i.e. F E S S Ref. Dhingra5th/edp 188,6th/edp 175;Maqbool 11 th/edp206


FESS is Functional Endoscopic Sinus Surgery.
• Current t r e a t m e n t o f choice o f a n t r o c h o a n a l p o l y p is endoscopic sinus surgery w h i c h has superceded o t h e r m o d e s o f p o l y p
removal.
• In this p r o c e d u r e all polyps are r e m o v e d u n d e r e n d o s c o p i c c o n t r o l especially f r o m t h e key area o f t h e o s t e o m e a t a l c o m p l e x .
This p r o c e d u r e helps t o preserve t h e n o r m a l f u n c t i o n o f t h e sinuses. FESS can be d o n e u n d e r local anesthesia a l t h o u g h general
anesthesia is preferred
• Caldwell-Luc o p e r a t i o n is a v o i d e d these days.
2 0 . A n s . is b i.e. Occurs in t h e first d e c a d e o f life Mohan Bansal p 310
2 1 . A n s . is d i.e. a s s o c i a t e d w i t h b r o n c h i a l a s t h m a
Ref. Scott Brown 7th/ed Vol 2 Chapter 121 p 1550; Dhingra 5th/ed pp 185,186,6th/ed, p 172; Logan Turner 10th/edp373;
Mohan Bansal p 310
2 2 . A n s . is b i.e. E t h m o i d a l p o l y p Mohan Bansalp 308

Ethmoidal Polyps

• They are mostly seen m adults. 0

• Etiology—usual cause of ethmoidal polyps is allergy 0

• "Allergic nasal polyps are rarely, if ever seen in childhood. They are only seen in childhood in association with
mucoviscidosis." —Turner 10th/ed p373
• E t h m o i d a l polyps are also associated w i t h :
- Bronchial asthma
- Aspirin intolerance
- Cystic fibrosis
- Nasal mastocystosis
- Syndromes like: Kartageners/Young syndrome/Churg-Strauss syndrome
• It is generally b i l a t e r a l .
0

• A p p e a r as m u l t i p l e , sessile or p e d u n c u l a t e d masses like a b u n c h o f g r a p e s


0

• Insensitive t o t o u c h a n d d o n o t bleed o n p r o b i n g . 0

• Recurrence is c o m m o n after r e m o v a l . 0

- Friends m a n y questions have been asked o n differences b e t w e e n e t h m o i d a l p o l y p s a n d a n t r o c h o a n a l polyps


- A n d believe m e s o m e t i m e s in exams t h e r e is so m u c h m i n d b l o c k t h a t simple questions like this seem c o n f u s i n g . So I a m
g i v i n g y o u a m n e m o n i c w h i c h w i l l h e l p y o u in r e m e m b e r i n g t h e characteristics o f e t h m o i d a l polyps.

Ethmoidal Polyps - Features

,^Jnemonic
Adult B M R
Adult - It is seen in adults
28 |_ SECTION I Nose and Paranasal Sinuses

2 3 . A n s . is c i.e. A m p h o t e r i c i n B " Ref. Dhingra 5th/ed p 186,6th/edp 173; Logan and Turner Wth/edpp 52,54
• This p a t i e n t is having e t h m o i d a l p o l y p (because polyps are m u l t i p l e a n d bilateral)
• M a i n e t i o l o g y of polyps is allergy.
• Medical t r e a t m e n t o f polyps is t h e same as t h a t f o r allergic rhinitis w h i c h consists of:
- Antihistaminics
- S t e r o i d s — h e l p f u l in patients w h o c a n n o t t o l e r a t e a n t i h i s t a m i n e or have asthma a l o n g w i t h polyps. It is also useful t o
p r e v e n t recurrence after surgery
- Decongestants such as epinephrine, phenylephrine, xylometazoline, etc.
• A n t i f u n g a l s (e.g. A m p h o t e r i c i n B) have n o role in t r e a t m e n t o f polyps.
2 4 . A n s . is b i.e E x t r a n a s a l e t h m o i d e c t o m y Ref. Dhingra 5th/edp 186,6th/edp 173
T r e a t m e n t of e t h m o i d a l p o l y p
• Simplepo/ypecfomy: W h e n t h e r e are o n e or t w o p e d u n c u l a t e d polyps.
• Intranasal ethmoidectomy: Indicated w h e n polyps are m u l t i p l e a n d sessile.
• Extranasalethmoidectomy:Tr\\s is indicated w h e n polyps recur after intranasal procedures.
• Transantralethmoidectomy: Indicated w h e n i n f e c t i o n a n d p o l y p o i d a l changes are also seen in t h e maxillary a n t r u m . In this case
a n t r u m is o p e n e d by Caldwell-Luc a p p r o a c h a n d t h e e t h m o i d a l air cells a p p r o a c h e d t h r o u g h t h e m e d i a l wall o f t h e a n t r u m .

NOTE

These days, ethmoidal polypi are removed by endoscopic sinus surgery (FESS) which is theTOC.

2 5 . A n s . is a i.e. Nasal p o l y p Ref. Fundamental of Physics, Halliday Resnic 6th/ed p 356; Turner 10th/ed p 54
"Bernoulli's theorem states that if the speed of a fluid element increases as it travels along a horizontal streemline, the fresher
of the fluid must decrease and conversely." — F u n d a m e n t a l of Physics, Halliday Resnic 6th/ed, p 356
Nasal polyps f o l l o w Bernoulli's t h e o r a m a s —
"The increased speed o f t h e air f l o w i n g t h r o u g h t h e nose decreases t h e pressure in t h e nasal cavity (Bernoulli's t h e o r e m ) w h i c h
pulls d o w n t h e polyp."
2 6 . A n s . is a, b a n d d i.e. Nasal p o l y p s ; A s p i r i n s e n s i t i v i t y ; a n d B r o n c h i a l a s t h m a
Ref. Scott Brown 7th/ed Vol 2 p 1472; Internet search - wikipedia.org; Mohan Bansal p 307
S a m t e r ' s t r i a d is a medical c o n d i t i o n consisting o f asthma, aspirin sensitivity, a n d nasal/ethmoidal polyposis. It occurs in m i d d l e
age (twenties and thirties are the most common onset times) a n d may n o t i n c l u d e any allergies.
• Most commonly, the first symptom is rhinitis.
• The disorder typically progresses t o asthma, t h e n polyposis, w i t h aspirin sensitivity c o m i n g last.
• The aspirin reaction can be severe, i n c l u d i n g an asthma attack, anaphylaxis, a n d urticaria in s o m e cases. Patients typically react
t o o t h e r NSAIDs such as i b u p r o f e n , a l t h o u g h paracetamol is generally considered safe.
• A n o s m i a (lack o f smell) is also t y p i c a l , as t h e i n f l a m m a t i o n reaches t h e o l f a c t o r y receptors in t h e nose.

Cause

The cause o f Samter's t r i a d is u n k n o w n , b u t it is w i d e l y believed t h a t t h e disorder is caused by an a n o m a l y in t h e arachidonic acid


cascade, w h i c h causes u n d u e p r o d u c t i o n o f leukotrienes, a series o f chemicals involved in t h e body's i n f l a m m a t o r y response. W h e n
p r o s t a g l a n d i n p r o d u c t i o n is b l o c k e d by NSAIDs like aspirin, t h e cascade shunts entirely t o leukotrienes, p r o d u c i n g t h e severe
allergy-like effects.

Treatment

• Medical: T h e preferred t r e a t m e n t n o w is d e s e n s i t i z a t i o n t o aspirin, u n d e r t a k e n at a clinic specializing in such t r e a t m e n t .


Leukotriene antagonists a n d i n h i b i t o r s ( m o n t e l u k a s t , zafirlukast, a n d z i l e u t o n ) are h e l p f u l in t r e a t i n g Samter's.
• Surgical: Occasionally, surgery may be required t o r e m o v e polyps,[3] a l t h o u g h t h e y t y p i c a l l y recur, particularly if desensitization
is n o t u n d e r t a k e n .
• Diet: A d i e t l o w in omega-6 oils (precursors o f arachidonic acid), a n d h i g h in omega-3 oils, may also help.
27. A n s is b i.e. Inferior m e a t u s Ref. Dhingra 5th/edp418,6th/edp 411,412; Tuli Ist/edp 495,2nd/edp 459;
Scott Brown 7th/ed Vol 2 pp 1491,1492

Caldwell-Luc Operation

• It was earlier d o n e in case o f chronic maxillary sinusitis w i t h an a i m t o r e m o v e " i r r e v e r s i b l y " d a m a g e d mucosa o f maxillary sinus
a n d t o facilitate gravitational drainage a n d aeration via an inframeatal a n t r o s t o m y .
• It was predominantly being used for persistent chronic rhinosinusitis w h e n medication, lavage and inferiormeatal antrostomy has failed.
• But it is n o w n o t being used - as it is n o t t h e normal ciliated respiratory epithelium which replaces t h e nasal mucosa b u t fibrous tissue,
• hich can o b l i t e r a t e t h e cavity a n d lead t o cyst f o r m a t i o n
• Contraindications - It s h o u l d n o t be d o n e in c h i l d r e n as it can d a m a g e t h e secondary d e n t i t i o n .
CHAPTER 3 Granulomatous Disorders of Nose, Nasal Polyps and Foreign Body in Nose

Surgery Done through j


1. Caldwell-Luc operation Inferior meatus

2. Antral puncture Inferior meatus

3. Dacryocystorhinostomy Middle meatus

2 8 . A n s . is b i.e. infraorbital n e r v e p a l s y Ref. Scott Brown 7th/ed Vol2p 1494


M/C C o m p l i c a t i o n o f Caldwell-Luc o p e r a t i o n is i n j u r y t o i n f r a o r b i t a l nerve w h i c h occurs is 2 1 % cases.
2 9 . A n s . is a, b a n d d i.e. U/L foetid d i s c h a r g e ; P r e s e n t s w i t h U/L n a s a l o b s t r u c t i o n ; a n d I n a n i m a t e s is MC t h a n a n i m a t e s
Ref. Dhingra 5th/ed 176,6th/edp 161; LT 10th/ed, p 62; Scott Brown 7th/ed, Vol 1,p1186

F o r e i g n B o d i e s in C h i l d r e n c a n b e

Animate Inanimate
• • Examples are screwworms, larvae, maggots and black carpet • These are more common
beetles • Examples are peas, beans, dried pulses, nuts, paper, cotton wool and
pieces of pencil

Clinical Features

• Unilateral f o u l smelling discharge in a child is p a t h o g n o m i c o f a f o r e i g n b o d y


• It can lead t o vestibulitis

Treatment

• Removal w i t h forceps or b l u n t h o o k u n d e r LA

Indications o f giving G A in Nasal Foreign B o d y R e m o v a l

• U n c o o p e r a t i v e or very apprehensive child


• T r o u b l e s o m e b l e e d i n g if t h e f o r e i g n b o d y is f i r m l y e m b e d d e d in g r a n u l a t i o n tissue
• Posteriorly placed f o r e i g n body.
• If a f o r e i g n b o d y is s t r o n g l y suspected b u t can't be f o u n d .
3 0 . A n s . is a i.e. Foreign b o d y Ref. Dhingra 5th/ed p 176,6th/edp 161; Logan Turner 10th/edp 63
"A unilateral nasal discharge is nearly always due to a foreign body and if discharge has an unpleasant smell, it is pathognomic."
—Logan Turner Wth/ed, p 63
"If a child presents with unilateral, foul smelling nasal discharge, foreign body must be excluded." —Dhingra Sth/ed p 176,6th/ed 161
3 1 . A n s . is c i.e. D e p o s i t i o n of c a l c i u m a r o u n d f o r e i g n b o d y in n o s e
Ref. Dhingra Sth/edp 176,6th/edp 161; Tuli 1st/ed,p 149; Scott-Brown 7th/edVol 1 p 1186; Mohan Bansal p 349
Rhinoliths are calcareous masses w h i c h result d u e t o d e p o s i t i o n o f salts-like c a l c i u m a n d m a g n e s i u m carbonates a n d p h o s p h a t e s
a r o u n d t h e nucleus o f a f o r e i g n b o d y .
For more details, see text part.
3 2 . A n s . is a i.e. C h l o r o f o r m d i l u t e d w i t h w a t e r Ref. Dhingra 5th/ed p 178,6th/ed p 162
Chloroform water or vapor must be instilled in order to anesthetize or kill the maggots and so release their grip from the skin.
• M a g g o t s are larval f o r m s o f flies, particularly o f t h e genus c h r y s o m y i a . 0

° Patient m a y present as a s i m p l e case o f epistaxis. 0

• M a g g o t s cause extensive d e s t r u c t i o n o f nose, sinuses, soft tissues t o face, palate or a r o u n d t h e n o s e . 0

• Death m a y occur f r o m m e n i n g i t i s . 0

3 3 . A n s . is a - R h i n o s c l e r o m a Ref. Dhingra 6th/ed p 156


Rhinoscleroma is a chronic g r a n u l o m a t o u s disease caused by Gram negative bacillus called Klebsiella rhinoscleromates o r Frisch
bacillus
-

34. A n s . is c i.e. M u c o r m y c o s i s Ref. Dhingra 6th/edp 159

Mucormycosis
• It is a f u r g a l i n f e c t i o n o f nose a n d paranasal sinuses w h i c h m a y p r o v e r a p i d l y fatal
• It is seen i n u n c o n t r o l l e d diabetes or in t h o s e t a k i n g i m m u n o s u p p r e s s i v e d r u g s
For m o r e d e t a i l s — r e f e r t o p r e c e d i n g t e x t .
3 5 . A n s . is a i.e. m u c o v i s c i d o s i s Ref. Dhingra 6th/edp 175
" M u l t i l e nasal p o l y p i in c h i l d r e n m a y be associated w i t h mucoviscidosis." —Dhindra 6th/edp175
4
C H T ER
Inflammatory Disorders of
asal Cavity

| RHINITIS Treatment

• Bed rest
Classification (Table 4.1)
• Vitamin C
T a b l e 4 . 1 : Classification o f rhinitis • Antihistaminics and anti-inflammatory drugs
o A n t i b i o t i c s if secondary i n f e c t i o n occurs.
Acute inflammation Chronic Inflammation

• Acute Rhinitis Specific


• Acute nasal Nasal syphilis, tuberculosis CHRONIC INFLAMMATORY CONDITIONS
diphtheria Lupus and leprosy o Nasal syphilis
• Rhinoscleroma
Tuberculosis of nose
• Rhinosporidiosis
Lupus vulgaris - Details discussed in c h a p t e r o n
• Sarcoidosis
Leprosy granulo matous disease o f t h e nose
Nonspecific: o
Rhinoscleroma
• Atrophic rhinitis
• Rhinitis sicca • Rhinosporidiosis •


5

• Rhinitis caseosa
Allergic
• Seasonal allergic rhinitis
11 A L L E R G I C RHINITIS

It is a n i m m u n o g l o b i n E (IgE) m e d i a t e d i m m u n o l o g i c a l

• Perennial allergic rhinitis
• Vasomotor rhinitis response o f nasal mucosa t o a i r b o r n e allergens.
Clinically allergic rhinitis is o f 2 types (Table 4.2).
T a b l e 4.2: Types o f allergic rhinitis
ACUTE INFLAMMATORY CONDITION
Seasonal Perennial

| A C U T E RHINITIS/CORYZA Symptoms appear in and Symptoms are present


around a particular season throughout the year
generally March-May or
• Frequently referred t o as common cold.
August-September.
• Seen in adults a n d school g o i n g c h i l d r e n .
• Caused by viruses specially rhinovirus, influenza a n d It is because of pollens of In this case-house dust,
parainfluenza virus, ECHO virus, adenoviruses a n d retroviruses some particular grass or perfumes, sprays, drugs,
• Sencondary invaders are Streptococci, Staphylococci, flowers which act as allergen tobacco, smoke, chemical,
Pneumococci, H. influenza a n d M. catarrhalis. fumes, etc. act as allergen
In morning symptoms Symptoms are not as severe as
Clinical Features are usually worse and are in seasonal type
aggravated by dry windy
• There is b u r n i n g / t i c k l i n g sensation at t h e back o f t h e nose condition
f o l l o w e d by nasal stuffiness, r h i n o r r h e a a n d sneezing.
• Low-grade fever. Pathogenesis
• I n i t i a l l y d i s c h a r g e is w a t e r y a n d p r o f u s e b u t b e c o m e s It is a t y p e I h y p e r s e n s i t i v i t y r e a c t i o n . It i n v o l v e s excessive
m u c o p u r u l e n t later d u e t o secondary bacterial invasion. p r o d u c t i o n o f IgE a n t i b o d i e s , i.e. i t is an atopic reaction.
CHAPTER 4 Inflammatory Disorders of Nasal Cavity
J 31
Clinical features • They have no effect o n nasal c o n g e s t i o n w h i c h is a late phase
reaction a n d is m e d i a t e d by cytokines a n d leukotrienes
No age or sex p r e d i l e c t i o n
• Side effects - Sedation, d r y m o u t h , decreased work
Onset is a t 12-16 years o f age (i.e. adolescence). Peak preva-
p e r f o r m a n c e . These side effects can b e r e d u c e d b y u s i n g
lence is d u r i n g t h i r d and f o u r t h decade.
second g e n e r a t i o n antihistaminies.
Patients present w i t h i t c h i n g o f eyes a n d nose, sneezing,
profuse w a t e r y discharge, postnasal drip, c o n c o m i t t a n t c o u g h - Corticosteroids
i n g a n d w h e e z i n g , nasal o b s t r u c t i o n .
M e c h a n i s m of a c t i o n : They act o n t h e late phase reaction a n d
p r e v e n t a significant influx o f i n f l a m m a t o r y cells. Corticosteroids
can be g i v e n e i t h e r
Noe: Rhinorrhea and sneezing are more c o m m o n in seasonal
rhinitis than perennial rhinitis Intranasally Systemically

• Minimal systemic absorption • Necessary for severe


Signs therefore no systemic side intractable symptoms
effects • Can cause all systemic side
Nose:
• Approved for use in children effects of steroids
Nasal mucosa is pale, b o g g y , h y p e r t r o p h i c a n d m a y a p -
• Its is the DOC treatment of • Can suppress HPA axis
pear b l u i s h .
allergic rhinitis in young adults therefore to be used with
Transverse crease is present o n t h e nose d u e t o u p w a r d
and children caution
r u b b i n g o f nose (allergic salute).
Turbinates are swollen. Drugs u s e d - Triamcinolone, Budesonide, Fluticasone a n d
Ear: Otitis m e d i a w i t h effusion d u e t o b l o c k a g e o f Eustachian Mometasone
t u b e is a possibility in c h i l d r e n
P h a r y n x : Granular pharyngitis. Decongestants
L a r y n x : Edema o f t h e vocal cords a n d hoarseness o f voice. Act o n a adrenergic receptors o f nasal mucosa a n d respiratory t r a c t
E y e s : Dark circles, i.e. allergic shiners are seen u n d e r t h e eyes. I
Vaso c o n s t r i c t i o n
Investigations

• B l o o d t e s t s : T TLC, T DLC (eosinophilia) Decrease t u r b i n a t e c o n g e s t i o n


• N a s a l s m e a r : Eosinophils seen 1
• S k i n t e s t s : Are d o n e t o i d e n t i f y t h e allergen: No effect o n rhinorrhea or sneezing I m p r o v e d nasal p a t e n c y
Prick test
Scratch test NOTE
I n t r a d e r m a l test Intranasal decongestant, i.e. oxymetazoline can cause rebound nasal
congestion and dependency if used for more than 3-4 days (rhinitis
medicamentosa) 0

Noe: Prick test is preferred over the others since the other two are
less reproducible, more dangerous and may give false positive Intranasal Cromolyn
result. It is used before t h e onset o f s y m p t o m s a n d c o n t i n u e d t h r o u g h o u t
t h e exposure. It is very safe mast cell stabilizer a n d prevents t h e i r
• R A S T ( R a d i o a l l e r g o s o r b e n t Test): Serum IgE m e a s u r e m e n t d e g r a n u l a t i o n despite t h e f o r m a t i o n o f a n t i g e n a n t i b o d y c o m p l e x .
is d o n e in vitro, ( n o t d o n e n o w )
• N e w e r tests like i m m u n o C a p are preferred. Leukotriene Inhibitor

They are b e i n g t r i e d for t h e m a n a g e m e n t o f allergic rhinitis b u t


Treatment
are less effective t h a n antihistamines a n d intranasal steroids, e.g.
Monteleukast.
Environment Control Like

Reduce h o u s e h o l d h u m i d i t y t o < 5 0 % , w a s h linens in h o t water, Immunotherapy


e l i m i n a t e cockroaches, remove carpets a n d pets. • Hyposensitization is d o n e w h e n o t h e r t y p e s o f t r e a t m e n t are
n o t effective.
Medical Measures/Pharmacotherapeutic Measurces
• Subcutaneous route is used t o give gradually increasing doses

Antihistaminics o f k n o w n allergen.
• I m m u n o t h e r a p y suppresses t h e f o r m a t i o n o f IgE and raises t h e
T h e y are f r e q u e n t l y used as a first-line t h e r a p y because m o s t
o f t h e m are available w i t h o u t a prescription titer o f IgG antibodies.

A n t i h i s t a m i n e s b l o c k t h e early phase reaction m e d i a t e d by • D i s a d v a n t a g e : It has t o be g i v e n f o r a sufficiently l o n g t i m e


histamines (2-3 year).
32 J_ SECTION I Nose and Paranasal Sinuses

Contraindications to Immunotherapy
NOTE
Coexistent a s t h m a
For undergraduate students—saline i r r i g a t i o n is an i m p o r t a n t
Patients t a k i n g (3-blocker adjuvant to treatment as it helps to avert intranasal stasis and reduces
O t h e r m e d i c a l / I m m u n o l o g i c a l disease crusting. Its use not only increases the efficacy of intranasal topical
Age < 5 yr medications but also improves ciliary function.

Pregnancy
Other Drugs which can be Used
Surgery • A n t i c h o l e n e r g i c s like i p r a t r o p i u m b r o m i d e as t h e y b l o c k
Ref. Scott Brown 7th/ed Vol2pp 1400,1401 p a r a s y m p a t h e t i c i n p u t a n d so decrease r h i n o r r h e a . A v o i d
• Nasal surgery m a y be r e q u i r e d w h e n there is a m a r k e d septal in patients o f n a r r o w angle g l a u c o m a , BPH or b l a d d e r neck
d e v i a t i o n or b o n y t u r b i n a t e e n l a r g e m e n t (Grade D), w h i c h obstruction.
• Azelastine spray - It w o r k s in case o f v a s o m o t o r rhinitis b u t has
makes t o p i c a l nasal sprays usage d i f f i c u l t .
a b i t t e r taste w h i c h precludes its f r e q u e n t use.
• It is never t h e first line o f t r e a t m e n t .
• Mucosal h y p e r t r o p h y (Grade C) is preferably dealt medically,
| H Y P E R T R O P H I C RHINITIS
since after surgery t h e p r o b l e m t e n d s t o recur w i t h i n m o n t h s .

Characterized by t h i c k e n i n g o f mucosa, submucosa, seromucinous


| V A S O M O T O R RHINITIS glands, p e r i o s t e u m a n d b o n e .

• N o n Allergic Rhinitis Symptoms


• Persists t h r o u g h o u t year
• Nasal o b s t r u c t i o n
• Tests o f nasal allergy are negative.
• Thick a n d sticky nasal discharge.

Pathogenesis
Signs
Parasympathetic overactivity.
• H y p e r t r o p h y o f t u r b i n a t e s : especially inferior t u r b i n a t e s .
• M u l b e r r y like a p p e a r a n c e of n a s a l m u c o s a is s e e n . 0

Symptoms
• Does n o t p i t o n pressure.
• M o r e c o m m o n in e m o t i o n a l l y unstable persons especially in • Shows little shrinkage w i t h vasoconstrictor drugs.
w o m e n o f 2 0 - 4 0 years.
• Paroxymal s n e e z i n g — j u s t after g e t t i n g o u t o f b e d in m o r n i n g . Treatment

• Nasal o b s t r u c t i o n . To relieve nasal o b s t r u c t i o n by r e d u c i n g t h e size o f t u r b i n a t e s by:


• Excessive clear r h i n o r r h e a . Cauterization
• Postnasal d r i p . Submucosal d i a t h e r m y
Cryosurgery
Signs Partial or t o t a l t u r b i n e c t o m y
-
S u b m u c o u s resection o f t u r b i n a t e bone/laser t r e a t m e n t .
• Congested a n d h y p e r t r o p h i c nasal mucosa.
• Mucosa o f t u r b i n a t e s m a y give m u l b e r r y like a p p e a r a n c e a n d
| RHINITIS SICCA
is pale t o d u s k y red in color.
• No eye s y m p t o m s seen. • Seen in patients w o r k i n g in h o t , d r y a n d d u s t y s u r r o u n d i n g s .
• The respiratory ciliated c o l u m n a r e p i t h e l i u m o f a n t e r i o r p a r t
Complications
o f nose u n d e r g o e s s q u a m o u s m e t a p l a s i a w i t h a t r o p h y o f
• Nasal p o l y p s e r o m u c i n o u s glands.
• H y p e r t r o p h i c rhinitis a n d sinusitis. •
Treatment
T r e a t m e n t ( T a b l e 4.3) • Correction o f o c c u p a t i o n a l s u r r o u n d i n g s
• A n t i b i o t i c a n d steroid o i n t m e n t
T a b l e 4 . 3 : T r e a t m e n t o f v a s o m o t o r rhinitis
• Nasal d o u c h i n g .
Medical Surgical

• Avoidance of provoking • Treatment of complications | ATROPHIC RHINITIS/OZAENA


symptoms • Vidian neurectomy
Chronic i n f l a m m a t o r y disease characterized by progressive a t r o p h y
• Avoidanceofprovokingsymptoms
o f t h e nasal mucosa a n d t h e u n d e r l y i n g b o n e o f t h e t u r b i n a t e s .
(oral/nasal decongestants)
T h e r e is associated e x c e s s i v e c r u s t i n g w h i c h leads t o nasal
• Topical and nasal steroids o b s t r u c t i o n in spite o f a b n o r m a l p a t e n c y o f nasal passages.
CHAPTER 4 Inflammatory Disorders of Nasal Cavity 33
Etiology The purpose of the solution is to loosen and remove the crusts
and the thick tenacious secretions.
2 5 % g l u c o s e in glycerin:
nemonic
F o l l o w i n g removal o f t h e crust t h e nose is p a i n t e d w i t h
HERNIA
2 5 % glucose in g l y c e r i n .
Hereditary
Glucose—Inhibits proteolytic organisms, Glycerine—is a
Endocrinal pathology—Starts at puberty. Stops after menopause
hygroscopic agent.
Racial factors—seen more in Whites and Yellow races
Nutritional deficiency: deficiency of vitamin A, D, E and iron may O t h e r L o c a l a n t i b i o t i c s : Kemicetine antiozaena solution: 1
be responsible for it m l contains c h l o r a m p h e n i c o l ( 9 0 m g ) , estradiol d i p r o p i o n a t e
Infective: Klebsiella ozanae, Diphtheriods, P. vulgaris, E. coli, (0.64 m g ) , Vit D2 (900IU) a n d p r o p y l e n e g l y c o l
Staphylococci, Streptococci Potassium i o d i d e : by m o u t h t o increase t h e nasal secretion
A u t o i m m u n e p r o c e s s — c a u s i n g destruction of nasal, H u m a n placental extract is g i v e n in t h e f o r m o f submucosal
neurovascular and glandular elements may be the cause injections
Other drugs:
Pathology Rifampicin, S t r e p t o m y c i n t o decrease t h e o d o r a n d crusts.
Estradiol spray t o T vascularity o f nasal mucosa
• Ciliated columnar epithelium is lost and is replaced by stratified
Placental extract i n j e c t e d submucosally.
squamous type.
• A t r o p h y o f s e r o m u c i n o u s glands. Surgical
• Turbinates are resorbed leading t o w i d e n i n g o f nasal chambers.
• Young's o p e r a t i o n : 0

Closure o f b o t h t h e nostril f o l l o w i n g elevation o f t h e nasal


Clinical Features
vestibular folds. They are o p e n e d after 6 m o n t h s .
• Seen in females at puberty. • Modified Young's o p e r a t i o n :
• Patient herself is anosmic but a foul smell comes from her making Partial closure o f t h e nostril leaving b e h i n d a 3 m m hole.
her a social outcast. This remains for a p e r i o d o f 2 years.
• Nasal obstruction (in spite of roomy nasal cavities due to large • N a r r o w i n g o f t h e nasal cavity by - ( L a u t e n s l a g e r ' s o p e r a t i o n )
crusts filling the nose) and epistaxis. Submucosal i n j e c t i o n o f t e f l o n paste
Insertion of fat, cartilage, b o n e or t e f l o n strips u n d e r t h e
Signs m u c o p e r i o s t e u m o f f l o o r a n d lateral wall o f nose
Section and medial displacement of lateral wall of nose
• Roomy nasal cavities w i t h greenish large crusts w i t h shriveled
turbinates.
| RHINITIS C A S E O S A / N A S A L C H O L E S T E A T O M A
• Septal p e r f o r a t i o n may be present.
• Nose m a y s h o w saddle d e f o r m i t y . It isa chronic inflammation o f t h e nose characterized by accumulation
• A t r o p h i c changes may be seen in t h e p h a r y n g e a l mucosa. o f offensive cheesy material r e s e m b l i n g c h o l e s t e a t o m a .
• A t r o p h i c changes may be seen in t h e larynx- Atrophic laryngitis
Features
• Eustachian t u b e o b s t r u c t i o n can lead t o hearing loss.
. Usually U/L
Investigations • The nose gets filled w i t h w h i t i s h offensive debris w i t h invasion
o f t h e b o n y structures a n d t h e soft tissues o f t h e face.
X-ray PNS (Water's v i e w ) — T h i c k e n i n g o f t h e walls o f t h e sinuses
Treatment
Treatment
• Removal o f debris by s c o o p i n g it o u t
Medical • Repeated irrigation
• W a r m n a s a l a l k a l i n e s o l u t i o n : 280 m l w a r m w a t e r + 1 p a r t o f
the following powder:
S o d i u m b i c a r b o n a t e (28.4 g) + S o d i u m b i b o r a t e (28.4 g) Rhinitis m e d i c a m e n t o s a "
+ 2 parts o f S o d i u m c h l o r i d e (56.7 g) {Remember—BBC) Caused by excessive use of topical decongestant nasal drops. 0


SECTION I Nose and Paranasal Sinuses

QUESTIONS

C o m m o n cold is c a u s e d p r i m a r i l y by: [Kamatka 94] All are t r u e a b o u t o z a e n a e x c e p t : [UP 03]


a. Viruses b. Bacteria a. C o m m o n in female
c. Fungi d. Allergy b. It is usually unilateral
E a r l y m e d i a t o r s of allergic rhinitis a r e : [PGI 03] c. Nasal cavity is filled w i t h greenish crusts
a. Leukotriene b. IL-4 d. Atrophic pharyngitis
c. IL-5 d. Bradykinin 10. A l k a l i n e d o u c h s o l u t i o n of n o s e d o e s not c o n t a i n :
e. PAF a. NaCI b. Na biborate
In Allergic rhinitis n a s a l m u c o s a is: [MP 03] c. N a H C 0 3 d. Glucose
a. Pale and swollen b. Pink and swollen 11. Young's o p e r a t i o n is d o n e for:
c. Atrophied d. Bluish and atrophied [JIPMER02][Jharkhand06, MP 03] [FMGE 2013]
All o f t h e f o l l o w i n g surgical p r o c e d u r e s are u s e d for a l - a. Allergic rhinitis b. Atropic rhinitis
lergic rhinitis e x c e p t : [AIIMS 04] c. Vasomotor rhinitis d. Idiopathic rhinitis
a. Radiofrequency ablation o f t h e inferior turbinate 12. V i d i a n n e u r e c t o m y is d o n e in: [CUPGEE 97]
b. Laser ablation o f t h e inferior turbinate a. Vasomotor rhinitis
c. Submucosal placement o f silastic in inferior turbinate b. Rhinintis sicca
d. Inferior t u r b i n e c t o m y c. Allergic sinusitis
All a r e i m p l i c a t e d in e t i o l o g y of a t r o p h i c rhinitis e x c e p t : d. Epistaxis
[DNB 02] 1 3 . M u l b e r r y a p p e a r a n c e of n a s a l m u c o s a l m e m b r a n e is s e e n
a. Chronic sinusitis b. Nasal deformity in: [MP2006]
c. DNS d. Strong hereditary factors a. Coryza
W h i c h of t h e f o l l o w i n g o r g a n i s m s is k n o w n t o c a u s e b. Atrophic rhinitis
A t r o p h i c Rhinitis: [MP 07] c. Maxillary sinusitis
a. Klebsiella pneumoniae d. Chronic hypertrophic rhinitis
b. Klebsiella ozaenae 14. Merciful a n o s m i a is s e e n i n : [FMGE2013]
c. Streptococcus pneumoniae a. Atrophic rhinitis
d. Streptococcus foetidis b. Allergic rhinitis
C a u s e of n a s a l o b s t r u c t i o n in a t r o p h i c rhinitis: c. Ethmoidal polyposis
[PGI 00, 97] d. Wegener's granulomatosis
a. Crusting b. Polyp 1 5 . Rhinitis m e d i m e n t o s a is d u e to: [NEET pattern]
c. Secretions d. DNS a. Nasal decongesants b. Steroid
All are t r u e r e g a r d i n g a t r o p h i c rhinitis e x c e p t : [AP 04] c. Anthihistamics d. Surgery
a. More c o m m o n in males 1 6 . Allergic rhinitis t r e a t m e n t i n c l u d e all e x c e p t :
b. Crusts are seen [NEET pattern]
c. Anosmia is notice a. Antibiotics b. Avoiding allergen
d. Young's operation is useful c. Corticosteroids d. Surgery

EXPLANATIONS AND REFERENCES


1. A n s . is a i.e. V i r u s e s Ref. Dhingra 5th/ed p 168,6th/edp 152; Mohan Bansal p 299
C o m m o n cold/coryza/Acute"Rhinitis is p r i m a r i l y caused by viruses, e.g. A d e n o v i r u s , Picorna virus, Rhinovirus, Coxsackie a n d ECHO
viruses. Secondary Invasion by Bacteria Occurs Later.

NOTE

Mode of infection: - Droplet infection


Incubation period: - 1-4 days
Clinical features: - Burning sensation - Rhinorrhea
- Sneezing - Nasal stuffiness

2. A n s . is a, b, c, d a n d e i.e. L e u k o t r i e n e ; IL4, I L 5 , B r a d y k i n i n ; a n d PAF


Ref. Robbin's 7th/ed pp 208,209; Current otolaryngology 2nd/ed pp 267,268; Dhingra 5th/ed p 180,6th/ed pi 67
Allergic rhinitis is Type 1 hypersensitivity reaction
i

CHAPTER 4 Inflammatory Disorders of Nasal Cavity _J 35

Pathology

In i n d i v i d u a l s w h o have g e n e t i c p r e d i s p o s i t i o n t o allergy

Allergen exposure

4- leads t o
IgE a n t i b o d y p r o d u c t i o n
4
Attaches t o mast cell (by Fc end)

On s u b s e q u e n t exposure t o t h e same allergen

4.
It attaches itself t o IgE a n t i b o d y ( w h i c h in t u r n is a t t a c h e d t o mast cell) by its F ab end
I
D e g r a n u l a t i o n o f mast cell
i
Release o f m e d i a t o r s
Like h i s t a m i n e , l e u k o t r i e n e , c y t o k i n e s
0 0 0

P r o s t a g l a n d i n s , Platelet a c t i v a t i n g f a c t o r
0 0

Called as Early phase/Humoral reaction

Early phase occurs w i t h i n 10-15 m i n s (max 30 mins) o f allergen exposure


It is d u e t o release o f m e d i a t o r s viz. h i s t a m i n e , c y t o k i n e , Prostaglandins, leukotrienes, platelet a c t i v a t i n g f a c t o r
Release o f h i s t a m i n e causes s y m p t o m s like - sneezing, r h i n o r r h e a , i t c h i n g , vascular p e r m e a b i l i t y , v a s o d i l a t a t i o n , g l a n d u l a r
secretion
4
Release o f cytokines a n d leukotrienes in t h e eraly phase causes i n f l u x o f i n f l a m m a t o r y cells (eosinophils)
4

Called as later phase o f cellular reaction

Occurs 2-8 hours after initial sensitization


Causes s y m p t o m s like Nasal c o n g e s t i o n a n d postnasal d r i p

3. A n s . is a i.e P a l e a n d s w o l l e n Ref. Scott Brown 7th/edVol2 Chapter]09p 1393; Dhingra Sth/edp 181,6th/edp 167
In allergic rhinitis - on e x a m i n a t i o n f o l l o w i n g features are seen.

In N o s e In E y e s In Ear In l a r y n x a n d p h a r y n x

• Nasal mucosa is pale, swollen, • Edema of lids • Retracted tympanic membrane • Child may show adenoid
hypertrophic and hyperplasia due to mouth
breathing

• Turbinates are swollen • "Congestion and cobble stone • Serous otitis media due to • Granular pharyngitis edema of
appearance of conjunctiva blockage of Eustachian tube vocal cords

• Watery and mucoid discharge is • Dark circles under the eye k/a • Hoarseness of voice
present allergic shiners

• Allergic salute i.e. a transverse crease is seen on nose due t o upward r u b b i n g of nose HIS %fti>UZ'4 «jSi .5>.' f r i t mSHA . . 6

4. A n s . is c i.e. S u b m u c o s a l p l a c e m e n t of sialistic in inferior t u r b i n a t e


Ref. Turner lOth/edpp 39,53; Scott Brown 7th/ed Vol 2, Chapter 104, pp 1400,1401
• Surgery is d o n e in a case o f allergic rhinitis w h e n o t h e r m e t h o d s have failed or w h e n t h e r e is m a r k e d septal
d e v i a t i o n o r b o n y t u r b i n a t e e n l a r g e m e n t w h i c h makes tossspical nasal spray usage d i f f i c u l t
• It s h o u l d never be used as first line o f t r e a t m e n t .
36 [_ SECTION I Nose and Paranasal Sinuses

Surgery

To relieve nasal obstruction To relieve rhinorrhea J


To relieve obstruction, turbinate reduction or turbinate resection is done by following Vidian neurectomy is done to relieve rhinorrhoea:
methods:

a. Submucosal diathermy—to fibrose, the vascular spaces of inferior turbinates a. Excision of vidian nerve
b. Cryosurgery b. Diathermy/division of vidian nerve
c. Laser cautery
d. Radiofrequency ablation
e. Partial excision o f turbinate
f. Submucosal t u r b i n e c t o m v
g. Radical turbinectomy

NOTE

Submucosal injection of teflon or placement of sialistic is the treatment option for Atropic rhinitis.

5. A n s . is c i.e. DNS Ref. Dhingra 5th/edp 170,6th/edp 153

Atrophic Rhitnitis

Primary Secondary
The exact etiology is not known Secondary rhinitis can be due to:
It can be due to: - Specific infections like:
H = H e r e d i t a r y factors • Syphilis
E = Endocrinal disturbance because it starts at puberty and cease after • Leprosy
menopasuse. Female > Male.Therefore endocrinal cause is possibility. • Rhinoscleroma
R = Racial factors -White and Yellow races are susceptible - Longstanding p u r u l e n t sinusitis
N = Nutritional deficiency of Vit A, D and iron - Radiotherapy to nose
I = Infective (organisms like Klebsiella ozaenae, diphtheroids, - Surgical removal of turbinates
P. vulgaris, E. coli, Staphylocci, Streptococci) - Deviated nasal s e p t u m
A = Autoimmune process

NOTE

DNS can lead to unilateral atrophic rhinitis on the wider side. 0

6. A n s . is a i.e. Klebsiella ozaenae


Ref. Scott Brown 7th/ed Vol 2 Chapter 115 p 1465; Dhingra 5th/ed p 170,6th/edp 154;Mohan Bansalp313
Organism k n o w n t o cause a t r o p h i c Rhinitis are:
• Coccobacillus foetidus ozaena
• Diphtheroid bacillus
• Klebsiella ozaenae j Ref. Scott Brown 7th/ed Vol 2 p 1465
• Bordettela bronchiseptica
• Pasteurella multocida -
• P. vulgaris
• £ coli
• Staphylococcus Ref. Dhingra 5th/edp 170,6th/edp 154
• Streptococcus
7. A n s . is a i.e. C r u s t i n g Ref. Logan Turner lOth/edp 40; Dhingra 5th/edpp 170,171 ;6th/edp 152,154;Mohan Bansalp 313
8. A n s . is a i.e. More c o m m o n in M a l e s Ref. Dhingra 5th/ed pp 146,l70andl71,6th/edp 152,154
9. A n s . is b i.e. It is u s u a l l y unilateral Ref. Scotts Brown 7th/ed Vol2 Chapter 115p 1465, Dhingra 6th/edp 153,164
1 0 . A n s . is d i.e. G l u c o s e Ref. Dhingra 5th/edp 171,6th/edp 154
1 1 . A n s . is b i.e. A t r o p h i c Rhinitis
Ref. Dhingra 5th/ed p 171 6th/edp 152; Scott Brown 7th/ed Vol 2, Chapter 155 p 1466; Mohan Bansal p 314

Atrophic Rhinitis

• Characterized by excessive c r u s t i n g a n d a t r o p h y o f nasal mucosa a n d t u r b i n a t e bones.


CHAPTER 4 Inflammatory Disorders of Nasal Cavity j37

M o r e c o m m o n in f e m a l e s . 0

Age—Usually starts at p u b e r t y a n d ceases after m e n o p a u s e . 0

It is always b i l a t e r a l except in case o f DNS w h e r e a t r o p h i c rhinitis is seen o n t h e w i d e r side.


0

Clinical Features

Due to excessive crusting and atrophy there is:


Foul smell f r o m nose
Patient herself is a n o s m i c 0

Nasal o b s t r u c t i o n 0

Epistaxis i f crusts are r e m o v e d


O/E: • N a s a l cavity is f u l l o f greenish/gray b l a c k dry c r u s t s 0

Atrophy of turbinates
Nasal mucosa is pale.
Pharynx—Atrophic pharyngitis may be seen
L a r y n x — A t r o p h i c laryngitis m a y be seen w h i c h can lead t o c o u g h a n d hoarseness o f voice
E a r — O b s t r u c t i o n o f Eustachian t u b e can cause serous otitis m e d i a
P N S — S m a l l / u n d e r d e v e l o p e d a n d have t h i c k walls. They appear o p a q u e o n X-ray

M a n a g e m e n t of Atrophic Rhinitis

Medical Surgical

• Warm nasal Alkaline • Young's operation


• Douche made up o f : - Closure o f t h e b o t h nostril f o l l o w i n g elevation o f
- Sodium bicarbonate (1 Part) Vestibular folds. It is opened after 6 months.
- Sodium biborate (1 Part) Added to 280 ml of water • Modified young's operation
- Sodium chloride (2 Part) — - Partial closure o f t h e nostril leaving behind a 3 m m hole
In water (280 ml) - This remains for a period of 2 years
2 5 % glucose in glycerin • Submucosal injection of teflon paste to narrow the cavity.
• Local antibiotics • Lautenslager's operation
• Kemicetine antiozaena solution - Surgical procedures aimed at medializing the lateral nasal
- Potassuim Iodide wall using substances like:
- Human placental extract Paraffin, teflon, Polythene, cartilage

A l s o k n o w : Kemicetine a n t i ozaena s o l u t i o n .
It contains: • Chloromycetin •

• Estradiol
• Vitamin D 2

1 2 . A n s . is a i.e. V a s o m o t o r Rhinitis Ref. Dhigra 5th/ed p 183;6th/edp 170; Scott Brown 7th/ed Vol 2 p 1412
Excessive r h i n o r r h e a in v a s o m o t o r rhinitis n o t corrected by medical t h e r a p y a n d b o t h e r s o m e t o t h e p a t i e n t , is relieved by s e c t i o n -
i n g t h e p a r a s y m p a t h e t i c s e c r e t o m o t o r fibers t o nose, i.e. v i d i a n n e u r e c t o m y .

NOTE

The parasympathetic/secretomotor supply o f t h e nose comes t h r o u g h the vidian nerve (also called the nerve of pterygoid canal). It is formed by
greater superficial petrosal branch of facial nerve j o i n i n g deep petrosal nerve derived f r o m plexus around internal carotid artery (sympathetic
nerve supply).

1 3 . A n s . is d i.e. C h r o n i c h y p e r t r o p h i c rhinitis Ref. Dhingra 5th/ed p 169; 6th/ed p 153; Mohan Bansalp 337
M u l b e r r y like appearance o f nasal mucosa is seen in chronic h y p e r t r o p h i c rhinitis
[For details k i n d l y see t h e p r e c e d i n g t e x t ]
1 4 . A n s . is a i.e. a t r o p h i c rhinitis Ref. Dhingra 6th/edp 154
In a t r o p h i c rhinitis, there is f o u l smell f r o m t h e nose, m a k i n g t h e p a t i e n t a social outcast t h o u g h t h e p a t i e n t himself is unaware o f
t h e smell d u e t o m a r k e d anosmia w h i c h accompanies t h e d e g e n e r a t i v e changes.This is called as merciful a n o s m i a .
1 5 . A n s . is a i.e. N a s a l d e c o n g e s t a n t s Ref. Mohan Bansal Ist/ed 331
Rhinitis m e d i c a m e n t o s a : T h e l o n g t e r m use o f cocaine a n d t o p i c a l nasal decongestants (cause r e b o u n d c o n g e s t i o n ) leads t o rhinits
medicamentosa.s
1 6 . A n s . is a antibiotics Ref. Dhingra 6th/edp 168-9; Mohan Bansal Ist/edp 327-30
N o w Friends, y o u actually d o n o t need any reference or e x p l a n t i o n t o answer this q u e s t i o n as it is o b v i o u s a n t i b i o t i c s d o n o t have
any role in t r e a t i n g allergy.
Rest all o p t i o n s - a v o i d i n g allergens, corticosteroids a n d surgery can be used as m a n a g e m e n t o p t i o n s for allergic rhinitis f o r m o r e
details see t h e p r e c e d i n g t e x t .
-

CHAPTER

Epistaxis

Retrocolumeilar Vein

Epistaxis is bleeding from inside the nose. L o c a t i o n : Just b e h i n d t h e columella at t h e anterior e d g e o f t h e


little's area.
• The retrocolumeilar vein o f this area t h e n runs a l o n g t h e f l o o r
AREAS OF NASAL BLEED
o f t h e nose t o anastomise w i t h t h e various plexus o f t h e lateral
wall o f t h e nose.
Little's A r e a
• C o m m o n site of v e n o u s bleeding in y o u n g p e o p l e (<35 yrs).
• M o s t c o m m o n site f o r epistaxis in c h i l d r e n a n d y o u n g adults.
• L o c a t i o n : A n t e r o i n f e r i o r p a r t o f t h e nasal s e p t u m 0
Woodruffs Plexus
Arteries contributing: S p h e n o p a l a t i n e a r t e r y (also
0

(Fig. 5.1) called as artery o f epistaxis) • L o c a t i o n : Found in t h e lateral nasal wall inferior t o t h e posterior
- Anterior ethmoidal e n d o f inferior t u r b i n a t e .
- Septal branch of greater palatine 0

• C o n t r i b u t i n g v e s s e l s : Anastomosis b e t w e e n s p h e n o p a l a t i n e
artery
artery a n d posterior p h a r y n g e a l artery.
- Septal branch o f superior l a b i a l 0

artery • Browne's a r e a : Located at t h e p a r t e n d o f nasal s e p t u m .


These arteries f o r m t h e Kiesselbach's plexus. 0 • Features:
It is a venous plexus
C o m m o n cause o f posterior epistaxis.
This area is called as little's area as it was identified by James
Little in 1879. It is also called as locus valsalvae and is the
confluence o f internal and external carotid artery.This vascular CLASSIFICATION OF EPISTAXIS
area is the most c o m m o n site of nose bleed in children and
young adults. It gets dried due t o the effect of inspiratory Classification I
current and easily traumatised due t o frequent picking
A c c o r d i n g to Scott Brown 7th/ed Vol2p 1600
(fingering) o f nose.

Anterior epistaxis: B l e e d i n g f r o m a source a n t e r i o r t o t h e


plane o f t h e p i r i f o r m a p e r t u r e . This includes b l e e d i n g f r o m t h e
anterior s e p t u m a n d rare bleeds f r o m t h e vestibular skin a n d
mucocutaneous junction.
Anterior ethmoidal Posterior
ethmoidal Posterior epistaxis: Bleeding f r o m a vessel situated posterior t o
artery and vein
artery and vein t h e p i r i f o r m aperture.This allows f u r t h e r subdivision i n t o lateral
Kiesselbach's wall, septal a n d nasal f l o o r b l e e d i n g .
(little area) Sphenopalatine
artery and vein
NOTE
Septal branch
For undergraduate students nobody can challenge above definition
of superior labial Greater palatine
artery and vein but in case a short note is asked an anterior and posterior epistaxis then
artery and vein
the followingTable 5.1, given on next page o f t h e guide from Dhingra
Fig. 5.1: Blood s u p p l y o f nasal s e p t u m should also be reproduced.
CHAPTERS Epistaxis

Table 5.1: Types of epistaxis and their features

Anterior Epistaxis Posterior Epistaxis

Blood flows out from the front of nose Blood flows back into the throat

Incidence More c o m m o n Less common

Site Mostly from Little's area or anterior part of lateral wall Mostly from posterosuperior part of nasal cavity; often difficult to
localise the bleeding point

Age Mostly occurs in children or young adults After 40 years of age'

Cause Mostly trauma Spontaneous; often due to hypertension or arteriosclerosis

Bleeding Usually m i l d , can be easily c o n t r o l l e d by local Bleeding is severe, requires hospitalization; postnasal packing
pressure or anterior pack often required

C l a s s i f i c a t i o n II ADULT RECURRENT EPISTAXIS

Epistaxis can also b e classified as:


W h e n recurrent bleeds occur in adults, secondary epistaxis is m o s t
• C h i l d h o o d e p i s t a x i s : i.e. if it occurs in age < 16 yrs likely therefore t h e causes listed b e l o w are t h e same for Recurrent/
• A d u l t e p i s t a x i s : i.e. if it occurs in age > 16 yrs s e c o n d a r y Epistaxis. Except f o r NSAIDs/aspirin use w h i c h can
cause recurrent epistaxis
C l a s s i f i c a t i o n III • C o a g u l o p a t h y secondary t o liver disease, k i d n e y disease, l e u -
Primary kemia or myelosuppression
• Trauma
B e t w e e n 7 0 % a n d 8 0 % o f all cases o f epistaxis are i d i o p a t h i c , • Post s u r g e r y : As after inferior t u r b i n e c t o m y , iatrogenic d a m -
s p o n t a n e o u s b l e e d s w i t h o u t any p r o v e n p r e c i p i t a n t or causal age t o a n t e r i o r e t h m o i d a l a r t e r y d u r i n g e n d o s c o p i c sinus
factor. This is k/a p r i m a r y epistaxis. surgery or d a m a g e t o internal c a r o t i d artery d u r i n g posterior
e t h m o i d or s p h e n o i d sinus surgery.
Secondary • Patients o n w a r f a r i n
Those cases w h e r e t h e cause o f epistaxis is d e f i n e d like t r a u m a , • Hereditary h e m o r r h a g i c telangiectasia
• Tumors-Juvenile nasopharyngeal angiofibroma hemangio-
surgery or a n t i c o a g u l a n t overdose.
pericytoma.
• M/C c a u s e of epitaxis in a d u l t s : H y p e r t e n s i o n .
| E P I T A X I S IN C H I L D R E N

Scott Brown 7th/ed Vol 1 p1064 MANAGEMENT OF NOSE BLEED


• Epistaxis is c o m m o n and usually i n n o c u o u s event in c h i l d h o o d
First-aid M e t h o d s
• It is rare in c h i l d r e n < 2 years
• Peak prevalence is in 3-8 y e a r s of age. • Nasal p i n c h i n g
• There is a seasonal variation w i t h a h i g h e r prevalence in t h e • A p p l y i n g ice cold w a t e r t o head or face or give ice packs t o
w i n t e r m o n t h s , d u e t o greater f r e q u e n c y o f u p p e r respiratory d o r s u m o f nose.
tract infections or t o t h e d r y i n g effect o f inspired air o f m o d e r n • Trotter's m e t h o d : Old fashioned m e t h o d o f c o n t r o l l i n g e p i -
central h e a t i n g systems. staxis. Make t h e p a t i e n t sit u p w i t h a cork b e t w e e n his t e e t h
• M/C site of o r i g i n of b l e e d — A n t e r i o r p a r t o f nasal s e p t u m a n d a l l o w h i m t o bleed till he becomes hypotensive.
(because this p a r t o f nasal mucosa is t h i n a n d is exposed t o
d r y air currents). T r e a t m e n t in H o s p i t a l
• M/C site of bleeding-Little's area
Sedation
• M/C c a u s e of E p i s t a x i s - l d i o p a t h i c
• 2 n d M/C c a u s e : Digital trauma/Nose p r i c k i n g in little's area • Pethidine is given t o allay t h e fear a n d anxiety o f t h e p a t i e n t .
w h i c h is d u e t o crusting w h i c h occurs because o f URTI.
Anterior Nasal Packing
C a u s e of R e c u r r e n t Epistaxis in C h i l d r e n • If b l e e d i n g continues, nose s h o u l d be packed w i t h a r i b b o n

Allergic rhinitis g a u z e s o a k e d in n e o s p o r i n a n t i s e p t i c c r e a m f o r 2 4 t o 4 8
hours. Merocel packs can be used as an alternative t o r i b b o n
Retained nasal f o r e i g n b o d y
gauze p a c k i n g ( a l t h o u g h costly b u t gives less d i s c o m f o r t t o
Use o f nasal sprays as intranasal steroid sprays
the patient).
H e m o r r h a g i c disease as in - ITPP, v o n w i l l e b r a n d disease
Vascular a b n o r m a l i t i e s - A/V m a l f o r m a t i o n s , h e m a n g i o m a Posterior Nasal Packing
A n g i o f i b r o m a (Suspected in adolescent boys) • If b l e e d i n g does n o t s t o p by anterior nasal packing, it indicates
Nasal parasitosis/Nasal mycosis posterior b l e e d i n g , a n d postnasal p a c k i n g s h o u l d be d o n e .
1 SECTION I Nose and Paranasal Sinuses

Posterior nasal packing can cause cardiovascular complications Anterior a n d posterior ethmoidal arteries are ligated
like p u l m o n a r y h y p e r t e n s i o n a n d c o r p u l m o n a l e since i t leads b e t w e e n inner canthus o f eye a n d m i d l i n e o f nose. Internal
t o sleep apnea. m a x i l l a r y artery is ligated by Caldwell-Luc a p p r o a c h t h r o u g h
its posterior wall in p t e r y g o p a l a t i n e fossa.
| V E S S E L L I G A T I O N IN U N C O N T R O L L A B L E B L E E D S

• External carotid artery ligations: Operation o f choice in Hereditary hemorrhagic telangiectasia or Esler-Weber Rendu
Elderly a n d d e b i l i t a t e d patients in a n t e r i o r epistaxis. disease:
Indication: bleeding f r o m t h e external carotid artery Hereditary hemorrhagic telangiectasia area inolves the anterior
system w h e n all conservative m e t h o d s have failed part of nasal septum and causes recurrent episodes of profuse
bleeding. It is managed by KTP or Nd Yag Laser or by septoder-
Site f o r l i g a t i o n : a b o v e t h e o r i g i n o f s u p e r i o r t h y r o i d
moplasty
artery.
• M a x i l l a r y a r t e r y ligation: Performed in t h e p t e r y g o p a l a t i n e
fossa. It is p e r f o r m e d in posterior bleeds.
• Ligation m e t h o d o f choice is Endoscopic sphenopalatine artery Hierarchy of arteries used for ligation in uncontrollable epistaxis:
l i g a t i o n (ESPAL). It is d o n e after e x p o s i n g t h e s p h e n o p a l a t i n e » Sphenopalatine artery (ESPAL)
f o r a m e n b y p u t t i n g an incision in t h e m i d d l e t u r b i n a t e a n d > Internal maxillary artery
l i g a t i n g t h e s p h e n o p a l a t i n e artery. • External carotid artery
• Anterior/posterior ethmoidal artery

• •


CHAPTER 5 Epistaxis

QUESTIONS

C o m m o n site of b l e e d i n g : [PGI 08] b. Internal maxillary artery ligation


a. Woodruff's plexus b. Brown area c. Anterior and posterior nasal pack
c. Little's area d. Vestibular area d. Anterior nasal pack
Woodruff's p l e x u s is s e e n at: [AP95;TN99;AP03] 14. S o u r c e of e p i s t a x i s a f t e r l i g a t i o n of e x t e r n a l c a r o t i d
a. Anteroinferior part o f superior turbinate a r t e r y is: [AIIMS 93]
b. Middle turbinate a. Maxillary artery b.. Greater palatine artery
c. Posterior part o f inferior turbinate c. Superior labial artery d. Ethmoidal artery
d. Anterior part o f inferior turbinate 1 5 . If p o s t e r i o r epistaxis c a n n o t be c o n t r o l l e d , w h i c h a r t e r y
Little's a r e a is s i t u a t e d in n a s a l c a v i t y in: is l i g a t e d : [Kolkata 00]
a. Anteroinferior b. Anterosuperior a. Posterior ethmoidal arteryb. Maxillary artery
c. Posteroinfesion d. Posterosuperior c. Sphenopalatine artery d. External carotid artery
M a i n v a s c u l a r s u p p l y of little's a r e a is all except: 16. In c a s e of u n c o n t r o l l e d e p i s t a x i s , l i g a t i o n of i n t e r n a l
a. Septal branch o f superior labial artery m a x i l l a r y a r t e r y is to b e d o n e in t h e : [Kolkata 01 ]
b. Nasal branch o f sphenopalatine artery a. Maxillary antrum b. Pterygopalatine fossa
c. Anterior ethmoidal artery c. A t t h e n e c k d. Medial wall of orbit
d. Palatal branch of sphenoplatine 1 7 . T r e a t m e n t of c h o i c e in r e c u r r e n t e p i s t a x i s in a p a t i e n t
W h i c h a r t e r y d o e s not c o n t r i b u t e to little's a r e a : with hereditary hemotelangiectasis: [Kolkota05]
[PGI 98] [FMGE 2013] a. Anterior ethmoidal artery ligation
a. Anterior ethmoidal artery b. Septal branch o f facial artery b. Septal dermatoplasty
c. Sphenopalatine artery d. Posterior ethmoidal artery c. External carotid artery ligation
M o s t c o m m o n c a u s e for n o s e b l e e d i n g is: [AIIMS 95] d. Internal carotid artery ligation
a. Trauma t o Little's area 18. C a u s e s of epistaxis are all except: [NEET Pattern]
b. AV aneurysm a. Allergic rhinitis b. Foreign b o d y
c. Posterosuperior part o f nasal septum c. Tumor d. Hypertension
d. Hiatus semilunaris 1 9 . M o s t c o m m o n site of n o s e b l e e d in c h i l d : [NEETPattern]
M/C c a u s e of epistaxis in 3 y e a r s o l d c h i l d : [PGI 98] a. W o o d r u f f area b. Brown area
a. Nasal polyp b. Foreign b o d y c. Little's area d. None
c. Upper respiratory catarrh d. Atrophic rhinitis 20. Posterior epistaxis is c o m m o n l y s e e n in: [NEETPattern]
In a 5-year-old child, m o s t c o m m o n c a u s e of unilateral a. Children w i t h ethmoidal polyps
epistaxis is: [PGI 97] b. Foreign bodies o f t h e nose
a. Foreign b o d y b. Polyp c. Hypertension
c. Atrophic rhinitis d. Maggot's d. Nose picking
R e c u r r e n t e p i s t a x i s in a 15-year-old f e m a l e t h e m o s t 21. Kiesselbach's p l e x u s is s i t u a t e d o n t h e : [DNB 2005, 11]
c o m m o n c a u s e is: [JIPMER 90] a. Medial wall of the middle ear
a. Juvenile nasopharyngeal fibroma b. Lateral wall of the nasopharynx
b. Rhinosporiodiosis c. Medial wall o f t h e nasal cavity
c. Foregin b o d y d. Laryngeal aspect o f epiglottis
d. Hematopoietic disorder 22. Posterior epistaxis o c c u r s f r o m : [Kerala 2010]
D i a g n o s i s in a 10-year-old b o y w i t h r e c u r r e n t expistaxis a. Woodruffs plexus b. Kiesselbach's plexus
10.
a n d a u n i l a t e r a l n a s a l m a s s is: [SGPGI05] c. Atherosclerosis d. Littles area
a. Antrochoanal polyp b. Hemangioma 2 3 . A child w i t h unilateral n a s a l obstructin a l o n g w i t h a m a s s
c. Angiofibroma d. Rhinolith in c h e e k a n d p r o f u s e a n d r e c u r r e n t e p i s t a x i s :
Epistaxis in e l d e r l y p e r s o n is c o m m o n in: [Al 04] [FMGE2013]
11.
a. Foreign b o d y b. Allergic rhinitis Glomus t u m o r
c. Hypertension d. Nasopharyngeal carcinoma Antrochoanal polyp
12. S y s t e m i c c a u s e s of epistaxis a r e all except: [UP 02] Juvenile nasal angiofibroma
a. Hypertension b. Anticoagulant treatment Rhinolith
c. Hereditary telangiectasia d. Hemophilia 2 4 . w h i c h is k n o w n as a r t e r y of epistaxis
13. A 7 0 y e a r s a g e d p a t i e n t w i t h epistaxis, p a t i e n t is h y p e r - a. Anterior ethmoidal A
t e n s i v e w i t h BP = 2 0 0 / 1 0 0 m m Hg. O n e x a m i n a t i o n no b. Sphenopalatine A
a c t i v e b l e e d i n g n o t e d , n e x t s t e p of m a n a g e m e n t is: c. Greater palatine A
a. Observation d. Septal branch of superior labial A
42 I SECTION I Nose and Paranasal Sinuses

EXPLANATIONS AND REFERENCES

1. A n s . is a , b a n d c i.e. Woodruff's p l e x u s , Brown's a r e a ; a n d Little's a r e a


2. A n s . is c i.e. Posterior p a r t of inferior t u r b i n a t e Ref. Mohan Bansal p 297
3. A n s . is a i.e. A n t e r o r i n f e r i o r
4. A n s . is d i.e. Palatal b r a n c h of s p h e n o p a l a t i n e a r t e r y
5. A n s . is d i.e. Posterior e t h m o i d a l a r t e r y
Ref. Dhingra 5th/edpp 190,191,6th/edp 176; Scott Brown 7th/ed Vol 2 p 1597; Mohan Bansal p 293

C o m m o n Sites of Bleeding

Located Formed by Characteristic

L i t t l e ' s area (M/C site o f A n t e r o i n f e r i o r part o f nasal • Anterior ethmoidal artery M/C site of bleeding
Epistaxis) septum
Septal branch of superior labial
Artery

Septal branch of sphenopalatine


artery

Greater palatine artery

Woodruff's area Under the posterior e n d of Sphenopalatine artery • It is a venous plexus


inferior turbinate
Posterior pharyngeal artery • Common cause of posterior
epistaxis

Retrocolumeilar vein Behind t h e c o l u m e l l a at t h e • M/C site of venous bleeding in


anterior edge of little's area children

Brown's area Posterior part of septum Posterior part of septum • Site for hypertensive posterior
epistaxis

6. A n s . is a i.e T r a u m a t o t h e little's a r e a Ref. Dhingra 5th/edp 190,6th/edp 176; Mohan Bansal p 293
• Little area (also called as Kiesselbach's plexus) is a h i g h l y vascular area in t h e anteroinferior p a r t o f nasal s e p t u m j u s t a b o v e t h e
vestibule
• It is t h e m o s t c o m m o n site f o r nasal b l e e d i n g as this area is exposed t o t h e d r y i n g effect o f inspiratory c u r r e n t a n d t o f i n g e r nail
trauma.
7. A n s . is c i.e. U p p e r r e s p i r a t o r y c a t a r r h Ref. Scott Brown 7th/ed Vol 1 p 1064
• Friends -1 k n o w s o m e o f y o u m u s t b e t h i n k i n g f o r e i g n b o d y as t h e answer b u t it is n o t t h e m o s t c o m m o n cause.
• MIC cause of epistaxis in children is idiopathic.
2 n d M/C c a u s e of e p i s t a x i s in c h i l d r e n is
Infection/Trauma
I
D e v e l o p m e n t o f crusts
I
Nasal picking/Digital t r a u m a
I
Nasal b l e e d

Still if y o u have d o u b t read t h e f o l l o w i n g lines o f Scotts B r o w n :


"Epistaxis - Children are especially susceptible to nose bleeds due to extensive vascular supply to nasal mucosa and the frequency with
which they develop upper respiratory tract infections." —Scott Brown 7th/ed Vol 1 pi 063
"Epistaxis is more common in children with upper respiratory allergies." —Scott Brown 7th/ed Vol 1 pi 063
"There is a seasonal variation with a higher prevalence in the winter months perhaps due to the greater frequency of upper respiratory
tract infections." —Scott Brown 7th/ed Vol 1 pi 063
8. A n s . is a i.e. F o r e i g n b o d y Ref. Dhingra 4th/edp 153; 5th/ed p 176; 6th/ed 161; SKDe5th/ed p 245
M o s t c o m m o n cause o f unilateral epistaxis in c h i l d r e n is Foreign body.
CHAPTER 5 Epistaxis

I n case o f Foreig n Body o f Nose "The child presents with unilateral nasal discharge which is often foul smelling and occasionally
blood-stained." —Dhingra 5th/ed p 176,6th/ed p!61
9. A n s . is d i.e. H e m a t o p o i e t i c d i s o r d e r Ref. Read Below
As such this answer is n o t g i v e n a n y w h e r e b u t w e can c o m e t o t h e correct answer by exclusion
Option "a" is Juvenile nasopharyngeal f i b r o m a .
It is seen in adolescent males a n d is t h e r e f o r e t h e m o s t c o m m o n cause o f recurrent epistaxis in males a n d n o t in females.

—Dhingra 5th/ed p 261,6th/edp 346


Option " b " i s Rhinosporidiosis is a cause o f epistaxis b u t usually occurs in y o u n g males f r o m India. —Turner 10th/edp61
Option " c " i s Foreign b o d y w h i c h is a cause o f epistaxis in c h i l d r e n a n d is n o t c o m m o n l y seen in 15 years o f age.
So w e are left w i t h h e m a t o p o i e t i c disorder w h i c h can be seen in a 15 years o l d female. — D h i n g r a 5th/edp 1766th/edpi61
1 0 . A n s . is c i.e. A n g i o f i b r o m a Ref. Dhingra 4th/ed p 230,5th/ed p 261,6th/ed p 246
Recurrent epistaxis in a 10-year-old b o y w i t h unilateral n a s a l m a s s is d i a g n o s t i c of j u v e n i l e nasopharyngeal f i b r o m a .
For details, see chapter on Pharyngeal Tumor.
1 1 . A n s . is c i.e. H y p e r t e n s i o n Ref. Maqbool 11 th/ed p 180; Mohan Bansal p295
A c c o r d i n g t o Scott Brown 7th/ed Vol 2 p 1600 - M/C c a u s e of a d u l t epistaxis is idiopathic t h o u g h a n u m b e r o f factors increase
its chances like use o f NSAIDs a n d alcohol. It f u r t h e r says t h e r e is n o p r o v e n association b e t w e e n h y p e r t e n s i o n a n d a d u l t Epistaxis,
b u t still
"Elevated blood pressure is observed in almost all epistaxis admissions. This apparent hypertension in acute admissions may be a result
of anxiety associated with hospital admission and the invasive techniques used to control the bleeding." —Dhingra 6th/ed p 167
But still t h e answer t o this q u e s t i o n is h y p e r t e n s i o n by r u l i n g o u t o t h e r o p t i o n s :
• O p t i o n a - f o r e i g n b o d y - is a cause o f epistaxis in c h i l d r e n a n d n o t in elderly age g r o u p
• O p t i o n b - allergic rhinitis - does n o t lead t o epistaxis —Dhingra 5th/edp 181
• Nasopharyngeal carcinoma does cause epistaxis a n d is seen in elderly age g r o u p b u t is n o t t h e m o s t c o m m o n cause as in itself
n a s o p h a r y n g e a l carcinoma is n o t c o m m o n .
"Nasal tumors seldom present as epistaxis in isolation Juvenile nasopharyngeal angiofibroma and hemangiopericytoma are rare vascular
tumors which can present with severe or recurrent epitaxis in association with nasal obstruction." —Dhingra 5th/ed p 263
• Hence o u r answer by exclusion is h y p e r t e n s i o n .
• The answer is f u r t h e r s u p p o r t e d by M a q b o o l 11 th/ed p 180 w h i c h says:
"Hypertension is a very common disease and causes epistaxis frequently in elderly patients."
1 2 . A n s is d i.e. H e m o p h i l i a Ref. Scott Brown 7th/ed Vol2p 1605

Epistaxis in A d u l t

Primary Secondary

No cause is i d e n t i f i e d b u t may be d u e t o : Cause is i d e n t i f i e d a n d it is d u e t o :

• Use of NSAIDs • Coagulopathy secondary to liver disease/kidney disease/leukemia or myelosuppression


• Use of alcohol • Trauma
• Hypertension (role not proven) • Post surgery like inferior turbinectomy, Endoscopic sinus surgery
• Warfarin intake (anticoagulant treatment)
• Hereditary hemorrhagic telangiectasia

H e m o p h i l i a is a Secondary Cause o f Epistaxis in Children Ref. Scott Brown 7th/ed Vol 1 p 1065
Hence t h e answer is d i.e. h e m o p h i l i a w h i c h is n o t a cause o f secondary epistaxis b u t is i m p l i c a t e d in t h e e t i o l o g y o f p r i m a r y epistaxis
t h o u g h its role is d o u b t e d t h e r e also.
1 3 . A n s . is a i.e. O b s e r v a t i o n Ref. Scott Brown 7th/ed, Vol 1 pi 065
• We d o n o t need any reference t o answer this particular q u e s t i o n as t h e answer is h i d d e n in t h e q u e s t i o n only.
• The q u e s t i o n itself says t h a t n o active b l e e d i n g is s e e n — s o n o need t o d o a n y t h i n g j u s t observe t h e p a t i e n t a n d because his
B/P is 200/100 m m Hg w h i c h is q u i t e h i g h , give h i m a n t i h y p e r t e n s i v e drugs.
44 [_ SECTION I Nose and Paranasal Sinuses

ALSO KNOW
-
M a n a g e m e n t strategy f o r a d u l t p r i m a r y epistaxis

Patient presents with epistaxis

Resuscitation (Pinch the ala nasa) i

Initial examination

i
Vessel NOT located Vessel located
T
Endoscopy
I
Direct therapy


1
It identifies the point of bleeding in 8 0 % cases and also Direct therapy
enables targeted hemostasis of bleeding vessel using (In case of anterior
insulated hot wire cautery or bipolar electrodes epistaxis -> bipolar
\
diathermy electrocautery)

If vessel is NOT located on Endoscopy

Indirect therapy
In the form of


Nasal packing
• Hot water irrigation

" Continued bleeding


\
• Posterior packing
• Ligation of
- Sphenopalatine artery
- Internal maxillary artery
- External carotid artery
-Anterior/posterior ethmoidal artery
• Septal surgery if epistaxis is due to
: prominent septal deviation or
vomeropalatine spur
• Embolization of artery

1 4 . A n s . is d i.e. E t h m o i d a l a r t e r y Ref. Dhingra 5th/ed p 189,6th/edp 178; Mohan Bansal Ist/edp 35; Scott Brown 7th/edVol2 p 1599

N o s e is S u p p l i e d b y

Internal carotid artery External c a r o t i d a r t e r y

• Anterior ethmoidal artery Facial Artery


• Posterior ethmoidal artery • Superior labial artery
Maxillary a r t e r y
* • Greater palatine artery
• Branches of sphenopalatine artery (nasopalatine, post nasal septal branches and posterior lateral nasal
branches
• Anterior superior dental artery

In t h e Q u e s t i o n

• Greater palatine artery
• Superior labial artery
• Maxillary a r t e r y •
CHAPTER 5 Epistaxis J, 45
Are all branches o f external c a r o t i d artery.
If external c a r o t i d artery is l i g a t e d , t h e source o f epistaxis w i l l be e t h m o i d a l a r t e r y w h i c h is a b r a n c h o f I n t e r n a l c a r o t i d a r t e r y .
1 5 . A n s . is c i.e. S p h e n o p a l a t i n e a r t e r y Ref. Scott Brown 7th/ed Vol 2 pp 1603,1606
Ligation t e c h n i q u e is reserved for intractable b l e e d i n g w h e r e t h e source c a n n o t be located or c o n t r o l l e d by o t h e r t e c h n i q u e s .

The hierarchy o f arteries used f o r l i g a t i o n is:


• Sphenopalatine artery
• Internal maxillary a r t e r y
• External carotid A r t e r y
• Anterior/posterior e t h m o i d a l a r t e r y

Earlier the most common artery ligated was maxillary artery but now endonasal sphenopalatine artery ligation (ESPAL) is the ligation of choice

"ESPAL is the current ligation of choice controlling bleeding in over 90% of cases with a low complication rate."
Ref. Scotts Brown 7th/ed Vol2p 1606

Endonasal Sphenopalatine Ligation

• It is t h e m o s t p o p u l a r p r o c e d u r e f o r l i g a t i o n a n d has replaced internal maxillary artery l i g a t i o n .


Can be d o n e u n d e r LA/GA
„ . . . .• . . . . , .... f .. .
• Incision is g i v e n 8 m m anterior a n d u n d e r t h e p o s t e r i o r e n d o f m i d d l e t u r b i n a t e
• S p h e n o p a l a t i n e artery is ligated in t h e s p h e n o p a l a t i n e f o r a m e n 0

• Success r a t e - 1 0 0 %

• C o m p l i c a t i o n s very rare - r e b l e e d i n g , i n f e c t i o n a n d nasal adhesions

Internal Maxillary Artery Ligation


Earlier it was t h e ligation p r o c e d u r e o f choice f o r u n c o n t r o l l e d b l e e d i n g :
• Internal maxillary a r t e r y is ligated in t h e p t e r y g o p a l a t i n e fossa using a Caldwell-Luc a p p r o a c h (3rd p a r t o f t h e artery is l i g a t e d ) 0

• CSuccess
o m p l i c arate
t i o n-s - Sinusitis, d a m a g e t o i n f r a o r b i t a l n e r v e , o r o a n t r a l f i s t u l a , d e n t a l d a m a g e a n d a n e s t h e s i a , a n d rarely
o p h t h a l m o p l e g i a a n d blindness.
External c a r o t i d artery l i g a t i o n a n d a n t e r i o r a n d posterior e t h m o i d a l a r t e r y ligation is n o t c o m m o n l y d o n e .
1 6 . A n s . is b i.e. P t e r y g o p a l a t i n e f o s s a Ref. Scott Brown 7th/ed Vol 2 p 1603; Mohan Bansalp 296


Site
• Sphenopalatine artery Sphenopalatine foramen
• Internal maxillary artery Pterygopalatine fossa •

• External carotid artery Above the origin of superior thyroid artery


• Ethmoidal arteries Between inner canthus o f eye and midline o f nose

1 7 . A n s . is b i.e. S e p t:al
al dermoplasty
Ref. Dhingra 5th/ed p 193,6th/edp 180; Scott Brown 7th/ed Vol 2, p 1605; Mohan Bansal 1st/edp297
• Hereditary hemotelangiectasia (HHT) or Osler-Weber-Rendu disease is an autosomal d o m i n a n t c o n d i t i o n affecting b l o o d vessels
in t h e skin, m u c o u s m e m b r a n e s a n d viscera
• The g e n e t i c a b n o r m a l i t y is located t o c h r o m o s o m e 9 a n d 12
-'assical features:
Telangiectasia
- A/V m a l f o r m a t i o n s
- Aneurysms
- Recurrent epistaxis (seen in 9 3 % cases)
46 [_ SECTION I Nose and Paranasal Sinuses

Management
Recurrent epistaxis in HHT
I
1
No blood transfusion required Blood transfusions required
T
Mild Moderate
I
Severe

Laser photo coagulation

Nasal closure
Packing, cautery antifibrolytic
agents, systemic/topical X
estrogens Young's operation
T
Septal dermoplasty where
anterior part of septal mucosa is
excised and replaced by a split
skin graft.

1 8 . A n s . is a i.e. allergic rhinitis Ref. Dhingra 6th/edp 176, 167;Mohan Bansal Ist/edp 294
A m o n g s t t h e o p t i o n s g i v e n , f o r e i g n body, t u m o r , h y p e r t e n s i o n all can lead t o epistaxis.
R e m e m b e r : M a n y nasal p r o b l e m s can lead t o epistaxis viz nasal t r a u m a , viral rhinitis, chronic infections o f nose (which lead t o
crust f o r m a t i o n like a t r o p h i c rhinitis, rhinits sicca, TB o f nose), f o r e i g n bodies in nose ( m a g g o t s a n d n o n living), DNS, neoplasms
( h e m a n g i o m a , p a p i l l o m a , carcinoma or sarcoma).
-L , ... .. . . . . .
T w o nasal c o n d i t i o n s w h i c h d o n o t lead t o epistaxis:
• Nasal p o l y p s
. Allergic rhinitis
P h a r y n g e a l c o n d i t i o n s w h i c h lead t o epistaxis:
• Adenoiditis
• Juvenile a n g i o f i b r o m a
• Malignant tumors
1 9 . A n s . is c i.e. Little's a r e a Ref. Mohan Bansal Ist/ed p 294
"The m o s t c o m m o n site o f b l e e d i n g in c h i l d r e n a n d y o u n g p e o p l e is Little's area."
2 0 . A n s . is c i.e. h y p e r t e n s i o n Ref. Dhingra 6th/ed p 178 Table 33.1; Mohan Bansal Ist/ed p 294
A n t e r i o r epistaxis
In a n t e r i o r epistaxis, b l o o d f l o w s f r o m arterior nasal o p e n i n g
It is m o r e c o m m o n t h a n p o s t e r i o r nasal b l e e d i n g
The c o m m o n sites o f b l e e d i n g are Little's area a n d a n t e r i o r p a r t o f lateral nasal wall
It is usually m i l d a n d c o n t r o l l e d by local pressure or anterior p a c k i n g
It m o s t l y affects c h i l d r e n a n d y o u n g adults a n d t h e M/C cause is t r a u m a .
Posterior epistaxis
Posterior nasal b l e e d i n g w h i c h is less c o m m o n , b u t m o r e severe, occurs s p o n t a n e o u s l y
M o s t o f t h e patients are m o r e t h a n 4 0 years o f age
The b l e e d i n g site w h i c h is d i f f i c u l t t o localise is m o s t l y p o s t e r i o r superior p a r t o f nasal cavity
The M/C cause is h y p e r t e n s i o n a n d arteriosclerosis
Bleeding is so severe t h a t it requires hospitalisation a n d posterior nasal p a c k i n g
2 1 . A n s . is c i.e. m e d i a l w a l l of n a s a l c a v i t y Ref. Dhingra 6th/edp 176
Kiesselbach's plexus is situated in t h e anterior inferior p a r t o f nasal s e p t u m ( w h i c h forms t h e m e d r o l wall o f nose) j u s t a b o v e t h e
vestibule.
2 2 . A n s . is a i.e. Woodruffs p l e u x Ref. Dhingra 6th/ed p 450
E x p l a n a t i o n : Repeat
2 3 . A n s . is c i.e. J u v e n i l e n a s a l a n g i o f i b r o m a Ref. Dhingra 6th/ed p 246
A c h i l d p r e s e n t i n g w i t h unilateral nasal o b s t r u c t i o n a l o n g w i t h mass in cheek a n d profuse a n d recurrent epistaxis s h o u l d i m m e d i -
ately raise t h e suspicion f o r Juvenile a n g i o f i b r o m a , details o f w h i c h are d e a l t in chapter o n 'Tumors o f pharynx'.
2 4 . A n s . is b s p h e n o p a l a t i n e a r t e r y Ref. internet search
The s p h e n o p a l a t i n e a r t e r y (nasopalatine artery), a branch o f maxillary artery a n d is c o m m o n l y k n o w n as A r t e r y o f Epistaxis.
CHAPTER
Diseases of Paranasal
Sinus—Sinusitis

Development
SINUSITIS • Maxillaryand e t h m o i d sinusesarepresentat birth, w h i l e s p h e n o i d
sinus is r u d i m e n t a r y at b i r t h a n d frontal sinus is recognizable at
I ANATOMY AND PHYSIOLOGY O F PARANASAL SINUSES
6 years o f age and is f u l l y d e v e l o p e d b y p u b e r t y .
Paranasal sinuses are a g r o u p o f air c o n t a i n i n g spaces t h a t s u r r o u n d •

t h e nasal cavity.

M a x i l l a r y sinus Frontal s i n u s

• Well developed at birth (1 st to develop) • Develops 2 years after birth


• Most common site of bacterial sinusitis • Characteristic feature—Pott's puffy tumor
• Most common site of noninvasive fungal sinusitis • Mucocele
• On X-ray: visible at 4-5 months • Ivory osteoma
• Completely developed by 9 year of age (at the time of second • X-ray visible at 6 years of age
dentition) • Maximum size achieved by puberty
• Largest sinus in the body

Ethmoidal sinus Sphenoidal sinus

• Well developed at birth • Develops 5 years after birth


• Clinically ethmoid cells are divided by the basal lamina into anterior • Least common sinusitis
ethmoid group which opens into middle meatus and posterior • Major cause of cavernous sinus thrombophlebitis
ethmoid group which opens into superior meatus • X-ray: appears by 4 year of age.
- Ant group includes cells: (a) Ager nasic cells (b) Ethmoidal bulla • Bones of Bertin also called sphenoidal turbinates initially cover the
(c) Supraorbital cells (d) Fronto-ethmoid cells (e) Haller cells anterior wall of sinus, but after 10 years, fuse with it.
- The posterior group includes onodi cells
• Leads t o orbital cellulitis
• Adenocarcioma seen mostly in w o o d worker
• X-ray: visible at 1st year of age and complete by puberty
• Most common cause of acute sinusitis in children. 0

Functions of Paranasal Sinus Blood Supply and Nerve Supply of Paranasal Sinuses

Possible f u n c t i o n s o f paransal sinus (PNS) are:


Sinuses Arteries Nerves
Air c o n d i t i o n i n g , i.e. w a r m i n g
Reduction o f skull w e i g h t Frontal Supraorbital, Supraorbital,
Increase t h e o l f a c t o r y area (in animals) Maxillary supratrochlear supratrochlear
Anterior Maxillary (main) and facial Maxillary
Heat i n s u l a t i o n
ethmoidal Anterior ethmoidal Anterior ethmoidal
Vocal resonance
Posterior Posterior e t h m o i d and Posterior e t h m o i d a n d
Provide mechanical r i g i d i t y t o skull ethmoidal sphenopalatine sphenopalatine
Pressure d a m p e n i n g Sphenoidal Posterior e t h m o i d a n d Posterior e t h m o i d a n d
Secretion o f m u c u s t o keep nasal c h a m b e r s m o i s t . sphenopalatine sphenopalatine
SECTION I Nose and Paranasal Sinuses

Development and Growth of Paranasal Sinuses

Sinus At birth A d u l t size Growth Radiological appearance (Age)

Maxillary Present 15 years Biphasic growth: Birth—3 years, 7-12 year 4-5 months
Ethmoid Present 12 years Size increases up t o 12 years 1 year
Frontal Absent 13-18 years Invades frontal bone (2-4 yrs), size increases until teens 6 years
Sphenoid Absent 12-15 years Reaches sella turcica (7 yrs), dorsum sellae (late teens), basisphenoid 4 years
(adult)

Extra Edge M a x i l l a r y sinusitis


• E t h m o i d a l sinuses are w e l l d e v e l o p e d a t b i r t h , hence infants Pain site: u p p e r j a w w i t h radiation t o t h e g u m s a n d t e e t h .
a n d c h i l d r e n b e l o w 3 years o f age are m o r e likely t o have acute It is aggravated by c o u g h i n g a n d s t o o p i n g .
e t h m o i d i t i s ; b u t a b o v e t h i s age, m a x i l l a r y sinusitis is m o r e Headache in Frontal region.
c o m m o n l y seen. Tenderness: Over the cheeks.
• Periodictiy is a characteristic feature of frontal sinus
Postnasal drip.
infections in w h i c h t h e pain increases g r a d u a l l y o n w a k i n g
Frontal sinusitis
u p a n d b e c o m e s m a x i m u m b y m i d d a y , starts d i m i n i s h i n g b y
H e a d a c h e : Over t h e f r o n t a l sinus area in t h e f o r e h e a d .
e v e n i n g , h e n c e also called office headache.
• T r e p h i n a t i o n o f f r o n t a l s i n u s is d o n e i f p a i n a n d pyrexia Pain is t y p i c a l l y periodical in n a t u r e . "
persist d e s p i t e o f medical t r e a t m e n t f o r 48 hours. Often calledas Office Headache. as 0
m a x i m u m pain occurs
• A n t r a l l a v a g e in acute maxillary sinusitis is d o n e o n l y w h e n b y m i d d a y a n d decreases by e v e n i n g
m e d i c a l t r e a t m e n t has f a i l e d a n d t h e p a t i e n t has s t a r t e d • Tenderness: A l o n g t h e f r o n t a l sinus floor j u s t above t h e
s h o w i n g signs o f c o m p l i c a t i o n s . This is d o n e u n d e r cover o f medial canthus.
antibiotics, o t h e r w i s e osteomyelitis o f t h e maxilla m a y set i n .
• Edema o f upper eyelid.
• D e n t a l i n f e c t i o n s are i m p o r t a n t cause o f maxillary sinusitis
E t h m o i d sinusitis
because o f r e l a t i o n o f roots o f molars a n d premolars w i t h t h e
M o r e o f t e n i n v o l v e d in infants a n d y o u n g c h i l d r e n . 0

f l o o r o f maxillary sinus.
P a i n : Over t h e nasal b r i d g e a n d inner canthus o f eye a n d
• Foramina o f Breschet are v e n o u s d r a i n a g e channels located in
t h e posterior w a l l o f Frontal sinus. is referred t o parietal e m i n e n c e .
T e n d e r n e s s is a l o n g inner canthus.
Edema o f t h e u p p e r a n d l o w e r eyelids.
| ACUTE SINUSITIS
Sphenoiditis
• It is acute i n f l a m m a t i o n o f t h e paranasal sinuses o f > 7 days Rare e n t i t y o n its o w n
a n d less t h a n 4 weeks d u r a t i o n . Occurs s u b s e q u e n t l y t o ethmoiditis/pansinusitis
Severe occipital or vertical headache a n d is s o m e t h i m e s

• M/C Sinus involved in adults in order of frequency: referred t o m a s t o i d process. 0

Maxillary > Frontal > Ethmoid > Sphenoid P a i n m a y b e f e l t r e t r o o r b i t a l l y d u e t o close p r o x i m i t y

• M/C sinus i n v o l v e d in children = Ethmoidal sinus


J w i t h V t h nerve.
Postnasal d r i p seen o n posterior rhinoscopy.

Etiology
Noe: Vertical headache with postnatal discharge is suggestive of
Secondary bacterial i n f e c t i o n f o l l o w i n g viral rhinitis.
sphenoid sinusitis.

Causative organisms Examination


• M/C—Streptococcus pneumoniae O n A n t e r i o r R h i n o s c o p y : Red, s h i n y a n d s w o l l e n mucous
• 2nd M/C—H. inferenzae
m e m b r a n e is seen near t h e o s t i u m o f t h e sinus, a n d t r i c k l e o f pus
• Others—Moraxella
m a y also be seen.

Clinical Features Investigation

As per RhinosinusitisTask Force d e f i n i t i o n : • T r a n s i l l u m i n a t i o n test:


• Major s y m p t o m s of sinusitis i n c l u d e facial p a i n , pressure, In maxillary sinus—-absence o f infra-orbital crescent o f
c o n g e s t i o n , nasal o b s t r u c t i o n , nasal/postnasal discharge, l i g h t a n d p u p i l l a r y g l o w indicate sinusitis.
h y p o s m i a , a n d fever. In Frontal sinusites cits t r a n s i l l u m i n a t i o n is n o t very i n -
• M i n o r s y m p t o m s are headaches, halitosis, a n d d e n t a l p a i n . formative
• D i a g n o s i s requires two major criteria or one major and two X-ray PNS: To d e m o n s t r a t e f l u i d level, pus or opacity.
minor criteria.
CHAPTER 6A Diseases of Paranasal Sinus—Sinusitis J 49

• R e c e n t l y , e n d o s c o p i c s i n u s s u r g e r y is r e p l a c i n g r a d i c a l
o p e r a t i o n s o n t h e sinuses a n d provides g o o d d r a i n a g e a n d
R a d i o l o g i c a l V i e w s f o r Each Sinus
v e n t i l a t i o n . It also avoids external incisions.
Maxillary Frontal Ethmoids Sphenoid
fl F U N G A L S I N U S I T I S
Best-Water's view Caldwell's Caldwell's Lateral and
(also called as view view Basal view
• Fungal infection occurs mostly in traumaticcases w i t h c o m p o u n d
occipitomental (occipitofrontal (but best is
fractures, in u n c o n t r o l l e d diabetics, d e b i l i t a t e d patients, such as
or nose chin or nose lateral view)
carcinoma, and in patients o n immunosuppressants, antibiotics
position) and forehead view)
or steroids.
Basal view
• More c o m m o n fungal speciesare/4sperg;7/us(M/C),/Acf/nomyces,
Mucor, Rhizopus or Absidia species o f f u n g u s .
NOTE • May occur in n o n invasive or invasive f o r m .
• C o m m o n e s t o r g a n i s i m i n v o l v e d i n n o n i n v a s i v e f o r m is
• In acute sinusitis—diagnosis is mainly made on clinical ground and
Aspergillus fumigatus f o l l o w e d b y Dematiaceous species
there is little role for imaging. (Bipolaris, Curvularia, Alternaria).
• The first investigatioin usually done in past was plain X-ray but it is • N o n invasive f o r m m a y e i t h e r p e r s e n t as a f u n g a l ball o r
not done nowadays. The plain CT scan without contrast is the first a l l e r g i c f u n g a l r h i n o s i n u s i t i s (AFRS) a n d u s u a l l y a f f e c t
line of screening study o f t h e nose and paranasal sinuses these days i m m u n o c o m p e t e n t individuals.

Treatment C o m p l i c a t i o n s o f Paranasal Sinus I n f e c t i o n

Local Mucocele/Mucopyocele
• Medical:
Mucous retention cyst
A n t i b i o t i c s are given f o r m i n i m u m — 2 weeks (10-14 days) Osteomyelitis
A m o x i c i l l i n + clavulanic acid. - Frontal bone (more common)
Nasal d e c o n g e s t a n t s : They should n o t be given f o r - Maxilla
m o r e t h a n 2 w e e k s else p a t i e n t m a y d e v e l o p Rhinitis
Orbital Preseptal inflammatory edema of lids
medicamentosa.
Subperiosteal abscess
Analgesics
Orbital cellulitis
Steam i n h a l a t i o n
Orbital abscess
• S u r g e r y : It is n o t d o n e i n acute sinusitis e x c e p t in case o f
Cavernous sinus thrombosis
i m p e n d i n g c o m p l i c a t i o n s like o r b i t a l cellulitis. Superior orbital fissure syndrome

Intracranial Meningitis
| CHRONIC SINUSITIS
Extradural abscess
Subdural abscess
• W h e n s y m p t o m s o f sinusitis persist f o r m o r e t h a n 3 m o n t h s —
Brain abscess
Chronic state develops.
Descending Otitis media
• O r g a n i s m s : M i x e d aerobic a n d anaerobic.
infections Pharyngitis
Tonsillitis
N o t e : Maxillary sinus is most commonly involved in chronic Laryngitis
sinusitis.
—' ' ! ! !
| ORBITAL COMPLICATIONS
Diagnosis
• M o s t l y seen in children
D i a g n o s i s is d o n e b y nasal e n d o s c o p y a l o n g w i t h e n d o s c o p y
g u i d e d c u l t u r e f r o m m i d d l e meatus. This can b e s u p p l e m e n t e d
w i t h CT scan o f nose a n d PNS In children the orbital complication of sinusitis are due t o
ethmoiditis.
Treatment In adults, it is due to frontal sinusitis

Medical
• Patients c o m p l a i n o f h i g h fever, w i t h pain in eye o n t h e side-
• Antibiotics, Mucolytics, Nasal Irrigation, Cortcosteroidsto reduce o f lesion, chemosis, p r o p t o s i s a n d d i p l o p i a . Vision m a y b e
mucosal swelling associated w i t h t h e i n f l a m m a t o r y response. diminished.

Surgical
Superior Orbital Fissure S y n d r o m e
• I n d i c a t i o n : If m e d i c a l t r e a t m e n t g i v e n f o r a p e r i o d o f 3-4
weeks fail. • Occurs s u b s e q u e n t t o s p h e n o i d o i t i s .
SECTION I Nose and Paranasal Sinuses

Treatment
• Features • Broad s p e c t r u m a n t i b i o t i c s for 4-6 weeks.
- Deep orbital pain • Surgical drainage o f t h e sinus t h r o u g h f r o n t o n a s a l d u c t .
- Frontal headache
- Progressive paralysis of III, IV and VI nerve (first nerve t o get
Osteomyelitis ofthe Maxilla
s involved) cranial nerve.
M o r e o f t e n i n infants a n d c h i l d r e n because o f t h e presence o f
s p o n g y b o n e in t h e anterior wall o f t h e Maxilla.
Orbital Apex Syndrome

Superior o r b i t a l fissure s y n d r o m e w i t h i n v o l v e m e n t o f o p t i c nerve


| DENTAL COMPLICATIONS
a n d maxillary nerve.
• Second p r e m o l a r a n d t h e first m o l a r are directly in relation t o
Treatment t h e f l o o r o f t h e Maxillary sinus.

A n t i b i o t i c s , analgesics a n d nasal d e c o n g e s t a n t s . Therefore, acute sinusitis may produce dental pain.


Surgical d e c o m p r e s s i o n in case o f visual loss.
1 SYSTEMIC COMPLICATIONS
Cavernous Sinus Thrombosis
• Toxic s h o c k s y n d r o m e : Is rare, b u t p o t e n t i a l l y fatal.
Usually results f r o m i n f e c t i o n o f e t h m o i d a n d s p h e n o i d sinuses. O r g a n i s m : Staphylococcus aureus.
• Clinical f e a t u r e s : S y m p t o m s : Fever, h y p o t e n s i o n , rash w i t h d e s q u a m a t i o n
Onset is abrupt with fever chills and rigor a n d m u l t i s y s t e m failure.
Involvement oflilrd, Nth, Vth and Vlth cranial nerve
Chemosis of conjunctiva (1 st a n d 2 n d division)
1 CHRONIC COMPLICATIONS
Pupils are dilated a n d fixed (due t o i n v o l v e m e n t o f s y m -
p a t h e t i c plexus a r o u n d c a r o t i d artery). MucoCele/Pyoceles
Decreased vision (due t o o p t i c nerve d a m a g e ) .
Definition
Decreased sensation in distribution of Vth nerve (ophthalmic
division) a n d e n g o r g e m e n t o f retinal vessels. Epithelial-lined; m u c u s - c o n t a i n i n g sac c o m p l e t e l y filling t h e sinus

• Treatment: A n t i b i o t i c s in h i g h doses f o r 4-6 weeks a n d a n d capable o f e x p a n s i o n .

d r a i n a g e o f i n v o l v e d sinus.
Etiopathogenesis

NOTE
O b s t r u c t i o n a n d s u b s e q u e n t sinus i n f e c t i o n or i n f l a m m a t i o n
Cavernous sinus t h r o m b o s i s can be differentiated f r o m other
orbital complications as their is B/L involvement in cavernous sinus Features
thrombosis. • C o m m o n in p a t i e n t s : 4 0 - 7 0 years.
• Males > Females
| OSTEOMYELITIS

Osteomyelitis is i n f e c t i o n o f t h e b o n e m a r r o w . • Sinuses affected in order of frequency: Frontals > ethmoids >


sphenoids > maxillary
Organism Causing
Frontal Sinus Mucocele
• Staphylococcus
• Site: Superomedial q u a d r a n t o f t h e o r b i t .
• Streptococcus
• S y m p t o m s : Displacement o f t h e eyeball—Forward, d o w n w a r d
• Anaerobes
a n d laterally, diplopia, d e e p nasal or periorbital pain a n d f r o n t a l
headache.
Osteomyelitis o f t h e Frontal B o n e is Most C o m m o n as: • F e a t u r e s of t h e s w e l l i n g : cystic, non-tender, egg-shell
• It is a diploic b o n e a n d t h e lesion is essentially t h r o m b o p h l e b i t i s crackling m a y be seen.
of diploic bone. • X-ray: C l o u d i n g o f f r o n t a l sinus w i t h loss o f t h e s c a l l o p e d
margins (characteristic o f f r o n t a l sinus).
• It f o l l o w s i n f e c t i o n o f f r o n t a l sinus.
• It is c o m m o n i n adults since t h i s sinus is n o t d e v e l o p e d i n
Mucocele of Anterior Ethmoid
infants a n d c h i l d r e n
• Presents as a r e t e n t i o n cyst.
• Compress t h e nasolacrimal r e g i o n - Epiphora
Clinical Feature
• Causes a b u l g e in m i d d l e meatus o f nose.
• Fever, malaise, headache.
• Puffy s w e l l i n g u n d e r t h e p e r i o s t e u m o f f r o n t a l b o n e (Pott's Mucocele of Maxillary Sinus

puffy tumor). Asymptomatic.


CHAPTER 6A Diseases of Paranasal Sinus—Sinusitis

Mucocele of Sphenoid Sinus Indications of FESS


• Presents as superior o r b i t a l fissure s y n d r o m e or o r b i t a l apex
syndrome. FESS is indicated in rhinosinusitis, sinonasal polyposis, mucoceles,
• E x o p h t h a l m o s is always present. a n d allergic f u n g a l sinusitis.
• Pain is localized t o o r b i t or f o r e h e a d .
• T r e a t m e n t is Endoscopic Sinus Surgery (surgery o f choice) o r Contraindications
external e t h m o i d e c t o m y a n d s p h e n o i d e c t o m y .
Intracranial c o m p l i c a t i o n s f o l l o w i n g acute sinusitis, i n v o l v e m e n t
o f lateral wall a n d floor o f maxillary a n t r u m , p a t h o l o g y localized
Treatment
t o lateral recesses o f frontal sinus.
F r o n t o e t h m o i d a l M u c o c e l e s : Radical f r o n t o e t h m o i d e c t o m y
u s i n g a n e x t e r n a l m o d i f i e d Lynch-Howarth's incision w i t h free A .. .
d r a i n a g e o f f r o n t a l sinus i n t o t h e m i d d l e meatus. S o m e can b e Complications of FESS
r e m o v e d endoscopically. Major complications are orbital (Periorbital ecchymosis,
Emphysema, O p t i c nerve injury) a n d intracranial injury. (CSF leak,
M e n i n g i t i s , c a r o t i d artery injury).
Remember:
Other complications include major hemorrhage from
• Acute sinusitis Symptoms for < 4 weeks sphenoplatine and ethmoidal arteries, injury to nasolacrimal duct,
' Subacute sinusitis Symptoms for 4-12 weeks
rhinorrhea anosmia, and synechiae formation.
• Chronic sinusitis Symptoms for > 12 weeks
• Recurrent sinusitis 4 or more episodes of sinusites each
year, lastingformore than 7-10 days. J Note: Optic nerve injury occurs in posterior ethmoidal and
sphenoidal sinus surgeries, while carotid artery injury occurs in
SURGERIES FOR SINUSITIS ^ surgeries of the sphenoid sinus.
. . .

Indications of Nasal Endoscopic Surgery


CALDWELL-LUC'S SURGERY

nemonic
Maxillary antrum is entered through the sublabial route to clear the
Selected = Selected Tumor Resection disease inside. Antrum is connected to the nose through a nasoantral
Indians = Inflammation of sinus = Rhinosinusitis window made via the inferior meatus.
Prime = Polyps removal (Ethmoidal/Antrochoaral)
Minister = Mucocele of frontoethmoid/sphenoid
Indications
Don't Dacrocystorhinostomy
Speak = Septoplasty (Endoscopic) • Dental o r i g i n maxillary sinusitis.
Correct = Choanal atresia repair/CSF leak • Recurrent a n t r o c h o a n a l p o l y p in an a d u l t ( c o n t r a i n d i c a t e d i n
Fluent = removal of Foreign body children)
English = Epistaxis (Endoscopic cautery) • Foreign bodies in t h e a n t r u m
• Dental cyst
| FUNCTIONAL ENDOSCOPIC SINUS S U R G E R Y (FESS) • Oroantral fistula
• Fractures o f maxilla
It is the surgery of choice in most sinusitis. It uses nasal endoscopes of • As a n a p p r o a c h t o p t e r y g o p a l a t i n e fossa ( m a x i l l a r y a r t e r y
varying angulation (0°, 30°, 45°, 70°) to gain access to the outflow tracts ligation/Vidian neurectomy) and ethmoids (transantral
and ostia of sinuses, employing atraumatic surgical techniques with ethmoidectomy).
mucosal preservation to improve sinus ventilation and mucociliary
Can you Take Biopsy by this Approach in Maxillary Carcinoma?
employing atraumatic surgical techniques with mucosal preservation
! Note: No. Biopsy via Caldwell-Luc's is a contraindication in malignancy
to improve sinus ventilation and mucociliary clearance."
maxilla as it leads t o spread o f t h e neoplasm t o the cheek.

F E S S is b a s e d o n 3 p r i n c i p l e s
M/C C o m p l i c a t i o n i s -
• Site o f p a t h o g e n e s i s in sinusitis is*osteomeatal c o m p l e x .
• M u c o c i l i a r y clearance o f t h e sinuses is always d i r e c t e d t o w a r d Infra-orbital anesthesia/neuralgia d u e t o t r a c t i o n o n t h e nerve.
t h e natural o s t i u m .
• The mucosal p a t h o l o g y in sinuses reverts b a c k t o n o r m a l once Important Clinical Vignettes
t h e sinus v e n t i l a t i o n a n d m u c o c i l i a r y clearance is i m p r o v e d . • Lund-Mackay s t a g i n g is used i n r a d i o l o g i c a l assessment o f
NOTE chronic rhinosinusitis.The scoring is based o n CTscan f i n d i n g s
o f t h e sinuses (Maxillary, f r o n t a l , s p h e n o i d , arterior e t h m o i d
In FESS = Opening is made via middle meatus. a n d posterior e t h m o i d )
• Lund-Kennedy Endoscopic scores-
The Basic Steps of F E S S In this staging system endoscopic appearance o f nose is seen for:
U n c i n e c t o m y ( i n f u n d i b u l o t o m y ) , anterior e t h m o i d e c t o m y , m i d d l e 1. Presence o f p o l y p
meatal antrostomy, posterior ethmoidectomy, s p h e n o i d o t o m y 2 Presence o f discharge
f o l l o w e d by f r o n t a l recess clearance. 3. Presence o f edema, scarring or a d h e s i o n a n d c r u s t i n g .
SECTION I Nose and Paranasal Sinuses

•STIONS

1. Which sinus is NOT a part of paranasal sinus? [MP 09] 13. Ethmoidal sinusitis is more common with: [AIIMS 97]
a. Frontal b. Ethmoid a. Fireworkers b. Woodworkers
c. Sphenoid d. Pyriform c. Chimney smokers d. None
2. Sinus not present at birth is: [Maharashtra 02] 14. Sphenoid sinusitis pain is referred most commonly to:
a. Ethmoid b. Maxillary a. Occiput b. Vertex
c., Sphenoid d. None c. Frontal d. Temporal region
3. Maxillary sinus achieves maximum size at: [Manipal 06] 15. Best view for evaluating sphenoid sinus is: [PGI 98]
a. At birth b. At primary dentition a. Water's with open mouth b. Schuller's view
c. At secondary dentition d. At puberty c. Towne's view d. Lateral view
4. Which among the following sinuses is most commonly 16. Best view for frontal sinus: [AIIMS Nov 2010]
affected in a child: [PGI99] a. Caldwell b. Towne
a. Sphenoid b. Frontal c. Water's d. Lateral view
c. Ethmoid d. Maxillary 17. For veiwing superior orbital fissure-best view is:
5. In acute sinusitis, the sinus most often involved in [AIMS 97]
children is: [UPSC07] a. Plain AP view b. Caldwell view
a. Maxillary Sphenoid c. Towne view d. Basal view
c. Ethmoid Frontal 18. Complications of acute sinusitis: [PGI 03]
6. Sinusi least involved in: [UP 08] a. Orbital cellulitis b. Pott's puffy tumor
a. Maxillary Ethmoid c. Conjunctival chemosis d. Subdural abscess
c. Frontal Sphenoid e. Pyocele
7. Common organisms causing sinusitis: [AI01] 19. Complication of sinus disease include: [AIIMS 93]
a. Pseudomonas a. Retrobulbar neuritis
b. Moraxella catarrhalis b. Orbital cellulitis
c. Streptococcus pneumoniae c. Cavernous sinus thrombosis
d. Staphylococcus epidermidis d. Superior orbital fissure syndrome
e. H. influenzae e. All of the above
8. Common organisms causing sinustitis: [PGI01] 20. Orbital cellulites is a complication of: [MP 09]
a. Pseudomonas a. Parasinusitis b. Faciomaxillary trauma
-?i3jtib.
JIM) M S T J I t U . . / JtV J o H o 111
1
c. Endoscopic sinus surgery d. All of these
c. Moraxella catarrhalis 21. Angular vein infection commonly causes thrombosis
d. Streptococcus pnenumoniae •
of: [TN]
a. Cavernous sinus b. Sphenoidal sinus
e. Staphylococcus epidermidis^
H. Influenzae Jt c. Petrosal sinus d. Sigmoid sinus
22. A patient with sinus infection develops chemosis, B/L
9. Which of the following is the most common etiological
proptosis and fever, the diagnosis goes in favor of:
agent in paranasal sinus mycoses? [AIIMSMay06]
[PGI 99]
a. Aspergillus sp b. Histoplasma
a. Lateral sinus thrombosis
c. Conidiobolus coronatus d. Candida albicans
b. Frontal lobe abscess
10. Which among the following is true regarding fungal
c. Cavernous sinus thrombosis
sinusitis: [PGI 01]
d. Meningtitis
a. Surgery is required for treatment
23. Most definitive diagnosis of sinusitis is: [AIIMS 92]
b. Most common organism is Aspergillus niger
a. X-ray PNS b. Proof puncture
c. Amphoterecin B IV is used for invasive fungal sinusitis c. Sinoscopy d. Transillumination test
d. Hazy appearance on X-ray with radiopaque density 24. Pathognomic feature of Maxillary sinusitis is: [UP 07]
e. Seen only in immunodeficient conditions a. Mucopus in the middle meatus
11. All of the following are diagnostic criteria of allergic b. Inferior turbinate hypertrophy
Fungal sinusitis (AFS) except: [Al 08] c. Purulent nasal discharge
a. Areas of High attuenuation on CT scan d. Atrophic sinusitis
b. Orbital invasion 25. The best surgical treatment for chronic maxillary sinusitis
c. Allergic eosinophilic mucin is: [MP 02]
d. Type 1 Hypersitivity a. Repeated antral washout
12. Periodicity is a characteristic feature in which sinus b. Fiberoptic endoscopic sinus surgery
infection: [COMED06] c. Caldwell-Luc s operation
a. Maxillary sinus infection b. Frontal sinus infection d. Horgans operation
c. Sphenoid sinus infection d. Ethmoid sinus infection
CHAPTER 6A Diseases of Paranasal Sinus—Sinusitis 53

26. Frontal mucocele presents as: [PGI 96] 33. Multiple R e s p o n s e Q u e s t i o n :


a. Swelling above medial canthus, b e l o w t h e floor o f frontal All a r e t r u e a b o u t m u c o r m y c o s i s , e x c e p t :
sinus a. Lymph invasion
b. Swelling above eyebrow lateral t o grabella b. Angio invasion
c. External proptosis c. Long t e r m deferoxanine therepy
d. Intianasal swelling d. Septate hyphae
2 7 . M u c o c e l e is c o m m o n l y s e e n in s i n u s : [DNB 07] e. May lead t o blindness
a. Frontal b. Maxillary 3 4 . C a v e r n o u s sinus t h r o m b o s i s f o l l o w i n g sinusitis results
c. Ethmoid d. Sphenoid in all of t h e f o l l o w i n g s i g n s e x c e p t :
28. M o s t c o m m o n site for o s t e o m a is: [MP 08] a. Constricted pupil in response t o light
a. Maxillary sinus b. Ethmoid sinus b. Engorgement o f retinal veins u p o n o p h t h a l m o s c o p i c ex-
c. Frontal sinus d. Sphenoid sinus amination
2 9 . A 2-year-old child w i t h p u r u l e n t n a s a l d i s c h a r g e , f e v e r c. Ptosis o f eyelid
a n d p a i n s i n c e 2 m o n t h s . His f e v e r is 102-103°C, a n d
d. Ophthalmoplegia.
l e u c o c y t e c o u n t is 1 2 0 0 0 c u / m m . X-ray P N S s h o w e d
3 5 . A 24-year-old f e m a l e w i t h l o n g s t a n d i n g h i s t o r y o f
o p a c i f i c a t i o n of left e t h m o i d a l a i r cells. T h e c u l t u r e o f
sinusitis p r e s e n t w i t h fevers, h e a d a c h e (recent origin)
t h e e y e d i s c h a r g e w a s n e g a t i v e . W h i c h of t h e f o l l o w i n g
and personality changes; Fundus examination revealed
w o u l d b e m o s t useful f u r t h e r step in e v a l u a t i o n of t h i s
p a p i l l e d e m a . M o s t likely d i a g n o s i s is:
patient? [Al 10]
-
H a. Frontal lobe abscess b. Meningitis
a. CTscan
I' c. Encephalitis d. Frontal b o n e osteomyelitis
b. Urine culture
36. First p a r a n a s a l s i n u s to d e v e l o p at b i r t h is:
c. Blood culture
[NEET Pattern]
d. Repeat culture o f t h e eye discharge
a. Maxillary b. Ethoidal
30. FESS means: [Mahara 02]
c. Frontal d. Sphenidal
a. Factual endoscopic sinus surgey
3 7 . A n t r u m o f H i g h m o r e is a n o t h e r n a m e for: [NEETPattern]
b. Functionl endonasal sinus surgery
c. Factual endonasal sinus surgery a. Maxillary b. Ethmoid

d. Functionl endoscopic sinus surgery c. Sphenoid d. Frontal


31. E n d o s c o p i c n a s a l s u r g e r y is i n d i c a t e d i n : [Manipal 04] 38. Bilateral proptosis a n d bilateral 6 t h n e r v e p a l s y in s e e n
a. Chronic sinusitis b. Epistaxis [NEET Pattern]
c. Both d. None Cavernous sinus thrombosis
32. I n d i c a t i o n s of F E S S : [NEET Pattern] Meaningitis

a. Inverted papilloma b. Nasal allergic polyposis Hydrocephalus •

c. Mucocele d. Ca maxilla orbital Cellulitis


EXPLANATIONS AND REFERENCES

1. A n s . is d i.e. Pyriform Ref. Dhingra 5th/ed p 201, 6th/ed p 187; Mohan Bansalp 37
Paranasal sinuses are air c o n t a i n i n g cavities in certain bones o f skull. They are f o u r o n each side. Clinically, paranasal sinuses have
been divided into t w o groups.

1 Anterior group
. Posterior g r o u p 1
It includes: It includes:

- Maxillary sinus - Posterior ethmoidal sinus (opens in superior meatus)

- Frontal sinus

- Anterior ethmoidal sinus - Sphenoid sinus (opens in sphenoethmoidal recess)

^ ^ T E j [

All of them open in the middle meatus 0

2. A n s . is c i.e. S p h e n o i d s i n u s Ref. Scott Brown 7th/ed Vol 2, p 1320; Mohan Bansal Ist/ed p 39

1
SECTION I Nose and Paranasal Sinuses

D e v e l o p m e n t and g r w o t h o f paransal sinuses

Sinus At birth Adult size Growth Radiological appearance (age)

Maxillary Present 15 years Biphasic growth: Birth—3 years, 7-12 year 4-5 months
Ethmoid Present 12 years Size increases up to 12 years 1 year
Frontal Absent 13-18 years Invades frontal bone (2-4 yrs), size increases until teens • 6 years
Sphenoid Absent 12-15 years Reaches sella turcica (7 yrs), dorsum sellae (late teens), basisphenoid (adult) 4 years

A n s . is c i.e At s e c o n d a r y d e n t i t i o n Ref. Maqbool 11th/edp 148; Turners 10th/edp9


• Maxillary sinus is t h e first sinus t o d e v e l o p at b i r t h .
• It is c o m p l e t e l y d e v e l o p e d b y 9 years o f age, i.e. a p p r o x i m a t e l y at t h e t i m e o f secondary d e n t i t i o n .
A n s . is c i.e E t h m o i d Ref. Tuli Ist/edp 190; Dhingra Sth/edp207,6th/edp 193
A n s . is c i.e. E t h m o i d

M o s t c o m m o n sinusitis in c h i l d r e n is E t h m o i d .
M o s t c o m m o n sinusitis in adults is Maxillary.

"Ethmoidal sinuses are well developed at birth, hence infants and children below 3 years of age are more likely to have acute
ethmoiditis; but after this age, maxillary antral infections are more commonly seen." — T u l i Ist/ed p 190
"Ethmoid sinuses are more often involved in infants and young children." — D h i n g r a 5th/ed p 207,6th/ed p 193
A n s is d i.e. S p h e n o i d Ref. Dhingra 5th/ed p 207,6th/ed p 193; Turner 1 Oth/ed p 48
"Isolated involvement of sphenoid sinus is rare. It is often a part of pansinusitis oris associated with infection of posterior ethmoidal sinus."
...Dhingra 6th/edp 193
"The sphenoid sinus is rarely affected on its own" —Turner 10th/ed p 48
In Nutshell r e m e m b e r :

M/c sinus affected in a d u l t s — m a x i l l a r y


M/c sinus affected in c h i l d r e n — E t h m o i d
Sinus w h i c h is least a f f e c t e d — S p h e n o i d

7. A n s . is c a n d e i.e. Streptococcus pneumoniae; a n d H. influenzae


8. A n s is b, c a n d e i.e. M o r a x e l l a , S t r e p t o c o c c i a n d H. influenzae
Ref. Harrison 17th/ed p 205; Scott Brown 7th/ed Vol 2p1441; Mohan Bansal p 299
According to Harrison 17th/edp205
" A m o n g c o m m u n i t y - a c q u i r e d cases, S. pneumoniae a n d n o n t y p a b l e Haemophilus influenzae are t h e m o s t c o m m o n p a t h o g e n s ,
a c c o u n t i n g for 5 0 - 6 0 % o f cases. Moraxella catarrhalis causes disease in a signigicant percentage ( 2 0 % ) o f c h i l d r e n b u t less o f t e n in
adults. O t h e r streptococcal species a n d Staphylococcus aureus cause o n l y a small percentage o f cases, a l t h o u g h t h e r e is increasing
c o n c e r n a b o u t c o m m u n i t y strains o f m e t h i c i l l i n - resistant S. aureus (MRSA) as an e m e r g i n g cause."
According to Nelson 18th/ed,pp 1749,1750
"The bacterial p a t h o g e n s causing acute bacterial sinusitis in c h i l d r e n a n d adolescents i n c l u d e Streptococcus pneumoniae (= 3 0 % ) ,
n o n t y p a b l e Haemophilus influenzae ( = 2 0 % ) . "
According to scotts Brown 7th/ed, p 1441
• M/C O r g a n i s m causing sinusitis in adults is also Streptococcus pneumoniae f o l l o w e d by H. influenza.
In c h i l d r e n :
• M/C is Streptococcus pneumoniae ( 3 0 - 4 3 % ) f o l l o w e d by b o t h H. influenza a n d Moraxella catarrhalis ( 2 0 - 2 8 % each)
A n s . is a i.e. Aspergillus sp Ref. Maqbool 11 th/edp 225; Scott Brown 7th/ed Vol 1and2 p 1452; Mohan Bansalp317
M o s t c o m m o n t y p e of f u n g a l infection of n o s e a n d p a r a n a s a l s i n u s e s a r e d u e to Aspergillus.
A. fumigatus > A. niger > A. flqyus are t h e m o s t f r e q u e n t offenders.
1 0 . A n s . is a, c a n d d i.e. S u r g e r y is r e q u i r e d for t r e a t m e n t ; A m p h o t e r e c i n B IV is u s e d for i n v a s i v e f u n g a l sinusitis; a n d S e e n
o n l y in i m m u n o d e f i c i e n t c o n d i t i o n s Ref. Maqbool 11 th/edp 225; Scott Brown 7th ed Vol2 p 1455; Mohan Bansalp 317,318

F U N G A L SINUSITIS

Most c o m m o n cause: Aspergillus


Most common species: A. fumigatus > A. niger > A. flavus. —Maqbool 11th/edp225
Other offenders are: Mucor, Rhizopus, Altemaria
• F u n g a l infection c a n b e of f o l l o w i n g t y p e s :
i. Fungus ball

ii. Allergic f u n g a l rhinosinusitis
iii. Chronic or i n d o l e n t invasive f u n g a l sinusitis
iv. A c u t e f u l m i n a n t f u n g a l rhinosinusitis
CHAPTER 6A Diseases of Paranasal Sinus—Sinusitis

F u n g u s Ball

Fungus ball occurs in adults females


M/C a g e n t : Aspergillus
Most c o m m o n sinus i n v o l v e d - Maxillary > s p h e n o i d sinus
M/C s y m p t o m - unilateral postnasal discharge
Most I m p o r t a n t Investigation-CT scan
Fungus ball is t h e m a i n f u n g a l rhinosinusitis in an i m m u n o c o m p e t e n t p a t i e n t .
Surgery is t h e m o s t effective t r e a t m e n t f o r f u n g u s ball.

Allergic Fungal Rhinosinusitis

• AFS is a noninvasive f u n g a l rhonosinusistis


• Dermatiaceous species are t h e f u n g a l agents m o s t l y responsible f o r AFRs.
• Seen in i m m u n o c o m p e t e n t hosts w i t h allergy t o f u n g u s .
• Clinical a n d biological criteria f o r diagnosis is still u n d e r d e b a t e , a n d i n c l u d e nasal polyps, t h i c k m u c i n , hypersensitivity t y p e I
for f u n g u s , eosinophilic m u c i n .
• Sinus opacities w i t h b o n e e x t e n s i o n are f r e q u e n t l y seen o n CT scan.
• Diagnosis o f all allergic f u n g a l rhinosinusitis is s u p p o r t e d by allergic a n d f u n g a l criteria (Refer t o Ans. 11 f o r criteria).
• Treatment = Antifungals

-
Chronic or Indolent Invasive Fungal Rhinosinusitis

Chronic invasive f u n g a l rhinosinusitis is a rare p a t h o l o g y o c c u r r i n g m o s t l y in i m m u n o c o m p e t e n t patients.


Aspergillus is t h e m o s t f r e q u e n t a g e n t isolated in this p a t h o l o g y .

Acute Fulminant Fungal Rhinosinusitis

• F u l m i n a n t invasive f u n g a l rhinosinusitis occurs in i m m u n o c o m p r o m i s e d patients (HIV, diabetes, c h e m o t h e r a p y )


• Early diagnosis a n d c o n t r o l o f p r i m a r y i m m u n o l o g i c a l disorders is essential f o r t h e prognosis.
T h u s f r o m t h e a b o v e d e s c r i p t i o n it c a n b e c o n c l u d e d
Option - a - Surgery is r e q u i r e d f o r t r e a t m e n t - (correct) as in all f o r m s o f f u n g a l sinusitis - s o m e or t h e o t h e r f o r m o f surgery
is required.
Option - b M/c o r g a n i s m is Aspergillus niger.
Incorrect - M/c is A fumigatus ( M a q b o o l 11 t h / e d p 228)
Option - c A m p h o t e r i c i n IV is used f o r invasive f u n g a l sinusitis
Correct - Ref. Dhingra 5th/ed p2W, 6th/ed p 196
Option - d Hazy appearance o n X-ray w i t h r a d i o p a q u e d e n s i t y
Correct - Sinusitis gives hazy appearance o n X-ray
Option - e Seen o n l y in i m m u n o d e n c i e n t c o n d i t i o n
I n c o r r e c t - o n l y t h e acute f u l m i n a n t f o r m is m o r e c o m m o n in i m m u n o d e f i c i e n t state whereas others are seen in i m m

11.
. . . — .n .o c. o.m p e t e n t hosts.
A n s . is b i.e. Orbital i n v a s i o n
Ref. Current Diagnosis and Treatment in Otorhinology 2nd/ed p 276; Scott Brown 7th/ed Vol 2 pp 1452-1454; Ear Nose and Throat
Histopathology 2nd/edp 152; Patterson's Allergic Disease 6th/edp 778; Allergy and Immunology: An Otolaryngic Approach (2001)7239

Allergic fungal sinusitis is a noninvasive form of fungal sinusitis as such orbital invasion is not its feature.

. Bent and Kuhn Criteria for Allergic Fungal Sinusitis (AFS) C T scan findings in A F S

1. Type 1 hypersensitivity (confirmed by history, skin test or serology most Areas of High attenuation surrounded by a thin zone of low
important criteria) attenuation
2. Nasal polyposis CT scan reveals pansinusitis and polyposis
3. Asthma
4. Unilateral predominance
5. Eosinophilic mucus demonstrating fungal elements, charcot-leyden crystal
6. Peripheral eosinophilia
7. Positive fungal culture
8. Charachteristic Radiological Findings (CT, MRI) absence of tissue invasion by
fungus
9. Radiographic bone erosion
SECTION I Nose and Paranasal Sinuses

Allergic F u n g a l Sinusitis (AFS): F o r m of C h r o n i c Sinusitis

• AFS in a u n i q u e t y p e o f noninvasive sinusitis caused b y T y p e I or T y p e III h y p e r s e n s i t i v i t y r e a c t i o n t o f u n g a l organisms t h a t


0

c o m e in c o n t a c t w i t h t h e nasal sinus mucosa.


• It is s e e n in a n i m m u n o c o m p e t e n t h o s t w i t h allergy to f u n g u s .
0

• IgE levels are h i g h in patients o f AFS


• It is m o s t l y caused by dermatiaceous species {Bipolaris, cunicularia,
0
alternaria) and rarely by aspergilus.
• M/C in y o u n g e r a g e grp (~ 3 0 years)
• Clinically patients present w i t h Nasal polyposis w h i c h can be U/L or B/L (U/L > B/L).
• The classic rhinoscopic f i n d i n g in AFS is t h i c k , tenacious p e a n u t b u t t e r like inspissated m u c u s in one or m o r e paranasal sinuses.

Histological e x a m i n a t i o n o f this'allergic mucin'reveals:


- Embedded eosinophils 0

- Charcot-Leyden crystals (eosinophil b r e a k d o w n p r o d u c t s ) .


0

- Extramucosal f u n g a l h y p h a e ( w i t h o u t tissue invasion).


0

• E o s i n o p h i l s are i n c r e a s e d in b l o o d
• X-ray s h o w s — b o n y extension
• O n CT scan —> Sinus o p a c i t e s w i t h e x t e n s i o n s e e n
• Treatment consists o f removal o f all m u c i n a l o n g w i t h either topical or systemic antifungals. Prednisone is also given a l o n g w i t h it.
• I m m u n o t h e r a p y is b e i n g t r i e d for its t r e a t m e n t .
• Recurrence is c o m m o n

Extra E d g e
Stage Endoscopic finding
Stage 0 No mucosal edema or allergic mucin
Stage 1 Mucosal edema with or without allergic mucin
Stage 2 Polypoid oedema with/without allergic mucin
Stage 3 Sinus polyps with fungal debris or allergic mucin.

1 2 . A n s . is b i.e. Frontal sinus infection Ref. Dhingra 5th/edp 206,6th/edp 192-193


Pain o f f r o n t a l sinusitis shows characteristic periodicity, i.e. comes u p o n w a k i n g , gradually increases and reaches its peak by m i d d a y
a n d t h e n starts subsiding. It is also called "office headache" as it is present o n l y d u r i n g office hours.
1 3 . A n s . is d i.e. None Ref. Dhingra 5th/edp 207,6th/ed p 193
Acute e t h m o i d sinusitis is o f t e n associated w i t h i n f e c t i o n o f o t h e r sinuses. E t h m o i d sinuses are m o r e o f t e n involved in infants a n d
y o u n g c h i l d r e n . It is the m o s t c o m m o n sinus i n v o l v e d in infants a n d c h i l d r e n .

Also Know

• Pain of e t h m o i d i t i s is localized over t h e b r i d g e o f t h e nose, m e d i a l and d e e p t o t h e eye. It is aggravated by m o v e m e n t s of


t h e eyeball.
• Orbital cellulitis is an early c o m p l i c a t i o n in such cases.
• Nasal d i s c h a r g e — o n anterior rhinoscopy, pus may be seen in m i d d l e or superior meatus d e p e n d i n g o n t h e i n v o l v e m e n t o f
0

anterior or posterior g r o u p o f e t h m o i d sinuses.


• S w e l l i n g of the m i d d l e t u r b i n a t e .
1 4 . A n s . is a a n d b w h e r e b > a i.e. V e r t e x > O c c i p u t Ref. Dhingra 5th/ed p 207; Turner 10th/ed p 35; Maqbool 11 th/ed p 208; Tuli 1st/
edp 188

Sometimes an easy seeming q u e s t i o n like this o n e can be really difficult t o answer.

According to Dhingra
• A c u t e s p h e n o d i t i s : 'Headache - usually localized to the occiput or vertex. Pain may also be referred to the mastoid region.'
- Dhingra 5th/ed p 207,6th/ed p 194

i.e. b o t h o p t i o n s a and b are correct.

A c c o r d i n g t o Tuli
"Sphenoidal pain—It gives rise to occipital or vertical headache and sometimes is referred to mastoid process. Pain may be felt behind
the eyeball due to close proximity with Vth nerve." - Tuli ist/ed p 188
i.e. again b o t h o p t i o n s a a n d b are c o r r e c t .
CHAPTER 6A Diseases of Paranasal Sinus—Sinusitis

According to Maqbool

"In sphenoid infection -the pain is usually referred to the vertex or occiput." - Maqbool 11 th/ed p 208

Now read w h a t Turner has to say


"The pain ofSphenoiditis, which is relatively uncommon, is localized to the top of head. It may produce pain over the trigeminal distribution
because ofthe close proximity of these nerve." Turner lOth/ed p 35
So b o t h o p t i o n s a i.e. o c c i p u t a n d o p t i o n b i.e. vertex are correct, b u t b > a.

Also Know

Pain felt in area

Maxillary sinus Along the infraorbital margin and referred to upper teeth or gums on affected side (along the distribution of superior orbital
nerve) Pain is aggravated on stooping or coughing.

Frontal sinus Pain localized over forehead. It has a characteristic periodicity

Ethmoid sinus Pain localized over the nasal bridge, inner canthus and behind the ear.

1 5 . A n s . is d i.e. Lateral v i e w Ref. Turner 1 Oth/ed p 18; Dhingra 5th/ed p 445,6th/ed p 434
1 6 . A n s . is a i.e C a l d w e l l v i » w
"Lateral view is best for the sphenoid sinus.'

ALSO KNOW

S o m e o t h e r v i e w s a n d t h e s i n u s e s best s e e n b y t h e m :
• Occipitomental/Water's v i e w - Maxillary antrum.
• Occipitofrontal/Caldwell v i e w - Frontal sinus and ethmoid sinuses
• S u b m e n t o v e r t i c a l /Basal v i e w - Sphenoid, posterior ethmoid and maxillary sinus

17. A n s . is b i.e. C a l d w e l l v i e w Ref. Dhingra 5th/ed p 445,6th/ed p 434


o In plain A P v i e w a n d Towne's v i e w w e can see t h e Temporal b o n e a n d sinus, z y g o m a , z y g o m a t i c arch a n d m a n d i b l e .
• Superior o r b i t a l fissure can be seen by caldwell v i e w a n d water's view.
1 8 . A n s . is a, b, c a n d d i.e. O r b i t a l cellulitis; Pott's puffy t u m o r ; C o n j u n c t i v a l c h e m o s i s ; a n d S u b d u r a l a b s c e s s
Ref. Scotts brown 7th/ed Vol2pp 1539,1540; Mohan Bansalp 305

Complications of Sinusitis—Acute Sinusitis

L o c a l (due t o local s p r e a d ) S y s t e m i c (due t o h e m a t o g e n o u s s p r e a d )

• Frontal sinusitis can cause Brain abscess (can occur as a result of local spread as well
- Subperiosteal abscess/or pott's puffy tumor hematogenous spread secondary to maxillary sinusitis
- Osteomyelitis associated with dental disease)
• E t h m o i d sinusitis can cause Meningitis
- Orbital cellulites Toxic shock syndrome
The stages of orbital cellulitis are:
- Preseptal cellulitis (infection anterior to orbital septum)
- Postseptal cellulitis or orbital cellulitis without abscess (i.e. infection posterior to
orbital septum)
- Subperiosteal abscess (pus collects-beneath the periosteum)
- Orbital abscess (pus collects in orbit)
- Cavernous sinus thrombosis/abscess (includes chemosis)
• Maxillary sinusitis - no acute complications
• Sphenoid sinusitis can lead to
- Cavernous sinus thrombosis
- Intracranial complications

M u c o c e l e a n d Pyocele are d u e t o Chronic Sinusitis

NOTE

If infection in the frontal sinus spreads t o the marrow of frontal bone, localized osteomyelitis w i t h bone destruction can result in a doughy
swelling o f forehead, classically called as 'Pott's Puffy Tumor'. Surgical drainage and debridement should be done in this case.
58 T SECTION I Nose and Paranasal Sinuses

1 9 . A n s . is e i.e. All of t h e a b o v e Ref. Tuli Ist/ed p 196; Scott Brown 7th/ed Vol 2 pp 1539,1540; Mohan Bansalp 305
As D i s c u s s e d in Previous Q u e s t i o n :
• There is no c o n f u s i o n r e g a r d i n g o r b i t a l cellulitis, a n d cavernous sinus t h r o m b o s i s b e i n g t h e c o m p l i c a t i o n s o f sinusitis.
• Dhingra does not m e n t i o n Retrobulbar neuritis as o n e o f t h e c o m p l i c a t i o n s o f sinusitis b u t according t o Tuli Ist/edp 196. Posterior
g r o u p o f sinuses can lead t o neuritis w i t h i m p a i r e d vision.

Complications of Posterior G r o u p of Sinuses

Superior o r b i t a l fissure s y n d r o m e / o r b i t a l apex s y n d r o m e .


> Cavernous sinus t h r o m b o s i s .
-

Neuritis w i t h i m p a i r e d vision.
• Oroantral fistula/sublabial fistula.
2 0 . A n s . is d i.e. All of t h e s e Ref. Scott Brown's 7th/ed Vol 2 p 1485; Parson disease of eye 20th/edp 457
Orbital cellulitis can occur as a c o m p l i c a t i o n o f sinusitis a n d injuries. As far as endoscopic sinus surgery is c o n c e r n e d , it can lead t o
o r b i t a l a n d intracranial c o m p l i c a t i o n s so o r b i t a l cellulitis can occur in it also.
2 1 . A n s . is a i.e. C a v e r n o u s s i n u s Ref. Dhingra 5th/ed p214,6th/edp 201; Mohan Bansal p 307
2 2 . A n s . is c i.e. C a v e r n o u s s i n u s t h r o m b o s i s
Cavernous sinus t h r o m b o s i s is a c o m p l i c a t i o n o f o r b i t a l cellulites (As explanation in Ans. 18)

Cavernous SINUS INFECTION

R o u t e of S p r e a d
• E t h m o i d s i n u s (most common) via o p h t h a l m i c veins
• S p h e n o i d sinus by d i r e c t spread.
• Frontal sinus via supraorbital a n d o p h t h a l m i c veins.
• O r b i t b y o p h t h a l m i c veins.
• U p p e r lid v i a A n g u l a r v e i n a n d o p h t h a l m i c v e i n s .
• Ear by petrosal v e n o u s sinuses.

Clinical Features

• Onset is a b r u p t w i t h fever, chills a n d rigor


• It is bilateral
• I n v o l v e m e n t o f IIIrd, IVth, V t h a n d V l t h cranial nerves
• Chemosis of conjunctiva
• Proptosis ofthe eye with l i m i t e d m o v e m e n t s
• Papilledema
• Pupils are d i l a t e d a n d fixed
• Decreased vision
• Decreased sensation in d i s t r i b u t i o n o f V t h nerve ( o p h t h a l m i c division)
• Progressive o p h t h a l m o p l e g i a (specially f o r lateral gaze)

Treatment

It is a life-threatening c o n d i t i o n . A n t i b i o t i c s are given in h i g h doses f o r 4-6 weeks.

• The incidence o f all o r b i t a l c o m p l i c a t i o n s i n c l u d i n g cavernous sinus t h r o m b o s i s are c o m m o n in pediatric age g r o u p .


• Visual p r o b l e m s are present f r o m stage III o n w a r d o f o r b i t a l cellulitis i.e. f r o m stage o f abscess f o r m a t i o n .
• O r b i t a l c o m p l i c a t i o n s are a l m o s t always secondary t o e t h m o i d rhinosinusitis b u t may occur w i t h f r o n t a l rhinosinusitis also, j

2 3 . A n s . is c i.e. S i n o s c o p y Ref. Scott Brown 7th/ed Vol2p 1442; Current Otolaryngology 2nd/ed p 277; Turner lOth/edp 43
A c c o r d i n g t o Scott Brown's 7th/ed Vol 2 p 1142
"There are many possible methods to make diagnosis of rhinosinusitis but there is much debate related to best method. It has become
increasingly clear that the diagnosis ofABRS (acute bacterial rhinosinusitis) is best made on clinical grounds and criteria."
But this o p t i o n is n o t g i v e n .
Scoff Brown's further says:
"At this time, a maxillary sinus tap with cultures, revealing pathogenic organism remains the gold standard for the diagnosis ofABRS,
although there is increasing interest in the role of endoscopic-guided middle meatal cultures, in lieu ofmaxillcfry sinus tap. It has even
been suggested that endoscopically guided cultures may be a preferred culture technique to maxillary sinus taps, as they can identify
patients with ethmoid infection." Scott Brown 7th/edVol2p 1442
CHAPTER 6A Diseases of Paranasal Sinus—Sinusitis

According to Current otolaryngology 2nd/ed p 277


• In a c u t e b a c t e r i a l rhinosinusitis: Endoscopy is useful t o c o n f i r m t h e diagnosis a n d t o o b t a i n cultures at t h e m i d d l e meatus.
• In c h r o n i c sinusitis: Gold standard is maxillary sinus aspiration (and s u b s e q u e n t c u l t u r e o f aspirated material) b u t e n d o s c o p y
w h i c h helps in visualizing t h e sinus as w e l l as o b t a i n i n g , is s l o w l y g a i n i n g i m p o r t a n c e .
So nowadays sinoscopy is a better o p t i o n t h a n p r o o f p u n c t u r e w h i c h has b e c o m e obsolete as it causes a lot o f p a t i e n t d i s c o m f o r t .

Also Know

I m a g i n g m o d a l i t y o f choice f o r sinus disease - CT s c a n 0

2 4 . A n s . is a i.e. M u c o p u s in m i d d l e m e a t u s Ref. Dhingra 5th/ed p 205


• C h a r a c t e r i s t i c finding of m a x i l l a r y sinusitis on R h i n o s c o p y is pus or m u c o p u s in in t h e m i d d l e meatus.
• Mucosa a n d t u r b i n a t e s m a y appear red a n d s w o l l e n .

; R e m e m b e r : Dental infections are an i m p o r t a n t source o f maxillary sinusitis.

2 5 . A n s . is b i.e. Fiber optic e n d o s c o p i c s i n u s s u r g e r y


Ref. Current otolaryngology 2nd/ed pp 279,280; Dhingra 5th/edpp 205,209

M a n a g e m e n t o f A c u t e S i n u s i t i s (Maxillary) is M a i n l y C o n s e r v a t i v e w i t h t h e H e l p o f

i. A n t i b i o t i c s - a m p i c i l l i n / a m o x i c i l l i n
ii. Nasal d e c o n g e s t a n t d r o p s
iii. Steam i n h a l a t i o n
iv. Analgesics
v. H o t f o m e n t a t i o n
-

• Rarely w h e n medical m a n a g e m e n t fails


• In acute maxillary sinusitis antral lavage is d o n e .

Chronic Sinusitis

M e d i c a l m a n a g e m e n t - It is t h e t r e a t m e n t o f choice

i. A n t i b i o t i c s ( d e p e n d i n g o n culture)
ii. Nasal a n d systemic steroids
iii. Antihistaminics
iv. Decongestants

Surgery

E n d o s c o p i c sinus s u r g e r y Open surgery

"Recently endoscopic sinus surgery is replacing radical operations on It is rarely required


the sinuses and provides good drainage and ventilation. It also avoids M/c operation done - Caldwell-Luc antrostomy
external incision" - Dhingra 5th/edp 209

"The improvement in symptoms with functional endoscopic sinus surgery may be expected in > 9 0 % patients."- Current otolaryngology 2nd/ed p 279

2 6 . A n s . is a i.e. Swelling a b o v e m e d i a l c a n t h u s , b e l o w the floor of frontal s i n u s


2 7 . A n s . is a i.e. Frontal Ref. Dhingra Sth/edp 211, 6th/ed p 198;Tuli1st/edp 196; Scott Brown 7th/ed Vol 2 p 1531
A m u c o c e l e is an epithelial l i n e d , m u c u s c o n t a i n i n g sac c o m p l e t e l y f i l l i n g t h e sinus and capable o f e x p a n s i o n :
• M u c o c e l e are most commonly f o r m e d in Frontal sinus f o l l o w e d b y e t h m o i d , s p h e n o i d a n d maxillary sinuses.
• M u c o c e l e o f f r o n t a l sinus presents as a s w e l l i n g in t h e f l o o r o f f r o n t a l sinus a b o v e t h e inner (medial) canthus. It displaces t h e
eyeball f o r w a r d , d o w n w a r d a n d laterally.
IOC = CT scan
T O C = Endoscopic sinus surgery

According to Dhingra, 6th/ed p 198

• Least c o m m o n sinus assopciated w i t h M u c o c e l e f o r m a t i o n is s p h e n o i d .


• But Scott Brown 7th/ed Vol 2 p 1531 says:
- M o s t o f t h e cases o f m u c o c e l e o f s p h e n o i d sinus are referred t o neurosurgeons. Therefore, it seems it is less c o m m o n b u t
actually t h e sinus least i n v o l v e d by m u c o c e l e is maxilla.
60 ]_ SECTION I Nose and Paranasal Sinuses

2 8 . A n s . is c i.e. Frontal s i n u s Ref. Scott Brown 7th/ed Vol 2 p 1521


• Craniofacial osteomas are b e n i g n t u m o r s o f t e n o r i g i n a t i n g in t h e paranasal sinuses
• The f r o n t a l sinus is t h e m o s t f r e q u e n t l o c a t i o n f o l l o w e d by t h e e t h m o i d , maxillary a n d s p h e n o i d sinus, respectively
• Age o f p r e s e n t a t i o n = second t o f i f t h d e c a d e w i t h a m a l e - f e m a t e ratio - 3:1.
• Presentation:
- Generally t h e y are an i n c i d e n t a l f i n d i n g o n r a d i o g r a p h y
- It m a y p r o d u c e s y m p t o m s like -
- Visual i m p a i r m e n t

- Intracranial neurological c o m p l i c a t i o n s like m e n i n g i t i s or p n e u m o c e p h a l u s w i t h seizure.

Management
Removal by e n d o s c o p i c sinus surgery.
2 9 . A n s . is a i.e. C T s c a n Ref. PL Dhingra 5th/edpp 213-208.
The child is presenting w i t h fever a n d p u r u l e n t nasal discharge w i t h X-ray PNS s h o w i n g opacification o f e t h m o i d a l sinus, i.e. p r o b a b l y
t h e c h i l d is h a v i n g chronic sinusitis (as it is present for t h e past 2 m o n t h s ) w i t h an acute e x a c e r b a t i o n . N o w t h e m o s t d r e a d e d
c o m p l i c a t i o n o f e t h m o i d a l sinusitis is o r b i t a l c o m p l i c a t i o n .
"Orbital complication -most ofthe complications, follow infection ofethmoids as they are separated from the orbit only by a thin lamina
of bone - lamina papyracea. Infection travels from these sinuses either by ostitis or a thrombophlebitic process of ethmoidal veins."
- Dhingra Sth/edp 213
The best m e t h o d t o assess t h e status o f e t h m o i d a l air cells a n d its c o m p l i c a t i o n s is CTscan.
"CTis particularly useful in ethmoid and sphenoid sinus infections and has replaced studies with contrast material."
- Dhingra 5th/edp 209
3 0 . A n s . is d i.e. F u n c t i o n l e n d o s c o p i c s i n u s s u r g e r y
$ 1. A n s . is c i.e. B o t h Ref. Dhingra 5th/ed p 429,6th/ed p419; Head and Neck surgery DeSouza p127;
Scott Brown 7th/ed Vol 2 p 1481

E n d o s c o p i c S i n u s S u r g e r y is I n d i c a t e d i n

0 nemonic
India's Selected Prime Minister Don't Correct Speak Fluent English
• India's - Inflammation of sinus, i.e. sinusitis viz.,
- Recurrent acute sinusitis
- Chronic bacterial sinusitis unresponsive t o medical treatment
- Fungal sinusitis
- Polypoid sinusitis/sinonasal polyposis —Maqbool 1) th/ed, p21'6
(Functional Endoscopic sinus surgery)
• Selected - Selected tumor resection
• Prime - Polyp (Antrochoanal/ethmoidal)
• Minister - Mucocele of frontoethmoid or sphenoid sinus
• Don't - Dacrocystorhinostomy
• Speak - Septoplasty—endoscopic
• Correct - Choanal atresia and CSF leak repair.
• Fluent - Removal of Foreign body from nose or sinus
• English - Epistaxis (control of epistaxis by endoscopic cautery)

32. A n s is a, b a n d c i.e. i n v e r t e d p a p i l l o m a , n a s a l allergic p o l y p o s i s ; a n d m u c o c e l e


Ref. Scott Brown 7th/ed Vol 1pp 1481,1523-1524

Functional Endoscopic Sinus Surgeries are Indicated -

O n l y in sinonasal i n f l a m m a t o r y disease i n c l u d i n g sinusitis, polyposis, mucoceles a n d AFRS. In case o f i n v e r t e d p a p i l l o m a a n d , Ca


maxilla, endonasal route/endoscopes are b e i n g used b u t t h e y are n o t f u n c t i o n a l surgeries.

NOTE

In carcinoma maxilla, biopsy should n o t be taken via Caldwell-Luc as it leads t o spread o f t h e neoplasm t o cheek.

3 3 . A n s . is b, c a n d d i.e. a n g i o i n v a s i o n , long-term d e f e r o x a m i n e t h e r a p y a n d s e p t a t e h y p h a e .
Ref. (Current Otolaryngology 3rd/ed p 295)


CHAPTER 6A Diseases of Paranasal Sinus—Sinusitis J 61

• M u c o r m y c o s i s is caused by Rhizopus species, Rhizomucus a n d Absidia species.


• Intitially, t h e disease runs a subtle course w i t h o n l y fever a n d r h i n o r r h e a . Latter o n , it invades t h e o r b i t a n d intracranial cavity
w i t h rapid loss o f vision, m e n i n g i t i s , cavernous sinus t h r o m b o s i s a n d m u l t i p l e cranial nerve palsies.
• It has m a r k e d p r e d i l e c t i o n f o r vascular invasion leading t o w i d e s p r e a d t h r o m b o s i s , tissue necrosis, a n d g a n g r e n e .
• Characteristic nasal f i n d i n g is a d a r k necrotic t u r b i n a t e s u r r o u n d e d by pale mucosa blackish discharge a n d crusts.
• M/C site is m i d d l e t u r b i n a t e f o l l o w e d by m i d d l e meatus a n d s e p t u m .
• Investigation o f choice is MRI, w h i l e b i o p s y is c o n f i r m a t o r y .
T r e a t m e n t : Includes a m p h o t e r i c i n - B , h e p a r i n , hyperbaric o x y g e n , a n d d e b r i d e m e n t .
34. A n s . is a i.e. C o n s t r i c t e d pupil in r e s p o n s e t o light Ref. Dhingra 5th/edp214
Ptosis a n d o p h t h a l m o p l e g i a occur in cavernous sinus t h r o m b o s i s d u e t o i n v o l v e m e n t o f III, IV a n d V cranial nerves. Retinal vessels
are also e n g o r g e d b u t pupies are f i x e d a n d d i l a t e d ( n o t c o n s t r i c t e d ) .
35. A n s . is a i.e. F r o n t a l l o b e a b s c e s s Ref. Read below
• Patient is presenting w i t h fever, headache a n d personality changes w h i c h is typical of frontal l o b e abscess (which is a c o m p l i c a t i o n
1
o f chronics sinusitis). In m e n i n g i t i s a n d encephalitis a l t h o u g h p a t i e n t presents w i t h fever a n d headache, b u t personality changes
are n o t seen.
• Frontal b o n e osteomyelitis (Pott's p u f f y t u m o r ) presents as d o u g h y swelling o n f o r e h e a d .
36 A n s . is is a i.e. M a x i l l a r y
37, A n s is a i.e. M a x i l l a r y Ref. Mohan Bansal Ist/edp 37
Maxillary sinus is also called as a n t r u m of h i g h m o r e a n d is t h e first t o d e v e l o p in h u m a n fetus.
It is t h e largest paranasal sinus (15 m l capacity in adults).
38. A n s . is a i.e. C a v e r n o u s s i n u s t h r o m b o s i s Ref. Dhingra 6th/edp 204
Friends alwasy r e m e m b e r in cavernous sinus t h r o m b o s i s t h e r e is bilateral o r b i t a l i n v o l v e m e n t whereas in o r b i t a l cellulitisit, i t is
unilateral.
Differences b e t w e e n o r b i t a l cellulitis a n d cavernous sinus t h r o m b o s i s

Orbital cellulitis C a v e r n o u s sinus t h r o m b o s i s


Source Commonly ethmoid sinuses Nose, sinuses, orbit, ear and pharynx
Onset and progress Show Abrupt
Crania nerve involvement Involved concurrently with complete ophthalmoplegia Involved individually and progressively
Side Usually involve affected side eye Involves both eyes
Toxemia Absent Present
Fever Present High temperature with chills
Mortality Less Very high


-

'9£9Bf.
CHAPTER
Diseases of Paranasal
Sinus—Sinonasal Tumor

. Features:
INONASAL TUMOR It s h o w s f i n g e r - l i k e e p i t h e l i a l i n v a s i o n s i n t o t h e
underlying stroma of the e p i t h e l i u m rather than on
| PREDISPOSING FACTORS
surface so-called i n v e r t e d p a p i l l o m a
It is usually unilateral and is a locally aggressive tumor.
• Nickel w i t h d u r a t i o n o f exposure ( a p p r o x i m a t e l y 18-36 years)
Patients c o m p l a i n o f U/L nasal o b s t r u c t i o n rhinorrhea a n d
p r e d i s p o s e s t o s q u a m o u s cell c a r c i n o m a a n d a n a p l a s t i c
unilateral epistaxis
carcinoma.
In 1 0 - 1 5 % cases t h e r e m a y be associated s q u a m o u s cell
• H a r d w o o d a n d s o f t w o o d predisposes t o A d e n o c a r c i n o m a o f
carcinoma (i.e. Premalignant c o n d i t i o n ) .
e t h m o i d a l sinus.
• T r e a t m e n t : Medical m a x i l l e c t o m y is t h e t r e a t m e n t o f choice. It
can be p e r f o r m e d by lateral r h i n o t o m y or sub labial d e g l o v i n g
Other Agents
approach.These days e n d o s c o p i c a p p r o a c h is preferred.
• Hydrocarbons • They have a t e n d e n c y t o recur after surgical removal (as it is
• M u s t a r d gas multicentric).
• Radium dial w o r k e r s : Soft tissue sarcoma
• Welding/soldering 1 MALIGNANT TUMORS OF NOSE
• A g e at p r e s e n t a t i o n : 5 t h decade
• S e x : Male: Female = 2:1 S q u a m o u s C e l l C a r c i n o m a is t h e M o s t C o m m o n
Histological Type of T u m o r
1. M/C m a l i g n a n c y o f nasal skin = Basal cell carcinama
• Also k n o w n as nose pickers cancer
2. M/C b e n i g n t u m o r o f nose = Capillary h e m a n g i o m a (arises
• Site: Lateral wall o f nose is m o s t c o m m o n l y i n v o l v e d .
f r o m nasal s e p t u m )
• Nasal cancer may be an e x t e n s i o n f r o m maxillary or e t h m o i d
3. M/C b e n i g n t u m o r o f paranasal sinus = Osteoma (M/C site
cancer.
f r o n t a l sinus)
• Metastasis is rare.
4. M/C m a l i g n a n t t u m o r o f a nose a n d PNS = S q u a m o u s cell
• A g e : Seen in m e n > 50 years o f age
carcinoma f o l l o w e d by a d e n o c a r c i n o m a .
• T r e a t m e n t : is c o m b i n a t i o n o f r a d i o t h e r a p y a n d surgery.

Papilloma Malignant Melanoma


• Site: Skin o f t h e nasal vestibule a n d t h e a n t e r i o r p a r t o f t h e • A g e : > 50 year
septum. • G r o s s : bluish-black-polypoidal mass.
• T r e a t m e n t : Cautery/cryotherapy • M o s t c o m m o n site: A n t e r i o r p a r t o f nasal s e p t u m
• T r e a t m e n t : W i d e surgical excision.
I n v e r t e d Papilloma/Transitional cell papilloma/Schnei-
derian Papilloma/Ringertz tumor Olfactory Neuroblastoma (Esthesioneuroblastoma)

• A g e : 4 0 - 7 0 years ( = 50 years) • Neuroendocrine t u m o r


• Sex: Male > Female • A g e : T w o p e a k s — o n e at 1 1 - 2 0 years a n d s e c o n d o n e at
• Site: Lateral nasal wall in m i d d l e meatus rarely o n t h e s e p t u m 50-60 years
• It is associated w i t h human papilloma virus 0
• It is M/C in females
CHAPTER 6B Diseases of Paranasal Sinus—Sinonasal Tumor J 63
• S i t e : U p p e r p a r t (upper t h i r d ) o f t h e nasal cavity. It can spread Lymphatic Spread
intracranially; requires anterior craniofacial resection f o l l o w e d
• Nodal metastases are u n c o m m o n
b y RT/CT.
• Earliest metastasis occurs t o Retropharyngeal l y m p h n o d e
• C o m m o n e s t LN involved is s u b m a n d i b u l a r l y m p h n o d e .
A d e n o i d Cystic Carcinoma

• S i t e : A n t r u m a n d Nose Diagnosis
• O n m i c r o s c o p i c e x a m i n a t i o n : Swiss - cheese p a t t e r n is seen. • Biopsy
• Has a p o t e n t i a l o f perineural spread • CECT o f Nose a n d PNS (Best investigation)

| PARANASAL SINUS TUMOR Classification

1. O h n g r e n ' s Classification:
Benign Neoplasms
o A n imaginary plane d r a w n e x t e n d i n g between medial
Osteoma canthus o f eye a n d angle o f m a n d i b l e .
• C o m m o n e s t site: M a n d i b l e • G r o w t h s a b o v e t h i s p l a n e have p o o r e r p r o g n o s i s t h a n
• C o m m o n e s t site in the u p p e r j a w : F r o n t o e t h m o i d a l area t h o s e b e l o w it.
• Most c o m m o n sinus involved is Frontal > Ethmoids > Maxillary
sinus
• Features:
M o s t o f t h e m are clinically silent
If close t o t h e o s t i u m , it can lead t o f o r m a t i o n o f mucocele.

Malignant Tumors of Paranasal Sinus


Etiology
Seen m o r e c o m m o n l y in p e o p l e w o r k i n g i n :
• H a r d w o o d f u r n i t u r e i n d u s t r y leads t o a d e n o c a r c i n o m a o f
e t h m o i d a n d u p p e r nasal c a v i t y (called as w o o d w o r k e r s
carcinoma)
• N i c k e l r e f i n i n g leads t o s q a m o u s cell Ca a n d a n a p l a s t i c
carcinoma.
• Leather i n d u s t r y
• M a n u f a c t u r e o f m u s t a r d gas

Fig. 6.1: Ohngren's classification.Ohngren's line is an i m a g i n a r y


NOTE
line (OL), w h i c h extends b e t w e e n medial canthus a n d t h e angle o f
While h a r d w o o d is a carcinogen for sinonasal adenocarcinoma, m a n d i b l e , divides t h e maxilla i n t o t w o regions anteroinferior (Al)
softwood exposure increases risk of squamous cell carcinoma. a n d posterosuperior (PS). Al g r o w t h s are easy t o manage and have
better prognosis t h a n Ps t u m o r s .
Histology Coutesy: Textbook of Diseases of Ear, Nose and
• 8 0 % Sqamous cell C a Q Throat, Mohan Bansal. Jaypee Brothers, p 357
• O t h e r s : A d e n o c a r c i n o m a , A d e n o i d cystic carcinoma,
2. T N M Classification a n d S t a g e g r o u p i n g s of the p a r a n a s a l
M e l a n o m a a n d sarcomas
• Site: M/c Maxillary a n t r u m f o l l o w e d by e t h m o i d sinus, f r o n t a l sinuses. This classification is n o t i m p o r t a n t f r o m P G
a n d s p h e n o i d series E n t r a n c e point of view.
• A g e : Seventh decade o f life 3. L e d e r m a n ' s classification (Fig. 6.2):
• S e x : Male > Female
2 h o r i z o n t a l lines o f Sebileau are d r a w n :
• S y m p t o m s : Silent for l o n g t i m e .
One - Passing t h r o u g h floors o f o r b i t
Early features Late features depend on the Other - T h r o u g h f l o o r o f antra
spread

• Nasal stuffiness • Medial - Nasal cavity, ethmoids Thus Dividing this Area into
• U/L Epistaxis • Anterior - Cheek
• Facial paraesthesia or pain • Inferior - alveolus leading to • Suprastructure - e t h m o i d , s p h e n o i d , f r o n t a l sinus
• Epiphora Malocclusion, loose teeth • Mesostructure - maxillary sinus a n d respirator area o f nose
• Dental pain leading • Superior - Orbit leading t o • Infrastructure-alveolar process
t o frequent change of Diplopia, Proptosis loss of vision
Treatment
dentures • Posterior - Pterygoid plates
leading to tresmus Intracranial • For s q u a m o u s cell c a r c i n o m a — r a d i o t h e r a p y or surgery.
spread can also occur • Surgery—Total or Extended m a x i l l e c t o m y
SECTION I Nose and Paranasal Sinuses

Fig. 6.2: Lederman's classification. T w o h o r i z o n t a l lines o f Sebi-leau, o n e passing t h r o u g h t h e o r b i t floors(l) a n d o t h e r t h r o u g h antral


floors(ll), d i v i d e t h e area i n t o t h r e e regions: Suprastructure (ss), m e s o s t r u c t u r e (ms), a n d Infrastructure (Is).The vertical line (III) at t h e plane
of m e d i a l w a l l o f o r b i t separates e t h m o i d sinuses a n d nasal fossa f r o m t h e maxillary sinuses.
Coutesy: Textbook of Diseases of Ear, Nose and Throat, Mohan Bansal. Jaypee Brothers, p 357

• Incision U s e d : Weber-Ferguson i n c i s i o n (see s e c t i o n o f • Stage III a n d IV: C o m b i n e d radiation and surgery. Radiotherapy
pictorial questions) can be given before o n after surgery (preferably postoperatively)
• Both r a d i o t h e r a p y and surgery have equal results in stage I
Ethmoid Sinus Malignancy
a n d II.
• Prognosis: 5 year cure rate o f 3 0 % . • O f t e n i n v o l v e d f r o m e x t e n s i o n o f maxillary carcinoma.
• Prognosis—poor


CHAPTER 6B Diseases of Paranasal Sinus—Sinonasal Tumor

QUESTIONS
Inverted papiloma: [PGI 02; PGI Nov 09] Wood workers are associated sinus C a : [PGI Dec 06]
a. Is c o m m o n in children Arises f r o m lateral wall a. Adeno Ca
c. Always b e n i g n Can be premalignant b. Squamous cell Ca
e. Causes epistaxis Recurrence is rare c. Anaplastic Ca
2. T r u e a b o u t i n v e r t e d p a p i l l o m a : [PGI Dec 08] d. Melanoma
a. Arises mainly from nasal s e p t u m Early maxillary c a r c i n o m p r e s e n t s a s : [PGI 90]
b. C o m m o n in children a. Bleeding per nose b. Supraclavicular l y m p h node
c. Risk o f malignancy c. Tooth pain d. Nasal discharge
d. Postoperative radiotherapy useful 8. C a m a x i l l a r y s i n u s s t a g e III ( T 3 NO MO), t r e a t m e n t o f
e. Also k n o w n as Scheiderian papilloma choice is/Ca maxillary s i n u s is t r e a t e d b y :
3. I n v e r t e d p a p i l l o m a is c h a r a c t e r i z e d by all e x c e p t : [TN 06; AP 05; AIIMS 01, AIIMS 97]
Radiotherapy
[MP 06]
Surgery + Radiotherapy
Also called as Schneiderian papilloma
Chemotherapy
Seen more often in females
Chemotherapy + Surgery
Presents w i t h epistaxis and nasal obstruction
T r u e a b o u t B a s a l Cell C a r c i n o m a [PGI 04]
Originates f r o m lateral wall o f nose
a. Equal incidence in male and female
C o m m o n a b o u t t u m o r s o f PNS a n d Nasal C a :
C o m m o n e r on the trunk
[PGI Dec 06]
Radiation is the only treatment
a. Squamous cell Ca is t h e MC t y p e
C o m m o n l y metastasize
b. A d e n o Ca is t h e MC t y p e Chemotherapy can be given
c. Melanoma can occur
10. W h i c h o f t h e f o l l o w i n g n a s a l t u m o u r s o r i g i n a t e s f r o m
M o s t c o m m o n m a l i g n a n c y in m a x i l l a r y a n t r u m is: the olfactory mucosa? [Al 12]
[PGI 93]
a. Neuroblastoma
a. M u c o e p i d e r m o i d Carcinoma
b. Nasal glioma
b. Adeno cystic Ca c. Esthesioneuroblastoma
c. Adenocarcinoma d. Antrochoanal polyp
d. Squamous cell Ca

EXPLANATIONS AND REFERENCES


1. A n s . is b, d a n d e i.e. Arises f r o m lateral w a l l ; C a n b e p r e m a l i g n a n t ; a n d C a u s e s epistaxis
2. A n s . is c a n d e i.e. Risk of m a l i g n a n c y ; a n d a l s o k n o w n a s S c h n e i d e r i a n p a p i l l o m a
3. A n s . is b i.e. S e e n m o r e o f t e n in f e m a l e s
Ref. Dhingra 5th/ed p 216; Logan Turner Wth/ed p 56; Current Otolaryngology 2nd/ed p 289,6th/ed p 202; Mohan Bansal p354
• Inverted papilloma is a transitional cell papilloma also called Schneiderian papilloma/Ringertz Tumor.
• It is t h e m o s t common benign neoplasm ofthe nose and paransal sinuses.
• It presents in t h e middle age (40-70 years) as soft friable mass r e s e m b l i n g p o l y p .
• More common in males (male:female = 5:1).
• Arises a l m o s t exclusively f r o m t h e lateral wall ofthe nose a n d o n l y occasionally f r o m t h e s e p t u m . Can e x t e n d i n t o t h e e t h m o i d
a n d maxillary sinuses.
• It is always unilateral. 0

• Patients m a i n l y c o m p l a i n o f nasal obstruction, rhinorrhea a n d unilateral epistaxis-Current s 2nd/edp 289


• There is c o i n c i d e n t a l m a l i g n a n c y elsewhere in t h e u p p e r respiratory t r a c t in a b o u t 4 % o f t h e cases a n d malignant transformation
ofthe tumor occurs in about 8% cases.
• T r e a t m e n t is a d e q u a t e local excision. If it arises in maxillary sinus, t h e n a medical m a x i l l e c t o m y is surgery o f chace. If it arises
in t h e e t h m o i d a l sinus, an external e t h m o i d e c t o m y is d o n e . If it arises f r o m nose, t r e a t m e n t is w i d e surgical excision by lateral
rhinotomy.
• It has a t e n d e n c y t o recur even after r e m o v a l .
A n s . a a n d c i.e. S q u a m o u s cell C a is t h e m o s t c o m m o n t y p e ; a n d M e l a n o m a c a n occur
Ref. Dhingra 5th/ed pp 217,219,6th/ed p 205
A n s . is d i.e. S q u a m o u s cell A Ref. Dhingra 5th/edpp 219,220,6th/ed p 205
• M o r e t h a n 8 0 % o f t h e m a l i g n a n t t u m o r s o f paransal sinus a n d o f nose are o f s q u a m o u s cell variety. Rest are A d e n o c a r c i n o m a ,
A d e n o i d cystic carcinoma, M e l a n o m a a n d various types o f sarcomas.
SECTION I Nose and Paranasal Sinuses

• M a x i l l a r y s i n u s is t h e m o s t f r e q u e n t l y i n v o l v e d s i n u s . O t h e r sites i n decreasing order are nasal cavity, e t h m o i d sinuses,


f r o n t a l a n d s p h e n o i d sinus.
6. A n s . is a i.e. A d e n o C a Ref. Dhingra 5th/edp 219,6th/edp 205
• Workers o f f u r n i t u r e i n d u s t r y d e v e l o p a d e n o c a r c i n o m a o f t h e Ethmoids a n d u p p e r nasal cavity. While t h o s e e n g a g e d in Nickel
refining g e t s q u a m o u s cell a n d Anaplastic c a r c i n o m a .
7. A n s . is a a n d c i.e. B l e e d i n g p e r n o s e ; a n d Tooth p a i n
Ref. Current Otolaryngology 3rd/edp312; Scott Brown 7th/ed Vol 2, p 2424; Mohan Bansal p 358

Maxillary Carcinoma

• It is seen m o r e c o m m o n l y in t h e 7 t h decade o f l i f e . 0

• Males are affected m o r e c o m m o n l y t h a n f e m a l e s 0

• Since cancer is c o n f i n e d t o t h e b o n y walls o f t h e sinus cavity, maxilary cancer usually present very late; t h e o n l y early s y m p t o m s
may be loosening o f t e e t h , f r e q u e n t change o f dentures, pain in maxillary t e e t h , epistaxis a n d infra-orbital neuralogias/numbness.
These are d u e t o i n v o l v e m e n t o f t h e alveolus, nasal cavity a n d infra-orbital nerve by t h e t u m o r . {Kindly read the text for more details)
8. A n s . is b i.e. S u r g e r y + R a d i o t h e r a p y
Ref. Dhingra 5th/edp 222,6th/ed p 205; Current Otolaryngology 2nd/ed p 290; Mohan Bansal p358
For stage III s q u a m o u s cell carcinoma, a c o m b i n a t i o n o f r a d i o t h e r a p y a n d surgery gives b e t t e r result t h a n either alone. As far as
paranasal sinus is c o n c e r n e d - R a d i o t h e r a p y can b e g i v e n either before o r after surgery, generally a f u l l course o f p r e o p e r a t i v e
t e l e c o b a l t t h e r a p y is g i v e n f o l l o w e d by surgical excision o f t h e g r o w t h by t o t a l or e x t e n d e d m a x i l l e c t o m y (incision used—Weber-
Ferguson incision).

9. A n s . is e i.e. C h e m o t h e r a p y c a n b e g i v e n Ref. Current Otolaryngology 3rd/edpp 238,239; Scott Brown 7th/ed Vol2,pp 1705,1706

B a s a l Cell C a r c i n o m a
• Usually seen in m i d d l e age a n d a b o v e (40-80 years)
• M/C in Males
• M a i n e t i o l o g y is UV exposure.
. Usually seen a b o v e a line j o i n i n g angle o f m o u t h a n d ear l o b u l e .
• C o m m o n e s t site is inner canthus o f eye.
• C o m m o n e s t variety is N o d u l a r (painless shiny n o d u l e ) . Later it f o r m s an ulcer w i t h hard raised edges.
• It is a locally i n f i l t r a t i n g t u m o r w h i c h m a y e r o d e s u r r o u n d i n g tissue. Hence also k n o w n as Rodent ulcer.
• No l y m p h a t i c / b l o o d s t r e a m spread.
• Diagnostic p r o c e d u r e o f chace is W e d g e biopsy.
• T r e a t m e n t o f choice is w i d e surgical excision.
• C h e m o t h e r a p y in t h e f o r m o f t o p i c a l 5 % i m i q u i m o d , t o p i c a l 5 f l u o r o u r a c i l is also b e i n g u s e d .
• In p a t i e n t s > 6 0 years = Radiotherapy is t h e t r e a t m e n t .

N o t e — M o h s S u r g e r y is b e i n g d o n e i n B a s a l C e l l C a c r i n o m a

It involves sequential excision o f t h e t u m o r u n d e r frozen section c o n t r o l w i t h 1 0 0 % e v a l u a t i o n o f t u m o r margins. Specimens are


evaluated o n a h o r i z o n t a l basis ( n o r m a l frozen sections give us o n l y 1 0 % t u m o r m a r g i n a n d specimen is evaluated o n a vertical
basis.)
M o h s surgery is useful f o r basal cell carcinoma arising in d i f f i c u l t areas like inner canthus w h e r e w i d e excision m a y n o t be practical
a n d f o r recurrent t u m o r s .

1 0 . A n s . c i.e. Esthesioneuroblastoma Ref. Dhingra 5th/edp 217-218,6th/ed p 204; Current Otolaryngology 3rd/edp313

Esthesioneuroblastoma
Esthesioneuroblastoma (ENB), also k n o w n as olfactory neuroblastoma, is a rare neoplasm o r i g i n a t i n g f r o m o l f a c t o r y n e u r o e p i t h e l i u m
superior t o m i d d l e t u r b i n a t e . T h e y are initially unilateral a n d can g r o w i n t o t h e adjacent sinuses, contralateral nasal cavity a n d t h e y
can spread t o o r b i t and brain. It can cause paraneoplastic s y n d r o m e by secreting vasoactive petides. Since it can spread intracranially
craniofacial resection is t h e surgery o f choice. C o m b i n a t i o n t h e r a p y (Surgery + RT + CT) is used in m a n a g e m e n t .

NOTE

Contrary t o other nasal malignancies it is M/C in females


CHAPTER

Oral Cavity

| SUBMUCOUS FIBROSIS 11 Surgical


• I n d i c a t e d in a d v a n c e d cases t o relieve t r i s m u s .
• C h r o n i c i n s i d i o u s process c h a r a c t e r i z e d by f i b r o s i s in
• Includes release o f fibrosis followed by skin grafting or use o f flaps.
s u b m u c o s a l l a y e r s of oral cavity.
• Joshi in 1953 first described this c o n d i t i o n in India.
| TUMORS OF ORAL CAVITY
Etiology
• M o s t c o m m o n cancer o f oral cavity in W o r l d : Co tongue
• P r o l o n g e d local irritation: Due t o mechanical a n d c h e m i c a l • M o s t c o m m o n cancer o f oral cavity in India: Buccal mucosa >
i r r i t a t i o n caused by c h e w i n g b e t e l n u t , areca n u t , t o b a c c o , etc Anterior tongue
• D i e t a r y d e f i c i e n c y : V i t a m i n A, Zinc a n d a n t i o x i d a n t s . • Most common t y p e o f oral cancer: Squamous cell carcinoma.
—Bailey and Love 24th/ed p 704
• Localized c o l l a g e n disease.
• R a c i a l : m a i n l y affects Indians.
Etiology a n d Risk Factors for T u m o r of Oral Cavity
• In India it is most common in p o o r s o c i o e c o n o m i c status.
^^nemonic: (6S)
Pathology
• Smoking/tobacco chewing
• Epithelial a t r o p h y a n d s u b m u c o s a l fibroelastic t r a n s f o r m a t i o n • Spiritis (alcohol)
l e a d i n g t o t r i s m u s a n d d i f f i c u l t y in p r o t r u d i n g t h e t o n g u e . • Sharp jagged tooth and ill-fitting dentures
• It is a p r e m a l i g n a n t c o n d i t i o n . • Sepsis
• Leukoplakia a n d s q u a m o u s cell c a r c i n o m a m a y be associated • Syndrome of Plummer-vinson (iron deficiency anemia)
• Syphilitic glossitis
w i t h it ( m a l i g n a n t t r a n s f o r m a t i o n = 3 t o 7 . 6 % cases).

• Premalignant conditions
Clinical Features
Leukoplakia (most c o m m o n )
Most common in ages b e t w e e n 20 a n d 4 0 years. Erythroplakia ( m a x i m u m risk)
Intolerance t o spicy f o o d . Chronic hyperplastic candidiasis

Soreness o f m o u t h w i t h c o n s t a n t b u r n i n g sensation. • C o n d i t i o n s i n c r e a s i n g risk


Redness a n d repeated vesicular e r u p t i o n s o n palate a n d pillars. Oral s u b m u c o s a fibrosis
Syphilitic glossitis
D i f f i c u l t y in o p e n i n g m o u t h f u l l y a n d p r o t r u d i n g t h e t o n g u e .
Sideropenic dysphagia
• Risk is d o u b t f u l
Blanching of mucosa over soft palate, facial pillars and buccal mucosa Oral lichen planus
(the three most common sites for submucous fibrosis). Discoid lupus e r y t h e m a t o s u s
Dyskeratosis c o n g e n i t a .

Treatment Investigation

• Incisional b i o p s y is r e c o m m e n d e d in all cases.


Medical
• Fine-needle aspiration c y t o l o g y (FNAC) is d o n e f o r l u m p in neck
• A v o i d irritant factors. especially suspicious l y m p h nodes.
Treat a n e m i a a n d v i t a m i n deficiencies. • M a g n e t i c r e s o n a n c e i m a g i n g (MRI), w h e n a v a i l a b l e , is
Topical i n j e c t i o n o f steroids c o m b i n e d w i t h hyalase. i n v e s t i g a t i o n o f choice.

SECTION II Oral Cavity

Staging Carcinoma Tongue

Irrespective o f site same s t a g i n g is r e c o m m e n d e d f o r all oral cavity T1 a n d T 2 : T r a n s o r a l partial glossectomy.


tumor. T 3 andT4:Transmandibular/Transcervical t o t a l glossectomy.
T stage CSDT11/e,p286 In case o f l y m p h n o d e (LN) i n v o l v e m e n t —> Radical neck dissection
at t h e t i m e o f glossectomy.
T1 Tumor 2 cm or less in greatest dimension

12 Tumor more than 2 cm but not more than 4 cm in Carcinoma Mandible


greatest dimension
• For freely m o b i l e t u m o r — w i d e local excision
T3 Tumor more than 4 cm in greatest dimension • F o r t u m o r a d h e r e d t o lingual aspect—marginalmandibulectomy

T4 Tumor invades adjacent structures • For t u m o r adhered t o g i n g i v a — m a r g i n a l mandibulectomy


• For t u m o r i n v o l v i n g alveolar margin—segmental
N stage
mandibulectomy
NX Regional lymph nodes cannot be assessed

NO No regional lymph node metastasis | PENTIGEROUS (FOLLICULAR CYST)

N1 Metastasis in a single ipsilateral clinically positive node,


• D e n t i g e r o u s is a cyst w h i c h envelops t h e w h o l e or p a r t o f t h e
3 cm or less in greatest dimension
crown o f t h e uninterrupted permanent tooth.
N2 Metastasis in
• Seen in: 3 rd
- 4 t h
decade.
N2a Single ipsilateral lymph node more than 3 cm but • M o s t c o m m o n site: m a n d i b u l a r 3rd m o l a r t o o t h
not more than 6 cm in greatest dimension
• M o s t c o m m o n t y p e : Central t y p e , i.e. t h e cyst s u r r o u n d s t h e
N2b Multiple ipsilateral lymph nodes, none more than
crown o f t h e tooth
6 cm in greatest dimension
• Cyst l i n i n g : Non-keratinzing stratified s q u a m o u s e p i t h e l i u m .
N2c Bilateral or contralateral lymph nodes, none more
The f l u i d inside is cholesterol rich
than 6 cm in greatest dimension
N3 Metastasis in a lymph node more than 6 cm in • R a d i o g r a p h y : Well-defined radiolucency associated w i t h t h e
greatest dimension crown o f t h e interrupted t o o t h
M Stage • T r e a t m e n t : Enucleation w i t h t h e r e m o v a l o f t h e associated
tooth
MX Presence of distant metastasis cannot be assessed

MO No distant metastasis
| DENTAL CYST
Ml Distant metastasis
• Dental cyst (radicular cyst, p e r i o d o n t a l cyst) are i n f l a m m a t o r y
M o s t c o m m o n Site for
cysts w h i c h occur as a result o f p u l p d e a t h especially in t h e
Carcinoma Most c o m m o n site permanent tooth.
Lip carcinoma Vermilion of lower lip It is t h e m o s t c o m m o n cystic lesion in t h e j a w
Peak incidence: - 4 t h decade
Tongue carcinoma Lateral border
6 0 % f o u n d in t h e maxilla
Cheek carcinoma Angle of mouth
Egg-shell crackling: May be elicitable d u e t o cortical t h i n n i n g
Larynx carcinoma Glottis C o n t e n t : Straw-colored f l u i d , rich in cholesterol
Nasopharynx carcinoma Fossa of rosenmuller R a d i o g r a p h : The cysts are r o u n d / o v o i d radiolucencies w i t h
Ranula Floor of mouth beneath the tongue sclerotic m a r g i n

Epulis Root of teeth


| ODONTOGENIC KERATOCYST
*
Treatment
(Ref. Current Otolaryngology 3rd/ed p 380 onward • Arise f r o m t h e r e m n a n t o f d e n t a l lamina
S q u a m o u s cell cancers o f oral cavity are primarily treated surgically, • Have a t e n d e n c y t o recur f o l l o w i n g surgical excision
w h i l e t h o s e o f o r o p h a r y n x are p r i m a r i l y t r e a t e d w i t h radiotherapy. • Can arise a n y w h e r e in t h e m a n d i b l e or maxilla
• Has a p o t e n t i a l t o b e c o m e m a l i g n a n t
Carcinoma Lip (Most common cancer of oral cavity)

T1andT2: Surgery is TOC: Flap r e c o n s t r u c t i o n ( A b b e , Treatment


Estlander's f l a p ) is d o n e i f
Excision
m o r e t h a n 1/3rd is i n v o l v e d .
T3andT4: Combined radiation a n d • Radical if m a l i g n a n c y is suspected.
surgery/vermilionectomy or • Enucleation w i t h mechanical, c u r e t t a g e using m e t h y l e n e b l u e
lip shave.
f o l l o w e d b y a p p l i c a t i o n o f carnoy's s o l u t i o n .
CHAPTER7 Oral Cavity j71

Malignancy varies inversely with t h e size of gland ( 9 0 % o f m i n o r


I S I A L O L I T H I A S I S ( S T O N E IN S A L I V A R Y G L A N D )
salivary g l a n d t u m o r s are m a l i g n a n t ) .
• 8 0 - 9 0 % o f calculi d e v e l o p in w h a r t o n s d u c t o f s u b m a n d i b u l a r All salivary g l a n d t u m o r s are m o s t l y present in p a r o t i d g l a n d
gland. Stensons d u c t o f parotid constitutes 1 0 - 2 0 % and except a d e n o i d cystic carcinoma w h i c h is seen m o s t c o m m o n l y
sublingual only 1 % . in m i n o r salivary glands a n d s q u a m o u s cell carcinoma w h i c h
• 8 0 % s u b m a n d i b u l a r s t o n e s are r a d i o p a q u e w h i l e p a r o t i d is seen m o s t c o m m o n l y s u b m a n d i b u l a r g l a n d .
stones are radiolucent. Most common Benign tumor/overall tumor of salivary
• T r e a t m e n t : It d e p e n d s o n site: glands in children is h e m a n g i o m a ~~\Scott'-Brown's 7th/ed
If stone is l y i n g w i t h i n t h e s u b m a n d i b u l a r d u c t ; anterior t o Most common m a l i g n a n t salivary g l a n d
t h e crossing o f lingual nerve, stone can be r e m o v e d by l o n - t u m o r in children - M u c o e p i d e r m o i d J vol 1, p 1248
g i t u d i n a l incison over t h e d u c t . D u c t s h o u l d be left o p e n . 2 n d most common m a l i g n a n t t u m o u r in c h i l d r e n — A c i n i c cell
If stone is distal t o lingual nerve, it s h o u l d be t r e a t e d w i t h cancer.
s i m u l t a n e o u s excision o f s u b m a n d i b u l a r g l a n d . For m o s t t u m o r types t h e r e is a s l i g h t f e m a l e p r e p o n d e r a n c e
• Parotid stones are r e m o v e d surgically by e x p o s i n g t h e d u c t M o s t c o m m o n e t i o l o g i c a l a g e n t f o r salivary g l a n d t u m o r is
a n d stone is released. exposure t o radiation
M o s t salivary g l a n d t u m o r s are insidious in onset a n d g r o w
| S A L I V A R Y G L A N D T U M O R S ( T A B L E 7.1) _ _ slowly. Pain is e x t r e m e l y u n c o m m o n
Most helpful i m a g i n g t e c h n i q u e f o r salivary g l a n d t u m o r
• Major salivary g l a n d t u m o r are m o s t l y b e n i g n . are c o n t r a s t e n h a n c e d c o m p u t e d t o m o g r a p h y (CT) and
• M i n o r salivary g l a n d t u m o r are m o s t l y m a l i g n a n t . G a d o l i n i u m MRI (is preferred)
• In c h i l d r e n > 5 0 % salivary g l a n d t u m o r s are m a l i g n a n t . Open surgical biopsy is contraindicated in salivary g l a n d
• Most common t u m o r o f major salivary glands/most c o m m o n t u m o r s as it seeds t h e t u m o r t o t h e s u r r o u d i n g tissue.
b e n i g n salivary g l a n d t u m o r — p l e o m o r p h i c a d e n o m a . Investigation of choice f o r salivary g l a n d swellings - FNAC. as
• Most common m a l i g n a n t t u m o r o f m a j o r salivary g l a n d s - MRI c a n n o t distinguish b e t w e e n b e n i g n a n d m a l i g n a n t lesions
M u c o e p i d e r m o i d carcinoma. Treatment is exicision n o t enucleation as t u m o r has microscopic
• Most common m a l i g n a n t t u m o r o f m i n o r salivary g a l n d s - extensions o u t s i d e t h e capsule.
A d e n o i d cystic carcinoma. M a j o r i t y o f salivary g l a n d t u m o r s are radioresistant.
• Most common site o f m i n o r salivary glands t u m o r - H a r d palate.

T a b l e 7.1: S u m m a r y o f salivary g l a n d t u m o r

Tumor type Most c o m m o n site Important feature Management

Pleomorphic Adenoma Parotid gland tail M/C benign salivary gland t u m o r " Superficial parotidectomy
(Mixed Tumor) (superficial lobe) (Patey's operation)

M/C tumor of major salivary g l a n d "

Affects women around 40 years 0

In pleomorphic adenoma of sub-mandibular gland


m/c age affected is 60 yrs 0

8 0 % of parotid pleomorphic adenomas arise in


superficial l o b e 0

Encapsulated but sends pseudopods into


surrounding glands (so enucleation is not done as
treatment)

Malignant transformation occurs in 3-5% of cases

Facial nerve infiltration indicates carcinomatous


change

Warthin's tumor/ Parotid gland It is the second M/C benign tumor of salivary Superficial parotidectomy
Adenolymphoma exclusively (M/c site glands
being lower part of Can also arise from cervical nodes
parotid overlying Smoking its risk
angle of mandible) It never involves facial nerve
It shows hot spot in 99Tcm scan which is diagnostic

Contd...
SECTION II Oral Cavity

Contd...

Tumor type Most c o m m o n site Important feature Management

• It is the only salivary gland tumor which is more


common in men

Adenoid cystic Minor salivary gland • M/C cancer of minor salivary gland followed by Radical parotidectomy
Minor salivary gland adenocarcinoma and mucoepidermoid carcinoma followed by postoperative
carcinoma (Cylindroma) • Invades perineural space and lymphatics radiotherapy if margins are
• M/C head and neck cancer associated with positive
perineural invasion
• Unlike other salivary gland tumors it is more
radiosenstive

Mucoepidermoid Parotid gland • M/C malignant salivary gland tumor in children Superficial/Total
carcinoma • M/C malignant tumour of parotid parotidectomy + radical neck
• M/C radiation induced neoplasm of salivary gland dissection
carcinoma
• Consists of mixture of squamous cells, mucous-
secreting cells, intermediate cells and clear or
hydropic cells
• Mucin producing tumor is low-grade type;
squamous cell T/m is high grade type

Acinic cell adeno carcinoma Exclusively parotid • Rare tumor with low-grade malignancy Treatment is radical excision
gland affecting •- Tends to involve the regional lymph nodes Only tumor which responds to
w o m e n mostly radiotherapy so, irradiation

Squamous cell carcinoma Submandibular gland • Arises from squamous metaplasia of the lining
therapy is useful epithelium o f t h e ducts

CERVICAL SWELLING • Branchial fistulas are t h o s e d e r i v e d f r o m 2 n d branchial cleft


a n d o p e n externally in t h e lower t h i r d o f neck, near t h e anterior
M i d l i n e s w e l l i n g o f neck ( f r o m a b o v e d o w n w a r d ) is k/a cervical b o r d e r o f s t e r n o c l e i d o m a s t o i d . Its internal orifice is located in
swelling t h e tonsillar fossa.

9 nemonic
Lymph Ludwigs angina
Features

• Cysts a n d sinuses are lined by stratified s q u a m o u s e p i t h e l i u m .


Node Enlarged submental lymph nodes
• C o n t e n t : Straw-colored f l u i d rich in cholesterol.
Sublingual Sublingual dermoid
• B r a n c h i a l cysts: Present in t h e t h i r d decade.
Likes Lipoma
The Thyroglossal cyst • B r a n c h i a l s i n u s : Present since b i r t h .
Sweet Subhyoid bursitis • M a l e : Female = 3:2.
Girl Goiter • 6 0 % o f t h e m are present o n left side.
Living (in) Lipoma
Sites of o c c u r r e n c e of t h e c y s t :
Retro Retrosternal goiter
U p p e r neck (most common)
Thymus Thymic swelling
Lower neck
Parotid g l a n d
nemonic Pharynx and p o s t e r i o r t r i a n g l e
(Though a little weird but is very helpful) Lymph Nodes Sublingual
gual
Likes The Sweet Girl Living (in) Retro Thymus. Treatment

Excision o f t h e cyst a n d fistula.


BRACHIAL CYST AND BRACHIAL FISTULA

Remnants o f t h e brachial apparatus, present in fetal life | THYROGLOSSAL CYST


Branchial cysts are characteristically f o u n d a n t e r i o r a n d d e e p It is a cystic swelling w h i c h arises f r o m t h e r e m n a n t o f thyroglossal
t o t h e u p p e r t h i r d o f t h e s t e r n o c l e i d o m a s t o i d muscle. duct.
CHAPTER 7 Oral Cavity

D e v e l o p m e n t of T h y r o g l o s s a l Cyst Differences b e t w e e n Thyroglossal Cyst and


T h y r o g l o s s a l Fistula
• Thyroglossal t r a c t passes d o w n f r o m f o r a m e n c e c u m o f t h e
t o n g u e b e t w e e n genioglossi muscle in f r o n t , passing b e h i n d Thyroglossal cyst Thyroglossal fistula
t h e hyoid bone t o the upper border of thyroid cartilage
Congenital Never congenital, always acquired
u l t i m a t e l y e n d i n g in t h e p y r a m i d a l l o b e o f t h y r o i d g l a n d .
Present anywhere along f o l l o w i n g i n f e c t i o n / inadequate
• N o r m a l l y this t r a c t disappears by t h e 5 t h - 10th w e e k except thyroglossal tract cyst removal
in t h e lower p a r t f o r m i n g isthmus o f t h y r o i d .
Most common site subhyoid Median fistula of neck
• S o m e t i m e s , a p a r t o f it m a y r e m a i n p a t e n t g i v i n g rise t o a
Moves upward on protrusion Moves upward on protrusion of
cystic s w e l l i n g d u e t o r e t e n t i o n o f secretions r e s u l t i n g in
o f t o n g u e as w e l l as o n tongue
thyroglossal cyst.
swallowing

| NECK DISSECTIONS

Types
Thyroglossal Thyroid
- Hyoid bone
Radical Neck Dissection (RND)
duct cartilage
• S t r u c t u r e s r e m o v e d : en bloc r e m o v a l o f t h e l y m p h nodes
Thyroid- Anterior a n d l y m p h bearing areas f r o m t h e m a n d i b l e (above) t o clavicle
• Thyroglossal
(below) a n d f r o m m i d l i n e t o t h e anterior b o r d e r t o trapezius.
duct
• A d d i t i o n a l structures r e m o v e d :

Fig. 7.1: Sites o f thyroglossal d u c t cyst S u b m a n d i b u l a r gland/tail o f p a r o t i d

• Epithelial lining: Pseudostratifiedciliated/columnarsquamous Internal j u g u l a r vein

• I m p o r t a n c e : S q u a m o u s carcinoma may arises in t h e cyst. Sternocleidomastoid, o m o h y o i d


Spinal accessory nerve a n d cervical plexus
Clinical Features
Modified Radical and Seletive Neck Dissection
• Age: Althoughcongenitalcanbeseenatanyagefrombirth
u p t o 70 years. (Mostly present b e t w e e n 15 a n d Modified Radical neck Selective neck dissection
30 years). dissection

• Position: M i d l i n e in 9 0 % cases. Modification of RND where one Modification o f RND that


• In 1 0 % cases, it occurs an o n e side in w h i c h 9 5 % or more non-lymphatic structures preserves one or more lymphatic
are o n left side a n d 5 % o n r i g h t side. are preserved viz. compartment normally removed
as a part of RND.
• Clinically: Swelling moves sideways only. On p r o t r u d i n g t h e
t o n g u e or o n d e g l u t i t i o n — i t moves u p w a r d . • J - Internal Jugular vein Types:
• A - Spinal accessory nerve • Supraomohyoid: removes LN
Treatment in level I to III (indication: oral
cavity primaries)
S i s t r u n k ' s o p e r a t i o n ( s t e p l a d d e r s u r g e r y ) in w h i c h t r a c t is
Sterno cleidomastoid • Lateral neck dissection:
c o m p l e t e l y excised a l o n g w i t h m i d d l e o f h y o i d b o n e .
muscleJugular lymphatic Removes LN level II to IV
chain and cervical (indication: laryngeal Ca)
NOTE
lymphatics are removed
If body of hyoid is not removed recurrence occurs in 8 5 % cases. Indication: Single node • Posterolateral neck dissection:
Recurrence after removal of hyoid = 2-8% < 3 cm removes LN in level II to IV
In cases of infected thyroglossal cyst: abscess should be incised (indication: thyroid Ca)
and drained.
After complete subsidence of inflammatory reaction Functional Neck Dissection
(approximately 6 weeks) thyroglossal cyst and its epithelial tract Preservation o f all t h r e e n o n - l y m p h a t i c structures viz. i n t e r n a l
should be excised j u g u l a r spinal accessory a n d sterno-cleidomastoid.
Carcinoma arising in the thyroglossal cyst are:
- Papillary adenocarcinoma (85%)
Extended Radical Neck Dissection
- Follicular adenocarcinoma (15%) Removal o f a d d i t i o n a l l y m p h n o d e g r o u p s (paratracheal, superior,

- Adenocarcinoma mediastinal parapharyngeal) and n o n l y m p h a t i c structures (external


carotid a r t e r y XII nerve, X nerve paraspinal muscles a n d p a r o t i d
- Squamous carcinoma
glands).
SECTION II Oral Cavity

NOTE Level 4 Nodes along the lower third of IJV between cricoid
cartilage and clavicle.
L e v e l s of l y m p h n o d e s in n e c k
Level 5These nodes lie in posterior triangle of neck including
• Level 7 Includes submental and submandibular lymph nodes.
transverse cervical and supraclavicular nodes.
• Leve/2Nodes lie along the upper one-third of IJV between base of teve/6Theseare nodes in anterior compartment including
skull and hyoid bone prelarygeal, pretracheal and paratracheal groups.
• Level3 Nodes along the middle third of IJV between hyoid bone Level 7lncludes nodes of upper mediastinum below suprasternal
and upper border of cricoid cartilage. notch.

| FRACTURE OF THE NOSE

It is t h e most common facial b o n e t o g e t f r a c t u r e d . 0

C l a s s i f i c a t i o n N a s a l o f F r a c t u r e ( T a b l e 7.2)
T a b l e 7.2: Classification o f nasal fracture

Class 1 fracture Class 2 fracture Class 3 fracture

Chevallet Jarjavay

• Depressed nasal fracture • Involve the nasal bone, the frontal process • Caused by high velocity trauma
• Fracture line runs parallel to the dorsum and o f t h e maxilla and the septal structures • Naso orbit ethmoidal fracture
the nasomadilary suture line • Ethmoidal labyrinth and the orbit are spared • Ethmoidal labyrinth is involved
• Nasal septum is not involved generally in • Here, the quardrilateral cartilage gets • Presents with multiple fracturesof the roof of
this injury dislocated from the maxillary crest ethmoid, orbit and sometimes extends as far
• It is involved only in severe cases • Treatment: Closed reduction of the nasal back as the sphenoid and parasellar regions
• Features: Does not cause gross lateral bone fracture with open reduction o f t h e (CSF leak and pneumocranium seen)
• Treatment: Fracture reduction done either septum • Treatment: Open reduction and displace-
immediately or after 5-7 days, once edema ment internal fixation
settles

NOTE S y m p t o m s of Nasal Fracture

Distal part o f t h e nasal bone is very thin and therefore more • M o s t c o m m o n s y m p t o m : epistaxis
susceptible to injury. • External nasal d e f o r m i t y
Untreated nasal bone fractures lasting for more than 21 days • Nasal o b s t r u c t i o n d u e t o b l o o d c l o t
require open reduction • Palpation:
Any cerebrospinal fluid (CSF) leak persisting for more than 2 weeks Tenderness present
have to be considered for repair.
Crepts present
Foreceps used in:
• Watery nasal discharge indicates CSF leak d u e t o fracture o f
- Reduction of nasal bone - Walsham forcep
c r i b r i f o r m plate in r o o f o f nose.
- Reduction of septal facture - Asch forcep

| FRACTURE OF MAXILLA

Le f o r t classified fracture o f maxilla i n t o t h r e e types (Table 7.3)


T a b l e 7.3: Classification o f le f o r t t y p e fracture

Le Fort type 1 fractures Type 2 (Pyramidal fracture) Type 3 (Craniofacial dysostosis)

• Type 1 (transverse Guerin fracture) separates This fracture involves the pterygoid plates, Facial skeleton separates from the cranial base
the palate from midface and by definition fronto nasal maxillary buttress and often the Fracture line passes from Root of nose
involve the pterygoid plates bilaterally skull base via the ethmoid bone

(a) Le forte 1 (Guerin) (b)Le forte 2 (Pyramidal) (c) Le forte 3 (Craniofacial dysjunction)

Le Fort f r a c t u r e s

Contd.
CHAPTER 7 Oral Cavity
J75
Contd.

Le Fort t y p e 1 fractures Type 2 (Pyramidal fracture) Type 3 (Craniofacial dysostosis)


Fracture line passes through the floor o f t h e • Fracture line passes from floor o f t h e maxilla I
maxilla on both sides I Ethmoid frontal junction
I Through zygomatic maxillary suture line I
Above the nasal cavity floor Superior orbital tissue
# i
... I•
;
I
Floor of the orbit Lateral wall of orbit
And through the nasal septum
I I i
Lacrimal bone Zygomaticotemporal suture
Inferior parts o f t h e medial and lateral
4- I
pterygoids
Nasion Temperozygomatic suture
I
Upper part of pterygoid
• This type of fracture results in a mobile • Infraorbital nerve damaged
palate but a stable upper midface

• Orbital floor is always inclined

• This fracture has a pyramidal appearance


and results in palatal and midface mobility

It is also called as'floating maxilla fracture

Le fort fracture ll/lll are associated with CSF rhinorrhea

| ZYGOMATIC FRACTURE (TRIPOD FRACTURE)

Z y g o m a t i c Fracture is t h e second M/C facial fracture (after nasal


bone).
• C o m m o n l y c a l l e d Tripod F r a c t u r e S i n c e t h e B o n e B r e a k s
at three Places
• Z y g o m a t i c o f r o n t a l or F r o n t o z y g o m a t i c suture
• Infraorbital r i m

• Z y g o m a t i c o t e m p o r a l suture (Fig. 7.2)

Features
• Ecchymosis o f p e r i o r b i t a l r e g i o n w i t h i n 2 h o u r s o f i n j u r y is
pathognomic
• S t e p — d e f o r m i t y at t h e infraorbital m a r g i n
• Flattening o f t h e malar p r o m i n e n c e
• Anesthesia in t h e d i s t r i b u t i o n o f t h e i n f r a o r b i t a l nerve
• Trismus Fig. 7.2: Left z y g o m a ( t r i p o d ) f r a c t u r e s h o w i n g t h r e e sites o f

• Restricted ocular m o v e m e n t f r a c t u r e . (1) Z y g o m a t i c o f r o n t a l ; (2) Z y g o m a t i c o t e m p o r a l ; (3)

• Periorbital e m p h y s e m a Infraorbital
Coutesy: Textbook of Diseases of Ear, Nose and
• Diplopia
Throat, Mohan Bansal. Jaypee Brothers, p 344
Diagnosis
| CEREBROSPINAL FLUID RHINORRHEA
• Water's v i e w a n d exaggerated water's v i e w X-ray
• CT scan (orbit) (Scoff Brown 7th/ed, vol2p 1636-1639)
• It is t h e f l o w o f CSF f r o m nose (due t o leakage o f CSF f r o m t h e
Treatment
•subarachnoid space i n t o nasal cavity).
O n l y displaced fractures are t o be t r e a t e d • Usual sites o f CSF leak are c r i b r i f o r m p l a t e > f r o n t a l sinus
O p e n r e d u c t i o n a n d internal w i r e f i x a t i o n is carried o u t . (posterior walls) > f l o o r o f t h e anterior cranial fossa.
SECTION II Oral Cavity

Etiology

T r a u m a t i c (Acute/delayed) Atraumatic

Accidental . Iatrogenic Due t o raised ICT Normal pressure leaks


I • Headlight Intranasal surgery like Tumors • Congenital dehiscence o f
In Le fort II and polypectomy Hydrocephalus nasal roof
Lefort III Fracture • Endoscopic sinus surgery Destructive bony lesions like • Focal atrophy
craniotomy granuloma • Osteomyelitic erosion
• Transphenoidal hypophysectomy

NOTE b e d , stool softeners, a n d a v o i d a n c e o f nose b l o w i n g , s n e e z i n g


a n d s t r a i n i n g . Prophylactic a n t i b i o t i c s can b e used t o p r e v e n t
Historically the M/C cause of CSF rhinorrhea was head injury with
m e n i n g i t i s . A c e t a z o l a m i d e d e c r e a s e s CSF f o r m a t i o n . T h e s e
involvent of cribriform plate of e t h m o i d but now M/C cause is
measures can be c o m b i n e d w i t h l u m b a r drain if i n d i c a t e d .
Iatrogenic trauma surgery.
Surgical repair can be d o n e by t h e f o l l o w i n g :
CSF can escape from following routes:
• N e u r o s u r g i c a l intracranial a p p r o a c h .
• Middle/posterior fossa via mastoid cavity, sphenoid sinus
• E x t r a d u r a l a p p r o a c h e s such as external e t h m o i d e c t o m y for
• Anterior cranial fossa via:
c r i b r i f o r m p l a t e a n d e t h m o i d area, trans-septal s p h e n o i d a l
- Frontal, Ethmoid Sphenoid sinus
a p p r o a c h f o r s p h e n o i d a n d o s t e o p l a s t i c flap a p p r o a c h f o r
- Cribriform plate
f r o n t a l sinus leak.
- From middle ear via eustachian tube • Transnasal endoscopic: With the advent o f endoscopic
s u r g e r y f o r nose a n d sinuses, m o s t o f t h e leaks f r o m t h e
Clinical Features a n t e r i o r cranial fossa a n d s p h e n o i d sinus can b e m a n a g e d
endoscopically w i t h a success rate o f 9 0 % w i t h first a t t e m p t .
Unilateral, clear w a t e r y discharge d r i p p i n g o n l o o k i n g d o w n , w h i c h
Principles o f repair i n c l u d e :
increases o n c o u g h i n g , sneezing or e x e r t i o n .
D e f i n i n g t h e sites o f b o n y defect.
Diagnosis
Preparation o f g r a f t site.
On Examination Underlay g r a f t i n g o f t h e fascia extradurally f o l l o w e d b y
p l a c e m e n t o f mucosa (as a free graft or p e d i c l e d flap).
• R e s e r v o i r s i g n : To elicit CSF r h i n o r r h e a
If b o n y d e f e c t is larger t h a n 2 c m , i t is r e p a i r e d w i t h
• A f t e r b e i n g s u p i n e —> t h e p a t i e n t is m a d e t o sit u p in t h e
cartilage (from nasal s e p t u m or auricular concha) f o l l o w e d
u p r i g h t p o s i t i o n w i t h t h e neck f l e x e d . If t h e r e is s u d d e n rush
by p l a c e m e n t o f mucosa.
o f clear f l u i d , it indicates CSF r h i n o r r h e a .
• H a n d k e r c h i e f test: s t i f f e n i n g o f t h e h a n d k e r c h i e f occurs w i t h NOTE
rhinitis (due to presence of mucus) b u t n o t in CSF r h i n o r r h e a . CSF leak from frontal sinus often requires osteoplastic flap operation
• D o u b l e r i n g or h a l o sign is seen in b l o o d stained CSF fluid. and obliteration o f t h e sinus with fat.
• Nasal endoscopy: w i t h / w i t h o u t fluorescein—can help in
diagnosis | BLOW OUT FRACTURE OF ORBIT

Biochemical Examination
• B l u n t t r a u m a t o t h e o r b i t leads t o increase i n i n t r a o r b i t a l
• G l u c o s e a n d c h l o r i d e c o n c e n t r a t i o n : Glucose level o f > 30 p r e s s u r e a n d so o r b i t g i v e s w a y t h r o u g h t h e f l o o r a n d
m g % is c o n f i r m a t o r y for CSF. m e d i a l w a l l . There is h e r n i a t i o n o f t h e o r b i t a l c o n t e n t s i n t o
• P t r a n s f e r r i n o n e l e c t r o p h o r e s i s : Presence o f (3 transferrin
2 2 t h e maxillary a n t u m . This is k n o w n as o r b i t a l b l o w o u t . This
is p a t h o g n o m i c for CSF r h i n o r r h e a . This is t h e o n l y test w h i c h h e r n i a t i o n o f o r b i t a l c o n t e n t s i n t o t h e m a x i l l a r y a n t r u m is
s h o u l d b e used t o c o n f i r m CSF'rhinorrhea. Besides CSF, P 2 visualized radiologically as a convex o p a c i t y b u l g i n g i n t o t h e
transferrin is present in p e r i l y m p h a n d aqueous h u m o r . a n t r u m f r o m above. This is k n o w n as t e a r d r o p sign.
• A n o t h e r p r o t e i n called t h e b e t a trace protein is also specific • The s y m p t o m s i n c l u d e e n o p h t h a l m o s , d i p l o p i a , r e s t r i c t e d
f o r CSF a n d is w i d e l y used in Europe. It is secreted by meninges u p w a r d gaze and i n f r a o r b i t a l anesthesia.
a n d c h o r o i d plexus. Facilities t o test these proteins are n o t easily • Forced deduction test: Detects extraocular muscle e n t r a p m e n t
available e v e r y w h e r e . in b l o w o u t fractures.
• I m a g i n g m o d a l i t y of choice: To diagnose t h e site o f l e a k — T 2
w e i g h t e d MRI. | FRACTURE OF MANDIBLE

Treatment Fracture o f m a n d i b l e is classified b y D i n g m a n s c l a s s i f i c a t i o n


d e p e n d i n g o n location. Condylar fractures ( 3 5 % ) are m o s t c o m m o n
Early cases o f post-traumatic CSF r h i n o r r h e a can be m a n a g e d b y
f o l l o w e d by t h o s e o f angle ( 2 0 % ) , b o d y ( 2 0 % ) a n d s y m p h y s i s ( 1 5 % )
conservative measures such as b e d rest, e l e v a t i n g t h e head o f t h e
of mandible.
CHAPTER 7 Oral Cavity

CLINICAL VIGNNETTES TO REMEMBER 6. To delineate t h e area f r o m w h i c h b i o p s y s h o u l d be t a k e n


in oral leisons-supravital staining w i t h t o l u i d i n e b l u e dye is
1. Vestibule is seen in ear (in inner ear b o n y l a b y r i n t h ) , nose
used.
(skin lined p o r t i o n o f nose), larynx (part a b o v e ventricular
7. A 40-year-old chronic cigratte smoker presents w i t h reddish
bands) a n d oral cavity.
shiny plaques in t h e f l o o r o f m o u t h . M o s t c o m m o n D/D
2. L N o f t o n g u e is j u g u l o m y l o h y o i d LN (as f r o m all parts o f
is-Erythroplakia.
t o n g u e , l y m p h a t i c s finally drain i n t o j u g u l o m o h y o i d LN).
8. A 42-year-old male w h o is a sale's m a n a g e r in a l e a d i n g firm
3. M/C l y m p h n o d e enlarged in t o n g u e m a l i g n a n c y = S u b m a n -
presents w i t h grayish atropic area in t h e l o w e r lip d u e t o
d i b u l a r LN (as M/C site for Ca t o n g u e = lateral aspect w h i c h
l o n g s t a n d i n g s u n l i g h t exposure. The m o s t i m p o r t a n t D/D
drains i n t o s u b m a n d i b u l a r LN)
is actinic chelosis. (Note: Actinic chelosis is c o m m o n in males
4. In XII nerve paralysis deviates t o paralyzed side o n p r o t u s i o n
> 4 0 years a n d can lead t o s q u a m o u s cell carcinoma).
d u e t o action o f unaffected genioglossus muscle o n o p p o s i t e
9. M/C site f o r m a n d i b u l a r fracture = c o n d y l a r fracture.
side.
10. Pneumocephalus can be seen in fracture o f f r o n t a l sinus.
5. For lip r e c o n s t r u c t i o n Abbe-Estlander flap (Fig. 7.4) is used
11. 1 st/Most i m p o r t a n t step in m a n a g e m e n t o f f a c i o m a x i l l a r y
w h i c h is based o n labial artery. O t h e r flaps w h i c h c o u l d be
t r a u m a - Airway m a n a g e m e n t
used are Karapandzic flap, Gillie's fan flap. 12. Palatal m y o c l o n u s is seen in m u l t i p l e sclerosis.
13. A14-years-old boy presents w i t h fever, sore-throat ulcers and
cervical l y m p h n o d e enlargement.Throat-swab is positive for
beta h e m o l y t i c streptococcus a n d was p u t o n penicillin b u t
he d e v e l o p e d r u b e l l i f o r m rash a n d s y m p t o m s w o r s e n e d -
Diagnosis is - Infectious m o n o n u c l e o s i s (also k/a g l a n d u l a r
fever). Caused by EBV. G o l d standard test f o r d i a g n o s i n g this
c o n d i t i o n - EBV a n t i b o d i e s . M a n a g e m e n t - s t e r o i d s .
14. In a case o f recurrent e d e m a o f uvula a n d laryngeal e d e m a
- always suspect hereditary a n g i o n e u r o t i c e d e m a (HANE).
Paitents m a y also have e d e m a o f g u t . It is caused d u e t o
deficiency o f e n z y m e C1 esterase inhibitor.
15. Behcet's syndrome - is oculo o r o genital syndrome character-
ized by a t r i a d o f —
• A p t h o u s like ulcers in oral cavity. The e d g e o f t h e ulcer is
characteristically p u n c h e d o u t .
• Genital ulceration
• Uveitis
16. Taste buds are h i g h e s t in circumvallate papillae > Foliate
papillae > F u n g i f o r m papillae.
F i g . 7.4: A b b e estander flap
T h e r e are p r a c t i c a l l y n o b u d s i n f e l l i f o r m p a p i l l a e .

Clinical C o n d i t i o n Seen i n
• Black membrane in mouth Vincent argina

• Grayish white membrane on tonsils + B/L cervical lymphadenitis in a febrile patient Diphtheria
• Cystic translucent swelling in the floor of mouth Ranula
• Opaque swelling in midline in the floor of mouth Dermoid cyst

• Black hairy tongue Chronics smokers, Drugs like lasanopra zole, antibiotic use.

• Fissured tongue Syphilis, Vit B deficiency, Anemia

• Wickham's striae Lichen planus


78 I SECTION II Oral Cavity

QUESTIONS

Fordyce's (Spots) G r a n u l e s in oral cavity a r i s e f r o m : c. External beam radiotherapy


[AIIMS 04] d. Chemotherapy
a. Mucous glands b. Sebaceous glands 12. A patient w i t h C a t o n g u e is f o u n d to h a v e l y m p h n o d e s
c. Taste buds d. Minor salivary glands in t h e lower n e c k . T h e t r e a t m e n t of c h o i c e for t h e l y m p h
True a b o u t a p h t h o u s ulcer: [PGI June 05] n o d e s is: [AIIMS 01]
a. Viral predisposition a. Lower cervical neck dissection
Recurrent ulcer b. Suprahyoid neck dissection
Deep ulcers c. Teleradiotherapy
Involves t h e mucosa o f t h e hard palate d. Radical neck dissection
Steroids given as treatment 1 3 . C a r c i n o m a of b u c c a l m u c o s a c o m m o n l y d r a i n to t h e
3. R e g a r d i n g r a n u l a all are t r u e e x c e p t : [MAHE05] f o l l o w i n g l y m p h n o d e s sites: [Al 97]
a. Retention cyst a. Submental b. Submandibular
b. Arises f r o m submandibular gland c. Supraclavicular d. Cervical
c. Translucent 14. M e t a s t a s i s of c a r c i n o m a buccal m u c o s a g o e s to:
d. Plunging may be a feature [AIIMS 96]
4. True r e g a r d i n g R a n u l a : [AI01] a. Regional l y m p h node b. Liver
a. It is also called as epulis c. Heart d. Brain
b. It is a cystic swelling in t h e floor o f m o u t h . 15. S q u a m o u s cell c a r c i n o m a of m a x i l l a w i t h T 3 NONO
c. It is a t y p e of thyroglossal cyst s t a g i n a t r e a t m e n t is:
d. It is a t y p e o f mucus retention cyst a. Radiotherapy
5a. P r e m a l i g n a n t leison of oral cavity i n c l u d e s : [PGI Nov 10] b. Maxillectomy
a. Erythroplasia b. Fordyce spots c. Radiotherapy and maxillectomy
c. Leukoplakia d . Keratoacanthoma d. Maxillectomy and c h e m o t h e r a p y
e. A p h t h o u s ulcer 16. A 70-year-old m a l e w h o has b e e n c h e w i n g tobacco for the
5 b . Risk f a c t o r s for o r o p h a r y n g e a l region c a r c i n o m a : past 50 y e a r s p r e s e n t w i t h a six m o n t h s history of large,
a. Sideropenic dysphagia b. Oral submucous fibrosis f u n g a t i n g , soft p a p i l l a r y lesions in t h e oral cavity. T h e
c. Erythroplakia d. Leukoplakia lesion h a s p e n e t r a t e d into the m a n d i b l e . L y m p h n o d e s
e. Chronic hypertrophic candidiasis a r e not p a l p a b l e . T w o b i o p s i e s t a k e n f r o m t h e l e s i o n
W h i c h of t h e following is p r e m a l i g n a n t c o n d i t i o n : proper show benign appearing papillomatosis with
[AIIMS 91] h y p e r k e r a t o s i s a n d a c a n t h o s i s infiltrating t h e s u b j a c e n t
a. Chronic glossitis b. Submucous fibrosis t i s s u e s . T h e m o s t likely d i a g n o s i s is: [A1041
c. Hypertrophic glossitis d. A p h t h o u s stomatitis a. Squamous cell papilloma b. Squamous cell carcinoma
T h e most c o m m o n p r e m a l i g n a n t condition of oral c. Verrucous carcinoma d. Malignant mixed t u m o r
c a r c i n o m a is: [Al 95,96] 17. A 80-year-old p a t i e n t p r e s e n t w i t h a m i d l i n e t u m o r
a. Leukoplakia b. Erythroplakia of t h e lower j a w , i n v o l v i n g t h e a l v e o l a r m a r g i n . He is
c. Lichen planus d. Fibrosis e d e n t u l o u s . T r e a t m e n t of c o k e : [Al 01]
T h e m o s t c o m m o n s i t e of o r a l c a n c e r a m o n g Indian a. H e m i m a n d i b u l e c t o m y
p o p u l a t i o n is: [Al 04] b. C o m m a n d o operation
a. Tongue b. Floor o f m o u t h c. Segmental m a n d i b u l e c t o m y
c. Alveobuccal complex d. Lip d. Marginal m a n d i b u l e c t o m y
C a r c i n o m a t o n g u e m o s t f r e q u e n t l y d e v e l o p s from:[AI 02] 18. A n old m a n w h o is e d e n t u l o u s s q u a m o u s cell c a r c i n o m a
a. Tip p. Lateral border in b u c c a l m u c o s a t h a t h a s d e v e l o p e d infiltrated to t h e
c. Dorsal p o r t i o n d. All portions equally a l v e o l u s . Following is not indicated in t r e a t m e n t :
1 0 . A patient h a s c a r c i n o m a of right t o n g u e o n its lateral a. Radiotherapy
b o r d e r of a n t e r i o r 2/3rd, w i t h l y m p h n o d e of size 4 c m b. Segment m a n d i b u l e c t o m y
in level 3 o n left s i d e of t h e n e c k , s t a g e of d i s e a s e is: c. Marginal m a n d i b u l e c t o m y involving removal o f outer table
[AIIMS May 07] only
a. NO b. N1 d. Marginal m a n d i b u l e c t o m y involving removal o f upper half
c. N2 d. N3 of mandible
11. A patient presented with a 1x1.5 cms growth on the 1 9 . W h i c h C a has best p r o g n o s i s : [AIIMS 98]
lateral b o r d e r of t h e t o n g u e . T h e t r e a t m e n t i n d i c a t e d a. Carcinoma lip
would be: [AIIMS 02] b. Carcinoma cheek
a. Laser ablation c. Carcinoma t o n g u e
b. Interstitial brachytherapy d. Carcinoma palate
CHAPTER 7 Oral Cavity

2 0 . True s t a t e m e n t a b o u t oral c a n c e r is/are: [PGI 04] 3 1 . Acinic cell c a r c i n o m a of t h e s a l i v a r y g l a n d a r i s e m o s t


a. Most c o m m o n in buccal mucosa o f t e n in t h e : [AI06]
b. Systemic metastasis u n c o m m o n a. Parotid salivary gland
c. Responds t o radiotherapy b. Minor salivry glands
d. Surgery is treatment o f choice c. Submandibular salivary glands
e. Syphilis a n d dental irridation predisposes d. Sublinguial salivary glands
21. In c a r c i n o m a of lower lip s e c o n d a r i e s are s e e n i n : [Al 91] 3 2 . A Warthin's t u m o r is: [AIIMS 03, 05]
a. Upper cervical LN b. Supraclavicular LN a. An a d e n o l y m p h o m a o f parotid gland
c. Axillary LN d. Mediastinal LN b. A pleomorphic adenoma o f t h e parotid
2 2 . Calculus is m o s t c o m m o n l y s e e n in w h i c h s a l i v a r y g l a n d : c. A carcinoma o f t h e parotid
[AIIMS June 99] d. A carcinoma o f submandibular salivary gland
a. Sublingual b. Palatal 33. All of t h e f o l l o w i n g are t r u e r e g a r d i n g Warthin's t u m o r
c. Parotid d . Submandibular except: [AIIMS 02]
2 3 . T h e m o s t c o m m o n t u m o r of t h e s a l i v a r y g l a n d is: a. More c o m m o n in females
[Al02; AIIMS 98] b. C o m m o n l y involve t h e parotid glands
a. M u c o e p i d e r m o i d t u m o r b. Warthin's t u m o r c. They arise f r o m the epithelial and the l y m p h o i d cells
c. Acinic cell t u m o r d. Pleomorphic adenoma d. 1 0 % are bilateral
2 4 . M o s t c o m m o n s a l i v a r y g l a n d t u m o r in c h i l d r e n : 3 4 . T r e a t m e n t of choice for Warthin's t u m o r is:
[AIIMS 99]
[AIIMS 01 ;AI 98]
a. L y m p h o m a b. Adenoid cystic Ca
a. Superficial parotidectomyb. Enucleation
c. Pleomorphic adenoma d. M u c o e p i d e r m o i d Ca
c. Radiotherapy d. Injection of a sclerosing agent
2 5 . All are t r u e for p l e o m o r p h i c a d e n o m a e x c e p t : [PGI 99]
3 5 . M u c o e p i d e r m o i d c a r c i n o m a of p a r o t i d arises f r o m :
a. Arises f r o m parotid
[PGI 99]
b. May turn into malignant
a. Mucus secreting and epidermal cells
c. Minor salivary gland can be affected
b. Excretory cells
d. None
c. Myoepithelium cells
2 6 . T r e a t m e n t of choice for p l e o m o r p h i c a d e n o m a :
d. Acinus
[AIIMS 96, 98, 01; Al 97; PGI 95, 99]
3 6 . True s t a t e m e n t [s] a b o u t s a l i v a r y g l a n d t u m o r s : [PGI 04]
a. Superficial parotidectomy
a. Pleomorphic adenoma can arise in submandibular gland
b. Radical parotidectomy
b. Warthin's t u m o r arises f r o m submandibular gland
c. Enucleation
c. Pleomorphic a d e n o m a is most c o m m o n t u m o r o f s u b -
d. Radiotherapy
mandibular gland
27. R a m a v a t i , a 40-year-old f e m a l e , p r e s e n t e d w i t h a
d. Acinic cell Ca is most malignant
progressively i n c r e a s i n g l u m p in t h e p a r o t i d r e g i o n .
e. Frey's syndrome can occur after parotid surgery
O n oral e x a m i n a t i o n s , t h e tonsil w a s p u s h e d m e d i a l l y .
37. In s u r g e r y of s u b m a n d i b u l a r s a l i v a r y g l a n d , n e r v e o f t e n
B i o p s y s h o w e d it t o b e p l e o m o r p h i c a d e n o m a . The
a p p r o p r i a t e t r e a t m e n t is: [AIIMS 01 ] involved: [PGI June 97]
a. Hypoglossal b. Glossopharyngeal
a. Superficial parotidectomy
b. Lumpectomy c. Facial d. Lingual
c. Conservative total parotidectomy 38. In w h i c h of t h e f o l l o w i n g c o n d i t i o n s s i a l o g r a p h y i s
d. Enucleation contraindicated: [AI05/AI07]
a. Ductal calculus b. Chronic parotitis
2 8 . Which of t h e following is not a n indication of radiotherapy
in p l e o m o p h i c a d e n o m a of p a r o t i d : [Al 04] c. Parotid obstruction d. Acute sialadenitis
a. Involvement o f deep lobe 3 9 . M o s t c o m m o n c a u s e of unilateral p a r o t i d s w e l l i n g in a
b. 2nd histologically benign recurrence 27yroldmaleis: [AI01]
c. Microscopically positive margins a. Warthin's t u m o r b. Pleomorphic adenoma
d. Malignant transformation c. Adenocarcinoma d. Hemangioma
29. M i x e d t u m o r s of t h e s a l i v a r y g l a n d s a r e : [Al 06] 4 0 . True a b o u t Ludwig's a n g i n a : [PGI 07]
a. Most c o m m o n in submandibular gland a. Involves b o t h submandibular and sublingual spaces
b. Usually malignant b. Most c o m m o n cause is dental infection

c. Most c o m m o n in parotid gland c. Bilateral
d. Associated w i t h calculi d. Spreads by lymphatics
30. In w h i c h o n e of t h e f o l l o w i n g h e a d a n d n e c k cancer 41. L u d w i g ' s a n g i n a is c h a r a c t e r i z e d by all t h e f o l l o w i n g
p e r i n e u r a l i n v a s i o n is m o s t c o m m o n l y s e e n : [A105] except: [AI94]
a. Adenocarcinoma a. Cellulitis of the floor of the m o u t h
b. Adenoid cystic carcinoma b. Caused by anaerobic organisms
c. Basal cell carcinoma c. A p h t h o u s ulcers in the pharynx
d. Squamous cell carcinoma d. Infection spreads t o retropharyngeal space
SECTION II Oral Cavity

4 2 . True a b o u t q u i n k e d i s e a s e : [PGI June 05] [June 04] n o s e a n d slight difficulty in b r e a t h i n g . Next step in
a. Bacterial infection b. Peritonsillar abscess management: [AIIMS 07]
c. Vocal cord edema d. Edema of uvula a. IV antibiotics for 7-10 days
43. Le Fort's fracture d o e s not involve: [Kerala 89] b. Observation in hospital
a. Z y g o m a b. Maxilla c. Surgical drainage
c. Nasal b o n e d. Mandible d. Discharge after 2 days and follow-up o f t h e patient after 8
44. C r a n i o f a c i a l d i s s o c i a t i o n is s e e n i n : [SGPGI05, TN 06] weeks
a. Le Fort 1 fracture b. Le Fort 2 fracture 58. Ideal t i m e of c o r r e c t i n g fracture of n a s a l b o n e is:
c. Le Fort 3 fracture d. Tripod fracture [Kolkata 00]
4 5 . Tear d r o p s i g n is s e e n in: [SGPI05] a. Immediately b. After few days
a. Fracture o f floor of orbit b. Fracture o f lateral wall of nose c. After 2 weeks d. After 3-4 weeks
c. Le Fort's fracture d. Fracture o n zygomatic arch
4 6 . Clinical f e a t u r e s of fracture z y g o m a is/are: [PGI Nov 09] Miscellaneous
a. Cheek swelling b. Trismus 5 9 . G r a y i s h w h i t e m e m b r a n e in t h r o a t m a y be s e e n in all o f
c. Nose bleeding d. Infraorbital numbness t h e following infections e x c e p t : [Al 97]
e: Diplopia a. Streptococcal tonsilitis b. Diphtheria
47. F r a c t u r e z y g o m a s h o w s ail t h e f e a t u r e s e x c e p t : [At 97] c. Adenovirus d. Ludwig's angina
a. Diplopia b. CSF rhinorrhea 60. Black color patch in t h e m o u t h is s e e n i n : [AI91]
c. Epistaxis d. Trismus a. Acute tonsillitis b. Peritonsillar abscess
4 8 . T r i p o d fracture is s e e n i n : [MP 08] c. Vincent's angina d . Leukemia
a. Mandible b. Maxilla 6 1 . T r e n c h m o u t h is: [UP 07]
c. Nasal b o n e d. Zygoma a. Submucosal fibrosis
4 9 . W h i c h is not s e e n in fracture m a x i l l a : [AIIMS 91] b. Tumor at uveal angle
a. CSF rhinorrhea b. Malocclusion c. Ulcerative lesion o f t h e tonsil
c. Anesthesia upper lip d. Surgical emphysema d. Retension cyst o f t h e tonsil
50. C S F r h i n o r r h e a o c c u r s d u e to fracture of: [AIIMS 97] 6 2 . T h e t y p i c a l characteristic of d i p h t h e r i c m e m b r a n e is:
a. Roof o f orbit [Delhi 96]
b. Cribriform plate of ethmoidal bone Loosely attached
c. Frontal sinus Pearly w h i t e in color
d. Sphenoid bone Firmly attached and bleeds on remove
5 1 . T h e m o s t c o m m o n site of leak in C S F r h i n o r r h e a is: Fast c o m p o n e n t occasionally
[AI05] 6 3 . O r o d e n t a l fistula is m o s t c o m m o n after e x t r a c t i o n of:
Ethmoid sinus [DNB 00]
Frontal sinus A. 2 incisor
n d
B. 1 premolar
st

Petrous part o f temporal b o n e C. 2 premolar


n d
D. 1 molarst

Sphenoid sinus 6 4 . S u b m a x i l l a r y calculi c a n be v i s u a l i z e d by X-ray in:


5 2 . C S F r h i n o r r h e a is s e e n i n : [PGI June 03] a. 2 0 % cases b. 5 0 % cases
a. Lefort's fracture Type I b. Nasal fracture c. 6 0 % cases d. 8 0 % cases
c. Nasoethmoid fracture d. Frontozygomatic fracture e. 1 0 0 % cases
5 3 . True a b o u t C S F r h i n o r r h e a is: [PGI 02] 65. M a n a g e m e n t of p e r s i s t e n t c a s e s of C S F r h i n o r r h e a is:
a. Occurs d u e t o break in cribriform plate [FMGE 2013]
b. Contains glucose a. Head low postion o n bed b. Straining activities
c. Requires immediate surgery c. Endoscopic repair d. All of t h e above
d. Contains less protein 66. T h e most c o m m o n site of oral cancer a m o n g indian
54. I m m e d i a t e t r e a t m e n t of C S F r h i n o r r h e a r e q u i r e s : p o p u l a t i o n is: [NEETPattern]
[AIIMS 97] a. Tongue b. Floor o f m o u t h
a. Antibiotics and o b s e r v a t i o n " c. Alveobuccal c o m p l e x d . Lip
b. Plugging w i t h paraffin guage 67. In J a r j a w a y fracture of n a s a l b o n e , t h e fracture line is:
c. Blowing o f nose [NEET Pattern]
d. Craniotomy a. Oblique b. C o m m i n u t e d
5 5 . C S F r h i n o r r h e a is d i a g n o s e d by: [AI07] c. Vertical d. Horizontal
a. Beta-2 microglobulin b. Beta-2 transferrin 6 8 . T r i p o d fracture is s e e n i n : [NEETPattern]
c. Thyproglobulin d . Transthyretin a. Mandible b. Maxilla
56. T h e p a t h o g n o m o n i c test for CSF in s u s p e c t e d C S F c. nasal bone d. Zygoma
rhinorrhea is: [MP 07] 6 9 . A p a t i e n t p r e s e n t w i t h e n o p h t h a l m o s after a t r a u m a t o
face by blunt object. T h e r e is no fever a n d n o e x t r a o c u l a r
a. Glucose concentration b. Handkerchief test
m u s c l e palsy. D i a g n o s i s is: [NEETPattern]
c. Halo sign d. Beta-2 transferrin
57. After l a p a r o s c o p i c a p p e n d e c t o m y , p a t i e n t h a d fall a. Fracture maxilla b. Fracture z y g o m a
f r o m b e d o n h e r n o s e after w h i c h s h e h a d s w e l l i n g in c. Blow o u t fracture d. Fracture e t h m o i d
CHAPTER 7 Oral Cavity J 81

EXPLANATIONS AND REFERENCES

1. A n s . is b i.e. S e b a c e o u s g l a n d Ref. Scott Brown's Otolaryngology 7th/ed vol 2 p 1824; Harrison 17th/ed p 128;
Dhingra Sth/ed p 205,6th/ed p 220; Turner 10th/ed p 233; Mohan Bansal p379

Fordyce's Spot

• Yellowish lesions in buccal a n d labial mucosa.


• They are e c t o p i c sebaceous glands w i t h n o e r y t h e m a t o u s halo.
• Seen in u p t o 8 0 % o f p o p u l a t i o n .
• No clinical significance.

Also R e m e m b e r :
• Forchhiemer spots: seen in rubella, infectious mono nucleosis and scarlet fever.
• Rothe's spots: Infective endocarditis
• Rose spots: Typhoid fever
Kopliks spot: Measles (above the second molar). J

2. A n s . is a, b a n d e i.e. Viral p r e d i s p o s i t i o n ; R e c u r r e n t ulcer; a n d S t e r o i d s g i v e n as t r e a t m e n t


Ref. Dhingra 5th/edp 230,6th/ed p218; Mohan Bansal p 381-2
Aphthous ulcers are recurrent and superficial ulcers, usually involving movable mucosa i.e. inner surfaces ofips, buccal mucosa, tongue,
floor of mouth and soft palate, while sparing mucosa ofthe hard palate and gingivae.

Etiology

Is u n k n o w n is b u t d u e t o may be:
• N u t r i t i o n a l deficiency o f vit. B12, folic acid a n d iron.
• Viral i n f e c t i o n
• H o r m o n a l changes

Treatment

• Topical steroids and cauterization w i t h 1 0 % silver nitrate

o Remember:
• Recurrence is c o m m o n in ulcers.
• M/C cause of viral oral ulcer = Herpes simplex type I
• Painless oral ulcers are seen in—syphilis
^ Bechet's syndrome is oral ulcers + genital ulcers + eye disease (iridocyclitis and retinal vasculitis) + vascular malformation.

-
3. A n s . is b i.e. A r i s e s f r o m s u b m a n d i b u l a r g l a n d
Ref. Dhingra 5th/ed p 237,6th/ed p 224; Surgical Short Cases 3rd/edp 45,46; Mohan Bansal p 403
4. A n s . is b i.e. It is a cystic s w e l l i n g in t h e floor of m o u t h .
Ranula
• T h i n w a l l e d bluish r e t e n t i o n cyst. 0

• Seen in t h e f l o o r o f m o u t h o n o n e side o f t h e f r e n u l u m . 0

• It arises d u e t o o b s t r u c t i o n o f d u c t o f s u b l i n g u a l salivary q l a n d .
• It is a l m o s t always unilateral.

Clinical Features

• Seen m o s t l y in c h i l d r e n a n d y o u n g adults.
• O n l y c o m p l a i n — s w e l l i n g in t h e f l o o r o f m o u t h
• Cyst may r u p t u r e s p o n t a n e o u s l y b u t recurrence is c o m m o n
SECTION II Oral Cavity

O/E
Bluish in color - Brilliantly t r a n s l u c e n t 0

L y m p h nodes are n o t e n l a r g e d

Types
Simple: Situated in f l o o r o f m o u t h w i t h o u t any cervical p r o l o n g a t i o n .
Deep/plunging: Ranula w h i c h e x t e n d s t o t h e neck t h r o u g h t h e muscles o f m y l o h y o i d .
Such p r o l o n g a t i o n appears in s u b m a n d i b u l a r r e g i o n .

Management
Surgical exicision o f ranula a l o n g w i t h s u b l i n g u a l salivary g l a n d is t h e ideal t r e a t m e n t .

NOTE
Cavernous ranula is a type of lymphangioma which invades the fascial planes of neck

• M/C D/D o f ranula = s u b l i n g u a l d e r m o i d ( o p a q u e m i d l i n e swelling)


0

• D u r i n g excision o f ranula = M/C nerve w h i c h can be d a m a g e d is lingual n e r v e . 0

5 a . A n s . is a, c i.e. E r y t h r o p l a k i a ; a n d L e u k o p l a k i a
5b. A n s . is a, b, c, d i.e. S i d e r o p e n i c d y s p h a g i a , O r a l s u b m u c o u s fibrosis, E r y t h r o p l a k i a , L e u k o p l a k i a
6. A n s . is b i.e. S u b m u c o u s fibrosis Ref. Devita 7th/ed p 982; Bailey and Love 25th/ed p 735
L e s i o n s a n d c o n d i t i o n s of t h e oral m u c o s a a s s o c i a t e d w i t h a n i n c r e a s e d risk of m a l i g n a n c y .

Premalignant conditions C o n d i t i o n s i n c r e a s i n g risk Risk is d o u b t f u l

- Leukoplakia - Oral submucosa fibrosis - Oral lichen planus


- Erythroplakia - Syphilitic glossitis - Discoid lupus erythematosus
- Speckled erythroplakia - Sideropenic dysphagia (Paterson-Kelly syndrome) - Dyskeratosis congenita.
- Chronic hyperplastic candidiasis

... Bailey and Love 25th/edp 735

• Friends in the table 46.2 given in Bailey and Love, Leukoplakia is not included in conditions associated with increased risk but in the description
just given below it - leukoplakia is specially mentioned.
• Premalignant lession is morphologically altered tissue where canccer is more likely t o occur e.g. Leukoplakia whereas premalignant condition is a
generalised state where these is significantly increased risk of cancer, e.g. syphilis, submucous fibiosis.

7. A n s . is a i.e. L e u k o p l a k i a Ref. Devita 7th/ed p 982; Bailey and Love 25th/ed p 735; Mohan Bansal p 376-7
"Leukoplakia is the most common premalignant oral mucosal lesion." Mohan Bansalp 377
"The malignant potential of erythroplakia is 17 times higher than in leukoplakia." Mohan Bansal p 376

Remember:

Most common p r e m a l i g n a n t c o n d i t i o n f o r oral cancer Leukoplakia or speckled leukoplakia


Premalignant c o n d i t i o n w i t h highest risk f o r oral cancer Erythroplakia. (M/C Site = lower alveolar margin and floor of
mouth)
Painless oral ulcers are seen in-syphilis
Bechet's s y n d r o m e = oral ulcers + g e n i t a l ulcers + eye disease (iridocyclitis a n d retinal vasculitis) + vascular m a l f o r m a t i o n .

Important Points on Leukoplakia

• Clinical white patch t h a t can't be characterized clinically or p a t h o l o g i c a l l y as any o t h e r disease is leukoplakia.


• M o s t c o m m o n site is buccal mucosa a n d oral commissures.
Tobacco s m o k i n g a n d c h e w i n g are m a i n e t i o l o g i c a l factor.
If p a t i e n t stops s m o k i n g for 1 year, it w i l l disappear in 6 0 % o f cases.
Features suggestive malignant change in leukoplakia are i n d u r a t i o n , speckled or n o d u l a r appearance.
Chances o f m a l i g n a n t changes in leukoplakia increases w i t h increases in age of lesion and age of patient.
All lesions m u s t be b i o p s i e d a n d sent for h i s t o l o g y as it has 2 - 8 % risk o f m a l i g n a n c y .
CHAPTER 7 Oral Cavity J 83

1 Lesion Treatment 1
- Hyperkeratosis Follow-up at 4 monthly interval/chemopreventive drugs
- Dysplasia Surgical excision or C 0 laser exicison
2

Remember:
C h e m o p r e v e n t i v e d r u g s used in oral m a l i g n a n c y :
• Vit. A, E, C • Betacarotene
• lavonoids • Celecoxib

8. A n s . is c i.e. A l v e o b u c c a l c o m p l e x Ref.ASI1 st/ed p 348; Oncology and Surgery Journal 2004 p 161
F r e q u e n c y of v a r i o u s c a n c e r of oral c a v i t y in India a r e : Buccal mucosa 3 8 %
• Anterior t o n g u e 1 6 %
• Lower alveolus 1 5 %
So, most common site o f oral cancer a m o n g Indian p o p u l a t i o n is buccal mucosa or in this q u e s t i o n alveobuccal c o m p l e x (due t o
t h e i r p r e d i l e c t i o n for p a n c h e w i n g w h e r e t o b a c c o is kept in l o w e r g i n g i v o b u c c a l suldus).

Remember:
a. Most common site o f oral cancer in w o r l d : T o n g u e
b. Most common histological variety o f oral cancer: S q u a m o u s cell carcinoma
c. M/C histological variety of lip carcinoma - s q u a m o u s cell carcinoma
d. Oral m a l i g n a n c y w i t h best prognosis = lip cancer
e. M/C site f o r Ca lip = lower lip
f. Oral m a l i g n a n c y w i t h w o r s t prognosis = f l o o r o f m o u t h .

9. A n s . is b i.e. Lateral b o r d e r Ref. Dhingra 5th/ed p 240,6th/ed p 227; Scott Brown 7th/ed vol 2 p 2552; Mohan Bansal p 407
"Most common site of carcinoma tongue is middle of lateral border or the ventral aspect of the tongue followed by tip and
dorsum." Dhingra6th/ed,p227

Cancer M o s t c o m m o n site

• Lip Vermillion of lower lip


• Tongue Lateral border
• Cheek Angle of mouth
• Nasopharyngeal carcinoma Fossa of rosenmuller •

• Larynx Glottis

10. A n s . is c i.e. N2
Ref. Schwartz 9th/ed p 491; Devita Oncology 7th/ed p 665,672,689; Dhingra 5th/ed p 241,6th/ed p 228; Mohan Bansal p 406
Classification of s t a g e of t u m o r of oral c a v i t y b a s e d on s i z e of l y m p h n o d e .

< 3cms between 3 cm and 6 cm >6 cm

Stage N1 Stage N2 Stage N3

In t h e g i v e n q u e s t i o n Size o f l y m p h n o d e is 4 c m so it b e l o n g t o stage N2
For d e t a i l e d classification See t e x t g i v e n in t h e b e g i n n i n g . •

R e m e m b e r : For all head a n d neck cancers except t h e nasopharynx, t h e ' N ' classification system is u n i f o r m .

1 1 . A n s . is a i.e. L a s e r a b l a t i o n Ref. Schwartz 8th/ed p519; 9th/ed p 492; Current Otolaryngology 3rd/ed p 382
"The carbon dioxide laser may be used for excision of early tongue cancers (T1) or for ablation of premalignant lesion."
Patient in the question has tumor of 1.5x1 cm. So, comes under 77.

Remember:
• T r e a t m e n t o f choice for small (T1-T2) t o n g u e cancer is w i d e local exicision transorally. (Transoral partial glossectomy)
• For small T1-T2 lesions radiotherapy is not used n o w . — C u m m i n g s Otolaryngology 4th/edp 1597
• T3 and T4 Stage tumors treated by transmandibular or transcervical total glossectomy.
» Tongue base tumors are treated by chemoradiation (S/B 7th/ed vol 2 p 2554)

1 2 . A n s . is d i.e. Radical n e c k d i s s e c t i o n Ref. Bailey and Love 25th/ed p716; Mohan Bansal p 408
SECTION II Oral Cavity

M a n a g e m e n t o f Neck N o d e s in O r o p h a r y n g e a l Cancers

If t h e n o d e s a r e clinically n e g a t i v e (i.e. t h e r e is occult m e t a s t a s i s )


• Generally t o n g u e cancers a n d t o a lesser e x t e n t f l o o r o f m o u t h cancers give rise t o o c c u l t metastases
• It is always g o o d t o actively treat cervical l y m p h nodes in even absence o f o b v i o u s disease.

Management

• In C a t o n g u e w i t h no n o d e s In C a of floor of m o u t h a n d m a n d i b u l a r a l v e o l a r w i t h n o n o d e s

i . A
Extended s u p r a o m o h y o i d neck dissection (i.e. removal Supra o m o h y o i d neck dissection
o f LN levels I, II, III and IV) in c o n t i n u i t y w i t h (i.e. removal o f LN levels I, II a n d III in c o n t i n u i t y w i t h p r i m a r y t u m o r )
primary t u m o r
If l y m p h nodes are i n v o l v e d - o p t i o n s are:
• Selective s u p r a o m o h y o i d neck dissection (for stage N1)
• Radical neck dissection (for all o t h e r stages)
N o w in t h e q u e s t i o n , t h e size a n d n u m b e r o f nodes i n v o l v e d is n o t g i v e n b u t it is given t h a t ' l y m p h nodes in t h e l o w e r neck' are
i n v o l v e d . So t h e o p t i o n s u p r a o m o h y o i d dissection is ruled o u t (as it is d o n e in case o f either o c c u l t metastasis or single ipsilateral
n o d e < 3 cm) a n d t h e o b v i o u s answer is radical neck dissection.
1 3 . A n s . is b i.e. S u b m a n d i b u l a r Ref. Dhingra 5th/ed p 240,6th/edp 227
• M/C l y m p h n o d e i n v o l v e d in any oral m a l i g n a n c y is S u b m a n d i b u l a r LN
• M a x i m u m LN metastases is seen in cancer t o n g u e f o l l o w e d by f l o o r o f m o u t h .
• L y m p h a t i c metastasis is least in lip cancer f o l l o w e d by hard palate.

1 4 . A n s . is a i.e. R e g i o n a l l y m p h n o d e Ref. Devita 7th/edp 682; Schwartz 9th/edp 494

Tumors of Buccal Mucosa


"Tumors in this area have a propensity to spread locally and to metastasize to regional lymphatics" —Schwartyz 9th/ed pp 494,495
1 5 . A n s . is c i.e. R a d i o t h e r a p y a n d m a x i l l e c t o m y Ref. Scott Brown's 7th/ed vol 2 p. 2427
• For s q u a m o u s cell c a r c i n o m a , a c o m b i n a t i o n o f r a d i o t h e r a p y a n d surgery gives b e t t e r results t h a n either alone. Radiotherapy
can be g i v e n before or after surgery.
• For a d e n o c a r c i n o m a o r m e l a n o m a of m a x i l l a r a d i o t h e r a p y is ineffective so o n l y surgery is d o n e .
1 6 . A n s . is c i.e. V e r r u c o u s c a r c i n o m a
Ref. Scott Brown 7th/ed vol 2 p 2561; Diagnostic Histopathology of Tumors by Fletcher 2nd/ed Vol I, pp 211,212
A l t h o u g h M/C variety o f buccal cancer is s q u a m o u s cell cancer, Verrucous carcinoma is a variety o f well-differentiated s q u a m o u s
cell c a r c i n o m a w h i c h is locally aggressive i n v o l v i n g t h e b o n e b u t l y m p h n o d e metastasis is u n c o m m o n . Histologically, these t u m o r s
s h o w m a r k e d hyperkeratosis a n d acanthosis w i t h dysplasia l i m i t e d t o deeper layers. Repeated biopsies r e p o r t it as s q u a m o u s p a p i l -
loma.
"Histologically verrucious carcinoma are characterized by marked acanthosis, hyperkeratosis often with broad bullous process
showing central columns of keratin. There is no cytological evidence of malignancy."
1 7 . A n s . is c i.e. S e g m e n t a l m a n d i b u l e c t o m y Ref. Cummings Otolaryngology 4th/ed p 1608; Oncology and Surgery 2004 p 169
• Surgery is t h e t r e a t m e n t o f choice in m a n d i b l e cancers.
• Radiotherapy is c o n t r a i n d i c a t e d as it can lead t o osteoradionecrosis o f m a n d i b l e .
• M a n d i b l e is m a n a g e d surgicals by m a r g i n a l or s e g m e n t a l resection.
• Marginal (rim) resection keeps t h e o u t e / l o w e r r i m (1 c m t h i c k m a n d i b l e i n t a c t t o m a n t a i n cosmesis.)
• It is i n d i c a t e d w h e n t h e r e is i n v o l v e m e n t o f p e r i o s t e u m o n l y or w i t h m i n i m a l alveolar/cortical i n v o l v e m e n t .
• S e g m e n t a l resection removes a full s e g m e n t o f m a n d i b l e creating a d e f e c t w h i c h necessitates r e c o n s t r u c t i o n .

Indications

(i) W h e n t h e r e is gross i n v o l v e m e n t o f cancellus b o n e


(ii) (Even minimal involvement only in an edentulous mandible.) 0

(iii) I n v o l v e m e n t o f inferior alveolar canal (earlier h e m i m a n d i b u l e c t o m y was b e i n g done)


(iv) In previously irradiated m a n d i b l e (where m a r g i n a l resection m a y lead t o a p a t h o l o g i c a l f r a c t u r e o f t h e w e a k n e d bone)
1 8 . A n s . is a i.e. R a d i o t h e r a p y Ref. Read below

Explanation

The m o s t a p p r o p r i a t e answer here is Radiotherapy.The t r e a t m e n t o f choice here is segmental m a n d i b u l e c t o m y . From t h e discus-


sion above o n m a n a g e m e n t o f m a n d i b l e in oral malignancy, it is e v i d e n t t h a t marginal m a n d i b u l e c t o m y has little role in surgical
CHAPTER 7 Oral Cavity

m a n a g e m e n t o f an e d e n t u l o u s m a n d i b l e , b u t r a d i o t h e r a p y is a b s o l u t e l y c o n t r a i n d i c a t e d because it can t r i g g e r osteoradionecrosis


here. Hence, it s h o u l d be selected as t h e m o s t a p p r o p r i a t e answer here.
1 9 . A n s . is a i.e. C a r c i n o m a Lip Ref. Cummings otolaryngology 4th/ed p 1594, 1602; Mohan Bansal p 406

Oral m a l i g n a n c y w i t h best prognosis is carcinoma lips.

5-Year S u r v i v a l R a t e s i n c a n c e r l i p -

Site Lip Tongue Palate Cheek -


Stage 1 and II 90% 75% 80% 65-75%

Stage III and IV • 50% 40% 40% 5 0 % (Stage III); 50% (Stage IV)

As is clear f r o m a b o v e t e x t f o r some stage carcinoma lip has h i g h e s t 5-year survival rate or has t h e best prognosis.

C A R C I N O M A LIPS

• M/C site f o r Ca lips = v e r m i l i o n o f l o w e r lip


• Lower lip cancer has a b e t t e r prognosis t h a n u p p e r lip carcinoma as l o w e r lip metastases t o s u b m e n t a l a n d s u b m a n d i b u l a r
nodes w h i l e u p p e r lip in a d d i t i o n also involves preaural a n d p a r o t i d l y m p h nodes.
• M/C histiological t y p e o f lip cancer ( b o t h u p p e r lip a n d l o w e r lip) is s q u a m o u s cell cancer.
• Basal cell carcinoma is m o r e c o m m o n in u p p e r lip.
. M/C e t i o l o g i c factor - solar r a d i a t i o n
• T r e a t m e n t o f choice o f Ca lips is = surgery
• Oral cancer w i t h w o r s t prognosis is f l o o r o f m o u t h c a r c i n o m a .

20. A n s . is b, c, d a n d e i.e. S y s t e m i c m e t a s t a s i s u n c o m m o n ; R e s p o n d s to r a d i o t h e r a p y ; S u r g e r y is t r e a t m e n t of choice; a n d


S y p h i l i s a n d d e n t a l irridation p r e d i s p o s e s Ref. Dhingra 5th/edp 238,6th/edp 226; Bailey and Love 25th/edp 740
• M o s t c o m m o n site of oral c a v i t y c a r c i n o m a in w o r l d is t o n g u e ; In India it is buccal mucosa, (so o p t i o n a is incorrect)
• Tumors of oral cavity are radiosensitive but because of its serious complications (Xerostomia; Mandibular necrosis) it is not
indicated as primary treatment. Surgery is the treatment of choice in tumors of oral cavity. (So option c ard d both are correct)
• As discussed in t h e p r e c e d i n g t e x t - e t i o l o g i c a l factors f o r oral cancers are:
6Sviz : • Smoking
• Spirit
<-u • i. .i
• Sharp j a g g e d t o o t h
<» Sepsis
• S y n d r o m e o f Plummer-vinson
. Syphilitic glossitis ( o p t i o n e is correct)
• Thus o p t i o n i.e. is syphilis a n d d e n t a l irradiation predispose is correct.
• M/c m e t h o d o f spread o f oral cancers is by local invasion a n d l y m p h a t i c spread.
• Systemic metastasis is rare (i.e. o p t i o n b is correct)
2 1 . A n s . is a i.e. U p p e r cervical LN Ref. Dhingra 5th/edp 239,6th/ed p 227
• As discussed earlier M/C l y m p h n o d e involved in any oral m a l i g n a n c y is s u b m a n d i b u l a r LN. In carcinoma o f lips also - s u b m e n t a l
a n d s u b m a n d i b u l a r nodes are i n v o l v e d first. A t cater stages, d e e p cervical g r o u p o f LN's m a y g e t i n v o l v e d .
• S u b m e n t a l a n d s u b m a n d i b u l a r are i n c l u d e d in u p p e r cervical LN or level 1 l y m p h nodes
22. A n s . is d i.e. S u b m a n d i b u l a r
Ref. Bailey and Love 25th/ed p 755; Current Otolaryngology 2nd/ed p 299; CSDT 13th/ed pp 239-240; Mohan Bansal p393
S t o n e f o r m a t i o n is most common in s u b m a x i l l a r y (submandibular) g l a n d ( 8 0 - 9 0 % cases) followed by parotid g l a n d ( 1 0 - 2 0 % ) .
It can occur at any age w i t h a p r e d i l e c t i o n for m e n .
• P r e d i s p o s i n g factors f o r s t o n e f o r m a t i o n are systemic disease ( H y p e r p a r a t h y r o i d i s m , hypercalcemia, g o u t , diabetes a n d
h y p e r t e n s i o n ) therefore s u b m a n d i b u l a r calculi c o n t a i n p r i m a r i l y c a l c i u m p h o s p h a t e a n d h y d r o x y a p a t i t e a n d are r a d i o p a q u e
a n d visualized on X-ray
• Parotid g l a n d calculi are less r a d i o p a q u e
• M/C p r e s e n t a t i o n - Recurrent swelling a n d pain in t h e s u b m a n d i b u l a r g l a n d exacerbated w i t h e a t i n g .
. IOC t o d e t e c t stones - CT scan (CSDT/13th/edp 240)
• Sialography is n o t d o n e r o u t i n e l y a n d is c o n t r a i n d i c a t e d in a p a t i e n t o f sialadenitis. 0
86 \_ SECTION II Oral Cavity

Management

D e p e n d i n g o n t h e size o f stone a n d t h e site at w h i c h it is located, it can be r e m o v e d by:


. Intraoral e x t r a c t i o n
• Surgical excision
• Endoscopic removal
2 3 . A n s . is d i.e. P l e o m o r p h i c a d e n o m a
Ref. Devita 7th/edp 725; Bailey and Love 24th/edp 730; Scott's Brown 7th/ed vol 2 p 2476; Mohan Bansal p395
"Pleomorphic adenoma is the commonest tumor found at any site and outnumbers all other tumors in major glands."
—Scott's Brown 7th/ed Vol 3rd/ed p 2476
"Pleomorphic adenomas or benign mixed tumors are the M/C neoplasms of salivary gland" —Current Otolaryngology 3rd/ed p 329
• Most common tumor of salivary g l a n d Pleomorphic adenoma
• Mostcommon b e n i g n t u m o r o f salivary g l a n d Pleomorphic adenoma
• Mostcommon m a l i g n a n t t u m o r o f major salivary g l a n d : Mucoepidermoid carcinoma
• Mostcommon m a l i g n a n t t u m o r o f m i n o r salivary g l a n d : Adenoid cystic carcinoma
• Mostcommon b e n i g n a n d overall t u m o r o f p a r o t i d : Hemangioma (Current Otolaryngology3rd/ed
in c h i l d r e n (specially < 1 yr) p332;Maqbool 12th/edp209)
• Most common m a l i g n a n t t u m o r in c h i l d r e n : Mucoepidermoid (Maqbool 12th/edp 209)
• Most common radiation i n d u c e d n e o p l a s m o f salivary g l a n d Mucoepidermoid carcinoma.
2 4 . A n s . is c i.e. P l e o m o r p h i c a d e n o m a Ref. Scott Brown 7th/ed Vol 1 p 1248
"The commonest benign tumor encoun tered is pleomorphic salivary adenoma accounting for approximately 30% of all pediatric salivary
neoplasma. The majority occur within the parotid gland."
Most common m a l i g n a n t t u m o r o f salivary g l a n d in c h i l d h o o d : Mucoeidermoid carcinoma, approximately 50% followed by acinic
cell carcinoma (20%)—Scoff Brown 7th/ed Vol 1 p 1248, Current otolaryngology 3rd/ed, p 341
2 5 . A n s . is d i.e. N o n e Ref. Current Otolaryngology 2nd/ed pp 307-308; 3rd/edp 329,330; Dhingra 5th/edp247
.
Pleomorphic Adenoma

It is t h e M/C b e n i g n t u m o r o f salivary g l a n d s 0

• It can arise f r o m t h e p a r o t i d , s u b m a n d i b u l a r or o t h e r m i n o r salivary glands o f palate a n d p h a r y n x


0 0 0

- Dhingra 5th/ed p 247; Scotts Brown 7th/ed Vol 2 p 2475


They represent ~ 6 0 - 7 0 % o f all p a r o t i d t u m o r s a n d 9 0 % o f s u b m a n d i b u l a r b e n i g n t u m o r s
M/C age g r o u p affected is f o u r t h decade
M/C g l a n d involved - p a r o t i d g l a n d
M/C site affected in p a r o t i d g l a n d is - tail o f p a r o t i d g l a n d
They are s l o w g r o w i n g painless t u m o r s
Histologically, t h e y c o n t a i n b o t h epithelial a n d m e s e n c h y m a l e l e m e n t s a n d are therefore called as m i x e d t u m o r s .
It can rarely u n d e r g o m a l i g n a n t t r a n s f o r m a t i o n (current o t o l a r y n g o l o g y 2nd/ed p 3 0 8 , 3 r d / e d p 330)
T O C - Surgery - C o m p l e t e surgical excision o f t h e t u m o r w i t h u n i n v o l v e d margins is t h e r e c o m m e n d e d t r e a t m e n t , for e x a m p l e , if
t h e t u m o r is in superficial l o b e - superficial p a r o t i d e c t o m y is t h e surgery o f choice.
Prognosis is excellent w i t h a 9 5 % non-recurrence rate.
2 6 . A n s . is a i.e. Superficial p a r o t i d e c t o m y
Ref. Dhingra 5th/ed p 247,6th/ed p 234; Current otolaryngology 2nd/ed p 308,3rd/ed, p 330 and; Short Cases of Surgery 3rd/ed p 77
T r e a t m e n t o f choice for p l e o m o r p h i c a d e n o m a is superficial p a r o t i d e c t o m y b u t , if t h e d e e p lobe o f p a r o t i d is i n v o l v e d , t o t a l p a r o t i -
d e c t o m y is d o n e .
2 7 . A n s . is c i.e. C o n s e r v a t i v e total p a r o t i d e c t o m y Ref. Schwartz 8th/ed p 540
"For parotid tumors that arise in lateral lobe superficial parotidectomy with preservation ofCN VII is indicated. If the tumor extends in to
deep lobe of parotid, a total parotidectomy with nerve preservation is performed."
In this q u e s t i o n tonsil is p u s h e d m e d i a l l y i.e. d e e p lobe o f p a r o t i d is also i n v o v l e d , so conservative t o t a l p a r o t i d e c t o m y w i l l be d o n e .
2 8 . A n s . is b i.e. 2 n d histologically b e n i g n r e c u r r e n c e Ref. Devita 7th/edp725
Radiotherapy is indicated for m a l i g n a n t recurrence n o t for b e n i g n recurrence.

Indications of Radiotherapy in Salivary Gland Tumor

• Low-grade n e o p l a s m w i t h close or positive m a r g i n


• Facial nerve i n v o l v e m e n t
• M u l t i p l e regional n o d e metastasis
• High-grade h i s t o l o g y
Deep l o b e i n v o l v e m e n t
CHAPTER 7 Oral Cavity

o Perineural invasion
• Recurrence o f m a l i g n a n t t u m o r s .
29. A n s . is c i.e. M o s t c o m m o n in p a r o t i d g l a n d
Ref. Bailey and Love 24th/edp 731; Robbins 7th/ed pp 791,792; Dhingra 5th/ed p 247,6th/ed p 234; Mohan Bansal p 395
M i x e d t u m o r s o f salivary glands are p l e o m o r p h i c a d e n o m a s (as t h e y have b o t h epithelial a n d m e s e n c h y m a l elements)
"80% of salivary gland tumor occur in parotid. Of these tumors approximately 75-80% are pleomorphic adenoma (mixed tumor)."

NOTE ^^^^^^^p^^^^^^^^^B^^W^^^^^^^^^^^P^a^i^jjyi
M/C site for all salivary gland tumors is parotid gland except for:
• Adenoid cystic carcinoma = M/C site is minor salivary gland.
• Squamous cell carcinoma = M/C site is submandibular gland.

30. A n s . is b i.e. A d e n o i d cystic c a r c i n o m a


Ref. Schwartz 8th/edp 539; Bailey 24th/ed p 685; Dhingra 5th/edp 248,6th/edp 235; Current Otolaryngology 2nd/ed p315,3rd/ed p 338
Perineural invasion is the most constant microscopic finding in adenoid cystc carcinoma.

A d e n o i d Cystic C a r c i n o m a (Cylindroma)

Mostcommon m a l i g n a n t t u m o r o f s u b m a n d i b u l a r glands.
Mostcommon m i n o r salivary glands t u m o u r .
Mostcommon site m i n o r salivary g l a n d .
Characterized by its t e n d e n c y t o invade perineural space a n d lymphatics a n d t h u s causes pain (which m a y be a p r o m i n e n t a n d
early s y m p t o m ) and VII nerve paralysis.
. . . . . .
Skip lesions a o n g nerves are c o m m o n .
It is a treacherous tumor as it appears b e n i g n e v e n w h e n it is m a l i g n a n t ,
t can metastasize t o y m p h nodes
They are h i g h l y recurrent.
Local recurrence after surgical excision are c o m m o n a n d can occur as late as 20 years after surgery. Distant metastases g o t o
l u n g , b r a i n a n d bone.
T r e a t m e n t o f choice is radical parotidectomy irrespective of its benign appearance under the microscopy.
Radical neck dissection is n o t d o n e unless nodal metastases are present
Postoperative radiation is g i v e n if margins o f resected specimen are n o t free o f t u m o r

EXTRA EDGE

• The m o s t c o m m o n histologic s u b t y p e ( 4 4 % ) is t h e cribriform t y p e , characterized by a " S w i s s - C h e e s e " p a t t e r n o f vacuolated


area. It has i n t e r m e d i a t e prognosis
• The t u b u l a r s u b t y p e has t h e best prognosis w h i l e solid s u b t y p e has t h e w o r s t prognosis
31. A n s . is a i.e. Parotid s a l i v a r y g l a n d Ref. Schwartz 8th/edp 539; Robbin's 7th/edp 794; Current Otolaryngology 2nd/edp315
"80-90% occur in the parotid gland and most ofthe remaining occur in submandibular gland" -Current Otolaryngology 2nd/ed p316.

Remember: All salivary g l a n d t u m o r are most common in p a r o t i d g l a n d except adenoid cystic carcinoma [most common in minor
salivary gland) a n d quamous cellcarinoma (most common is s u b m a n d i b u l a r gland).

~ . . . . . - .. -
I m p o r t a n t Points a b o u t Acinic Cell C a r c i n o m a

• Affect exlusively parotid gland


• Low-grade malignancy
• Hypercellularturnor w i t h relative absence of stroma. It is enclosed in a f i b r o u s capsule
• T r e a t m e n t is radical excision.
32. A n s . is a i.e. A n a d e n o l y m p h o m a o f p a r o t i d g l a n d
33. A n s . is a i.e. M o r e c o m m o n in f e m a l e s
34. A n s . is a i.e. Superficial p a r o t i d e c t o m y
Ref. CSDT 13th/edp 257; Current Otolaryngology 2nd/ed p 308; 3rd/ed, 330; Dhingra 5th/ed p 248,6th/ed p 234; Mohan Bansal p396
• Warthin's t u m o r o r p a p i l l a r y c y s t a d e n o m a l y m p h o m a t o s u m o r a d e n o l y m p h o m a is 2 n d m o s t c o m m o n b e n i g n t u m o r
accounting for 5 % of parotid gland tumors.
• It arises e x c l u s i v e l y f r o m p a r o t i d g l a n d .
• It a l m o s t always occur in older males, (in 5 t h t o 7 t h decade)
• There is increased risk in smokers

88 [_ SECTION II Oral Cavity

• Most common site is tail of parotid


• It is bilateral in 70% cases.
• It consists o f papillary cystic p a t t e r n lined w i t h c u b o i d a l a n d c o l u m n a r cells w i t h core o f l y m p h o i d tissue.
• Treatment o f choice is superficial p a r o t i d e c t o m y b u t because o f its b e n i g n nature and since it can be easily diagnosed cytologically,
surgical removal is not always necessary especially in older or unhealthy persons.

Remember:
• It is o n l y salivary g l a n d t u m o r t h a t produces hot spot in 99Tcm scan so its preoperative diagnosis is made without biopsy.
• It never involves facial nerve i.e. it never b e c o m e s m a l i g n a n t .
• It is t h e o n l y salivary g l a n d t u m o r w h i c h is m o r e c o m m o n in males

3 5 . A n s . is a i.e. M u c u s s e c r e t i n g a n d e p i d e r m a l cells
Ref. Robbin's 7th/edp 793; Dhingra 5th/edp 248,6th/ed p 235; Current Otolaryngology 3rd/ed p 337
Muco epidermoid carcinoma is the M/C type of malignant salivary gland tumor.
M u c o e p i d e r m o i d t u m o r consists of f o l l o w i n g cells:
• S q u a m o u s cells • Mucus secreting cells
. I n t e r m e d i a t e h y b r i d cells . Clear or h y d r o p i c cells.
( p r o g e n i t o r o f o t h e r cells)
• No m y o e p i t h e l i a l cells are s e e n

ALSO KNOW

M u c o e p i d e r m o i d t u m o r s similar t o o t h e r t u m o r s is m o r e c o m m o n in p a r o t i d and has a f e m a l e p r e d o m i n a n c e .


They are m a l i g n a n t t u m o r s w h i c h are slow g r o w i n g a n d can invade facial nerve
Histologically, t h e greater is t h e ratio o f e p i d e r m o i d e l e m e n t , t h e m o r e m a l i g n a n t is t h e behavior o f t h e t u m o r
They are m o r e aggressive in m i n o r salivary glands as c o m p a r e d t o major salivary glands.
Low-grade t u m o r s are m o r e c o m m o n in c h i l d r e n

Management

Low-grade T u m o r s High-grade t u m o r

Total parotidectomy with preservation of facial nerve • Total parotidectomy


• Facial nerve may be sacrificed if it is invaded by tumor
• Radical neck dissection may be done.

3 6 . A n s . is a, c a n d e i.e. P l e o m o r p h i c a d e n o m a c a n a r i s e in s u b h m a n d i b u l a r g l a n d ; P l e o m o r p h i c a d e n o m a is t h e m o s t c o m m o n
t u m o u r o f s u b m a n d i b u l a r g l a n d ; a n d Frey's s y n d r o m e c a n o c c u r after parotid s u r g e r y Ref. Scott Brown 7th/ed Vol 1 p 1248,
MB p 395-396

Lets Analyse Each Option Separately

• O p t i o n a - Pleomorphic a d e n o m a can arise in s u b m a n d i b u l a r g l a n d .


This is correct as "Pleomorphic adenoma - It can arise from the parotid, submandibular or other minor salivary glands"
-Dhingra 5th/ed p247,6th/ed p 234
• O p t i o n b - Warthin's t u m o r arises f r o m s u b m a n d i b u l a r g l a n d .
This is absolutely incorrect as"Warthin's tumor is found almost exclusively in the parotid gland."
-Curren t Otolaryngology 2nd/ed p308, 3rd/ed p338
• O p t i o n c - Pleomorphic a d e n o m a is t h e M/C t u m o r o f s u b m a n d i b u l a r g l a n d
This is correct as -"Plemorphic adenoma - represent approximated 60-70% of all parotid tumors and 90% of submandibular benign
tumors." —Current Otolaryngology 3rd/ed p 329
• O p t i o n d - Acinic cell Ca is m o s t m a l i g n a n t
This is w r o n g because - "Acinic cells carcinomas are low-grade malignancies."
- Current otolaryngology 2nd/edp316,3rd/edp 338
• O p t i o n e - Frey's s y n d r o m e (gustatory sweating) is a universal sequelae f o l l o w i n g p a r o t i d surgery. -Bailey and love 25th/edp 763

Freys Syndrome (Gustatory sweating)

• Usually manifests several m o n t h s after p a r o t i d o p e r a t i o n .


• Characterized by sweating a n d f l u s h i n g o f t h e preauricular skin d u r i n g mastication.
• Occurs d u e t o aberrant i n n e r v a t i o n o f sweat glands by p a r a s y m p a t h e t i c s e c r e t o m o t o r fibers o f p a r o t i d g l a n d , so instead o f
causing salivary secretions f r o m p a r o t i d , t h e y cause secretions f r o m sweat glands.
CHAPTER 7 Oral Cavity

Treatment
- M o s t l y reassurance.
- In s o m e cases t y m p a n i c n e u r e c t o m y is d o n e w h i c h intercepts these parasympathetic fibers at t h e level o f m i d d l e ear.
3 7 . A n s . is a, c a n d d i.e. h y p o g l o s s a l , facial a n d l i n g u a l n e r v e .
Ref. Bailey and Love 25th/ed p757; Scott Brown 7th/ed Vol 2 pp 2487,88

Sub Mandibular Gland Surgery

• Unlike t h e parotid g l a n d w h e r e o n l y a part o f t h e g l a n d is r e m o v e d , t o t a l resection o f t h e s u b m a n d i b u l a r g l a n d is always indicated


for t u m o r s o f s u b m a n d i b u l a r g l a n d .
• Before p e r f o r m i n g t h e surgery, t h e p a t i e n t s h o u l d be w a r n e d a b o u t t h e f o l l o w i n g serious or f r e q u e n t c o m p l i c a t i o n s
- D a m a g e to m a r g i n a l b r a n c h of facial n e r v e :
- This m a y result in t e m p o r a r y or p e r m a n e n t weakness o f t h e angle o f m o u t h .
- Lingual and hypoglossal nerve damage:
« This results m o r e f r e q u e n t l y , if g l a n d is b e i n g r e m o v e d f o r chronic sialadenitis rather t h a n t u m o r
• It leads t o m o t o r d y s f u n c t i o n o f t o n g u e w h i c h impairs a r t i c u l a t i o n a n d mastication
- Cosmetic defects
3 8 . A n s . is d i.e. A c u t e s i a l a d e n i t i s Ref. Sutton 7th/ed p 535; Diseases of Salivary Gland by Rankow and Prolayes, p 55;
Current surgical diagnosis and treatment 3rd/ed p 240
Sialography:
"Use of sialography during period of an acute inflammation of salivary system is contraindicated." —Sutton 7th/ed p 535
"Sialography is no longer routinely used and is contraindicated in patients with acute sialadenitis." —CSDT 13th/ed p 240

A L S O K N O W - Sialography

M a i n Indications of S i a l o g r a p h y

• Salivary d u c t stones
• Stricture
• Fistula, p e n e t r a t i n g i n j u r y
• I n t r a g l a n d u l a r a n d s o m e t i m e s extra g l a n d u l a r mass lesions.

Contraindications

• Iodine allergy
• A c u t e sialadenitis

Contrast

• Water s o l u b l e media ( M e g l u m i n e diatrizoate)

ALSO KNOW
• M/c o r g a n i s m leading t o bacterial sialadenitis - Staphylococcus
• M/c site o f sialadenitis - Parotid Gland
• M/c site o f sialolithiasis - S u b m a n d i b u l a r Gland
3 9 . A n s . is b i.e. P l e o m o r p h i c a d e n o m a Ref. CSDT 13th/ed p 257; Dhingra 5th/edp 247
• P l e o m o r p h i c a d e n o m a or b e n i g n m i x e d t u m o r accounts f o r 8 0 % o f p a r o t i d t u m o r s a n d 6 0 % o f all salivary g l a n d t u m o r s .
• M o s t c o m m o n site is p a r o t i d g l a n d t h o u g h it can arise f r o m s u b m a n d i b u l a r g l a n d , salivary g l a n d o f palate u p p e r lip a n d buccal
mucosa.

R e m e m b e r : T h o u g h Warthin's t u m o r occurs m o s t c o m m o n in males, b u t m o s t c o m m o n t u m o r in males still is p l e o m o r p h i c a d e n o m a . |

4 0 . A n s . is a a n d b i.e. Involves b o t h s u b m a n d i b u l a r a n d s u b l i n g u a l s p a c e s ; a n d m o s t c o m m o n c a u s e is d e n t a l infection


4 1 . A n s . is c i.e. A p h t h o u s ulcer in p h a r y n x . Ref. Dhingra 5th/edpp277,278,6th/edp263,264; Logan Turner Wth/edpp 107,108
• Ludwig's A n g i n a is a rapidly s p r e a d i n g cellulitis o f t h e f l o o r o f t h e m o u t h w h i c h involves s u b m a n d i b u l a r space secondary t o
dental infection.
• S u b m a n d i b u l a r space is d i v i d e d i n t o s u b l i n g u a l space (above t h e m y l o h y o i d ) a n d s u b m a x i l l a r y space ( b e l o w t h e m y l o h y o i d
muscle) (Fig. 7.3).
• B a c t e r i o l o g y : M i x e d i n f e c t i o n s i n v o l v i n g b o t h aerobes a n d anaerobes are c o m m o n like, alpha h a e m o l y t i c s t r e p t o c o c c i ,
staphylococci bacteriodes a n d £ coli. H. influenzae is a rare cause. 0
90 J_ SECTION II Oral Cavity

• Clinical f e a t u r e s : In Ludwig's a n g i n a , t h e r e is usually cellulitis •

o f t h e tissue rather t h a n frank abscess.


• M a r k e d d i f f i c u l t y in s w a l l o w i n g ( o d y n o p h a g i a ) .
• Varying degrees o f t r i s u m u s .
• O n i n v o l v e m e n t o f s u b l i n g u a l space, f l o o r o f t h e m o u t h is
s w o l l e n , e d e m a t o u s a n d t o n g u e seems t o be p u s h e d u p a n d
Tongue
back.
• On i n v o l v e m e n t o f t h e s u b m a x i l l a r y space, t h e s u b m e n t a l a n d
s u b m a n d i b u l a r regions b e c o m e s w o l l e n a n d t e n d e r a n d i m p a r t Sublingual
w o o d y - h a r d feel. T o n g u e is progressively p u s h e d u p w a r d a n d space
b a c k w a r d t h r e a t e n i n g t h e airway.
• Laryngeal e d e m a m a y appear, if it spreads t o p a r a p h a r y n g e a l Mylohyoid
Submandibular- muscle
or r e t r o p h a r y n g e a l space. gland Submaxillary
space
Treatment

Anterior belly of
• digastric
• Systemic antibiotics Fig. 7.3: A n a t o m y o f s u b m a n d i b u l a r space
• I a n d D s h o u l d be p o s t p o n e d as l o n g as possible because pus -

is s e l d o m f o u n d .
• T r a c h e o s t o m y is r e q u i r e d if airway is e n d a n g e r e d .

NOTE

If incision and drainage for Ludwig's angina is done 4- GA-there are increased chances of aspiration and shock as tongue is pushed up and back in
Ludwig angina.

4 2 . A n s is d i.e. E d e m a of u v u l a Ref. Scott Browns 6th/ed pp 4,5,10

Quincke Disease

• A c u t e e d e m a o f t h e uvula is called as Quincke's disease.


• Etiology is u n k n o w n ; b u t it is related t o
(a) Allergy
Other causes include
(b) Trauma (foreign body, iatrogenic)
r
(c) Infection
- Viral pharyngitis - Candidiasis
- Syphilis - TB
(d) Tumors = Squamous cell carcinoma
• Clinical f e a t u r e s : Trickling or irritating sensation in t h e t h r o a t t o g e t h e r w i t h sensation o f g a g g i n g
• T r e a t m e n t : Edema usually settles down spontaneously. IV hydrocortisone may help.
- C o l l e c t i o n o f pus in t h e peritonsillar space is k n o w n as Quinsy and not Quincke.
- Recurrent e d e m a o f uvula w i t h occasional laryngeal e d e m a is seen in hereditary a n g i o n e u r o t i c e d e m a (HANE)
4 3 . A n s is d i.e. M a n d i b l e Ref. Dhingra 5th/ed pp 198,199,6th/ed p 185; Mohan Bansal p346

Le Fort's Fractures Involve



Nasal s e p t u m Maxilla

• P t e r y g o i d plates Floor o f o r b i t
- -
• Superior o r b i t a l fissure < Lacrimal b o n e
• Z y g o m a t i c processes ( f r o n t o z y g o m a t i c a n d t e m p o r o z y g o m a t i c )

44 A n s . is c i.e. Le Fort 3 f r a c t u r e Ref. Dhingra; 5th/ed p 199,6th/edp 185; Scott's Brown 7th/ed Vol 2 Chapter 128, p 1623
In Le Fort 3 fracture, t h e r e is c o m p l e t e separation o f facial bones f r o m t h e cranial bones i.e. craniofacial d i s s o c i a t i o n / d y s j u n c t i o n
occurs.
45. A n s . is a i.e. F r a c t u r e of floor of orbit Ref. Dhingra Sth/ed p 198,6th/edp 184
As discussed in t h e o r y s e c t i o n - T e a r D r o p " s i g n is a radiological sign seen in b l o w o u t fracture o f o r b i t . It signifies e n t r a p m e n t a n d
h e r n i a t i o n o f o r b i t a l c o n t e n t t h r o u g h a defect in f l o o r o f o r b i t i n t o maxillary a n t r u m .
4 6 . A n s . is a, b, c, d a n d e i.e. C h e c k s w e l l i n g ; t r i s m u s ; n o s e b l e e d i n g ; Infraorbital n u m b n e s s a n d d i p l o p i a .
4 7 . A n s . is b i . e . C S F r h i n o r r h e a
4 8 . A n s is d i.e. Z y g o m a Ref. Dhingra 5th/ed p 197,6th/ed p 183; Mohan Bansal p 344
CHAPTER 7 Oral Cavity J 91

C l i n i c a l F e a t u e r s o f Z y g o m a F r a c t u r e : ( a l s o k/a T r i p o d F r a c t u r e )

. Flattening o f m a a r prominence
. Swelling o f cheeks
. Ecchymosis o f lower eyelids
• U n i l a t e r a l epistaxis
. N u m b n e s s over infraorbital p a r t o f face
. D i p l o p i a a n d restricted o c u l a r m o v e m e n t s
• T r i s m u s d u e to d e p r e s s i o n o f z y g o m a o n u n d e r l y i n g c o r o n o i d process
• Periorbital e m p h y s e m a d u e t o escape o f air f r o m t h e maxillary sinus o n nose b l o w i n g — D h i n g r a , 6th/edp 183
• Step d e f o r m i t y o f infraorbital m a r g i n . —Dhingra, 6th/edp 183

NOTE •

• After nasal bones, zygoma is the second most frequently fractured bone
The fracture and displacement can best be viewed by water's view
• T/t - only displaced fractures require o p e n reduction and inletral wire fixation.

4 9 . A n s . is d i.e. S u r g i c a l e m p h y s e m a Ref. Dhingra 5th/ed p 199,6th/edp 185;Tuli 1st/edp 201; Mohan Bansal p 344
Fracture of maxilla as we have already discussed is classified as Le Fort l/Le Fort ll/Lefort III.

Clinical F e a t u r e s of M a x i l l a — C o m m o n to All T y p e s

• Malocclusion of teeth • Elongation o f m i d face


• U n d u e m o b i l i t y o f maxilla

Specific Clinical Features


• CSF r h i n o r r h e a is seen in Le Fort II a n d Le Fort III f r a c t u r e as c r i b r i f o r m plate is injured.
• Injury t o i n f r a o r b i t a l nerve is seen in Le Fort II fracture. -Tuli Ist/ed p201
• So anesthesia w i l l be seen in area o f s u p p l y o f infraorbital nerve i n j u r y viz. cheek a n d u p p e r lip (area o f s u p p l y o f i n f r a o r b i t a l
nerve). - BDC4th/edp 118
5 0 . A n s . is b i.e. C r i b r i f o r m p l a t e of e t h m o i d b o n e Ref. Logan and Turner lOth/edp 28;
5 1 . A n s . is a i.e. E t h m o i d s i n u s Scott Brown 7th/ed Vol2 pp 1636-1639
Historically, m o s t c o m m o n cause o f CSF r h i n o r r h e a was head i n j u r y w i t h i n v o l v e m e n t o f c r i b r i f o r m plate o f e t h m o i d b o n e h o w e v e r
N o w m o s t c o m m o n c a u s e of C S F r h i n o r r h e a is i a t r o g e n i c t r a u m a / s u r g e r y

NOTE

According to Logan and Turner lOth/ed p 28

• M o s t c o m m o n area o f f r a c t u r e o f CSF r h i n o r r h e a is t h e c r i b r i f o r m plate o f t h e e t h m o i d b o n e as it is e x t r e m e l y t h i n


• O t h e r possible areas are -
• Posterior wall o f t h e f r o n t a l sinus
• Floor o f a n t e r i o r cranial fossa
- In t h e previous e d i t i o n o f Scott Brown - i t was g i v e n m o s t c o m m o n site o f leak in CSF r h i n o r r h e a is - roof of e t h m o i d s i n u s
> cribriform plate > s p h e n o i d s i n u s
- But n o w in latest e d i t i o n it is n o t g i v e n .
5 2 . A n s . is c a n d d i.e. N a s o e t h m o i d f r a c t u r e ; a n d F r o n t o z y g o m a t i c f r a c t u r e
Ref. Logan and Turner Wth/edp 28; Dhingra 5th/edp 199,6th/edp 182; Mohan Bansal p 348

C S F R h i n o r r h e a O c c u r s in

Fracture o f maxilla in Le Fort t y p e II a n d t y p e III. (as c r i b r i f o r m plate is i n j u r e d here) and also in nasal f r a c t u r e class III
5 3 . A n s . is a , b a n d d i.e. O c c u r s d u e to b r e a k in cribriform p l a t e ; C o n t a i n s g l u c o s e a n d ; C o n t a i n s less p r o t e i n
Ref. Turner lOth/edp 28; Dhingra 5th/edp 178,6th/edp 163-165

Let us see e a c h option Separately

• O p t i o n a - Occurs d u e t o break in c r i b r i f o r m plate. This is c o r r e c t


• O p t i o n b - CSF contains glucose a n d o p t i o n d It has less p r o t e i n s
• In c o m p a r i s o n t o nasal secretions - CSF contains m o r e o f glucose a n d less o f proteins (Turner lOth/edp 28) hence b o t h o p t i o n s
b a n d d are correct
SECTION II Oral Cavity

• O p t i o n c - Requires i m m e d i a t e surgery
• This is n o t absolutely correct as:
- Early cases o f post t r a u m a t i c CSF r h i n o r r h e a are m a n a g e d conservatively. Only those cases w h e r e CSF r h i n o r r h e a occurs
persistently
- Surgical management should be done
5 4 . A n s . is a i.e. Antibiotics a n d O b s e r v a t i o n Ref. Dhingra 5th/edp 179,6th/edp 164
• E a r l y c a s e s o f p o s t t r a u m a t i c CSF r h i n o r r h e a are m a n a g e d conservatively (by placing t h e p a t i e n t in p r o p p e d u p p o s i t i o n ,
a v o i d i n g b l o w i n g o f nose, sneezing a n d straining) a n d
. Prophylactic antibiotics (to p r e v e n t m e n i n g i t i s ) .
• P e r s i s t e n t c a s e s are t r e a t e d surgically by nasal e n d o s c o p y or by intracranial route.
A c c o r d i n g t o Scott-Brown's 7th/ed Vol 2 p 1 6 4 1 -
E n d o s c o p i c closure of C S F l e a k is now t h e t r e a t m e n t of choice in majority of patients b u t it should n o t be d o n e immediately,
first p a t i e n t should be subjected t o diagnostic evaluation a n d after site o f leakage is c o n f i r m e d , it s h o u l d be closed endoscopically.
5 5 . A n s . is b i.e. Beta-2 t r a n s f e r r i n
5 6 . A n s . is d i.e. Beta-2 t r a n s f e r r i n
Ref. Scott-Brown's Otolaryngology 7th/ed Vol 2, Chapter 129 p 1638; Mohan Bansal p 348; Dhingra, 6th/ed, p 164 Table 29.1
• The o n l y test t h a t s h o u l d be used t o d e t e r m i n e if a sample is CSF or n o t , is i m m u n o f i x a t i o n of beta-2 transferin.
• Beta-2 transferrin is a p r o t e i n i n v o l v e d in ferrous i o n t r a n s p o r t a n d is f o u n d in CSF, p e r i l y m p h a n d aqueous h u m o r
• The sensitivity o f t h e test is 1 0 0 % a n d specificity 9 5 %
• There are certain c o n d i t i o n s w h i c h can cause a b n o r m a l transferrin m e t a b o l i s m a n d t h u s p f o r m a t i o n in b l o o d w h i c h c o u l d
2

p o t e n t i a l l y lead t o false-positive result:

These c o n d i t i o n s are:
(a) Chronic liver disease
(b) I n b o r n errors o f g l y c o g e n m e t a b o l i s m
(c) Genetic v a r i a n t f o r m o f transferrin
(d) Neuropsychiatric disease
(e) Rectal c a r c i n o m a

For this reason, some a u t h o r s r e c o m m e n d t a k i n g a s i m u l t a n e o u s b l o o d sample t o exclude t h i s possible source o f error.

ALSO KNOW

• I m a g i n g m o d a l i t y o f choice t o d e t e c t t h e site o f leak in CSF r h i n o r r h e a is T 2 w e i g h t e d MRI


• H i g h r e s o l u t i o n CT can d e t e c t CSF r h i n o r r h e a in u p t o 8 4 % cases b u t its result s h o u l d be i n t e r p r e t e d w i t h c a u t i o n , as if t h e r e is/
has been a previous skull base surgery it w i l l a l m o s t i n e v i t a b l y s h o w a large defect in absence o f a t r u e leak.
• Historically, m a n y dyes ( m e t h y l e n e b l u e , i n d i g o c a r m i n e ) w e r e used f o r diagnosis o f CSF r h i n o r h e a b u t in recent t i m e o n l y
fluorescein is b e i n g used. It is used in cases w h e r e site o f leak is u n c e r t a i n or there is t h e possibility o f m o r e t h a n o n e defect.
5 7 . A n s . is c i.e. S u r g i c a l d r a i n a g e Ref. Tuli 1st/ed p 148; Current Otolaryngology 2nd/ed pp 252,253
• The p a t i e n t in t h e q u e s t i o n h a d fall f r o m b e d f o l l o w i n g w h i c h t h e r e is a s w e l l i n g in nose a n d s l i g h t d i f f i c u l t y in b r e a t h i n g .
• This p a t i e n t has p r o b a b l y h a d septal h a e m a t o m a w h i c h s h o u l d be d r a i n e d i m m e d i a t e l y u n d e r LA.
• For details o f septal hematoma-Ref. t o t h e Chapter-Diseases o f Nasal S e p t u m .
5 8 . A n s . is b i.e. after few d a y s Ref. Scott's Brown 7th/ed chapter 127 Vol 2 p 1612; Dhingra 6th/ed p 182; Tuli 2nd/ed p 208

M a n a g e m e n t Protocol for Nasal Fractures/Injuries

• M o s t o f t h e patients ( ~ 7 0 - 8 0 % ) d o n o t require any active t r e a t m e n t , as m a n y d o n o t have a nasal fracture a n d t h o s e t h a t d o ,


t h e fracture is n o t displaced.
- Soft tissue swelling can p r o d u c e t h e m i s l e a d i n g appearance o f a d e f o r m i t y w h i c h disappears as t h e s w e l l i n g subsides. Such
p a t i e n t s require o n l y reassurance a n d t o p i c a l vasoconstrictors t o alleviate c o n g e s t i o n a n d o b s t r u c t i v e s y m p t o m s . A re-
e x a m i n a t i o n s h o u l d be carried o u t after 5 days, if t h e r e is u n c e r t a i n t y a b o u t t h e need f o r r e d u c t i o n .
• I m m e d i a t e surgical i n t e r v e n t i o n in acute phase is required in case o f cosmetic d e f o r m i t y a n d nasal o b s t r u c t i o n caused by septal
hematoma
• For rest o f t h e cases t h e o p t i m a l t i m e f o r clinical assessment is a r o u n d 5 days, by w h i c h t i m e t h e e d e m a w i l l have subsided
a n d any u n d e r l y i n g d e f o r m i t y apparent. Review at 5 days allows sensible p l a n n i n g f o r r e d u c t i o n o f t h e fracture o n an elective
o p e r a t i n g list w i t h i n t h e n e x t 2-3 days.
By 7 days t h e b o n y d e f o r m i t y w i l l be easily palpable a n d still m o v a b l e . Further delay makes effective r e d u c t i o n less likely a n d
s o m e t i m e s i m p o s s i b l e w i t h o u t m a k i n g o s t e o t o m i e s . T h u s best t i m e t o reduce fracture o f nasal b o n e is b e t w e e n 5 a n d 7 days. In
c h i l d r e n , h e a l i n g can take place even m o r e q u i c k l y a n d earlier i n t e r v e n t i o n is i n d i c a t e d .
CHAPTER 7 Oral Cavity

59. Ans. is d i.e. Ludwig's angina Ref. Dhingra 5th/ed p 274; Harrison 17th/edp 210; Mohan Bansal p 544

M e m b r a n e in T h r o a t is C a u s e d b y

• Pyogenic organisms viz. Streptococci, Staphylococci causing membranous tonsillitis


• Diphtheria
• Vincent's angina (Caused by fusiform bacilli and spirochetes: Borrelia vincentii)
• Candidiasis/monoliasis/oral thrush —Maqbool llth/edp280
• Infectious mononucleosis
• Agranulocytosis
• Leukemia
• Aphthous ulcers
• Traumatic ulcers

From the above list it is clear that streptococcus (option 'a'). i d diphtheria (Option 'b') causes membrane over throat.
This leaves us w i t h 2 options—Adenovirus and Ludwigs angina
Harrison 17th/ed, p210 says about Adenovirus pharyngitis:
"Since pharyngeal exudate may be present on examination, this condition is difficult to differentiate from streptococcal pharyngitis."
So adenovirus may also be associated w i t h membrane in throat b u t Ludwig's angina is infection o f t h e submandibular space and
never presents w i t h membrane over the tonsil/throat.
So amongst the given o p t i o n s — L u d w i g s angina is the best option.

60. Ans. is c i.e. Vincent's angina Ref. Logan Turner 10th/edpp 87,88

Vincent's A n g i n a : (Ulcerative Gingivitis/Trench mouth)


• Was c o m m o n during first w o r l d war (due t o lack of oral hygiene) and is less c o m m o n now.
• Caused by fusiform bacillus and spirochetes: Borrelia vincentii.

• It manifests as necrotizing gingivostomatitis w i t h oropharyngeal ulcerations and dark gray membrane.

O/E
• Membrane generally present on one tonsil but may involve the g u m soft, and hard palate.
• It appears as grayish black slough which bleeds w h e n it is removed.
• Ulcers are visible on tonsil after removal of membrane.
• Membrane reforms after removal.

• Characteristic smell in breath (halitosis), so also called as Trench m o u t h .

Treatment
• Systemic antibiotics: Penicillin, Erythromycin, Metronidazole.
• Warm sodium bicarbonate gargles.
• Barrier nursing o f t h e patient as disease is infectious.
61. Ans. is c i.e. Ulcerative lesions of tonsil Ref. Turner Wth/ed pp 87,88
Trench mouth/Vincent's angina is ulcerative gingivostomatitis.
62. Ans. is c i.e. Firmly attached and bleeds on removal Ref. Dhingra 5th/ed pp 308,309,6th/edp 260
• In diphtheria: membrane is dirty grey in color.
• It extends beyond the tonsils, on t o the soft palate and posterior pharyngeal wall.
• It is adherent and its removal leaves a bleeding surface.
• Cervical l y m p h nodes particularly t h e jugulodigastric l y m p h node are enlarged and become tender, giving a bull neck
v
appearance
6 3 . Ans is d i.e. 1 molar
st
Ref. Dhingra 5th/ed 200
64. Ans is d i.e 8 0 % of cases Ref. Bailey and Love 24th/ed p 723; 25th/ed p 755
8 0 % of all salivary stones occur in the submanidbular glands because their secretions are highly viscous. 8 0 % of submandibular
stones are radiopaque and can be identified on plain radiograph.
6 5 . Ans is c i.e. Endoscopic repair Ref. Dhingra 6th/edp 164
As discussed is preceeding text. CSF rhinorrhea can be managed by
SECTION II Oral Cavity

I 1

Conservative approach Surgical repair: In persistent cases surgical repair is performed by


• Bed rest • Neuro surgical intracranial approach
• Elevating head of bed • Endoscopic repair
• Stool softeners • Extradural approach
• Avoidance of sneezing/straining activities

66. Ans is c i.e. Alveobuccal complex Ref. ASI Ist/edp 348


Repeat
67. Ans is d i.e. Horizontal Ref. Login Turner Wth/edp 21
In j a r y a w y fracture:The fracture line is horizontal
68. Ans is d i . e . zygoma Ref. Dhingra 6th/ed p 183
Expl: Fracture o f zygoma is called as tripod fracture as w h e n the bone fractures, it is separated at its three processes viz zygoma-
ticofrontal, zygomatico temporal and infraorbital.
68. Ans is c i.e. Blow out fracture Ref. Dhingra 6th/edp 184
Blow o u t fracture is isolated fracture of orbital floor, w h e n a large blunt object strikes the globe.
It presents w i t h :
• Ecchymosis of lid, conjunctiv a and sclera
• Enophthalmos
• Diplopia due t o displacement of eye ball
• Anaesth esia of cheek and upper lip, if infraorbital nerve is involved.


PHARYNX

Anatomy of Pharynx, Tonsils and Adenoids


Head and Neck Space Inflammation
Lesions of Nasopharynx and Hypopharynx including Tumors of Pharynx
Hot Topics

CHAPTER
Anatomy of Pharynx,
Tonsils and Adenoids

PHARYNX • Lateral Wall - Pharyngeal opening of Eustachian tube


is situated 1.25 cm behind the posterior
Pharynx extends f r o m the base of skull t o lower border of cricoid end of inferior turbinate." It is b o u n d e d
cartilage. Its length is 12-14 cm and w i d t h is 3.5 cm at base t o 1.5 above and behind by an elevation k/a
cm at pharyngoesophageal junction, which is the narrowest part torus tubaris, behind which is a recess
of digestive tract (apart from appendix).
called fossa ofRosenmuller. 9
Anatomically pharynx is divided into 3 parts (Fig. 8.1):
• Nasopharynx Contents of Nasopharynx
• Oropharynx
• Adenoids/Nasopharyngeal tonsil: Subepithelial collection
• Hypopharynx/Laryngopharynx of l y m p h o i d tissue at the junction of roof and posterior wall
of nasopharynx (Fig. 8.2). Q

Nasopharynx
• N a s o p h a r y n g e a l B u r s a : Epithelial l i n e d m e d i a n recess
Shape and Boundaries e x t e n d i n g f r o m p h a r y n g e a l mucosa t o t h e p e r i o s t e u m
• Extends f r o m base of skull t o a plane passing t h r o u g h hard of b a s i o c c i p u t . Represents a t t a c h m e n t o f n o t o c h o r d t o
palate or soft palate i.e. C2 level. pharyngeal endoderm d u r i n g embryonic life. Abscess of this
• Upper chamber - Large rectangular or oval shape bursa is called asThornwald's disease."
Lower chamber Tubular • Rathke pouch: Reminiscent of buccal mucosal invagination t o
Roof Basisphenoid and basioccipital form the anterior lobe of pituitary. Represented by a dimple above
Posterior wall C, vertebrae
adenoids. A craniopharyngioma may arise from Rathke pouch.
Floor Soft palate anteriorly
Nasopharyngeal isthmus posteriorly
Anterior wall Choanae

Eustachian tube
Sinus of
Morgagni

Mucosa
Pharyngo-
basilar fascia

Superior constrictor

Middle constrictor

Inferior constrictor

Fig. 8.2: Pharyngeal wall (Coronal section)


Courtesy: Textbook of Diseases of Ear, Nose and Throat,
• Mohan Bansal Jaypee Brothers, p 50
8.1: Anatomy of pharynx
98 ]_ SECTION III Pharynx

Sinus of Morgagni: Space between base o f skull and upper Epiglottis Uvula of soft palate
border of superior constrictor muscle (Fig. 8.2).
Base of tongue

Structures passing through this space Aryepiglottic fold


Cuneiform tubercle
> Eustachian tube
1
Levator palate muscle Corniculate tubercle
' Ascending palatine artery Posterior surface over
arytenoid cartilage
Pyriform sinus
P a s s a v a n t ' s r i d g e : I t is a n e l e v a t i o n f o r m e d b y f i b e r s o f lateral wall
palatopharyngeus and superior constrictor. Soft palate makes firm
Pyriform sinus
contact w i t h this ridge t o cut off nasopharynx f r o m oropharynx medial wall
during deglutition and speech. Cricoid cartilage
prominence.
Epithelial lining of Nasopharynx Cricoid cartilage
lower border
• Functionally nasopharynx is the posterior extension o f nasal
cavity so it is lined by pseudostratified ciliated columnar epi- Esophagus
thelium.

Oropharynx Fig. 8.3: Structures of hypopharynx. Posteriorview of laryngopharynx.


Courtesy: Textbook of Diseases of Ear, Nose and Throat,
Extends f r o m hard palate above t o hyoid bone below: Mohan Bansal. Jaypee Brothers, p 56
Boundaries (Fig. 8.2)
Posterior pharyngeal wall lying o p - is referred to ear in carci-noma
Posterior Wall
posite C2 and C3 pyriform sinus via this nerve.
Postcricoid region: Lies between upper and lower
Anterior Wall a. Base o f t o n g u e — p o s t e r i o r t o
border of cricoid lamina
circumvallate papillae
Commonest site o f carcinoma
b. Lingual tonsils
in females suffering from Plum-
c. Valleculae—is a depression lying
mer-Vinson syndrome
between base of t o n g u e and an-
• Posterior pharyngeal wall: - Extendsfrom hyoid bone t o cr
terior surface of epiglottis.
coarytenoid joint.
Lateral Wall a. Palatine (faucial) tonsil
b. Anterior pillar (palatoglossal arch) M u s c l e s of P h a r y n x
" f o r m e d by palatoglossus muscle
Extrinsic Muscle
c. Posterior pillar (palatopharyngeal
a r c h ) f o r m e d by palatopharyn-
Q S u p e r i o r c o n s t r i c t o r : It arises f r o m p t e r y g o i d h a m u l u s ,
geus muscles pterygomandibular ligament and posterior end of myelohyoid line.
Inferior Boundary a. Upper border of epiglottis • Middle constrictor: It is a fan-shaped muscle which arises from
lesser and greater cornu of hyoid bone.
b. Pharyngoepiglottic folds
• Inferior constrictor: It has t w o parts:
Upper part i.e. thyropharyhgeus with oblique fibers arising
H y p o p h a r y n x / L a r y n g o p h a r y n x (Lower part of P h a r y n x )
f r o m oblique line of thyroid cartilage.
Lies between body of hyoid t o lower border of cricoid cartilage, Lower part i.e. cricopharyngeus arises from lateral side of
opposite 3,4,5, and 6 cervical vertebrae. cricoid cartilage and transverse fibers f r o m cricopharyn-
Subdivided into three regions (Fig. 8.3): geal sphincter. Constrictor muscles are supplied t h r o u g h
• Pyriform sinus (fossa)—Bounded by: pharyngeal plexus. Inferior constrictor is suplied by recur-
Superiorly Pharyngoepiglottic folds rent laryngeal nerve. Killian's dehiscence is a gap be-
tween oblique and transverse fibers of inferior constrictor.
Interiorly Lower border of cricoid
Laterally Thyrohyoid membrane and
Intrinsic Muscles
thyroid cartilage
Medially Aryepiglotticfold. • Staphylopharyngeus
Posterolateral surface of aryt- • Salpingopharyngeus
enoids and cricoid cartilages • Palatopharyngeus

Importance Forms lateral channel for f o o d


Foreign bodies may lodge here Arterial S u p p l y of P h a r y n x
Internal laryngeal nerve runs • Ascending pharyngeal branch o f external c a r o t i d , artery.
submucosally here thus easily ac- Ascending palatine branch of facial artery (branch of external
cessible for anesthesia and pain carotid), greater palatine branch of maxillary artery.
CHAPTER 8 Anatomy of Pharynx, Tonsils and Adenoids 99
• Venous drainage is t h r o u g h pharyngeal plexus into internal Peritonsillar vein in
Pharyngobasilar fascia loose areolar tissue
jugular vein.
Superior constrictor muscle
Nerve Supply Tonsillar capsule
Buccopharyngeal fascia l
Soft palate
It is by pharyngeal plexus of nerves which is formed by: Crypta magna
• Branch of vagus (Xth nerve)/Motor supply. Medial pterygoid muscle
Primary crypt
• Branches of glossopharyngeal (IXth nervej/Sensory supply.
Secondary crypt
• Sympathetic plexus.
Tonsil
Lymphatic Drainage of Pharynx
Facial artery
• Nasopharynx
Styloglossus Tongue
Nasopharynx drains i n t o u p p e r deep cervical nodes
Submandibular salivary gland
either directly or indirectly t h r o u g h retropharyngeal or
parapharyngeal nodes. Fig. 8.4: Bed of tonsil
Nasopharynx also drain into spinal accessory chain o f Courtesy: Textbook of Diseases of Ear, Nose and
nodes in t h e posterior triangle of the neck. Throat, Mohan Bansal. Jaypee Brothers, p 54

3
Medial surface o f each tonsil has 15-20 crypts, t h e largest
of which is called Intratonsillar cleft or crypta magna (which
Rouviere's node represents persistence of the ventral p o r t i o n of t h e second
This most superior node of the lateral group of retropharyngeal pharyngeal pouch).
Tonsillar bed (Fig. 8.4) is formed f r o m w i t h i n — o u t w a r d by:
\^ lymph nodes. , ... _„^„_ _.,
Pharyngobasilar fascia
m M

• Oropharynx Superior constrictor (above) and palatopharyngeus muscle


Styloglossus (below)
Lymphatics f r o m the oropharynx drain into upper jugular
Buccopharyngeal fascia
particularly the jugulodigastric (tonsillar) nodes.
The soft palate, lateral and posterior pharyngeal walls and
the base o f t o n g u e also drain into retropharyngeal and Important Relationships
parapharymgeal nodes and f r o m there t o the j u g u l o d i - 1
The styloid process lies is relation to lower part of tonsillar fossa,
gastric and posterior cervical group. Note: Lymphatics of therefore, a hard elongated swelling felt in the posterior wall
base of t o n g u e drain bilaterally. of tonsil may be on enlarged styloid and tonsillectomy is the
approach for excision of elongated styloid.
• Hyphopharynx
1
Glossopharyngeal nerve lies in relation to posterior pole of
Pyriform sinus drains into upper jugular chain a n d
tonsil which is the cause of earache in peritonsillar abscess and
then to deep cervical group of lymph nodes. Note: Py- after tonsillectomy
riform fossa have rich lymphatic network and carcinoma > Internal carotid artery lies lateral to tonsil so aneurysm of
of this region has high frequency o f nodal metastasis. Internal Cartdid Artery can cause pulsatile tonsil
Postcricoid region drains into parapharyngeal and para-
tracheal group of lymph nodes. Nerve Supply
Posterior pharyngeal wall drains into parapharyngeal • By the tonsillar branch o f t h e 9th nerve.
lymph nodes and finally t o deep cervical lymph nodes. • Upper part of the tonsil is supplied by: Lesser palatine Nerve.

Waldeyer's Ring Blood Supply


• It is a group of lymphatic organs guarding the oropharynx and • Tonsillar branch of facial artery (main source) and is the most
nasopharynx in the f o r m of a ring. common arterial cause of bleeding during tonsillectomy. 0

The ring is bounded above by pharyngeal tonsil (adenoids) and • Ascending palatine artery (branch of facial artery.)
tubal tonsil, below by lingual tonsil and on left and right side • Dorsal lingual branch of lingual artery.
by palatine tonsils and lateral plaryngeal bands. • Greater/descending palatine branch of maxillary artery.
• Tonsillar branch of ascending pharyngeal artery (Fig. 8.5).

| PALATINE TONSIL Venous Drainage

• Palatine tonsil is specialized subepithelial lymphoid tissue situ- Paratonsillar vein: Also called as Tonsillar vein
ated in tonsillar sinus on the lateral wall of oropharynx.
Lymphatic Drainge
• It is almond shaped.
• Tonsillar fossa is bounded by palatoglossal fold in front and Jugulodigastric lymph nodes.
palatopharyngeal fold behind.
• Tonsils are l i n e d by: Non-keratinized stratified squamous Development
epithelium.
0
Tonsils develop from ventral part of second pharyngeal pouch.


iooT SECTION III Pharynx

Signs
Superficial Maxillary artery
- Inflammed tonsils, pillars, soft palate, uvula
temporal artery -Descending
palatine artery - Tongue may be red i.e. strawberry t o n g u e

-Crypta magna - Bilateral jugulodigastric lymph nodes are enlarged and tender. 0

Tonsillar branch
Secondary crypt Diagnosis
of ascending- y
pharyngeal artery -Primary crypt • Pus can be squeezed f r o m the crypts of tonsils
• Throat culture w i t h blood agar plate
Ascending-
palatine artery
Treatment
Tonsillar artery-
• Antibiotics: Crystalline pencillin for 7-10 days. 0

• Analgesics
Facial artery-

External- Complication
-Dorsal lingual branches of •
carotid artery
lingual artery
HP nemonic
Fig. 8.5: Blood supply and crypts of tonsil ORA (N)TGE
Courtesy: Textbook of Diseases of Ear, Nose and O - Acute otitis media
Throat, Mohan Bansal. Jaypee Brothers, p 55
R - Rheumatic fever and scarlet fever
A - Abscess:
DISEASES OF TONSIL - Peritonsillar
- Parapharyngeal
| ACUTE TONSILLITIS - Cervical
(N)T - Chronictonsillitis/Chronicadenotonsillarhypertrophy
Most commonly seen in school going children but can be seen in G - Glomerulonephritis (Post streptococcal)
adults. E - Subacute bacterial endocarditis

Microbiology
• Most c o m m o n : Group B-hemolytic streptococcus (GABHS) 0

• Others: Staphylococcus, Haemophilus, and Pneum coccus. Recently, a temporal association between pharyngotonsillitis
• Viral causes: Adenovirus > Ebstein-Barr virus > Influenza virus induced by group A, B-hemolytic streptococci and a new set of
obsessive compulsive disorders (OCDs) and Other tics has been
T y p e s of Tonsillitis recognized. This has been called as PANDAS (Pediatric Autoimmune
The components o f a normal tonsil are: Neuropsychiatric Disorder associated with Streptococcal infection)
• Surface epithelium (continuous w i t h oropharyngeal lining)


Crypts
Lymphoid tissue
^
Differential Diagnosis of Membrane Over the Tonsil
Thus tonsillitis is classified depending on the component involved: 1
Trauma
• Acute catarrhal or superficial tonsillitis:Tonsillitis is a part of 1
Tumors of tonsil and aphthous ulcer
generalized pharyngitis and is mostly seen in viral infections. • Infections: Candidal Infection
• Acute follicular tonsillitis: Infection spreads into the crypts Diphtheria
w h i c h become filled w i t h purulent material, presenting at the Tonsillitis—membranous
• Forrest
openings of crypts as yellowish spots.
• Acute membranous tonsillitis: It is a stage ahead o f acute VIAL Vincent angina (Caused by fusiform bacilli and
follicular tonsillitis w h e n exudation f r o m the crypts coalesces Borrelia vincentii)
t o f o r m a membrane on the surface of tonsil. Infectious mononucleosis
• Acute parenchymatous tonsillitis: Here t h e substance o f A - Agranulocytosis
tonsil is affected. Tonsil is uniformly enlarged and red. L - Leukemia

Prodromal Symptoms

• Fever, headche, malaise, general bodyache
• In acute phase—sore throat Pyogenic membranous tonsillitis is caused by streptococci,
• Dysphagia, earache, trismus staphylococci)
. Foul breath w i t h coated tongue
CHAPTER 8 Anatomy of Pharynx, Tonsils and Adenoids

T o n s i l l e c t o m y ( T a b l e 8.1) Commonest site: Upper pole of tonsil.


Etiology: Generally occurs as a complication of acute tonsillitis,
Indications
but may arise denovo w i t h o u t a preceding history of tonsillitis.
A. Tonsillar Indications (Table 8.1) It is generally unilateral.
Table 8.1: Indications for tonsillectomy Age group: Young adults between 20 and 39 years of age.
Children rarely affected.
Tonsillectomy
Organisms: Mixed flora (anaerobes and aerobes)/Groups A
Absolute Indications Relative Indications beta-hemolytic streptococcus
• Huge hypertrophic tonsil • Recurrent tonsillitis
causing oropharyngeal Clinical Features
obstruction • High-grade fever w i t h chills and rigor
• Suspected malignancy of • 2nd attack of Quinsy • Unilateral throat pain
tonsil • Chronic Tonsillitis • Ipsilateral referred Odynophagia cranial nerve IX supplies tonsil
• Tonsillitis causing febrile seizures as well as ear
• Tonsillitis in a cardiac valvular • Hot potato voice/Plummy voice
disease patient • Ipsilateral earache (referred pain via IXth cranial nerve)
• Long-term management of IgA
• Foul breath
nephropathy
• Trismus (due t o spasm of pterygoid muscles w h i c h are in close
• Severe infectious mononucleosis
proximity t o superior constrictor)
with upper airway obstruction

On Examination
Criteria for Recurrent Tonsillitis - Tonsils, pillars and soft palate are congested and swollen o n
1
Sore throat should be due to tonsillitis the involved site.
• Five or more episodes of tonsillitis per year - Uvula is swollen and pushed t o opposite side.
> Symptoms for at least 1 year - Mucopus covering tonsillar area
• The episodes should be disabling which prevent normal - Cervical l y m p h nodes are enlarged
functioning - Torticollis: patient keeps neck tilted t o side of abscess.

B. Non-tonsillar Indications for Tonsillectomy


Treatment
As an approach for elongated styloid process and glossopharyngeal
nerve, c o m p l e t e excision o f branchial fistula and as a part o f Hospitalization
uvulopalatopharyngoplasty in obstructive sleep apnea. • IV fluids, antibiotics analgesics
• I and D: If there is bulging of soft palate or if adequate response
C o n t r a i n d i c a t i o n ( T a b l e 8.2) is not seen w i t h i n 24 hours o f t h e antibiotic therapy.
Table 8.2: Contraindication of tonsillectomy • Interval tonsillectomy: Tonsillectomy done after 6 weeks
of quinsy.
Absolute Others • Hot tonsillectomy/abscess tonsillectomy: Tonsillectomy
• Polio epidemic • Acute tonsillar infection performed in the acute stage
• Submucous cleftpalate • Age < 3 years
• Recent acute upper respiratory
| ADENOIDS (LUSCHKA TONSIL)
tract injection

Important Points on Tonsillectomy o Adenoids are nasopharyngeal tonsils, situated at the j u n c t i o n


of roof and posterior wall o f t h e nasopharynx.
• Position of patient during tonsillectomy: Rose position:
• M e t h o d of p e r f o r m i n g t o n s i l l e c t o m y : Dissection a n d • They are present at birth, enlarge up t o 6 years of age and then
snaring m e t h o d atrophy and completely disappear by 20 years o f age.
• M/C complication of tonsillectomy: Hemorrhage • Unlike palatine tonsils, they have no crypts and no capsule
• Average blood loss during tonsillectomy: 50 t o 80 ml and are lined by pseudo-stratified ciliated columar epithelium
• Average blood loss during Adenoidectomy: 80 t o 120 ml (stratified squamous in Tonsil).
• M/C cause of bleeding during tonsillectomy: Paratonsillar • Not visible on X-ray in infants < 1 m o n t h of age. Clinically seen
vein (Dennis Browne vein) by the 4 t h m o n t h .
• M/C arterial cause of bleeding during tonsillectomy->Tonsillar
branch of facial artery (called as artery of tonsillar hemorrhage)
Blood Supply

| PERITONSILLAR ABSCESS (QUINSY) Adenoids receive blood supply from:


• Ascending palatine branch of facial pharyngeal branch o f
It is collection o f pus between the fibrous capsule of the tonsil,
external carotid.
usually at the upper pole and the superior constrictor muscles of
the pharynx: o Pharyngeal branch of the third part of maxillary artery
102^ SECTION I Pharynx

• Ascending cervical branch o f inferior thyroid artery o f thyro Contd...


cervical trunk
Nasal S y m p t o m s Aural Symptoms General
Lymphatica Drainage Symptoms/
A d e n o i d facies
Is into upper jugular nodes directly or indirectly via retro pharyngeal
• High arched
and parapharyngeal nodes.
palate
Nerve Supply • Systemic
symptoms
Through CN IX and X (It is also responsible for referred pain to
- Pulmonary
ear d u e to adenoiditis)
- Hypertension

Differences b e t w e e n Palatine Tonsils a n d A d e n o i d s


Diagnosis
Adenoids Palatine Tonsils
Soft tissue lateral radiograph reveals size of adenoid (CT has no
Number Single One on each side role in diagnosis).
Site Nasopharynx Tonsillar fossa in
oropharynx Treatment

Crypts or Furrows Only furrows Only crypts Adenoidectomy


Capsule Absent Present • Traditional method -Transoral curettage
Epithelium Ciliated columnar Squamous • Newer method - Endoscopic a d e n o i d e c t o m y w i t h forcep,
stratified suction diathermy and microdebrider

In adults after 20 years of Absent present Indications Contraindication


age
• Obstructive sleep apnea • Submucous cleft of palate (as
it can lead to postoperative
velopharyngeal insufficiency)
DISEASES OF ADENOID
• Glue ear

| ADENOID HYPERTROPHY Relative:


• Snoaring/UARS
Etiology
• Chronic sinusitis (Scott Brown's
Rhinits, Sinusitis, Allergy and tonsilitis 7th/edVol 1 p 1084)

C l i n i c a l S y m p t o m s ( T a b l e 8.3)
For U n d e r g r a d u a t e Students
Table 8.3: Clinical s y m p t o m of adenoid hypertrophy
• Rhinolalia clausa: It is toneless voice with no nasal component.
Nasal S y m p t o m s Aural Symptoms General Causes:
Symptoms/ Adenoid hypertrophy
A d e n o i d facies B/L nasal polyp
Hypertrophic turbinates
•obstruction
B/L Nasal
(M/C
•Conductive hearing
loss due to tubal
•face
Elongated dull
Nasal allergy
Nasopharyngeal angiofibroma.
symptom) obstruction
• Structures passing b e t w e e n u p p e r border of superior
• Wet bubbly nose
• Dull expression constrictor muscle and base of s k u l l —

•Sinusitis •ofRecurrent
acute
attacks
•Open mouth Levater veti palatani
Eustachian t u b e
Ascending palatine artery
• Epistaxis •Otitis media •Crowded
teeth
upper
• Structures passing between superior and middle
constructors—
• Voice change
• CSOM
•Hitched
upper lip
up
Glossopharyngeal nerve
Stylopharyngeus muscle
Voice is
toneless, loses •Serous OM •appearance
Pinched
of
• Structures passing between middle and inferior constrictor
muscle
nasal quality nasal ala Superior laryngeal artery and vein
(Rhinolalia
Internal laryngeal branch of superior laryngeal nerve
clausa) Contd...
CHAPTER 8 Anatomy of Pharynx, Tonsils and Adenoids

QUESTIONS

i. Which ofthe following part is NOT included in hypophar- 12. A 5-year-old patient is scheduled of for tonsillectomy. On
ynxis: [UP 01] the d a y of surgery he had running nose, temperature,
a. Pyriform sinus b. Post cricoid region 37.5°C and dry cough. Which of the following should be
c. Anterior pharyngeal wall d. Posterior pharyngeal wall the most appropriate decision for surgery? [Al 06]
Which of t h e following structures is seen in orophar- a. Surgery should be canceled
ynx? [TN06] b. Can proceed for surgery if chest is clear and there is no
a. Pharyngotympanic tube b. Fossa of Rosenmuller history of asthma
c. Palatine tonsil d. Pyriform fossa c. Should get X-ray chest before proceeding for surgery
3. The lymphatic drainage of pyriform fossa is to:[Delhi 96] d. Cancel surgery for 3 weeks and patient to be on antibiotic
a. Upper deep cervical nodesb. Prelaryngeal node 13. Tonsillectomy: following peritonsillar abscess is done
c. Parapharyngeal nodes d. Mediastinal nodes after weeks: [PGI 97,98]
Killian's dehisence is seen in: [MH 00] a. 1-3 weeks b. 6-8 weeks
a. Oropharynx b. Nosophrynx c. 4-6 weeks d. 8-12 weeks
c. Cricopharynx d. Vocal cords 14. Mostcommonpostoperativecomplicationoftonsilectomy
5. 6-year-old child with recurrent URTI with mouth breath- is: [PGI 85]
ing a n d failure to grow with high arched palate a n d a. Palatal palsy b. Hemorrhage
impaited hearing is: [AIIMS May 07,2012] c. Injury to uvula d. Infection
a. Tonsillectomy 15. Secondary hemorrhage after tonsillectomy develops:
b. Grommet insertion [Aim
c. Myringotomy with grommet insertion a. Within 12 hrs b. Within 24 hrs
d. Adenoidectomy with grommet insertion c. Within 6 days d. Within 1 months
Regarding adenoids true is/are: [PGI 02] 16. Ramu, 15 years of age presents with hemorrhage 5 hours
a. There is failure to thrive after tonsillectomy. Treatment of choice is: [AIIMS 99]
Mouth breathing is seen a. External gauze packing b. Antibiotics and mouth wash
CT scan should be done to assess size c. Irrigation with saline d. Reopen immediately
High-arched palate is present 17 Contraindication of adenotonsillectomy: [PGI 04]
Immediate surgery even for minor symptoms a. A g e < 4 y r b. Poliomyelitis
7. Indication for Adenoidectomy in children include: c. Haemophilus infection d. Upper RTI
[AP00] 18. In which of the following locations, there is collection of
Recurrent respiratory tract infections pus in the quinsy: [AIIMS 04]
Middle ear infection with deafness a. Peritonsillar space b. Parapharyngeal space
Recurrent allergic rhinitis c. Retropnaryngeal space d. Within the tonsil
Multiple adenoids 19. True about quinsy is: [PGI 02]
8. T h e inner Waldeyer's group of lymph nodes does not a. Penicillin is used in treatment
include: [AP 93 test I- General; TN 86,00] b. Abscess is located in capsule
a. Submandibular lymph node c. Commonly occurs bilaterally
b. Tonsils d. Immediate tonsillectomy should be done
c. Lingual tonsils e. Patient presents with toxic features and drooling
d. Adenoids 20. 7-year-old child has peritonsillar abscess presents with
T h e most common organism causing acute tosillitis is: trismus, the best treatment is: [AIIMS 96]
[TN95] a. Immediate abscess drain orally
a. Staph aureus b. Anaerobes b. Drainage externally
c. Hemolytic streptococci d. Pneumococcus c. Systemic antibiotics up to 48 hours then drainage
10. All ofthe following cause a gray-white membrane on the d. Tracheostomy
tonsils, except: [AIIMS May 04]
a. Infectious mononucleosis NEET PATTERN QUESTIONS
b. Ludwig's angina
c. Streptococcal tonsillitis 21. A 6-year-old boy presented to ENT OPD with recurrent
d. Diphtheria URTI, mouth breathing and impaired hearing. The boy
11. Tonsillectomy is indicated in [AI94] w a s d i a g n o s e d a s h a v i n g a d e n o i d h y p e r t r o p h y for
a. Acute tonsillitis which adenoidectomy was done and grommet inserted;
b. Aphthous ulcers in the pharynx 1 week after surgery the boy was again brought to the
c. Rheumatic tonsillitis OPD with torticollis. Which of t h e following are true
d. Physiological enlargement about above clinical scenario. 0
[NEETPattern]
SECTION III Pharynx

a. Atlantoaxial subluxation is the cause for his torticollis b. Tracheostomy


b. The condition is M/C in children with Down's syndrome c. Tonsillectomy
c. Torticollis is not a complication after adenoid surgery and d. Myringoplasty
it is a sheer coincidence
e. Adenoidectomy
d. Adenoidectomy should not have been done in the patient
23. Crypta magna is seen in: [APPG2011]
as adenoids would have spontaneously regressed
a. Nasopharyngeal tonsil b. Tubal tonsil
22. Figure based question
c. Palatine tonsil d. Lingual tonsil
24. The palatine tonsil receives its arterial supply from all
of the following except: [FMGE2013]
a. Tonsillar branch of facial artery
b. Ascending palatine artery
c. Sphenopalatine artery
d. Dorsal lingual artery
25. After tonsillectomy, secondary haemorrhage occurs:
[Neet Pattern]
a. Within 24 hours b. After 2 weeks
c. 5-10 post operative days d. After 1 month
26. Torrential bleed during tonsillectomy is due to:
[Neet Pattern]
a. Facial artery b. Tonsilar artery
Identify the position o f t h e patient during surgery and c. Paratonsillar vein d. None
select the surgeries from the following list w h e r e it is 27. Tonsillectomy is contraindicated in: [DNB 2005]
used: a. Small atrophic tonsils b. Quinsy
a. Submucous Resection of nasel septum c. Poliomyelitis epidemic d. Tonsillolith

EXPLANATIONS AND REFERENCES

1. Ans. is c i.e. Anterior pharyngeal wall Ref. Mohan Bansal p 56; Dhingra 6th/ed p 241
2. Ans. is c i.e. Palatine tonsil Ref. Scott Brown's 7th/ed Vol 2 pp i944,1945; Mohan Bansal p 52; Dhingra 6th/ed p 240

Pharynx is divided into -

Nasopharynx Hypopharynx/Laryngopharynx Oropharynx j


Important contents of Nasopharynx It is further divided into Major structures included in it are:
• Adenoids • Pyriform sinus • Liagual tonsil
• Nasopharyngeal bursa • Postcricoid region • Palatine tonsil
• Rathke pouch • Post pharyngeal wall • Soft palate
• Sinus of Morgagni • Tongue base
• Passavant ridge

3. Ans. is a i.e. Upper deep cervical nodes Ref. Tuli ist/ed pp 231,232; Dhingra 5th/edp 257
• Pyriform sinus drains into upper jugular chain and t h e n t o deep cervical g r o u p o f l y m p h nodes.
• Postcricoid region drains into parapharyngeal and paratracheal group o f l y m p h nodes.
• Posterior pharyngealwall drains into parapharyngeal l y m p h nodes and finally t o deep cervical l y m p h nodes.
4 Ans. is c i.e. Cricophary nx Ref. Scott Brown's 7th/ed Vol 2 Chapter 155 p 2045; Dhingra 5th/ed p 253,6th/ed p 238
\
K i l l i a n ' s D e h i s c e n c e (Fig. 8.6)

• It is an area o f weakness between the t w o parts o f inferior constrictor muscle—sub thyropharyngeus and cricopha-ryngeus
• A pulsion diverticulum of pharyngeal mucosa can emerge posteriorly t h r o u g h the Killian's dehiscence called as Zenker's
diverticulum or pharyngeal pouch.
• Since it is an area of weakness it is one of the sites of esophageal perforation during instrumentation and scopy—hence also
called 'Gateway of Tears'.
CHAPTER 8 Anatomy of Pharynx, Tonsils and Adenoids J 105
Also Know:

Killian-Janieson's space - It lies between cricopharyngeus and circular fibres


o f t h e esophagus.
Thyropharyngeus
Lamier Hackemann's space - It lies between circular and longitudinal fibers
of esophagus.
Killian's
5. Ans. is d i.e. Adenoidectomy with grommet insertion
dehiscence
Ref. Scott Brown 7th/ed Vol 1 p 896-906
Cricopharyngeus
The child is having recurrent URTI w i t h high arched palate and failure t o
grow which indicates child is having adenoids and since there is impaired Circular and
hearing it means child has developed otitis media as a complication. longitudinal
fibers of
Hence logically t h e child should be treated w i t h adenoidectomy w i t h Zenker's-
esophagus
g r o m m e t insertion. This is further supported by following lines f r o m Scott diverticulum 1

Brown.
"Currentpractice is to perform adenoidectomy as an adjunct to the insertion
of ventilation tubes." —Scott Brown's 7th/ed Vol 1, p 902
Fig. 8.6: Inferior constrictor muscle and
6. Ans. is a, b, d i.e. f here is failure to thrive; Mouth breathing is seen, Killian's dehiscence
and High arched palate Ref. Dhingra 5th/ed pp 258,259,6th/ed p 243-244; Logan Turner 1 Oth/ed p 367; Mohan Bansal p 52

Explanation

• High arched palate and m o u t h breathing are features of hypertrophied adenoids which leads t o adenoid facies
• In adenoids as a consequence o f recurrent nasal obstruction and URTI, child develops failure t o thrive
• Size of adenoids may well be assessed using lateral radiograph of nasopharynx, and CT scan is not necessary (Ruling out o p t i o n
c). Surgery is indicated only in hypertrophy causing severe symptoms. (Ruling out o p t i o n e)
7. Ans. is b i.e. Middle ear infections with deafness Ref. Dhingra 5th/edp 442,6th/ed p 131

I n d i c a t i o n s for A d e n o i d e c t o m y

Absolute Relative Indications



• Obstructive sleep apnea • Snoring UARS
• Recurent otitis media with effusion (glue ear) • Chronic sinusitis

There is growing evidence in literature for adenoidectomy as a first-line surgical intervention for chronic rhinosinusitis in children who have failed
maximal medical treatment -Scoff Brown7th/edVol 1 p 1084

8. Ans. a i.e. Submandibular lymph nodes Ref. Current Otolaryngology 2nd/ed pp 340,341; Scott Brown 7th/ed Vol2p1793

Explanation

Submandibular nodes d o not form part of Waldeyer's lymphatic ring.They form part o f t h e outer group of lymph nodes into which
efferents f r o m the constituents of the Waldeyer's lymphatic ring may drain.
Waldeyer ring consists o f (See fig in pictorial Q's at the back):
1. Adenoids (nasopharyngeal tonsil)
2. Tubal tonsil (Fossa of Rosenmuller)
3. Lateral pharyngeal bands
\ 4. Palatine tonsils
5. Nodules (Post pharyngeal wall)
6. Lingua! tonsils
9. Ans. is c i.e. Hemolytic streptococci Ref. Dhingra 5th/edp 341,6th/ed p 288; Current Otolaryngology 2nd/edp 341

Explanation

Group A beta-hemolytic streptococci is the M/C organism causing acute tonsillitis


106[

Other causes are:


• Staphytococci
• Pneumococci
• H. influenza
10. Ans b i.e. Ludwig angina
Ref. Dhingra Sth/ed p274,6th/ed p 259-260
Kindly see the preceding text for explanation.
11. Ans. is c i.e. Rheumatic tonsillitis
Ref. Scott Brown's 7th/ed Vol 2 pp 1989,1990, Vol 7 p 1232; Dhingra 5th/ed p 438,6th/ed p 428 Mohan Bansal p 567
Kindly see t h e preceding text for indications of tonsillectomy.
12. Ans. is d i.e. Cancel surgery for 3 weeks and patient to be on antibiotic
Ref. Logan Turner's Wth/edpp 365,366, Current
Otolaryngology 2nd/ed, p 178, Dhingra 6th/edp 428
"There are no absolute contraindications to tonsillectomy. As such tonsillectomy is an elective operation and should not be undertaken
in presence of respiratory tract infections or during the period of incubation of after contact with one ofthe infectious disease, if there is
tonsillar inflammations. It is much safer to wait some 3 weeks after an acute inflammatory illness before operating because ofthe greatly
increased risk of postoperative haemorrhage." - Turner 7 Oth/ed pp 365,366

Tonsillectomy and Adenoidectomy

"Patient may present with upper respiratory tract infections. Surgery for these patients should be postponed until the infection is resolved.
Usually 7-14 days. These patients may develop a laryngospasm with airway manipulation. This complication carries the potential for
significant morbidity and even mortality." - Current Otolaryngology 2nd/edp 173
13. Ans. is b i.e. 6-8 weeks Ref. Turner 7 Oth/ed p 86; Head and Neck Surgery by Chris DeSouza Vol 2 p 1583
• Friends, Dhingra and Turner have a different opinions on this one.
• According to Turner 10th/ed p 8 6 — " T h e tonsils should be removed 6-8 weeks.following a Quinsy."
• According to Dhingra 6th/ed p 265—"Tonsils are removed 4-6 weeks following an attack of Quinsy."
• According to Head and Neck Surgery-
• Quinsy - "Most people would practise interval tonsillectomy for these patients, deferring surgery for 6 weeks following resolution of
an attack."- Head and Neck Surgery by Chris de Souza Vol2p1583
So, after reading all t h e above texts -1 think 6-8 weeks is a better option.
14. Ans. is b i.e. Hemorrhage
Ref. Dhingra 5th/edp441,6th/edp 430; Maqbool 17 th/ed, p 288; Scott Brown's 7th/ed Vol2p 1994; Mohan Bansalp 577
15. Ans. is c i.e. Within 6 days Ref. Mohan Bansalp 571, Dhingra 6th/ed p 430
"The main complication is hemorrhage which occurs in 3-5% patients"—Head and Neck Surgery de Souza Vol 2 p 1588
"Most common complication following tonsillectomy is hemorrhage." —Maqbool 11 th/ed p 288
"Reactionary hemorrhage is the most feared complication post tonsillectomy because ofthe risk of airway obstruction, shock and ulti-
mately death." —Scott Brown's 7th/ed Vol2p1994

Hemorrhage can be

| Primary Reactionary Secondary I


Occurring at the time of operation Occurring within 24 hours of surgery Seen between the Sth to 10th postoperative day

Most common time of hemorrhage after tonsillectomy is within 4 hrs of surgery

16. Ans. is d i.e. Reopen immediately Ref. Turner 10th/ed p 366


"Reactionary hemorrhage occurs within a few hours ofthe operation and may be severe. It may occur after operation and is treated by a
return to the theater when the vessle is ligated under anesthesia." —Turner 10th/ed p 366

Also Know
• In case of secondary hemorrhage - Generally 2° hemorrhages are self-limiting and bleeding usually stops by t h e t i m e patient
reaches hospital.
• Suction o f t h e clot or gargling w i t h diluted hydrogen peroxide is the only treatment required in most cases.
• If bleeding recurs, topical epinephrine may be applied t o t h e tonsillar fossa. (Head and Neck Surgery Chris de Souza Vol2p 1589)
• Return t o operation theatre for placing ligature is rarely needed
CHAPTER 8 Anatomy of Pharynx, Tonsils and Adenoids J 107

Remember

• Reactionary hemorrhage are more c o m m o n in


- Adults (> 70 years)
Males
- Patients w i t h h/o infections
- Infectious mononucleosis
- Recurrent acute tonsilitis - Scott Brown's 7th/ed Vol 2 p 1995

.... ...
Indications for blood transfusion in a case of Tonsillectomy
- End-stage
Hypertension renal disease -
- Reduced hemoglobin and hematocrit In all these patients, if secondary hemorrhages occur - immediately return t o
OT t o
17. Ans. is b, c a n d d i.e. Poliomyelitis; Haemophilus avoid severe
infection; and complications
Upper RTI
Ref. Turner 1 Oth/ed pp 365,366; Mohan Bansal p 568
• As explained earlier,Tonsillectomy should not be performed during epidemics of poliomyelitis.This is because there are evidences
that the virus may gain access t o the exposed nerve sheaths and give rise t o the fatal bulbar form o f t h e disease.
• It should not be undertaken in the presence of respiratory tract infections or during the period of incubation of after contact
w i t h one o f t h e infectious disease (i.e. Haemophilus) or if there is tonsillar inflammation.
• It is safer t o wait for 3 weeks after an acute inflammatory disease, before performing tonsillectomy
According of Turner- Tonsillectomy can be performed at any age, if there are sufficient indications for their removal.
According to Dhingra - 6th/ed, p 428, Children < 3 years (Not < 4 years as given in the options) are poor candidates for surgery. So
tonsillectomy should not be done in t h e m .
According to Head and Neck Surgery de Souza -
"As tonsillar tissue has a role in the development ofthe immune system, it is advisable that surgery should be delayed until the age of 3
whenever possible."—Head and Neck Surgery Chris de Souza Vol2p 1587
18. Ans. is a i.e. Peritonsillar space Ref. Dhingra 5th/edpp 278,279,6th/edp 264
Quinsy is collection of pus in the peritonsillar space which lies between the capsule of tonsil and superior constrictor muscle i.e.
peritonsillar abscess.
19. Ans. is a and e Penicillin is used in treatment and Patient presents with toxic features and drooling
Ref. Logan Turner 10th/ed p 86; Dhingra 5th/ed p 279,6th/ed p 248; Scott's Brown 7th/ed Vol 2 pp 7 996,7 997
• Quinsy is collection of pus outside the capsule (not in capsule) in peritonsillar area
• t is usually unilateral
• Patient present w i t h toxic symptoms due t o septicemia as well as local symptoms (e.g. dribbling of saliva from mouth)
• Antibiotics: High-dose panicillin. (IV benzipenicillin) is the DOC. In patients allergic t o penicillin, erythromycin is t h e DOC. If
antibiotics fail t o relieve the condition w i t h i n 48 hours, t h e n the abscess must be opened and drained.
20. Ans. is c i.e. Systemic antibiotics up to 48 hours and then drainage
Ref. Harrison 77th/edp211;Scott's brown 7th/ed Vol2p 1997; Turner 10th/edp 86
Treatment o f quinsy include IV antibiotics and if it fails t o relieve the condition in 24-48 hours, the abscess must be opened and
drained.
2 1 . Ans. is a and b i.e. Antlantoaxial subluxation is the cause for his torticollis and the condition is M/C in children with Down's
syndrome. Ref. Curent otolaryngology 3rd/edp 363
Torticollis can occur as a complication of adenoidectomy due t o ligamentous laxity secondary t o inflammatory process following
adenoidectomy. It is called as Grisel syndrome.
This is M/C in patients of Down syndrome as children w i t h Down's already have asymptomatic atlantoaxial instability which m a n i -
fests after surgery.
22. T h e position drawn in figure is'Rose position'where patient lies supine with head extended by placing a pillow under the
shoulder—Rose position is used during. Ref. Dhingra 5th/edpp 438,439-442; Mohan Bansalp 569
i. Tonsillectomy
v ii. Adenoidectomy
iii. Tracheostomy
23. Ans. is c i.e. Palatine tonsil Ref. Dhingra 6th/edp 257
The medial surface of palatine tonsils is covered by non keratinizing stratified squamous epithelium which dips into the substance
of tonsil in t h e f o r m of crypts. One of these crypts is very large and deep and is called crypta magna or intratonsillar deft.
24. Ans. is c i.e. Sphenopalatine artery Ref. Dhingra 6th/edp 257
The tonsils are supplied by five arteries viz.
1. Tonsillar branch o f facial artery
108[ SECTION III Pharynx

2. Ascending pharyngeal artery f r o m external carotid artery


3. Ascending palatine, branch of facial atery
„ 11-11- U f ,
4. Dorsal lingual branches of lingual artery
5. Descending palatine branch of maxillary artery
25. Ans. is c i.e. 5-10 post operative days Ref. Dhingra 6th/ed p 430
Already expalined
26. Ans. is c i.e. Paratonsillar vein
- M/C cause of bleeding during tonsillectomy: Paratonsillar vein (Denis Browne vein)
- M/C arterial cause of bleeding during tonsillectomy: Toreillar branch of facial artery (K/a artery of tonsillar haemorrhage)
27. Ans. is c i.e. Poliomyelitis epidemic Ref. Dhingra 6th/edp 257
Already expalined.

•.sod

• •
9
A P T E R
Head and Neck Space
Inflammation
• •

| RETROPHARYNGEAL SPACE [ RETROPHARYNGEAL ABSCESS

Retropharyngeal space lies between the buccopharyngeal fascia Acute Retropharyngeal Abscess
covering the pharyngeal constriction muscles and the prevertebral
• Most c o m m o n l y seen in children below 6 years w i t h a peak
fascia covering t h e vertebrae and prevertebral muscles. A midline
incidence between 3 and 5 years.
fibrous raphe divides the space into t w o lateral compartments
(spaces of Gillette); one on each side. This an abscess of Retro
Cause
pharyngeal space causes unilateral bulge (Fig. 9.1).
Suppuration o f retropharyngeal lymph nodes due t o infection at
• Extension: f r o m the skull base t o the bifurcation of trachea.
its draining sites—adenoids, nasopharynx, posterior nasal sinuses
• Boundaries:
or nasal cavity.
Anterior: Buccopharyngeal fascia covering the pharyn-
geal constrictor muscle Adults
Posterior: Prevertebral fascia Cause: Penetrating injuries t o the posterior pharyngeal wall or the

3
Laterally: Carotid sheath cervical esophagus
• Contents: Retropharyngeal nodes. Rarely: Acute mastoiditis

CN IX, X, XI, XII Streptococcus viridans (46%)


and sympathetic trunk M/c organism
Staphylococcus aureus (26%)
Parotid gland

Clinical Features
Internal
jugular • Dysphagia
vein • Fever
Styloid • Difficulty in breathing—Stridor or Croupy cough
process • Torticollis

Buccopharyngeal On Examination
fascia
Palatopharyngeal Unilateral bulge in the posterior pharyngeal wall
muscle (Friends, d o n o t m u g up these features—as their is abscess—
Tonsil obviously fever will be present.
Since it is situated in retropharynx it w i l l — l e a d t o a b u l g e in
Palatoglossal
muscle posterior pharyngeal wall and torticollis. It will press trachea and
esophagus, so, it will cause difficulty in breathing and dysphagia.)

Fig. 9.1: Deep neck spaces for abscesses Treatment


Abbreviations:CH, Cranial nerves IX, X, XI and XII and sympathetic trunk; IC,
Internal carotid artery; IJV, Internal jugular vein; M, Masseter muscle; MT, • I and D w i t h o u t general anesthesia (due t o risk of rupture of
Medial pterygoid muscle; PP, Parapharyngeal space; PT, Peritonsillar space; abscess during intubation)
PV, Prevertebral space; RP, Retropharyngeal space • Antibiotics
Courtesy: Textbook of Diseases of Ear, Nose and • Tracheostomy: If large abscess causing mechanical obstruction
Throat, Mohan Bansal. Jaypee Brothers, p 539 o f t h e airway.
110^ SECTION III Pharynx

Chronic Retropharyngeal Abscess C. vertebra

Cause Prevertebral fascia


- Tuberculosis o f t h e cervical spine
- TB o f t h e retropharyngeal lymph nodes secondary t o t u b e r c u - Carotid sheath
losis o f t h e deep cervical lymph nodes Parapharyngeal
space
Features Retropharyngeal
- Discomfort in the throat space
- Pain n
- Fever Mandible
- Progressive neurological signs and symptoms due t o spinal
cord compression.
- Neck may show tubercular l y m p h nodes
Fig. 9.2: Relation of parapharyngeal space

Investigation
Medially: Eustachian tube, Pharynx, and Palatine tonsil,
X-ray medial pterygoid muscle
Radiological criteria t o diagnose retropharyngeal abscess: Posteriorly: Vertebral and Prevertebral muscles
• Widening of retropharyngeal space (> 3/4th diameter of cor- Anteriorly: Pterygoid muscles and interpterygoid fascia
responding cervical vertebra (Fig. 9.2)
• Straightening of cervical space •

• Presence of gas shadow


Medial wall of the parapharyngeal space is the lateral wall of the
Treatment peritonsillar space. It is proved by the superior constrictor muscle.
• Antituberculous therapy (ATT)
External drainage: • Styloid process divides the space into 2 compartments
Drainage t h r o u g h cervical incision
High abscess: vertical incision along the posterior border
of sternocleidomastoid muscle"
Low abscess: vertical incision along the anterior border o f Anterior c o m p a r t m e n t Posterior c o m p a r t m e n t
sternocleidomastoid muscle" ( r e l a t e d t o t o n s i l l a r fossa (related t o posterior part o f
medially m e d i a l p t e r y g o i d lateral pharyngeal wall medially
Danger Space muscle laterally) and p a r o t i d g l a n d laterally)
It lies just posterior to retropharyngeal space in between the Contents Contents
alar fascia (anteriorly) and prevertebral fascia (posteriorly). These
• Pterygoids • Neurovascular bundle
are the two layers of preverterbral layer of deep cervical fascia.
It contains only loose connective tissue and extends from skull • Tensor villi palati • Internal carotid artery
base to mediastinum. So the infection of this space can cause • Internal Jugallar vein
mediastinitis. During the surgical drainage, both the dangerous • IX, X, XI, XII Cranial nerves
and retropharyngeal spaces are treated as one unit. A dissecting • Sympathetic chain
finger is used to disrupt the partition between these two spaces.
• Upper deep cervical nodes

w PARAPHARYNGEAL ABSCESS (ABSCESS OF LATERAL


P H A R Y N G E A L SPACE)

A n a t o m y of P a r a p h a r y n g e a l s p a c e
Parapharyngeal Abscess

The parapharyngeal space communicates w i t h the retropharyngeal,


parotid, submandibular, carotid and visceral spaces.

Etiology
(Pharyngomaxillary space)
Infection in parapharyngeal space can occur t h r o u g h
Parapharynx lies o n either side o f t h e superior part o f pharynx i.e.
the nasopharynx and oropharynx.
Pharynx, tonsils, and adenoids infections
• It is pyramidal in shape w i t h base at the base of skull and apex
Teeth : Dental infections (Or extraction of lower
at hyoid bone.
third molar tooth) in 4 0 % cases.
• Relations:
Ear : Petrositis and Bezold's abscess
L a t e r a l l y : Medial ptyergoid muscle and mandible; deep
External trauma : Penetrating injuries of the neck
lobe o f t h e parotid
CHAPTER 9 Head and Neck Space Inflammation

Clinical Symptoms and Signs Predisposing Factor


• Very poor dental hygiene
Anterior compartment Posterior c o m p a r t m e n t • Debilitated patient
• Tonsil is pushed medially • Bulge in pharyngeal wall • Seen in y o u n g adults and middle-aged persons
• Trismus (due to spasm of behind the posterior pillar .

medial pterygoid muscles) Features

• External swelling behind • IX, X, XI, XII palsy Clinical


the angle of the jaw (at the • Homers syndrome due to • Necrotizing gingivitis, i.e. gums are covered w i t h necrotic
posterior part of middle third involvement of sympathetic membrane
of sternocleidomastoi) • Bleeding of gums
chain
• Odynophagia • Ulceration of mucosa of tonsils, pharynx and m o u t h
• Parotid bulge
• Minimal trismus or tonsillar • Patients parent w i t h low-grade pyrexia and sore t h r o a t
• Torticollis
prolapse
On Examination
Grayish black membrane is present on one tonsil b u t may involve
Investigation of Choice: CTscan g u m , soft and hard palate. The m e m b r a n e bleeds w h e n i t is
removed. It gives a characteristic smell t o the breath.
Treatment
• Admission t o hospital for intravenous (IV) antibiotics (penicillin/ Treatment
cefuroxime) is t h e baseline treatment • Sodium bicarbonate gargles.
• Failure t o respond t o conservative treatment or clinical dete- • Penicillin + Metronidazole
rioration should p r o m p t surgical abscess drainage • Dental care
• Abscess drainage is done through a collar incision in the neck at
the level of hyoid-bone under general anaesthesia
KERATOSIS PHARYNGIS
Feature—Benign Condition:
VINCENT'S ANGINA (TRENCH MOUTH/ULCERATIVE
GINGIVITIS) > Horny excrescences on the tonsillar surface, pharyngeal wall or
lingual tonsils which appear as white/yellow dots which cannot
be wiped off.
Organisms
> No constitutional symptoms
• Spirochete » Treatment: Reassurance
• Borellia vincentii
Thornwaldt's disease (Pharyngeal Bursitis)
• Anaerobe
• Bacillus fusiformis Persistence of Lushka pouch (Pharyngeal bursa) causes
thornwaldt's cysts which may get infected and form an abscess
in the nasopharynx called as thronwaldt's disease as pharyngeal
bursitis.


SECTION III Pharynx

QUESTIONS

1. A male Shyam, age 30 years presented with trismus, fe- 7. True statement about chronic retropharnygeal abscess:
ver, swelling pushing the tonsils medially and spreading [PGI 03]
laterally posterior to the middle sternocleido-mastoid. a. Associated with tuberculosis of spine
He gives H/O excision of 3rd molar few days back for b. Causes psoas spasm
dental caries. The diagnosis is: [AIIMS 01] c. Suppuration of Rouviere lymph node
a. Retropharyngeal abscess b. Ludwig's angina d. Treatment by surgery
c. Submental abscess d. Parapharyngeal abscess 8. Retropharyngeal abscess, false is... [AIIMS Nov 10]
2. A postdental extraction patient presents with swelling a. It lies lateral to midline
in posterior one third of the sternocleidomastoid, the b. Causes difficulty in swallowing and speech
tonsil is pushed medially. Most likely diagnosis is: c. Can always be palpated by finger at the post pharyngeal
a. Retopharyngeal abscess b. Parapharyngeal abscess wall
c. Ludwig angina d. Vincent angina d. It is present beneath the vertebral fascia.
3. Parapharygeal space is also known as: [PGI June 05] 9. Infection of submandibular space is seen \n:[Manipal08]
a. Retropharyngeal space b. Pyriform sinus a. Ludwig angina b. Vincent angina
c. Lateral pharyngeal space d. Pterygomaxillary space c. Prinzmetal angina d. Unstable angina
4. The medial bulging of pharynx is seen in: [AI9.1] 10. Middle age diabetic with tooth extraction with ipsilateral
a. Pharyngomaxillary abscess swelling over middle one-third of sternocleidomastoid
b. Retropharyngeal abscess and displacement of tonsils towards contralateral side:
c. Peritonsillar abscess [NEET Pattern]
d. Paratonsillar abscess a. Parapharyngeal abscess b. Retropharyngeal abscess
5. Trismus in parapharyngeal abscess is due to spasm to: c. Ludwigs angina d. None
[PGI 98] 11. W h i c h o f t h e f o l l o w i n g is n o t t r u e a b o u t a c u t e
a. Masseter muscle b. Medial pterygoid retropharyngeal absess: [NEET Pattern]
c. Lateral pterygoid d. Temporalis a. Dysphagia
6. M o s t c o m m o n c a u s e o f c h r o n i c r e t r o p h a r y n g e a l b. Swelling on posterolateral wall
abscess: [Kolkata 01] c. Torticollis
a. Suppuration of retropharyngeal lymph node d. Caries of cervical spine is usually a common cause
b. Caries of cervical spine 12. Thornwaldt cyst: [Neet Pattern]
c. Infective foreign body a. Laryngeal cyst b. Nasopharyngeal cyst
d. Caries teeth c. Ear cyst d. None

EXPLANATIONS AND REFERENCES

1. Ans. is d i.e. Parapharyngeal abscess


2. Ans. is b i.e. Parapharyngeal abscess
Ref. Turner Wth/edp 106;Tuli Ist/edp 260,2nd/edp 268; Mohan Bansal p 542; Dhingra 6th/ed p 267
History of dental caries —

+
Trismus
+
Swelling pushing t h e tonsils medially Indicate parapharyngeal abscess
+
Swelling spreading posterior t o t h e sternocleidomastoid or
Presenting w i t h a swelling in middle 1/3 of sternocleidomastoid
rd
_

3. Ans. is c and d i.e. Lateral pharyngeal space; and Pterygomaxillary space


Ref. Dhingra 5th/ed p 281,6th/ed p 267; Mohan Bansal p 538
4. Ans. is a i.e. Pharyngomaxillary space
• Parapharyngeal space is also called lateral pharyngeal space and pharyngomaxillary space.
* Pharyngomaxillary abscess is a synonym for parapharyngeal abscess (which is also called Lateral Pharyngeal abscess).
CHAPTER 9 Head and Neck Space Inflammation J 113
5. Ans. is b i.e. Medial pterygoid muscle Ref. Dhingra 5th/edp282,6th/ed p 268
Trismus in parapharyngeal abscess is due to spasm of medial pterygoid muscle.

• Styloid process divides the pharynx into anterior and posterior compartment
• Trismus occurs in infection of anterior compartment whereas torticollis (due to spasm of paravertebral muscles) occurs in the infection of
posterior compartment.

6. Ans. is b i.e. Caries of cervical spine


7. Ans. is a, c and d i.e. Associated with tuberculosis of spine; and Suppuration of Rouviere lymph node; and Treated by
surgery Ref.Dhingra5th/edp281,6th/edp266-267
• Chronic retropharyngeal abscess is associated w i t h caries o f cervical spine or tuberculous infection of retropharyngeal lymph
nodes secondary t o tuberculosis of deep cervical nodes (i.e. suppuration of Rouviere nodes)
• It leads t o discomort in throat, dysphagia, fluctuant swelling of postpharyngeal wall.
• Retropharyngeal abscess doesnot lead t o psoas spasm.

Treatment

• Incison and drainage of abscess


• Full course of ATT

HBHHMMBHHBHHBHI

Most common cause of acute retropharyngeal abscess.

T
I
Children Adults
• Suppuration of retropharyngeal lymph nodes secondary to infection • Due to penetrating injury of posterior pharyngeal wall or cervical
in the adenoids, nasopharynx, posterior nasal sinuses or nasal cavity esophagus

8. Ans. is d i.e. It is present beneath vertebral fascia Ref. Dhingra 5th/edpp 280,281,6th/edp 266-267; Mohan Bansalp 543
• Retropharyneal space lies behind the pharynx between the buccopharyngeal fascia covering pharyngeal constrictor muscles
and the prevertebral facia (i.e. behind the pharynx and in front of prevertebral fascia)
• Thus o p t i o n d, i.e. it lies beneath the vertebral fascia is incorrect.
• On physical examination, may reveal bulging o f t h e posterior pharyngeal wall, although this is present in < 5 0 % of infants w i t h
retropharyngeal abscess. Cervical lymphadenopathy may also be present. There will be as smooth swelling on one side o f t h e
posterior pharyngeal wall w i t h airway impairment.
• Dysphagia and difficulty in breathing are prominent symptoms as the abscess obstructs the air and food passages
9. Ans. is a i.e. Ludwig angina Ref. Dhingra 5th/edp 277,6th/ed p 263; Mohan Bansal p 543

Submandibular Space

• It lies between mucous membrane of floor of m o u t h and tongue on one side and superficial layer of deep cervical fascia extending
between t h e hyoid bone and mandible on other side.
• It is divided into 2 compartments by mylohyoid muscle
- sublingual space - above the mylohyoid
v
- submaxillary s p a c e - b e l o w the mylohyoid
Infection of submandibular space is called Ludwig angina
Bacteriology: Infections involved b o t h aerobes and anaerobes. The M/c causative organism are rhemolytic Streptococci,
Staphylococci and bacteroides.
114[ SECTION III Pharynx

Ludwig angina

Etiology Clinical features Treatment

- Dental infection - 8 0 % cases - Odynophagia (difficulty in IV antibiotics


Root of premolar - sublingual swallowing) Incision and drainage
space infection - Trismus
Root of molar - submaxillary space - Infection of dsublingual space
- Submandibular sialadenits Swelling of floor of mouth +
- Fracture of mandible tongue is pushed up and back Sublingual Submaxillary
- Infection spreads to submaxillary space space

and submental space I I
- Cellulitis of floor is present Intraoral External
- Tongue is pushed up and back
leading to decreased airway

10. Ans. is a i.e. Parapharyngeal abscess Ref. Dhingra 6th/edp267


Already explained
11. Ans. is d i.e. Caries of cervical spine is usually a common cause Ref. Dhingra 6th/edp266
As discussed previously, M/C cause acute retropharyngeal alscess in children is suppuration of retro pharyngeal lymphnodes
secondary t o infection of aderoids, nasopharynx and nasal cavity.
The M/C cause of acute retropharyngeal abscess in adutls is penetrating injury of posterior pharyngeal wall orcerivcal esophagus.
Rest all options are clinical features seen in acute retropharyngeal abscess.
12. Ans. is b i.e. Nasopharyngeal cyst
See preceding text for explanation.

- ' •

-
Lesions of Nasopharynx and
• I

CHAPTER
Hypopharynx including
Tumors of Pharynx
V
-

Most common site is posterior part of nasal cavity close to the


NASOPHARYNX (ALSO CALLED AS EPIHARYNX) margin of sphenopalatine foramen.
Seen almost exclusively in males of 10-20years {testosterone
| ANATOMY
dependent tumor seen in prepubertal to adolescent males).
• Oval in shape extends vertically from the skull base (basioc- Locally invasive vasoformative tumor consisting of endothe-
ciput and basisphenoid) to soft palate (horizontal line passing lium lined vessels with no muscle coat.
through the hard palate) The major blood supply is from internal maxillary artery.
• Posteriorly related to arch of axis (C1) vertebra and anteriorly
to choana. Clinical Features
• Lateral wall is characterized by pharyngeal opening of Eusta- Symptoms
chian tube bounded above and behind by the torus tubarius Symptoms depend on spread of tumor to nasal cavity, paranasal
(cartilage) sinuses, pterygomaxillary fossa, infratemporal fossa, cheek, orbits
(through inferior orbital fissure), cranial cavity (most common site
Fossa ofRosenmuller
is middle cranial fossa).
• Also known as pharyngeal recess or lateral recess, it is a recess
• Most common symptom - Spontaneous profuse and recurrent
posterolateral to the torus.
epistaxis.
• This is the most common site of origin of carcinoma nasopharynx.
• Progressive nasal obstruction, denasal speech, hyposmia/
Sinus of Morgagni anosmia, broadening of nasal bridge.
• Otalgia conductive hearing loss, serous otitis media, due to
• The pharynx is covered by overlapping constrictor muscles in the
eustachian tube obstruction.
entire extent except superiorly between the skull base and upper
• Pink or purplish mass obstructing one or both choanae in
border borer of superior constrictor muscle, bridged only by the
nasopharynx.
pharyngobasilar fascia. This area is known as sinus of Morgagni,
• Tumor in the orbit causes proptosis and frog-face deformity;
which allows entry to Eustachian tube, levator palate, and ascend-
diplopia and diminshed vision.
ing palatine artery.
• Swelling of cheek
• It is the routeof spread of malignancy of nasopharynx outside
• Tumor in infratemporal fossa can cause trismus and bulge of
to the parapharyngeal space.
parotid.
• Passavant's ridge is formed by fibers of palatopharyngeus muscle
• II, III, IV, V, VI cranial nerve can be involved.
(or sometimes by inner fibers of superior constrictor). It encircles
the posterior and lateral walls of nasopharynx forming a U-shaped
sling against which the soft palate is approximated to maintain
Juvenile nasopharyngeal angiofibroma: In an adolescent male,
competence during deglutition. profuse recurrent episodes of nosebleed suggests juvenile
nasopharyngeal angiofibroma until proven otherwise
| TUMORS OF NASOPHARYNX
Sign
Nasopharyngeal Fibroma/juvenile Nasopharyngeal
Splaying of nasal bones
Angiofibroma
Pink or purplish mass obstructing one or both choanae in
• Most common benign tumor of nasopharynx (but overall an- nasopharynx
giofibroma is rare). Swelling of cheek and fullness of face.
116[ SECTION III Pharynx

Diagnosis Etiology
• Soft tissue lateral film of nasopharynx and X-ray of paranasal • Genetic: It is most c o m m o n in Southern China.
sinuses, base o f skull. • Viral: Epstein-Barr virus has identified in t u m o r epithelial cells
• CT scan of h e a d with contrast enhancement is n o w t h e IOC. It of most undifferentiated and nonkeratinizing squamous cell
shows extent, bony destruction or displacements and anterior carcinoma.
b o w i n g o f t h e posterior wall of maxilla due t o t u m o r enlarging • Environmental: Burning of incense or w o o d (polycyclic hydro-
in pterygopalatine fossa (called as antral sign/Holman miller carbon); smoking of tobacco and o p i u m ; air pollution; nitrosa-
sign) which is pathognomic of angiofibroma. mines f r o m dry salted fish along w i t h vitamin C deficient diet.
• MRI is done t o veiw the soft tissue extension and is c o m p l e -
mentary to CT scan. Clinical Features
• Carotid angiography-Shows extent o f t h e tumor, its vascularity • It usually affects males. The t u m o r occcurs at much younger
and feeding vessel age than other cancers; its incidence begins t o rise after 2nd
• Biopsy is contraindicated. decade and peaks by 5th decade.
• Most common manifestation is upper neck swelling due to
Treatment cervical lymphadenopathy since nasopharynx is richly supplied
• Surgical excision is treatment of choice. by lymphatics.
• Preoperative embolization a n d estrogen therapy or cryo- • Unilateral neckswelling is more common than bilateral swelling
therapy or radiotherapy reduce blood loss in surgery. • Most common l y m p h node involved jugulodigastric (upper
deep cervical) lymph node.
• Earliest lymph node involved is retropharyngeal l y m p h n o d e .
Preoperative embolzation ofthe tumor reduces its blood supply
and causes less bleeding, if tumor removal is performed within Spread of Tumor Findings
24-48 hour of embolization before collaterals have time to Nasal obstruction; epistaxis
V. Nose and orbit
develop. Properative angiography also helps to find any feeders
from internal caroitid system. 2. Eustachian tube Serous/suppurative otitis media
leading t o U/L deafness and
tinnitus
Approach:
Surgical approach of choice = Midfacial degloving a p - 3. Parapharyngeal space Cranial nerve palsies IX, X, XI, XII;
proach t o nasopharynx. Horner's syndrome; trismus
Transpalatal approach is for tumor confinedto nasophar- 4. Foramen lacerum and Ophthalmic symptoms and facial
ynx, (called as Wilson approach) ovale pain (CN III, IV, V, VI) (Cavernous
Lateral r h i n o t o m y done for larger tumors involving nasal sinus thrombosis)
cavity, paranasal sinuses.
5. Retropharyngeal nodes Neck pain and stiffness
Other Approaches:
• Transpalatine + Sublabial (Sardana's approach) 6. Krause's nodes These LN's are s i t u a t e d in
• Extended Denker's approach. the j u g u l a r f o r a m e n . Their
enlargement compresses CN IX,
X, and XI and produce jugular
foramen syndrome.
Recurrence is not uncommon after surgery (30-50%).
Recurrence rates can be reduced by meticulous dissection of 7. Distant metastases Secondaries in bone (most
sphenopalatine foramen. Recurrences usually become evident common) lung, liver
within 2-3 years of initial resection.
Most c o m m o n cranial nerve palsy in nasopharyngeal
carcinoma is V cranial nerve followed by VI nerve whereas M/C
multiple cranial nerves involved are IX and X.
It is not a fast-growing tumor.

Nasopharyngeal Carcinoma Remember


> Presence of unilateral serous otitis media in an adult should
U n c o m m o n in India except in North-East region where people are
raise suspicion of nasopharyngeal growth.
predominantly of m o n g o l o i d origin. • Trotter's triad: Seen in Nasopharyngeal cancer. Includes
• M/c in southern states of China, Taiwan and Indonesia. conductive deafness; ipsilateral tempoparietal neuralgia due to
• It is m o s t c o m m o n t u m o r of head and neck which give rise t o involvement ofCN V; palatal paralysis due to CN IX. Also called
secondaries w i t h occult primary. as sinus of morgagni syndrome
• Most c o m m o n site is fossa of rosenmuller in the lateral wall o f < Unlike other squamous cell carcinoma, it can metastasize to
nasopharynx. posterior triangle (level V) in the absence of jugular lymph
• M o s t c o m m o n t y p e o f nasopharyngeal c a r c i n o m a — N o n . node involvement.
keratinising undifferentiated carcinoma followed by keratinis- • Neck is the M/C: Site of clinically occult primary cancer of
tonsillar fossa, tongue base, pyriform sinus and nasopharynx.
ing carcinoma.
CHAPTER 10 Lesions of Nasopharynx and Hypopharynx including Tumors of Pharynx J 117

Diagnosis Rhadomyosarcoma

• Most i m p o r t a n t is examination o f postnasal space by naso- • It is the M/c malignant t u m o r of nasopharynx in children.
pharyngeal mirror or nasopharyngoscope. • Orbit is the M/c site of rhadomyosarcoma in t h e head and
• Biopsy of nasopharynx is considered the first necessary inves- neck region.
tigation for nasopharyngeal canceroma if a suspected lesion
is f o u n d . Nasopharyngeal Chordoma
• Imaging modality of choice - MRI w i t h gadolinium and fat
• It originates f r o m the notochord.
suppression.

Treatment
• Irradiation /streatment of choice (external beam radiotherapy)
HYPOPHARYNX
on as nasopharyngeal carcinoma is highly radiosensitive.
• In stage I and II only radiotherapy is done. In stage III and IV | ANATOMY
chemoradiation is the treatment stage
Hypopharynx extends f r o m the floor of vallecula t o the lower
• Radical neck dissection is required for persistent nodes w h e n
border of the cricoid.
p r i m a r y has been controlled and in postradiation cervical
metastasis. It has three parts:
1. Pyriform sinus
Complications of Radiotherapy 2. Posterior pharyngeal wall
• Xerostomia of radiotherapy (M/c c o m m o n complication be- 3. Postcricoid (see adjacent figure)
cause both major and minor salivary glands are well w i t h i n
the field of irradiation)
H TUMORS OF HYPOPHARYNX
« Mucositis, altered taste sensation, dental caries
• Radiation otitis media w i t h effusion, rhinosinusitis H y p o p h a r y n x C a n c e r (Fig. 10.1)
• Radionecrosis o f skull base
• Most c o m m o n type o f t u m o r of hypopharynx is - squamous
• Radiation myelitis
cell carcinoma. 0

• Encephalomyelitic change
• Optic atrophy Etiology
• M o d e r n radiotherapy techniques like intensity-modulated
• Alcohol
radiation therapy (IMRT) have decreased the incidences of
these complications. • Tobacco
• Vitamin A deficiency
Lhermitte's Sign • Iron deficiency/plummer-vinson syndrome is an i m p o r t a n t
• U n c o m m o n complication etiology for carcinoma postcricoid.
• Cause: Due t o radiation t o the cervical spinal cord • Low cholesterol levels.
• Features: Lightening - like electrical sensation spreading into
b o t h arms, d o w n the dorsal spin, and into b o t h legs on neck
flexion.

Feature CA Pyriform Sinus CA Postcricoid CA Posterior Pharyngeal Wall


Mostcommon of all hypopharyngeal cancer 2nd most common Least common (10%)
(60%) hypopharyngeal cancer (30%)
Age and sex Mostly males > 40 years • Mostly females • Mostly males > 40
• May be seen as early as 20-30
years
Clinical features Generally symptomless and diagnosed late
Presenting symptoms Pricking/sticking sensation in throat Progressive dysphagia Dysphagia, hemoptysis

Presenting sign Enlarged lymph nodes Enlarged lymph nodes


Paratracheal lymph nodes
Lymphatic spread Upper deep cervical nodes Retropharyngeal lymph nodes
(Bilateral)
Treatment of choice Early growth-radiotherapy Poor prognosis with both surgery Early growths-radiotherapy Later surgery
and radiotherapy
SECTION III Pharynx

Arytenoid and Posterior • There is an incoordination between the descending peristaltic


aryepiglottic fold pharyngeal wall wave and circopharyngeus muscle at the upper esophageal
sphincter leading t o abnormally high intraluminal pressure and
mucosal herniation through the weakarea of Killian's dehiscence.
• M/c s y m p t o m is dysphagia; initially i n t e r m i t t e n t w h i c h b e -
comes progressive later o n .
• It is associated w i t h regurgitatiovn of food and cough. Patient
may experience halitosis and regurgling sounds in neck.
• The gurgling sensation palpation of neck is known as Boyce sign.
Medial wall of • Diagnosis is by Barium swallow + videofluoroscopy.
pyriform sinus Postcricoid region
• Malignancy can develop in 0.5-1 % cases.

Treatment

• Endoscopic stapling o f the diverticulo esophageal septum


(Earlier excision of diverticulum with cricopharyngeal myotomy
was considered t o be the treatment of choice
• In patients not fit for major procedures Dohlman's surgery (En-
doscopic cautery of diverticulo esophageal septum) may suffice

Zenker's Diverticulum is not a true diverticulum

A true diverticulum contains all layers of the esophageal wall


while false diverticula consists primarily of only mucosa and
submucosa.
Fig. 10.1: Sites of hypopharyngeal cancer. (A) indirect Zenker's diverticulum is a pulsion diverticulum that arises
laryngoscopic; (B) posterior views of laryngopharynx because elevated intraluminal pressure forces the mucosa and
Courtesy: Textbook of Diseases of Ear, Nose and submucosa to herniate through the muscle layer and hence is
Throat, Mohan Bansal. Jaypee Brothers, p 450 a false diverticulum.

Inferior constrictor
muscle of pharynx Plummer-Vinsion (Paterson-Brown-Kelly) S y n d r o m e
Thyropharyngeus • Mostly affects females more than 40 years.

Killian's dehiscence Clas sical features include


Progressively increasing dysphagia for solids (due t o Webs
Cricopharyngeus
in postcricoid region).
Zenker's diverticulum Iron deficiency anemia.
Circular fibers Glossitis and stomatitis
Koilonychia (spooning of nails).
Longitudinal fibers
- Achlorhydria
Esophagus • Smooth tongue devoid of papillae.
• Craked lips and corners of m o u t h .
• Barium swallow shows web in the postcricoid region due t o
subepithelial fibrosis in the region.
Fig. 10.2: Zenker's d i v e r t i c u l u m o f h y p o p h a r y n x h e r n i a t i n g
• 2 % cases develop postcricoid carcinoma.
t h r o u g h the Killian's dehiscence between the thyropharyngeal and
cricopharyngeal parts o f t h e inferior constrictor muscle Treatment
Courtesy: Textbook of Diseases of Ear, Nose and
• Correction of anemia
\ Throat, Mohan Bansal. Jaypee Brothers, p 463
• Dilatation o f t h e w e b b e d area by esophageal bougies.
| BENIGN HYPOPHARYNGEAL LESIONS

Z e n k e r ' s D i v e r t i c u l u m ( P h a r y n g e a l P o u c h ) (Fig. 10.2) Remember


• It is a posterior pharyngeal pulsion diverticulum t h r o u g h the 1
In cancer of pyriform fossa: The referred ear pain is through
Killian's dehiscence (area o f weakness), between the t h y r o - CN X (superior laryngeal nerve, branch of vagus nerve).
pharyngeus and cricopharyngeus parts of inferior constrictor 1
Laryngeal crepitus: Post-laryngeal crepitus is present in normal
muscle. persons and absent in patients with postcricoid malignancy, j
CHAPTER 10 Lesions of Nasopharynx and Hypopharynx including Tumors of Pharynx

QUESTIONS
1. Most common site of origin of nasopharyngeal angiofi- c. Surgery is treatment of choice
broma: [Al 00] d. Radiotherapy can be given
a. Roof of Nasopharynx b. At sphenopalatine foraman e. Recurrence is c o m m o n
c. Vault ofskull d. Lateral wall of nose 11 Most appropriate investigation for angiofibroma is:
2 Nasopharyngeal angiofibroma is: [TN91] [AIIMS 97]
a. Benign a. Angiography b. CTscan
b. Malignant c. MRI scan d. Plain X-ray
c. Benign but potentially malignant 12, A 2 years child presents with B/L nasal pink masses. Most
d. None o f t h e above important investigation prior to undertaking surgery
3. A 1 0 y e a r s child has unilateral nasal obstruction epistaxis, is: [AI97]
swelling over cheek, the diagnosis is: [AIIMS 99] a. CTScan b. FNAC
a. Nasal polyp b. Nasopharyngeal carcinoma c. Biopsy d. Ultrasound
c. Angiofibroma d. Foreign bodies 13 A 10-year-old boy presents with nasal obstruction and
4. C h a n d u a 15-year-aged boy presents with unilateral intermittent profuse epistaxis. He has a firm pinkish mass
nasal blockade mass in the cheek and epistaxis; likely in the nasopharynx. All of the following investigations
diasnosisis: [AI01] [UPSC98]
are done in this case except:
a. Nasopharyngeal Ca b. Angiofibroma
a. X-ray base of skull b. Carotid angiography
c. Inverted papilloma d. None o f t h e above >sy
c. CTscan d.
5. In angiofibroma of nasopharynx all are correct except:
14. IOC for angiofibroma is:
[Kolkata 00]
a. CTscan MRI
a. Common in female
c. Angiography Plain X-Ray
b. Most c o m m o n presentation is epistaxis
15. A n 18-year-old boy presented with repeated epistaxis
c. Arises from roof of nasopharynx
and there was a mass arising from the lateral wall of his
d. In late cases frog-face deformity occurs
nose extending into the nasopharynx. It was decided to
6. Angiofibroma bleeds excessively because: [DNB 0 1]
operate him. All o f t h e following are true regarding his
a. It lacks a capsule
management except: [AIIMS 02]
b. Vessels lack a contractile component
a. Requires adequate amount of blood to be transfused
c. It has multiple sites of origin
b. A lateral rhinotomy approach may be used
d. All o f t h e above
c. Transpalatal approach used
7. Clinical features of nasopharyngeal angiofibroma are:
d. Transmaxillary approach
[PGI 02]
16. Treatment of choice for angiofibroma: [RJ02]
3rd to 4th decades
a. Surgery b. Radiotherapy
Adolescent male
c. Both d. Chemotherapy
Epistaxis and nasal obstruction is the cardinal symptom
17. A 9 years boy presents with nasal obstruction, proptosis,
Radiotherapy is the Rx of choice
r e c u r r e n t e p i s t a x i s f r o m 3-4 y e a r s . M a n a g e m e n t
Arises from posterior nasal cavity
includes: [PGI Nov 10]
A 14 years boy presented with repeated epistaxis, and
a swelling in cheek. Which of these statements may be a. Routine radiological investigations
correct: [PGI 02] b. Embolization alone should be done
a. Diagnosis is nasopharyngeal angiofibroma c. Surgery is treatment of choice
Contrast CT scan should be done to see the extent d. Embolization followed by surgery
High propensity to spread via lymphatics e. Conservative management is sufficient
Arises from roof of nose 18. Radiotherapy is used in treatment of angiofibroma when
Surgery is therapy of choice it involves: [MP 04]
True about juvenile nasopharyngeal angiofibroma: a. Cheek b. Orbit
[PGI June 06] c. Middle cranial fossa d. Cavernous sinus
fc. Surgery is treatment of choice 19. Most common site for nasopharyngeal carcinoma:
b. It is malignant tumor [AIIMS 97; MP 02]
c. Incidence in females a. Nasal septum b. Fossa of rosenmuller
d. Hormones not used in Rx c. Vault of nasopharynx d. Anterosuperior wall
e. Miller's sign positive 20. Nasopharyngeal Ca involve: [PGI 02]
10. True about nasopharyngeal angiofibroma: [PGI Dec 03] a. Nasal cavity b. Orophaynx
a. Commonly seen in girls c. Oral cavity d. Tympanic cavity
b. Hormonal etiology e. Orbit
SECTION III Pharynx

21. Nasopharyngeal Ca is caused by: [AIIMS 98] 31. Which of the following is NOT true about nasopharyngeal
a. EBV b. Papillomavirus carcinoma? [Al 10]
c. Parvovirus d. Adenovirus a. Bimodal age distribution
22. Most common presentation in nasopharyngeal b. EBV is implicated as etiological agent
carcinoma is with: [Al 97] c. Squamous cell carcinoma is common
a. Epistaxis b. Hoarseness of voice d. Nasopharyngectomy and lymph node dissection is mainstay
c. Nasal stuffiness d. Cervical lymphadenopathy of treatment
23. A 70-year-old male presents with Neck nodes. 32. Treatment of choice in nasopharyngeal carcinoma:
Examination reveals a Dull Tympanic Membrance, deaf- [Al 98; PGI Dec 05 FMGE 2013]
ness and tinnitus and on evaluation Audiometry gives a. Radiotherapy b. Chemotherapy
Curve B. The most probable diagnosis is: [Al 08] c. Surgery d. Surgery and radiotherapy
a. Nasopharyngeal carcinoma 33. True about plummer Vinson syndrome: [PGI 06]
b. Fluid in maddle ear a. Web is M/C in lower esophagus
c. Tumor in interior ear b. Web is M/C in Mid esophagus
d. Sensorineuronal hearing loss c. Web is M/C in postcricoid region
24. Nasopharyngeal Ca causes deafness by: d. It occurs due to abnormal vessels
[PGI Nov 05; PGI Dec 07] e. Reduced motility of esophagus
Temporal bone metastasis 34. A p a t i e n t p r e s e n t s w i t h r e g u r g i t a t i o n o f f o o d w i t h
Middle ear infiltration foul smelling breath and intermittent dysphagia and
Serous effusion diagnosis is: [AI01]
Radiation therapy a. Achalasia cardia b. Tracheoesophageal fistula
2 5 . Horner's syndrome is caused by: [PGI 97] c. Zenker's diverticulum d. Diabetic gastropathy
a. Nasopharyngeal carcinoma metastasis 35. All of the following are true about Zenker's diverticulum
b. Facial bone injury except:
c. Maxillary sinusites a.
It is an acquired condition
d. Ethmoid polyp b.
It is a false diverticulum
26. Trotter's triad is seen in carcinoma of: [Corned 08] c.
Barium swallow, lateral view is the investigation of choice
a. Maxilla b. Larynx d.
Out pouching of anterior pharyngeal wall above cricopha-
c. Nasopharynx d. Ethmoid sinus ryngeus muscles
27. Trotter's triad includes all o f t h e following except: 36. Frog face deformity of nose caused by: [NEETPattern]
[AI09] a. Rhinoscleroma b. Angiofibroma
a. Mandibular Neuralgia Deafness c. Antral polyp d. Ethmoidal polyp
c. Palatal palsy Seizures 37. Which of the following is not true for juvenile angiofi-
28. True about Trotter's triad [PGI Dec 08] broma: [NEET Pattern]
a. Conductive deafness Involvement of CN VI a. Biopsy for diagnosis b. Benign tumor
c. Involvement of CN X d. Palatal paralysis c. Surgical excision d. Second decade
e. Associated with nasopharyngeal angiofibroma 38. M o s t c o m m o n p r e s e n t a t i o n i n nasopharyngeal
29. Nasopharyngeal Ca: [PGI 02] carcinoma: [NEET Pattern]
a. Epistaxis b. Hoarseness of voice
-
M/c nerve involve is vagus
Unilateral serous otitis media is seen c. Nasal stuffiness d. Cervical lymphadenopathy
Treatment of choice radiotherapy 39. Trotter's triad includes all except: [NEETPattern]
Metastasized to cervical lymph node a. Sensory disturbance over distribution of 5th cranial nerve
EBV is responsible b. Diplopia
30. Which among the following is not true regarding naso- c. Conductive deafness
pharyngeal carcinoma: [PGI 01] d. Palatal palsy
a. Associated with EBV infection 40. 70-years-old man with cervical lymphadenopathy. What
Starts in the fossa of Rosenmuller can be the cause: [NEET Pattern]
Radiotherapy is the treatment of choice a. Nasopharyngeal carcinoma
Adenocarcinoma is usual b. Angiofibroma
If elderly patients present with unilateral otitis media, it is c. Acoustic neuroma
highly suggestive d. Otosclerosis

CHAPTER 10 Lesions of Nasopharynx and Hypopharynx including Tumors of Pharynx

EXPLANATIONS AND REFERENCES


I
1. Ans. is b i.e. at Sphenopalatine foramen
2. Ans. is a i.e. Benign Ref. Dhingra 5th/edp 261,6th/ed p 246; Scott-Brown's 7th/ed Vol 2 p 2437; Mohan Bansal p 437
• Nasopharyngeal fibroma is the mosf common benign tumor of nasopharynx.
• Mostcommon site is posterior part o f nasal cavity close t o the margin of Sphenopalatine foramen.
• Though it is a benign tumor, it is locally invasive and destroys the adjoining structures.
• Juvenine Angiofibroma is uncommon, benign and extremely vascular tumor that arises in the tissues within the
sphenopalatine foramen." - Scott-Brown's 7th/ed Vol 2 p 2437
3. Ans. is c i.e. Angiofibroma
4. Ans. is b i.e. Angiofibroma Ref. Dhingra 5th/ed p 261,6th/edp 246; Scott-Brown's 7th/ed Vol 2 p 2437; Mohan Bansal p 437
• '

T h i s is T y p i c a l P r e s e n t a t i o n o f N a s o p h a r y n g e a l F i b r o m a / A n g i o f i b r o m a
• Nasopharyngeal fibroma is most common benign t u m o r of nasopharynx.
• Mosf common site is posterior part of nasal cavity close t o the margin of sphenopalatine foramen.
• Sex : Seen almost exclusively in males (testosterone dependent).
• Age : 10-20 years (2nd decade).
• Clinical features: - Most common s y m p t o m is profuse and recurrent epistaxis
- Progressive nasal obstruction
- Denasal speech
- Conductive hearing loss and serous otitis media
- Mass in nasopharynx
- Broadening o f nasal bridge
- - Proptosis
- Frog-face deformity
- Swelling of cheek
- Involvement of cranial nerves II, III, IV, VI.

So friends, remember—if the Question says a boy with age 10-20 years presents with swelling of cheek and recurrent epistaxis - Do not think
of anything else but -'Nasopharyngeal fibroma'

5. Ans. is a i.e. Common in female Ref. Read below


Let us see each option separately here:
Option Correct/Incorrect Reference Explanation
Option a = Common in female Incorrect Dhingra 5th/edp 261, It is seen almost exclusively in male
6th/ed, p 246; Scott-
Brown's 7th/ed Vol 2 p
2437
Option b = M/c presentation is Correct Dhingra 5th/ed p 261, Profuse and recurrent epistaxis is the M/c presentation
epistaxis 6th/ed, p 246; Scott-
Brown's 7th/ed Vol 2 p
2438
Option c = Arises from roof of Partly correct Dhingra 5th/ed p 261, This statement is partly correct as earlier it was thought to
nasopharynx 6th/ed,p 246 arise from roof of nasopharynx or anteriorwall of sphenoid.
But now it is believed to arise from posterior part of nasal
\ - >- s v 8s fi * 0
cavity close to sphenopalatine foramen.

Option d Correct Dhingra 5th/ed p 262, In later stages, it can lead to broadening of nasal bridge,
6th/ed, p 246 proptosis, i.e. frog-like deformity.
In late cases frog-like deformity
seen

S/B = Scott-Brown 7th/ed


Thus o p t i o n a, i.e. Common in females is absolutely incorrect and the o p t i o n of choice here.
122 [_ SECTION III Pharynx

6. Ans. is b i.e. Vessels lack a contracture component Ref. Dhingra 5th/edp 261
Angiofibroma as the name implies is made of vascular and fibrous tissues in varying ratios "Mostly the vessels are just endothelium
lined spaces with no muscle coat. This accounts for the severe bleeding as the vessels lose the ability t o contract, and also, bleeding
cannot be controlled by application of adrenaline." - Dhingra 5th/ed p 261,6th/ed p 246
7. Ans. is b, c a n d e i.e. Adolescent male; Epistaxis and nasal obstruction is the cardinal symptom and arises from posterior
nasal cavity. Ref. Dhingra Sth/ed pp 261 -3,6th/ed p 246; Mohan Bansal p 437-8

Nasopharyngeal Angiofibroma

• Most c o m m o n l y seen in adolescent males (i.e. option b is correct)


• Most c o m m o n age of presentation = second decade of life (option a incorrect)
• Arises f r o m posterior nasal cavity close t o sphenopalatine foramen (option e is correct)
• Epistaxis and nasal obstruction are the most c o m m o n presentation, (correct)
• Recurrent severe epistaxis accompanied by progressive nasal obstruction are the classical symptoms of juvenile angiofibromas
at the t i m e of presentation." - Scoff-Brown 7th/ed Vol2p 2438
• TOC is surgical excision (i.e. o p t i o n d is incorrect)
8. Ans. is a, b and e i.e. Diagnosis is nasopharyngeal angiofibroma; Contrast CT scan should be done to see the extent; and
Surgery is therapy of choice
9. Ans. is a and e i.e. Surgery is treatment of choice; and Miller's sign positive
10. Ans. b, c, d and e i.e. Hormonal etiology; Surgery is treatment of choice; Radiotherapy can be given; and Recurrence is
common Ref. Dhingra 5th/edpp 261 -3,6th/edp 246-9
Boy of 14 years presenting with repeated epistaxis with swelling in cheek points toward angiofibroma
It is a benign tumor, so it does not spread by lymphatics but is locally invasive.
Exclusively seen in males between 10-20 years (i.e. testosterone dependent).
It arises f r o m posterior part of nasal cavity (near sphenopalatine frames).
Contrast CT is the investigation of choice as the extent o f t h e t u m o r can be seen.
On CT scan (which is the IOC) pathognomic finding is anterior b o w i n g of the posterior wall of the maxillary sinus, called the
Holman miller sign/Antral sign. Ref. Dhingra 5th/edp 262,6th/edp 248
Surgery is the treatment of choice.
Hormonal therapy w i t h estrogen reduces the vascularity o f t u m o r and helps in successful resection.
Radiotherapy can also be used especially for intracranial extension and in case of recurrences
[Dhingra 5th/edp263], 6th/edp, 249 Table 49.2
Recurrence is the most c o m m o n complication
"Recurrence is by far the most common complication encountered and is reported in 2 5 % patients."
[Scott-Brown's 7th/ed, Vol 2, p 2442]
11. Ans. is b i.e. CT scan
12. Ans. a i.e. CT scan
13. Ans. is d i.e. Biopsy
14. Ans. is a i.e. CT scan Ref. Dhingra 5th/edp262,6th/edp 252; Mohan Bansalp 437

Diagnosis of Nasopharyngeal Fibroma


CT scan of head with contrast enhancement is now the IOC. It shows the extent, bony destruction or displacements and
anterior b o w i n g o f t h e posterior wall of maxillary sinus (called as antral sign) which is pathognomic of angiofibroma.
• MRI is complimentary t o CT and is done especially t o see the soft tissue extension.
• Carotid angiography shows the vascularity and feeding vessels. It is done when embolization is planned before operation.
• Biopsy is contraindicated.
15. Ans. is d i.e.Transmaxillary approach
16. Ans. a i.e. Surgery
17. Ans. is a, c and e i.e. Routine radiological investigations; Surgery is the IOC; and Embolization followed by surgery
\ Ref. Dhingra 5th/ed pp 262,263,6th/ed p 252
18 years male

Repeated epistaxis Indicates the patient has


+ nasopharyngeal angiofibroma
Mass arising from the lateral wall of
nose and extending t o nasopharynx
CHAPTER 10 Lesions of Nasopharynx and Hypopharynx including Tumors of Pharynx

Treatment

• Surgical excision is the treatment of choice.


• Before surgery at least 2-3 liters of blood should be given. —Tuli Ist/edp 253
• Preoperative embolization and estrogen therapy or cryotherapy reduce b l o o d loss in surgery. —Dhingra 6th/edp249

Approach

• Transpalatine approach—done for t u m o r confined t o nasopharynx.


• Lateral rhinotomy approach—done for large tumors involving, nasal cavity, paranasal sinuses and orbit. Nowadays, it is the best
approach.

Other Approaches

• Sardana's approach - Transpalatine + Sublabial.


• Transhyoid and transmandibular approach.
• Transzygomatic approach.

ALSO KNOW

Other modalities of treatment in nasopharyngeal angiofibroma.

Radiotherapy Hormonal Chemotherapy

• For intracranial extension of tumor when • Since tumor occurs in young males • Doxorubicin, vincristine and dacarbazine are
it derives its blood supply from Internal testosterone has been implicated for its used for residual with recurrent lesions
carotid artery growth. Antitestosterone are being tried for
management
• Recurrent angiofibromas are treated with • Diethylstilbestrol with Flutamide
intensity modulated radiotherapy

18. Ans. is c i.e. Middle cranial fossa Ref. Dhingra 6th/ed p 249
Radiotherapy is useful only for advanced cases o f t h e tumor.
Extent of juvenile nasopharyngeal angiofibroma and surgical approach

Location Approach

A. Nose and nasopharynx Transpalatal or endoscopic


B. Nose, nasopharynx maxillary antrun and pterygopalatine tossa Lateral rhinotomy with medial maxillectomy
OR
Endoscopic
OR
Le Fort 1
C. As in B + Infratemporal fossa Extended lateral rhinotomy
Or <i?m'wlOJ
Infratemporal fossa approach
OR
Maxillary swing approach
D. As in C +Check extension Extended lateral rhinotomy
E. As in B + C +Intracranial Combined intracraial and extracranial approach (craniotomy + one of th
extracranial approaches)
OR
Radiation if intracranial part is inaccessible
F. Residual or recurrent disease (extracranial) Observation OR repeat surgery or radiation if inaccessible
G. Intracranial residual or recurrent Stereotactic radiation (X or gamma knife)

19. Ans. is b i.e. Fossa of Rosenmuller


Ref. Dhingra 5th/ed p 264,6th/ed p 251; Mohan Bansal p439
Nasopharyngeal Carcinoma

• Nasopharyngeal carcinoma most c o m m o n l y arises f r o m


• It is mostly seen in men between Sth-7th decade. fossa of rosenmuller in lateral wall of nasopharynx.
124|_ SECTION III Pharynx

• Most c o m m o n type of nasopharyngeal carcinoma is Non keratinising undifferentiated carcinoma followed by squamous
cell carcinoma
° Most c o m m o n manifestation is cervical lymphadenopathy because of rich lymphatic network.

• Most c o m m o n cranial nerve palsy in nasopharyngeal carcinoma is VI cranial nerve.
• Treatment o f choice is irradiation.
20. Ans. is a, d and e i.e. Nasal cavity; Tympanic cavity; and Orbit Ref. Dhingra 5th/edp 265,6th/edp 250
Nasopharyngeal cancer arises f r o m fossa of Rosenmuller." In the lateral wall of nasopharynx and can spread t o various sites.

R o u t e s of S p r e a d a n d C l i n i c a l F e a t u r e s o f N a s o p h a r y n g e a l C a r c i n o m a (Fig. 1 0 . 3 )

Ophthalmic symptoms
and facial pain (CN III, IV, V, VI) -

Cranial nerves
''palsies (IX, X, XII Foramen lacerum am
Horner's ovale •

9 Parapharyngeal
syndrom /
| Eustachian tube
Pterygoid ^J^' P s a c e

muscles SSBk Nose and


(Trismus) \ X^iPv.
orbit

^Retropharyngeal Distant
Neck p a i n * ^ n o c l e s
metastases
v and stiffness
Cervical nodes

Upper-jugular and
3
osterior trangle nodes enlargemerj

Fig. 10.3: Routes of spread and clinical features of nasopharyngeal cancer


21. Ans. is a i.e. EBV Ref. Dhingra 5th/ed p 264,6th/ed p 250; Mohan Bansal p439

Etiology of Nasopharyngeal Carcinoma

• Genetic: It is mostcommon in China.


• Viral: Epstein-Barr virus is closely associated w i t h nasopharyngeal cancer. Epstein-Barr virus has identified in t u m o r epithelial
cells (not lymphocytes) of most undifferentiated and nonkeratinizing squamous cell carcinoma.
• Environmental: Burning of incense or w o o d (polycyclic hydrocarbon); smoking of tobacco and o p i u m ; air pollution; nitrosamines
from d r y salted fish along w i t h vitamin C deficient diet have been linked t o the etiology of nasopharyngeal cancer.
22. Ans. is d i.e. Cervical lymphadenopathy Ref. Dhingra 5th/ed p 264,6th/edp 252; Scott-Brown's 7th/ed Vol 2 p 2451
"The most common complain at presentation is the presence of an upper neck swelling. Unilateral neck swelling is much more
common although bilateral metastasis also occur." - Scott-Brown's 7th/ed Vol 2 p 2457
23. Ans. is a i.e. Nasopharyngeal carcinoma Ref. Dhingra 5th/edpp 261,262,6th/edp 251-2
Let us see t h e complains and examination findings one by one.
• A 70 years male is presenting w i t h neck nodes (which could either be due t o infection or due t o malignancy, but malignancy is
more consistent w i t h the age).
• On examination—ear shows coincidental findings viz.Tympanic membrane appears dull and audiometry shows curve B which
means fluid is present in middle ear (Ruling o u t option "d" i.e. SNHL).

F l u i d in M i d d l e E a r c a n b e S e e n in

Serous otitis media

Tympanic membrane appears red, bulging On examination either cholesteatoma Tympanic membrane appears dull and
in early stages and later in the stage of granulation or perforation will be seen audiometry shows B type of curve. So,
resolution usually a small perforation is seen serous otitis media is a possibility

Also here patient will have fever and Neck nodes will not be the presenting So,
excruciating earache (which is the chief CSOM ruled out
complain) So, ASOM ruled out
CHAPTER 10 Lesions of Nasopharynx and Hypopharynx including Tumors of Pharynx J 125

Serous otitis media/Glue ear:


It is mostly seen in school-going children. If serous otitis media is seen in adults (that t o o males) - always think of Nasopharyngeal
carcinoma.
"Presence of unilateral serous otitis media in an adult should raise the suspicion of a nasopharyngeal growth"
—Dhingra 5 th/ed p 264,6th/ed p 251
The diagnosis further consolidated by-
• Age of patient = 70 years (mostcommon age for nasopharyngeal carcinoma = 5th - 7th decade).
• Sex of patient = male (nasopharyngeal carcinoma is most common in males).
• Presenting s y m p t o m = Presence of neck nodes.
• (It is the mostcommon presenting symptom of nasopharyngeal carcinoma).

Presence of unilateral serous otitis media in an adult should always raise suspicion of nasopharyngeal growth.
J
24. Ans. is c i.e Serous effusion Ref. Dhingra 5th/ed p 264,6th/ed p 251; Scott-Brown's 7th/ed Vol 2 p 2458
Nasopharyngeal carcinoma spreads to Eustachian tube, blocks it and causes Serous Otitis Media which in turn causes Conductive hearing
loss.
25. Ans. is a i.e. Nasopharyngeal Carcinoma Metastasis Ref. Dhingra 5th/edp 264,6th/edp 251; Mohan Bansalp 439
Nasopharyngeal carcinoma can cause Horner's syndrome due t o involvement of cervical sympathetic chain.
26. Ans. is c i.e. Nasopharynx Ref. Dhingra 5th/ed p 264; Mohan Bansalp 439
27. Ans. is d i.e. Seizures
28. Ans. is a, c and d i.e. Conductive deafness; Involvement of CN X; and Palatal Paralysis
Ref. Dhingra 6th/ed p 251; Mohan Bansal p 439
Trotter's triad - seen in nasopharyngeal carcinoma is characterised by (Fig. 10.4):
Conductive deafness
(due to Eustachian dysfunction)

Ipsilateral Palatal paralysis


temporoparietal neuralgia (due to involvement of CN X)
(due to involvement of CN V)
Fig. 10.4: Trotter triad
29. Ans. is b, c, d and e i.e. Unileteral serous otitis media is seen; TOC radiotherapy; Metastaised to cervical LN; and EBV is
responsible Ref. Dhingra 5th/ed pp 264-6,6th/ed p250-251
• Epstein-Barr virus is associated w i t h etiology of nasopharyngeal carcinoma (i.e. o p t i o n e is correct)
• Most common presentation - Cervical lymphadenopathy (i.e. o p t i o n d is correct)
• Most common nerve involved - VI nerve - —Ref. Current Otolaryngology 2nd/edp 365; Mohan Bansalp 439
• Nasopharyngeal carcinoma leads t o obstruction of Eustachian tube and serous otitis media.
• Presence of Unilateral Serous otitis media in an adult should raise suspicion of nasopharyngeal g r o w t h .
• TOC is radiotherapy because o f t h e difficulty in obtaining adequate surgical margins.
30. Ans. is d i.e. Adenocarcinoma is usual Ref. Dhingra 5th/ed pp 264,265,6th/ed p 250-251

Mostcommon histological type of nasopharyngeal carcinoma - Squamous cell carcinoma.


Rest ofthe options have been explained earlier.

31. Ans. is d i.e. Nasophyrangiectomy and lymph node dissection is mainstay of treatment
\ Ref. Dhingra 5th/ed pp 264-6,6th/ed p 250-252
In nasopharyngeal carcinoma, radiotherapy is the mainstay of treatment. Radical neck dissection is required for persistent nodes
w h e n primary has been controlled.
For details on nasopharyngeal carcinoma, kindly see preceding text.
32. Ans. is a i.e. Radiotherapy Ref. Dhingra 5th/edp 266,6th/ed p 252; Mohan Bansal p 439-40
TOC for nasopharyngeal fibroma - Surgery
TOC for nasopharyngeal carcinoma - Radiaton
TOC for advanced carcinoma - Chemotherapy + Radiation
126 J_ SECTION III Pharynx

33. Ans is c, i.e. Web is M/C in post cricoid region Ref. Dhingra 5th/edp 351,6th/ed p 343; Mohan Bansal p 46 1
In Plummer-Vinson syndrome patients present w i t h dysphagia due t o web in the postcricoid region and due t o incoordinated
swallowing secondary t o esophageal spasm
For more details see preceding text
34. Ans. is c i.e. Zenker's Diverticulum Ref. Dhingra 5th/ed pp 289-90,6th/ed p 274
• In Zenker's diverticulum patients present w i t h intermittent dysphagia + regurgitation o f food + foul smelling breath.
• Later on the dysphagia becomes progressive.
• In case o f achalasia cardia patients present w i t h dysphagia t o liquids initially which later on progresses t o involve solids also.
• ' In trachea esophageal fistula patients present w i t h cough during meals causing difficulty in eating.
35. Ans. is d i.e. Outpouching of anterior pharyngeal wall above crsicopharyngeus muscle
Ref. Dhingra 5th/ed pp 289-90,6th/ed p274
Zenker's diverticulum is an acquired posterior pharyngeal pulsion diverticulum in which only the mucosa and submucosa herni-
ate t h r o u g h the Killian's dehiscence. It is a false diverticulum. IOC is barium study.
36. Ans. is b i.e. Angiofibroma Ref. Dhingra 6th/edp 246
Already explained
37. Ans. is a i.e. biopsy for diagnosis Ref. Dhingra 6th/ed p 247
As discussed earlier biopsy is never done for diagnosis of nasopharyngeal fibroma as it is extremely vascular t u m o r and is attended
by profuse bleeding.
38. Ans. is d i.e. Cervical lymphadenopathy Ref. Dhingra 6th/edp 252
Cerivcal lymphadenopathy is the M/C presentation of nasopharyngeal carcinoma. It may be the only manifestation in some cases.
39. Ans. is b i.e. Diplopia Ref. Dhingra 6th/edp 251
Nasopharyngeal can cause conductive deafness (eustachian t u b e blockage,) ipsilateral temporoparietal neuralgia (involvement of
CN V) and palatal paralysis (CNX) collectively called Trotters triad.
40. Ans. is a i.e. Nasopharyngeal carcinoma sRef. Dhingra p 250-2
Already explained

CLINICAL VIGENNETTES

Clinical presentation Diagnosis


• A 35-year-old male, presented with trismus, fever, swelling Parapharyngeal abscess
pushing the tonsil medially and spreading laterally posterior to
sternocleidomastoid toward the middle. He gives h/O extraction of
3 molar tooth due to dental caries, a few days back.
rd

• A 14-year-old boy presented with repeated epistaxis and swelling in Nasal angiofibroma
cheek
• A 70-year-old male presents with neck nodes. Examination reveals Nasopharyngeal carcinoma
dull tympanic membrane, deafness and tinnitus on evaluation
audiometry shows curve B.
• A 70-year-old made presents with regurgitation, no diurnal cough, Zenker's diverticulum
dysphagia and has gurgling sensation on palpating neck.
• A pale emaciated 45-year-old female complains of dysphagia for Postcricoid carcinoma
both solid and liquid with absence of laryngeal crepitus and B/L
pooling of saliva
• A middle-aged female complains of foreign body sensation in throat Globus pharyngeus, (Functional disorder where patient complains of
No organic lesion could be detected. lump in throat, no true dysphagia on clinical examination) Everything is
• Dhingra 5th/ed, p 353 normal T/t= Reassuarance
\


CHAPTER
-

Pharynx Hot Topics


I SNORING Table 11.1: Causesls of snoring in adults

Site Cause
It is an undesirable disturbing sound that occurs during sleep. It is
Nose (Nasopharynx) • Septal deviation
estimated that 2 5 % o f adult males and 1 5 % of adult females snore. • Nasal hypertrophy
Its prevalence increase w i t h age.s • Nasal polyp
• Nasal tumor
Definition of T e r m s Oral cavity (oropharynx) • Elongated self palate/uvula
• Large base of tongue
• Sleep apnea: It is cessation of breathing that lasts for 10 s • Tongue tumor
or more during sleep. Less than five such episodes is normal. Larynx (laryngopharynx) • Laryngeal stenosis
• A p n e a index: It is number of episodes o f apnea in 1 hour • Omega shaped epiglottis
• Hypopnoea: It is reduction of airflow. Some define it ias Others • Obesity
• Use of alcohol, sedatives,
drop of 5 0 % of airflow f r o m the base line associated w i t h
hypnotics
an EFG defined arousal or 4 % drop in oxygen saturation.
• Respiratory disturbance index (RDI). Also called a p n e a - Sites of S n o r i n g
hyponoea index. It is the number of apnea and hypopnoea
Site of snoring may be soft palate, tonsillar pillars or hypopharynx.
events per hour. Normally RDI is less than five. Based on
RDI, severity o f apnea has been classified as mild, 5-14; Symptomatology
moderate, 15-29; and severe > 30. • Excessive loud snoring is socially disruptive and forms snoring-
• Arousal index. It is number of arousal events in 1 h. Less spouse syndrome and is the cause of marital discord sometime
t h a n four is normal. leading t o divorce.
• Sleep efficiency. Minutes of sleep divided by minutes in • I n addition, a snorer with obstructive sleep apnea may manifest
bed after lights are turned off. with:
Excessive d a y t i m e sleepiness (measured an e p w o r t h
• Multiple sleep latency test or nap study. Patient is given
sleepiness scale)
four or five scheduled naps usually in the daytime. Latency
Morning headaches
period f r o m wakefulness t o the onset of sleep and rolling
General fatigusse
eye movement (REM) sleep are measured. It is performed Memory loss
w h e n narcolepsy is suspected or daytime sleepiness is Irritability and depression
evaluated objectively. Decreased libido
I ncreased risk of road accidents

Etiology
Treatment
• In children most c o m m o n cause is adenotonsillar hypertrophy. Avoidance of alcohol, sedatives and hypnotics.
• In adults see Table 11.1 Reduction of weight.
128[ SECTION III Pharynx

• Sleeping on t h e side rather than or the back. P e r m a n e n t t r a c h e o s t o m y is t h e " g o l d s t a n d a r d " o f


• Removal of obstructing lesion in nose, nasopharynx, oral cavity, treatment but it is not accepted socially and has complications
hypopharynx and larynx. Radiofrequency has been used for of its own. It is usually not a preferred option b y patients.
volumetric reduction of tissues of turbinates, soft palate and
Surgical procedures used in OSA include:
base of t o n g u e .
• Tonsillectomy and/or adenoidectomy.
• Performing uvulopalatoplasty (UPP) surgically with cold
• Nasal surgery: Nasal obstruction may be the primary or the
knife or a s s i s t e d w i t h r a d i o f r e q u e n c y (RAUP) or laser
aggravating factor for OSA. septoplasty t o correct deviated
(LAUP).
nasal s e p t u m , removal o f nasal polyps and r e d u c t i o n o f
turbinate size help t o relieve nasal obstruction.
| SLEEP APNEA
• Oropharyngeal surgery: Uvulopalatoplasty (UPP) is the
Apnea means no breathing at all. There is no movement of air at most common procedure performed for snoring and OSA. It
the level of nose and m o u t h . It is of three types: is 8 0 % effective in snoring but OSA is relieved only in 5 0 % .
1. Obstructive: There is collapse o f t h e upper airway resulting • A d v a n c e m e n t g e n i o p l a s t y with hyoid s u s p e n s i o n : It is
in cessation o f airflow. Other factors may be o b s t r u c t i v e done in patients where base of t o n g u e also contributes t o
conditions of nose, nasopharynx, oral cavity and oropharynx,
OSA. Patients w i t h retrognathia and micrognathia are also
base of tongue or larynx.
the candidates.
2. Central: Airways are patent but brain fails to signal the muscles
• Tongue base radiofrequency: Radiofrequency (RF) is used in
t o breathe.
five t o six sittings t o reduce the size of tongue.
3. Mixed: It is combination of both types.
• Maxillomandibular advancement osteotomy: Osteotomies
P a t h o p h y s i o l o g y of O b s t r u c t i v e S l e e p A p n e a (OSA) are performed on mandibular ramus and maxilla. Osteomy of
the maxilla is like a Le Fort I procedure.
Apnea during sleep causes hypoxia and retention of carbon dioxide
which leads t o pulmonary constriction leading t o congestive heart
Important Clinical points to R e m e m b e r
failure bradycardia and cardiac hypoxia leading t o left heart failure,
and cardiac arrhythmias sometimes leading t o sudden death. • Investigation o f choice f o r dysphagia = Barium
During sleep apnea, there are frequent arousals which cause sleep videofluoroscopy
fragmentation, daytime sleepiness and other manifestations. • D y s p h a g i a t o solids is g e n e r a l l y d u e t o m e c h a n i c a l
obstruction (e.g. tumors) —> endoscopy should be done
Treatment • Dysphagia t o liquids is generally due t o motility disorders —>
esophageal manometry should be done
Non Surgical Treatment
• Change in lifestyle. | RIGID E S O P H A G O S C O P Y
• Positional therapy: Patient should sleep on the side as supine
position may cause obstructive apnea. A rubber ball can be • Anesthesia - General Anaesthesia
fixed t o the back of shirt t o prevent adopting supine position. • Position - Boyce position (Similar t o direct laryngoscopy)
• Intraoral devices: They alter t h e position o f m a n d i b l e or • C/l of rigid esophagoscopy - cervical spine injury, Aneurysm
tongue t o open the airway and relieve snoring and sleep apnea. of arch of aorta, Recent Ml, Trismus
Mandible advancement device (MAD) keeps t h e mandible In most of these cases new generations of flexible gastroscopes
can be used successfully.
forward while t o n g u e retaining device (TRD) keeps t o n g u e in
anterior position during sleep. They help improve or abolish • M/C complication of rigid esophagoscopy - Perforation
snoring. MAD is also useful in retrognathic patients. • M/C site for perforation - Just above cricopharyngeal
Q.I. All are true about esophagoscopy except: [PGI 06]
• CPAP (continuous positive airway pressure): It provides
a. Compress the posterior part of tongue
pneumatic splint t o airway and increase its calibre. O p t i m u m
b. Tip o f t h e esophagoscope lies in pyriform fossa
airway pressure for device t o open the airway is determined
c. Should be inserted from right side
d u r i n g sleep study and is usually kept at 5-20 cm H 0 . About 2
d. Epiglottis should be lifted up
4 0 % of patients find the use of CPAP device comberome to carry e. Incisiors must act as fulcrum
w i t h t h e m w h e n travelling and thus stop using it. Ans. is e i.e. Incisors must act as fulcrum
When CPAP is n o t tolerated, a BiPAP (bilevel positive airway Ref. Dhingra 5th/ed pp 436,437
pressure) device is used. It delivers positive pressure at t w o
fixed levels—a higher inspiratory and a lower expiratory Explanation
pressure. Now an autotitrating PAP (APAP) is also available
w h i c h continuously adjusts the pressure. Esophagoscopy Procedure

• Hold the scope in a pen-like fashion and introduce it into the


Surgical Treatment m o u t h f r o m the right side of tongue and then t o w a r d the
It is indicated for failed or noncompliant medical therapy. midline.
CHAPTER 11 Pharynx Hot Topics J 129
Never rotate t h e endoscope on the fulcrum o f t h e upper teeth, Move the head slightly t o the right while passing the cardia.
rather it should be lifted up. (Identified by redder and more velvety mucosa)
Lift u p the epiglottis after passing t h r o u g h the t o n g u e base t o
identify the arytenoids. Tip o f t h e scope should be introduced
into the pyriform sinuses o n either t o inspect t h e m before A rigid bronchoscope can be used for performing esophagoscopy
passing behind the arytenoids. but not vice versa.
Open up the cricopharyngeal sphincter by slow sustained
pressure, never apply force on the sphincter for it can result in Esophageal Perforation
undue spasm and perforation • Features: Fever after esophagoscopy.
Once the esophagus is entered keep the lumen in constant view • Diagnosis: Swallow study confirms the diagnosis.
Lower the head o f t h e patient while negotiating the aortic and • Treatment: Early intervention t o repair is most desirable. Drain
bronchial constriciton the perforation t o prevent complications.
LARYNX

12. Anatomy of Larynx, Congenital Lesions of Larynx and Stridor


13. Acute and Chronic Inflammation of Larynx, Voice and Speech Disorders
14. Vocal Cord Paralysis
15. Tumor of Larynx

A -• • • " • • •• • ••••>••

» "

- , iia t!
CHAPTER
Anatomy of Larynx, Congenital
Lesions of Larynx and Stridor
- •

Larynx develops f r o m tracheobronchial groove, a m i d l i n e Epiglottis


diverticulum o f foregut.
Development starts in t h e 4 t h week of embryonic life.
Most o f t h e anatomical characteristics of larynx develop by the
3rd m o n t h of fetal life. Hyoid bone
Angle o f t h e thyroid cartilage at birth: Cartilago triticea
Thyrohyoid
Males : 110 degree membrane Superior horn of
Females : 120 degree Thyroepiglottic thyroid cartilage
The angle remains till puberty. ligament
Corniculate
Level ofthe larynx: cartilage
- At birth : till C3 Thyroid Arytenoid
- By 5 years : till C6 cartilage cartilage
- 15-20 years : C7
Muscular process
Descent o f t h e larynx continues t h r o u g h o u t life.
Vocal cord length: Inferior horn of
Infants : 6 - 8 mm Lamina of thyroid cartilage
Adult males : 17-23 mm cricoid cartilage
- Adult females : 12-17 mm

| EXTERNAL FEATURES O F LARYNX

Laryngeal Cartilages
Fig. 12.2: Posterior view of larynx showing cartilages and ligaments
Laryngeal cartilages are 9 in number and derived f r o m Courtesy: Textbook of Diseases of Ear, Nose and Throat,
4 , 5 and 6
th th th
arches. Mohan Bansal. Jaypee Brothers, p 62.

• Larynx has 9 cartilages of which 3 are paired and 3 are unpaired:

Paired Unpaired
Thyrohyoid
ligament - Arytenoid - Thyroid
- Lateral - Corniculate - Cricoid
- Median - - Cuneiform - Epiglottis
Thyroid-^
Ossification ofthe various laryngeal cartilages: •
notch \
Cricothyroid — . \ Hyoid 2 years
joint
) Thyroid a n d Cricoid Early 20s
Cricoid
cartilage Arytenoid Late 30s

Trachea
cartilage j^iiuillillliJ
• Vocal process DO NOT ossify
Fig. 12.1: Laryngeal framework—anterior view • No ossification occurs in the cuneiform or the corniculate
Courtesy: Textbook of Diseases of Ear, Nose and Throat, cartilage.
Mohan Bansal. Jaypee Brothers, p 62.
134|_ SECTION IV Larynx

Histology of L a r y n g e a l C a r t i l a g e s Pre-epiglottic Space:


Anteriorly: T h y r o h y o i d m e m b r a n e and u p p e r part o f
Hyaline cartilages (ossify) Elastic cartilages (Do not ossify) thyroid cartilage
• Thyroid cartilages All the other cartilages Posteriorly: Infrahyoid part o f t h e epiglottis
• Cricoid cartilages Superiorly: Hyoepiglottic ligament
• Basal part of arytenoid
cartilage
Preepiglottic
Epiglottis space
NOTE Paraglottic
Other example of elastic cartilage is auricular cartilage space
Thyroid
cartilage Qudrangular
Characteristics of Individual L a r y n g e a l C a r t i l a g e s Pyriform membrane

Thyroid Cartilage fossa

• Largest cartilage, hyaline in nature. Fig. 12.3: Diagram t o show pre-epiglottic and
• It is V shaped and consists of right and left lamina. Which meet paraepiglottic space
anteriorly in midline and f o r m an angle (Adams angle) • Paraglottic S p a c e is c o n t i n u o u s m e d i a l l y w i t h t h e pre-
• Adams angle: epiglottic space.
Male : 90 degree Boundaries: • Laterally - Thyroid cartilage
Female : 120 degree • Medially - Quadrangular membrane and
• The outer surface of each lamina is marked by an o b l i q u e Conus elasticus
line which extends f r o m superior thyroid tubercle t o inferior • Posteriorly - Anterior reflection of pyriform
thyroid tubercle. sinus.
Joints of L a r y n x
Oblique line gives attachment t o : Cricoarytenoid Joint ~~| Synovial Joints
Thyrohyoid Criocthyroid Joint
Sternothyroid
Inferior constrictor muscle
Larynx of infants differ from the adults as:
Cricoid Cartilage - It is situated high up (C2-C4) and funnel shaped/conical
• It is hyaline cartilage and shaped like a ring, (the only complete (Adults - Cylindrical in shape) with narrow epiglottis
- Cartilages are soft and collapse easily on forced inspiration.
cartilaginous ring in the airway)
Epiglottis is omaga shaped It has more of submucosal space
• It articulates w i t h arytenoid cartilage. Cricoarytenoid j o i n t is
- The narrowest part of infantile larynx is the junction of
a synovial j o i n t 0
subglottic larynx with trachea" ^

Arytenoid Cartilage
M e m b r a n e s of T h y r o i d
• They are 2 small pyramid shaped cartilages. It articulates w i t h
cricoid lamina. • Thyrohyoid membrane: connects the thyroid cartilage t o the

o
hyoid bone. Its median and lateral parts are thickened t o form
the median and and lateral thyrohyoid ligaments.

It has a vocal process for : attachment of vocal folds.


It has»muscular process for : attachment of posterior Structures Piercing it are Superior Laryngeal Vessels and
cricoarytenoid and lateral cricothyroid^ Internal Laryngeal Nerve j
• Its apex articulates w i t h corniculate cartilage.
Q u a d r a t e m e m b r a n e is a f i b r o e l a s t i c m e m b r a n e w h i c h
C o r n i c u l a t e (Cartilage of Santorini) a n d C u n e i f o r m
extends between the border of the epiglottis and the aryteniod
(Cartilage of Wrisberg)
cartilage. It has upper border called the aryepiglottic fold and
• Are fibroelastic cartilages. Corniculate cartilages are conical; a lower margin called as vestibular fold.
cuneiform cartilages are rod shaped. Cricovocal membrane: This triangular fibroelastic membrane
• Corniculate carlitage articulates t h r o u g h a synovial j o i n t w i t h hasfree upper border (vocal ligament), which stretches between
apices o f arytenoids cartilage. middle of thyroid angle to the vocal process of arytenoids. The
lower border is attached t o the arch of cricoid cartilage.
Epiglottis Conus elasticus: The t w o sides of cricovocal membranes
• It is Fibroelastic cartilage which is leaf shaped in adults and form conus elasticus. Subglottic foreign bodies sometimes
omega shaped in children. get impacted in the region o f conus elasticus.
CHAPTER 12 Anatomy of Larynx, Congenital Lesions of Larynx and Stridor J 135
Cricothyroid membrane: The anterior part of conus elas- Vestibular and vocal folds divide the cavity of larynx into 3
ticus is thick and forms cricothyroid membrane, which parts
connects thyroid cartilage t o cricoid cartilage. • T h e p a r t a b o v e t h e v e s t i b u l a r f o l d - Vestibule of larynx/
supraglottis
• The part between the vestibular and vocal f o l d - Sinus of
Any airway obstruction above the vocal cord due to tumor or morgagni/ventricle of larynx/glottis
foreign body can be quickly, easily and effectively bypassed • The part below the vocal folds - Infraglotticpart
by piercing the cricothyroid membrane (cricothyrotomy). •

Subglottic foreign bodies sometimes get impacted in the Clinical Correlation


region of conus elasticus. • Most of the laryngeal foreign bodies are seen in supraglottic
region lying above the vocal cords.

| INTERIOR OF THE LARYNX

The anterior part of sinus of morgagni is prolonged upward as


Inlet of the l a r y n x
a diverticulum between the vestibular fold and the lamina of
thyroid cartilage, this extension is called as the Saccule of larynx.
Anteriorly Bounded on sides Posteriorly
The secretion of mucusglands in the saccule provide lubrication
Free edge of Aryepiglottic folds Mucous membrane over for vocal cords.region
the epiglottis the interarytenoid fold

- Cavity of larynx extends from inlet of larynxto the lowerborder Clinical Correlation
o f t h e cricoid cartilage. Laryngo cole: This abnormally enlarged and distended saccule
- W i t h i n t h e cavity o f larynx, t h e r e are 2-folds o f m u c o u s contains air.
membrane on each side.The u p p e r f o l d is called as vestibular
Retention cyst: The obstruction of d u c t o f mucous gland in
fold (false vocal cords) and the lower fold is called as vocal fold
saccule can result in retention cyst.
(True vocal cords).
The space between the right and left vestibular fold is called as
-
Rima vestlbulai and the space between vocal f o l d is called as Vocal Folds
Rima glottidis. It is t h e narrowest part of larynx. • Are t w o f o l d like structures which extend from the middle of
the angle of the thyroid cartilage t o the vocal process of the
arytenoids posteriorly.
Rima glottidis is the narrowest part of larynx in adults whereas in
infants the narrowest part of larynx is subglottic region.
Mucous membrane of larynx: The anterior surface and
upper half of the posterior surface of epiglottis, the upper
parts of aryepiglottic fold and the vocal folds are lined by
Hyoid bone non keratinizing stratified squamous epithelium. Rest of the
Thyrohyoid laryngeal mucous membrane is covered with pseudostratified
membrane ciliated columnar epithelium.

Lymphatic drainage o f t h e larynx

Thyroid - Above the glottis : To upper deep cervical nodes


cartilage - Below the glottis : Tolowerdeepcervicalnodechainthrough
the pre-laryngeal and pretracheal lymph
nodes
Vocal fold Glottis : Lymphatics in vocal cords are very scanty,
hence g l o t t i c carcinoma rarely shows
lympatic metastasis.

Cricdtracheal - Delphian node - Prelaryngeal LN's in the region of thyroid isthmus are
membrane called Delphian nodes.

Tracheal
cartilage Nerve Supply

• Superior laryngeal nerve: arises f r o m the inferior ganglion of


Fig. 12.4: Coronal section o f larynx vagus and receives a branch from superior cervical sympathetic
Courtesy: Textbook of Diseases of Ear, Nose and g a n g l i o n . It enters t h e larynx by piercing t h e t h y r o h y o i d
Throat, Mohan Bansal. Jaypee Brothers, p 63. membrane.
36 [_ SECTION IV Larynx

It divides at the level of greater cornu of hyoid into: | EXAMINATION OF LARYNX


Internal laryngeal nerve:
Sensory (It supplies the larynx above the vocal cords i.e. I n d i r e c t L a r y n g o s c o p y (IL)
supraglottic area) Done using a laryngeal mirror
Secretomotor
External laryngeal nerve—supplies cricothyroid muscle
The superior laryngeal nerve ends by piercing the inferior
Or Remember —The posterior rhinoscopy mirror is smaller and its
constrictor of pharynx and unites w i t h ascending branch shaft is bayonet shaped, while the shaft of the laryngeal mirror
is straight.
of recurrent laryngeal nerve. This branch is k/a Galen's
anastomosis and is purely sensory.
Structures which can be Blind areas which cannot be
Recurrent laryngeal nerve: visualized
visualized b y l L

M o t o r branch Sensory branch • Larynx (with trachea rings) • Laryngeal surface of epigottis/
• Parts of oropharynx (tongue infrahyord epiglottis
Supplies all the intrinsic muscles Supplies larynx below the level of
base and vallecula) • Ventricle of larynx
ofthe larynx except cricothyroid the vocal folds
• Hypopharynx/laryngopharynx • Subglottis
(which is supplied by external
laryngeal nerve, a branch of part viz. • Anterior commissure
- Pyriform sinus • Apex of pyriform fossa
superior laryngeal nerve).
- Posterier wall of
hypopharynx
- Postcricoid region

Both superior laryngeal nerves and recurrent laryngeal nerves are


branches of vagus nerves (CNX) which carry the fibres of cranial part
of accessory nerve (CNIX) The movenents of both the cords are observed when patient
takes deep inspiration (abduction of cords) and says "Aa"
• Laryngeal muscles: All muscles are paired except transverse (adduction cords) and "Eee" (for adduction and tension)
arytenoid

Action Muscle Responsible


Base of tongue
Abductor: • Posterior cricoarytenoid
Median
Adductor: • Lateral cricoarytenoid Vallecula glossoepiglottic fold
• Interarytenoid (transverse Lateral
Anterior
arytenoids) glossoepiglottic fold
commissure — Epiglottis
• Thyroarytenoid (external part)
Tracheal rings Ventricular band
Tensor: • Cricothyroid 11

Vocal cord
Relax vocal cord: • Thyroarytenoid (internal part)
Ary-epiglottic
• Vocalis
fold
Opener (ofthe laryngeal inlet): • Thyroepiglotticus Arytenoid
Closure ofthe laryngeal inlet: • Aryepiglotticus
• Inter arytenoids (oblique part)
Fig. 12.5: Structures seen on indirect laryngoscopy
Arterial Supply
D i r e c t L a r y n g o s c o p y (Fig. 12.6)
• Up to vocal folds: by superior laryngeal artery, a branch of
Done using a rigid endoscope
superior thyroid artery.
Position of patient - Boyce position/Barking-dog position
• Below vocal folds: by inferior laryngeal artery, a branch of
inferior thyroid artery. Contraindications
The cricothyroid artery is a branch of superior thyroid artery and • Cervical spine injury
passes across the upper part of cricothyroid ligament t o supply • Aneurysm of arch of aorta
the larynx. • Recent cardiac illness

Venous Drainage

Superior laryngeal vein —> Internal jugular vein In these condition and in voice disorders - Transnasal flexible
Inferior laryngeal vein —> Inferior thyroid vein
CHAPTER12 Anatomy of Larynx, Congenital Lesions of Larynx and Stridor

• If neck of sac is blocked and it gets infected, pyocele is formed.

Investigation
• X-ray: A n t e r o p o s t e r i o r v i e w w i t h a n d w i t h o u t valsalva
maneuver.

• Indirect laryngoscopy helps t o make the diagnosis.

Treatment
• Excision o f t h e saccule at its neck together w i t h removal o f t h e
upper half of thyroid lamina.
• Endoscopic marsupialization of internal laryngocele

NOTE
In adults laryngocele may be associated with saccule carcinoma.
Wt, ' :• v

10-15 cm Laryngomalacia/Congenital Laryngeal Stridor

Fig. 12.6: Direct Laryngoscopy Mostcommon congenital anomaly o f larynx. 0

Most commo n condition causing inspiratory stridor at or


Microlaryngoscopy shortly a f t e r b i r t h (within first 2 weeks of life).
0

In most cases, it is asymptomtic.


• Combination of laryngoscope and operating microscopy done
M:F = 1:1
for precision in surgeries on vocal cord. Focal length of the lens
There is a b n o r m a l flacidity o f laryngeal c a r t i l a g e . Stridor
0

of microscope used in microlaryngos copy = 400 m m .


occurs as a result of sucking of supraglottic structures into the
laryngeal inlet on inspiration
| CONGENITAL LESIONS O F LARYNX AND STRIDOR
Manifests afterbirth (within first 2 weeks of life) and may persist
t h r o u g h o u t infancy. (peak age - 6-9 months)
0

Laryngocele
Usually disappears by t w o years of age. 0

Definition Inspiratory stridor is worse during exertion such as crying and


Laryngocele is an air-filled cystic swelling due t o dilatation o f feeding so stridor is i n t e r m i t t e n t . 0

saccule.The saccule is a diverticulum of mucous membrane which Strangely, stridor worsens during sleep, and positional variations
starts f r o m the anterior part of venticular cavity and extends upward occur—stridor is worse when patient is in supine position.
between vestibular folds and lamina of thyrid cartilage. When it It decreases when child is placed in prone position and in
abnormally enlarges, it forms the air containing sac - Laryngocele.
hyperextension.
Sometimes associated w i t h cyanosis - (Dhingra 5th/ed, p 34)
Type
Cry is normal.
Laryngoscopy f i n d i n g — O m e g a shaped e p i g l o t t i s . 0

External 3 0 % Internal 2 0 % Mixed type 5 0 %


Aryepiglottic folds are tall, t h i n and foreshortened.
Sac arises f r o m t h e The dilatation Treatment is conservative:
laryngeal ventricle and remains confined to Reassure the patient.
expands into the neck larynx Early antibiotic therapy for URI.
through the thyrohyoid
1 0 % patients need surgical intervention which includes
membrane 0

supraglottoplasty (ary epiglottoplasty).


• In severe cases, tracheostomy may be needed. 0

Causes
Raised transglottic air pressure as in t r u m p e t players, glass blowers
or w e i g h t lifters. Childrentwith laryngomalacia have high prevalence of gastro
esophageal reflux disease (50-100%) and second synchronous
Clinical Features airway lession (17%) i
• Majority cases are asymptomatic.
• The internal laryngocele produces hoarseness o f voice and
Laryngeal Web/Atresia
may produce dyspnea due t o pressure changes.
• The external laryngocele presents as a cystic swelling in neck • Mostly congenital but may be acquired.
w h i c h increases in size on coughing or performing Valsalva • Congenital web is due t o incomplete recanalization of larynx.
• It presents w i t h hoarseness, cough and if large - obstruction • Mostcommon site: Anterior 2/3rds o f t h e vocal cord.
t o t h e airway. • Webs have a concave posterior margin.
138^ SECTION IV Larynx

Symptom • Thin web - cut w i t h a knife or C 0 laser


2

The child presents w i t h congenital airway obstruction (stridor), • Thick web-Excision via laryngofissure followed by placement
weak cry or aphomia. of silicon keel (MC Naughter keel) and subsequent dilation.

Stridor
All patients need genetic screening and cardiovascular evaluation It is noisy respiration due t o upper airways obstruction (i.e. f r o m
especially of aortic arch. external naves up t o trachea. Causes of stridor have been given in
flow charts 12.1.
Treatment
• Tracheostomy - often required

Causes of stridor

r
A-l Children (laryngeal causes) A-ll Children ( Extra laryngeal causes)
1
B Adults

T I T T f
1

Congenital Inflammatory Traumatic Neoplasm Foreign bodies Neurological disorders


Infantile larynx • Acute laryngitis • Birth trauma • Papillomas • Vocal card palsy
Laryngeal web/stenosis • Diphtheria • Burns and scalds • Cyst • Tetany
Congential • Allergic edema • Accidental • Tumors
hemangioma/cysts • Laryngotracheo
Laryngomalacia bronchitis (croup)

E x t r a l a r y n g e a l c a u s e s in c h i l d r e n •

A-ll Children (Extralaryngeal causes)

r T T 1
b c •

(Congenital) (Neoplasm) (Traumatic) (Foreign body) (Inflammatory) •

• Pierre Robin syndrome • Submental cellulitis


• To fistula • Neck abscesses
• Vascular loops (Retropharyngeal)

C a u s e s in a d u l t s

Adults
I
I 1
a b c Others
Infective Traumatic Allergic Neoplastic Neurological Tetany
• Edema of larynx • Angioneurotic • Laryngeal cancer • Bilateral abductor I (calcium)
• Epiglottitis edema • Thyroid palsy
neoplasm
• Neck • Medias

Flow chart 12.1 t o show causes of stridor.


• In children, chronic stridor is due to congenital lesions, mostly due to laryngomalacia.


• In children acute stridor is mostly due to acute upper respiratory tract infection.
• In adults stridor is uncommon chronic stridor may indicate laryngeal carcinoma.
CHAPTER 12 Anatomy of Larynx, Congenital Lesions of Larynx and Stridor

IMPORTANT CLINICAL CONCEPTS FOR NEET


• The only intrinsic muscle o f t h e larynx which lies outside the laryngeal cartilages ciliated columnar
framework is cricothyroid.
• In thyroidectomy, the nerve c o m m o n l y injured is external branch of superior
laryngeal nerve.
• Posterior cricoarytenoid is the only abductor of vocal cord. Reinke's space
• Epiglottis is omega shaped in neonates and infants. (Lamina propria)
• Vocal cords have practically no lymphatics except for a small delphain node which
Vocal ligament
lies on cricothyroid membrane (lymphatic watershed of larynx).
• Aryepiglottic fold has the richest lymphatic supply in larynx.
• Keyhole glottis is seen in thyroarytenoid weakness.
Vocalis muscle
• Flag sign is seen in bilateral adductor palsy. (Thyroarytenoid)
• In examination o f neck, absence o f laryngeal crepitus indicates a postcricoid
g r o w t h or an abscess in the postcricoid area.
• Thyroid cartilage is hyaline cartilage and so calcifies - calcification occurs earlest
in it. It starts by 20 years of age and is fully ossified by 7th decade of life. Thyroid
cartilage calcifies in a figure of 8 pattern. Malignancies of larynx which invade
thyroid cartilage destroy this pattern.
• Reinke's space: This potential space has scanty subepithelial connective tissues and lies under the epithelium of vocal cords. It is
b o u n d e d by¬
- Above and below: Arcuate lines.
Anterior: Anterior commissure.
Posterior: Vocal process of arytenoids.
Reinke's edema: Edema of Reinke's space results in fusiform swelling o f t h e membranous vocal cords.



-

• .


140[

QUESTIONS

1. All of the following are paired except: [PGI Nov 05] 14. Laryngocele arises from: [AIIMS May 05,08]
a. Interarytenoids b. Corniculate a. Anterior commissure b. Saccule of the ventricle
c. Vocal cords d. Cricothyroids c. True cords d. False cords
e. Thyroid 15. Laryngocele arises as herniation of laryngeal mucosa
2. Laryngeal cartilage forming complete circle: [TN08] through the following membrance: [Al 06]
a. Arytenoid b. Cricoid a. Thyrohyoid b. Cricothyroid
c. Thyroid d. Hyoid c. Cricotracheal d. Crisosternal
3. True about larynx in neonate: [PGI 03] 16. Most common congenital anomaly of larynx:
a. Epiglottis is large and omega shaped [TN 99; Delhi-08]
b. Cricoid narrowest part a. Laryngeal web b. Laryngomalacia
c. It extends C4,5,6 vertebrae c. Laryngeal stenosis d. Vocal and palsy
d. Tongue is small in comparison to oral cavity 17. Regarding laryngomalacia: [PGI 02]
e. Funnel shaped a. Most common cause of stridor in newborn
4. Narrowest part of infantile larynx is: [Assam 95, RJ 05] b. Omega-shaped epiglottis
a. Supraglottic b. Subglottic c. Inspiratory stridor
c. Glottic d. None o f t h e above d. Requires immediate surgery
5. Abductor of vocal cord is: [Kerala 95] e. Stridor worsens on lying in prone position
a. Cricothyroid b. Posterior cricoarytenoid 18. Which is not true about laryngomalacia? [Al 12]
c. Lateral cricoarytenoid d. Cricohyoid a. Omega-shaped epiglottis
6. All are elevators of larynx except: [AP04] b. Stridor increases on crying, but decreases on placing the
a. Thyrohyoid b. Digastric child in prone position
c. Stylohyoid d. Sternohyoid c. Most common congenital anomaly o f t h e larynx
7. Sensory nerve supply of larynx below the level of vocal d. Surgical management ofthe airway by tracheostomy is the
cordis: [AIIMS 98; Al95] preferred initial treatment
a. External branch of superior laryngeal nerve 19. About laryngomalacia, all are true except: [PGI 08]
b. Internal branch of superior laryngeal nerve a. MC neonatal respiratory lesion
c. Recurrent laryngeal nerve b. Decreased symptoms during prone position
d. Inferior pharyngeal c. Self-limiting by 2-3 years of age
8. Supraglottis includes all o f t h e following except: d. Omega-shaped epiglottis seen
a. Aryepiglottic fold e. Surgery is treatment of choice
b. False cord 20. Most common mode of treatment for laryngomalacia is:
c. Lingual surface of epiglottis
[UP 07]
d. Laryngeal surface of epiglottis
a. Reassurance b. Medical
9. Epilarynx include (s): [PGI Nov 10]
c. Surgery d. Wait and watch
a. Suprahyoid epiglottis b. Infrahyoid epiglottis
21. MC cause of intermittent stridor in a 10-day-old child
c. False cords d. Posterior commissure
shortly afterbirth is: [Al 01; AIIMS 95]
10. The water cane in the larynx (saccules) are present in:
a. Laryngomalacia b. Foreign body
[UP 07]
c. Vocal nodule d. Hypertrophy of turbinate
a. Paraglottic space b. Pyriform fossa
22. Most common cause of stridor in children is: [UP 07]
c. Reinke's space d. Laryngeal ventricles
a. Laryngomalacia b. Congenital laryngeal paralysis
11 Vocal cord is lined by: [Delhi 96]
c. Foreign body in larynx d. Congenital laryngeal tumors
a. Stratified columnar epithelium
23. Causes of congenital laryngeal stridor is/are: [PGI 00]
b. Pseudociliated columner epithelium
c. Stratified squamous epithelium a. Laryngomalacia b. Laryngeal papillomatosis
d. Cuboidal epithelium c. Subglottic papilloma d. Laryngeal stenosis
12., Inlet of larynx is formed by: [Kolkata 03] e. Hemangioma of larynx
a. Ventricular fold b. Aryepiglottic fold 24. Main treatment of congenital laryngeal stridor is:
c. Glossoepiglottic fold d. Vocal cord [Jipmer 04]
13. A neonate while suckling milk can respire without dif- a. Tracheostomy
ficulty due to: [AIIMS Nov 10] b. Steroid therapy
a. Start sofl palate b. Small tonque c. Reassurance to the child's parents
c. High larynx d. Small pharynx d. Amputating epiglottis


I

CHAPTER 12 Anatomy of Larynx, Congenital Lesions of Larynx and Stridor J 141


25. Stridor is caused by all except: [AP 96] 32. Microlaryngoscopy was started by: [MH 03]
a. Hypocalcemia b. Asthma a. Bruce Benjamin b, Kleinsasser
c. Epiglotitis d. Laryngeal tumor c. Chevalier Jackson d. None
26. A 2-year-old boy presenting with sudden severe dyspnea, 33. The procedure that should precede microlaryngoscopy
most common cause is: [Bihar 06] is: [AI91]
a. Foreign body b. Bronchiolitis a. Pharyngoscopy b. Esophagoscopy
c. Asthmatic attack d. None c. Rhinoscopy d. Laryngoendoscopy
27. Stridor in adults is most commonly caused by: [Delhi 96] 34. Laryngeal mirror is warmed before use by placing:
a. Reinke's edema b. Malignancy [Karn. 89]
c. Acute severe asthma d. Toxic gas inhalation a. Glass surface on flame b. Back of mirror on flame
28. The most com mon cause of laryngeal stridor in a 60-year- c. Whole mirror into flame d. Mirror in boiling water
old male is: [JIPMER91]
a. Nasopharyngeal carcinoma
b. Thyroid carcinoma NEET PATTERN QUESTIONS
c. Foreign body aspiration
d. Carcinoma larynx 35. Abbudctor of vocal cord is: [NEET Pattern]
a. Posterior cricoarytenoid b. Lateral cricoarytenoid
29. Laryngofissure is: [Jipmer 04]
c. Cricothyroid d. None of the above
a. Opening the larynx in midline
36. Which of the following is the only intrinisic muscle of
b. Making w i n d o w in thyroid cartilage
larynx that lies outside the laryngeal framework:
c. Removal of arytenoids
[NEET Pattern]
d. Removal of epiglottis
a. Cricothyroid
30. In an direct laryngo scopy which of the following can be
b. Superior constrictor
visualized: [PGI Dec 01]
c. Cricopharyngeus
a. Cricothyroid b. Lingual surface of epiglottis
d. Lateral cricothyroid
c. Arytenoids d. Pyriform fossa
37. Palpatory thud, audible slap is seet in: [NEETPattern]
e. Tracheal cartilage
a. Tracheal foreign body
31. Which ofthe following is difficult to visualize or examine
b. Bronchial foregion body
on indirect laryngoscopy? [MH-PGM-CET07;MH08]
c. Laryngeal foreign body
a. True vocal cord b. Anterior commmissure
d. None
c. Epiglottis d. False vocal cord

EXPLANATIONS AND REFERENCES

1. Ans. is a and e i.e. Interarytenoid; and Thyroid


Ref. BDC, Vol 3,4th/ed pp 240,244; Scptt-Brown's 7th/ed Vol 2 p 2133; Dhingra 5th/edp 299,6th/ed p 282; Mohan Bansal p 62
• All intrinsic muscles of larynx are paired except transverse arytenoid/interarytenoid.
• As far as cartilages of larynx are concerned 3 are paired and 3 are unpaired.

• Unpaired cartilage Paired cartilage


Thyroid cartilage Arytenoid
Cricoid Corniculate
Epiglottics Cuneiform

Vocal cords are also paired structures.


2. Ans. is b i.e. Cricoid Ref. BDC, Vol III, 4th/ed p 240; Dhingra 5th/ed p 299,6th/ed p 282; Mohan Bansal p 62

Cartilage
Thyroid cartilage V shaped on cross section. Has 2 lamina right and left which are placed at an angle of 90° in males and 120° in females
Cricoid cartilage Ring shaped, (it is the only complete ring present in the air passages)
Epiglottic cartilage Leaf shaped in adults, omega shaped in infants and neonates
Arytenoid cartilage Pyramid shaped
Corniculate cartilage Cone shaped
Cuneiform cartilage Rod shaped
142^ • •
SECTION IV Larynx

Also know: The thyroid, cricoid and basal parts of arytenoid cartilages are made up of hyaline cartilage. They ossify after the age
of 25 years. The other cartilages, e.g. epiglottis, corniculate, cuneiform and processes o f t h e arytenoid are made of elastic cartilage
and do not ossify.
3. Ans. is a, b and e i.e. Epiglottis is large and omega shaped; Cricoid narrowest part; and Funnel shaped
Ref. Miller Anaesthesia Sth/ed p 2090; Tuli ist/ed p 284; Scott-Brown's 7th/ed Vol 2 p 2131; Mohan Bansal p 67; Dhingra 6th/ed p 285

Infant's Larynx Differs from Adult in:


1. It is situated high up (C2 - C4).° (in adults = C3 - C6)
2. Of equal size in both sixes (in adults it is larger in males)
3. Larynx is funnel shaped
4. The narrowest part ofthe infantile larynx is the junction of subglottic larynx with trachea and this is because cricoid cartilage is very small
5. Cartilages:
a. Epiglottis is omega shaped, soft, large and patulous.
b. Laryngeal cartilages are soft and collapse easily
c. Thyroid cartilage is flat
d. Arytenoid cartilage is relatively large
6. The cricothyroid and thyrohyoid spaces are narrow
7. The submucosal tissue is thick and loose and becomes oedematous in response inflammation
8. Vocal cords are angled and lie at level of C8
9. Trachea bifurcates at level of T9

Narrowest part of adult larynx is Rima Glottidis.


4. Ans. is b i.e. Subglottis Ref. Scott-Brown's 7th/ed Vol 2 p 2131; Dhingra 5th/edp 303,6th/ed p 285
The infantile larynx
" The diameter of cricoid cartilage is smaller than the size of glottis, making subglottis the narrowest part." - Dhingra 5th/ed p 303
"Rima glottidis (Glottis) is the narrowest part of larynx in adults whereas in infants the narrowest part of larynx is subglottis region."
—Mohan Bansal p 6
5. Ans. is b i.e. Posterior cricoarytenoid Ref. BDC, Vol 3,4th/ed p 245; Dhingra 5th/ed p 300, 6th/ed p 283
Remember: Posterior cricoarytenoid is the only abductor of vocal cord.
Adductors of vocal cord are:
T = Thyroarytenoid
A = Transverse arytenoid
L = Lateral cricoarytenoid

-
nemonic
Add TALC i.e. Adductors are TALC.

6. Ans. is a i.e. thyrohyoid Ref. BDC4th/ed Vol3p 243 Table 16.2; Mohan Bansal p 66
Elevation o f larynx is carried o u t by - Thyrohyoid and mylohyoid - BDC4th/ed Vol3p 243

Movement Muscles
1. Elevation of larynx Thyrohyoid, mylohyoid
2. Depression of larynx Sternohyoid, sternothyroid, omohyoid
3. Opening the inlet of larynx Thyroepiglotticus
4. Closing of inlet of larynx Aryepiglotticus
5. Abductor of vocal cord Posterior cricoarytenoids
6. Adductor of vocal cord T-Thyroarytenoid
A - Transverse arytenoids
L - Lateral cricoarytenoid
C - Cricothyroid
Tensor of vocal cord Cricothyroid
Relaxor of vocal cord Thyroarytenoid

-
CHAPTER 12 Anatomy of Larynx, Congenital Lesions of Larynx and Stridor

A L S O K N O W - According to Dhingra 5th/ed p 301,6th/ed p 284

Elevators of larynx can be

r 3
Primary Secondary
Primary act directly and include Act indirectly as they are attached to hyoid bone
Thy - Thyrohyoid Mylohyoid (main)
Style - Stylopharyngeus Stylohyoid
Shall - Salpingopharyngeus Geniohyoid
Prevail - Palatopharyngeus Digastric

Ans. is c i.e. Recurrent laryngeal nerve Ref. BDC, Vol 3,4th/edp 246; Mohan Bansalp 66; Dhingra 6th/edp 298
Nerve supply of larynx
• Sensory:
- The internal laryngeal nerve supplies the mucous membrane up t o the level o f t h e vocal folds.
- The recurrent laryngeal nerve supplies below the level o f t h e vocal folds.
. Motor:
- All intrinsic muscles o f t h e larynx are supplied by the recurrent laryngeal nerve except for the cricothyroid
which is supplied by the external laryngeal nerve.
Ans. is c i.e. Lingual surface of epiglottis Ref. Logan Turner Wth/edp 171
• The lingual surface of epiglottis and vallecula are a part of oropharynx according t o Logan Turner Wth/edp 171
• According t o Dhingra 6th/edp307
"Whole of epiglottis is included in supraglottic area."
• According to Scott-Brown's 7th/ed Vol 3 p 2132- w h o l e of epiglottis is included in the supraglottic.
• But since here we have t o choose one o p t i o n . Therefore, I am going w i t h Turner.
Ans. is a a n d c i.e. Suprahyoid epiglottis and Arytenoids
Ref. Dhingra 6th/ed p 307; Stell and Manran's
Head and Neck Surgery 4th/ed p 233
• The larynx is divided into supraglottis, glottis and subglottic region for
the purpose o f classification o f its tumor. Pyriform fossa

The division is based on the lymphatic drainage. False cord


The glottic area has virtually n o lymphatic drainage and
Supraglottis
so acts as a w a t e r s h e d . The area above t h e g l o t t i s , i.e. Ventricle
supraglottic drains upward via superior lymphatics into t h e Glottis; 1.0 cm
True cord
upper deep cervical group of nodes whereas the area below . i
the vocal cords, i.e. subglottic, drains t o the prelaryngeal and Subglottis
pretracheal glands.

Classification of sites and various subsities under each site in larynx (AJCC classification, 2002)

Site Subsite
Supraglottis • Suprahyoid epiglottis (both lingual and laryngeal surfaces)
• Infrahyoid epiglottis •
• Aryepiglottic folds (laryngeal aspect only)
• Arytenoids
• Ventricular bands (or false cords) •

Glottis True vocal cords including anterior and posterior commissure


Subglottis Subglottis up to lower border of cricoid cartilage

Suprahyoid epiglottis, infrahyoid epiglottis, aryepiglottic folds and arytenoids together are called epilarynx

10. Ans. is d i.e. Laryngeal ventricles Ref. Dhingra 5th/edp301,6th/edp 284; BDC, Vol 3,4th/edp 242; Mohan Bansalp 64,65
It is a diverticulum of mucous membrane which starts from t h e anterior part of laryngeal ventricle extending between t h e vestibular
folds and lamina of thyroid cartilage. The saccule has plenty o f mucous glands whose main purpose is t o lubricate t h e vocal cords
(vocal cord is devoid of mucous glands) and hence is k n o w n as water can of larynx.
144^ SECTION IV Larynx

When distended the saccule can protrude through the thyrohyoid membrane in the neck and is known as Laryngocele.

A L S O KNOW
Boyer's space - another name for the pre-epiglottic space which lies in front of epiglottic beneath the hyoid bone.

11. Ans. is c i.e. Stratified squamous epithelium


Ref. Dhingra5th/edp 302,6th/edp285;Scott-Brown's 7th/ed Vol2p2137;Mohan Bansalp65
Whole of larynx is lined by ciliated columnar epithelium except the vocal cords and upper part of vestibule which is lined by strati-
fied squamous epithelium.

ALSO KNOW

Mucous glands are distributed all over the larynx except the vocal cords, which is lubricated by mucus from glands w i t h i n the sac-
cule. The squamous epithelium o f vocal fold is, therefore prone t o desiccation if these glands cease t o function as in radiotherapy.
12. Ans. is b i.e. aryepiglottic fold Ref. BDC 4th/ed Vol 3 p 242; Dhingra 6/e p 284
Inlet of the larynx is bounded by:
Anteriorly - Epiglottis
Posteriorily - Interarytenoid fold of mucous membrane
On each side by - Aryepiglotic fold
13. Ans. is c i.e. High larynx Ref. Dhingra 6th/ed p 285
Infant's larynx is positioned high in the neck level of glottis being opposticto C3 or C4 at rest and reaches C1 or C2 during swal-
lowing. This high position allows the epiglottis t o meet soft palate and make anasopharyngeal channel for nasal breathing during
suckling.The milkfeed passes separately over the dorsum of t o n g u e and the side of epiglottis, thus allowing breathing and feeding
t o go on s i m u l t a n e o u s l y .
14. Ans. is b i.e. Saccule of the ventricle Ref. Dhingra 5th/ed p 34, 6th/ed p 295
15. Ans. is a i.e. Thyrohyoid Ref. Turner 10th/ed p 168; Mohan Bansal Ist/ed p 487
Laryngocele is an air-filled cystic swelling which occurs due to dilatation of saccule.
Saccule is a diverticulum arising from anterior part of ventricle/sinus of larynx.
0

External laryngocele is one in which distended saccule herniates through the thyrohyoid m e m b r a n e and presents as a reduc-
0

ible swelling in the neck, which increases in size o n coughing or performing Valsalva.
16. Ans. is b i.e. Laryngomalacia
17. Ans. is a, b and c i.e. Most common cause of stridor in newborn; Omega shaped epiglottis; and Inspiratory stridor
18. Ans. d i.e. Surgical management of the airway by tracheostomy is the preferred initial treatment
Ref. Dhingra 5th/edp314,6th/edp 285; Turner 10th/ed, pp 385,386; Current Otolaryngology 2nd/ed, pp 462,463; Mohan Bansal p514

Laryngomalacia

• It is the M/C congenital anomaly of the larynx


• It is the M/C condition causing inspiratory stridor afterbirth.
• The stridor worsens during sleep and when baby is in supine position (not in prone position). Rather when the child is placed
in prone position it is relieved.
• On laryngoscopy - Epiglottis is omega shaped and aryepiglottis folds are floppy.

Treatment

Conservative Management
19. Ans. is b a n d e i.e. Decreased symptoms during prone position and Surgery is treatment of choice
Ref. Dhingra 5th/edp 314; Current Otolaryngology 2nd/ed p 462; Mohan p514

Option Correct/ Incorrect Reference Explanation


Option a Not sure Current 2nd/ed p462 Laryngomalacia is the most common cause
M/C neonatal respiratory lesion of stridor in infants and is also the most
common congenital laryngeal abnormality
but whether it is M/c neonatal respiratory
lesion is not sure.

Contd..
CHAPTER 12 Anatomy of Larynx, Congenital Lesions of Larynx and Stridor

Contd.

Option Correct/ Incorrect Reference Explanation

Option b Incorrect Current Otolaryn-gology 2 n d / - Stridor in laryngomalacia is intermittent &


Decreased s y m p t o m s d u r i n g ed p 4 6 2 ; Dhingra 5th/ed p 314, not constant.
prone position 6th/edp295 - It is worse during sleep or when patient
is in supine position
It is improved in - prone position
Option c Correct Dhingra 5th/ed p 314; Current "It manifests at birth or soon after and
Self limiting by 2-3 years of age Otolaryngology 2nd/ed p 462 usually disappears by 2 years of age"
- Dhingra 5th/ed p 314,6th/ed p 295
Option d - Correct Dhingra 5th/ed p 314, 6th/ed p On direct laryngoscopy
Omega shaped epiglottis seen 295; Current Otolaryngology 2nd/ - Epiglottis is omega shaped
ed p 462 - Aryepiglottic folds- floppy
- Arytenoids - prominent

Option e Incorrect Dhingra 5th/ed p 314, 6th/ed p Treatment is mostly conservative


Surgery is T/t of choice 295; Current Otolaryngology 2nd/ Surgery is required in only 10% cases
ed p 462 Indications o f surgery
- Severe stridor
- Apnea
- Failure to thrive
- Pulmonary hypertension
- Corpulmonale

20. Ans. is a i.e. Reassurance Ref. Dhingra 5th/ed p 314,6th/edp 295; Turner 1 Oth/edp 386; Current Otolaryngology 2nd/ed p 463
In most patients laryngomalacia is a self-limiting condition.
Treatment of laryngomalacia is reassurance to the parents and early antibiotic therapy for upper respiratory tract infections.
Tracheostomy is required only in severe respiratory obstruction.
Surgical intervention (supraglottoplasty i.e. reduction of redundant laryngeal mucosa) is indicated for 1 0 % of patients. Main indica-
tions for surgery are:
• Severe stridor
• Apnea
• Failure to thrive
• Pulmonary hypertension
• Corpulmonale
21. Ans. is a i.e. Laryngomalacia Ref. Turner lOth/ed, p 385; Current Otolaryngology 2nd/edp 462
Laryngomalacia is the most common cause of inspiratory stridor in neonates.
The stridor in case of laryngomalacia is not constantly present, rather it is intermittent. So laryngomalacia is also the M/C cause of
intermittent stridor in neonates.
22. Ans. is c i.e. Foreign body in larynx Ref. Ghai 6th/ed, p 341
Read the question carefully.
It says most c o m m o n cause of stridor in children—which is not laryngomalacia, it usually resolves spontaneously by t h e age of 2
years and is rare after that.
"Foreign body aspiration should always be considered as a potential cause of stridor and airway obstruction in children."
- Ghai 6th/ed,p 341

A L S O KNOW
Most common causes of chronic stridor in children is long-term intubation causing laryngotracheal stenosis.

23. Ans. is a, d and e i.e. Laryngomalacia; Hemangioma of larynx; and laryngeal stenosis
Ref. Tuli Ist/edp 295; Current Otolaryngology 2nd/edp 463; Mohan Bansalp 474

1 Causes of congenital laryngeal stridor:


• Infantile larynx • Laryngeal web/stenosis • Vocal cord paralysis
• Laryngomalacia • Laryngeal cyst • Cricoarytenoid joint fixation
• Congenital hemangioma (subglottic)
• Posterior laryngeal cleft
146^ SECTION IV Larynx

24. Ans. is c i.e. Reassurance to child's parent Ref. Dhingra 5th/ed p314,6th/ed p 295
Congenital laryngeal stridor is synonymous w i t h laryngomalacia. Hence, management remains the same i.e. reassurance t o childs
parent.
25. Ans. is b i.e. Asthma Ref. Dhingra 5th/ed p 315
• First you should know what exactly upper and lower airway means:
i. Upper airway: The airway from the nares and lips t o the lower border of larynx (includes nose, pharynx, larynx).
ii. Lower airway: From the lower border of the terminal bronchioles (includes various level of bronchioles up t o terminal
bronchioles).
• Stridor usually implies upper airway obstruction, so the level of obstruction is above the level of trachea (P) (from nares
t o the larynx).
• Wheezing andronchi are signs of lower airway obstruction.
Epiglottitis and laryngeal tumors are common causes of stridor and do not need explanation.
Hypocalcemia leads t o tetany which causes stridor.
Asthma leads to wheezing or ronchi (lower airway obstruction)
Also know - Stridor is a harsh noise produced by t u r b u l e n t air flow t h r o u g h a partially obstructed upper airway.
It can be:
- Inspiratory i.e. originates from supraglottis glottis and pharynx
- Expiratory i.e. originates f r o m thoracic trachea
- Biphasic i.e. originates from subglottis and cervical trachea
Hence, stridor is mainly of laryngeal and tracheal origin.
26. Ans. is a .i.e. Foreign body Ref. Scott-Brown's 7th/ed Vol 1 pll 17; Dhingra 5th/edp315,6th/edp295
In case of stridor w i t h acute airway obstruction (i.e. dyspnea) always history of any foreign body ingestion should be taken.
27. Ans. is b i.e. Malignancy Ref. Read below
The answer to this question can be derived by exclusion.
Reinke's edema leads to hoarseness of voice and not stridor. (Dhingra 5th/ed, p311,6th/ed p 292) Ruling out option 'a'.
° Acute severe asthma also does not lead of stridor.
• Toxic gas inhalation does not lead t o stridor. So we are left w i t h one o p t i o n i.e. malignancy.
28. Ans. is d i.e. Carcinoma larynx Ref. Dhingra 5th/edpp 315-317,6th/edp 296-297; Mohan Bansal p 474
Most common cause for stridor in 60 years old male will be carcinoma larynx as carcinoma larynx occurs in males (predominantly) at the
age of40-70years.
Mostcommon and earliest s y m p t o m of subglottic cancer is stridor.

• Nasopharyngeal cancer does not lead to stridor


• Thyroid cancer causes stridor rarely.
• Foreign body aspiration is a common cause of stridor in children and not adults.

29. Ans. is a i.e. Opening the larynx in midline Ref. Stedman Dictionary, p 937
Laryngofissure: Opening the larynx in midline.
30. Ans. is a , b, c, d and e i.e. Cricopharynx; Lingual surface of the epiglottis; Arytenoids; Pyriform fossa; a n d Tracheal
cartilage. Ref. Dhingra 5th/ed p 432,6th/ed p 384; Tuli 1 st/ed, p 527
Structures seen on Indirect laryngoscopy are:
• Larynx: Epiglottis, aryepiglottic folds, arytenoids, cuneiform and corniculate cartilage, ventricular ands, ventricles, true cords,
anterior commissure, posterior commissure, subglottis and rings of trachea.
• Hypopharynx: Both pyriform fossae, post-cricoid region, posterior wall of laryngopharynx.
• Oropharynx: Base of tongue, lingual tonsils, valleculae, media and lateral glosso-epiglottic folds.

In indirect laryngoscopy - The hidden ares of larynx viz. Anterior Commisure, Ventricle and Subglottic area are not seen properly.

31. Ans. is b i.e. Anterior commissure Ref. Dhingra 5th/edp 432,6th/ed p 384 p 70; Tuli Ist/ed, p 527; Mohan Bansal p 70
Hidden areas of larynx viz. infrahyoid epiglottis, anterior commissure, ventricles and subglottic region and apex of pyriform fossa
are difficult t o visualize by indirect laryngoscopy.
32. Ans. is b i.e. Kleinsasser Ref. Maqbool 11st/edp 323
"The presen t day microsurgical techniques ofthe larynx are a credit to Kleinsasser." - Maqbool! 1 th/ed p 323
CHAPTER 12 Anatomy of Larynx, Congenital Lesions of Larynx and Stridor

33. Ans. is d i.e. Laryngoendoscopy Ref. Scott-Brown's 7th/ed Vol 2 p 2236


The answer is not given directly but the following lines of Scott-Brown's leave no d o u b t about the answer—"Microlaryngoscopy
concentrates mainly on the glottic area in cases where the diagnosis is already established and unlike direct laryngoscopy, is not
primarily concerned w i t h other areas of larynx which should have been assessed preoperatively."
It is clear direct laryngoscopy (or laryngoendoscopy as given in the options) should always be done prior to microlaryngoscopy.
34. Ans. is a i.e. Glass surface on flame Ref. Tuli Ist/ed p 234
Laryngeal mirror is warmed by:
• Dipping t h e mirror in warm water.
• Heating the glass surface against some heat such as bulb or spirit lamp.
35. Ans. is a i.e. Posterior crico arytenoid Ref. Dhingra 6th/edp283
Posterior crico arytenoids are the only abductors of vocal cord.
Adductors o f vocal cord can be memorised by mnemonic TALC as discussed earlier
36. Ans. is a i.e. Cricothyroid Ref. Dhingra 6th/edp 284, Fig's 56.5,56-6
Cricothyroid muscle is the only intrinsic muscle w h i c h is supplied by external laryngeal nerve and lies outside t h e laryngeal
framework
37. Ans. is a i.e. Tracheal foreign body Ref. Dhingra 6th/ed p 321
A foreign body in trachea may move up and d o w n the trachea between the carnia and the undersurface of vocal cords causing
"audible slap" and "palpatory thud."
Symptoms and signs of foreign bodies at different levels

Site o f f o r e g i n bodies Symptoms and signs


Larynx • Complete obstruction leading to death
• Partial obstruction: stridor, hoarseness, cough, respiratory
difficulty
Trachea • Choking, striodor, wheeze, cough palpatory thud, audible slap
Bronchi • Cough, wheeze and diminished air entry to lung forms a "triad"
• Respiratory distress with swelling of foreign body
• Lung collapse, emphysema, pneumonitis, bronchiectasis or lung
abscess are late features

-
-

- i - !


-


Acute and Chronic Inflammation


CHAPTER
of Larynx, Voice and
Speech Disorders

| ACUTE LARYNGOTRACHEOBRONCHIT1S (CROUP) Investigation

• X-ray:"Steeple sign" i.e. symmetric steeple or funnel-shaped


• It is a dangerous infection seen mostly in children which i n -
narrowing of subglottic region.
volves whole of tracheobronchial tree.

Organism Treatment

• Mostly viruses (parainfluenza type 1 and 2 and influenza A). • Broad-spectrum penicillin (for secondary bacterial infecton)
• In adults it can be caused by: • IV steroids, if child is in distress.
H. simplex • Humidified air
Cytomegalovirus . IV fluids
Influenza virus • Nebulization w i t h adrenaline
Superimposed bacterial infection [Hemolyticstreptococci] In despite above measures respiratory o b s t r u c t i o n increases
usually occurs intubation/tracheostomy is done.

Features
Indications for Intubation
• Age g r o u p — m o s t common in 6 months t o 3 years although
« Rising Co level
2

children < 7 years are susceptible. > Worsening neurologic status


• Male > Female > Decreasing respiratory rate

Pathology

• Mucosal swelling especially in s u b g l o t t i c area. Subglottic | ACUTE EPIGLOTTITIS (SUPRAGLOTTIC LARYNGITIS)


edema is most characteristic pathological f e a t u r e0

• Production o f thick tenacious mucus w h i c h can hardly be • It is acute inflammatory condition of the supraglottic struc-
expectorated. tures viz.
• Pseudomembrane formation Epiglottis
• All these can lead t o airway obstruction. Aryepiglottic fold and arytenoids
• Most common organism in children: H. influenza—type B
Clinical Features • In adults it can be caused by:
Group A streptococci, S. pneumoniae, S. aureus, Klebsiella
Onset is gradual w i t h prodrome of upper respiratory symptoms
pneumoniae
Fever usually low grade
Recently, Neisseria meningitidis has been recognized as a
Painful croupy cough (barking cough)
cause of fulminant life threatening supraglottitis.
Hoarseness and stridor (initially inspiratory; then biphasic)
Upper Airway obstruction which is visible in the form of supra- Clinical Features
sternal and intracostal recession.

?
• Age g r o u p — m o s t l y seen in 3-6 years but can occur in adults
also.
• There is usually a short history w i t h rapid progression.
• Acute laryngotracheo bronchitis is the M/C cause of infectious • Starts w i t h URI and fever (sometimes > 40°C).
\^ respiratory obstruction in children
CHAPTER 13 Acute and Chronic Inflammation of Larynx, Voice and Speech Disorders J 149
• Sore throat and dysphagia are the most c o m m o n presenting • Exact cause is not k n o w n .
symptoms in adults. Can be due to: • Repeated attacks of acute inflammation
• Dyspnea and stridor are the most c o m m o n presenting symp- • Smoking
toms in children. • Voice abuse
• Child prefers sitting position w i t h hyperextended neck (tripod • Pollution
sign). • Chronic cough
• Drooling of saliva present as child has dysphagia. • Chronic sinusitis
• Voice is not affected.
• Stridor is u n c o m m o n in adults but tachycardia which is dispro- Types of Chronic Laryngitis
portionate to pyrexia is an i m p o r t a n t sign which preceedes
• Hyperemic
airway obstruction.
• Hypertrophic
The pseudostratified ciliated epithelium changes t o squamous
Signs
type. There may be hyperplasia and keratinization (leukoplakia of
• Epiglottis f o u n d cherry red and swollen on indiect laryngos- squamous epithelium of the vocal cords).
copy

II.
• Care should be taken when depressing the tongue for examina- CONTACT ULCERS/PACHYDERMIA LARYNGITS/
t i o n as it can lead t o the glottic spasm. CONTACT GRANULOMA

Investigations • Due to faulty voice production vocal processes of arytenoid


Lateral soft tissue X-ray of neck shows: rub against each other which leads t o an area of heaped u p
mucosa on one vocal process which fits into ulcer like depres-
• Swollen epitglottis (Thumb sign) 0

sion on the opposite side.


• Absence of deep well-defined vallecula (valleculla sign)
• It is a type of chronic hypertrophic laryngitis.
• It mainly affects posterior third of vocal fold which corresponds
Treatment
t o vocal process of arytenoid cartilage.
• Intubation/tracheostomy regardless o f t h e severity of respira-
t o r y distress is the t o p m o s t priority Etiology
• Hospitalization
It is multifactorial:
• Immediate I.V antibiotics ampicillin/2nd and 3rd generation
• Vocal abuse is the main offending cause
cephalosporins • Seen in men w h o smoke/drink alcohol excessively.
• Ceftriaxone is t h e antibiotic of choice 0

« Steroid Others
• Adequate hydration t o be maintained
Emotional stress
• Humidification/0 inhalation
Gastroesophageal reflux
2

• If household contacts of the patient w i t h H. influenzae epi- Chronic throat clearing and infections postural drip
glotittis include an unvaccinated child under the age o f 4, Allergy
all members o f the household (including the patient) should Idiopathic
receive prophylactic rifampin for 4 days to eradicate carriage
of H. influenzae. -Ref. Harrison 17th/ed, p213 Lesions
• Main complication: Death from respiratory arrest. • Saucer like lesions f o r m e d by heaping of granulation tissue
• Site: Medical edge of the vocal cord at the vocal process
| P S E U D O C R O U P (SUBGLOTTIC LARYNGITIS) Lesion is B/L and symmetrical

Age: Children < 3 years


There is no epithelial defect (as is seen in true ulcers).
Pathology: Mucosal swelling is found on or near the undersurface It doesnot undergo malignant charge
o f t h e vocal cords and in the subglottic region.
Clinical features: Clinical Features
• Starts abruptly
Seen exclusively in males > 30 years.
• No fever/mild fever
• The only symptom is hoarseness of voice.
• Voice is raw resembling barking of seals.
Diagnosis is m a d e b y b i o p s y w h i c h shows a canthosis a n d
• Dry cough
hyperkeratosis.
Treatment: Moist air

Treatment
| CHRONIC LARYNGITIS
• Voice rest for a long period of time and voice therapy if required
• Management of psychological stress and GERD
• Chronic inflammation of mucosa of larynx.
• Microlaryngoscopic excision of granuloma
150^ SECTION IV Larynx

1 ATROPHIC LARYNGITIS/LARYNGITIS SICCA Vocal cords show shallow ulcers w i t h undermined edges
{mouse nibbled appearance)—Characteristic feature
• Characterized by atrophy of laryngeal mucosa and crust for- Pseudoedema o f t h e epiglottis called as Turban epiglottis
mation. Swelling in interarytenoid region giving a mammilated
• Usually occurs as a part of atrophic rhinitis caused by Klebsiella
appearance
ozaenae and atrophic pharyngitis.
ii Diagnosis
Pathologically
• Chest X-ray
• Respiratory e p i t h e l i u m shows squamous metaplasia w i t h
loss o f cilia, mucous producing glands and foul smelling crust • Sputum for AFB
formation Treatment: ATT
• Most common site:
False cords 1 LUPUS O F T H E LARYNX
Posterior region and subglottic region
It is an indolent tubercular infection associated w i t h lupus of nose
Clinical Features and pharynx.
• Mostly seen in females:
Hoarseness o f voice w h i c h improves t e m p o r a r i l y on
Site affected: Anterior part of the larynx (Epiglottis >
coughing and on removing of crust
Aryepiglottic fold > ventricular bands)
There may be dry irritating cough and dyspnea due t o
obstructing crusts.
Patient may complain of b l o o d stained t h i c k m u c o i d
Clinical Features
discharge. — M a q b o o l 11 th/ed, p 335
Crusts are foul smelling and mucosa bleeds when they • It is a painless condition and the patient is asymptomatic.
are removed. • No association w i t h pulmonary tuberculosis.
i
Crusts may also be seen in trachea Prognosis: Good

Treatment
| SYPHILIS OF THE LARYNX
• Treat the underlying cause (poor nutrition, generalized infection
rarely syphillis). • All stages of disease can be manifested.
• Laryngeal sprays w i t h glucose in glycerine or oil of pine helps • Primary stage: Mucosal ulceration: Primary chancre
to loosen the crust. Secondary stage: Multiple vesicles and papular lesions.
• Microlaryngoscopic removal of crust is new modality of treat- Tertiary stage: Gummatous lesion
ment
• Expectorants containing a m m o n i u m chloride or iodide also
help t o loosen t h e crust. 9
Sites affected: Anterior part of the larynx i.e. epiglottis and
\^ aryepiglottic fold. ^ ^
| TUBERCULAR LARYNGITIS

Commonly associated w i t h pulmonary TB | LEPROSY


Rarely: blood-borne infection
• Most commonly affects the anterior part of larynx.
• Epiglottis and aryepiglottic folds are affected first
Sites Affected

• All regions can be affected. | REINKE EDEMA
• Predilection for the posterior part of larynx. (Interarytersid region >
vertricular bands > vocal cord > epiglottis) B/L Symmetrical swelling o f t h e whole o f t h e membranous part
of vocal cord occurring due t o edema o f t h e subepithelial space
(Reinke's space).
Clinical Features ...

Weakness of voice w i t h periods of aphonia is earliest symptom Etiology


Hoarseness, cough, dysphagia, odynophagia.
Referred otalgia Chronic irritation of vocal cords due to:
Laryngeal examination: Voice misuse, Heavy smoking, Chronic sinusitis,
Hyperemia and ulceration of vocal cord w i t h impairment Laryngooesophageal reflex.
of abduction—first sign Myxoedema

•CHAPTER 13 Acute and Chronic Inflammation of Larynx, Voice and Speech Disorders

Clinical Features • Triangular gap near posterior commissure in weakness of


• Seen in middle age (40-60 years). interarytenoid.
• Key hole appearance of glottis when b o t h muscles viz. t h y r o -
• Most common symptom: hoarseness of voice.
arytenoid and interarytenoids are involved.
• Patient uses false vocal cords for voice production therefore
voice is low pitched and rough. Treatment: Voice rest
On examination: There is bilateral symmetrical swelling of the
vocal cords. | HYPONASALITY

Treatment Called as Rhinolalia clausa.


Decortication: A circumscribed strip of epithelium is removed from Lack of nasal resonance.
one side o f vocal cord while preserving the vocal ligament. Other Defect is blockage of nose or nasopharnx due t o c o m m o n
side t o be operated after 3-4 weeks. cold, nasal allergy, polyps nasal growths, adenoids or naso-
• Voice rest and speech therapy. pharyngeal mass.

HYPERNASALITY

VOICE AND SPEECH DISORDERS It is called as rhinolalia aperta.


Words w i t h little nasal resonance are resonated t h r o u g h nose.
DYSPHONIA PLICA VENTRICULARIS (VENTRICULAR Defect: failure of nasopharynx t o cut off f r o m oropharynx or

0 DYSPHONIA) abnormal communication between oral and nasal cavities.

Features Voice production is by false cords (ventricular PUBERPHONIA


folds) rather than true vocal cord.
Cause can be functional (psychogenic) or organic • Presence of high pitched voice of childhood in adult males.
eg in case of impaired function of true cords as • Seen in boys w h o are emotionally immature, feel insecure and
in paralysis, fixation or tumors. show excessive attachment to their mothers.
Quality of voice Rough, low-pitched and unpleasant.
Diagnosis On i n d i r e c t l a r y n g o s c o p y false cords Treatment
a p p r o x i m a t e p a r t i a l l y or c o m p l e t e l y and • Training the boy t o produce low-pitched voice.
obscure the view of true cords on phonation
Treatment Functional cases are dealt w i t h voice therapy
and psychological counseling. The condition
is difficult t o treat if, it is caused bylaryngeal
disorders.
CONDITIONS CAUSING
SPEECH DISORDERS
FUNCTIONAL APHONIA
VOCAL CORD NODULE (SINGER'S/SCREAMERS
• Mostly seen in emotionally labile females (in age group 15-30 NODULES)
years)
• Patient communicates w i t h whisper but coughing is normal. It is localized epithelial hyperplasia and is a bilateral condition.
• Aphonia is sudden and w i t h o u t any accompanying laryngeal
Seen symmetrically on the free edge of vocal cord, at the junc-
symptoms/No vocal cord palsy 0

tion of anterior one third, w i t h the posterior t w o thirds (i.e. area


On Laryngoscopic examination: Vocal cords are seen in abducted of maximum vibration of cord).
position and fail t o adduct on phonation; however, adduction of Seen in singers, actors, teachers and hawkers.
vocal cords is seen on coughing.
Females > males in adults whereas in children it is more c o m -
mon in boys.
Treatment
Most c o m m o n age group = 20-30 years.
Main cause-Misuse or abuse of voice.
Reassurance and psychotherapy.
Patients complain of hoarseness of voice, which worsens by
evening due t o fatigue.
• PHONOSTHENIA
Indirect laryngoscopy shows—pinkish white nodules at the
• Weakness of voice due t o fatigue o f phonatory muscles due t o
j u n c t i o n of anterior one third and posterior t w o thrids.
voice abuse or laryngitis.
• Thyroarytenoid, interarytenoid or b o t h may be affected.
Treatment
Symptoms: Easy fatiguability of voice.
• Voice rest and speech therapy
Signs: Indirect laryngoscopy. • Microlaryngoscopic excision of nodules- -Using microsurgical
• Elliptical space between cords in weakness of thyroarytenoid. instruments or laser.
SECTION IV Larynx

| V O C A L CORD POLYP Mogiphonia: It is a psychoneurotic disorder in which phonic


spasm occurs in professional voice users, w h e n t h e y appear in
• Usually unilateral at the junction of anterior and middle third public. Initially, the voice is normal but soon the vocal cords
of vocal cord. get adducted and person cannot speak.

Etiology

• Voice abuse, chronic irritation like smoking.


• Sudden s h o u t i n g results in h e m o r r h a g e and submucosal False vocal cord
edema. (vestibular fold)

Management

• Microlaryngeal excision. Aryepiglottic


-
fold
| EXTRA EDGE

• Gutzmann's pressure test if positive confirms puberphonia. Fig. 13.1: Diagram t o show vocal nodules
In this test, t h y r o i d prominence is pressed backwards and Bilateral vocal nodules at the junction of anterior
downwards producing low tone voice. one-third and posterior two-thirds of vocal cords
• Ortner's syndrome consists of cardiomegaly and paralysis of Courtesy: Text book of Diseases of Ear, Nose and
recurrent layngeal nerve. Throat, Mohan Bansal. Jaypee Brothers, p 485

;

ri

-
CHAPTER 13 Acute and Chronic Inflammation of Larynx, Voice and Speech Disorders J 153

QUESTIONS

1. Epiglottitis in a 2-year-old child occurs most commonly a. It is a hyperkeratotic lesion present within t h e anterior
due to infection with: [AIIMS May 05] 2/3rd o f t h e vocal cords
a. Influenza virus b. Staphylococcus aureus b. It is not premalignant lesion
c. Haemophilus influenzae d. Respiratory syncytial virus c. Diagnosis is made by biopsy
2. Which ofthe following is the etiological agent most often d. On microscopy it shows acanthosis and hyperkeratosis
associated with Epiglottitis in children? [AIIMS Nov 04] 11. A middle-aged male comes to the outpatient depart-
a. Streptococcus pneumoniae ment (OPD) with the only complaint of hoarseness of
b. Haemophilus influenzae type b voice for the past 2 years. He has been a chronic smoker
c. Neisseria sp. for 30 years. On examination, a reddish area of mucosal
irregularity overlying a portion of both cords w a s seen.
d. Moraxella ca tarrhalis
Management would include all except: [Al 03]
3. T h u m b sign in lateral X-ray of neck seen in: [PGI Dec 04]
a. Cessation of smoking
a. Epiglottitis b. Internal hemorrhage
b. Bilateral cordectomy
c. Saccular cyst d. Ca epiglottis
c. Microlaryngeal surgery for biopsy
e. Vallecular cyst
d. Regular follow-up
4. In acute epiglottis, common cause of death is: [Delhi 96]
12. Steeple sign is seen in: [SGPGI05; UP 05]
a. Acidosis b. Respiratory obstruction
a. Croup b. Acute epiglottitis
c. Atelactasis d. Laryngospasm
c. Laryngomalacia d. Quinsy
5. The antibiotic of choice in acute epiglottitis pending
13. True about laryngitis sicca: [PGIJune 05]
culture sensitivity report is: [01] a. Caused by Klebsiella ozaena
a. Erythromycin b. Rolitetracycline b. Caused by Klebsiella rhinoscleromatosis
c. Doxycycline d. Ampicillin c. Hemorrhagic crust formation seen
6. A 1-year-old infant has biphasic stridor, barking cough d. Antifungal are effective
a n d difficulty in breathing since 3-4 days. He has high- e. Microlaryngoscopic surgery is a modality of treatment
grade fever a n d leukocyte count is increased. Which of 14. Wrong about Laryngitis sicca: [PGIJune 04]
the following would not be a true statement regarding a. Also known as Laryngitis atrophica
the clinical condition of the child? [Al 10] b. Caused by Klebsiella ozaena
a. It is more c o m m o n in boys than in girls c. Caused by Rhinosporodium
b. Subglotic area is the common site of involvement d. Common in women
c. Antibiotics are mainstay of treatment 15. Reflux laryngitis produces: [PGI Dec 04]
d. Narrowing of subglottic space with ballooning of hypo- a. Subglottic stenosis i b. Ca larynx
pharynx is seen c. Cord fixation d. Acute supraglottitis
7. Pachydermia laryngitis - M/C site of involvement e. Laryngitis
a. Arytenoids cartilage 16. Tubercular laryngitis affects primarily: [TN01]
b. Posterior 1/3 and anterior 1/3 commissure a. Anterior commissure
c. Anterior 1/3 commissure b. Posterior commissure of larynx
d. Vestibular fold c. Anywhere within the larynx
8. The cause for contact ulcer in vocal cords is: d. Superior surface of larynx
17. True about TB larynx: [PGI 02]
[Kerala 94,95]
a. 'Turban'epiglottis b. Odynophagia
a. Voice abuse b. Smoking
c. Cricoarytenoid fixation d. Ulceration of arytenoids
c. TB d. Malignancy
e. Paralysis of vocal cord
9. Which of the following statements is not true for contact
18. Mouse-nibbled apperance of vocal cord is seen in:
ulcer? [AIIMS 03]
[CUPGEE01]
a. The commonest site is the junction of anterior 1 /3rd and
a. TB b. Syphillis
middle 1/3rd of vocal cord and gastroesophageal reflux is
c. Cancer d. Papilloma
the causative factor
19 Infection involving anterior larynx: [MP 01]
b. Can be caused by intubation injury
a. TB b. Sarcoidosis
c. The vocal process is the site and is caused/aggravated by c. Syphilis d. All the above
acid reflux 20. Reinke's edema is seen in: [JIPMER 98; Karn 01 ]
d. Can be caused by adductor dysphonia a. Vestibular folds
10. In a patient hoarseness of voice was found to be having b. Edges of vocal cords
p a c h y d e r m i a laryngitis. All of the following are ture c. Between true and false vocal cords
except: [AIIMS 02] d. In pyriform fossa
154[ SECTION IV Larynx

21. Reinke's layer seen in: [CMC] 29. Androphonia can be corrected by doing: [Aim]
a. Vocal cord b. Tympanic membrane a. Type 1 thyroplasty b. Type 2 thyroplasty
c. Cochlea d. Reissner's membrane c. Type 3 thyroplasty d. Type 4 thyroplasty
2 2 . Pharyngeal Pseudosulcus is seen secondary to: [Al 09] 30. Key nob appearance is seen in: [MP 08]
[AIIMS Nov 2012] a. Functional aphonia
a. Vocal abuse b. Laryngopharyngeal reflux b. Puberphonia
c. Tuberculosis d. Corticosteroid usage c. Phonasthenia
23 In dysphonia plica ventricularis, sound is produced by d. Vocal cord paralysis
[AIIMS 99]
31. Most common location of vocal nodule: [UP 04; PGI 00]
a. False vocal cords b. True vocal cords a. Anterior 1/3 and posterior 2/3 junction
c. Ventricle of larynx d. Tongue b. Anterior commissure
24. Features of functional aphonia: [PGI June 06]
c. Posterior 1/3 and anterior 2/3 junction
a. Incidence in males
d. Posterior commissure
b. Due to vocal cord paralysis
32. True about vocal nodule is/are: [PGI 00]
c. Can cough
a. Also known as screamer's node
d. On laryngoscopy vocal cord is abducted
b. Occur at junction of ant. 1 /3rd and post. 2/3rd of vocal cords
e. Speech therapy is the treatment of choice
c. Most common presentation is aphonia
2 5 . Habitual dysphonia is characterized by: [PGI Dec 04]
d. Microlaryngoscopic surgery is not useful
a. Poor voice in normal environment
33. A c c o r d i n g to E u r o p e a n L a r y n g e a l Society, subliga-
b. Related t o stressful events
mentous cordectomy is classfied as: [AIIMS May 11 ]
c. Treatment is vocal exercise and reassurance
a. Type I b. Type II
d. Whispering voice
e. Quality of voice is constant c. Type III d. Type IV
26. Rhinolalia clausa is associated with all of the following 34. Change in pitch of sound is produced by which muscle:
except: [AI07] [Jharkhand 04]
a. Allergic rhinitis b. Palatal paralysis a. Post cricoarytenoids b. Lateral cricoarytenoids
c. Adenoids d. Nasal polyps c. Cricothyroid d. Vocalis
27. in a patient with hypertrophied a d e n o i d s , the voice 35. Following is not true about spasmodic dysphonia/TA/12]
abnormality that is seen is: [JIPMER 00; Karn. 01] a. Patient with the abductor type have strained and strangled
a. Rhinolalia clausa b. Rhinolalia aperta voice
c. Hot potato voice d. Staccato voice b. Botulinum toxin is the standard treatment for it
28. Young man whose voice has not broken is called: c. Multiple sittings of botulinum toxin A is required for its
a. Puberphonia b. Androphonia treatment
c. Plica ventricularis d. Functional aphonia d. It affects the muscles o f t h e larynx
• •

EXPLANATIONS AND REFERENCES


1. Ans. is c i.e. Haemophilus influenzae
2. Ans is b i.e. Haemophilus influenzae type B
Ref. Dhingra 5th/ed, p 307; Ghai 6th/ed, p 340; Harrison 17th/ed, pp 212,213, Scott-Brown's 7th/ed Vol 2, p 2250; Mohan Bansal p479
• Most common organism causing epiglottitis in children is H. influenzae type B.
• Though the introduction of Hib vaccine has reduced the annual incidence acute epiglottitis but still most o f t h e pediatric cases
seen today are due to haemophilus influenzae B. —Harrison 17th/edp212
• In adults it can be caused by group A streptococcus, S. pneumoniae, S. aureus and Klebsiella pneumoniae
Ans. is a i.e. Epiglottitis Ref. Dhingra 5th/edp 308; Scott-Brown's 7th/ed Vol 2 p 2250; Mohan Bansal p 479

ALSO KNOW
Steeple sign i.e. narrowing of subglottic region is seen in chest X-ray of patients of laryngotracheobronchitis (i.e. croup).
A plain lateral soft tissue radiograph of neck shows the following specific features:
• Thickening o f t h e e p i g l o t t i s — t h e thumb sign
• Absence of a deep well-defined vallecula—the vallecula sign
Ans. is b i.e. Respiratory obstruction Ref. Scott's Brown 7th/ed vol-2 pg 2251; Logan Turner 10th/edp 390; Mohan Bansalp 480
Acute Epiglotlitis
"The main complication is death from respiratory arrest due to acute airway obstruction" -Scott's Brown 7th/ed pg 225
• Respiratory arrest is more likely in patients w i t h rapidly progressive disease and occurs w i t h i n hours of onset of t h e illness
• Other complications are rare but include epiglottic abscess, pulmonary edema secondary t o relieving airway obstruction and
thrombosis of internal jugular vein (Lemierre's syndrome)
CHAPTER 13 Acute and Chronic Inflammation of Larynx, Voice and Speech Disorders J 155
5. Ans. is d i.e. ampicillin Ref. Turner Wth/edp 390

Well friends, there is some controversy over this one.


• Let's, first see what Dhingra 5/e, p 308, has t o say:
• Ampicillin or third generation cephalosporin are effective against H. influenzae and are given by parenteral route."
However, books like Turner and Harrison do not agree with Dhingra about ampicillin being the drug of choice.
Harrison 17/e, p 212 says:
"Once the airway has been secured and specimens of blood and epiglottis tissue have been obtained for cultrue, treatment with IV
antibiotics should be given to cover the most likely organism particularly H. influenzae. Because rates of ampicillin resistance in this organism
have risen significantly in recent years, therapy with a beta lactam / beta lactamase inhibitor combination or a second or third generation
cephalosporin is recommended. Typically, ampicillin / sulbactam, cefuroxime, cefotaxime or ceftriaxone is given, with clindamycin and
trimethoprim-sulfamethoxazole reserved for patients allergic to beta lactams."
So, according t o Harrison DOC are:
• Ampicillin + Sulbactam (Not ampicillin alone)
• Cefuroxime
• Cefotaxime
• Ceftriaxone

According t o Scott's Brown 7/ed vol-2 pg-2251


"The antibiotics of choice are second and third generation cephalosporin. Ampicillin was often prescribed but resistant H.influenza
are n o w emerging".
Now, lets read what Turner W/e, p 390 has t o say:
"Treatment is to put the child in an atmosphere of moist oxygen. Sedation must be given cautiously, if at all, in case the respiratory
centre is depressed. Chloramphenicolis the antibioticofchoice and it should be given intramuscularly or preferably intravenously. Amoxycillin
or ampicillin is no longer advised as haemophilus organism are now sufficiently often resistant to make its use inappropirate."
Neither 2nd/3rd generation cephalosporins nor chloramphenicol is give in the o p t i o n . Hence we will have t o opt for amplicillin
as no other opiton is correct.

| Remember: DOC for epiglottitis -2nd/3rd generation cephalosporin. Treatment w i t h amplicillin is not that effective due t o b lactamase
production by Hib. Prophylaxis w i t h Rifampicin for 4 days is advocated in unimmunized household contacts < 4 years of age and in all
5
immunocompromised contact.

6. Ans. is c i.e. Antibiotics are mainstay of treatment Ref. Dhingra Sth/ed p 308; Mohan Bansalp 478
CROUP (laryngotracheitis and laryngotracheobronchitis)
• Croup is a c o m m o n respiratory illness in childhood.
• It is an inflammatory condition involving the larynx, trachea and bronchi.
• Most c o m m o n site involved is subglottis.
• P<jtf?o/ogy-There is some degree of laryngeal inflammation, loose areaolartissue of subglottis swells up; resulting in hoarseness,
a barking cough and varying degrees of respiratory distress over time.
• Etiology - Mostly it is viral in origin. Most c o m m o n viruses involved are parainfluenzae 1 and 2. Others are influenza A and B,
respiratory syncytial virus, adenovirus and measles. Bacterial super-infection can occur in cases of laryngotracheobronchitis
and laryngotracheobronchopneumonitis.
• Age - most c o m m o n l y seen between the ages of 1 and 6 years w i t h a peak incidence being around 18 months of age and the
majority o f cases below 3 years of age.
It is more c o m m o n in boys than girls.
Laryngotracheitis generally starts w i t h several days of rhinorrhea, pharyngitis, low-grade fever and a mild cough. Over the
next 12 t o 48 hours, a progressively worsening "barky"cough, hoarseness and inspiratory stridor are noted, secondary t o some
degree o f upper airway obstruction and laryngeal inflammation. The onset is often rapid and typically in the early morning
hours (e.g. 2:00 am).
On examination, the child will be noted t o have coryza, a hoarse voice, and varying degrees of pharyngeal inflammation,
tachypnea, and stridor. More severe cases may involve nasal flaring, moderate tachypnea, retractions and cyanosis. Some
children w i t h croup may not be able t o maintain adequate oral intake of fluids. Alveolar gas exchange is usually normal, w i t h
hypoxia seen only in severe cases.
The diagnosis is usually made on clinical grounds. Laboratory studies add little t o the diagnosis of croup if bacterial infection
is not suspected. White blood cell counts may be elevated above 10,000 w i t h a predominance of polymorphonuclear cells.
Chest radiographs may show subglottic narrowing (in 5 0 % of children w i t h croup) called as "Steeple sign".
SECTION IV Larynx

• The most important diagnostic consideration is distinguishing acute epiglottitis from acute laryngotracheitis. Epiglottitis describes
a bacterial infection o f t h e epiglottis. It is most commonly caused by H. influenzae type B. In epiglottitis fever is o f very high
grade, patient has a toxic look, there is marked stridor and odynophagia On chest X-ray t h u m b sign is seen.

Management

• Once the diagnosis of croup is made, mist therapy, corticosteroids and epinephrine are the usual treatments. Since croup is chiefly
viral in etiology, antibiotics play no role. Mist therapy (warm or cool) is t h o u g h t t o reduce the severity of croup by moistening
the mucosa and reducing the viscosity of exudates, making coughing more productive. For patients w i t h mild symptoms, mist
therapy may be all that is required and can be provided at home.
• For more severe cases, further intervention may be required like oxygen inhalation by mask, racemic epinephrine given by
nebulizer, corticosteroids and intubation or tracheostomy.
7. Ans. is a i.e. arytenoid cartilage Ref. Scott's Brown 7th/ed vol-2 pg2196
Pachyderma laryngitis affects the medial surface of arytenoid cartilage, in particular the vocal processes.

ALSO KNOW

Condition
Tuberculosis Posterior half of larynx
Syphilis Anterior commissure and anterior 1/3 of vocal cord
Leprosy Anterior part of larynx including epiglottis and aryepiglottic fold
Vocal nodule Junction of anterior 1/3 and posterior 2/3 of vocal cord
Glottic cancer Free edge and upper surface of anterior 1/3 of true vocal cord.

8. Ans. is a i.e. Voice abuse Ref. Maqbool 11 th/edp 334;Mohan Bansalp 486
Aetiology of contact ulcers is mutli factorial but the most important cause is:
• Voice abuse (faulty production of voice rather than excess use). - Maqbool
• Smoking as a cause for contact ulcer is given only in Dhingra and is not supported by Scoffs Brown or Maqbool.
9. Ans. is a i.e. The commonest site is the junction of anterior 1/3rd and middle 1/3rd of vocal cord and gastroesophageal
reflux is the causative factor Ref. Scotts Brown 7th/ed Vol-2 pg 2196-2197
10. Ans. is a i.e. It is a hyperkeratotic lesion present within the anterior 2/3rd of the vocal cords
Ref. Dhingra 5th/ed p31l; Maqbool 17 th/ed pg 334; Scotts Brown 7th/ed vol-2 pg2197
The mostcommon site for contact ulcers is vocal processes o f t h e arytenoid cartilage.
Contact ulcers: / Vocal process granuloma / arytenoids granuloma / intubation granuloma.
• Nearly exclusively seen in men over the age of 30 years. •

• Commonly located over the posterior part o f vocal processes of arytenoid cartilage.
• Can be unilateral or bilateral -

• It is multifactorial in aetiology:
- Vocal abuse (most important Etiological factor) t a l k i n g in a h a b i t u a l l y low p i t c h e d creaky, h y p e r f u n c t i o n a l manner
(.-. o p t i o n d is correct)
- Prolonged intubation
- Esophageal dysfunction (such as gastroesophageal reflux, hiatus hernia, dysmotility).
• Symptoms
- Low pitch quality of voice ( m o s t prominent feature).
- Irritation and pain in larynx which worsens on phonation or coughing and it can radiate t o ear.
• Management
- Voice therapy along w i t h anti reflux medications.
- In persistant cases microlaryngeal excision may be required t o confirm the diagnosis and exclude malignancy.
11. Ans. is b i.e. Bilateral cordectomy Ref. Dhingra 6th/ed p 292-293,309
Middle aged man + Chronic smoking + Hoarseness of voice + Bilateral reddish area of mucosal irregularity on cords
All these indicate that either it is pachydermia laryngitis or it can be early carcinoma:
• Both the conditions can be distinguished by biopsy only so option "c" is correct.
• In either conditions: smoking is a causative factor and should be stopped.
• Regular follow up is a must in either o f t h e conditions.
• Bilateral cordectomy is not required even if it is glottic cancer because early stages of glottic cancer are treated by radiotherapy.
• Management of pachydermia is microsurgical excision of hyperplastic epithelium (cordectomy has no role).
CHAPTER 13 Acute and Chronic Inflammation of Larynx, Voice and Speech Disorders J 157

12. Ans. is a i.e. Croup Ref. Ghai Pediatric 6th/ed p 339; Current Otolaryngology 2nd/ed p 472
Chest X-ray in croup (Laryngotracheobronchitis) reveals a characteristic narrowing o f t h e subglottic region called steeple sign.
13. Ans. is a, c a n d e i.e. Caused by Klebsiella ozaena; Hemorrhagic crust formation seen; and Microlaryngoscopic surgery
Ref. Dhingra 5th/ed p312; Scott Brown 6th/ed Vol. I, p 512,513; Mohan Bansal p 481
14. Ans. is c i.e. Caused by Rhinosporidium. Ref. Dhingra 6th/edp 293
For details see text
15. Ans. is a, b a n d e i.e. Subglottic stenosis; Ca Larynx; Laryngitis
• There are lots of controversies regarding the reflux laryngitis secondary t o reflux gastrointestinal disease. But now some studies
d o c u m e n t that there is a clear relation between the t w o .
• Reflux laryngitis may have the following sequlae:
- Bronchospasm
- Chemical pneumonitis
- Refractory subglottic stenosis
- Refractory contact ulcer
- Peptic laryngeal granuloma
- Acid laryngitis (Heart burn, burning pharyngeal discomfort, nocturnal chocking due to interarytenoid pachydermia)
- Laryngeal Carcinoma (According to recent reports laryngeal reflux is the cause of laryngeal carcinoma in patients who are life
time non-smokers).

L a r y n g o p h a r y n g e a l Reflux

Here classical GERD symptoms are absent. Patients have more o f daytime/upright reflux w i t h o u t the nocturnal/supine reflux o f
GERD. In laryngopharyngeal reflux esophageal motility and lower esophageal sphincter is normal, while upper esophageal sphincter
is abnormal. The traditional diagnostic tests for GERD are not useful in LPR.
Symptom Chronic or Intermittent dysphonia, vocal strain, foreign body sensation, excessive throat mucus, Postnasal discharge and
cough. Laryngeal findings: Interarytenoid bunching, Posterior laryngitis and subglottic edema (Pseudosulcus)

S e q u e l a e o f L a r y n g o p h a r y n g e a l Reflux

• Subglottic stenosis
• Carcinoma larynx
• Contact ulcer/granuloma
• Cricoarytenoid j o i n t fixity
• Vocal nodule/polyp
. Sudden infant deaths
• Laryngomalacia (Association)
Treatment is in similar lines as GERD, but we need t o give proton p u m p inhibitors at a higher dose and for a longer duration (at
least 6-8 months).
16. Ans. is b i.e. Posterior commissure of larynx Ref. Dhingra 5th/edp 312,6th/ed p 293
Tuberculosis affects posterior part of larynx more than anterior part.
Parts affected are: Inter arytenoid fold > Ventricular bands > Vocal cords > Epiglottis
17. Ans. is a, b a n d d i.e. Turban epiglottis; Odynophagia; a n d Ulceration of arytenoids
18. Ans. is a i.e. T B Ref. Dhingra 5th/ed pg312,6th/ed p 293; Mohan Bansal p 481
• Tuberculosis of larynx is always secondary t o pulmonary TB.
• Tubercle bacilli reach the larynx by bronchogenic or haematogenous routes.
• Mostly affects males in middle age group.

• Affects posterior part o f (Posterior Commissure) larynx more than anterior part.

Clinical Features
• Weakness of voice (earliest symptom), odynophagia, dysphagia.
• Pain radiates t o the ears.
• Laryngeal examination shows:
- Vocal cord: Mouse nibbled ulceration
- Arytenoids: show ulceration.
- Interarytenoid region is swollen giving a mammillated appearance"
- Epiglottis shows: Pseudoedema and is called as 'turban epiglottis'.
- Surrounding mucosa is pale.
158[ SECTION IV Larynx

I n°te friO!i^wij^^
Earliest sign = Adduction weakness

Remember: Knob like epiglottis and Button hole Epiglottis is seen in leprosy

19. Ans. d i.e. all of the above Ref. Scott's Brown 7th/edpg 2267

Syphilis Larynx is rarely involved. If Larynx is involved it presents as diffuse erythematous papules (secondary
stage) and nodular infiltrates coalescing into painless ulcers (tertiary stage) with epiglottis and
aryepiglottic folds being principally involved (i.e. anterior part involved).

Sarcoidosis It is a slowly progressive disease with laryngeal involvement in less than 5% cases.
Laryngeal appearance is similar to that of T:B with suprglottic structures being involved
primarily i.e. anterior part involved.

T:B As discussed earlier it involves posterior part more than anterior part.
The parts being involved in the order:

(i) Interarytenoid fold


(ii) Ventricular bands Hence - though less involved but anterior
(iii) Vocal cords part of larynx may be involved.
(iv) Epiglottis ^t

20. Ans. is b i.e. Edges of vocal cords Ref. Dhirgra 5th/edp 311,6th/ed p292; Mohan Bansal Ist/ed p 486
21. Ans. is a i.e. Vocal cord

Reinke's E d e m a


It is diffuse edema o f t h e Reinke's space (of vocal cords) leading t o irreversible fusiform swelling o f t h e vocal cord—usually bilateral.

Commonest etiology is smoking t h o u g h extra esophageal reflux, vocal strain and hypothyroidism has also been implicated.

Patient has a low-pitched hoarse voice; may present as stridor in severe cases.

Treatment is superior cordotomy (incising the superior surface of vocal cord preserving the medical vibrating edge) t h r o u g h
microlaryngoscopy t o decompress the edema fluid. The mucosal flap is then replaced after t r i m m i n g off the excess epithelium.
22. Ans. is b i.e. laryngopharyngeal reflex Ref. Ballenger's Otolaryngology 17th/ed p 886; Scott Brown's 7th/ed p 2238)

Vocal Sulcus/Laryngeal Sulcus

It is a groove along the mucosa and can be classified into three types:

Laryngeal sulcus

Laryngeal Pseudosuicus (Pseudosulcus Laryngeal True Sulcus (Suleusvergeture) Sulcus vocalis


Vocalis)
Pseudosulcus arises due to swelling of the True sulcus is related to scarring of the vocal Seen in deeper layers of ligament
subglottic area secondary to laryngotracheal fold in the phonatory strking zone
reflux. It refers t o i n f r a g l o t t i c edema
e x t e n d i n g f r o m arterior commissure t o
posterior larynx
The pseudosulcus is located between the true This is located within the true vocal folds at the
vocal folds and the subglottic swelling site ofthe adherence of vocal fold epithelium
to the vocal ligament

• It is believed that vocal sulcus / laryngeal sulcus are more common in Indian subcontinent.
• They frequently present with persistent dysphonia following puberty.

Management

Phonosurgical treatment, i.e. either excising the sulcus, injecting collagen or fat t o boost the underlying layer or giving a parallel
incision in the mucosa running in cephalad to cordal direction t o break up the linear scar and vocal fold.
CHAPTER 13 Acute and Chronic Inflammation of Larynx, Voice and Speech Disorders j 159
23. Ans. is a i.e False vocal cord Ref. Dhingra 5th/ed pg 334,6th/edp313; Mohan Bansalp 497
In dysphonia plica ventricularis voice is produced by false vocal cords (ventricular folds).
24. Ans. is c and d i.e. Can cough; and on laryngoscopy vocal cord is abducted
Ref. Dhingra 5th/ed p 334,6th/edp 314; Mohan Bansal p 497
• Functional aphonia or hysterical aphonia is a functional disorder mostly seen in emotionally labile females in th age group of 15-30 years.
• Laryngoscopy Examination shows vocal cord in abducted position and fails t o adduct on phonation, however adduction is seen
o n coughing, indicating normal adductor function.
• Treatment: - Reassurance o f t h e patient of normal laryngeal function and psychotherapy.
- Speech therapy has no role in it.
25 Ans. is a, c, d a n d e i.e. Poor voice in normal environment; Treatment is vocal exercise and re-assurance; Whispering voice;
and Quality of voice is constant
• When a person always uses a poor voice in normal circumstances, is called habitual dysphonia. It is not related t o stressful events
and seems t o be a habit.
• The distinguishing characteristics o f habitual and psychogenic functional dysphonia are:

Habitual dysphonia Psychogenic functional dysphonia


Quality of voice is always poor Previous good voice quality

Very gradual onset of voice problem Abrupt change in voice quality.


Quality of voice is nearly constant changing with circumstances Inconstant quality of voice

The voice fails repeatedly after prolonged speaking Voice fails repeatedly in situationsof emotional stress.

Some patients w i t h habitual dysphonia need vocal excercises and very little counseling. Others are cured by a few counseling ses-
sions and no voice practice at all.
26. Ans. is b i.e. Palatal paralysis Ref. Dhingra 5th/edp 334-335,6th/edp315; Mohan Bansalp 497
27. Ans. is a i.e. Rhinolalia clausa
• Rhinolalia clausa is lack of nasal resonance (hyponasality).
• It is seen in conditions which block the nose or nasopharynx. So will be see in case of allergic rhinitis, adenoids and nasal polpys.
• Palatal paralysis will lead t o hypernasality and not hyponasality.
28. Ans. is b i.e. Puberphonia Ref. Dhingra 5th/edp 334,6th/edp315, Mohan Bansalp 497
• In males at t h e t i m e o f puberty, the voice normally drops by an octave and becomes low pitch.
• It occurs because vocal cords lengthen
• Failure of this change leads t o persistence of childhood high pitched voice and is called as puberphonia
• It is seen in boys w h o are emotionally insecure and show excessive attachment t o their mothers. Their physical and sexual
development is normal

Treatment

Training the body t o produce low pitched voice.

• G u t z m a n n pressure test: In this test thyroid prominence is pressed backward and downward producing low tone voice.
• If this test is positive it indicates puberphonia.

29. Ans. is d i.e. Type IV thyroplasty Ref. Dhingra 5th/edp 321

Thyroplasty

Type Procedure Indication


Type 1 Medialisation of vocal cord Unilateral vocal cord paralysis, vocal cord atrophy and sulcus vocalis
Type 2 Lateralisation of vocal cord Spasmodic dysphonia
Type 3 Shortening (relaxation) or cord For lowering vocal pitch as in puberphonia
Type 4 Lengthening (Stretching) of cord For elevating the pitch as in androphonia

30. Ans. is c i.e. Phoneasthenia Ref. Dhingra 5th/edp 334,6th/edp 314


Phonoasthemia is weakness of voice due t o fatigue of phonatory muscles i.e. either thyroarytenoids or intrarytenoids or b o t h
O/E - on Indirect laryngoscopy - 3 features may be seen
160^ SECTION IV Larynx

• I
Elliptical space between the cords in Triangular gap near posterior commissure Keyhole appearance of glottis when both
case of weakness of thyroarytenoid in weakness of interarytenoid thyroarytenoids are involved.

31. Ans. is a i.e. Anterior 113 and posterior 2/3 junction Ref. Dhingra Sth/ed p 322,6th/ed p 303; Mohan Bansal p 485
32. Ans. is a and b i.e. Also known as screamer's node; and Occur at junction of ant. 1 st/3rd and post. 2nd/3rd of vocal cords
Ref. Dhingra 5th/edp 322,6th/ed p 303; Current Otolaryngology 2nd/edp 432; Mohan Bansalp 485
• Vocal nodules are also called singers or screamers nodes.
• They are also the most c o m m o n cause of persistent dysphonia in children
• Mostcommon site - at the j u n c t i o n of anterior 1/3 and posterior 2/3 of vocal cords.
• Most common cause - voice abuse.
• Most common presentation - Hoarseness of voice.
• O/E -They appear as bilateral w h i t e asymmetric nodules (< 3 mm) on the vocal cord
Management: First line of therapy is speech therapy
Microlaryngoscopic surgery should be reserved for cases which do not respond t o voice therapy or if diagnosis is not clear.
33. Ans. i s ' b ' i . e . T y p e III
• The European Laryngological Society is proposing a classification of different layngeal endoscopic cordectomies in order t o
ensure better definitions of post-operative results.
• The w o r d "cordectomy" is used even for partial resections because is the term most often used in the surgical literature.
• The classification comprises eight types o f cordectomies.
- Tyepe I: A subepithelial cordectomy, which is resection o f t h e epithelium
- Type Iii Asubligamental cordectomy, which is a resection of the epithelium, Reinke's space and vocal ligament.
- Type III: Transmuscular cordectomy, which proceeds t h r o u g h the vocalis muscle
- Type IV: Total cordectomy;
- Type Va: Extended cordectomy, which encompasses the contralateral vocal fold and the anterior commissure
- TypeVb: Extended cordectomy, which includes the arythnoid
- Type Vc: Extended cordectomy, which encompasses the subglottis
- TypeVd: Extended cordectomy, which includes the ventricle.
34. Ans. is c i.e. Cricothyroid Ref. PL Dhingra 3rd p 337
Sorry for this one

- • - • .

-
CHAPTER

14 Vocal Cord Paralysis

| NERVE SUPPLY OF LARYNX

The main cranial nerve innervating the larynx is the vagus nerve On the right side recurrent laryngeal N originates from vagus
via its branches; superior laryngeal nerve (SLN) and recurrent and on left side it has a longer course since'it originates in
laryngeal nerve (RLN). mediastimum at the level of arch of aorta and it is more vulnerable
to injury.
• Superior laryngeal nerve: arises f r o m the inferior ganglion
of vagus and receives a branch from superior cervical sympa-
thetic ganglion. It enters the larynx by piercing the thyrohyoid
membrane.
Muscle Actions
• It divides at the level of greater corner of hyoid into: > Inorder to have a better understanding of the effects of nerve
(i) Internal laryngeal nerve: palsies: a summary ofthe nerve supply and actions of intrinsic
- Sensory (It supplies the larynx above the vocal cords) muscles is given. In the table:
- Secretomotor
Muscle Supplied by Action
(ii) External laryngeal nerve-supplies cricothyroid muscle
- The superior laryngeal nerve ends by piercing t h e Cricothyroid SLN Tensor, Adductor
inferior c o n s t r i c t o r of pharynx and unites w i t h Posterior cricothyroid RLN Abductor
ascending branch o f recurrent laryngeal nerve. This
Lateral cricoarytenoid RLN Adductor
branch is k/a galen's anastomosis & is purely sensory.
• Recurrent laryngeal nerve: Interarytenoids RLN Adductor
Vocalis RLN Adductor
Motor branch Sensory branch V J
Supplies all the intrinsic muscles Supplies below the level of the
ofthe larynx expect cricothyroid vocal folds

P o s i t i o n of t h e V o c a l c o r d in H e a l t h a n d D i s e a s e

Position o f t h e cord Location of the cord f r o m Situation in


midline
Health Disease

Median Midline Phonation RLN paralysis

Paramedian 1.5 mm Strong whisper RLN paralysis

Intermediate (cadaveric) 3.5 mm. This is neutral position of Paralysis of both recurrent and
cricoarytenoid joint. Abduction superior laryngeal nerves
and adduction take place from
this position

Gentle abductin 7 mm Quiet respiration Paralysis of adductors

Full abduction 9.5 mm Deep inspiration

-
162[ SECTION IV Larynx

| SUPERIOR LARYNGEAL NERVE PALSY

Vocal cord Unilateral Paralysis

Vestibular fold Muscle affected Cricothyroid-Adductor, Tensor


Features • Voice not severely affected and recovers fast.
• Pitch of the voice cannot be raised
Aryepiglottic fold
• Ipsilateral cord:
- Bowed and floppy
- Increased length
- Cords sag down during inspiration and bulge
FA SA C PM M up during expiration

Fig. 14.1: Vocal cord positions • U/L Anaesthesia of larynx above the level of
Abbreviations: M, Median; PM, Paramedian; C, Cadaveric vocal cord
(Inter-mediate); SA, Slight abduction; FA, Full abduction
Treatment: No treatment
Courtesy: Text book of Diseases of Ear, Nose and
Throat, Mohan Bansal. Jaypee Brothers, p 491
Bilateral Paralysis
High vagal nerve palsy: Vagus nerve invovlement in the skull from
• Featuers - voice is breathy and weak.
parapharyngeal space - till jugular foramen.
• High chances of aspiration as there is bilateral anaesthesia of
• Features: Left side is more c o m m o n l y involved
• B/L paralysis occurs in 6 % cases. supraglottic part.
• Male: Female = 8:1 •

Treatment
C a u s e s of Vocal C o r d Palsy • Tracheostomy may be required.
Idiopathic • Epiglottopexy t o close the laryngeal inlet, t o protect the lungs
Malignancy: - Bronchial (50%) Oesophageal (20%) from repeated aspiration, may be done.
- Thyroid (10%) Nasopharyngeal care
| RECURRENT LARYNGEAL NERVE PALSY
noma/ 2 0 %
- Glomus tumor, lymphoma.
U/Labductor Paralysis
Surgical t r a u m a (Oesophageal, l u n g , t h y r o i d , radical neck
dissection). Recurrent laryngeal nerve palsy leads t o ipsilateral paralysis of all
Non-surgical trauma (Road traffic accident, Otner's syndrome). intrinsic laryngeal muscles except cricothyroid.
Viral factors: Infectious mononucleosis, Influenza. • Affected cord: Paramedian position (vocal cord does not move
Bacterial causes: T.B., syphilis. laterally on deep inspiration)
Features:
Miscellaneous causes: Hemolytic anemia, collagen disorder,
Slight hoarseness, which improves over the days.
Diabetes, alcoholism. Gullain Barre syndrome
Voice tires w i t h use.
Treament: Speech therapy

LAWS RELATED TO NERVE PALSIES


> Semons law: States that in a gradually advancing organic lesion
of recurrent laryngeal nerve or its fibres in the peripheral trunk, Causes of Left Recurrent laryngeal Nerve palsy:
3 stages can be observed. • Pancoast tumor of lung
1 stage . Only abductor paralyzed
st
• Mitral stenoses-due to enlarged left atrium (k/a Ortners syndrome)
• Vocal cord in the midline • Aneurysm of arch of aorta
• Adduction still possible • Apical TB ki-.fj:
2 stage . Additional contracture of the abductors. Cord
n d

immobilized in the median position. B/L Recurrent l a r y n g e a l n e r v e palsy-(B/L Abductor


3 stage • Adductors paralysed. Cords are present in the
rd

paralysis)
cadaveric position (Intermediate position)
• Wagner and Grossman theory: States that in the abscence M/C cause = T h y r o i d surgery and neuritis
of cricoarytenoid joint fixation, an immobile vocal fold lying
in the paramedian position has a total Unilateral recurrent Features
laryngeal nerve palsy, while an immobile vocal fold in the
• Both cords lie either in the median or in the paramedian posi-
lateral (cadaveric) position has combined paralysis of superior
tion due t o unopposed action of critothyroid muscle.
and recurrent laryngeal nerves.
• Voice is good
CHAPTER 14 Vocal Cord Paralysis J 163
• Dysponea/stridor: May be present as airway is inadequate. - Chances of aspiration are present.
• Stridor becomes worse on exertion or d u r i n g an attack o f Treatment - Cord medialization.
acute laryngitis. Surgery for medialization of the cord: (Type I thyroplasty)
Intracordal injection: Teflon and collagen
Treatment
Arytenoid rotation
• Emergency tracheostomy as an emergency procedure

Nerve- muscle pedicle reinnervation.
• In long term cases choice is between a permanent trachesotomy Recurrent laryngeal nerve reinnervation
w i t h a speaking valve or a surgical procedure t o lateralize the Muscle / cartilage implant
cord.The former relieves stridor, preserves good voice but has
the disadvantage of a tracheostomy hole in the neck.The latter B/L A d d u c t o r P a r a l y s i s (M/C C a u s e = F u n c t i o n a l -> F l a g
relieves airway obstruction but at the expense of a g o o d voice, s i g n is s e e n )
however, there is no tracheostomy hole in the neck.
• Widening the respiratory a i r w a y without a p e r m a n e n t •
Position of the cord: B/ L Cadeveric
tracheostomy (endoscopic or through external cervical •Features:
approach). Aim is t o widen the respiratory airway t h r o u g h Aphonia - Aspiration
larynx. Inability of cough - Bronchopneumonia
This can be achieved by (i) arytenoidectomy w i t h suture, w o o d - There is also total anaesthesia o f t h e larynx.
man procedure, Dowine procedure, (ii) artenoidopexy (fixing
Treatment
the arytenoid in lateral position), (iii) lateralization of vocal cord
and (iv) laser cordectomy (removal of one cord). • Where recovery expected:
• These operations have now been replaced by less invasive Tracheostomy w i t h cuff
techniques such as: Epiglottopexy
(i) Transverse cordotomy (kashima operation). Vocal cord plication
(ii) Partial arytenoidectomy. • If neurological lesion is progressive and irreversable total lar-
(iii) Reinnervation procedures.-Aim t o innervate paralyzed yngectomy t o prevent aspiration and lung infection
posterior cricoarytenoid muscle by implanting a nerve-
muscle pedicle of sternohyoid or o m o h y o i d muscle w i t h
Isshiki's thyroplasty: It is an innovative procedure developed to
its nerve supply f r o m ansa hypoglossi. These procedures
improve the laryngeal mechanics:
have not been very successful.
Types:
(iv) Thyroplasty t y p e II
> Type 1: Medialization of the cord
> Type 2: Lateralization of the cord
COMBINED SUPERIOR AND RECURRENT LARYNGEAL » Type 3: Shortening the cord (lowers the vocal pitch)
NERVE PALSY > Type 4: Lengthening of the cord (to increase the pitch) to
correct androphonia. The male character low pitch voice is
U/L a d d u c t o r p a r a l y s i s converted to female pitch voice.
Note
(Both superior and recurrent laryngeal nerve gone).
Carcinoma bronchus is the most common cause of left RLN
There occurs unilateral paralysis of all laryngeal muscles except
palsy, while thyroid surgery affects right RLN (as RLN is close
the inter arytenoid w h i c h receives innervation f r o m both the sides.
to inferior thyroid artery, so increased chances of injury during
• Position of the cord: U/L Cadaveric position (3.5 m m f r o m
thyroidectomy).
midline)
• Features: Voice produced is weak and husky

• •
SECTION IV Larynx

QUESTIONS

1. Which of the following muscle is not supplied by recur- 12. W h i c h one of the f o l l o w i n g lesions of v o c a l cord is
rent laryngeal nerve: [PGI Dec 08] dangerous to life: [UPSC01,02]
a. Post cricoarytenoid b. Thyroarytenoid a. Bilateral adductor paralysis
c. Lateral cricoarytenoid d. Cricothyroid b. Bilateral abductor paralysis
e. Interarytenoids c. Combined paralysis of left side superior and recurrent
2. Cricothyroid muscle is supplied by: [Jharkhand2003] laryngeal nerve
a. Superior laryngeal nerve b. External laryngeal nerve d. Superior laryngeal nerve paralysis
c. Vagus nerve d. Glossophryngael nerve 13. In complete bilateral palsy of recurrent laryngeal
3. Position of vocal cord in cadaver is: [DNB 2000] nerves, there is: [AIIMS Nov. 03]
a. Median b. Paramedian a. Complete loss of speech with stridor and dyspnea
c. Intermediate d. Full Abduction b. Complete loss of speech but not difficulty in breathing
4. Why vocal cord looks pale? [TN2005] c. Preservation of speech with severe stridor and dyspnea
a. Vocal cord is muscle, lack of blood vessels network d. Preservation of speech and not difficulty in breathin
b. Absence of mucosa, no blood vessels 14. In bilateral abductor paralysis which of the following is
c. Absence of sub mucosa, no blood vessels seen:
d. Absence of mucosa with blood vessels a. Vocal cord in paramedian position
5. Right sided vocal cord palsy seen in: [AIIMS 99] b. Voice is affected early
a. Larynx carcinoma c. Stridor & dyspnoea occurs
b. Aortic aneurysm d. Vocal cord lateralization done
c. Mediastinal lymphadenopathy e. Hoarseness occurs
d. Right vocal nodule 15. The voice in a patient with bilateral abductor paralysis
6. The most common cause of vocal cord palsy is: [UPSC05] of larynx is: [AP2005]
a. Total thyroidectomy b. Bronchogenic carcinoma a. Puberuophonia
c. Aneurysm of aorta d. Tubercular lymph nodes. b. Phonasthenia
7. Left sided vocal cord palsy is commonly due to:[TN2005] c. Dysphonia plicae ventricularis
a. Left hilar bronchial carcinoma d. Normal or good voice
b. Mitral stenosis 16. In B/L, abductor palsy of vocal cords following is done
c. Thyroid malignancy except: [PGI 98]
d. Thyroid surgery a. Teflon paste b. Cordectomy
8. Vocal cord palsy is not associated with: [AP 2003] c. Nerve muscle implant d. Arytenoidectomy
a. Vertebral secondaries 17. Injury to superior laryngeal nerve causes: [AIIMS]
b. Left atrial enlargement a. Hoarseness b. Paralysis of vocal cords
c. Bronchogenic carcinoma c. No effect 8. Loss of timbre of voice
d. Secondaries in mediastinum 18. Paralysis of recurrent laryngeal nerve true is: [Bihar 05]
9. Bilateral (B/l) recurrent laryngeal nerve palsy is/ are a. Common in (Lt) side b. 5 0 % idiopathic
caused by: [PGI 00] c. Cord will be laterally d. Speech therapy given
a. Thyroid surgery 19. Partial recurrent larynegeal nerve palsy produces vocal
b. Thyroid malignancy cord in which position: [UP 96]
c. Aneurysm of arch of aorta a. Cadaveric b. Abducted
d. Viral infection c. Adducted d. Paramedian
e. Mitral valve surgery 20. U/L vocal cord palsy treatment includes: [PGI Nov 09]
10. Cause of B/L Recurrent laryngeal nurve palsy is/are: a. Isshiki type I thyroplasty b. Isshiki type II thyroplasty
[PGI Nov. 09] c. Woodmann operation d. Laser aartenoidectomy
a. Thyroid Ca b. Thyroid Surgery e. Teflon injection
c. BlonchogenicCa d. Aortic aneurysm 21. Type I thyroplasty is for: [AI03]
e. Cervical lymphadenopathy a. Vocal cord medialization
11. Bilateral recurrent laryngeal nerve palsy is seen in: b. Vocal cord lateralization
[Delhi 2008] c. Vocal cord shortening
a. Thyroidectomy d. Vocal cord lengthening
b. Carcinoma thyroid 22. In thyroplasty type 2, vocal cord is: [AP 2004]
c. Cancer cervical oesophagus a. Lateralized b. Medialized
d. All of the above c. Shorterned d. Lengthened
CHAPTER 14 Vocal Cord Paralysis _J 165

23. A 1 0 year old boy develped hoarseness of voice follow- 24. A patient presented with stridor and dyspnea which he
ing an attack of diphtheria. On examination, his Rt vocal developed after an attack of upper respiratory tract infection.
cord was paralysed. The treatment of choice for paralysed On examination he was found to havea 3mm glottic opening.
vocal cord will be: [AIIMS Nov. 05] All ofthe following are used in the management except:
a. Gel foam injection of right vocal cord a. Tracheostomy [AIIMS 02]
b. Fat injection of right vocal cord b. Arytenoidectomy
c. Thyroplasty type—I c. Teflon injection
d. Wait for spontaneous recovery of vocal cord d. Cordectomy

EXPLANATIONS AND REFERENCES

1. Ans. is d i.e. Cricothyroid


2. Ans. is a i.e. Superior laryngeal nerve Ref. Dhingra 5th/ed p317,6th/edp 298; Scotts brown 7th/ed p2139
All t h e muscles which play any role in movement of vocal cord are supplied by recurrent laryngeal nerve except the cricothyroid
muscle which receives its innervation f r o m the external laryngeal nerve - a branch of superior laryngeal nerve.
Ans. is c i.e. Intermediate Ref. Dhingra 5th/ed p318,6th/ed p299; Table 60.2
In cadaveric state - the position of vocal cord is intermediate (i.e. equal amount of adduction & abduction)
Ans. is c i.e. Absence of submucosa, no blood vessels Ref. Maqbool 71th/edp310
• Vocal cord are fibro elastic bands.
• They are f o r m e d by reflection o f t h e mucosa over vocal ligaments.
• They have stratified squamous epithelium w i t h no submucous layer
• Their blood supply is poor and are almost devoid of lymphatics. Hence vocal cords look pale in appearance.
Ans. is a i.e. Larynx carcinoma Ref. Dhingra 5th/ed p318; 6th/ed p 298
This question can be solved easily if you know the course of Left and Right recurrent laryngeal nerve.
As discussed in detail in text:
• LtRLN.: Arises f r o m vagus in the mediastinum at the level of arch of aorta loops around it and then ascends into the neck.
• Rt. RLN: Arises f r o m vagus at the level of subclavian artery, hooks around it and then ascends up.
So, any mediastinal causes viz mediastinal lymphadenopathy and aortic aneurysm w o u l d parlyse Lt. RLN. only (ruling o u t options
" b " a n d "d") Vocal nodule does not cause vocal cord palsy.
Laryngeal carcinoma especially glottic can cause U/L or B/L Vocal Cord paralysis -Conn's Current Theory
Ans. is a i.e. Total thyroidectomy
Ans. is a i.e. Left hilar bronchial carcinoma
Ans. is a i.e. Vertebral secondarces Ref Schwartz surgery 8th/ed p 509; Dhingra 5th/ed p 320,6th/ed p299
Vocal cord paralysis is most commonly iatrogenic in origin following surgery t o Thyroid, parathyroid, carotid or cardiothoracic
structures.

Right Left Both


• Mor~U t r 3 i i m p j Nsck
• INt:i_r\ LI d u l 1 Id
• Accidental trauma Thyroid surgery
• Benign or malignant thyroid disease
• Thyroid surgery • Thyroid disease (benign or malignant) Carcinoma thyroid
• Carcinoma cervical oesophagus • Thyroid surgery Cancer cervical oesphagus
• Cervical lymphadenopathy • Carcinoma cervical oesophagus Cervical lymphadenopathy
• Cervical lymphadenopathy
//. Mediastum
• Aneurysm of subclavian atery • Bronchogenic cancer (M/C)
• Carcinoma apex right lung • Carcinoma thoracic oesophagus
• Tuberculosis of cervical pleura • Aortic aneurysm
• Idiopathic • Enlarged left atrium
• Intrathoracic surgery
• Idiopathic

9. Ans. is a, b and di.e. Thyroid surgery; Thyroid malignancy; and Viral infection
10. Ans. is a, b a n d e i.e. Thyroid Ca, Thyroid Surgery and Cervical lymphadenopathy
11. Ans. is d i.e. all of the above Ref. Dhingra 5th/ed p21, 6th/ed p299; Turner 10th/ed pi 81; Current Otolaryngology 2nd/ed p 457

)
166|_ SECTION IV Larynx

Causes of bilateral recurrent laryngeal nerve palsy are:


• Idiopathic
• Post thyroid surgery
• Thyroid malignancy
• Carcinoma of cervical part of esophagus
• Cervical Lymphadenopathy

Peripheral neuritis causes high vagal palsy which leads to both superior as well as recurrent laryngeal nerve palsy i.e. bilateral complete palsy. Turner
10/e p. 181; Dhingra 5/e p. 318; 6/e, p301

12. Ans. is b i.e. bilateral abductor paralysis Ref. Dhingra 5th/edp 318-319; 6th/edp 300
• Most dangerous lesion of vocal cords is bilateral abductor paralysis (Bilateral RLN palsy).
• This is because recurrent laryngeal nerve palsy will lead t o paralysis of all laryngeal muscles except the cricothyroid muscle (as
it is supplied by superior laryngeal nerve). The cricothyroid muscle is an adductor & therefore this will leave both the cords in
median or paramedian position thus endangering proper airway, leading t o stridor and dyspnoea.
13. Ans. is c i.e. Preservation of speech with severe stridor and dyspnea
14. Ans. is a, c a n d d i.e. Vocal cord in paramedian position; Stridor and dyspnoea occurs; and Vocal cord lateralization done
15. Ans. is d i.e. normal or good voice Ref. Dhingra 5th/ed p 318; 6th/edp 300; Current Otolaryngology p 459-460

Bilateral Recurrent laryngeal nerve |

r I
Supplies all laryngeal muscles Except the cricothyroid
1 I
.-. its paralysis leads to paralysis of all laryngeal muscles | | It is spared in case of recurrent laryngeal nerve |

Leads to adduction of vocal cord


T
Vocal cord lie in median / paramedian position


Voice = normal
1.
In case of upper respiratory
infection
T
Dyspnea / Stridor occur
.•. it is a life threatening
condition

Management

• Lateralization of cord by arytenoidectomy, endoscopic surgery, thyroplasty type II, cordectomy


• In emergency cases -Tracheostomy may be required

Also know
• Generally patients with bilateral recurrent laryngeal nerve palsy have a recent history of thyroid surgery or rarely an advanced malignant
thyroid tumor.
» Most common presentation-Development of stridor following URI
» Since the voice ofthe patient is normal\it is diagnosed very late.

16. Ans. is a i.e Teflon paste Ref. Dhingra 5th/edp 319,6th/edp 300
• In Bilateral Abductor paralysis (i.e. bilateral paralysis of RLN), the cords lie in median or paramedian position due to unopposed action
of cricothyroid muscle.
• Since, b o t h the cords lie in median or paramedian position, the airway is inadequate causing dyspnea and stridor.
• Principle for managing such cases is: lateralisation o f t h e cord and not further medialization of cord by injection of Teflon
For more details see the proceeding text.

17. Ans. is d i. e. Loss of timbre of voice Ref Dhingra 5th/ed p 320; 6th/ed p 300
Paralysis of Superior Laryngeal Nerve -causes paralysis of cricothyroid muscle which is a tensor of vocal cord.


CHAPTER 14 Vocal Cord Paralysis J 167

Clinical Features

• Voice is weak and pitch cannot be raised.


• U/L Anaesthesia of larynx above the level of vocal cords causing occasional aspiration.
18. Ans. is a i.e. Common in (left) side Ref. Dhingra 5th/edp318,6th/ed p 299; Current otolaryngology 2nd/edp 457

Unilateral Recurrent Laryngeal Nerve Palsy

• More c o m m o n on left side than right side because o f t h e longer and more convoluted course o f t h e left recurrent laryngeal
nerve (Rt side is involved only in 3-30% cases) (i.e. option a is correct)
• Most unilateral vocal cord paralysis are secondary to surgery (i.e. o p t i o n b is incorrect)
• Unilateral injury t o recurrent laryngeal nerve leads t o ipsilateral paralysis of all intrinsic muscles except cricothyroid (which is
an adductor of vocal cord). The vocal cord thus assumes a median or paramedian position which does not move laterally on
deep inspiration (i.e. option c is incorrect)

Clinical Features

• Asymptomatic in 1/3 cases


rd

• In rest of the patients there may be some voice problem i.e. Dysphonia - the voice is hoarse & becomes weak w i t h use. This
gradually improves w i t h t i m e due t o compensation by the healthy cord which crosses the midline t o meet the paralysed one.
Generally no speech therapy is required (i.e. option d is incorrect).
19. Ans. is d i.e. Paramedian Ref. Dhingra 5/e,p 318; 6/e,p297

Nerve paralysed Muscles aftected Position of vocal cord

• Recurrent Larynageal Nerve All muscles of larynx except Cricothyroid (Which is an adductor) Median, paramedian
• Superior Laryngeal nerve Cricothyroid Normal but cord loses tension
• Both recurrent and superior laryngeal All muscles of larynx except interarytenoid which also receives Cadaveric position
nerve of one side innervation from opposite side.

20. Ans is a & e i.e. Isshiki type I thyroplasty & Teflon injection
Ref: Dhingra 5th/ed pg 320; 6/e, p 300 Logan & Turner W' /182,183
h

Combined (Complete) Paralysis (Recurrent & Superior Laryngeal nerve paralysis): Unilateral
It leads t o paralysis of all the muscles of larynx on one side except the cricoarytenoid which also receive innervations f r o m the o p -
0

posite side. Vocal cord o f t h e affected side will lie in the cadavaric position .The healthy cord is unable to approximate the paralysed
Q

side.This results in hoarseness of voice and aspiration occurs t h r o u g h the glottis.


Treatment
• Speech therapy - With proper speech therapy the healthy cord may approximate the paralysed cord.
• Procedures t o medialise the cord
- Injection of Teflon paste, lateral t o the paralysed c o r d 0

Thyroplasty type 1°
Muscle or cartilage i m p l a n t
0

Arthrodesis of cricoarytenoid joint (Also known as Reversed


Woodman's operation - Logan & Turner 10 /182) th

Woodman's operation 0
(External arytenoidectomy) is done in bilateral abductor paralysis- Logan & Turner I0th/183
Endoscopic laser arytenoidectomy & Isshiki type II thyroplasty is done for lateralization of cord (in bilateral abductor paralysis)"-Dhingra
0

5th/318,319&362

21. Ans. is a i.e. Vocal cord medialisation Ref. Dhingra5/e,p 321


22. Ans. is a i.e. Lateralised
Isshiki divided thyroplasty procedures into 4 categories to produce functional alteration of vocal cords:
• Type 1 : Medial displacement of vocal cord (done by injection of gel foam/Teflon paste) —-ft
• Type 2 : Lateral displacement of cord (done to improve the airway)
• Type 3 : Shortening (relax) the cord, t o lower the pitch (gender transformation f r o m female t o male)
• Type 4 : Lengthening (tightening) the cord, t o elevate the pitch (gender transformation f r o m male t o female)
SECTION IV Larynx

23. Ans. is d i.e. Wait for spontaneous recovery of vocal cord Ref. Dhingra 5/e, p318; 6/e, p 300 Nelson 17/e, p 888-889
Unilateral paralysis of cord due to neuritis (as in diphtheria) does not require any treatment as it recovers spontaneously.
The characteristic features of diphtherial neuropathy is that it recovers completely.
24. Ans. is c i.e Teflon injection Ref. Dhingra 5/e, p 318-319; 6/e, p 300
• Glottic diameter of 3 m m indicates that the patient is having laryngeal paralysis (due t o URTI).
• Because o f the narrowness of the opening, the patient is having stridor and dyspnea.
• Stridor and dyspnea can be managed by:
- Tracheostomy
- Fixing the cord in the lateral position by:
• Arytenoidectomy
• Aretynoid pexy
- Vocal cord lateralisation t h r o u g h endoscope
- Laser cordectomy
- Thyroplasty type II.
• Teflon injection is a method tomedialisethecord and is therefore of no use in this patient. It would rather aggravate the condition.

For a quiet respiration the glottic diameter should be 14 mm wide.

25. Ans. a i.e. Patients with the abductor type have strained and strangled voice Ref. Dhingra, 6/e, p 314.
• Spasmodic dysphonia (or laryngeal dysphonia) is a voice disorder characterized by involuntary movements or spasms of one or
more muscles ofthe larynx (vocal folds or voice box) during speech.
• The three types of spasmodic dysphonia (SD) are adductor spasmodic dysphonia, abductor spasmodic dysphonia and mixed
spasmodic dysphonia.

Adductor Spasmodic Dysphonia

• In adductor spasmodic dysphonia (ADSD), the addutor muscles of larynx go into spasm causing the vocal folds (or vocal cords)
t o adduct and stiffen.
These spasms make it difficult for the vocal folds t o vibrate and produce voice. Words are often cut off or difficult to start because
o f t h e muscle spasms. Therefore, speech may be choppy.
• The voice of an individual with adductor spasmodic dysphonia is commonly described as strained or strangled and full of effort.
Surprisingly, the spasms are usually absent while laughing, speaking at a high pitch, or speaking while singing, b u t singers can
experience a loss of range or the inability t o produce certain notes of a scale or with projection. Stress, however, often makes
the muscle spasms more severe.
• Larynx is morphologically normal

Abductor Spasmodic Dysphonia

• In abductor spasmodic dysphonia, sudden involuntary spasm of abductor muscle i.e. posterior cricoarytenoid causes the vocal
folds t o remain open.
• The vocal folds cannot vibrate w h e n they are open. The open position of the vocal folds also allows air t o escape f r o m the lungs
during speech. As a result, the voices of these individuals often sound weak, quiet and breathy or whispery.
• The condition is progressive and symptoms get aggravated during period of stress or when patient uses telephone.

Mixed Spasmodic Dysphonia

Mixed spasmodic dysphonia involves muscles that open the vocal folds a well as muscles that close the vocal folds and therefore
has features of b o t h adductor and abductor spasmodic dysphonia.

T r e a t m e n t in A d d u c t o r Dsysphonia
• Inj Botulinium toxin: Given in thyroarytenoid muscle, by percutaneous electromyography guided route t h r o u g h crico thyroid
space. Benefit lasts for 16 weeks so repeat injection is required. Result is g o o d .
• Voice therapy.
Treatment in abductor dysphonia-
• Inj Botulenium toxin: Given in posterior cricoarytenoid muscle by percutaneous or endoscopic route.
Result: Not very g o o d .
• Voice therapy
• Surgery-ln patients w h o d o n o t respond t o botulenium injection or voice therapy-Thyroplasty type I or fat injection may be given
CHAPTER

• 15 ^^^^^
Tumor of Larynx

Flow chart 15.1: Classification of laryngeal neoplasms • Interferon alfa can also be used as an adjuvant therapy in
patients w i t h severe disease b u t has several side effects like
LARYNGEAL NEOPLASM fever, chills, myalgia, arthralgia, headache, w e i g h t loss and
bone marrow suppression

Benign
1
Malignant] • Recurrence after removal is c o m m o n .

Ca larynx Adult onset papilloma


• Single, smaller in size, less aggressive and donot recur
Squamous
T
Granular cell Glandular
after surgery.
Chondroma Hemangioma • Most common age affected is 30-50 years and is more
papillomas tumour tumour
c o m m o n in males.
• It arises f r o m anterior half of vocal cord or anterior c o m -
missure.
I SQUAMOUS PAPILLOMAS
• Hoarseness is the presenting symptom.
Mostcommon benign tumour. • Treatment is same as of Juvenile papillomas

It is of two types:
1. Juvenile onset/Recurrent respiratory papillomatosis | CHONDROMA
(JORRP)/Multiple papillomatosis.
Most of t h e m arise f r o m cricoid cartilage and cause dyspnea
• Viral in origin, caused by HPV types 6 and 11 and less
commonly by subtypes 16 and 18 or lump in throat.
• Multiple sessile / pedunculated, friable papillomas which Mostly affect men in age group 40-60.
bleed on t o u c h . C 0 laser is useful for biopsy.
2

• Occurs in infants and young children - peak age 2 to4years. Management is: excision of tumor.

? • Most common site - vocal fold (first and predominant site)....


HEMANGIOMA

Infantile hemangioma involves the subglottic area and presents


I CSDT 12/e,p 971 w i t h stridor in first 6 months of life.
Other sites = other parts of larynx, nose, pharynx and trachea. • Tends t o involute spontaneously but a tracheostomy may be
needed t o relieve respiratory obstruction.
• Patient presents w i t h hoarsness - Later as the lesion p r o -
• Treated by C 0 laser.
gresses inspiratory dyspnea w i t h stridor develops.
2

Adult hemangioma involves vocal cord or supraglottic larynx.


• Most are cavernous type and can't be treated w i t h laser.
• Notreatment is required for asymptomatic cases, larger ones are
Vertical transmission also occurs. treated by steroids or radiation therapy.

Treatment GRANULAR CELL TUMOR

• Micro e n d o s c o p i c C0 laser excision of papillomas at fixed


2
Arise f r o m Schwann cells and is often submucosal.
interval (2, 4 and 6 month) according to individual need is the
Overlying epithelium shows pseudoepitheliomatous hyper-
treatment of choice. plasia which resemble well differentiated cancer.
170^ SECTION IV Larynx

| CANCER LARYNX of carcinoma of larynx. It is an i m p o r t a n t division and is based


on laymphatic drainage. The area above t h e vocal cords, i.e.,
More prevalent in India. supraglottis drains upwards via the superior lymphatic t o upper
Age: Most c o m m o n in age group 40-70 yrs. deep vervical group of lymph nodes. Vocal cords, i.e., glottis has
practically no lymphatics so, acts as a watershed. The area below
the glottis (subglottis) drains t o prelaryngeal and paratracheal
glands and then t o lower deep cervical nodes. Incidence of larynx
cancer by site:

Supraglottis cancer = 4 0 %
Glottic cancer = 59%
Subglottic cancer = 1 %

Classification

According t o site Ca larynx is divided into:


• a. Supraglottic cancer: Less common
• Majority of lesion is on epiglottis, false cords or ventricular
bands followed by aryepiglottic folds (laryngeal aspect
only); arytenoids.
• Symptoms:
Pain on swallowing is the mostfrequent initial symp-
tom. Devita 7/e, p 698
Mass in neck may be the first sign.
Hoaresness is a late symptom.
Pain may referred to ear by vagus nerve and auricular
Fig. 15.1: Carcinoma larynx. (A) Supraglottic; and (B) Glottic nerve of arnoid.
Courtesy: Text book of Diseases of Ear, Nose and Late symptoms include foul breath, dysphagia and
Throat, Mohan Bansal. Jaypee Brothers, p 504 aspiration.
Large t u m o r s can cause h o t p o t a t o voice/muffled
• Males > females: M/C in lower socio economic class
voice.
• Occurrence: Glottis (55-75%) > supraglottis (24-42%) > Sub-
Hemoptysis, stridor, dyspnea, aspiration pneumonia
glottis (1-2%) may also occur.
Spread:
Etiology
Locally to invade vallecula, base of t o n g u e and pyri-
form fossa.
Lymphatic: Greatest incidence of nodal spread, nodal
Tobacco smoking and alcohol are most important. Combination metastases occurs early and is bilateral Upper and
of alcohol and smoking increase the risk 15 fold. middle jugular nodes
are often involved.
• Previous neck irradiation. b. Glottic Cancer (M/C)
• Occupational exposure t o asbestos, mustard gas and petro- • Glottic cancer is the commoniest site. Mostly originates
leum products. on free edge and undersurface of Anterior 1 /3 of true vocal
• HPV-16 and 18 are also implicated. cord. Earliest to present (as hoarseness ), least predilection
0

• Premalignant conditions = Solitary papilloma, leukoplakia for neck node involvement and has the best prognosis. Due
and keratosis. t o the paucity of lymphatics, glottic malignancy is highly
radiosensitive.
Histopathology » Subglottic malignancy is the least c o m m o n site, last t o
present as stridor , has the worst prognosis since it involves
0

• 90 - 9 5 % of Ca larynx are squamous cell ca. the paratracheal and mediastinal nodes.
• Cordal lesions are well differentiated while supraglottic Ones
are anaplastic. Diagnosis

Site of L a r y n g e a l T u m o r s IOC = Direct laryngoscopy is used t o assess the extent of t u m o r


and for obtaining biopsy of the cancer.
As discussed previously, larynx is divided into supraglottic, glottic • CT: Very useful investigation t o find the extent of t u m o r and
and subglottic regions f o r t h e purpose of anatomical classification invasion of preepiglottic or paraepiglottic space.
CHAPTER 15 Tumor of Larynx

MRI: It is less suitable than CT due t o motion artifacts associated w i t h longer scanning time.
Supravital staining and biopsy: Toluidine blue is applied t o laryngeal lesion and then washed w i t h saline. Carcinoma in situ and
superficial carcinoma take dye while leukoplakia does not.

Staging: -Devita 7/e, p 698

TNM classification of cancer larynx (AJCC 2002) •

Primary Tumor (T)

Supraglottis
T1 Tumor limited to one subsite of supraglottis with normal vocal cord mobility

T2 Tumor invades mucosa of more than one adjacent sub-site of supraglottis or region outside the supraglottis, without fixation
of larynx.
T3 Tumor limited to larynx with vocal cord fixation and/or invades any of the following: postcoricoid area preepiglottic tissues,
paraglottic space, and/or minor thyroid cartilage erosion (e.g. inner cortex).
T4a Tumor invades through the thyroid cartilage and/or invades tissues beyond the larynx.
T4b Tumor invades prevertebral space, encases carotid artery or invades mediastinal structures
Glottis
T1 Tumor limited to one (T1 a) or both (T1 b) vocal cord(s) (may involve anterior or posterior commissure) with normal mobility
T2 Tumor extends to supraglottis and/or subglottis, or with impaired vocal cord mobility
T3 Tumor limited to the larynx with vocal cord fixation, and/or invades paraglottic space, and/or minor thyroid cartilage erosion
(e.g. inner cortex)
T4
Same as supraglottis
Subglottis
T1
Tumour limited to subglottis
T2
T3 Tumor extends to vocal cords with normal or impaired mobility

T4 Tumor limited to larynx with vocal cord fixation


Same as supraglottis

R e g i o n a l L y m p h N o d e s (N) opsy. If biopsy shows invasive carcinoma, radiotherapy is given


otherwise regular follow up is done.
• T, carcinoma: Radiotherapy is the treatment (as voice is pre-
served). These days mucolaryngoscopic surgery is the treatment
Cancer larynx first spreads to the cervical nodes. The next M/C site of
spread is lungs for this reason chest X-ray should be a part ofthe routine of choice.
metastatic evaluation (in all head and neck cancers). T Carcinoma with extension to anterior commissure: or
T Ca with extension to arytenoid: Radiotherapy is n o t
N x Regional l u m p h nodes cannot bejassessed. preferred because of the possibility of developing per-
N No regional lymph node metastatis
0
chondritis w h i c h w o u l d entail total laryingectomy. In
N, Metastasis in a single Ipsilateral lymph node, 3 cm or less
such cases some form of conservation surgery like vertical
in greatest dimension.
hemilaryngectomy or fronto lateral laryngectomy is done
N 2 Metastatis in a single ipsilateral lymph node, more than
t o preserve the voice.
3 cm b u t not more than 6 cm in greatest dimension, or
multiple ipsilateral lymph nodes, none more than 6 cm • T Carcinoma: Treatment depends on: (i) Mobility of vocal cords,
2

in greatest dimension, or bilateral or contralateral lymph and (ii) Involvement of anterior commissure and/or arytenoid:
nodes, nodes, not more than 6 cm in greatest dimension. Ifmobilityofcord is not impaired (cord is mobile) and anterior
N 2a Metastasis in a single ipsilateral lymph node more than 3 commissure and/or arytenoid not involved: Radiotherapy
cm b u t n o t more than 6 cm in greatest dimension. is the t r e a t m e n t of choice. In case o f recurrence total
laryngectomy or partial vertical laryngectomy is done.
Treatment If mobility of cord is impaired or anterior commissure and/
Glottic / Vocal cord carcinoma or arytenoid involved: Voice preserving conservative sur-
Stage dependent treatment include: gery such as vertical hemilaryngectomy or frontolateral
• Carcinoma in Situ (CIN): Best treated by transoral endoscopic laryngectomy is done. Total laryngectomy is done if there
C0 laser. If laser is n o t available stripping of vocal cord is done
2
is recurrence on follow up.
(Endo/microlaryngeal stripping) and the tissue is sent for b i -
SECTION IV Larynx

• T , T c a r c i n o m a : l n T lesions-TOC is primary chemoradiation


3 4 3 Vocal rehabilitation after L a r y n g e c t o m y
w i t h total laryngectomy as salvage surgery in residual lesion.
• Oesophageal speech:
Rough voice but loud and understandable
S u r g e r y in C A l a r y n x
• Artificial larynx:
1. Conservation Surgery Electrolarynx and trans oral pneumatic device.
• Total laryngectomy is done for most of the laryngeal cancers
• Tracheo oesophageal speech:
and the patient is left w i t h no voice and a permanenttracheos-
Neoglottis formation.
tome. Conservative laryngeal surgery is one which can preserve
voice and also avoid a permanent tracheal opening. However,
f e w cases w o u l d be suitable for this type of surgery and they
It is contraindicated in patients with distant metastain
should be carefully selected. Conservation surgery includes:
• Excision of vocal cord after splitting the larynx (cordectomy
3. Endoscpic resection with C o laser: 2

via laryngofissure).
Carcinoma of the mobile membrances vocal cord is n o w a
• Excision of vocal cord and anterior commissure region (partial
days treated w i t h excision via Co laser w i t h better results than
2

. frontolateral laryngectomy).
traditional radiotherapy.
• Excision of supraglottis, i.e. epiglottis, aryepiglottic folds, false
cord and verticle - a sort of transverse section of larynx above Complication of Treatment
t h e vocal cords (partial horizontal laryngectomy).
2. Total Laryngectomy Includes: Surgery: - Speech loss after laryngectomy.
• Resection of w h o l e o f larynx u p t o 1cm below the vocal cords. Radiation: - Laryngeal edema and o d y n o p h a g i a are
• Resction o f part of anterior wall of pharynx. most common complication after radiation
• Repair o f pharyngeal wall. for glottic or supraglottic lesion.
» Trachoestome formation above the suprasternal notch.

Also know
Indications of total laryngetomy > Glottic Ca carcinoma carries the best prognosis because of the
» T lesions (i.e. with cord fixed)
3 early diagnosis and relatively few lymphatics.
• All T lesions
4 » Most frequent site of recurrence in glottic Ca is around tracheal
» Invasion of thyroid or cricoid cartilage stoma in the base of tongue and in neck nodes.
> Bilateral arytenoid cartilage involvement » CT scan is the best investigation to find out the nature and
> Lesions of posterior commissure extent of growth besides direct laryngoscopy examination.
• Failure after radiotherapy or conservation surgery
» Tranglottic cancers, i.e. tumours involving supraglottis and
glottis across the ventricle, causing fixation ofthe vocal cord.


CHAPTER 15 Tumor of Larynx J173

QUESTIONS

1. Premalignant conditions for carcinoma larynx would 11. T h e m o s t c o m m o n a n d e a r l i e s t m a n i f e s t a t i o n of


include: [PGI 01] carcinoma of the glottis is: [Al 05, RJ-2006]
a. Leukoplakia b. Lichen planus a. Hoarseness b. Haemoptysis
c. Papillomas d. Smoking c. Cervical lymph nodes d. Stridor
e. Chronic laryngitis 12. Lymph mode metastasis in neck is almost never seen
2. Which of the following is precancerous lesion: [UP 00] with: [Al 96]
a. Pachydermia of larynx b. Laryngitis sicca a. Carcinoma vocal cords b. Supraglottic carcinoma
c. Keratosis of larynx d. Scleroma larynx c. Carcinoma of tonsil d. Papillary carcinoma thyroids
3. Of the following statements about Recurrent Laryngeal 13. Which of the following carcinomas commonly presents
papillomatosis are true. Except [AI-09] with neck nodes: [Al 95]
a. Caused by Human Papilloma Virus (HPV) a. Cricoid b. Glottic
b. HPV6 and HPV11 are most commonly implicated c. Epiglottis d. Anterior commissure
c. HPV6 is more virulent than HPV11 14. True statement about Infrglottic carcinoma larynx:
d. Transmission t o neonate occurs through contact w i t h a. Commonly spreads to mediastinal nodes [PGI 96]
mother during vaginal delivery b. Second most common carcinoma
4. True about juvenile respiratory papillomatosis: [PGI 00] c. Most common carcinoma
a. Affects children commonly d. Spreads to submetal nodes
15. The treatment of choice for stage I cancer larynx is:
b. Lower respiratroy tract can be involved
a. Radical Surgery [AIIMS 03, PGI 98]
c. May resolve spontaneously
b. Chemotherapy
d. Microlaryngoscopic surgery is treatment of choice
c. Radiotherapy
5. True about multiple papillomatosis: [PGI Dec. 05]
d. Surgery followed by radiotherapy
a. HSV is causative agent
16. Treatment of Ca larynx in stage. T1, MONO is: [AI00]
b. Radiotherapy treatment of choice
a. Radiotherapy b. Surgery Total laryngectomy
c. It is premalignant
c. Laser therapy d. Micro laryngoscopic surgery
d. It is more c o m m o n in 15 t o 33 yrs
17. For a mobile tumour on vocal cord, treatment is:
e. It recurs due to parturition
[AIIMS 92, AP 96]
6. True about Juvenile laryngeal papillomatosis:
a. Surgery b. Chemotherapy
a. Caused by HPV. [PGI May 2011]
c. Radiotherapy d. None of the above
b. No risk of recurrance after surgical removal
18. For carcinoma larynx stage III Treatment of choice:
c. Tends to disappear after puberty
a. Radiotherapy and Surgery [AIIMS 96]
d. Interferon therapy is useful
b. Chemotherapy with cisplatinum
7. Kamla 4 yrs of age presented in emergencywith mild re-
c. Partial laryngectomy with chemotherapy
spiratory distress. On laryngoscopy she was diagnosed
d. Radiotherapy with chemotherapy
to have multiple juvenile p a p i l l o m a t o s i s o f t h e larynx.
19. Treatment of choice in stage III carcinoma larynx is:
Next line of management is: [AIIMS 01]
[Al 98, RJ 2002]
a. Tracheostomy b. Microlaryngoscopy
a. Chemotherapy b. Surgery + radiation
c. Steroid d. Antibiotics c. Surgery + chemotherapy d. Only radiotherapy
8. All the following are true about Laryngeal carcinoma 20. Radiotherapy is the TOC for: [AIIMS Nov. 09]
except: [Al 94] a. Nasopharyngeal CaT N, b. Supraglottic CaT N
3

a. More common in females c. Glottic CaT N, 3 d. Subglottic CaT N 3 0

b. Common in patients over 40 years of age • 21. A patient of carcinoma larynx with stridor presents in
c. After laryngectomy, esophageal voice can be used casualty, immediate management is: [AIIMS 91 ]
d. Poor prognosis a. Planned tracheostomy
9. Features of laryngeal Ca: [PGI June 05] b. Immediate tracheostomy
a. Glottis is the MC site c. High dose steroid
b. Commonly metastasizes to cervical lymph node d. Intubate, give bronchodilator and wait for 12 hours, if no
c. Lesions seen at the edge of the vocal cord response, proceed to tracheostomy
d. Laryngeal compartments acts as barrier e. None
10. Supraglottic Ca present with: [PGI June 03] 22. Which of the following is not the indication of near total
a. Hot potato voice Laryngectomy? [AP2007]
b. Aspiration a. T3 stage
c. Smoking is common risk factor b. Anterior commissure involvement
d. Pain is MC manifestation c. Free lateral arytenoids
e. Lymph node metastasis is uncommon d. Interarytenoid plane involvement
174|_ SECTION IV Larynx

23. A p a t i e n t p r e s e n t s w i t h c a r c i n o m a of t h e l a r y n x a. Pulmonary surgery b. Electron beam therapy


involving t h e left false cord, left arytenoids a n d t h e c. Total laryngectomy d. Endoscopic removal
left aryepiglottic folds with bilateral mobile true cords. 28. Laryngofissure i s : [Jipmer04]
Treatment of choice is: [AIIMS Nov. 07] a. Opening the larynx is midline
a. Vertical hemilaryngectomy b. Making window in thyroid cartilage
b. Horizontal hemilaryngectomy c. Removal of arytenoids
c. Radiotherapy followed by chemotherapy d. Removal of epiglottis
d. Total laryngectomy 29. About total laryngectomy all is correct except:
24 A case of c a r c i n o m a larynx w i t h t h e involvement of [Bihar 2005]
anterior commissure a n d right vocal cord, developed a. Loss of smell b. Loss of taste
p e r i c h o n d r i t i s of t h y r o i d c a r t i l a g e . W h i c h of t h e c. Speech difficulty d. Difficult swallowing
following statements is true for the management of this 30. Laryngeal cartilage involvement, investigation of choice
case? [AIIMS May 06] is: [Bihar2003]
a. He should be given radical radiotherapy as this can cure a. CT b. MRI
early tumours c. Radionucleide scans d. X-ray
b. He should be trated with combination of 31. Laser used in laryngeal work? [AI20I0]
chemotherapy and radiotherapy a. Argon b. C 0 2

c. He should first receive radiotherapy and if residula tumour c. Holmium d. NdYag


is present then should under 32. Contraindication of supraglottic laryngectomy is/are:
go laryngectomy a. Poor pulmonary reserve [PGI Nov. 09]
d. He should first undergo laryngectomy and then b. Tumor involving pyriform sinus
post-operative radiotherapy c. Tumor involving preepiglottic space
25. Treatment of choice for carcinoma LarynxTI NOMO stage: d. Vocal cord fixation
a. External beam radiotherapy [AI02] e. Cricoid cartilage extension
b. Radioactive implants
c. Surgery
d. Surgery and radiotherapy
NEET PATTERN QUESTIONS
26. Select correct statements about Ca larynx: [PGI 02] 33. Juvenile papillomatoses is caused by-
a. Glottic Ca is the most common a. HPV b. EBV
b. Supraglottic ca has best prognosis c. CMV d. HSV
c. Lymphatic spread is the most common in 34. M a i n t e n a n c e of a i r w a y d u r i n g l a r y n g e c t o m y in a
subglottic Ca patient with carcinoma of larynx is best done by-
d. T1 tumor is best treated by radiotherapy a. Tracheostomy
e. Smoking predisposes b. Laryngeal mask airway
27. The preferred treatment of verrucouse carcinoma ofthe c. Laryngeal tube
larynx is: [UP 07] d. C o m b i t u b e
a-j

EXPLANATIONS AND REFERENCES


1. Ans. is a, c, and e i.e. Leukoplakia; Papillomas; and Chronic laryngitis Ref. Read below
2. Ans. is c i.e. Keratosis of larynx
Ref. Scotts Brown 7th/ed vol-2 pg-2221; Dhingra 5th/ed pg-323,6th/ed p 304; Mohan Bansal p 487
• Lichen planus has no malignant potential. ....Turner 10th/ed p 126
• Papilloma- "The malignant transformation from benign non keratining squamous papilloma to squamous cell carcinoma can occur
in children, but is rarely seen" - Current Otolaryngology 2nd/edp 471
• Leukoplakia is a white patch, in which there is epithelial hyperplasia along w i t h atypical cells. It is a premalignant condition.
Another name for leukoplakia is hyperkeratosis dyskeratosis - Scott's Brown 7th/ed vol-2 p 2221
• Smoking is a predisposing factor, not a premalignant condition.
• In some cases of chronic laryngitis, the laryngeal mucosa becomes dysplastic particularly over true vocal folds and is a premalignant
condition. ...Bailey 24th/edp 765
• Chronic inflammatory conditions of larynx like chronic laryngitis may develop into malignancy. ... Maqbool 11 th/edp 359
Keratosis of larynx/leukoplakia:-
It is epithelial hyperplasia o f t h e upper surface of one or b o t h vocal cords.
• Appears as a white plaque or warty g r o w t h on cord w i t h o u t affecting its mobility
CHAPTER 15 Tumor of Larynx
J 175

• Regarded as a precarcerous condition as Ca in situ develops frequently


• T/t=stripping of cords
3. Ans. is c i.e. HPV6 is more virulent than HPV 11:
Nelson's pediatrics 18th/l 772; Current Otorhinology 2nd/edpg-435/471'Pediatric ENT' by Graham. Scadding and Bull (2008) 7258

Recurrent Laryngeal Papillomatosis / Recurrent Respiratory Papillomatosis

Etiology

• Associated w i t h Human Papilloma Virus infection (HPV)


• HPV6 and HPV 11 are most commonly associated with laryngeal disease whereas HPV 16 and HPV 18 are less commonly associated.
• HPV11 is associated w i t h a more aggressive disease and makes the patient more prone t o malignant change
• Thus HPV 11 is more virulent

Epidemiology

• Most c o m m o n l y occur in children <5 years of age (2 -5 years)


• Male female ratio - same (first born vaginally delivered child of a teenage mother is most prone)

Transmission

Exact mode of transmission is not k n o w n .


• There is recognized transmission f r o m genital warts.
• Vertical transmission of virus from mother to child can occur either as ascending uterine infection or through direct contact in birth canal.

ALSO KNOW
Malignant transformation in a case of papilloma occurs most commonly in distal bronchopulmonary tree and prognosis is univer-
sally poor
4. Ans. a, b, c, and d i.e. Affects children commonly. Lower respiratroy tract can be involved. May resolve spontaneously, and
Microlaryngoscopic surgery is treatment of choice
Ref. Dhingra 5th/ed pp 324,325; Current Otolaryngology 2nd/ed p 471; Mohan Bansal p 488
As discussed in the previous questions - Juvenile respiratory papillomatosis:
a. Affects children commonly, (option a is correct)
b. Lower respiratory tract can be involved - t h o u g h larynx is the M/C site affected - M o u t h , pharynx, tracheobronchial tree and
oesophagus can all be affected
Hence o p t i o n b is correct
c. May resolveXspontaneously (Hence o p t i o n c is correct)
d. Micro laryngoscopic surgery is the Treatment of choice
C 0 laser surgery, which is a f o r m o f microlaryngoscopic surgery is the Treatment of choice
2

Hence o p t i o n d is also correct.


5. Ans. is c i.e. It is Premalignant Ref. Current Otolaryngology 2nd/ed pg-471,3rd/ed pg-453-454

Option Correct / Incorrect Reference Explanation

HSV is the causative Incorrect Current 2/e pg-471 It is caused by infection with human papilloma virus
Agent (Option a) (HPV) subtype 6 and 11 not by Herpes simplex virus i.e.
HSV is not the causative agent

Radiotherapy is theTOC (Option b) ncorrect Current 2/e pg-471 The primary treatment modality for respiratory
papillomatosis is surgery" Current Otolaryngology 2/e
pg-471

It is premalignant Option c) Correct Current 2/e pg-471 Juvenile papillomatosis due to subtype 11,16,18 can
undergo malignant transformations, though it is rare.

It is M/C in 15 to 33 yrs (Option d) Incorrect Current 2/e pg-471 Respiratory papillomatosisis m/c seen in children
Dhingra 6/e, p 305 between the ages 2 to 5 years although it can be seen
in adults in third decade also.
. - . .

Confd.
176[ SECTION IV Larynx

Contd.

Option Correct/Incorrect Reference Explanation


It recurs cause is due to parturition In correct Current 2/e pg-471 These are 2 different statements -
(Option e) Dhingra 5/e pg-324 1. Papilloma has a tendency to recur
6/e, p 305-306 2. Vertical transmission can occur from mother to child
at the time of parturition. Both these statements are
correct individually.
But - It recurs and cause of recurrence is parturition is
not correct

Also Remember
• Adult onset papilloma - seen in adults in the t h i r d decade
• It is less aggressive, less chances of malignant transformation and less chances of recurrence.

6. Ans. is a, c and d i.e. Caused by HPV, tends to disappear after puberty and Interferon therapy is useful (Ref. Read below)
As discussed in previous questions-Juveline Laryngeal Papillamatosis
• It is caused by HPV
• It tends t o disappear spont aneously after puberty Ref. Dhingra 5th/edp 324,6th/edp 305
• Interferon therapy is being tried t o prevent recurrence and has been f o u n d t o be useful Ref.Dhingra5th/edp325,6th/edp306
• Option b.i.e no risk of recurence after surgery is incorrect Ref: Dhingra 5/e, p 324,6/e o 306
7. Ans. is b i.e. Micro laryngoscopy Ref. Current Otolaryngology 2nd/edpg-471,3rd/edp 454-455
• The patient (a 4 years girl) in the question is presenting w i t h mild respiratory distress due t o multiple Juvenile papillomatosis
of larynx
• The management in such a case is microlarygoscopic surgery using C 0 laser t o ablate the lesion.
2

• Steroids and antibiotics have no role.


• Tracheostomy is reserved for those patients w h o have severe respiratory distress.
8. Ans. is a i.e. More common in females and d i.e. poor prognosis
Ref. Current Otolaryngology 2nd/ed pg-437 onwards; Mohan Bansalp 502,503

Cancer Larynx

Mostcommon histological type of laryngeal Ca - Squamous cell carcinoma (seen in 9 0 % cases)


It is more c o m m o n in males.
• Male: Female ratio is 4:1) (option a is incorrect)
• Most common age = 60-70 years. •

nemonic
Aetiology: Risk factors:- Mnemonic"CA LARGES"
C - Chronic laryngitis
A - Alcohol
L - Leukoplakia •

A ' - Asbestosis
R - Radiation
G - Mustard Gas
E - Exposure to petroleum products
S - Smoking

Prognosis of Laryngeal Cancer

• Cure for larynx cancer, defined as 5 year disease free survival is generally better than for other primary site tumors o f the
aerodigestive tract. This reflects the prevalence of primary glottic tumors over supraglottic tumors and the early age at which
glottic tumours are diagnosed (Hence o p t i o n d is incorrect)
• So option a and d are both incorrect b u t if one option is t o be chosen, go for option'a'.
9. Ans. is a, b, c and d i.e. All options are correct
Ref. Dhingra 5th/d p 302,327; 6th/ed p 308,309; Tuli Ist/ed p310; Mohan Bansal pp 502,503
• As discussed previously, larynx is divided into supraglottic, glottic and subglottic regions for the purpose o f anatomical
classification of carcinoma of larynx.
CHAPTER 15 Tumor of Larynx

• It is an important division and is based on lymphatic drainage.


• The area above the vocal cords i.e. supraglottis drains upwards via the superior lymphatics to upper deep cervical group of
lymphnodes.
• Vocal cords, i:e glottis has practically no lymphatics so, it acts as a watershed.
• The area below the glottis, (subglottis) drain to prelaryngeal and paratracheal glands and then to lower deep cervical nodes.
Hence option b and d are both correct
10. Ans. is a, c a n d d i.e. Hot potato voice; Smoking is common risk factor; Pain is the most common manifestation
Ref. Devita 7th/ed p 698; Scott's Brown 7th/ed vol-2 pg-2608; Mohan Bansalp 506

Supraglottic Cancer

It is the second mostcommon laryngeal cancer (most common is glottic cancer).


Mostcommon initial symptom - pain on swallowing, (option d is correct)
/Most common I first sign - mass is neck.
Small supraglottic lesions not extending to glottis - may present with globus or foreign body sensation and parasthesia
If exophytic they may cause hemoptysis
Large tumors can cause "hot potato voice" (Option 'a' is correct)
Hoarseness is a late symptom.
Smoking is a risk for all laryngeal carcinomas, (option c is correct)
Lymphatic spread occurs early in case of supraglottic cancer, (as it has rich supply of lymphatics)

Hoarseness of voice is the presenting symptom in glottic carcinoma.

11 Ans. is a i.e. Hoarseness Ref. Dhingra 5th/ed p 327,6th/ed p 309; Current Otolaryngology 2nd/ed pg-441,3rd/ed pg 460.
In glottic cancer. .
"Hoarseness of voice is an early sign because lesion of cord affects its vibratory capacity."
For details see the text.
12. Ans. is a i.e. Carcinoma of vocal cords. Ref. Dhingra 5th/ed p 327,6th/ed p 309
"There are very few lymphatics in vocal cords and nodal metastasis are practically never seen in cordal lesions unless it has spread beyond
the region of membranous cord."
13. Ans. is c i.e Epiglottis Ref. Dhingra Sth/edp 326-327,6th/ed pg-308-309.
Supraglottic cancers: Have earliest neck nodes involvement.
Presenting features is - pain on swallowing or neck mass.
Glottic cancers: No nodes involved presenting features is hoarseness.
Subglottic cancers: Nodal metastasis occurs to pretracheal, prelaryngeal nodes.
^~ ~" • Presenting feature is stridor.
In the options qwien-epiglottis belongs to supraglottis so it will present with neck nodes.

Remember: • Ca which presents with neck nodes = supraglottis Ca


• Highest lymphnode involvement occurs in - supraglottic Ca
' Hoarseness is the presenting symptom - Glottic Ca
• Stridor is the presenting symptom in Subglottic Ca.
• Laryngeal cancer with worst prognosis = subglottic Ca
• Ca with best prognosis = Glottic Ca

14. Ans. is a i.e. Commonly spreads to mediastinal nodes Ref. Dhingra 5th/edp 327,6th/ed p 309
• Subglottic cancer is the rarest of laryngeal cancer.
• Earliest presentation is a globus or foreign body sensation in throat followed by stridor or laryngeal obstruction.
• Hoarseness is a late feature and occurs due to involvement of glottis or recurrent laryngeal nerve.
• Lymphatic spread occurs to prelaryngeal, pretracheal, paratracheal and lower jugular nodes (i.e. mediastinal nodes.)
15. Ans. is c i.e Radiotherapy Ref. Dhingra 5th/edp 329-330;Mohan Bansalp 504
Friends remember 2 very important concepts regarding laryngeal Ca:
• If the site of larynx caner viz supra glottis, glottis or subglottis is not mentioned, the cancer should be considered glottic (since
it is the M/C variety)
• Generally stage I, II, III, IV means stage T T , T , T respectively.
1( 2 3 4
178^ SECTION IV Larynx

According to Dhingra

• Radiotherapy is the treatmnt of choice for all stage I cancers of larynx, which neither impair mobility nor invade cartilage
or cervical nodes.
• The greatest advantage of radiotherapy over surgery in Ca larynx glottic cancer is - preservation of voice.
It doesnot give good results:
• If cords are fixed
• In subglottic extension •i.e. stages T and T
3 4

• In cartilage invasion
• If nodal metastasis is present
But according t o Current otolaryngology 2/e pg-445. Current Recommendations by the American Society of Clinical Oncology are
that all patients w i t h stageT, o r T laryngeal cancer, should be treated initially w i t h the intent t o preserve the larynx.
2

Microlaryngeal Surgery

i.e. endoscopic removal of selected larynx by operating microscope and microlaryngeal dissection instruments is used for treating
early stages of cancer larynx.
The advantages of surgery compared to radiation are:
o A shorter treatment period (compared t o 6 - 7 weeks for radiation)
• Saving the o p t i o n of radiotherapy for recurrence
Drawback of Surgery - Poor Voice Quality
• Hence f r o m above discussion it can be concluded that microlaryngoscopic surgery / Radiotherapy is the TOC for stage I of
laryngeal cancer.
• In the o p t i o n - Surgery and not microlaryngoscopic surgery is given.
• Hence Radiotherapy is being taken as the correct option.
16. Ans. is d i.e. Microlaryngoscopic surgery Ref. Current Otolaryngology 2/e pg-446,445, Scott's Brown 7/ed vol2 pg-2610
• Now since in this question both microlaryngoscopic surgery and radiotherapy are given, we are opting for micro laryngoscopic surgery
which is a better option

• The answer is further supported by the table given in Scott Brown 7 /ed vol-2 pg-2610
lh

R e m e m b e r : Now TOC for early laryngeal cancer is microlaryngoscopic surgery.

The C0 laser is used for early supraglottic lesions (Current Otolaryngology 2/e pg-446-447)
2

Surgery (%) Radiotherapy (%)


Five year T, 100 91.7
T • 97.4 88.8
Survival 3. ' ;
100 71.1
Five year T,
Disease free survival T 2
. 78.7 60.1

17. Ans. is c i.e. Radiotherapy Ref. Dhingra 5/e, p 230-331

According to Dhingra

Radiotherpy is the treatment of choice for vocal cord cancer w i t h normal mobility.
Normal m o b i l i t y of cord suggests that g r o w t h is only limited t o the surface and belongs t o either stage T1 orT2.
TOC for stage T1 of glottic carcinoma - radiotherapy.
TOC for stage T2 of glottic carcinoma - depends on mobility o f t h e cord

If vocal cords are mobile (i.e. g r o w t h is limited t o surface) If local cords mobility is impaired (i.e. deeper invasion)
Radiotherapy/micro laryngeal surgery is TOC Conservative surgery like vertical hemilaryngectomy or frotolateral
hemilaryngectomy is TOC.

If cord mobility is imparied radiotherapy is not preferred because ofthe possibility of developing perichondritis which would entail total laryngectomy.
According to higher books - again micro laryngoscopic surgery isTOC in early cases but since this is not an option we are going with radiotherapy.
CHAPTER 15 Tumor of Larynx J 179
18. Ans. is a i.e. Radiotherapy and Surgery Ref. Dhingra 5/e, p 329-330; 6/e p 311 Current otolaryngology 2/e p. 446; MB pp 504,504
19. Ans. is b i.e. Surgery + Radiation

Treatment of Ca Larynx

Stage Site Treatment


Tl All site External beam radio therapy/Microlaryngeal surgery

12 Glottic and subglottic lesion Radiotherapy


Supraglottic lesion Supraglottic laryngectomy
T3andT4 All sites Total laryngectomy with neck dissection for clinically positive nodes and post operative radiotherapy
if nodes are not palpable

According to current otolaryngology 2/e pg- 446


• Advance stage larynx cancer (stage III and IV) was historically treated by dual modality therapy w i t h surgery and radiation.
• Surgery done was -Total laryngectomy

BUT NOW

In stage III - Organ preservation surgery along w i t h radiation (radiotherapy + chemotherapy) has become a standard of care in
most centres.

• Chemotherapy used is - 3 cycles of Cisplatinum and 5-FU


• In stage IV- organ preservation surgery is not useful because of cartilaginous invasion. Hence in stage IV. Total laryngectomy with radiotherapy
is still being done.

As per the options given in the question - there is no such o p t i o n as surgery + chemoradiation Hence we are going w i t h surgery
+ Radiotherapy as TOC in stage III cancer.
20. Ans. is 'a' i.e., Nasopharyngeal Ca T3N1
Ref: Dhingra 5/e, p263-266,6/e p 252 Cummings Otolaryngology: Head and Neck Surgery, 5/e, vol-2, Chapter-99
Treatment of nasopharyngeal carcinoma
• State I and II Radiotherapy

• Stage III and IV Radiotherapy + chemotherapy (preferred) or radiotherapy alone in some cases.

Now let's see about treatment of other options.


• Supraglottic T N 3 0 Total laryngectomy w i t h neck dissection followed by radiotherapy.
GlotticT N,3
Total laryngectomy + neck d i s s e c t i o n ! radiotherapy (In some centers organ preserving
surgery followed by chemoradiation is preferred).
SublotticCaT N 3 0
Total laryngectomy followed by post-operative radiation.
21. Ans. is b i.e. Immediate Tracheostomy
Ref: Turner 10/e,p 178
Carcinoma larynx presenting w i t h stridor means it is subglottic laryngeal carcinoma .Ideally in such cases emergency laryngectomy
should be performed.
"In the case of a large subglottic tumour presenting with respiratory obstruction a case could be made for doing an emergency laryngec-
tomy."
But it is not given in the options:
• Intubation can not be done as g r o w t h is seen in subglottic area therefore tube can not be put.
• Planned tracheostomy can not be done as patient is suffering f r o m stridor, which is an emergency.Therefore we will have t o do
emergency tracheostomy. With the precaution that the area of cancer should be removed w i t h i n 72 hours.
180|_ SECTION IV Larynx

22. Ans. is b i.e. anterior commissure involvement Ref: Current otolaryngology 2/e pg-448-449

Type of Laryngectomy Parts Removed Indications Comment


Hemilaryngectomy Removal of one vertical half of Tumor with:
larynx. • Subglottic extension < 1 cm Vocal cord reconstruction is done
below the true vocal cord in this case by transposing a flap
• A mobile affected cord of strap muscle or microvascular
• Unilateral involvement free flap to provide bulk against
• No cartilage invasion which the remaining unaffected
• No extra laryngeal soft tissue of cord can vibrate.
involvement
• For tumors with a T stage of
T,,T orT by pre epiglottic
2 3

involvement only Vocal cords


are mobile

Supra glottic Removal of the supraglottis or the • For tumors with a T stage of It can be done by laser or by
upper part of larynx T,,T orT by pre epiglottic
2 3 external approach
involvement only Side effect - Aspiration. For
• Vocal cords are mobile this reason patients with
• Cartilage is not involved borderline pulmonary function
• Anterior commissure is not (FEV1 < 50%) who cannot
involved tolerate chronic aspiration are
• Patient has good pulmonary generally not considered good
status/reserve candidates for supraglottic
• The base of tongue is not laryngectomy
involved past the circumvallate
papillae
• The apex of pyriform is not
involved
• FEV1 is predicted to be > 50%
Supracricoid laryngectomy It is a newer surgical technique It is done in those in which cancer Pulmonary function and prior
in which voice is preserved. is located at the anterior glottis radiation candidacy criteria
The true vocal cords, supra glottis, including the commissure or for supraglottic laryngectomy
thyroid cartilage are removed and those with more extensive pre applies for supracricoid
cricoid and ary-tenoid cartilage epiglottic space involvement laryngectomy as well
are preserved

Near Total laryngectomy It is more e x t e n d e d partial It should not- be offered t o • Aspiration can occur
laryngectomy procedure in which patients whose radiation • Pt is dependent on
only one arytenoid is preserved treatment has failed, those with tracheostomy for breathing
and a tracheo-sophageal conduit poor pulmonary reserve or those
is constructed for speech. with tumor involvement below
the cricoid ring. Patients with
large T3 and T4 leison with one
uninvolved arytenoid or with U/L
transglottic tumors with cord
fixation are candidates for this
surgery.

Total laryngectomy Entire larynx + Thyroid + cricoid Indications: Most important constraint is
cartilages are removed along with • T, malignancy speech problem w h i c h can
some upper tracheal rings and • As a salvage surgery in be o b t a i n e d b y t r a c h e o
hyoid bone, if possible. recurrences following oesophageal speech
chemoradiation forT3 esion
• It is TOC in perichondrites larynx
CHAPTER 15 Tumor of Larynx

23. Ans. is probably 'b' i.e. Horizontal

Laryngeal carcinoma Treatment plan


. i ~
Suptraglottic Ca

Tl X
T2 T3 and T4
I
Total laryngectomy
Radiation _CO,

Lung function Post op-radiotherapy


I to

Good Poor

Supraglottic Radiotherapy
laryngectomy ± to the priomary

Glottic Ca

•T1 carcinoma T1
i
Radiotherapy or £
Cord mobile
1
C 0 2 laser Cord mobility


T1 Ca with extension
to ant-commissure I
RT
impaired
or
(primary + involvement of
neck nodes)
£ or arytenoid
RT Fronto
sral par
laryngectomy
T
Fails
No Conservative

T
laryngectomy

£ Failure
Total laryngectomy (TL)
; T1 C a with extension to arytenoid
TL +
• same as above but surgery
is preferred •

Subqlottic Ca—I
[ '
j T and T ^lesioons
1 2 T and Ti
3

Radiotherapy TL + post op. RT


(Radiation portal
should include
upper mediatinum)

In t h e P a t i e n t

• Involvement of unilateral false cord, aryepiglottic folds and arytenoids with mobile cord suggest supraglottic cancer in T2 stage
(morem than one subsites of supraglottis are involved).
• ForT2 stage voice conservative surgery should be done. Supraglottis is excised by partial horizontal laryngectomy.
182|_ SECTION IV Larynx

Vertical hemilaryngectomy means excission of one half of the larynx on one side, i.e., vertical half is removed which include
vertical half of supraglottis, glottis and subglottis.
It is indicated for specificT and T glottic cancer
t 2

Horizontal hemilaryngectomy is the excision of supraglottis only sparing true vocal cords and arytenoids also k n o w n as
supraglottic laryngectomy.
It is indicated for specificT1 andT2 supraglottic cancers which d o n o t involve true vocal cord.
So, it is quite obvious, in supraglottic cancer horizontal hemilaryngectomy should be done t o remove supraglottis.

The most significant problem with partial laryngectomies (horizontal/vertical) is aspiration and subsequent pneumonia therefore patients with good
pulmonary reserve should only be selected.

24. Ans. is d i.e. He should first undergo laryngectomy and then post-operative radiotherapy
Ref. Dhingra 5/e pg-328,330,331; 6/e 310-311
Perichondritis of thyroid cartilage in a patient of Ca larynx suggests invasion of thyroid cartilage i.e. stage T4.
StageT4 lesions glottic cancer are managed by total laryngectomy w i t h neck dissection for clinically positive nodes and post opera-
tive radiotherapy if nodes are not palpable.
Indication of Total laryngectomy in Ca larynx - Current Otolaryngology 2/e pg-449; Dhingra 5/e, pg-330,6/e p 310
T lesions (i.e. w i t h cord fixed) not amenable t o chemoradiation or partial laryngectomy procedures
3

All T lesions
4

Invasion of thyroid or cricoid cartilage


Bilateral arytenoid cartilage involvement
Lesions of posterior commissure
Failure after radiotherapy or conservation surgery
Transglottic cancers i.e. tumors involving supraglottis and glottis across the ventricle, causing fixation o f t h e vocal cord.

Remember:Total laryngectomy is contraindicated in patients w i t h distant metastasis.

25. Ans. is a i.e. External beam radiotherapy Ref: Current otolaryngology 2/e pg-445,450,3/e, pg-469-470
As I have said earlier-Treatment for stage I of cancer larynx (glottic cancer) is either microlaryngoscopic surgery or Radiotherapy.
Since micro laryngoscopic surgery is not given we will go for Radiotherapy. Now the question arises which type of radiotherapoy
is used.

External B e a n Radiation or Brachytherapy

"External bean radiation is most often used to treat laryngeal and hypopharyngeal cancer."
"Brachytherapy is rarely used to treat laryngeal or hypopharyngeal cancer." -Oxford Basic referance
"Radiation given as the primary treatment for larynx cancer or as an adjuvant treatment after surgery is most often done using an external
beam technique, a dose of6000-7000 cGyis admistered to the primary site." -Current otolaryngology 3/e, pg-469-470
26. Ans. is a, d and e i.e. Glottic Ca is the most common; T tumor is best treated by radiotherapy; Smoking predisposes
1

Ref: current otolaryngology 2/e pg-440,441, Dhingra 5/e pg-326,327,329-330; 6/e p 308 onwarrds

Lets see each Option Separately

• Option a - Glottic CA is most c o m m o n is correct


Correct -
Incidence of larynx cancer by site -

Suprglottic 40%
Glottic 59%
Subglottic 1%

• Option b - Supraglottic Ca has best prognosis Incorrect....


• Supraglottic cancers are often silent and their only manifestation is presence of neck nodes which is a very late feature. Hence
it does not have a good prognosis. (Best prongosis is w i t h glottic cancer)
• Option c - Lymphatic spread is the M/C in subglottic CA
• Incorrect
• Lymphatic spread is more c o m m o n in supraglottic CA as it has a rich lymphatic supply.
Option d - T t u m o r are best treated by radiotherapy
CHAPTER 15 Tumor of Larynx J 183
Correct
T, tumors are best treated by micro laryngoscopic surgery/ radiotherapy
• Option e - Smoking predisposes - correct
Cigarette smoking and alcohol are 2 main predisposing factors for CA larynx
27. Ans. is d i.e. Endoscopic removal
Ref. Current otolaryngology 2/e d pg-444,3/e, p 463 Ref. Scotts Brown 7/ed vol-2 pg-2604 - Table -194.3 Turner 10/e, p 169

Verrucous Carcinoma

Verrcous carcinoma makes up only 1 - 2 % of laryngeal carcinomas.


The larynx is the second most c o m m o n site of occurence in the head and neck after the oral cavity.
Most c o m m o n site of involvement is vocal cord.

Grossly, verrucous carcinoma appears as a fungating, papillomatous, grayish w h i t e neoplasm.
Microscopically, it is well differentiated squamous cell carcinoma w i t h minimal cytological atypis.
It has low metastatic potential
Hoarseness is the most c o m m o n presented symptom. Pain and dysphagia may occur b u t are less c o m m o n .
Treatment o f most verrucous tumors is primary surgery. Endoscopic laser surgery is appropriate as the t u m o r is less aggressie
than usual squamous cell carcinoma.
28. Ans. is a i.e. Opening the larynx in midline Ref. Stedman dictionary, p 937
Laryngofissure: Opening the larynx in midline.
29. Ans. All are correct Ref: Scott-Brown's Otolaryngology 7/e, vol-2 pg- 2617,2618
Loss of functioning larynx causes problems in speech, swallowing, coughing, altered appearance, lifting, weight, laughing, crying,
smelling, tasting and even kissing.
30 Ans. is b i.e. MRI Ref: Ref. Scott-Brown's Otolaryngology 7/e, vol-2 pg- 2607
31 Ans. is b i.e. C 0 Laser
2 Ref. Dhingra 5< /edp 362,6/e p 357
h

Co laser is used in laryngeal surgery to excise vocal nodules, polyps, cysts, granulomas or juvenile laryngeal papilloma. Also used
2

in case of leukoplakia,^ lesion of vocal cord or localized leisions of supraepiglottis and infraglottis.

ALSO KNOW

• Co laser has wavelength 10,400nm


2

• It is the w o r k horse laser and has been used widely in ENT


• It can cut pericisely (0.3mm percision), coagulate bleeders and vaporise tissues

• Besides laryngeal surgery it is used in oropharyngeal surgery t o excise benign or malignant lesions and in plastic surgery

EXTRA EDGE

Use in ENT Comment


Argon laser • Used to treat port wine stain, hemangioma Lies in the visible spectrum of light
and telangiectasia Wave length 485-514 nm (blue green colour)
• Used to create hole in stapes footplate Easily transmitted through clear fluid eg. cornea, lens, vitreous humor
Absorbed by Hemoglobin
KTP Laser • Stapes surgery • Lies in the visible spectrum of light
• Endoscopic sinus surgery to remove polys or • Wavelength 532 nm
inverted papiltomas and vascular lesions
• Micro laryngeal surgery
• To remove tracheo bronchial leisons through
bronchoscope
Nd yad laser For debulking tracheo bronchial and Wavelength 1064 nm (lies in infra red zone of electro magnetic
oesophageal leisons for palliation, spectrum)
hereditary hemorrhagic telangiectasia and
turbinectomy

Diode laser Turbirate reduction, laser assisted Wavelength 600-1000 nm


stapedectomy and mucosa intact tonsillar
ablation

- -
184|^ SECTION IV Larynx

Note:- Gas preferred in laser surgery-is enflurane . 0

0 concentration in inhaled gases should not be more than 4 0 % .


2

Donot use N 0 2

32. Ans. is a, b, d and e i.e. Poor pulmonary reserve, Tumor involving pyriform sinus, Vocal cord fixation, Cricoid cartilage
extension Ref. Dhingra 5 /308 ,h

Ref: Current otolaryngology 2/ed pg-447-448; P.L Dhingra 5 /ed p 331; Logan and Turner 7 0 /ed p 174; Ballenger otolaryngology
th th
and
Head-Neck 16' /edp 1285h

Supra glottic laryngectomy: Removal o f t h e supraglottis or upper part of larynx.


It should be done if following conditions are fulfilled.
• For tumors w i t h a T stage of T ,T o r T by pre epiglottic space involvement only (Thus involvement of pre epiglottic space is not
2 3

a contraindication for supraglottic larynygectomy).


• Vocal cords are mobile
• Cartilage is not involved (which includes cricoid cartilage so o p t i o n e is correct)
• Anterior commissure is not involved
• Patient has good pulmonary reserve (i.e. Poor pulmonary reserve is a contraindication)
Base of the tongue is not involved past the circumvallate papillae
• The
The apex
apex oofftthhee pyriform
pyriform sinus
sinus isisinot involved, (i.e. involvement of pyriform sinus is a contraindication)
The FEV, is predicted t o be > 50)
So if above criteria are not filled it is a contraindication of supraglottic laryngectomy. Answer is further supported by following
lines o f : Bellinger
"Supraglottic laryngectomy should not be attempted if there is vocal cord fixation, extensive involvement of pyriform sinus, thyroidsor
cricoid cartilage invasion or extensive involvement of base of tongue (to or beyond circumvallate papilla)"-Ballenger otolaryngology
and Head and Neck Surgery 16th/ed pg-1285

ALSO KNOW

Supraglottic laryngectomy can be performed endoscopically using a C 0 laser or w i t h a standard external approach.
2

33. Ans. is a i.e. HPV Ref. Dhingra 6/e p 305


Already explained
34. Ans. is a i.e. tracheostomy Ref. Logan Turner 10/e p 178
During laryngectomy, airway of a patient is maintained by tracheostomy.


16. Anatomy of Ear 24. Otosclerosis
17. Physiology of Ear and Hearing 25. Facial nerve and its lesions
18. Assessment of Hearing Loss 26. Lesion of Cerebellopalatine
19. Hearing Loss Angle and Acoustic
20. Assessment of Vestibular Neuroma
Function 27. Glomus Tumor and other
21. Diseases of External Ear Tumors of the Ear
22. Diseases of Middle Ear 28. Rehabilitative Methods
23. Meniere's Disease 29. Miscellaneous
CHAPTER

Anatomy of Ear

Ear can be divided into three parts: I. External ear


External auditory meatus
II. Middle ear
III. Inner ear Scaphoid fossa
Triangular fossa
EXTERNAL EAR Cymba conchae
Auricular (Darwin's) tubercle
It consists o f (A) Pinna (B) External a u d i t o r y canal a n d (C)
Tympanic membrane Concha
Helix

| PINNA/AURICLE Antihelix
Tragus
• It is made of single yellow elastic cartilage except at the lobule, Intertragic notch
where it is absent Antitragus
• Its lateral surface has characteristic prominences and depres-
sions (as shown in figure) which are different in every individual
even among identical twins.This unique pattern is comparable Lobule
t o fingerprints and can allow for identification o f persons.
• The cartilage o f pinna is continuous w i t h t h e cartilage o f ex-
Fig. 1 6 . 1 : External features of auricle
ternal auditory canal.
Coutesy: Text book of Diseases of Ear, Nose and
• The cartilage is covered w i t h skin which is closely attached o n Throat, Mohan Bansal. Jaypee Brothers, p 3
lateral surface and slightly loose on medial surface. 0

• i n n e r v a t i o n of the pinna: (Fig. 16.2)


• The cartilage itself is avascular and derives its supply of nutri-
ents f r o m the perichondrium covering it.
Medial surface
Clinical importance - stripping o f t h e perichondrium f r o m the
cartilage as occurs following injuries that cause hematoma can 1. Auriculotemporal nerve 1. Lesser occipital n e r v e —
lead t o cartilage necrosis and so-called 'boxers ear'. supplies upper part
• Various landmarks on the pinna: see figure 16.1 2. Greater auricular nerve 2. Most of the medial surface is
• Incisura terminalis: Area between t h e crus o f the helix and supplied by great auricular
tragus. nerve (MAHE 07)
It is devoid o f cartilage 3. Auricular branch of vagus 3. Auricular branch of vagus
Clinical importance: An incision made in this area does also called as Arnold nerve
n o t cut t h r o u g h the cartilage and is used for endural ap-
4. Facial nerve (VII) 4. Facial nerve
proach in surgery.
• Pinna has 3 extrinsic muscle: 1. Auricularis anterior, 2. Auricu- Lymphatic Drainage:
laris superior and (3) Auricularis posterior.Theseareall attached From posterior surface -• lymph node at mastoid t i p
t o epicranial aponeurosis and supplied by the facial nerve From tragus and upper part o f anterior surface - Preau-
• Intrinsic muscles are 6 in number and are small, inconsistent ricular nodes
and w i t h o u t any useful information Rest of auricle —> upper deep cervical nodes
188[ SECTION V Ear

Flow chart 16.1: Relations of middle external auditory canal

I Anterior
Superior Z2C
• Temporomandibular joint
Superficial temporal A
Middle cranial fossa, and vein
temporal lobe
1
Auriculotemporal N
1
Parotid gland
• Preauricular lymph node

Medial
Middle
ear

Mastoid
K • Jugular bulb
Outside
world
Lateral

• Carotid
• Facial nerve
Posterior • Styloid process
• Parotid gland
• Digastric muscle

Fig. 16.2: Nervesupplyofrightpinna. (A) Lateral surface; (B) Medial surface



Interior
Coutesy: Text book of Diseases of Ear, Nose and Throat, ° N e r v e supply:
Mohan Bansal. Jaypee Brothers, p 4
Anterior wall and roof: Auriculotemporal nerve
Floor and posterior wall: Vagus (arnold nerve))

11
EXTERNAL AUDITORY CANAL/EXTERNAL Posterior wall also receives innervation f r o m : Facial nerve
ACOUSTIC MEATUS {Importance -Hypoesthesia o f t h e posterior meatal wall
is seen in case of facial nerve injury, known as Hitzel-
Length : 24-25 m m Q berger's s i g n )
Parts : Lateral/outer 1/3 Cartilaginous 0

Medial/inner 2/3 : Osseous0 | T Y M P A N I C M E M B R A N E (FIG. 16.3)


Shape : 'S'- shaped curve
It is the partition between external acoustic meatus and middle
External Auditory Canal develops from = First brachial cleft/groove 0

ear, i.e. it lies at medial end of external auditory meatus


Tympanic membrane is 9-10 m m tall, 8-9 m m w i d e and 0.1
Cartilaginous Part
m m thick and is positioned at angle of 55° t o floor.
Forms the outer/lateral 1/3 (8 mm) of external auditory canal It is shiny and pearly g r a y in color.
0

• Has a fissure/deficiency - in the anterior part called as Fissures Normal tympanic membrane is mobile w i t h maximum mobility
of S a n t o r i n i t h r o u g h which parotid or superficial mastoid
0
being in the peripheral p a r t . 0

infection can appear in the canal and like vice versa.


It has 2 parts:
• Skin covering is thick and has ceruminous glands (modified
apocrine sweat glands ), pilosebaceous glands and hair.
0 Pars tensa Pars f l a c c i d a / S h r a p n e l l ' s
• Ceruminous and pilosebaceous glands secrete wax (mixture of membrane
cerumen, sebum and desquamated cells is wax). It forms most of tympanic Situated above the lateral process
• Since hair is confined t o cartilaginous part - furuncles are seen membrane of malleus between the notch
only in the outer third of canal. 0
of Rivinus and the anterior and
posterior malleal folds
Bony Part
Periphery is thickened to It is more mobile
• It forms inner two-thirds (16 m m ) of external auditory canal.
0
form a fibro-cartilaginous ring
• Skin lining the bony canal is thin and it is devoid of hair and called theannulus tympanicus
ceruminous glands . 0
This ring is deficient above in Prussack's space is a shallow
• 5 m m lateral t o tympanic membrane, bony meatus is narrow the form of a notch called the recess within the posterior
and called Isthmus (Applied - Foreign bodies get lodged in it notch of Rivinus part of pars flaccida
and are difficult t o remove). The central part is tented
• Foramen of H u s c h k e is a deficiency present in anteroinferior
0
inward at the level of tip of
part of bony canal in children up t o 4 years of age, p e r m i t t i n g malleus and is called as umbo
infection t o and f r o m t h e t e m p o r o mandibular joint. Cone of light is seen radiating
Blood supply: It is also supplied by External carotid artery. from tip of malleus [o the
Lymphatic drainage—follows the auricle periphery in the anteroinferior
Relationship of external auditory canal - see flowchart 16.1 quadrant. 0
CHAPTER 16 Anatomy of Ear J 189

Pars flaccida Nerve supply of Tympanic membrane


Malleal fold . (Shrapnell's
membrane) • Lateral/outer surface
- Posterior
Anterior half: Auriculotemporal nerve
- Anterior Malleus Posterior half: Vagus nerve
lateral process
• Medial/inner surface
Long process Tympanic branch of glossopharyngeal nerve (k/a Jacob-
(Handle) son's nerve)
• Auriculotemporal nerve (CN V3): It is a branch of mandib-ular
Pars tensa
division of trigeminal nerve and supplies anterior half of lateral
surface of TM.
• CN X (vagus nerve): Its auricular branch (Arnold's nerve)
supplies t o posterior half of lateral surface of TM.
• CNIX(glossopharyngealnerve): Its tympanic branch (Jacobson's
Umbo Cone of light
nerve) supplies to medial surface of tympanic membrane.

Fig. 16.3: Tympanic membrane showing attic, malleus handle, MIDDLE EAR CLEFT (FIG. 16.5)
umbo, cone of light and structures of middle ear seen t h r o u g h it
on otoscopy Ear cleft in the temporal bone, consists of tympanic cavity (middle
Coutesy: Text book of Diseases of Ear, Nose and ear), Eustachian tube and mastoid air cell system.
Throat, Mohan Bansal. Jaypee Brothers, p 5 Aditus ad antrum
Mastoid
L a y e r s o f T y m p a n i c M e m b r a n e (Fig. 1 6 . 4 )

• Outer - Epithelial
• Middle - Fibrous
• Inter - Mucosal c o n t i n u o u s - t h e middle ear mucosa

Mesotympanum
When a tympanic membrane perforation heals spontaneously, it heals Hypotympanum
in two laryers as it is often closed by squamous epithelium before
fibrous elements develop.
Fig. 16.5: Parts of middle ear cleft
Arterial supply: Vessels are present only in connective tissue layer Coutesy: Text book of Diseases of Ear, Nose and
o f t h e lamina propria. Throat, Mohan Bansal. Jaypee Brothers, p 6
Arteries supplying tympanic membrane are:
| TYMPANIC CAVITY (MIDDLE EAR)
Mnemonic—
M = Maxillary artery It is divided into:
A = Post auricular Artery • Mesotympanum
M = Middle meningeal branch Artery • Epitympanum
• Hypotympanum
Ring of fibrocartilage in
tympanic sulcus

• Sometimes the portion of middle ear around the tympanic orifice


ofthe Eustachian tube is called as protympanum.
Mucosal layer • Anterior tympanic recess or supratubal recess refers to a small
Outer layer compartment within epitympanum seen on CT imaging.
• Boundaries of middle ear.
Middle ear cavity
• Middle ear is like a six sided box with a: roof, floor, medial wall,
Handle of malleus lateral wall, anterior wall, posterior wall

External auditory canal


Roof
• Fibrous layer
Is formed by a thin plate of bone called tegmen t y m p a n i . 0

• It separates tympanic cavity from the middle cranial fossa. 0

• Tegmen tympani is formed both by petrous and squamous


Fig. 16.4: Three layers o f tympanic membrane part of temporal bone and the petrosquamous line
Coutesy: Text book of Diseases of Ear, Nose and • which does not close until adult life can provide a route of
Throat, Mohan Bansal. Jaypee Brothers, p 5 access for infection into the extradural space in children.
190[ SECTION V Ear

Mesotympanum Epitympanum/Attic Hypotympanum


Lies opposite the pars tensa Lies above pars tensa and medial to lateral Lies below the level of pars tensa
attic wall and Shrapnell's membrane
Narrowest part of middle ear (Transverse
0
It is widest part of tympanic cavity Transverse diam = 4 m m
diam = 2 mm) (transverse diam = 6 mm)
Seen on routine otoscopy Prussak space lies in epitympanum Contains numerous hypotympanic air
Contains stapes bone, malleus (neck, short Contains incus (body and short process) cells
process and umbo), incus (long process), head of malleus
oval window, round window, stapedius Cannot be seen on routine otoscopy
muscle, tensor, tympani, cochleariform
process

F l o o r or Jugular wall (Paries tegmentalis) The posterior wall

It is a t h i n plate of bone which separates tympanic cavity from the It lies close t o t h e mastoid air cells. It has the f o l l o w i n g main
jugular b u l b . 0 features:
• At t h e j u n c t i o n o f floor and medial wall is a small opening
w h i c h allows entry of tympanic branch of glossopharyngeal
Epitympanum
nerve into t h e middle ear.
(Attic)
Ossicles
Anterior wall or carotid wall
Malleus
• It is a thin plate of bone which separates the cavity from internal Incus
carotid artery. Tympanic-_^~~7£\ / \ Stapes
membrane ^ ^ f ^ Oval window
• From above downwards features seen on anterior wall are
between fk \) Promontory
Canal f o r t e n s o r t y m p a n i (canal containing tensor tympani external \ _ L _ . Mesotympanum
muscle which extends t o the medial wall t o f o r m a pulley and middle ear
called as processus cochleariformis). The cochleariformis
process, serves a useful landmarkand denotes the location
Hypotympanum
of anterior most part of horizontal segment of facial nerve.
Opening for eustachian tube
Internal carotid artery (carotid canal) Fig. 16.7: Parts o f middle ear seen on coronal section
Coutesy: Text book of Diseases of Ear, Nose and
Throat, Mohan Bansal. Jaypee Brothers, p 6

Tegmen antri
and tympani Malleus
Processus
cochleariformis
Tympanic
semicircular /

15 mm membrane
Facial nerve
-Canal for Anterior
Posterior tensor tympani
Labyrinth
Oval window Eustachian
Round window tube
Internal
Medial carotid artery
Promontary
Jugular
bulb Inferior

Fig. 16.8: Six boundaries of t y m p a n u m . Medial wall is seen


Fig. 16.6: Dimensions o f t y m p a n u m t h r o u g h the tympanic membrane
Coutesy: Text book of Diseases of Ear, Nose and Coutesy: Text book of Diseases of Ear, Nose and
Throat, Mohan Bansal. Jaypee Brothers, p Throat, Mohan Bansal. Jaypee Brothers, p 7
CHAPTER 16 Anatomy of Ear J 191
Chorda tympani
nerve Lateral
semicircular
canal
Facial recess
Processus
Facial nerve in
cochleariformis
its canal -
Facial n e r v e ^ - ^ ^
Oval window-
Sinus tympani
Posterior
Posterior Pyramid"
Ponticulus - Tympanic
Sinus tympani - plexus
Subiculum -
Round window-
Fig. 16.9: Facial recess and sinus tympani relations w i t h
facial nerve and pyramidal eminence
Coutesy: Text book of Diseases of Ear, Nose and
Throat, Mohan Bansal. Jaypee Brothers, p 7
• Aditus-an opening t h r o u g h which attic communicates w i t h
t h e mastoid a n t r u m Fig. 16.11: Medial wall of middle ear
Coutesy: Text book of Diseases of Ear, Nose and
• A bony projection called the pyramid which contains stape-
Throat, Mohan Bansal. Jaypee Brothers, p 8
dius muscle.
• Facial nerve runs in the posterior wall just behind the pyramid. Medial wall
Facial recess (Fig. 16.9) also called suprapyramidal recess is a
It separates the tympanic cavity f r o m internal ear. It is formed by
collection o f air cells lying lateral t o facial nerve. It is b o u n d e d
medially by external genu o f facial nerve, laterally by chorda labyrinth. The main features on medial wall are (Fig. 16.11):
t y m p a n i nerve, superiorly by fossa incudis (in which lies the short • A bulge called as promontory formed by basal turn of cochlea. 0

process of incus) and anterolateral^ by tympanic membrane. • Fenestra vestibuli (oval w i n d o w ) lies posterosuperior (behind
0

and above) t o the p r o m o n t o r y and opens into scala vestibuli.


It is occupied by f o o t p l a t e of stapes fixed byannularligament.
Its size on average is 3.25 m m long and 1.75 m m wide
In the intact canal wall mastoidectomy, middle ear is approached
(posterior tympanotomy or facial recess approach) through the facial • Fenestra cochleae (round w i n d o w ) lies posteroinferior t o the
recess without disturbing posterior meatal wall. (Fig. 16.10) p r o m o n t o r y and opens into scala t y m p a n i o f cochlea. It is
closed by secondary tympanic membrane.The round w i n d o w
Sinus t y m p a n i (Infrapyramidal t y m p a n i ) : This deep recess lies
isss closest t o ampulla of posterior semicircular canal. Round
medial to the pyramid. It is bounded by the subiculum below and
w i n d o w is a triangular opening.
ponticulus above.
• Prominence of facial nerve canal (k/a Fallopian canal) lies above
Anterior
the fenestra vestibuli curving downward into posterior wall of
Short process of incus
. fio in fossa incudis middle ear.
Incudostapedial joint * • Anterior t o oval w i n d o w lies a hook-like projection called the
processus cochleariformis for t e n d o n of tensor t y m p a n i .
0 0

Chorda tympani nerve


• The cochleariform process marks the level o f t h e genu o f t h e
facial nerve which isan important landmark for surgery o f t h e
facial nerve.
Inferior
| EXTRA EDGE
Round window
Stapedius • The round w i n d o w opening is separated from the oval w i n d o w
muscle emerging
from pyramid opening by a bony ridge called the subiculum.
• The ponticulus - is another bony ridge below oval window.
Posterior • Medial t o the pyramid is a deep recess called as sinus tym-
pani which is bounded below by subiculum and above by
Fig. 16.10: Posterior tympanotomy. Structures of middle ear
ponticulus. It is the most in accessible site in the middle ear
seen t h r o u g h the opening of facial recess
and mastoid. Its importance is that cholesteatoma which has
Coutesy: Text book of Diseases of Ear, Nose and
extended upto it is difficult t o eradicate.
Throat, Mohan Bansal. Jaypee Brothers, p 7
192^ SECTION V Ear

The lateral wall (i) A n t e r i o r t y m p a n i c a r t e r y , (ii) I n f e r i o r t y m p a n i c a r t e r y ,


(iii) Stylomastoid artery
Is f o r m e d mainly by the tympanic membrane and t o some extent
by bony outer attic wall called the scutum".
Lymphatic drainage
Nerve supply of middle ear
Middle ear—> Retropharyngeal and Parotid nodes
Is by Tympanic Plexus. Eustachian t u b e —> Retropharyngeal group
• Tympanic plexus is formed by:
Tympanic branch of IX nerve (Jacobson nerve)
Caroticotympanic nerves from the sympathetic plexus Contents of T y m p a n i c Cavity
• They f o r m a plexus o n t h e p r o m o n t o r y and p r o v i d e • The tympanic cavity contains the
branches t o the t y m p a n i c cavity, Eustachian t u b e and • Ossicles
mastoid a n t r u m and air cells. • Muscles viz.
- Tensor tympani and stapedius
Blood supply • Chorda tympani
• Tympanic plexus
• Arteries supplying t h e walls and contents of the tympanic
cavity arise f r o m both the internal and external carotid system.
Arteries involved are:

A U D I T O R Y O S S I C L E S (FIG. 16.12)

These are malleus, incus and stapes (MIS)


Malleus Incus Stapes

• It is shaped like a mallet • It is shaped like an anvil • It is the shortest bone of the body
• It is placed most laterally • It is the largest of the three ossicles • It is shaped like a stirrupaz
• It is 7.5-9 m m long • It is placed medially t o malleus • It is placed most medially
• It comprises of head, neck, anterior • It has body, short process, long • Footplate of stapes is held on the oval w i n d o w
process, lateral process, manubrium process and lenticular process by annular ligament
and u m b o • Stapes consists of a capitulum, t w o crura
and f o o t plate
Also know:
Lenticular process is sometimes k/a, The average dimensions of foot plate are
-

the fourth ossicle as it is a sesamoid bone 3 m m long and 1.4 m m wide

D e v e l o p m e n t of Ossicles Short

• Maleus and incus develop mainly f r o m first brachial arch


(Meckels cartilage)
• Stapes develops mainly f r o m second brachial arch except the
f o o t plate w h i c h along w i t h annular ligament is derived f r o m
t h e otic capsule.
• Ossicles ossify by f o u r t h m o n t h of intrauterine life (first bones
in the body t o do so).

Joints o f t h e Ossicles

a. The incudomalleolarjoint Manubrium


Saddle j o i n t (handle)
b. Incudostapedial joint
Ball and socket j o i n t Footplate
Both o f t h e m are synovial joints.
Fig. 16.12: Middle ear ossicles
F u n c t i o n of Ossicle Coutesy: Text book of Diseases of Ear, Nose and
Throat, Mohan Bansal. Jaypee Brothers, p 8
• Ossicles conduct sound energy f r o m the tympanic membrane
t o oval w i n d o w and then t o inner ear fluid.
CHAPTER 16 Anatomy of Ear J 193

M u s c l e s o f T y m p a n i c C a v i t y : Tympanic Cavity has Two Muscles

Muscle Origin Insertion Innervation Function


Tensor tympani Cartilaginous pharyngo Upper part of handle of Contraction pulls handle
t y m p a n i c t u b e , greater malleus of malleus medially,
wing ofsphenoid.itsown tensing tympanic
bony canal membrane to reduce
the force of vibrations in
response to loud noise

Stapedius develops from A t t a c h e d t o inside o f Neck of stapes Branch of facial nerve Contraction usually
2nd Arch pyramidal eminence in response to loud
noises, pulls the stapes
posteriorly and prevents
excessive osscillation

| MASTOID ANTRUM
^^^^j^^^j^^^sjj^^^^M^i^S^^^M^^^fe^aj^^a MacEwen Triangle Trautman triangle
Types
Boundaries Boundaries
Sclerotic (20%) Above - supramastoid
Diploic (mixed) crest Posterior - sigmoid sinus
• It is an air sinus in the petrous temporal bone. Anteroinferior -
• Its upper anterior wall has the opening of aditus, while medial posterosuperior Anterior - bony labyrinth
wall is related t o posterior semicircular canal (SCC). margin of external
• Posteriorly lies the sigmoid sinus. auditory canal Superior - superior petrosal sinus
• The posterior belly of digastric muscle forms a groove in the Posterior - Tangent Importance: Infection into the posterior
base of mastoid bone. The corresponding ridge inside the drawn from zygomatic cranial fossa can spread through this
mastoid lies lateral not only t o sigmoid sinus but also t o facial arch triangle and can be approached by
nerve and is a useful landmark. removing the bone in between the triangle.
• The roof is f o r m e d by tegmen antri separating it f r o m middle I m p o r t a n c e : Spine
cranial fossa and temporal lobe o f b r a i n * of Henle lies in the
• Anteroinferior is the descending part of facial nerve canal (or triangle.
Fallopian canal). It is an i m p o r t a n t
• Lateral wall is formed by squamous temporal bone and is easily surgical landmark
palpable behind the pinna. for locating mastoid
• Mastoid develops f r o m squamous and petrous bone. antrum.
The mastoid a n t r u m but n o t the air cells are well developed at
MacEwan's triangle
birth. Pneumatization begins in the first year and is complete by
4 t o 6 years of age.

Korner's septum - Korner's septum is persistence of


c \
petrosquamous suture in the form of a bony plate which
separates superficial squamous cells from the deep petrosal cells. Spine of H e n l e — V
Korner's septum is surgically important as it may cause difficulty
in locating the antrum and the deeper cells, and thus lead to
incomplete removal of disease at mastoidectomy. Mastoid
antrum cannot be reached unless the Korner's septum has been
removed. j
^ "

Fig. 16.13: MacEwan's triangle

-
194T SECTION V Ear

Middle ear u j£°/)y • It serves the most i m p o r t a n t function o f hearing and e q u i -


librium.
• Parts: A. Bony labyrinth, B. Membranous labyrinth.

| B O N Y L A B Y R I N T H (FIG. 16.15)

• It lies in the temporal bone


• It consists of vestibule, the semicircular canals and the cochlea
which are filled with perilymph 0

Nasopharynx
Fig. 16.14: Right Eustachian t u b e Vestibule

• Central portion o f t h e bony labyrinth.


EUSTACHIAN TUBE • Posterosuperior wall: Has'5'openings of the semicircular canals.
• Medial wall:
It is a channel connecting the tympanic cavity with the nasopharynx.
Spherical Elliptical Pending of Vestibular
(Fig. 16.14)
recess recess aquaductof rest and
• It is also called pharyngotympanic tube. It is lined by Ciliated
vestibule cochlear
columnar epithelium.
• It helps t o equalize pressure on both sides of tympanic m e m - • For the macula • Forthe • Carries • Forthe
brane. of saccule macula of endolymphatic cochlear
• Length o f Eustachian t u b e is 36 m m (reached by the age o f • Carries fibers utricle duct nerve
7 years). of inferior
• Lateral third (i.e. 12 mm) is bony. In the lateral wall lies the oval w i n d o w (Fenestra vestibule)
• Medial 2/3 (i.e. 24 mm) is fibrocartilaginous.
• In adults it is placed at an angle of 45° with saggital plane, while Semicircular Canals
in infants it is short, wide and placed horizontally.
They are three in number, the lateral, posterior and superior, and
So in infants infections of middle ear are more c o m m o n .
lie at right angles t o each other.
• Muscles of Eustachian tube are tensor p a l a t i (dilator tube
0

• Ampulla: One end of each canal dilates t o f o r m the ampulla,


is a part of it) supplied by branch o f mandibular n e r v e and 0

which contains the vestibular sensory epithelium and opens


levator p a l a t i supplied by pharyngeal plexus t h r o u g h Xlth
0

independantly in vestibule.
cranial nerve. 0

• Crus commune: Formed by the non-ampullated ends o f the


• Arterial supply is through branches from ascending pharyngeal
superior and posterior semicircular canaal. So the 3 semicircular
artery, middle meningeal artery and artery of pterygoid canal
canals open in vestibule by "5"openings.
(both branches of maxillary artery).
• Venous drainage is t o the pterygoid venous plexus. Cochlea (Bony Cochlea)
• Nerve supply is by tympanic plexus.
• Has approximately two- and- one half t u r n s . 0

• Coils turn about a central cone called modiolus."


INNER EAR Stapes covering ovar window
It consists of a bony labyrinth contained w i t h i n the petrous
temporal bone along w i t h the membranous labyrinth.

Spherical recess -, Elliptical recess for utricle


for saccule
Posterior
Cochlea
Semicircular canals
Scala vestibuli
Superior
Osseous spiral lamina
(anterior)
Scala Posterior
tympani Lateral
(horizontal)
Helicotrema
Anterior Crus commune

Ampullated ends
Opening for
Cochlear aqueduct endolymphatic duct Fig. 16.16: Cochlea: Peri- and endolymphatic systems relations
w i t h cerebrospinal fluid (CSF)
Fig. 16.15: Bony labyrinth of left side. Coutesy: Text book ofDiseasses of Ear, Nose and
External features seen f r o m lateral side Throat, Mohan Bansal. Jaypee Brothers, p 14
CHAPTER 16 Anatomy of Ear

• Cochlea converts mechanical soundwaves to electrical signal • The length of basilar membrane increases as we proceed f r o m
which can be transmitted t o brain. This function is primarily the basal coil t o the apical coil. So higher frequencies o f sound
performed by cochlea hair cells. are heard at the basal coil, while lower ones are heard at the
• The modiolus houses spiral ganglion cells destined t o enervate apical coil.
cochlea hair cells, in an area called as Rosenthal canal.
• Arising f r o m the modiolus is a thin shelf of bone which spirals Utricle a n d Saccule
upward w i t h i n t h e lumen of the cochlea as the bony spiral • The utricle lies in the posterior part of bony vestibule.
lamina.
Spira lamina divides the cochlear canal into upper scala
vestibuli and lower scale tympani.The scala vestibuli and
t y m p a n i scala are continous w i t h each other t h r o u g h
helicotrema at the apex of cochlea. (Fig. 16.16)
Scala vestibuli is closed by the footplate of stapes, which
separates it f r o m the air-filled middle ear.
The scala t y m p a n i is closed by secondary t y m p a n i c
membrane.
Aqueduct of cochlea connects the scala tympani w i t h the
subarachnoid space.
Spiral lamina gives attachment t o the basilar membrane.

| M E M B R A N O U S L A B Y R I N T H (FIG. 16.17)

• It lies w i t h i n t h e osseus/bony l a b y r i n t h and is filled w i t h Fig. 16.18: Structure o f cochlear canal after its cut section
endolymphatic f l u i d . " Courtesy: Text book of Diseases of Ear, Nose and Throat, Mohan
• It is separated f r o m the bony labyrinth by perilymphatic f l u i d . 0
Bansal. Jaypee Brothers, p 15
• It consists of cochlear duct, utricle, saccule, semicircular ducts,
• It receives the five openings o f t h e three semicircular ducts.
endolymphatic d u c t and sac.
• It is connected t o the saccule through utriculosaccular ducts. 0

• The sensory epithelium o f t h e utricle is called the macula and


Cochlear Duct (Membranous Cochlea)
is concerned w i t h linear acceleration and deceleration.
0 0

• Also called membranous cochlea or the scala media. It is a


0 0
• The saccule also lies in the bony vestibule.
blind coiled tube. • Its sensory e p i t h e l i u m is also called the macula.
0
Its exact
• It appears triangular o n cross section and has three walls function is not k n o w n . It probably also responds t o linear ac-
f o r m e d by celeration and deceleration.
0 0

The basilar membrane, which supports the organ of c o r t i 0

The Reissner's memebrane which separates it f r o m the Semicircular Ducts


scala vestibuli (Fig. 16.18)
0

• They are three in number and correspond exactly t o the three


The stria vascularis, which contains vascular epithelium bony canals.
and is concerned w i t h secretion of e n d o l y m p h . 0

• They open in the utricle. The ampullated end of each duct


• Cochlear duct is connected t o the saccule by ductus reuniens 0
contains a thickened ridge of neuroepithelium called crista
ampullaris which responds t o angular acceleration.
0 0

Endolymphatic Duct and Sac

Endolymphatic duct is formed by the union of t w o ducts, one each


from the saccule and the utricle. It passes t h r o u g h the vestibular
0

aqueducts. Its terminal part is dilated t o f o r m endolymphatic sac


which lies under the dura on the posterior surface o f t h e petrous
bone.

Inner Ear Fluids and their Circulation

• There are t w o main fluids in the inner ear, p e r i l y m p h and


endolymph.
• Perilymph resembles extracellularfluid and is rich in Na ions . 0

It fills the space between the b o n y and the memebranous


0

Fig. 16.17: Membranous labyrinth of left side: External features labyrinth. It communicates w i t h CSF t h r o u g h the aqueduct
0

Coutesy: Text book of Diseases of Ear, Nose and of cochlea which opens into the scala tympani near the round
0

Throat, Mohan Bansal. Jaypee Brothers, p 15 window.


96 T SECTION V Ear

Endolymph fills the entire membranous l a b y r i n t h and re-


0
B l o o d S u p p l y of L a b y r i n t h
sembles intracellular f l u i d , being rich in K ions . It is secreted
0 0

• Blood supply of labyrinth is through labyrinthine a r t e r y which


0

by the secretory cells o f t h e stria vascularis o f t h e cochlea and


13

is a branch of anteroinferior cerebellar a r t e r y b u t may some-


0

by the dark cells (present in the utricle and near the ampullated times arise f r o m basilar artery.
ends of semicircular ducts). • It divides in the l a b y r i n t h - a s

Labyrinthine Artery (from anteroinferior cerebellar artery)

r
Common cochlear
1
Anterior-vestibular artery |
I

r
Vestibulocochlear artery Main cochlear artery
(to utricle and lateral &
superior canals)
(to cochlea, 8 0 % )

Cochlear anch Posterior vestibular artery


(to cochlear, 2 0 % ) (to saccule & posterior canal)

Venous Drainage Tragus develops f r o m the first branchial arch. The remaining
pinna develops second arch.
• Itisthroughthreeveins namely internal auditory, vein of cochlear
By the 20th week, pinna attains adult shape.
aq ueduct and vein of vestibular aqueduct which ultimately drain
into inferior petrosal sinus and lateral venous sinus.
Applied Anatomy
• Preauricular sinus: Due to defective fusion between 1 st and 2nd
Blood supply to the inner ear is independant of blood supply to arch hence it is situated between tragus and rest of pinna
middle ear and bony otic capsule, and there is no cross circulation Formed when 1 st and 2nd hillocks fail to fuse. Opening is found
between the two. in front of the ascending limb ofthe helix.
Blood supply to cochlea and vestibular labyrinth is segmental, • Anotia is complete absence of pinna and usually forms a part of
therefore, independent ischemic damage can occur to these the first arch syndrome
organs causing either cochlear or vestibular symptoms. • Microtia: It is developmental anomaly where size of pinna is
small.
1 DEVELOPEMENT OF EAR
External Auditory Canal
Pinna
In the sixth week of embryonic life, six tubercles (Hillocks of • External auditory canal (EAC) develops f r o m the firstbranchial
His) appear around the first branchial cleft .They progressively cleft.
g r o w and coalesce and f o r m the auricle. • External ear canal gets fully formed by the 28th week.

Applied Anatomy
Atresia of canal: The recanalization of meatal plug, which
begins from the deeper part near the tympanic membrane and
progresses outwards, forms the epithelial lining of the bony
meatus. This is the reason why deeper meatus is sometimes
developed while there is atresia of canal in the outer part.

Tympanic Membrane

It develops f r o m all the three germinal layers.


• Ectoderm: Outer epithelial layer is formed by t h e ectoderm.
• M e s o d e r m : The m i d d l e f i b r o u s layer develops f r o m t h e
mesoderm.

Fig. 16.19: Development of pinna (A) f r o m six hillocks o f His (B) • Endoderm: Inner mucosal layer is formed by the endoderm.
around the firstbranchial cleft (1 f r o m firstand 2-6 f r o m second
branchial arch) Middle Ear
Coutesy: Text book of Diseases of Ear, Nose and
• Endoderm of Tubotympanic Recess: The eustachian tube,
Throat, Mohan Bansal. Jaypee Brothers, p 19
tympanic cavity, attic, antrum and mastoid air cells are derived
J

CHAPTER 16 Anatomy of Ear J 197


f r o m the endoderm of tubotympanic recess which arises f r o m • The cochlea develops by 20 weeks of gestation and the fetus
the first and partly f r o m the second pharyngeal pouches. can hear in the w o m b o f the mother. The great Indian epic
of Mahabharata, which was written thousands of years ago,
mentions that Abhimanyu son of great warrior Arjun while in
11 his mother's w o m b heard conversation (regarding t h e art of
battle ground) of his mother and father.

Extra E d g e

• Structures of ear fully formed at birth:


Dhingra 4th/ed p 403; 5th/ed p 462; point 106.
Middlleear 0

External
auditory - Malleus"
canal (1st
branchial Incus 0

cleft) Stapes 0

Labyrinth 0

- Cochlea 0

Area of adult tympanic membrane is 90 m m 2 0


, of w h i c h only
55 m m is functional. Area of stapes footplate is 3.2 m m . Area
2 2

Fig. 16.20: Development of external and middle ears. 1 (Meckel's


cartilage) and 2 (Reichert's cartilage) branchial arches ratio (or hydraulic ratio) is 17:1. According t o other workers,
Coutesy: Text book of Diseases of Ear, Nose and functional area is 45 m m and area ratio is 14:1.
2

Throat, Mohan Bansal. Jaypee Brothers, p 20 Mastoid a n t r u m lies 12-15 m m deep f r o m t h e surface o f
suprameatal triangle in an adult. The thickness o f the bone
• First Branchial Arch: Malleus and incus develops f r o m meso-
overlying the antrum is only 2 m m at birth and then increases
d e r m o f t h e first arch.
«• Second Branchial Arch: The stapes superstructures develop at the rate of 1 m m per year.
from the second arch. Mastoid t i p does not develop till 2 years; hence postaural inci-
• Otic Capsule: The stapes footplate and annular ligament are sion t o open the mastoid before this age needs t o be modified
derived f r o m t h e otic capsule. See chapter for the details of t o avoid injury t o the facial nerve.
Branchial Apparatus. Vertical and anteroposterior dimensions of middle ear are 15
m m each while transverse dimension is 2 m m at mesotympa-
Inner Ear
n u m , 6 m m above at the epitympanum and 4 m m below in the
• Development o f the inner ear, which begins in third week of h y p o t y m p a n u m . Thus, middle ear is the narrowest between
fetal life, is complete by the 16th week. the u m b o and promontory.
• Auditory Placode: The auditory placode, which is thickened
Tympanic membrane develops f r o m all the three germinal
ectoderm of hind brain, gets invaginated and forms auditory
layers : ectoderm (outer epithelial layer) mesoderm (middle
0

vesicle (otocyst).
fibrous layer) and endoderm (inner mucosal layer).
• Auditory Vesicle: The auditory vesicle differentiates into e n -
dolymphatic d u c t and sac, utricle, semicircular ducts, saccule Boundaries of facial recess are facial nerve medially, chorda
and cochlea. tympanic (laterally) and fossa incudis (above).
• Development o f pars superior (semicircular canals and utricle) Eddy currents in the external auditory meatus do not allow
0

takes place earlier than pars inferior (saccule and cochlea). The water t o reach TM while swimming.
pars superior is phylogenetically older part of labyrinth. Organ o f corti is filled w i t h cortilymph.


198T SECTION V Ear

QUESTIONS

1. Ceruminous glands present in the ear are: 13. Distance of promontory from tympanic membrane:
[AIIMS May 05] [Delhi 05]
a. Modified eccrine glands b. Modified apocrine glands a. 2 m m b. 5 m m
c. Mucous gland d. Modified holocrine glands c. 6 m m d. 7 m m
2. Nerve supply for external ear are all except: [MAHE07] 14. Surface area of tympanic membrance: [Manipal 06]
a. Greater occipital nerve b. Greater auticular nerver a. 55 m m 2
b. 70 m m 2

c. Auriculotemporal nerve d. Lesser occipital nerve c. 80 m m 2


d. 90 m m 2

3. All of the following nerves supply auricle and extremal 15. The effective diameter of the tympanic membrane:
meatus except: [TN03] [UP 05]
a. Trigeminal nerve b. Glossopharyngeal nerve a. 25 m m 2
b. 30 m m 2

c. Auditory nerve d. Vagus nerve c. 40 m m 2


d. 45 m m 2

4. Which of the following nerves has no sensory supply to 16. Lever ratio of tympanic membrane is: [UP 01]
the auricle: [Al 12] a. 1.4-1 b. 1.3-1
a. Lesser occipital nerve c. 18.2-1 d. 1.5-1
b. Greater auricular nerve 17. "Cone of light" is due to: [AIIMS 96]
c. Auricular branch of vagus nerve a. Malleolar fold
d. Tympanic branch of glossopharyngeal nerve b. Handle of malleus
Skin over pinna is fixed: [JIPMER 95] c. Anterior inferior quadrant
a. Firmly on both sides b. Loosely on medial side d. Stapes
c. Loosely on lateral side d. Loosely on both side 18. In otoscopy, the most reliable sign is: [AIIMS 92]
6. Sensory supply of external auditory meatus is by: a. Lateral process of malleus
[PGI June 07] b. Handle of malleus
a. Pterygomandibular ganglion c. Umbo
b. Geniculate ganglion d. Cone of light
c. Facial nerve 19. Nerve supply of the tympanic membrane is by: [Al 95]
d. Auriculotemporal nerve a. Auriculotemporal b. Lesser occipital
Dehiscence of anterior wall of the external auditory canal c. Greater occipital d. Parasympathetic ganglion
cause infection in the parotid gland via 20. Nerve supply of tympanic memberane: [PGI Dec 02]
a. Fissure of Santorini b. Notch of ramus a. Auriculotemporal b. Auricular branch of vagus
c. Petrous fissure d. Retropharyngeal fissure c. Occipital NV d. Great auricular NV
8. What is the color o f t h e normal tympanic membrane? e. Glossopharyngeal NV
[CUPGEE96] 21. Which of the following is false about tympanic m e m -
a. Pearly white b. Gray brane? [Delhi 08]
c. Yellow d. Red a. Cone of light is anteroinferior
9. The most mobile part of the tympanic membrane: b. Shrapnell's membrane is also known as pars flaccida
[TN98] c. Healed perforation has three layers
a. Central b. Peripheral d. Anterior malleolar fold is longer than posterior
c. Both d. None of the above 22. Narrowest part of middle ear is: [PGI 97]
10. Pars flaccida of the tympanic membrance is also called a. Hypotympanum b. Epitympanum
[MP 07] c. Attic d. Mesotympanum
Reissner's membrane 23. Prussak's space is situated in: [MAHE 02]
Shrapnell's membrane a. Epitympanum b. Mesotympanum
Basilar membrane c. Hypotympanum d. Ear canal
Secondary tympanic membrane 24. All are components of epitympanum except:
11. Anterior wall of tympanic cavity contain: [PGI May 11] a. Body of incus b. Head of malleus
a. Promontry c. Chorda tympani d. Footplate of stapes
b. Bony part of pharyngotympanic tube 25. Sensory nerve supply of middle ear cavity is provided
c. Processus cochleariformis by: [AI95]
d. Pyramid a. Facial b. Glossopharyngeal
e. Tensor tympani muscle c. Vagus d. Trigeminal
12. The distance between tympanic membrane and medial 26. Tegmen seperates middle ear from the middle cranial
wall of middle ear at the level of center is: [PGI 00] fossa containing temporal lobe of brain by: [Karn. 06]
a. 3 m m b. 4 m m a. Medical wall of middle ear
c. 6 m m d. 2 m m b. Lateral wall of middle ear
CHAPTER 16 Anatomy of Ear J 199
c. Roof of middle ear 39. The length of Eustachian tube is: [AP99-JN06]
d. Anterior wall of middle ear a. 16 m m b. 24 m m
27. Facial recess or the posterior sinus is bounded by: c. 36 m m d. 40 m m
[TN 2003] 40. About Eustachian tube: [PGI June 02]
a. Medially by the vertical part of VII nerve a. 24 m m in length
b. Laterally by the chorda tympani b. Outer 1/3rds is cartilaginous
c. Above by the fossa includ is c. Inner 2/3rds is bony
d. All of the above d. Inner 2/3rds is cartilaginous
28. Floor of middle ear cavity is in relation with: [2001] e. Opens during swallowing
a. Internal carotid artery 41. True about Eustachian tube is/are: [PGI June 01]
b. Bulb o f t h e internal jugular vein a. Size is 3.75 cm
c. Sigmoid sinus b. Cartilagenous 1/3 and 2/3rd bony
d. Round w i n d o w c. Opens during swallowing
29. Promontory seen in the middle ear is: [PGI June 98] d. Nasopharyngeal opening is narrowest
a. Jugular bulge b. Basal turn of cochlea e. Tensor palati helps to open it
c. Semicircular canal d. Head of incus 42. True about Eustachian tube: [PGI Nov 10]
30. Process cochleariformis attaches to: [JIPMER 95] a. Length is 36 m m in adults and 1.6 to 3 m m in children
a. Tendon of tensor tympani b. Higher elastin content in adults
c. Ventilatory function of ear better developed in infants
b. Basal turns of helix
d. More horizontal in adults
c. Handle of malleus
i e. Angulated in infants
d. Incus
43. Eustachian tube opens into middle ear cavity at:
31. Mac Ewan's triangle is the landmark for: [MP98]
[UP 2000]
a. Maxillary sinus b. Mastoid antrum
a. Anterior walls b. Hypotympanum
c. Frontal sinus d. None
c. Superior surface d. Posterior wall
32. The suprameatal triangle overlies: [JIPMER 91]
44. Inner ear is present in which bone: [PGI 97]
a. Mastoid antrum b. Mastoid air cells
a. Parietal bone
c. Antrum d. Facial nerve
b. Petrous part of temporal bone
33. Anatomical l a n d m a r k indicating position of mastoid
c. Occipital bone
antrum: [CUPGEE96]
d. Petrous part of squamous bone
a. Suprameatal triangle
45. Inner ear bony labyrinth is: [Karn. 06]
b. Spine of Henle
a. Strongest bone in the body
c. Tip o f t h e mastoid process
b. Cancellous bone
d. None
c. Cartilaginous bone
34. All of the following form the boundary of MacEwen's
d. Membranous bone
triangle except: [Delhi 2008]
46. Cochlear aqueduct: [PGI June 98]
a. Temporal line
a. Connects internal ear with subarachnoid space
b. Posterosuperior segment of bony external auditory canal b. Connects cochlea with vestibule
c. Promontory c. Contains endoylymph
d. Tangent drawn to the external auditory meatus d. Same as S media
35. What is the type of joint between the ossicles of ear? 47. Infection of CNS spread in inner ear through:
[AI08] [AIIMSMay10,May1l]
a. Fibrous joint b. Primary cartilaginous a. Cochlear aqueduct b. Endolymphatic sac
c. Secondary cartilaginous d. Synovial joint c. Vestibular aqueduc d. Hyrtl fissure
36. Stapedius is supplied by: [JIPMER 92] 48. Crus commune is in: [Jharkhand 06]
a. Maxillary nerve b. Facial nerve a. Cochlea b. Middle ear
c. Auditory nerve d. Mandibular disese c. Behind retina d. Part of lens
37. Regarding stapedial reflex, which of the following is 49. Stapes footplate covers: [AIIMS May 03]
true: [MOO] a. Round window b. Oval window
a. It helps to enhance the sound conduction in middle ear c. Inferior sinus tympani d. Pyramid
b. It is a protective reflex against loud sound 50. Organ of corti is situated in: [Kerala 98]
c. It helps in masking the sound waves a. Scala media b. Sinus tympani
d. It is unilateral reflex c. Sinus vestibuli d. Saccule
38. Tensor tympani is supplied by: 51. Organ of corti is situated in: [TN06]
a. Anterior part of V nerve a. Basilar membrane
b. Posterior part of V nerve b. Utricle
c. IX nerve • c. Saccule
d. VII nerve. d. None of the above
J

200 T SECTION V Ear

52. Endolymphatic duct connects which structure: 6 3 . True regarding "Preauricular sinus" is: [MAHE 07]
a. Scala media to subdural space [Delhi05] a. Improper fusion of auricular tubercles
b. Scala vestibule to aqueduct of cochlea b. Persistent opening of first branchial arch
c. Scala tympani to aqueduct of cochlea c. Autosomal recessive pattern
d. Scala tympani to subdural space 64. Bone which is pneumatic: [PGI June 07]
53. Site where endolymph is seen: [Kerala 97] a. Maxillary b. Parietal
a. Scala vestibuli b. Scala media c. Temporal d. Frontal
c. Helicotrema d. Organ of corti e. Ethmoidal
54. Endolymph in inner ear: [AIIMS May 10] 65. Nerve of the pterygoid canal is also known a s : [PGI]
a. Is a filtrate of blood serum a. Arnold's nerve b. Vidian nerve
b. Is secreted by striae vascularis c. Nerve of Kuntz d. Criminal nerve of Grassi
c. Is secreted by basilar membrane
66. Singular nerve is a: [AP2007]
d. Is secreted by hair cells
a. Superior vestibular nerve supplying posterior semicircular
55. Labyrinthine artery is a branch of [AIIMS 91]
canal
a. Internal carotid artery
b. Interior vestibular nerve supplying post semicircular canal
b. Basilar artery

c. Superior vestibular nerve supplying anterior semicircular


c. Posterior cerebellar artery
canal
d. Anteroinferior cerebellar artery
d. Interior vestibular nerve supplying anterior semicircular
56. The following structure represents all the 3 components
canal
of the embryonic disc: [TN98]
67. Not correctly matched pair is: [TN2007]
a. Tympanic membrane b. Retina
a. Utricle and sacule -Semiciruclar canal
c. Meninges d. None o f t h e above
b. Oval window -Footplate of staps
57. Pinna develops from: [MH02]
c. Aditus ad antrum -MacEwen's triangle
a. 1st pharyngeal arch
b. 1 st and 3rd pharyngeal arch d. Scala vestibule -Reissner's membrane
c. 1 st and 2nd pharyngeal arch 68. In carcinoma base of tongue pain is referred to the ear
d. 2nd pharyngeal arch through: [Kerala 94]
58. Vertical crest at the internal auditory canal is: a. Hypoglossal nerve
[AIIMS May 11] b. Vagus nerve
a. Bill's bar b. Ponticulus c. Glossopharyngeal nerve
c. Cog d. Falciform crest d. Lingual nerve
59. Eustachian tube develops from: [PGI 97] 69. Spine of Henle is a: [MH2003]
a. 2nd and 3rd pharyngeal pouch a. Cortical bone
b. 1 st pharyngeal pouch b. Cancellous bone -

c. 2nd pharyngeal pouch c. Sclerotic bone i


d. 3rd pharyngeal pouch d. Long bone with Haversian system
60. All ofthe follwoing are ofthe size of adult at birth expect? 70. Which of the following is not a route of spread of infec-
[APPG06] tion from middle ear: [Al 12]
a. Tympanic membrane b. Ossicle a. Directly through openings such as round w i n d o w and oval
c. Tyamulus d. Mastoid antrum window
61. At birth the following structures are of adult size ex- b. By bony invasion
cept: [APPG06] c. Osteothrombotic route
a. Tympanic cavity b. Mastoid process d. Lymphatics
c. Malleus d. Tympanic ring 7 1 . Lateral wall of middle ear formed by: [FMGE 13]
62. Which of the following attain adult size before birth: a. Tegmen tympani
[AIIMS Nov 2010] b. Mastoid process
a. Ear ossicles Maxilla •
c. Promontory
c. Mastoid Parietal bone d. Tympanic membrane


CHAPTER 16 Anatomy of Ear _J 201

EXPLANATIONS AND REFERENCES

1. Ans. is b i.e. Modified apocrine glands Ref. IB Singh Histology 6th/ed p 214-215
Sweat glands are of 2 types

Eccrine / typical sweat glands Apocrine / Atypical sweat glands


• Distributed all over the body • Confined to some parts of body.
• Innervated by cholinergic nerves. • Innervated by adrenergic nerves.
• They open on the skin surface • They open into the hair follicle.
• Located on: Axilla, Mons pubis, Circumanal area, Areola, Nipple
Ceruminous glands of external acoustic meatus and ciliary glands of eyelids are modified apocrine
glands.

2. Ans. is a i.e Greater occipital nerve Ref. Dhingra Sth/ed p 5; 6th,'ed p 4 Scott Brown 7th/ed Vol. Ill pp 3106-3107
3. Ans. is c i.e. Auditory nerve
4. Ans. d i.e. Tympanic branch of glossopharyngeal nerve
:

N e r v e S u p p l y of Ear

External ear M i d d l e ear


Auricle/pinna External acoustic meatus Tympanic m e m b r a n e Cavity i.e. mucosa m a s t o i d , muscles a n t r u m , air
cells, a u d i t o r y t u b e .
Lateral surface Anterior wall and roof Lateral surface Tympanic plexus formed Tensor tympani by:
by auriculotempora by:
nerve
1. Upper 1/3 by Posterior wall and floor 1. A n t e r o i n f e r i o r part 1. Tympanic branch of Stapedius by facial nerve
auriculotemporal by auricular branch of by auriculo temporal glossopharyngeal
nerve vagus nerve nerve nerve.
2. Lower 2/3 by greater Posterior wall of auditory 2. Posteriosuperior part 2. Superior and inferior
auricular nerve canal also receives by auricular branch of Carotympanic nerves
innervations by facial vagus nerve (Sympathetic plexus
nerve through auricular around internal
branch of vagus carotid)

Medial surface Media surface


V. Upper 1/3 by lesser • Tympanic branch of
occipital nerve glossopharyngeal nerve
2. Lower 2/3 by greater (Jacobson nerve)
auricular nerve
3. Root of auricle by
auricular branch of
vagus nerve

Auriculotemporal nerve is a branch of mandibular nerve (branch of trigeminal nerve)

5. Ans. is b i.e. Loosely on medial side Ref. Dhingra 6th/edp 2, Sth/ed p 3


Skin over the pinna is closely adherent to the perichondrium on the lateral surface while it is loosely attached on the medial surface.
6. Ans. is d i.e. Auriculotemporal nerve Ref. Dhingra 6th/ed, p 4; 5th/edp.5; BDC 4th/ed, Vol. Hip - 254
7. Ans. is a i.e. Fissure of Santorini Ref: Dhingra 6th/edp 2,5th/ed p 4
• The cartilaginous part of external auditory canal has 2 deficiencies - the"fissures of santorini"through w h i c h infections can pass
from external ear t o parotid and vice versa.
• Fissure o f santorini is present in the cartilaginous part of auditory canal.
• The deficiency present in bony part is "Foramen of Huschke"seen in children up t o the age of 4. Through this infections of ear
can also pass t o parotid gland
I

202 f_ SECTION V Ear

| ALSO KNOW

In neonates, bony external meatus as the tympanic bone is not yet developed.
8. Ans. is a i.e. Pearly white Ref. Dhingra 5th/edp 61; Maqbool 11 th/edp 33; Turner 1 Oth/edp 240
Such a simple appearing question can also confuse us w i t h its options. Most o f t h e texts say that tympanic membrane is pearly
gray in color.
"Normal tympanic membrane is shiny and pearly gray in color." ... Dhingra 6th/ed p 55; 5th/ed p 61
"Tympanic membrane appears as a greyish white translucent membrane." ... Maqbool 71 th/ed p 33
"In health, the drum head presents a highly gray surface." ... Turner Wth/edp 240
So, neither option "a" i.e. pearly w h i t e nor option "fa" i.e. gray is fully correct but from ages the answer is taken as pearly white, so
I am in also taking option "a" i.e. pearly white as the correct o p t i o n .
9. Ans. is b i.e. Peripheral Ref. Dhingra 5th/ed p 78
"Movements of tympanic membrane are more at the periphery than at the center where malleus handle is attached."
10. Ans. is b i.e. Shrapnell membrane Ref: Dhingra 6th/edp2,5/epg-4
Pars flaccida /Shrapnell's membrane

Situated above the lateral process of malleus between the notch of Rivinus and the anterior and posterior malleal folds.

ALSO KNOW
• Reissner's membrane - Separates scala media f r o m scala vestibuli in the inner ear (Dhingra 6th/edp 70,5th/edp 12)
• Basilar membrane - Seen in scala media and supports the organ of corti (Dhingra 6/e p 10,5/ep 12)
• Secondary Tympanic Membrane - Closes the scala tympani at the site of round w i n d o w (Dhingra 5th/edp 11)
11. Ans. is e i.e. Tensor tympani muscle Ref. Dhingra 6th/edp7-8,5th/edp 6
The anterior wall has a t h i n plate of bone which separates the cavity f r o m internal carotid. It also has t w o openings; the lower one
for Eustachian tube and the upper one for the canal of tensor tympani muscle.
12. Ans. is d i.e. 2 mm
13. Ans. is a i.e. 2 mm Ref. BDC Vol. Ill 4th/ed p 258; Dhingra 6th/ed p450; point 129
" When seen in coronal section, the cavity ofthe middle ear is biconcave, as the medial and lateral walls are closest to each other
in the center."
The distances separating them are: • Near the roof 6 m m -> Epitympanum (Attic)
• In the centre 2 m m —> Mesotympanum
• Near the floor 4 m m —» H y p o t y m p a n u m
The medial wall of the tympanic cavity is formed by the labyrinth and the lateral wall is formed by the tympanic membrane.
14. Ans. is d i.e. 90 m m 2
Ref. Maqbool 11 th/ed p 19; Dhingra 6th/edp 446; point 8,5th/ed p 457; point 8
15. Ans. is d i.e. 45 m m 2

• Area of tympanic membrane is 90 m m . 2

• Effective area is 55 m m (approximately 2/3 o f t h e total area).


2 ,d

• Significance o f large area of tympanic membrane - The area of tympanic is much larger than area of stapes footplate, which
helps in converting sound o f greater amplitude but lesser force t o that of lesser amplitude and great force.
16. Ans. is b i.e. 1.3:1 Ref. Dhingra 6th/edp14,5th/edp 18

Lever-Action of Ossicles

Handle of malleus is 1.3 times longer than process o f t h e incus which constitutes for the lever-action.

Area ratio of tympanic membrane is 14:1


Lever ratio = 1.3:1
= Their product is 18:1 i.e. the pressure exerted at oval window.
This helps in the transformer action o f t h e middle ear (impedance matching mechanism) i.e. converting sound of greater amplitude
and less force t o that of lesser amplitude b u t greater force.
17. Ans. is b i.e. Handle of malleus Ref. Logan and Turner Wth/edp 240
Cone of Light
• Seen in anteroinferior quadrant o f t h e tympanic membrane is actually the reflection o f t h e light projected into the ear canal t o
examine it.
• This part reflects it because it is the only part of tympanic membrane that is approximately at right angles t o the meatus.
• This difference in different parts o f t h e tympanic membrane is due to the handle of malleus which pulls the tympanic membrane
Q u e r y o n 23 Q u e s t i o n s .

CHAPTER 16 Anatomy of Ear

and causes it t o tent inside.


Thus, the handle of malleus causes tenting and because of tenting the anteroinferior quardrant is at right angles t o the meatus and
thus reflects the light (leading t o cone light).
18. Ans. is a i.e. Lateral process of malleus Ref. Maqbool 11th/edp 33

Otoscopy

• Helps t o view the inside of external auditory canal.


• For proper view: Pinna is pulled
- Backward and upward in adults. 0

- Downward and outward in infants. 0

• The tympanic membrane appears as a grayish white, translucent membrane set obliquely inside the canal.
The important landmarks on membrane are:

Landmark Importance
• The short process: (Lateral process of malleus) It is the most important landmark as it is least obliterated in disease
• Anterior and posterior malleolar folds Separates pars tensa from pars flaccida
• Handle of malleus: It is directed downward and backward; ending at Cone of light radiates from it. Pars tensa is arbitrarily divided into four
the umbo quadrants by a vertical line passing along the handle of malleus and
horizontal line intersecting it at umbo

Since, short process/lateral process of malleus is least obliterated by diseases so I think it is the most reliable sign in otoscopy.

19. Ans. is a i.e Auricotemporal nerve


20. Ans. is a, b and e i.e. Auricotemoral nerve; Auricular branch of vagus nerve and Glossopharyngeal nerve
21. Ans. is c i.e. Healed perforation has three layers Ref. Dhingra 6th/ed p 2,3,5th/ed p 4,79

Let's see E a c h o p t i o n o n e by o n e

Option a - Cone of light is anteroinferior


This is correct - "A bright cone of light can be seen radiating from the tip of malleus to the periphery in the antero-inferior quadrant"
-Dhingra 5th/ed p 4
Option b - Shrapnell's membrane is also called as pars flaccida. This is absolutely correct - Dhingra 6th/edp2, Sth/edp 4
Option c - Healed perforation has 3 layers

This is incorrect
• When perforation of tympanic membrane heals, it heals in t w o layers and not in three layers. (Dhingra 6th/edp 55-56)
• "Healed chronic otitis media is the condition w h e n tympanic membrane has healed (usually by t w o layers) is atrophic and easily
retracted if there is negative pressure in the middle ear" - Dhingra 5/e p 79
Option d - Anterior malleal fold is longer than posterior fold. Well! it is not given anywhere that anterior fold is longer than posterior,
but we have to eliminate one o p t i o n and that definitely is o p t i o n 'c'.
22. Ans. is d i.e Mesotympanum
Ref. Maqbool 17 th/edp 20; BDC Vol. Ill 4th/edp 258; Dhingra 6th/edp 450; point 129,5th/edp 462;point 114
Vertical and anteroposterior dimensions of middle ear are 15 m m each while transverse dimension is 2 m m at mesotympanum,
6 m m above at the e p i t y m p a n u m and 4 m m below in the h y p o t y m p a n u m . Thus, middle ear is the narrowest between the u m b o
and promontory.
23. Ans. is a i.e. Epitympanum Ref. Dhingra 6th/ed p 449; point 149,5th/ed p 461; point 90; Maqbool llth/edp 13
Prussak's space lies medial t o pars flaccida, lateral t o the neck o f malleus and above the lateral process of malleus. Anteriorly,
posteriorly and superiorly, it is b o u n d e d by lateral malleal ligament. Posteriorly, it also has a gap t h r o u g h w h i c h the space
communicates w i t h epitympanum.
• Importance of this space is that the cholesteatoma may extend to posterior mesotympanum, under lateral incudal fold and
infection here does not drain easily and causes attic pathology.
24. Ans. is d i.e. Footplates of stapes Dhingra 6th/edp 5 Fig. 1.8,5th/edp 6, Fig. 1.5
It is clearly evident f r o m the diagram given on page 7 o f t h e guide that footplates of stapes is a part of mesotympanum and not
epitympanum.
25. Ans. is b i.e. Glossopharyngeal nerve Ref. Dhingra 6th/edp 8,5th/ed p 10
• The nerve supply of middle ear is derived f r o m tympanic plexus which lies over the promontory.
204^ SECTION V Ear

• The inferior ganglion o f t h e glossopharyngeal nerve gives off the tympanic nerve which enters the middle ear t h r o u g h the
tympanic canaliculus and takes part in formation of the tympanic plexus on the medial wall of middle ear.
• This distributes it fibres t o the middle ear, and also t o the auditory tube, aditus ad atrum mastoideum (aditus t o mastoid antrum).
Middle ear
Glossopharyngeal nerve —> Tympanic nerve/tympanic plexus Auditory tube
Mastoid antrum
26. Ans. is c i.e. Roof of middle ear Ref. Dhingra 4th/ed pg 5,5th/ed p 5,6th/ed p 5
• The roof o f middle ear is formed by a t h i n plate of bone called tegmen tympani. It separates tympanic cavity f r o m middle cranial
fossa.
• Tegmen t y m p a n i is formed by squamous and petrous part of temporal bone. 0

27. Ans. is d i.e. All of the above Ref. Dhingra 6th/e p 5, Sth/edp 6
Facial recess or Posterior sinus - It is a depression in the posterior wall o f t h e middle ear.
It is b o u n d e d by:
Medially-Vertical part of VIII nerve
Laterally - Chorda tympani
Above - Fossa incudis
Importance -This recess is important surgically, as direct access can be made t h r o u g h this into the middle ear w i t h o u t disturbing
posterior canal wall.
28. Ans. is b i.e. Bulb of internal jugular vein Ref. Dhingra 6th/ed p 5, Sth/edp 6; Scott Brown 7th/ed Vol. Ill p 3110
Floor of middle ear separates tympanic cavity (hypotympanum) from the jugular bulb

Superior

Anterior

Middle Tensor tympani


cranial fossa, muscle,
temporal lobe Eustachian tube

1 MediaFlH Inner ear Lateral

Aditus to mastoid

Inferior
Relation of middle ear
:

• At thejunction ofthe floor and the medial wall ofthe cavity there is a small opening that allows the entry ofthe tympanic branch of glossopharyngeal
nerve into the middle ear from its origin below the base of skull
• Anterior wall separates tympanic cavity from internal carotid artery
29. Ans. is b i.e. Basal turn of cochlea Ref. Dhingra 6th/edp5, Sth/ed p 6
Promontory is seen in the medial wall of middle ear and is due to basal coil of cochlea.

ALSO KNOW •

Medial wall o f middle ear is formed by labyrinth.


The main features on medial wall are:
• A bulge called as p r o m o n t o r y formed by basal t u r n of cochlea.
0

• Fenestra vestibuli (oval w i n d o w ) lies posterosuperior t o the promontory and opens into scala vestibuli. It is occupied by f o o t -
0

plate of stapes which is fixed by annular ligament. Oval w i n d o w is kidney-shaped


• Fenestra cochleae (round window) lies posteroinferior, t o the promontory and opens into scala tympani of cochlea. It is closed
be secondary tympanic membrane. The round w i n d o w is closest t o ampulla of posterior semicircular canal.
CHAPTER 16 Anatomy of Ear ]205
• Prominence of facial nerve canal lies above the fenestra vestibuli curving downward into posterior wall of middle ear.
• Anterior t o oval w i n d o w lies a hook-like projection called the processus cochleariformis for tendon of tensor t y m p a n i .
0 0

The cochleariform process marks the level o f t h e Genu o f t h e facial nerve which is an i m p o r t a n t landmark for surgery o f t h e facial
nerve.
Medial t o t h e pyramid is a deep recess called sinus t y m p a n i which is bounded by the subiculum below and ponticus above.
0

30. Ans. is a i.e. Tendon of tensor tympani Ref. Dhingra 6th/edp 5,5th/ed p 6
31. Ans. is b i.e. Mastoid antrum
32. Ans. is a i.e. Mastoid antrum
33. Ans. is a i.e. Suprameatal triangle
34. Ans. is c i.e. Promontory Ref.Dhingra6th/edp5,5th/edp7
Mastoid antrum is marked externally on the surface by suprameatal (Mac Ewen's) triangle. .

MacEwen's Triangle

It is bounded by:
MacEwan's triangle

a. Supramastoid crest
Spine of Henle
b. Posterosuperior segment of EAC
c. Tangent drawn t o external canal

• Temporal line
• Posterosuperior segment of bony external auditory canal.
• The line drawn as a tangent t o the external canal.
• It is an i m p o r t a n t landmark t o locate the mastoid antrum in the mastoid surgery.
35. Ans. is d i.e. Synovial joint Ref. Grays 38th/ed pp 485,617 and 1275

Joints of the ossicles


The incudomalleolar joint Incudostapedial joint
I • • • • • mn •
Saddle joint (variety of synovial joint) Ball and socket joint (type of synovial joint)

36. Ans. is b i.e. Facial nerve Ref. Dhingra 6th/ed p 5,5th/edp 10


37. Ans. is b i.e. It is a protective reflex against loud sounds Ref. Dhingra 5th/ed p 9-10,30
Stapedius muscle helps t o dampen very loud sound and thus prevents noise trauma t o the inner ear. It is supplied by VII nerve
(facial nerve). Lesions of facial nerve lead t o loss of stapedial reflex and hyperacusisor phonophobia i.e. intolerance t o loud sounds.
For more details see chapter - physiology of hearing and assessment of hearing loss o f t h e guide

1 note
WSBKKtK^sSMl^sSB^BSt
Stapedial reflex = Acoustic reflex

38. ?
Friends - According t o BDC 4/e Vol. Ill p 153, mandibular nerve has a main branch which after traveling a short course divides
into 2 i.e. anterior (small) and posterior trunk (large)
206 L SECTION V Ear

Branches

From the main trunk •


I
I
Meningeal branch Nerve to medial pterygoid supplies
T
Supplies the dura mater J:
of middle cranial fossa Otic ganglion Tensor veli palatini Tensor tympani muscles

-
From the anterior trunk


Sensory branch k/a Buccal nerve Motor branches

r I
Masseteric nerve Deep temporal nerve Nerve to lateral pterygoid

From the posterior trunk

r 1
Auriculotemporal nerve Lingual nerve Inferior alveolar nerve

T T
They are the 2 terminal branches

39. Ans. is c i.e. 36 mm


40. Ans. is d and e i.e. Inner 2/3rd is Cartilaginous; and Opens during swallowing
Ref. Logan and Turner 10th/ed p 227; Dhingra 6th/ed p 57,5th/ed p 63
• The Eustachian tube/auditory tube in the adult is 36 m m in length. From its pharyngeal end, it runs upwards, laterally and
backward.
• In infants, the t u b e is shorter, wider and is more horizontal.
• It has t w o parts—a pharyngeal cartilaginous part which forms 2/3rd (24 mm) of its length (i.e. inner or medial part) and a
tympanic bony part which forms remaining 1 /3rd (outer or lateral part). (12 mm). This is just reverse of external auditory canal

Remember: Mnemonic ICE 2/3: Inner part Cartilaginous in Eustachian tube and forms 2/3 part.

• The t w o parts meet at isthmus which is the narrowest part of tube.


• The fibers of origin of tensor palati muscles are attached t o lateral wall o f the tube. Contraction of this muscle during
swallowing, yawning and sneezing opens the tube and this helps in maintaining equality of air pressure on b o t h sides
of tympanic membrane. Contraction of levator palati muscles which runs below the floor of cartilaginous part also helps in
opening the tube.
• It is lined by pseudostratitified columnar ciliated epithelium (cartilaginous part contains numerous mucous glands).
41. Ans. is a, c and e i.e. Size is 3.7 cm; Opens during swallowing; and Tensor palati helps to open it
Ref. Dhingra 6th/edp 57,5th/ed p 63
• Eustachian t u b e is 36 m m in length, lateral 2/3 cartilaginous; isthmus is narrowest part.
• Remember mnemonic: ICE 2/3
• Normally the Eustachian tube remains closed
• Eustachian tube opens during swallowing, yawning and sneezing by the action of tensor and levator palati.
For more details, see previous answer
4 2 . Ans. is b i.e. Higher elastin content in adults Ref. Dhingra 6th/ed p57,5th/ed p 65
The Developing Humans: Kleith 8th/edp 431-32, Longman's Embryology Wth/ed pp 317-323
"Eustachian tube serves to ventilate the middle ear and exchange nasopharyngeal air in the middle ear. In children, ETis relatively narrow.
It is prone to obstruction when mucosa swell in response to infection or allergic challenge and it results in middle ear effusion"
0

—Cray's 40th/ed p 626


CHAPTER 16 Anatomy of Ear J 207
Eustachian Tube

• It connects nasopharynx w i t h the tympanic cavity. In adult, it is about 36 cm long and runs downward, forward medially f r o m
its tympanic end, f o r m i n g an angle of 45° w i t h the horizontal.
• The tympanic end of the tube is body and is situated in the anterior wall of middle ear. A little above the level of floor. The
pharyngeal end of the t u b e is slit like and is situated in the lateral wall o f t h e nasopharynx, 1-1.25 cm behind the posterior end
of inferior t u b i n a t e .
0

• It develops f r o m first arch and partly f r o m second arch.


Table (Dhingra 5/e, p 65): Differences between infant and adult Eustachian tube

Infant Adult
Length 13-18 cm birth (about half as long as in adult) 36 mm (31-38 mm)
Direction More horizontal , At birth it forms an angle of 10°
0
Forms an angle of 45° with the horizonal
with the horizontal At age 7 and later it is 45°
Angulation at isthmus No angulation Angulation persent
Bony versus cartilaginous Bony parts is slightly longer than 1/3 of the total Bony part 1/3; cartilagious part 32/3
length ofthe tube and is relatively wider
Tubal cartilaginous part Flaccid. Retrograde reflux nasopharyngeal Comparatively rigid, Remains closed and protects
Density of elastin at the hinge Less dense; tube does not efficiently close by recoil Density of elastin more and helps to keep the tune
closed by recoil of cartilage
Ostmann's pad of fat Less in-volume Large and helps to keep the tube closed

43. Ans. is a i.e. Anterior wall Ref. Dhingra 6th/edp5,5th/edp6; Scott Brown 7th/ed Vol. Ill p 3114 Fig. 225.13
Anterior wall o f tympanic cavity is formed by a t h i n plate of bone. It has 2 openings:
• Lower one for the Eustachian t u b e and upper one f o r t h e canal of tensor tympani muscle.
• Friends remember the diagram I have provided in Ans. 26 - It is important and helps in solving such questions.
44. A n s . i s b i.e. Petrous part temporal bone Ref. Turner 10th/ed p 228; BDC 4th/ed Vol. Hip 264
Inner ear lies w i t h i n the petrous part of temporal bone.
4 5 . Ans. is b i.e. Cancellous bone
Sorry for this one
46. Ans. is a i.e. Connects internal ear with subarachnoid space Ref. Dhingra 6th/edp 9
Cochlear aqueduct connects scala tympani w i t h the subarachnoid space. This is the reason why ottis media can lead to meningitis
Stapes Helicotrema

Scala media
(endolymph)

Subarachnoid
space
C.S.F

47. Ans. is a i.e. Cochlear aqueduct


Ref. Grey 40th/ed p635; Dhingra 5th/ed p112; http://Journalsleww.com/Otology, Pediatric audiology: Diagnosis, Technology and Man-
agement by Jane R. Madell, Carol Flexer2008, p. 28
• As we know that cochlear aqueduct (Aqueduct of Cochlea) is a connection between scala t y m p a n i (containing perilymph) and
the subarachnoid space (containing CSF). On occassions, particularly in young children, the Cochlear aqueduct is large and
open.
• Infection can spread to the inner ear f r o m the infected CSF or vice versa, via the cochlear aqueduct resulting in severe profound
hearing loss (meningitic labyrinthitis).
SECTION V Ear

48. Ans. is a i.e. Cochlea Ref. Dhingra 6th/edp 10,5th/edp 11

Vestibule
Bony labyrinth has 3 parts • Semicircular canals
_ Cochlea

Semicircular Canals

• There are 3 semicircular canals - the lateral, posterior and superior which lie in a plane of right angles t o one another
• Each canal has an ampullated end which opens independently into the vestibule and a non ampullated end
Superior S.C.C

Crus commune

Cochlea

Post S.C.C

Lateral S.C.C
Round window

• The non-ampullated ends o f posterior and superior canals unite t o f o r m a c o m m o n channel called the crus commune.
So the three canals open into the vestibule by 5 openings. Thus actually crus c o m m u n e is a part of semicircular canals but since
this o p t i o n is not given, we are taking the next best o p t i o n i.e. cochlea.

Also Remember

• Crista ampullaris: It is located in the ampullated end o f the three semicircular duct and is a receptor which responds t o angular
acceleration.
• Utricle and saccule lie in the bony vestibule, together they are called the otolith organ. Their sensory epithelium is called as
Macula which responds t o linear acceleration and deceleration
49. Ans. is b i.e. Oval window Ref. BDC4th/ed Vol. 3, p 258
• Footplate of stapes covers the oval w i n d o w and secondary tympanic membrane covers the round window.
• Mnemonic: SORT: • Stapes (footplate) covers
• Oval w i n d o w
• Round w i n d o w is covered by
• • Tympanic membrane (Secondary)
50. Ans. is a i.e. Scala media
51. Ans. is a i.e. Basilar membrane Ref. Dhingra 6th/ed p 9,5th/ed p 11
Bony cochlea has three compartments: Scala vestibuli •

• Scala tympani
• Scala media or the membranous cochlea.
The scala vestibuli and scala tympani are filled w i t h perilymph and communicated w i t h each other at apex of cochlea t h r o u g h an
opening called helicotrema . 0

Scala media is the membranous cochlea or cochlear duct. It has 3 walls:


• The basilar membrane, which supports the organ of corti.
• The Reissner's membrane which separates it f r o m the scala vestibuli.
• The stria vascularis which corftains vascular epithelium is concerned w i t h secretion of e n d o l y m p h .
52. Ans. is a i.e. Scala media to subdural space Ref. Dhingra 6th/ed p 9,5th/edp 12
Endolymphatic duct - It is a part of membranous labyrinth (Scala media)
• It is f o r m e d by union of saccule and utricle
• It connects scala media t o subdural space
• Its terminal part is dilated t o f o r m the endolymphatic sac
• Endolymphatic sac lies between the t w o layers o f dura on the posterior surface of petrous bone
• Surgical importance - Endolymphatic sac is exposed for drainage or shunt operation in Meniere's disease
CHAPTER 16 Anatomy of Ear J 209
Outer hair , 9 e l l s o f

cells Hensen
Tectorial
membrane
Inner hair
cells •

- •

Tunnel
Spiral Deiter's\
o f C o r t i

ganglion -Basilar c e l l s

membrane
Nerve fibers
Cochlear nerve (unmyelinated)
fibers (myelinated)

ALSO KNOW

• Ductus reuniens - connects cochlear duct t o saccule


• Aqueduct o f cochlea - connects scala t y m p a n i t o subarachnoid space
53. Ans. is b i.e. Scala media Ref. Dhingra 6th/ed p 10,5th/ed p 12
Scala vestibuli and scala tympani are filled with perilymph, whereas scala media/membranous cochlea is filled with endolymph.
Origin and absorption o f inner ear fluids.

Origin Absorption

• Perilymph (It resembles ECF and is rich in Nations) • Through aqueduct of cochlea to subarachnoid space
.- From CSF
- Direct blood filtrate from the vessels of spiral ligament
• Endolymph (It resembles ICF and is rich in K ions +
• Endolymphatic sac
- Secreted by stria vascularis or by the adjacent tissues of outer sulcus • Stria vascularis
- Derived from perilymph across Reissner's membrane

54. Ans. is b i.e. Is secreted by striae vascularis Ref. Dhingra 6th/edp 10,5th/edp 12
Already explained, kindly see previous answer and text for the explanation.
55. Ans. is d i.e. Anterior inferior cerebellar artery Ref. Dhingra 6th/ed p 11; 5th/ed p 13
Labyrinthine artery is a branch of anteroinferior cerebellar artery but can sometimes arise from basilar artery.
• It supplies w h o l e o f t h e inner ear.
Kindly see the preceeding text for more details
56. Ans. is a i.e. Tympanic membrane Ref. Dhingra 6th/ed p 12,5th/ed p 14
Tympanic membrane develops f r o m all the three germinal layers. Outer epithelial layer is f o r m e d by the ectoderm, inner mucosal
layer by t h e e n d o d e r m and t h e middle fibrous layer by the mesoderm.
57. Ans. is c i.e. 1 a n d 2
st n d
pharyngeal arch Ref. Dhingra 6th/edp 7 7,5th/edp 14

Pinna

It develops f r o m b o t h 1 and 2st n d


brachial arches
Period of d e v e l o p m e n t starts f r o m 4-6 weeks and adult configuration is attained by 2 0 th
week
• From the 1 arch
s t
2— arch
• Tragus Rest of the pinna
. Crus o f helix
• Adjacent fielix
• The tissue condensations o f t h e mesoderm o f t h e 1 and 2 st n d
brachial arch f o r m 6 hillocks o f His, which fuse t o f o r m t h e pinna

ALSO KNOW

• External auditory canal - develops from t h e 1 brachial cleft/groove st 0

• Tympanic membrane - develops from all 3 germ layers (Ecoderm, mesoderm and e n d o d e r m ) 0
210|_ SECTION V Ear

58. Ans. is a i.e. Bill's bar


Ref: Acoustic tumor: diagnosis and management; Charles M Luteje, Karen Jo Doyle, William F House, p 359
Bill's bar is t h e vertical crest extending superiorly from the crista falciformis, dividing the superior compartment of the lateral part
of internal auditory canal.
5 9 . Ans. is b i.e. First pharyngeal pouch and c i.e. 2 pharyngeal pouch
n d
Ref. IBSingh Embryology 8th/edp 110
The Eustachian tube, tympanic cavity, attic, antrum and mastoid develops from endoderm of tubotympanic recess which arises
from the first and partly from the second pharyngeal pouch. Since this question is of PGI - we are taking both 1 and 2 pouch as st n d

correct answer but if single o p t i o n is to be marked, it will be 1 pharyngeal pouch.


st

Pharyngeal Pouches and Derivatives

Pharyngeal pouch Ventral Dorsal


I Gets obliterated • Tubotympanic Recess
Proximal Distal
Eustachian tube Middle ear cavity tympanic antrum
II Palatine tonsil (part of this pouch persists
as intratonsillar cleft) -

III Thymus • Tubotympanic recess • Adenoids


IV Small amount of thymus tissue • Inferior parathyroid gland
V. Ultimobranchial body • Superior parathyroid glands

60. Ans. is d i.e. Mastoid antrum Ref Scotts Brown 7th/ed Vol. Ill p 3118
6 1 . Ans. is b i.e. Mastoid process Ref: Maqbool 11th/ed p 14
"Mastoid antrum is an air-filled sinus within the petrous part of temporal bone. It commincates with the middle ear by way ofthe aditus
and has mastoid air cells arising from its walls. The antrum, but not the air cells is well developed at birth"
—Scott Brown 7th/ed Vol. 3 p 3118
"Development of the mastoid air cell system does not occur until afterbirth, with about 90% of air cell formation being completed by the
age of six with the remaining 10% taking place up to age of 18" —Scotts Brown 7th/ed Vol. 3 p 3122
Hence, mastoid antrum which is not complete w i t h o u t its air cells, development is not complete at birth.
As far as - Q. 55 is concerned
Maqbool 11 th/ed p 14 says:
"The mastoid process is not present at birth and starts developing at the end ofthe first year and reaches its adult size at puberty."
"In infancy, the mastoid process being absent, the facial nerve emerges lateral to the tympanic portion from the stylo mastoid foramen
and is likely to get injured by the usual postaural incision." Maqbool 1 Ith/edp 14

f both mastoid antrum and mastoid process are given as options in any MCQ—always mark mastoid tip, as mastoid tip is a better option
because mastoid antrum per se is formed at birth but its air cells are not formed whereas whole of mastoid tip/mastoid process is not
present at birth.

62. Ans. is a i.e. Ear ossicles Ref. Pediatric Neuroradiology, edited by Paolo Tortori Donati 7/e, p 1362
• The ossicles begins to form during 4th week of gestation from the mesenchymal tissue.
• They originate as cartilaginous models that reach adult size by the 18th week of gestation. Ossification of malleus begins at 15th
week gestation, while stapes begins t o ossify at 18th week of gestation. At birth, the ossicles are of nearly adult size.

ALSO KNOW

Mastoid bone not the mastoid ptocess is almost the adult size at birth, while maxilla and parital bone grow in size as head grows.
63. Ans. is a i.e. Improper fusion of auricular tubercles Ref. Dhingra 6/e p 11,49; 5/e p 54
• This is c o m m o n l y seen at the root of helix and is due t o incomplete fusion of tubercles during
• Development of external ear
• It is a blind track lined by squamous epithelium
• It may get repeatedly infected causing purulent discharge
CHAPTER 16 Anatomy of Ear

• Abscess may also form


• Treatment is surgical excision of the track if the sinus gets repeatedly infected.

ALSO KNOW

Collaural Fistula
It is an anomaly of first brachial cleft:
• In this, there is one opening in the floor of external auditory meatus and another behind the angle of mandible close t o anterior
border of sternocleidomastoid
• Tract of fistula passes t h r o u g h parotid in close proximation t o facial nerve
• Treatment is excision of tract
64. Ans. is a and e i.e. Maxillary; and Ethmoidal Ref. BDC Handbook ofGeneral Anatomy 4th/ed p 32
Pneumatic bones are one which contain large air spaces lined by epithelium e.g.: maxilla, sphenoid, ethmoid, etc. They make the skull
light in weight, help in resonance of voice, and act as air conditioning chambers for the inspired air.
65. Ans. is b i.e. Vidian nerve Ref. Dhingra 5th/edp 154; Tuli Ist/edp 84
• Greater superifkial petrosal nerve joins the deep petrosal nerve to form the nerve of pterygoid canal or also called as Vidian nerve.
• Vidian nerve reaches pterygopalatine ganglion t o supply the lacrimal gland and mucous glands of nose, palate and pharynx.
Arnold nerve: It is a branch of cranial nerve X which carries fibers that supply sensory innervation t o the ear canal
Jacobson nerve: It is a branch of cranial nerve IX that runs along the promontory o f t h e middle ear supplying sensation and
parasympathetic fibers t o the parotid gland
66. Ans. is b i.e. Inferior vestibular nerve supplying the posterior semicircular canal Ref. Scott Brown 7th/ed Vol. 3 p 3120
• Inferior vestibular nerve passes t h r o u g h the inferior vestibular foramen t o supply the saccule.
• Just behind and below the inferior vestibular foramen is the foramen ofsinglare, which contains a branch of inferior vestibular
nerve called as the singular nerve'
• The singular nerve runs obliquely t h r o u g h the petrous bone close to the round w i n d o w t o supply the sensory epithelium in the
ampula o f t h e posterior semicircular canal.
67. Ans. is c i.e. Aditus ad antrum - Mac Ewen's triangle Ref. Scott Brown 7th/ed Vol. 3 p 3120
Let's analyze each o p t i o n separately.
Option a:
Utricle and saccule - Semicircular canal
Utricle lies bony vestibule and receives the five openings o f t h e three semicircular ducts/semicircular canals
Saccule also lies in the bony vestibule, anterior t o the utricle and together b o t h of there are called otolith organs.
Hence, this pair is correct
Option b:
Oval w i n d o w - footplate of stapes
Oval w i n d o w is closed by the footplate of stapes.
Hence this pair is also related t o each other
Option c:
Aditus ad a n t r u m - MacEwen's triangle
Aditus ad a n t r u m is an opening t h r o u g h which the attic communicates w i t h the antrum.
Mastoid a n t r u m and not the aditus is marked externally on by MacEwen's triangle
Hence, this pair is not correctly matched.
Option d:
Scala vestibule - Reissner's membrane
Reissner's membrane separates scala vestibule f r o m scala media
Hence, this pair is also related t o each other.
68. Ans. is d i.e. Lingual nerve ' Ref. Dhingra 6th/edp 228,5th/ed p 241
In carcinoma base of tongue pain is referred t o the ipsilateral ear because o f t h e c o m m o n nerve supply o f t h e t o n g u e (lingual nerve)
and ear (auriculotemporal nerve) f r o m the mandibular division o f t h e trigeminal nerve.
6 9 . Ans. is b. i.e. Cancellous bone
• T h e a n t u m lies above and behind a projection of bone called the spine of Henle—Maqbool 11/e, p 14
• Whether spine of Henle is cancellous bone is not given in it.
212|_ SECTION V Ear

70. Ans. d i.e. Lymphatics Ref. ENT, PL Dhingra 5th/ed p 84


Pathways of spread of infection from middle ear
Direct bone erosion
(e.g. cholestoma)

thrornbophlsbitis
I. Congenital dehiscence
ii. Patent sutures
iii. Previous skull fractures Middle
iv. Surgical dects eg stapedectomy
v. Oral/round window

of middleear- ^
• R o o f - T h i n plate called as tegmen t y m p a n i
Floor-Jugular bulbw
• Anterior wall - Internal carotid artery
. Posterior wall - Lies close t o mastoid air cells
• Medial wall - labyrinth
• Lateral wall - tympanic membrane


CHAPTER

Physiology of Ear and Hearing

| P H Y S I O L O G Y O F H E A R I N G - A U D I T O R Y PATHWAY

Hair cells are innervated by den- Dorsal cochlear nucleus


drites of biopolar cells of spiral Superior olivary nucleus
g a n g l i o n s i t u a t e d in Rosenthal Ventral cochlear nucleus I
c a n a l . A x o n s of t h e s e b i o p o l a r Lateral lemniscus
cells form the cochlear division I
of eighth nerve Auditory cortex <- Medial geniculate body <-• Inferior colliculus

Mnemonic E Eighth nerve


for auditory C Cochlear nuclei
pathway O Superior olivary nucleus
L Lateral Lemniscus
Auditory
1 Inferior colliculus cortex in
M Medial geniculate body temporal
lobe
A Auditory cortex Medial
Auditory radiations
The auditory fibers travel via the ipsilateral and contralateral routes in sublentiform part
of internal capsule geniculate body
and have multiple decussation points.Thus, each ear is represented
in both cerebral hemispheres.
Mid brain
The area o f cortex, concerned w i t h hearing is situated in the
superior temporal gyrus (Brodmann's area 41).

O r g a n of Corti

It is the sense organ o f hearing and is situated o n the basilar Pons


membrane in scala media.

Important components of the organ of Corti are:


1. Tunnel of Corti, w h i c h is formed by the inner and outer rods.
It contains a fluid called cortilymph. Cochlea
2. Cells: Fig. 17.1: Central auditory pathways
Sensory hair cells S u p p o r t i n g cells Coutesy: Text book of Diseases of Ear, Nose and
Throat, Mohan Bansal. Jaypee Brothers, p 21
Inner Outer • Deiter's cells
• Primarily afferent • Efferent • Pillar cells
• Resistant • Susceptible to • Hensen's cells
ototoxic drugs With age hair cells at base are lost more than at the apex. Significance-
so hearing loss is more for higher frequencies than lower.
and noise
214|_ SECTION V Ear

3. Tectorial Membrane b. An otolithic membrane, which is made up of a gelatinous mass


It consists o f gelatinous matrix w i t h delicate fibers. It overlies and on the top, the crystals of calcium carbonate called otoliths
0

the organ of Corti. The shearing force between the hair cells and or otoconia. Jhe
Q
linear, gravitational and head tilt movements
tectorial membrane produces the stimulus to hair cells. cause displacement of otolithic membrane and thus stimulate
the hair cells which lie in different planes.
| PHYSIOLOGY OF EQUILIBRIUM
Vestibular Nerve
Vestibular system - Peripheral receptors • Vestibular or Scarpa's ganglion is situated in the lateral part of
They are t w o types: the internal acoustic meatus.
• Cristae: They are located in the ampullated ends o f t h e three • The distal process of bipolar cells innervate the sensory epi-
semicircular canals.°These receptors respond t o a n g u l a r ac- 0 thelium o f t h e labyrinth while its central process aggregate t o
celeration and deceleration. 0 form the vestibular nerve.
• Maculae: They are located in otolith organs (i.e. utricle and
saccule). Macula o f t h e utricle lies in its floor in a horizontal
0 Central Vestibular Connections
plane. Macula of saccule lies in its medial wall in a vertical plane. • The fibers of vestibular nerve end in vestibular nuclei and some
They sense position of head in response to gravity and linear go t o the cerebellum directly.
acceleration. 0

S t r u c t u r e of a C r i s t a Ampulla of
semicircular duct
It has 2 types of hair cells: Cupula
• Type 1: Cells are flask-shaped w i t h a single large cup-like
nerve terminal, contains by polar cells.
• Type 2: Cell are cylindrical w i t h m u l t i p l e nerve terminals. Kinocilium
From the upper surface of each cell, project a single
Hair cells
hair, the kinocilium and a number of other cilia.
Crista ampullaris
S t r u c t u r e of M a c u l a

A macula consists mainly of t w o parts:


a. A sensory neuroepithelium, made up of type I and type II cells, Fig. 17.2: Structure o f ampullary end of semicircular duct.
similar t o those in the crista.

i
-


1. Otoacoustic emissions arise from: [AIIMS May 05; Al 10] 9. Movement of stapes causes vibration in: [DNB 02]
a. Inner hair cells a. Scala media b. Scala tympani
b. Outer hair cell c. Scala vestibuli d. Semicircular canal
c. Both inner and outer hair cells 10. Bones of middle ear are responsible for which of the
d. Organ of Corti following? [MH03]
2. Hair cell of organ of Corti supported by: [PGI Nov 09] a. Amplification of sound intensity
a. Onodi cells b. Deitercell b. Reduction of sound intensity
c. Hensencell d. Bullar cell c. Protecting the inner ear
e. Heller cell d. Reduction of impedance to sound transmission
3. Stapedial reflex is mediated by: [JIPMER 92] 11. Semicircular canals are stimulated by: [MP2000]
a. Vand VII nerves b. Vand VIII nerves a. Gravity b. Linear acceleration
c. VII and VI nerves d. VII and VIII nerves c. Rotation d. Sound
4. The cough response caused while cleaning the ear canal 12. Horizontal semicircular canal responds to: [UP 2005]
is mediated by stimulation of: [AIIMS Nov 02] a. Horizontal acceleration b. Rotational acceleration
a. The V Cranial nerve c. Gravity d. Anteroposterior acceleration
b. Innervation of external ear canal by C,, C 2 13. Angular movements are sensed by: [JIPMER 93]
c. The X Cranial nerve a. Cochlea b. Saccule
d. Branches of the VII Cranial nerve c. Utricle d. Semicircular canals
5. Perilymph contains: 14. All are correctly matched except: [TN07]
a. Na' b. K' a. Otolith - Made up of uric acid crystals
c. M g " d. CI b. Position of otolith - Changes with head position
6. Endolymph in the inner ear: [AIIMS May 09] c. Otoliths - Stretch receptors
a. Is a fiIterate of blood serum d. Otolith organs - Stimulated by gravity and linear accele-
b. Is secreted by stria vascularis ration
c. Is secreted by basilar membrane 15. Impedance matching occurs d/t-
d. Is secreted by hair cells a. Difference of surface are of tympanic membrane and foot
7. All ofthe following are concerned with auditory pathway plate
except: [Al 95] b. Semicircular canal fluid
a. Trapezoid body b. Medial geniculate body c. Utricle and Saccule
c. Genu of internal capsule d. Lateral lemniscus d. None
8. Higher auditory center determine: [AIIMS May 09] 16. Primary receptor cells of hearing-
a. Sound frequency b. Loudness a. Supporting cell b. Tectorial membrane
c. Speech discrimination d. Sound localization c. Tunnel of corti d. Hair cell

EXPLANATIONS AND REFERENCES


Ans. is b i.e. Outer hair cells
Ref. Dhingra 5th/ed, pp 32-33; 6/e p 27-28; Scott Brown 7th/ed Vol. 3 p 3277; Current Otolaryngology 2nd/ed pp 605-606

Otoacoustic Emissions (OAEs)

• Otoacoustic emissions are acoustic signals emitted from the cochlea t o middle ear and into the external ear canal where they
are recorded.
• They are low intensity sounds probably generated by acute mechanical contraction o f t h e outer hair cells
• They are produced either spontaneously or in response to the acoustic stimuli
• Sound produced by outer hair cells moves in reverse direction viz:
• Outer hair cell —> Basilar membrane —> Perilymph —> Oval w i n d o w —> Ossicles —> Tympanic membrne —> Ear canal.
• OAEs are present w h e n outer hajr cells are healthy.
• Abscence o f OAE indicate structurally damaged or non-functional outer hair cells. 0

• They do n o t dissappear in eighth nerve pathology as cochlear hair cells are normal. 0

Uses

a. OAEs are used as a screening test of hearing in neonates and t o test hearing in uncooperative or mentally challenged individuals
after sedation. Sedation does not interfere w i t h OAEs.
216|_ SECTION V Ear

b. They help t o distinguish cochlear from retrocochlear hearing loss as they are absent in cochlear but not in retrocochlear leisons.
c. OAEs are also useful in diagnosing retrocochlear pathology, especially auditory neuropathy.

E x t r a Edgje •

Type of O A E - Broadly OAEs are of 2 types

r 3
Spontaneous Evoked i.e. elicited by a sound stimulus
• Present in healthy normal hearing persons where hearing loss is Depending on the sound stimulus that used to elicit them, they are
not more man 30 dB of 2 types.
• May be absent in 5 0 % of normal persons
• Limited clinical use

Evoked by clicks
I Two tones are simultaneously presented to the cochlea to produce distortion
A series of click stimuli are presented to products
the ear & response recorded This tonal response is not present in single eliciting stimulus and is referred
TEOAEs are recorded as an amplitude/ to as a 'distortion'
time plot of the acoustic wave form DPOAE are attributed to the nonlinearity of motion of the outer hair cells,
'TEOAE is greater than 20 dB sound pressure particularly at low stimulus level
level (SPL); can be recorded from newborn DPOAEs are typically represented in a magnitude/frequency plot which is
while responses from children and adults called DP-gram.
range between 10 and 15 dB SPL Such a plot correlates with functional integrity of the cochlea
TEOAEs can be altered in presence of The two tones used to elicit the response are (F, & F ) called as primary
2

contralateral stimulation tones and have different but related frequencies; F is usually 1.2 times the
2

frequency of F,

2. Ans. is b , & c i.e. Deiter cell & Hensen cell


Ref. PL Dhingra 5th/ed p 16; Logan & Turner 1 Oth/ed pp 231 -32; Maqbool 11 th/ed p 18
"Supporting cell: Deiter's cells 0
are situated between the outer hair cells & provide support t o the later. Cells of Hensen 0
lie outside
t h e Deiter's cells"- PL Dhingra 5 /16 ,h

" H e n s e n cell: One of the supporting cells in the organ of Corti, immediately t o the outer side of the cells' of Deiters (www.drugs.
0

com/dict/hensen-cell)
Organ of Corti: components
Organ of Corti is the sense o r g a n o f hearing and situated o n basilar membrane
0

• Tunnel o f Corti: Formed by inner & outer rods


• Hair cells: They are important receptor cells of hearing & transducer sound energy into the electrical energy 0

- Inner hair cells: Form a single row and more important in the transmission of auditory impulse
- Outer hair cells: Arranged into three or four rows
• Supporting cell: Deiter cell & Hensen c e l l 0

• Tectorial membrane: Consists of gelatinous matrix w i t h delicate fibers


Hellar cells are ethmoidal air cell that extend along the medial roof of the maxillary sinus. They may exist as a discrete cells or the may
open into maxillary sinus or infundibulum 0
- Cummings Otolaryngology 4 /1162
,h

"Onodi cells are posterior & lateral extension of posterior ethmoidal cells. These cells can surround the optic nerve tract &put the nerve 0

at risk during surgery" 1


-Cummings Otolaryngology 4< /1'162
h

3. Ans. is d i.e. VII and VIII nerves Ref. Dhingra 5th/edp 30,6/e,p 24-25 Current Olotaryngology 2nd/edp 602.

Acoustic Reflex/Stapedial Reflex

It is based o n the fact that a loud sound, 70-100 dB above the threshold of hearing of particular ear, causes bilateral contraction o f
t h e stapedial muscle which can be detected by tympanometry. This can be seen both in the stimulating ear (ipsilateral ear) and in
the non stimulating ear (contralateral ear).

I/L = Ipsilateral C/L = Contralateral


CHAPTER 17 Physiology of Ear and Hearing J 217
Afferent Superior olivary Efferent
complex
Pathway A r
Pathway

Cochlear l/L ear C/L ear



nucleus motor neuron of motor neuron of
the facial nerve the facial nerve
J
Auditory nerve VII nerve I VII nerve I
(VIII nerve)
1
| Nn. to stapedius |
| Nn,tostapedius |
Cochlea
| Middle ear ]
I Middle ear I
Sound Middle ear
stimulus

Also know

Stapedial reflex can be used


• As an objective m e t h o d t o test hearing in infants and young children
• To detect malingers - as stapedial reflex is positive in people faking hearing loss
• To detect
Lesion Test response
a: Cochlear pathology Presence of stapedial reflex at lower intensities i.e. 40-60 dB means recruitment is positive i.e. cochlear pathology
b. VIII nerve lesion It eliminates both the contralateral and ipsilateral acoustic reflex when the affected ear is stimulated. But contralateral
and ipsilateral reflex are present if normal side is stimulated
c. VII nerve lesion Absence of stapedial reflex in presence of normal hearing indicates lesion of Vllth nerve proximal to the nerve of
stapedius
d. Brainstem lesion if ipsilateral reflex is present but contralateral reflex is absent, it indicates lesion in the area of crossed pathway in
the brainstem

4. Ans. is c i.e. The X cranial nerve Ref. BDC Wed, p Vol. 3, pi 85


"Irritation ofthe auriclar branches ofthe vagus in the external ear (by ear wax, syring, etc.) may reflexly cause cough, vomitting, or even
death due to sudden cardiac inhibition."
Auricular branch o f t h e vagus nerve is also known as Arnold's nerve or Alderman's nerve.

Also k n o w

Similarly irritation of recurrent laryngeal nerve by enlarged lymph nodes in children may also produce a persistent cough.
5. Ans. is a i.e. N a +
Ref. Dhingra 5th/ed p 12;6/ep 10; Current Otolaryngology 2nd/edp 583
6. Ans. is b. i.e. is secreted by stria vacularis [Ref. Dhingra 5/e p 12]
There are 2 main fluids in the inner ear.

Perilymph Endolymph
• Fills the space between the bony and membranous labyrinth i.e. it is • Fills the entire membranous labyrinth i.e. found in scala media
found in scala vestibuli and scala tympani
• Resembles extracellular fluid/CSF • Resembles intracellular fluid
• Rich in Na ions • Rich in K ions.

.• Marginal cells in the striae vascularis actively pump potassium into the membranous chamber to mantain the difference in the sodium
& potassium concentration.
• The difference in the chemical composition between perilymph & endolymph provides the electrochemical energy which powers the activities
of sensory cells.
218|_ SECTION V Ear

Also know

Striae vascularis forms the outer wall of scala media and sits w i t h i n spiral ligament.
Ans. is c i.e. G e n u of internal capsule Ref. Guyton Physiology 7 7 th/ed pp 657-658; Dhingra 5th/edp 17,6/e p 13
• Organ o f Corti is the sense organ of hearing and is situated on the basilar membrane.
• The actual sensory receptors in the organ o f Corti are t w o specialized types of nerve cells k/a hair cells.
• Inner hair cells - single layer
• Outer hair c e l l s - 3 or 4 layers
• The nerve fibers stimulated by the hair cells lead t o the'spiral ganglion of Corti'which lies in the modiolus (center) o f t h e cochlea.
• The spiral ganglia has bipolar cells - Its dendrites innervate the hair cells whereas the axons f o r m the cochlear division of CN
VIII and end in the dorsal and ventral cochlear nuclei located in the upper part of medulla.
Tecrorial membrane
Inner hair cells
Outer hair cells

Basilar fiber
Spiral ganglion
Cochlear nerve

Auditory Pathway

• All the fibres f r o m the dorsal and ventral cochlear nuclei synapse (k/a Trapezoid body) and the 2° order neurons pass mainly
t o the opposite side o f t h e brainstem t o terminate in the superior olivary nucleus
• A few 2° order fibers pass t o the superior olivary nucleus on the same side
• From t h e superior olivary nucleus, the auditory pathway passes upward t h r o u g h the lateral lemniscus
• Some o f t h e fibers terminate in the nucleus o f t h e lateral leminiscus but many bypass this nucleus and travel on t o the 'inferior
colliculus', where all or almost all fibers d o synapse.
• From there, the pathway passes t o the medial geniculate nucleus, where all fibers synapse.
• Finally t h e pathway proceeds b y w a y of auditory radiation t o auditory cortex, located mainly in the superior gyrus of temporal
bone.

In short pathway of hearing is:


Organ of Corti
I
Spiral ganglion
I •
Dorsal/ventral cochlear nuclei
I
(Trapezoid body)
i
Superior olivary nucleus
i
Lateral lemniscus
i
Inferior colliculus

Medial geniculate body


i
Auditory cortex.

-
CHAPTER 17 Physiology of Ear and Hearing J 219

Crossing over o f t h e fibers f r o m both sides occurs at 3 places in


the brain.
a. At the trapezoid body
b. In the commissure between the 2 nuclei of lateral lemnisci
c. In the commissure connecting the t w o inferior colliculi
Ans. is d i.e. Sound localization
Ref. Scott Brown 7th/ed Vol. 3p3144; Ganong 23rd/ed p213
I. Cochlear nuclear complex:
It is subdivided into

Dorsal cochlear nucleus Ventral cochlear nucleus


I I
It receives high frequency fibers It receives low frequency fibers
This shows t h a t t h e frequency distribution o f fibers in the
auditory nerve, is maintained across the cochlear nuclei as a
tonotopic map of neurons responding t o progressively higher
frequency f r o m one side t o other.
Dorsal cochlear nucleus is i m p o r t a n t in d e t e r m i n i n g w h a t
sounds are.
II. Superior olivary complex - It receives binaural information
and as such forms the first part o f the ascending auditory
pathway where major binaural comparisons are made.
It also helps in sound localization t o some extent and neurons
t h a t detect differences in sound intensity are located in the
lateral superior olive (LSO).
III. Inferior colliculus - It helps in understanding 'what a sound
is'.
• It provides a basis for recognizing patterns in sounds.
• It also has neurons which are involved in sound localization
• It is also involved in a u d i t o r y m o t o r responses. For ex-
a m p l e — t u r n i n g the head or moving the eyes in respond
t o sound.
IV. Auditory cortex - main function is sound localization"
"Sound localization is markedly disrupted by lesions ofthe auditory
cortex." -Ganong 23/ed p. 213
Ans. is c i.e. Scala vestibuli
[Ref. Dhingra Sth/edp 11&18,6/e 9, Tuli Ist/edp 18] Medulla
Friends, first let us quickly revise w h a t is scala vestibuli, t y m p a n i
and scala media. Trapezoid

Scala Vestibuli Auditory nervous pathways


It is filled w i t h perilymph.
• At one end it is closed by footplate of stapes and at the other end it continues w i t h scala t y m p a n i via the helicotrema. (So
obviously even w i t h o u t any further knowledge we can say, by c o m m o n sense that movement of stapes will cause movements
in scale vestibuli).

Scala Tympani

It is filled w i t h perilymph.
• At one end it is closed by secondary tympanic membrane of round w i n d o w and at one end it is connected t o scala vestibuli
via the helicotrema.
• It is also connected t o subarachnoid space by aqueduct of cochlea
Diagrammatic representation of perilymphatic system. CSF passes into scala t y m p a n i t h r o u g h aqueduct of cochlea.
220 [_ SECTION V Ear

Stapes Helicotrema


Scala media
(endolymph)

Subarachnoid
space
C.S.P

Scala Media

It is membranous cochlea and is filled w i t h endolymph


It appears triangular on cross section and has three walls formed by:
a. The basilar membrane, which supports the organ of Corti
b. The Reissner membrane, which separates it from the scala vestibuli
c. The stria vascularis, which contains vascular epithelium and is concerned w i t h secretion of endolymph.

Reissner's Scala vestibuli | Sound signal in the environment |


membrane

Cochlear duct
I
-
Collected by pinna
Passes through external auditory canal

vascularis T
| Strikes the tympanic membrane


T
Vibrations pass through the ossicles
I
Transmitted to stapes footplate
-
J'
Stapes footplate causes movement of perilymph in
scala vestibuli and in scala tympani via helicotrema
Osseous Basilar
spiral lamina membrane
This causes movement in basilar membrane

Scala tympani T
Stimulates the hair cells of organ of Corti

Section through cochlea to show scala media T


Hair cells act as transducers and convert
mechanical energy into electrical energy
(cochlear duct), scala vestibule and scala tympani
Now let us see the mechanism of hearing I
This electric impulse passes to auditory nerve"]

T
I Traverse the auditory pathway
I "
I

Scala vestibuli | Auditory cortex |


Cochlear of duct

Vestibule- Scala tympani


Oval •

window
Stapes

Round window
Physiology of ear
CHAPTER 17 Physiology of Ear and Hearing J 221
10, Ans. is d i.e. Reduction of impedance to sound transmission
Ref. Scott Brown 7th/ed, Vol. 3 p 3181; Dhingra Sth/ed p 18,6/e p 14,15,16
We have discussed in detail the mechanism of hearing in the previous question.
Broadly hearing mechanism can be divided into:
• Mechanical conduction of sound (done by middle ear).
• Transduction of mechanical energy into electrical impulses (done by sensory system of cochlea)
• Conduction of electrical impulse t o brain (i.e. auditory pathway)

Detailed Information
i. Conduction of sound:
It is done mainly by middle ear. Middle ear not just simply conducts the sound but converts sound of great amplitude & less
force to that of less amplitude and greater force. This function o f t h e middle ear is called as impedance matching mechanism
or the transformer action.
• This f u n c t i o n of middle ear is accomplished by

r
Lever action of ossicles T
Hydraulic action of tympanic membrane 1
Curved Membrane effect

Handle malleus is 1.3 times


longer than long process of
T
The area of tympanic membrane is much larger than
area of footplate of stapes -21:1.
T
I The movement of tympanic membrane is
j more at periphery than at center where handle
incus providing a mechanical But the effective vibratory area of tympanic membrane j of malleus is attached, this also provides some
advantage of 1.3. is only 2/3rd . So effective ratio is reduced to 14:1. leverage.
The product of area ratio and lever ratio is (14 * 1.3) = 18:1
(transformer ratio)

Transduction of mechanical energy to electrical impulse:


Movements o f t h e stapes footplate causes vibrations in scala vestibuli followed by scala tympani and is transmitted to the cochlear
fluids which brings about movement o f t h e basilar membrane.This sets up shearing force between the tectorial membrane and
the hair cells. The distortion of hair cells gives rise to electrical nerve impulse.

A sound wave, depending on its frequency, reaches maximum amplitude on a particular place on the basilar membrane, and stimulates that
segment (traveling wave theory of von Bekesy). Higher frequencies are represented in the basal turn of cochlea and the progressively lower-one
toward the apex.

4000

20,000^

8000

Frequency localization in the cochlea. Higher frequencies are


localized in the based t u r n and the progressively decrease toward the apex,
iii. Neural pathway/Auditory pathway:
• Hair cells get innervation f r o m bipolar cells of spiral ganglion. Central axons of these cells collect to form the cochlear nerve.
(Cochlear division of VIII nerve) and end in the cochlear nuclei (the dorsal and ventral on each side of medulla).
• From cochlear nuclei crossed & uncrossed fibers pass via superior olivary nucleus complex —> nucleus of lateral lemniscus
—> inflerior colliculus—> Medial geniculate body & finally reach the auditory cortex o f the temporal lobe.
1 1 . Ans. is c i.e. Rotation
12. Ans. is b i.e. Rotational acceleration
13. Ans. is d i.e. Semicircular canals Ref. Scott Brown 7th/ed p Vol. 3 p 3211, Dhingra 5th/ed p21,6/ep!7-18

-
SECTION V Ear

Vestibular systems includes


I

Semicircular canals/ducts Utricle and sacule (otolith organ)


I Peripheral 4-
Cristae •
receptors Macula

Located in the ampullated end ofthe


semicircular ducts
Respond to
I
Angular/Rotational acceleration and deceleration Linear acceleration/gravity/change in position of head

Ampulla of
semicircular duct
Otoliths
Cupula
Gelatinous
substance
Structure Suboupular
Kinocilium meshwork
Type II hair cell
Hair cells
Supporting cell
Crista ampullaris
Basement
membrane

It is a crest-like mound of connective tissue on


which the sensory epithelial cells lie. -

It has 2 parts
Cells are of 2 types,Type, I (flask-shaped) • A sensory neuroepithelium made of Type I and Type II cells
Type II (cyclindrical) (similar to crista)
From the uppe surface of each cell projects a single hair • An otolithic membrane which is made of gelatinous mass and
k/a kinocilium. on the top has crystals of calcium carbonate called as otolith. 0

When movement ofthe endolymph occurs toward The cilia ofthe hair cells project into the gelatinous layer
kinocilium discharge increases, and when it occurs away The linear, gravitation and head tilt movement causes displacement
from kinocilium discharge decreases ofthe otolithic membrane and thus stimulate the hair cells which
lie in different planes.
This stimulates sensory nerve endings which sends
impulses upward to the brain giving information about
the movement of head

Extra edge

Between the ultricle and saccule:


Utricle
Senses horizontal linear acceleration
Saccule
Senses vertical linear acceleration
Utricle + saccule
Both sense gravity and position of head in space
• Coriolis effect - It is a specific type of angular acceleration (i.e. sensed by semicircular ducts) that causes m o t i o n sickness in
space craft due t o rotation of earth.
• Type I cells correspond t o the inner hair cells of organ of Corti and Type II cells correspond t o the outer hair cells.
14. Ans. is a i.e. Otolith is made of uric acid crystals Ref. Dhingra 5th/edpp 20-22,6/e p 16, Tuli 1st/ed pp 23-24
Macula/otolith organs of utricle and saccule:
.... i i n I -i i n
• It acts like stretch receptor" and gravity acts as a stimulus"
• It has 2 main parts
|
I Sensory neuro epithelium Otolith membrane
1
1 1
Which has Type I (flask-shaped) & Type II (cylindrical) cells: It is a gelatinous mass with embedded otoliths
• Macula/otolith organ respond to Otolith/otoconia is made of crystals of calcium carbonate
a. Linear acceleration 0

b. Gravity 0

c. Change in position of head 0


CHAPTER 17 Physiology of Ear and Hearing J 223
Macula ofthe utricle responds to horizontal linear acceleration and that of saccule respnds to vertical linear acceleration.
15. Ans. is a. i.e. difference of surface area of tympanic membrane and foot plate Ref. Dhingra 6/ed, p 14
The area of tympanic membrane is much larger than area of stapes footplate, the average ratio being 21:1. As the effectice vibratory
area of tympanic membrane is only t w o thirds, the effectives areal ratio is reducted t o 14:1 w h i c h helps in impedance matching/
transformer action.
16. Ans. is d. i.e. hair cells Ref. Dhingra 6/ed, p 13
Hair cells: are i m p o r t a n t receptor cells of hearing and transduce sound energy into electrical energy.

-

-
CHAPTER

Assessment of Hearing Loss

| FUNCTIONAL ASSESSMENT OF HEARING In SNHL - AC > BC -> Low positive Rinne"


In severe SNHL-BC>AC —> False negative Rinne (Due t o t r a n -
scranial transmission of sounds to the normal ear)"
Tuning Fork Tests

They are:
• Qualitative test (as they indicate the type of hearing loss). A negative Rinne with 256, 512 and 1024 Hz shows air bone gap o f =
• Most c o m m o n used t u n i n g fork = 512 Hz. because of - Longer 15,30,45 dB respectively.
t o n e decay and distinct sound.
• Air conduction (AC) is tested by-—placing t u n i n g fork 1/2-1 Weber's Test
inch in front of external acoustic meatus. (It indicates integrity
In this test vibrating t u n i n g fork is placed in the middle of forehead
of tympano-ossicular chain).
and the patient is asked about the lateralization of sound t o left or
• Bone c o n d u c t i o n (BC) is tested by—placing t u n i n g fork on right ear or in which the sound is heard better. It is a very sensitive
mastoid bone or on forehead. (It indicates integrity of inner ear). test" and even less than 5 dB difference in 2 ears hearing level will
be indicated by this test.
Rinne Test
In Conductive Deafness
In this test, AC is compared w i t h BC o f t h e patient. Tuning fork is
• The sound is lateralized t o the deaf ear" and in bilateral c o n -
struck and placed in front o f external auditory meatus. When the ductive loss, sound is lateralized t o the more deaf ear or it is
patient stops hearing, move it on t o the mastoid bone and ask the centrally heard if b o t h ears are equally deaf.
patient if he/she still hears and then reverses the process. The object
In sensorineural hearing loss (SNHL):
is t o f i n d o u t whether the patient hears longer by air or by bone
• The sound is lateralized t o better hearing ear or is heard cen-
conduction. Rinne test will be negative in conductive deafness trally if b o t h ears are equally bad.
of more than 15 dB. Q

In normal ear:
Interpretation is as Follows • No lateralization of sound occurs.
• Normally, AC is 2 times better than BC- positive Rinne 0 • Weber test is quite sensitive as difference of only 3-5 dB hear-
ing level can result in lateralisation. Weber test readily detects
• In conductive deafness - BC > AC —> Negative Rinne"
false Rinne negative.

Test Normal Conductive deafness SN deafness


Rinne AC>BC (Rinne test positive) BC>AC (Rinne test negative) AC> BC (Rinne test positive)
Weber Not lateralized Lateralized to poorer ear Lateralized to better ear
ABC Same as examiner Same as examiner Reduced
Schwa bach Equal Lengthened Shortened
CHAPTER 18 Assessment of Hearing Loss

Remember:
• Ideal t u n i n g fork for testing hearing - 512 Hz.
• Gelle's test - Test for bone conduction.
Positive in normal persons and sensorineural deafness.
Negative in otosclerosis.

• Stenger's test /Chimani-Moos test/Lombard's test/Teel's test—They are t u n i n g fork test for detecting non-organic deafness (ma-
lingering).
• Most sensitive TFT-Weber's test (5 dB difference needed t o laterlize).
• Least sensitive TFT - Schwabach's test.
(TFT = Tuning fork test)
_ j
A b s o l u t e B o n e C o n d u c t i o n Test
Hearing is measured in decibels (dBs) which is a logarithmic scale.
In this test, bone conduction o f t h e patient is tested after occluding
the external auditory meatus and compared w i t h the BC of the Threshold of hearing at 0-1 OdB —> good hearing
examiner if he has a normal hearing. Threshold of hearing at 10-30dB —» mild hearing loss
Threshold of hearing at 30-60 dB —> moderate hearing loss
Conclusion
Threshold of hearing at 60-90 dB —> Severe hearing loss
• If b o t h the patient and examiner hear equally either hearing is
When > 90 dB —> Individual is deaf
normal in patient or there is conductive deafness.
• If patient ceases t o hear before examiner (i.e. ABC is reduced) Audiometry Symbols
- it indicates SNHL
• Blue line for left ear
S c h w a b a c h ' s Test Red line for Right ear (Remember R-R)
• Continuous line for air conduction
Bone conduction of the patient and examiner is compared, but
• Broken line for Bone conduction (Remember B-B)
meatus is not occluded.
Left ear Right ear
Conclusion Air conduction
• Schwabach is shortened in SNHL (Remember 3S). - Unmasked X O
• Schwabach is lengthened in conductive hearing loss. - Masked A

Bone conduction
Gelle's Test > <
- Unmasked
This test is done t o confirm the presence of otospongiosis. In this - Masked • E
test, BC is tested and a t t h e same time Siegle's speculum compresses No response
the air in the meatus. If hearing is reduced, it is normal; but in stapes
f i x a t i o n , bone conduction sound is not affected.
0
Air conduction
- Unmasked P
- Masked
A
Tuning fork tests are not 100% reliable, but are a useful screening
Bone conduction
test. They should be correlated with an audiogram. *<
- Unmasked
- Masked = 1 • j =

OtherTuning Fork Tests


» Stenger's test0

> Teel's test


0
One o f t h e most important yet confusing aspect of hearing test
> Lombard's test 0
is to ensure that the auditory function of each ear is measured
• These tests are done for those patients who feign deafness but
independently. In some situations, a noise is presented to the
actually are normal subjects.
non test ear to prevent it from responding to a signal presented
| AUDIOMETRY to test ear. This is called as masking.

Pure Tone Audiometry Masking is required for air c o n d u c t i o n w h e n ever the dif-
• It is a reliable m e t h o d of testing the hearing acuity and gives ference between the air c o n d u c t i o n presentation level and
information about quantity and quality of hearing loss. non test ear bone c o n d u c t i o n thresholds exceed a p p r o x i -
• Pure tones are given at various frequencies by increasing the mately 40.dB for the lower frequencies and 60 dB for higher
intensity at 5 dB steps and when the patient hears the sound,
frequencies.
it is recorded.
For BC testing, masking should be used w h e n there is any dif-
• Frequencies between 500 Hz and 3000 Hz are important as
these are speech frequencies. 0
ference in the AC and BC threshold.
I SECTION V Ear

Results
• In normal subjects or conductive hearing loss, SDS is 95 - 1 0 0 % .
Audiogram for normal ear, conductive hearing loss and serious
neural hearing loss are given at the back in section on pictorial • In cochlear lesions, SDS is low.
questions. • In retrocochlear lesions, SDS is very poor and roll over phe-
nomenon is present i.e. w i t h increase of intensity, score drops.
Speech Audiometry As poor discrimination score of less than 8 0 % affects t h e ability t o
In this audiometry, recorded spondee words are presented t o the understand speech, hence this test is useful t o find o u t if hearing
ear at various sound pressures. The patient is asked t o write the aid will be useful or not.
words, which are t h e n cross-checked w i t h the list.
Bekesy Audiometry
Speech Reception Threshold (SRT)
• It is a self-recording audiometer in which changes in the inten-
SRTof a person is the m i n i m u m intensity level (in dB) at which 5 0 %
sity and frequency are done automatically by the audiometer.
o f t h e spondee words can be repeated correctly.
• It is outdated these days.
S p e e c h D i s c r i m i n a t i o n S c o r e (SDS) o r O p t i m u m • Various graphs recorded in bekesy audiometer are give in
D i s c r i m i n a t i o n S c o r e (ODS) Fig's. 18.1 A t o 18.1 D

It is t h e m a x i m u m percentage of correct score w h e n phonetically


balanced single syllable words such as pin, day, bus, fun, and rum
are used. •

125 250 500 1000 2000 4000 8000 125 250 500 1000 2000 4000 8000
-20 -20
-10 -10
0 0
10 10
20 20
»

t y \ *«
;

30 30 A'A\A
* .* * V AW
40 40
N V
50 50 O *
60 60

70 70
V
80 80
90 90
100 100
110 110
120 120

Fig. 18.1 Fig. 18.1

Type I tracing - Normal person or conducting hearing loss Type II tracing - Cochlear lesion

The C and I tracings overlap in all frequencies The C and I tracings overlaps till 1000 Hz after
which C tracing drops by 15-20 dB

Test for Recruitment

• Recruitment is an abnormally rapid increase in loudness w i t h increasing sound intensity. Ear which does not hear low intensity sounds
will hear greater intensity sounds as loud or even louder than normal ear. 0
CHAPTER 18 Assessment of Hearing Loss

125 250 500 1000 2000 4000 8000 125 250 500 1000 2000 4000 8000
-20 -20
-10 -10
0 0
10 •» 10
20 20
f. . -
30 30 • * t

c—' •\ K A /•
40 40
\ ft
* V\*
50 50
60 60 \/vV
V V
AA A

70 70
80 80
90 90
100 100
110 110
120 120

Fig. 18.1 Fig. 18.1:

Type III tracing - retrocochlear lesion/neural lesion Type IV tracing - In acoustic nerve lesion or non organic
The C tracing drops t o > 20 dB below Type I tracing hearing loss The C and I tracings never overlaps

Left ear Right ear Left ear Right ear • In positive recruitment, ladder pattern becomes horizontal at
higher intensity.

Disadvantage
• Difference between the hearing thresholds of the t w o ears
should be atleast 25 dB.
• One ear should be normal.

T o n e D e c a y T e s t (or N e r v e F a t i g u e Test)

• Measure o f nerve f a t i g u e and is used t o detect retrocochlear


0

lesions. 0

• N o r m a l l y , a p e r s o n can hear a t o n e c o n t i n u o u s l y f o r
60 seconds.
• In nerve fatigue, he stops hearing earlier.
• A decay of more than 25 dB is diagnostic of retrocochlear lesions.

Impedance Audiometry

• Principle: It measures the change in the impedance o f t h e


middle ear system at the level o f t h e tympanic membrane as
a result of changes in the air pressure in the external auditory
A. Recruitment absent B. Recruitment present canal.

Fig. 1 8 . 2 : Recruitment phenomenon Uses:


• This p h e n o m e n o n of recruitment is seen in cochlear type of To differentiate between conductive and sensorineural hear-
SNHL, e.g. Meniere's disease and presbycusis . In normal per-
0 0
ing loss.
sons and conductive hearing loss, the test is negative. Differential diagnosis of conductive hearing loss.
Measurement of middle ear pressure and evaluation of Eusta-
A B L B Test of F o w l e r (Alternate Binaural L o u d n e s s
chian tube function.
Balancing) To differentiate between cochlear and retrocochlear type of
• In this test, a tone is alternatively played into normal and deaf sensorineural hearing loss.
ear, until the sound is heard equally in both ears. To identify the site of lesion in facial paralysis.
228 T SECTION V Ear

It consists of t y m p a n o m e t r y and acoustic reflex measurement. A C O U S T I C R E F L E X / S T A P E D I A L R E F L E X (Fig. 1 8 . 4 )


• T y m p a n o m e t r y : It is the measure o f change in t h e i m p e d -
Stapes footplate rocks in the oval w i n d o w and stiffens the ossicular
ance o f t h e m i d d l e ear system at t h e plane o f t y m p a n i c
chain and tympanic membrane.
m e m b r a n e - in response t o pressure changes in the external
a u d i t o r y canal. Method of performing Acoustic Reflex Test:
• Acoustic reflex: It is the measure of change in the impedance
Sound intensity used: 70-105 dB SL (sensation level).
of the middle ear system in response t o loud stimulus.

A tone of 85 dB sound pressure level (SPL) and low frequency


(220 Hz) is used for impedance audiometry in adults whereas in
infants and neonates a higher frequency probe tone (660 or 1000
Hz) must be used.

C u r v e s o f I m p e d e n c e A u d i o m e t r y (Fig. 18.3)

Muscle

Stapedius muscle
Stapes

Fig. 18.4: Acoustic reflex pathways

C a u s e s of A b s e n t Stapedial Reflex

Afferent Efferent
pathway pathway
Middle ear Otosclerosis VII nerve Facial palsy
diseases diseases
Ossicular Ramsay Hunt
-200 -100 0 +100 +200
discontinuity syndrome
Fig. 18.3: Curves of Impedence Audiometry Atelectasis
Colchlea/VIII Severe SNHL Stapedius Poststape-
Type of curve Condition nerve/superior Acoustic neuroma muscle dectomy
olivary c o m - Multiple sclerosis involve- Myasthenia
A curve Normal
plex lesion ment gravis
(Normal peak height and Eustachian tube obstruction
pressure). Bera: (Brainstem Evoked Response Audiometry)/ABR
As curve" Otosclerosis" (Auditory Biainston Response)/lndications
(It is also a variant of Tumors of middle ear
• For detection of deafness in difficult t o test cases like infants",
normal tympanogram but Fixed malleus syndrome
mentally retarted or malingers.
may be shallow) Tympanosclerosis
• For assessment of the nature of deafness (conductive or sen-
Ad curve Ossicular discontinuity sorineural) 0

(Variant of normal with high Post stapedectomy • For i d e n t i f i c a t i o n o f t h e site o f lesion in r e t r o c o c h l e a r


peak) Monometric ear drum pathologies"
B curve Fluid in middle ear" • To study t h e m a t u r i t y o f t h e CNS in n e w b o r n s , objective
(Flat or dome-shaped curve)" Secretory otitis media" assessment of brain-death.
Indicating lack of compliance Tympanic membrane perforation" • For assessing prognosis in a comatosed patients.
Grommet in ear" • To diagnose brainstem pathology example multiple sclerosis
C curve Retracted tympanic membrane or pontine t u m o r
(negative peak pressure) Faulty function of Eustachian tube/ • Unlike pure tone audiometry, BERA does not require subjective
Eustachian tube obstruction patient response.
;

-
CHAPTER 18 Assessment of Hearing Loss J 229
Principle 1 OTOACOUSTIC EMISSIONS

It is noninvasive technique t o find the integrity of central auditory Otoacoustic emissions (OAE) are low-intensity sounds, w h i c h are
pathway t h r o u g h the VIII nerve, pons and midbrain. produced by movements of the outer hair cells of t h e cochlea.
They are produced spontaneously and in response t o t h e acoustic
is converted into Electrical Passes from Cochlea
Sound in stimuli, OAE are picked u p by a miniature microphone, which is
impulse
the Cochlea placed snugly in the EAC. Absence of OAE indicates disorders o f
(Various wave< Auditory
forms) Produces outer hair cells.
cortex
This non-invasive objective test can diagnose damage t o ther
These waves are studied for latency, amplitude and morphology. outer hair cells due t o acoustic trauma and ototoxic drugs. It aids
Out o f t h e following waves generated the 1 st, 3rd and 5th waves in the assessment of hearing in infants. Sedation does n o t interfere
are most stable and t h e ones which are studies w i t h OAE.
The OAE travels t h r o u g h basilar m e m b r a n e , p e r i l y m p h ,
According t o Dhingra 4th/ed p 29 and Scott Browns 7th ed p 3283
oval w i n d o w , ossicles, t y m p a n i c m e m b r a n e a n d ear canal.
• Wave I = E = Distal part of eight nerve OAE are present in nerve hearing loss as the outer hair cells are
• Wave II = E = Proximal part of eight nerve normal.
• Wave III = C = Cochlear nucleus/Lower pons
Uses
• Wave IV = 0 = Superior olivary complex
• Screening test of hearing in neonates, uncooperative or m e n -
• WaveV=L = Lateral leminiscus — U p p e r pons tally challenged patients.
• WaveVI-VII = Inferior colliculus • Distinguish between cochlear (acoustic trauma and ototoxic
drugs) and retrocochlear hearing losses (auditory neuropathy).

i
-

• •

•3
-



SECTION V Ear

QUESTIONS
All are tunning fork test except: [UP 02/DNB 02] 13. A 38-year-old gentleman reports of decreased hearing
a. Schwabacktest b. Grant's test in the right ear for the last two years. On testing with a
c. Rinne'stest d. Weber's test 512Hz tuning fork the Rinne's test without masking is
Tuning fork of 512 FPS is used to test the hearing because negative on the right ear and positive on the left ear.
it is: [Karn. 06] With the Weber's test the tone is perceived as louder in
a. Better heard b. Better felt the left ear. The most likely patient has: [AIIMS Nov 02]
c. Produces over tones d. Not heard a. Right conductive hearing loss
Gelle's test is done in: [JIPMER 98] b. Right sensorineural hearing loss
a. Senile deafness b. Traumatic deafness c. Left sensorineural hearing loss
c. Otosclerosis d. Serous otitis media d. Left conductive hearing loss
4. Which one ofthe following test is used to detect malinge- 14. A m i d d l e - a g e d w o m e n p r e s e n t e d w i t h right s i d e d
ring? [TN07] hearing loss, Rinne's test shows positive result on left side
a. Stenger's test b. Bunge's test and negative result on right side Weber's test showed
c. Weber's test d Rinne's test lateralization to left side, diagnosis is: [AIIMS June 00]
Rinne's test is negative in: [AIIMS Nov 94] a. Right sided conductive deafness
a. Sensorineural deafness b. Acoustic neuroma b. Right sided sensorineural deafness
c. Tympanosclerosis d. Meniere's disease c. Left sided sensorineural deafness
Rinne's test negative is seen in: [JIPMER 92] d. Left sided conductive deafness
a. Presbycusis b. CSOM 15. One man had 30 dB deafness in left ear with Weber test
c. Labyrinthitis d Meniere's disease showing more sound in left ear and BC (Bone conduction]
Rinne's test is negative if minimum deafness is: more on left side and normal hearing in right ear, his test
[SRMC02] can be summarized as:
a. 15-20dB b. 25-30 dB a. Weber's test—left lateralized; Rinne test—right positive,
c. 35-40 dB d. 15-50dB BC>AC on left side
Positive Rinne test is seen in: [JIPMER91] b. Weber's test—right lateralized; Rinne test—left positive,
a. Otosclerosis b. CSOM AC>BCon right side
c. Wax impacted ear d. Presbycusis c. Weber's test—left lateralized; Rinne test—false positive on
9. Rinne's test is positive in: [AIIMS 91] right side, BC>AC on left side
a. Chronic suppurative otitis media d. Weber's left lateralized; Rinne test—equivocal, BC>AC on
b. Normal individual right side
c. Waxinear 16. A 38-year-old male presented with a suspected diagnosis
d. Otomycosis of suppurate labyrinthitis. A positive Rinne's test and
10. Weber test is best elicited by: [Al 02] positive fistula test was recorded on initial examination.
a. Placing t h e t u n i n g fork on t h e mastoid process and T h e patient refused treatment, a n d r e t u r n e d to the
comparing the bone conduction o f t h e patient with that emergency department after 2 weeks complaining of
o f t h e examiner deafness in the affected ear. On examination, fistula test
b. Placing the tunning fork on the vertex of the skull and was observed be negative. What is the likely expected
determining the effect of gently occluding the audtitory finding on repeating the Rinne test. [Al 09]
canal on the thereshold of low frequencies a. True positive Rinne's test b. False positive Rinne's test
c. Placing the t u n i n g fork on t h e mastoid process and c. True negative Rinne's test d. False negative Rinne's test
comparing the bone conduction in the patient 17. In p u r e t o n e a u d i o g r a m t h e s y m b o l X is u s e d to
d. Placing the tuning fork on the forehead and asking him t o mark: [JIPMER 02]
report in which ear he hears it better. a. Air conduction in right ear
11. In the right middle ear pathology, Weber's test will be: b. Air conduction in left ear
[AI04] c. Bone conduction in right ear
a. Normal b. Centralized d. No change in air conduction in right ear
c. Lateralized to right side d. Lateralized to left side 18. The " O " sign in a pure tone audiogram indicates:
12. Weber's test in conductive deafness: [CUPGEE 96] [AP2005]
a. Sound louder in normal ear a. Air conduction of right ear
b. Sound louder in diseased ear b. Air conduction of left ear
c. Bone conduction of right ear
c. Heard with equal intensity in both ears
d. Bone conduction of left ear
d. Inconclusive test
CHAPTER 18 Assessment of Hearing Loss

19. Tone decay test is done for: [ManipalOl] c. Free field audiometry
a. Cochlear deafness b. Neural deafness d. Behavioral audiometry
c. Middle ear problem d. Otosclerosis 32. In infant most sensitive audiometric screening is:
20. Al I are subjective tests for audiometry except: [PGI Dec 98]
a. Tone decay b. Impedance audiometry a. Electrocochleography

c. Speech audiometry d. Pure tone audiometry b. BERA
21. Impedance audiometry is for pathology of: [UP 04] c. Cortical evoked response audiometry
a. External ear b. Middle ear d. Tympanometry
33. To d i s t i n g u i s h b e t w e e n cochlear a n d post c o c h l e a r
c. Mastoid air cell d. Inner ear
damage test done is: [PGI Dec 97]
22. Impedance audiometry is done using frequency probe
a. Brainsterm evoked response audiometry
of: [Delhi 07]
b. Impedence audiometry
a. 220 Hz b. 550 Hz
c. Pure tone audiometry
c. 440 Hz d. 1000 Hz
d. Auditory cochlear potential
23. A lady has B/L hearing loss since 4 years which worsened
34. In normal adult wave v is generated from:
d u r i n g p r e g n a n c y . Type of i m p e d a n c e a u d i t o m e t r y
[J and K05, Delhi 08]
graph will be: [AIIMS May 07; Nov 06]
a. Cochlear nucleus
a. Ad b. As
b. Superior olivary complex
c. B d. C
c. Lateral lemniscus
24. Flat tympanogram is seen in: [PGI 00]
d. Inferior colliculus
a. ASOM b. Otosclerosis
35. Test of detecting damage to chochlea
c. Serous otitis media d. Ossicular chain disruption [MH PGM GET Jan 05; MH 00]
25. B-type tympanogram is seen in: [Bihar 04] a. Caloric test b. Weber test
a. Serous otitis media b. Ossicular discontinuity c. Rinne's test d. ABC test
c. Otosclerosis d. All o f t h e above 36. Threshold for bone conduction is normal and that for air
26. Flat and dome-shaped graph in tympanogram is found conduction is increased in disease of: [AP 96]
in: [RJ03] a. Middle ear b. Inner ear
a. Otosclerosis b. Ossicular discontinuity c. Cochlear nerve d. Temporal lobe
c. TM perforation d. Middle ear fluid 37. In monaural diplacusis the lesion is in the: [AP91]
27. In osteogenesis imperfecta, the tympanogram is: a. Cochlea b. Auditary nerve
[DNB 03] c. Brainstem d. Cerebrum .
a. Flat b. Noncompliance 38. Impedance audiometry is for pathology of:
c. High-compliance d. Low-compliance [NEET Pattern]
28. A young man presents with an accident leading to loss a. External ear b. Middle ear
of hearing in right ear. On otoscopic examination, the c. Mastoid air cell d. Inner cell
tympanic membrane was intact pure tone audiometry 39. Stapedial reflex is mediated by: [NEETPattern]
that shows an air-bone gap of 55 dB in the right with a. V a n d VII nerves
normal cochlear reserve. Which of the following will be b. V a n d VIII nerves
the like tympanometry finding: [Al 09] c. VII and VI nerves
a. As type tympanogram b. Ad type tympanogram d. VII and VIII nerves
c. B type tympanogram d. C type tympanogram 40. Vestibular evoked myogenic potential (VEMP) detects
29. High frequency audiometry is used in: [AIIMS May 09] lesion of: [AIIMS May 2012]
a. Otosclerosis b. Ototoxicity [AIIMS Nov 12] a. Cochlear nerve
c. Non-organic hearing loss d. Meniere's disease b. Superior vestibular nerve
30. W h i c h is t h e best test for s c r e e n i n g of the auditory c. Inferior vestibular nerve
function of neonates? [AIIMS May, Nov 12] d. Inflammatory myopathy
a. Pure tone audiometry 4 1 . In electrocochleography: [AIIMS May 2012]
b. Stapedial reflex a. It measures middle ear latency
c. Otoacoustic emissions (OAE) b. Outer hair cells are mainly responsible for cochlear
d Brainstem evoked auditory response microphonics and summation potential
31. Which is the investigation of choice in assessing hearing c. Summation potential is a compound of synchronus auditory
loss in neonates? [AIIMS May 11 ] nerve potential
a. Impedance audiometry d. Total AP represents endocochlear receptor potential to an
b. Brainstem evoked response audiometry (BERA) external auditory stimulus
232 J SECTION V Ear

EXPLANATIONS AND REFERENCES

1. Ans. is b i.e. Grant's test Ref. Dhingra 5th/edp 26,6th/ed p 22-23


Tuning Fork tests include.
• Rinne's test - Compares air conduction o f t h e ear w i t h bone conduction
• Weber test
• Absolute bone conduction test - Here bone conduction o f t h e patient is tested after occluding the meatus and then compared
w i t h BC o f t h e examiner
• Schwabach test - Here also BC o f t h e patient is compared w i t h the BC of a normal hearing person but meatus is n o t occluded.
• Bing test - It is a test of BC and examines the effect of occlusion of ear canal on hearing (i.e. external meatus is occluded and
released alternatively)
• Gelle's test - It is also a test of BC and examines the effect of increased air pressure in ear canal on hearing.

O t h e r T u n i n g Fork Tests

• Stenger test
• Teel's test For detecting malingering
• Lombard's test

Also Know

• To test air conduction - A vibrating t u n i n g fork is placed vertically about 2 cms away f r o m the opening of external auditory
meatus.
• Sound waves are transmitted: From tympanic membrane —> middle ear —> ossicles of inner ear.
• Thus AC tests both conducting mechanism and cochlea
• To test bone conduction - Vibrating t u n i n g fork is placed on the mastoid bone.
• Cochlea is stimulated directly by vibrations conducted t h r o u g h the skull
• Hence - BC is a measure of cochlear function only.
2. Ans. is a i.e. better heard Ref. Tuli Ist/ed p 28
Tuning fork tests can be done w i t h t u n i n g forks of different frequencies like 128, 256, 512, 1024, 2018 and 4096 Hz but most
c o m m o n l y used is 512 Hz because
• 'Tests are done with various tuning forks, but 572 Hz is the most commonly used as it has longer tone decay and sound is quite distinct
from ambient noise."
• Forks of lower frequencies produce a sense of bone vibration while those of higher frequency have a shorter decay t i m e and
therefore not c o m m o n l y used
3. Ans. is c i.e. osteosclerosis Ref. Dhingra Sth/ed p 27,6th/ed p 22
Gelle's test was once a popular test to find out stapes fixation in otosclerosis, but now it has been superseded by tympanometry.
In this test, bone conduction is tested and at the same time Siegel's speculum compresses the air in the meatus.

Principle
Normally, w h e n air pressure is increased in ear canal (by Siegel's speculum)

Normally, when air pressure is increased


in ear canal (by Siegel's speculum)

I Pushes
rusnes the
ine tympanic
tympanic membrane
memprane inward
inwara |

| Ossicles are pushed inward |


. 1 ,
1 t intralabyrinthine pressure |

. ^
Immobilty of basilar membrane ,
I i ed hearing i.e. test is positive [

But if ossicular chain is fixed or disrupted, no such phenomenon occurs i.e. test is negative.
CHAPTER 18 Assessment of Hearing Loss

Gelle's test is positive: In normal individuals, SNHL.


Gelle's test is negative: In case of fixed ossicular chain (otosclerosis) or if ossicular chain is disconnected.

Ans. is a i.e. Stenger's test [Ref. Dhingra 5th/ed p 42,6th/edp 37; Tuli Ist/ed p31]
Malingering/Nonorganic hearing loss (also called pseudohypacusis)
• Ocassionally patients wilfully or subconsciously exaggerate their hearing loss.

• This is functional hearing loss or pseudohypacusis or malingering
- The signs in the test behavior that suggest functional component include:
a. Inconsistent responses
b. Significant differences between the threshold obtained using ascending and descending administration of test stimuli
c. A discrepancy of > 8 dB between the SRT (speech reception threshold) and the pure tone average of 500-2000 Hz
d. Positive Stenger test

S t e n g e r Test
• It is used t o identify unilateral or asymmetrical functional hearing loss. It is based on the concept t h a t w h e n b o t h ears are
stimulated simultaneously by a t o n e equal in frequency and phase, t h e auditory percept is lateralized t o the ear w i t h better
hearing.
• If speech stimulus is used in Strengertest it is k/a Speech Stenger test or modified Stenger test.
• Other objective tests which can diagnose functional involvement are:
- acoustic reflexes: Pt saying hearing loss b u t normal acoustic reflex indicates NOHL
- auditory brainstem response
- otoacoustic emission

Also Know
Other t u n i n g fork tests which can be used t o detect malingering b u t are now outdated are:
• Teel's test
• Lombard's test
• Chamini-Moos test
• Gault test

, Ans. is c i.e. Tympanosclerosis


, Ans. is b i.e. CSOM
, Ans. is a i.e. 1 5 - 2 0 dB Ref. Dhingra 5th/edp 26, 6th/edp 22
As discussed in the text in Rinnies test—air conduction o f t h e ear is compared w i t h its bone conduction.

Principle and Result


• In normal individuals => AC > BC i.e. Rinne test is positive
• In case o f conductive deafness air conduction is decreased. Hence BC > AC i.e. Rinne test negative
• In U/L SNHL => air conduction is normal whereas BC is further decreased. Hence AC > BC i.e. Rinne test positive
• But in case of severe U/L SNHL - patients do not perceive any sound of t u n i n g fork by air conduction but respond t o bone
conduction testing which is in reality due t o transcranial transmission of sound f r o m the normal ear.
Hence there is false negative Rinne test (BC > AC).

1 Result Inference Seen in

Positive Air conduction> Bone conduction • Normal individuals


. SNHL

Negative Bone conduction> Air conduction • Conductive deafness

False negative • Severe SNHL

NOTE

' A negative Rinne test indicates a minimum air bone gap of 15-20 dB (Ans 7)

Now lets see Qs 5 and 6


Q.5 says Rinne's test is negative in -
We know negative Rinne test is seen in case of conductive deafness. Amongst the options given, only tympanosclerosis is a cause
for conductive deafness.
Again in Q.6 - only CSOM causes conductive deafness.
SECTION V Ear

8. Ans. is d i.e. Presbycusis


9. Ans. is b i.e. Normal individual Ref. Dhingra5th/edp26,41, 6th/edp22
Rinne's test is positive i.e. air conduction > bone conduction
It is seen in:
a. Normal individuals (Ans 9)
b. In case of sensorineural hearing loss (SNHL)
Amongst the options given in Q.8 only presbyacusis causes SNHL and therefore gives positive Rinne test

Presbycusis: It is sensorineural hearing loss associated with physiological aging process in the ear. It manifests at 65 years of age.

10. Ans. is d i.e. Placing the tuning fork on the forehead and asking him to report in which ear he hears better
Ref. Dhingra Sth/ed p 26, 6th/edp22

Method of testing
Rinne's test Placing the tuning fork on mastoid process and bringing it beside the meatus, when patient stops hearing it on mastoid
Weber's test Placing the tuning fork on forehead and asking him to report in which ear he hears better
Absolute bone Placing the tuning fork on mastoid process and comparing the bone conduction of the patient with that of examiner after
conduction occluding the meatus
Schwabach's test Test same as absolute bone conduction but meatus is not occluded

11. Ans. is c i.e. Lateralized to right side Ref. Dhingra 5th/ed p 26


12. Ans is b i.e. Sound louder in diseased ear
As discussed in the text in Weber's Test
• In normal Individuals - No lateralization of sound occurs as Bone conduction of both ears in normal and equal
• In conductive deafness - Lateralization of sound occurs t o the diseased ear (Ans 12)
• In SNHL - Lateralization of sound occurs t o the better ear.

It is a very sensitive test and even less than 5 dB difference in 2 ears hearing level can be indicated.

Also k n o w
BingTest Ref. Dhingra 6th/edp 22
It is a test of bone conduction and examines the effect of occlusion of ear canal on the hearing. A vibrating t u n i n g fork is placed
o n the mastoid while the examiner alternately closes and opens the ear canal by pressing on the tragus inward.
• Positive in normal and SNHL i.e. hears louder w h e n ear canal is occluded and softer w h e n ear canal is open.
• Negative in conductive hearing loss - i.e. no change

13. Ans. is b i.e. Right sensorineural hearing loss


14. Ans. is b i.e. Right sided sensorineural deafness Ref. Dhingra 5th/edp27 Table 4.1,6th/edp 22, Table 4.1

Rinne's Test

Negative on right side means either there is:


• Conductive deafness of Right side or
• Severe SNHL on right side (leading t o false negative Rinne test)
To differentiate between the 2 conditions: Let us see the result of Weber's test:
• Patient is complaining of decreased hearing in right ear and Weber's test is lateralized t o left ear (as stated in the question) i.e.
to the better ear.
• As discussed in the text: Weber's test is lateralized t o the better ear in case of SNHL.
So, diagnosis is right sided severe SNHL.

Remember: If Rinne's test is negative and Weber's test shows lateralization toward healthy side, it indicates severe SNHL

-
CHAPTER 18 Assessment of Hearing Loss J235

15. Ans. is a i.e. Weber's test - left lateralized; Rinne's right positive; BC > AC on left side
Ref. Dhingra 5th/ed p 26,27,6th/edp 22
Let us analyze each information provided in the question.
• This man has deafness of 30 dB in left ear.
• Weber's test is lateralized t o left ear i.e. deaf ear which means deafness is conductive type. (As in conductive deafness - Weber's
test is lateralized t o poorer ear).
This means Rinne test should be negative on left side (as in conductive deafness - Rinne test is negative). Ruling out options "b"
and "d".
In the question it is given hearing is normal on right side, so Rinne test will be positive on right side (because in case of normal
hearing - Rinne test is positive).
In t h e question itself it is given, bone conduction is more on left side.
So option "a" is correct i.e.:
Weber's test - left lateralized, Rinne test - right positive and BC>AC on left side.
16. Ans. is d i.e. False negative Rinne's test Ref. Dhingra 5th/ed p 26,6th/ed p 22
In t h e above question: Patient was suffering f r o m suppurative labyrinthitis which was not treated and led to total loss o f hearing
i.e. severe SNHL.
In severe SNHL: Rinne's test is false negative and because labyrinth is dead. Fistula test is negative.
False negative Rinne test as explained earlier occurs in case of severe SNHL because patient does not perceive any sound of t u n i n g
fork by air conduction but responds t o bone conduction due t o intracranial transmission of sound f r o m opposite healthy ear.

Fistula Test

The basis of this test is t o induce nystagmus by producing pressure changes in the external canal which are then transmitted t o the
labyrinth. Stimulation o f t h e labyrinth results in nystagmus and vertigo. Normally the test is negative because the pressure changes
in the EAC cannot be transmitted t o the labyrinth.
Positive Fistula Test is seen in:
• Erosion of horizontal semicircular canal (Cholesteatoma or fenestration operation)
• Abnormal opening in oval w i n d o w (post stapedectomy fistula) or round w i n d o w (rupture of round window).

A positive fistula test also implies that the labyrinth is still functioning.
• False-negative fistula test: Dead labyrinth, cholesteatoma covering site of fistula.
• False-positive fistula test (Positive fistula test w i t h o u t Fistula): Congenital syphilis, 2 5 % cases of Meniere's disease (Hennebert's sign.)

17. Ans. is b i.e. Air conduction in left ear Ref. Dhingra 5th/ed p 34 Fig 5.1; 6th/ed p 30,51; Current Otolaryngology 2nd/ed p 597
18. Ans. is a i.e. Air conduction in the right ear.
Symbols used in a u d i t o m e t r y — S e e thepreceeding text
19. Ans. is b i.e. Neural deafness Ref. Dhingra 4th/ed p 28,5th/ed p 31
Tone decay test is a measure of nerve fatigue (i.e. neural deafness) and is used t o detect retrocochlear lesions. A decay of more than
25 dB is diagnostic of retro cochlear lesion.
Method of doing the test and principle: A continuous tone of 5 dB above threshold in 500 Hz and 2000 Hz is given t o the ear and
person should be able t o hear it for 60 sec. The result is expressed as dB by which intensity has t o be increased so that the patient
car - hear the sound for 60 sec. If tone decay of >25 dB is present, it indicates retrocochlear leison e.g.—acoustic neuroma.
20. Ans. is b. i.e. Impedance audiometry Ref. Dhingra Sth/ed p 29; 6th/ed p 24; Tuli Ist/ed pp 31-35

Tests of hearing can be broadly classified as

I.
Subjective tests Objective tests
Where response depends on the Where response depends on automatically
patients e.g.: recorded
• Tuning fork tests • Impedance audiometry
• Pure tone audiometry • Stapedial/acoustic reflex
• Speech audiometry • Evoked response audiometry
- Speech recepton threshold - Electro cochleography
- Speech discrimination score -Auditory brainstem response
• Bekesy audiometry • Otoacoustic emissions
• ABLB test of foul ear
• Short increment—sensitivity index

• Tone decay test
236 ]_ SECTION V Ear

21. Ans. is b i.e. Middle ear Ref. Dhingra 5th/ed p 29; 6th/ed p 24; Current Otolaryngology 2nd/ed p 601
22. Ans. is a i.e. 220 Hz

Impedance Audiometry •

1. It is an objective test for h e a r i n g


0

2. It is very useful in children for assessing the hearing loss. 0

3. It consists of:

Tympanometry Acoustic reflex measurement


Principle: It is based on the amount of sound reflected back from the tympanic membrane when an 85dB Discussed in detail earlier
sound pressure level (SPL) low frequency (220 Hz) probe tone is introduced into the sealed ear canal, and
0

pressure in the canal is varied. By changing the pressures in the sealed auditory canal and then measuring
the reflected sound energy, it is possible to find the compliance or stiffness ofthe tympano-ossicular system
and thus find the healthy or diseased status ofthe middle ear

.•. impendance audiometry is used for diagnosing middle ear pathology.

• Various types of tympanograms are discussed in next question.


• For infants and neonates, tympanograms obtained using a 220 Hz probe may erroneously appear normal.
.'. a higher frequency probe tone (660 or 1000 Hz) must be used - (Current Otolaryngology 2nd/edp 601)

23 Ans is b i e As Ref. Dhingra Sth/ed p30, 97-98,6th/ed p87

Bilateral hearing loss


+
Occurring in afemale Allthesefeaturesindicatetowardotosclerosisasthecauseofdeafness
+
25 years of age

Accentuation of hearing loss during pregnancy



24. Ans. is a and c i.e. ASOM; and Serous otitis media
25. Ans. is a i.e. Serous otitis media
26. Ans is d i.e. Middle ear fluid Ref. Dhingra 6th/ed p 24, Sth/ed pp 30,69,71
• Flat (or dome-shaped) tympanogram is type B curve of tympanogram which is seen in case of fluid in middle ear.
• Fluid (i.e. pus) is seen in case of ASOM and sterile non purulent effusion is seen in case of serous otitis media. So, in b o t h these
conditions flat tympanogram/type B tympanogram will be seen.
27. Ans. is d i.e. Low compliance Ref. Scott Brown 7th/ed Vol. 3, p 3458, Dhingra 5th/ed pp 30,97,6th/ed p 87
This is a very interesting question - They are testing our knowledge as well as application ability.
Osteogenesis imperfecta is associated w i t h otosclerosis.
Vander Hoeve syndrome is a triad of:
Otosclerosis

Osteogenesis
imperfecta A Blue
sclera

Hence - Indirectly they are seen asking the type of tympanogram in otosclerosis.
Types of Tympanogram
Type A Normal tympanogram
Type AS Low-compliance tympanogram—Seen in case of fixation of ossicles i.e. otosclerosis or malleus fixation
Type Ad High-compliance tympanogram—seen in case of ossicular discontinuityy or laxed tympanic membrane
Type B Flat/Dome-shaped tympanogram—seen in case of middle ear fluid or thick tympanic membrane
Type C Negative compliance tympanogram—seen in case of retracted tympanic membrane

28. Ans. is b i.e. Ad type tympanogram Ref. Dhingra Sth/ed pp 27,30 and 34
This is also a very interesting question:
• The question says. Pure tone audiometry shows an air bone gap of 55 dB in the right ear w i t h normal cochlear reserve.

-
CHAPTER 18 Assessment of Hearing Loss

The air-bone gap in pure tone audiometry is a measure of total conductive deafness.
Hence - it means there is a conductive deafness of 55 dB in the right ear.
Next the question says - Patient has intact tympanic membrane so we have t o lookfor a cause of this 55 dB conductive deafness.

Average hearing loss seen in different lesions of conductive apparatus Ref. Dhingra Sth/ed p 34
1. Complete obstruction of ear canal 30 dB
2. Perforation of tympanic membrane 10-40dB
3. Ossicular interruption with intact drum 54 dB
4. Ossicular interruption with perforation 10-25 dB
5. Closure of oval window 60 dB

As is clear f r o m above table - w i t h tympanic membrane intact and a hearing loss of 55 dB is seen if ossicular chain is disrupted.
Hence- it is a case of ossicular discontinuity.
Tympanogram seen in ossicular discontinuity is a high compliance t y m p a n o g r a m i.e. Ad tympanogram.
29. Ans. is b i.e. ototoxicity Ref: Scoff Brown 7th/ed Vol. 3 p 3572; Audiology by Ross J. Roeser, Michael Valente,
Holly Hosford-Dunn 2nd/ed p 242; Ototoxicity by Peter S. Roland, John A. Rutka p 154
Conventional audiometry tests frequencies between 0.25 kHz-8 kHz, whereas high frequency audiometry tests in the region of
8 kHz-20 kHz. Some environmental factors, such as ototoxic medication like aminoglycosides and noise exposure, appear to be
more detrimental t o high frequency sensitivity than t o that of mid or low frequencies. Therefore, high frequency audiometry is
an effective m e t h o d of monitoring losses that are suspected to have been caused by these factors. It is also effective in detecting
t h e auditory sensitivity changes that occur w i t h aging

Ototoxic drugs like aminoglycosides typically affecting higher-frequency hearing first and progressing to lower frequencies.

• Otoacoustic emissions (OAE) are more sensitive at detecting auditory dysfunction than high-frequency pure fone audiometry.
OAEs also have the added advantage of being practical at bedside and do not require a soundproof room.
• Distortion product OAEs are more sensitive than transient evoked OAEs for the detection of early signs of ototoxicity.
30. Ans. is c i.e. Otoacoustic emissions
31. Ans. is b i.e. Brainstem evoked response audiometry (BERA)
32. Ans. is b i.e. BERA Ref. Logan and Turner's 10th/ed p 251,410-415; Anirban Biswas Clinical Audio Vestibulometry 3rd/ed p 68,99;
Dhingra 4th/ed p117; 5th/ed p 32,132
Methods of hearing assessment in infants and children... Dhingra 5th/131,6th/ed p!18

• Neonatal Screnning Procedure:'


- Arousal response cradule
- OAE

• Behavior Observation Audiometry
- Moro's reflex
- Cochleopapebral reflex
- Cessation reflex
• Distraction techinques
• Condition techniques
- Visual reinforcement audiometry •

- Play audiometry
• Objective tests
- ABR/BERA .
- OAE •

- Impedence audiometry

S c r e e n i n g t h e N e w b o r n for H e a r i n g l o s s •

- Screening newborn for hearing loss leads t o earlier detection and intervention in patients w i t h congenital hearing impairment.
- Early intervention can improve speech and language development, and educational achievement in affected patients.
238 T SECTION V Ear

Screening Tests for hearing loss

r
Auditory Brainstem Response (ABR/BERA) Otoacoustic Emissions (OAE)
• Measures the summation of action potential from the • OAEs are low intensity sounds produced by outer produced
8th cranial nerve (cochlear nerve) to the inferior colliculus of by outer hair cells of a normal cochlea.
the midbrain in response to a click stimulus. • OAEs are present when outer hair cells are healthy and are
• Non-inasive technique to find the integrity of central absent when they are damaged.
auditory pathways through the 8th cranial nerve, pons and • Help to test the function of cochlea.
midbrain. • Do not disappear in 8th nerve pathology as the cochlear
• Delayed or absent waves suggest a neurologic or cochlear hair cells are normal.
deficit. • OAE helps to distinguish cochlear from retrocochlear hearing
• Takes more time loss
• Child should be sedated • Takes less time
• Sedation not required.

Thus b o t h ABR (or BERA) and OAE are used as a screening proceduce in infants and newborn for hearing loss. In Qs 31 and 32 there
is no d o u b t as only BERA is given in options. Q 30 asks the screening procedure of choice and both OAE and BERA are given in
options. Nowhere it is mentioned which is the initial screening procedure of choice. I have chosen OAE as the answer for Q 30 as
suggested by following lines of COGDT 3/e, p 625
"As such, OAE testing is commonly used in newborn hearing screening because of its speed and non invasive nature." ...COGDT 3/e, p 625

• Infants w h o fail a screening test, require additional audiologic evaluation.
• In infants <8 months of age should be referred for diagnostic ABR/BERA.
• Hearing loss should be confirmed by visual reinforcement audiometry (VRA), when VRA can be performed reliably (>8 months
of age).
• Visual reinforcement audiometry (VRA) is the gold standard/Investigation of choice for hearing assessment for non-verbal
children.
• If VRA is n o t given in options or in infants <8 months (even is premature infants) and mortally retarded p e o p l e : Investigation
of choice is BERA.

33. Ans. is a i.e. Brainstem evoked response audiometry Ref. Dhingra 5th/ed p 32,6th/ed p 26
BERA testing objectively assesses the neural synchrony o f t h e auditory system f r o m the level of eight nerve to the midbrain
.-. It is very useful in distinguishing between cochlear pathology and retrocochlear pathology for SNHL
Cochlear SNHL-occurs due t o damage of hair cells mainly
Retrocochlear SNHL- occurs due t o lesion of Vlllth nerve or its central connection. Hence - BERA can diagnose a retrocochlear
pathology.
34. Ans. is c i.e. Lateral lemniscus Ref. Dhingra Sth/ed p 32,6th/ed p 27, Scott Brown 7th/ed Vol. 3 p 3283
• In normal persons during BERA testing, 7 waves are produced in the first 10 milli second
• The 1 st, 3rd and 5th wave are the most stable and are used in measurements
• These waves are studied for:
- Absolute latency
- I nter wave latency (between wave I and V)
- Amplitude

Site of Origin of Waves

Wave I Distal part of (Eighth nerve) E


Wave Proximal part of Eighth cranial nerve E
Wave Cochlear nucleus C
ave IV Superior olivary complex O
Wave V Lateral leminiscus L
Wave VI Inferior colliculus I
and VII
Mnemonic EE COLI
35. Ans. is d i.e. ABC test Ref. Dhingra Sth/ed p 26,6th/ed p 22
• As discussed earlier absolute bone conduction test is a t u n i n g fork test in which bone conduction of the patient is compared
w i t h BC o f t h e examiner after occluding the external auditory meatus of b o t h patient and examiner
• Bone conduction is a measure of cochlear function.
Hence, ABC test is used to detect damage t o cochlea.
CHAPTER18 Assessment of Hearing Loss J 239

Rinne's t e s t "
— M e a s u r e air c o n d u c t i o n
Weber's t e s t - •

Caloric t e s t - assesses v e s t i b u l a r f u n c t i o n
36. A n s . is a i.e. M i d d l e e a r Ref Dhingra Sth/ed p 26
T h r e s h o l d f o r air c o n d u c t i o n is increased (i.e. low frequency sounds are not heardwell) w h e r e a s t h a t o f b o n e c o n d u c t i o n in n o r m a l i.e.
B o n e c o n d u c t i o n > air c o n d u c t i o n w h i c h is seen in c o n d u c t i v e deafness. C o n d u c t i v e deafness occurs i n lesions o f e i t h e r e x t e r n a l
ear, t y m p a n i c m e m b r a n e , m i d d l e ear o r ossicles u p t o s t a p e d i o - v e s t i b u l a r j o i n t .
37. A n s . is a . i.e. C o c h l e a Ref. Tuli Ist/ed p 114
S u b j e c t i v e f e e l i n g o f diplacusis, hyperacusis o r f u l l n e s s i n t h e ear occurs in c o c h e a r p a t h o l o g y o r cohlear, s e n s o r i n e u r a l h e a r i n g
loss (SNHL).
Differences between Cochlear a n d Retrocochlear SNHL

Cochlear SNHL Retrocochlear SNHL


Hair cells are damaged mainly Lesion is of Vllth nerve or its central connections
Recruitment is present Recruitment is absent
No significant tone decay Tone decay is significant
5ISI (Short increment sensitivity index) is positive SISI is negative
Bekesy shows no gap between I and C tracing (Type II) Bekesy shows wide gap between i and C tracings (Type I
Speech discrimination is not highly impaired (SDS is low) and roll over Speech discrimination is highly impaired (SDS very poor) and roll over
p h e n o m e n o n is not present p h e n o m e n o n is present
Subjective feeling of diplacusis, hyperacusis or fullness in the ear No such sensation or feeling

SDS - Speech Discrimination Score

It is t h e m a x i m u m p e r c e n t a g e o f c o r r e c t score w h e n p h o n e t i c a l l y b a l a n c e d s i n g l e syllable w o r d s such as p i n , day, bus are u s e d .

Results

• In n o r m a l s u b j e c t o r c o n d u c t i v e h e a r i n g loss, SDS is 9 5 - 1 0 0 %
• In c o c h l e a r lesions SDS is l o w
• I n r e t r o c o c h l e a r lesions, SDS is v e r y p o o r a n d roll o v e r p h e n o m e n o n is p r e s e n t ( w h i c h m e a n s w i t h increase o f i n t e n s i t y , d r o p
o f score occurs)
38. A n s . is b i.e. M i d d l e e a r Ref. Dhingra 6th/edp 24
I m p e d e n c e a u t i o m e t r y is used t o find t h e h e a l t h o r diseased status o f m i d d l e ear.
39. A n s . is d i.e. VII a n d VIII n e r v e s Ref. Dhingra 6th/edp 25

Superior

olivary
complex

VII nerve
• •

40. A n s . is c i.e. i n f e r i o r v e s t i b u l a r n e r v e Ref. Current Otolaryngology 3rd/edp 64 7

Vestibular Evoked Myogenic Potentials

• T h e v e s t i b u l a r e v o k e d m y o g e n i c p o t e n t i a l (VEMP) are s h o r t l a t e n c y e l e c t r o m y o g r a m s t h a t are e v o k e d b y acostic s t i m u l i i n h i g h


i n t e n s i t y a n d r e c o r d e d f r o m surface e l e c t r o d e s o v e r t h e t o n i c a l l y c o n t r a c t e d s t e r n o c l e i d o m a s t o i d m u s c l e .
• T h e o r i g i n o f VEMP is t h e saccule.
• T h e r e s p o n s e p a t h w a y consists o f :
240 T SECTION V Ear

• Saccule; Inferior Vestibular Nerve, Lateral Vestibular Nucleus, Lateral Vestibulospinal Tract and Sternocleidomastoid muscle.
• The test provides diagnostic information about saccular and/or inferior vestibular nerve function.
• An intact middle ear is required for the response quality.
Waveform o f t h e response
The VEMP waveform is characterised by a
. Wave I-positive peak at 13-15 (p13)
• Wave II - negative peak at 21 -24 ms (p23)
Peak t o peak a m p l i t u d e of p i 3-23 is measured and asymmetries between the right and left side is noted (by calculating asymmetry
ratio AR) Abnormal AR is seen a case of
• Saccular hydrops (AR > 36%)
• Vestibular schwannoma oriqnating f r o m inferior vestibular nerve.
• Vestibular neuronitis
• Superior canal dehiscence syndrome.
4 1 . Ans. is b i.e. Outer hair cells are mainly responsible for cochlear microphonix and summation potential.
Ref. Mohan Bansal, Text book of Diseases of ENT Ist/ed p 24,25 and 145
Electrocochleography (EcoG) measures electrical potentials, which arise in cochlea and CN VIII in response to auditory stimuli within
first 5 milliseconds. It consists of following three types of responses
1. Cochlear microphonics
2. Summating potentials
3. Action potential of 8th nerve
Endocochlear potential, cochlear microphonics (CM) and summating potential (SP) are f r o m cochlea while the c o m p o u n d action
potential (AP) is f r o m the cochlear nerve fibers. Both CM and SP are receptor potentials similar t o other sensory end-organs.
• Endocochlear Potential: This resting potential o f + 8 0 mV direct current (DC) is recorded from scala media. This energy source
for cochlear transduction is generated from stria vascularis by Na+/K+ -ATPase p u m p . Endolymph has high K+ concentration. It
acts as a battery and helps in driving the current through the hair cells w h e n they move after exposure t o any sound stimulus.
• Cochlear Microphonics: Cochlear microphonics (CM) is an alternating current (AC) potential. Basilar membrane moves in
response t o sound stimulus. Changes occur in electrical resistance at the tips of OHC. Flow of K+ t h r o u g h the outer hair cells
produces voltage fluctuations and called CM.
Cochlear microphonics is absent in the part of cochlea where the outer hair cells are damaged.
• Summating Potential: Summating potential (SP) is a DC potential, which may be either negative or positive. It is produced by
hair cells. It follows the"envelop" of stimulating sound and is superimposed on cochlear nerve action poten-tial.This is a rectified
derivative of sound signal. Probably it arises f r o m IHCs w i t h a small contribution f r o m OHCs.
Summating potential of cochlea helps in the diagnosis of Meniere's diseases.
• C o m p o u n d (Auditory Nerve) Action Potential: It is the neural discharge of auditory nerve. It follows all or none phenomena
so has all or none response t o auditory nerve fibers. Each nerve fiber has o p t i m u m stimulus frequency for which the threshold
is lowest. A m p l i t u d e increases while latency decreases w i t h intensity over 40-50 dB range. The following features differentiate
itfromCMandSP:
a. No gradation b. Latency
c. Propagation d. Post-response refractory period

Method

The recording electrode (a thin needle) is placed on the prom-ontory through the tympanic membrane. The test can be done under
local anesthesia however children and anxious uncoop-erative adults need sedation or general anesthesia, which has no effect on
EcoG responses.
Uses
The main application of ECOG is t o help determine if a patient has Meniere disease. The amplitude o f t h e summating potential
(reflecting activity o f t h e hair cells) is compared w i t h that o f t h e c o m p o u n d action potential (reflecting whole nerve activity). If the
ratio is larger than normal (0.3-0.5), it is considered indicative of Meniere disease.The procedure is considered valid only the patient
is symptomatic. Now w i t h this background lets analyse each o p t i o n separately—
Option a - is incorrect as ECOG is a measure of electrical potential of inner ear (and not middle ear latency).
• Option b - is correct as explained above - Outer hair cells are mainly responsible for cochlear microphonics and Summation
Potential.
» Option c - is incorrect as it is not the summating potential b u t the action potential which is a c o m p o u n d of synchronous auditory
nerve potential.
° Option d - is incorrect as Action Potential represents neural potential and not the endocochlear receptor potential which is
represented by components arising f r o m organ of corti that i.e. SP and cochlear microphonics.

mi
Deafness can be of t w o types based o n its etiology viz:
• Congenital
• Acquired
Congenital causes o f deafness

Prenatal Perinatal Postnatal

Genetic Nongenetic Difficult labour • Alport's syndrome

• Waardenburgs syndrome • German measles, Kernicterus • Measles

• Pendred syndrome CMV, Rubella • Meningitis

• Ushers syndrome • Diabetes • Head injury

• Bing siebenmann aplasia • Syphilis • Renal tubular acidosis (type \l. distal)

• Mondini-alexander aplasia • Toxaemia • Loud noise


- Quinine
- Aminoglycosides
- Thalidomide
• Scheibe aplasia • Ototoxic drugs

• Bartters syndrome
• Klippel Feil syndrome
• Treacher Collins syndrome
• MELAS syndrome
• Trisomy 13,15,21
• Cretinism

Classification o f Acquired deafness

Conductive t y p e SN t y p e Mixed Sudden

• External ear • Head injuries • Blast injury • Vascular

Wax/Otomycosis/foreign • viral infections • CSOM • Trauma


bodies/otitis externa/atresia/
infection tumours

Ototoxicity • Mumps, measles • Otosclerosis • Viral infection

• Middle ear • Herpes • Ototoxicity

- Meningitis • Noise trauma • Meningitis

- Congenital defects • Tumours • CVA


Contd...
2421 SECTION V Ear

Contd...
Classification of Acquired deafness
Conductive t y p e SN t y p e Mixed Sudden
- Traumatic • Acoustic neuroma • Functional

- Otitis media (OM) • Meniere's disease


- Nonsuppurative OM • Ototoxicity
- Tuberculosis/syphilis • Presbycusis
- Otosclerosis • Hypertension
- Tumours • CVA
- Eustachian tube block • Diabetes
- E tube catarrh • Hypothyroidism
- Barotrauma • Smoking and alcoholism
• Psychogenic deafness

Deafness can also be classified as conductive type/sensorineural type based on the site of leison
Conductive hearing loss: Any disease process which interferes w i t h the conduction of sound t o reach cochlea causes conductive hearing
loss. The lesion may lie in the external ear tympanic membrane, middle ear, ossicles up t o stapediovestibullar joint.
Sensorineural hearing loss: Results from lesions o f t h e cochlea, Vlllth nerve or central auditory pathways. It may be present at birth
(congenital) or start later in life (acquired).

Differences b e t w e e n Conductive Hearing Loss and SNHL

Conductive hearing loss Sensorineural hearing loss i

• Disease process is limited to external ear and middle ear, including • Disease process is beyond the oval window in the inner ear
foot plate of stapes
• Rinne -ve • Rinne+ve
• Weber lateralised to worse ear • Weber lateralised to better ear
• ABC is normal • ABC shortened
• Pure tone audiometry shows bone air gap • PTA shows no bone air gap
• Low frequencies involved • High frequency hearing loss
• Hearing loss up to 50-60 dB • Hearing loss more than 60 dB
• Speech discrimination score (SDS) is good (95-100%) • Poor SDS in cochlear (low score) and retrocochlear (very low score) leison
• Test for recruitment is -ve • Positive recruitment in cochlear leison
• SISl of 15% • SISl above 60% in cochlear lesion
• No tone decay • A tone decay of 30 dB seen in retrocochlear lesion
• Impedance audiometry is a useful parameter • Impedance audiometry is not of much use
• BERA not of much use • BERA is a very useful diagnostic tool

D i f f e r e n c e s b e t w e e n C o c h l e a r arid R e t r o c o c h l e a r S N H L

Cochlear SNHL Retrocochlear SNHL


• Hair cells are damaged mainly • Lesion is of VIII nerve or its central connections
• Recruitment is present • Recruitment absent
• NO significant tone decay • Tone decay is significant
• SISl is positive • SISl is negative
• Bekesy shows no gap between 1 and C tracings (Type II) • Bekesy shows wide gap between 1 and C tracings (type III)
• Speech discrimination is not highly impaired (SDS) is low) and roll • Speech discrimination is highly impaired (SDS very poor) and roll
over phenomenon is not present over phenomenon is present
• Subjective feeling of displacusis, hyperacusis or fullness in the ear • No such sensation or feeling


•mi
CHAPTER 19 Hearing Loss

QUESTIONS

1. According to WHO classification, for severe degree of 13. A patient has bilateral conductive deafness, tinnitus with
Impairment of hearing is at: [TN 2004] positive family history. The diagnosis is: [AIIMS Nov. 93]
a. 26-40 dB b. 41-55 dB a. Otospongiosis b. Tympanosclerosis
c. 56-70 dB d. 71-91 dB c. Meniere's disease d. B/L otitis media
2. At which level sound is painful: [Jharkhand 2004] 14. Conductive deafness occurs in: [UP 07]
a. 100-120dB b. 80-85dB a. Travelling in aeroplane or ship
c. 60-65dB d. 20-25dB b. Trauma to labyrinth
3. Ear sensitive to: [Jharkhand 2003] c. Stapes abnormal at oval window
a. 500-3500 Hz b. 1000-3000 Hz d. High noise
c. 300-5000 Hz d. 5000-8000 Hz 15. A 55 years old female presents with tinnitus, dizziness
4. After rupture of tympanic membrane the hearing loss and h/o progressive deafness. Differential diagnosis
is: [PGIJune 99] includes all except: [AIIMS Nov. 01]
a. 10-40 dB b. 5-15 dB a. Acoustic neuroma
c. 20dB d. 300 dB b. Endolymphatic hydrops
5. W h i c h of the following conditions causes maximum c. Meningioma
hearing loss ? d. Histiocytosis-X
a. Ossicular disruption with intact tympanic membrane 16. Otitic barotrauma results due to: [PGI June 97]
b. Disruption o f malleus and incus w i t h intact tympanic a. Ascent in air b. Descent in air
membrane c. Linear acceleration d. Sudden acceleration
c. Partial fixation o f t h e stapes footplate 17. All are ototoxic drugs except: [RJ2000]
d. Ottitis media with effusion a. Streptomycin b. Quinine
Commonest cause of hearing loss in children is: c. Diuretics d. Propanolol
[AIIMS Dec. 95] 18. Post head injury, the patient had conductive deafness
a. CSOM b. ASOM and on examination, tympanic membrane was normal
c. Acoustic - neuroma d. Chronic secretory otitis media and mobile. Likely diagnosis is:
Commonest cause of hearing loss in children is: a. Distortion of ossicular chain
[CUPGEE95] b. Haemotympanum
a. Microtia w i t h atresia of external auditory meatus c. EAC sclerosis
b. Trauma d. Otosclerosis
c. Otitis media with effusion 19. All are causes of sensorineural deafness Except: [200 7]
d. Bony canal a. Old age
Commonest cause of deafness is: [AP 97] b. Cochlear otosclerosis
a. Trauma b. Wax c. Loud sound
d. Rupture of tympanic membrane
c. Acute mastoiditis d. Meniere's disease
All of the following can cause hearing loss except: 20. Virus causing acute onset sensorineural deafness:
[PGI Dec. 04]
[UP 2001]
a. Corona virus b. Rubella Measles
a. Measles b. Mumps
c. Mumps d. Adenovirus
c. Chickenpox d. Rubella
e. Rota virus
One of the following factors is not considered a high risk
10. 21. Sensorineural deafness may be feature of all, except:
criteria for development of deafness [Karn 94]
a. Nail-patella syndrome
a. Birth asphyxia
b. Distal renal tubular acidosis
b. Bacterial meningitis -
c. Bartter syndrome
c. Congenital [Torch] infections
d. Alport syndrome
d. Direct hyperbilirubinemia
22. Sensorineural deafness is seen in: [PGI June 02]
Hyperacusis is defined is:
11. [PGI Dec. 97]
Alport's syndrome
a. Hearing of only loud soundy
Pendred's syndrome
b. Normal sounds heard as loud and painful
Treacher-Collins syndrome
c. Completely deaf
Crouzon's disease
d. Ability to hear in noisy surroundings
Michel's aplasia
Conductive hearing loss is seen in all of the following
12. 23. Fluctuating recurring variable sensorineural deafness is
except: [Al 12]
seen in: [APPGI06]
a. Otosclerosis b. Otitis media with effusion
a. Serous otitis media b. Heamotympanum
c. Endolymphatic hydrops d. Suppurative otitis media
c. Perilabyrinthine fistula d. Labrinthine concussion
244 L SECTION V Ear

[PLANATIONS AND REFERENCES


1. Ans. is d i.e. 71 -91 dB Ref. Dhingra 5th/edpg 43,6th/edp 38 (Table 5.9)
Hearing loss and difficulty in hearing speech
WHO classification of degree of hearing Loss and Difficulty in Hearing Speech

Hearing threshold in better ear (average Degree o f i m p a i m e n t Ability t o understand speech


of 5,00,1000, 2000 Hz) (Who classification)
0-25 Not significant No sifinificant difficulty with faint speech
26-40 Mild Difficulty with faint speech
41-55 Moderate Frequent difficulty with normal speech
56-70 Moderately severe Frequency difficulty even with loud speech
71-90 Severe Can undertstand only shouted or amplified speech
Above 91 Profound Usually cannot understand even amplified speech

ALSO KNOW

C o m m o n Terminology

• Hearing loss is impairment of hearing and its severity may vary f r o m mild t o severe or profound.
• Deafness: It is used w h e n there is little or no hearing at all.

W H O Definition o f ' D e a f
The term deaf should be applied only t o those individual whose hearing impairment is so severe that they are unable t o benefit
from any t y p e of amplification.
According to Ministry of social welfare. Govt of India.
Deaf are those in w h o m the sense of hearing is non functional for ordinary purposes of life.
They do not- hear/understand sounds at all even w i t h amplified speech.
The cases included in this category are those w h o have either loss more than 90dB hearing loss in better ear or total hearing
loss in b o t h ears.
Partially hearing are those falling under any one o f t h e following categories
Category Hearing
Mild impairment Between 30-45 dB in better ear
Serious impairment Between 45-60dB in better ear
Severe impairment Between 60-90 dB in better ear

2. Ans. is a i.e. 1 0 0 - 1 2 0 dB


Intensity

Whisper 30 dB
Normal conversation 60 dB
Shout 90 dB

Discomfort of ear 120 dB
Pain in ear 130dB

Since the highest intensity given in the question is 100-120 dB Hence - we are taking it as our correct answer.
3. Ans. is a i.e. 5 0 0 - 3 5 0 0 HZ Ref. Guyton 11 th/ed p 657
Ear best perceives sound in the frequency o f 500 - 5000 HZ.
4. Ans. is a i.e. (10-40dB)
5. Ans. is a i.e. Ossicular disruption with intact tympanic membrane Ref. Dhingra 4th/ed p 30,5th/ed pg 34,6th/edp 29
Average hearing loss seen in different lesions of conductive apparatus:
Condition Average 7hearing loss
Closure of oval window 60 dB
Ossicular inerruption with intact TM 54 dB
Ossicular interruption with perforation 38 dB
Complete obstruction of ear canal 30 dB
TM perforation 10-40 dB


-

mi
CHAPTER 19 Hearing Loss

4. Ans. is clear i.e. rupture of tympanic membrane causes a loss of between 10-40 dB depending on the size of perforation.
Coming on t o ans 5.
• Hearing loss in otitis media w i t h effusion:
- Mean = 20-30dB. ... Internet search
• Hearing loss in ossicular fixation:
- Malleus fixation = 10-25dB - Dhingra 5th/ed pg 34,6th/ed p 29
- Stapes fixation = upto50dB ... Internet search
So it is clear - ossicular disruption w i t h intact tympanic membrane causes m a x i m u m hearing loss. Option 'b' (of Ans 5) can give
rise t o some confusion but o p t i o n b is disruption of malleus and incus (with stapes intact) whereas in option 'a'of ans 5, malleus,
incus and stapes are all disrupted which definitely will lead to more hearing loss.
6. Ans. is d i.e. Chronic secretory otits media Ref. Ghai 6th/edp 334; Ghai 7th/ed p 333; Current Otolaryngology 2nd/ed pg 658
7. Ans. is c i.e. Otitis media with effusion
"The mostcommon cause of conductive deafness in children is otitis media with effusion, which is typically of mild to moderate severity."
... Ghai 6th/edp334; Ghai 7th/edpg 333
Otitis media w i t h effusion / glue ear / chronic serous or secretory otitis media -

"It is the most c o m m o n cause of hearing loss in children in the developd world and has peak incidence at 2 and 5 years o f age" -
Current otolaryngology 2nd/ed pg 658
For more details on Secretory otitis media or Otitis media with effusion, see Chapter: Diseases of middle ear in this book.
8. Ans. is b i.e. Wax Ref. Dhingra 3rd/edp 68
Searching
9. Ans. is c i.e. Chicken pox Ref. OP Ghai 7/e p. 333
"The most common postnatal cause of acquired SNHL is meningitis, while the most common prenatal cause is intrauterine
infection (eg TORCH infections, syphilis, mumps, measles)". - OP Ghai 7th/ed p 333
10. A n s . i s N o n e Ref. Dhingra 6th/ed p 7 75-7 16and 5/e pg-128,130 Ghai 6/e, p335, table (13.1)
Risk factors for hearing loss in children (Recommendations of Joint commitee on infant hearing).
• Apgar score of 0-4 at 1 min or 0-6 at 5 mins (indicating birth asphyxia) 0

• Birth w e i g h t <1.5 k g 0

• Bilirubin (bilirubin> 2 0 m g % damages cochlear n u c l e i ) 0

• Craniofacial anomalies
• Drugs / ototoxic medications
• Family history of hearing loss
• Prenatal TORCH infection
• Bacterial meningitis
• Mechanical ventilation for > 5 days
• Stigmata or other findings associated w i t h a syndrome k n o w n t o cause SNHL/ conductive hearing loss.
11. Ans. is b i.e. Normal sounds heard as loud and painful Ref. Logan Turner 7Oth/edp 237; Maqbool 11 th/ed p 31

Hyperacusis Sensation of discomfort or pain on exposure t o normal sounds. Seen in injury t o nerve t o stapedius and in case
of congenital syphilis (Hennebert sign)
Displacusis Condition where same tone is heard as notes of different pitch in either ear
Paracusis willisii Condition where patient hears a sound better in presence of background noise. Seen in case of otosclerosis

ALSO KNOW

Tullio phenomenon condition where the subject gets attacks of vertigo / dizziness by loud sounds. It occurs in patients w i t h
labyrinthine fistula or those w h o have undergone fenestration operation.

12. Ans. c i.e. Endolymphatic hydrops Ref. 6th/edp 30 Table 5.7 and 5.2
Endolymphatic hydrops i.e meniers disease leads t o SNHL and not conductive hearing loss. All the rest can lead t o conductive
heaving loss
Congenital causes of conductive hearing loss
• Mental atresia
• Fixation of stapes footplate
SECTION V Ear

• Fixation o f malleus head


• Ossicular discontinuity
• Congenital cholesteatoma
Acquired causes of conductive hearing loss

External ear Any obstruction in the ear canal, e.g. wax, foreign body, furncle, acute inflammatory swelling, being or malignant
t u m o r or atresia of canal.
Middle ear (a) Perforation of tympanic membrane, traumatic or infective
(b) Fluid in the middle ear, e.g. acute otitis media, serous otitis media or h a e m o t y m p a n u m
(c) Mass in middle ear, e.g. benign or malignant t u m o u r
(d) Disruption of ossicles, e.g. trauma to ossicular chain, chronic suppurative otitis media, cholesteatoma
(e) Fixation o f ossicles, e.g. otosclerosis, tympanosclerosis, adhesive otitis media
(f) Eustachian tube blockage, e.g. retracted tympanic membrane, serous otitis media.
13. Ans. is a i.e. Otospongiosis Ref.Dhingra6th/edp30,875th/edp34,35,97
Conductive deafness means the disease process leading t o deafness is limited t o external ear tympanic membrane, middle ear
including the footplate of stapes.

Bilateral conductive deafness rules out meniere's disease (as it presents with SNHL).
Amongst the remaining three options, positive family history is seen mainly in case of otosclerosis (Otospongiosis) so it is our
answer.

14. Ans. is a, c and d i.e. Travelling in Aeroplane and Ship; Stapes abnormal at oval window; and High noise
Ref. Dhingra 5th/edpg 74,6th/edp66,30,33,35
Otitic Barotrauma or travelling in aeroplane/ship leads t o conductive hearing loss b u t sensorineural type of loss may also be
seen. ... Dhingra 6th/edpg. 66
• Trauma t o labyrinth leads t o SNHL. ... Dhingra 6th/edpg 33
• Abnormal attachment of stapes at oval w i n d o w (otosclerosis) will lead to conductive deafness. ... Dhingra 6th/edpg 30

High Noise / Acoustic trauma


~ 1 ~
Leads to

Damage of outer hair cells, disrupt organ of Rupture of tympanic membrane and disruption
corti and rupture the Reissner's membrane. of ossicular chain (in case of severe blast)
T
1 SNHL | |Conductive hearing loss.]

... Dhingra 5th/edpg 40,6th/edp 34


15. Ans. is d i.e. Histiocytosis X Ref. Dhingra5th/edp 38;Harrison 17th/edp2603
Acoustic neuroma and endolymphatic hydrops (Meniere's disease) can lead to SNHL and tinnitus (Dhingra 5/e p. 38). Meningioma
can cause deafness, and tinnitus as a part of Neurofibromatosis type 2 syndrome (Harrison 17th/edp 2603) and its peak incidence
occurs in middle age.
No where it is mentioned histiocytosis X causes deafness and tinnitus. Another point which goes against it is the age o f patient (55
years) as histiocytosis occurs mainly in children.
16. Ans. is b i.e. Descent in air Ref. Dhingra 5th/edp 74,6th/edp 66
Otitic Barotrauma: It this condition, Eustachian t u b e fails t o mantain middle ear pressure at ambient atmospheric level
Etiology: • Rapid descent during airflight
• Underwater diving
Compression in pressure chamber.
CHAPTER 19 Hearing Loss

Mechanism
• . -

ear pressure
I
Normal eustachian tube

X
Allows passage of air from middle ear to pharynx

if atmospheric pressure is more


by>90mmhg

Locking of eustachian tube

X
Sudden negative pressure in middle ear •

X
Retraction of tympanic membrane, hyperemia,
transudation with haemorrhage
r
Aero otitis media

. Severe earache, deafness, tinnitus, vertigo.


• On examination - Tympanic membrane is retracted, congested or there may be effusion in middle ear.
• Conductive type of hearing loss is present.

Precautions

• Avoid traveling during upper respiratory infection.


• Avoid sleep during descent.

• Do Valsalva or swallowing or yawning during descent of flight.

Treatment
• Middle ear ventilation should be restored by decongestants, catheterization or even myringotomy.
17. Ans. is d i.e. propranolol Ref. Scott's Brown 7th/ed vol-3 pg- 3568 Table 238 d. 7

Ototoxic Drugs

Class Examples Predominant ototoxic effects


1. Antimalarial Quinine Temporary hearing loss, tinnitus
% Analgesia, Antipyretics Aspirin Temporary hearing loss, tinnitus
"3. Aminoglycoside Amikacin, gentamicin, kanamycin, streptomycin, Permanent hearing loss and /or vestibular injury
Neomycin, netilmicin, tobramycin, ispamicin
4. Antineoplastics Cisplatin/ilarboplatin Permanent hearing loss and /or vestibular injury
5. Diuretics Ethacrynic acid, furosemide Temporary hearing loss
6. Industrial solvents Toluene benzene Permanent hearing loss in animals, inconclusive evidence in man.
7. Polypeptide antibiotics Viomycin, vancomycin Permanent vestibular injury and / or hearing loss
8. Macrolide antibiotics Erythromycin, azithromycin clarithromycin Temporary hearing loss

|Agents for w h i c h there have been isolated reports of ototoxicity are:


Arsenals, Bromides, chloramphenicol, chlorhexidine, erythromycin, Mercury, polymyxin B,Tetracycline, vinblastine and Vincristine
18. Ans. is a i.e. Distortion of ossicular chain Ref. Logan Turner 10th/edp 347
In post head injury, the conductive deanfess may occur due to:
• Fracture temporal bone (more commonly longitudinal) extending t o external canal: tympanic membrane is frequently t o r n and
inner ear is spared.

248 T SECTION V Ear

• Blood or CSF in external and middle ear.


• Damage t o ossicle (most frequent being incudo-stapedial joint) resulting in more severe and permanent conductive deafness.
• Aseptic necrosis of long process o f incus can lead t o late conductive deafness.

In the Question it is given:


Tympanic membrane is normal and mobile: In h e m o t y m p a n u m - tympanic membrane will appear red/blue [due t o presence of
blood pigments] so it is ruled out. ...Logan Turner 10th/ed p441
Otosclerosis and EAC sclerosis do not occur in case of head injury \ they are ruled o u t

ALSO KNOW

Causes of SNHL in case of head injury:


• Labyrinthine concussion
• Vestibular damage
19. Ans. is d i.e. rupture of tympanic membrane Ref. Dhingra 5th/ed pg 38,6th/ed p 32
SNHL - Sensorineural hearing loss:
• It results f r o m lesions o f cochlea, Vlllth nerve or central auditory pathways
• It may be present at birth i.e. congenital (explained in detail later) or may be acquired.

C o m m o n C a u s e s of A c q u i r e d S N H L

Infection of labyrinth - viral, bacterial or spirochetal


Trauma t o labyrinth (as in # of temporal bone)
Noise induced hearing loss II

Ototoxic drugs
Old age/Presbycusis
Meniere's disease
Acoustic neuroma
Sudden hearing loss
Systemic diseases like diabetes, hypothyroidism, kidney disease, A u t o i m m u n e diseases, multiple sclerosis, blood dyscrasias
Familial progressive SNHL.
From the above list-It is clear that Option a i.e. old age and o p t i o n c i.e. loud sound-cause SNHL
Since SNHL results f r o m lesions of cochlea - cochlear otosclerosis which is a variant of normal otosclerosis (which causes conductive
deafness) will cause sensorineural deafness (i.e. o p t i o n b is correct)
Perforation / Rupture of tympanic membrane causes conductive deafness and not- SNHL.

Loud noise can lead to both SNHL and conductive deafness (which occurs only in case of severe blast).

20. Ans. is b and c i.e. Rubella / mealses; and Mumps Ref. Scott Brown 7th/ed p 3579; OP Ghai 7th/ed p 333
"The most common postnatal cause of acquired SNHL is meningitis, while the most common prenatal cause is intrauterine infection
(eg TORCH infections, syphilis, mumps, measles)". - OP Ghai 7th/ed p 333
According t o Scotts Browth 7/e p. 3579 - Specific viruses like mumps and syphilis and encephalitis can cause sudden sensorineural
hearing loss.
21. Ans. is None Ref.+iarrison 16th/edp 1.692;17th/ed p 1794; Dhingra 5th/edpg 129,6th/edp 30,116; Maqbool 11 th/edp 116
22. Ans is a, b, c and e i.e. Alport's syndrome; Pendred's syndrome; Treacher-Collins syndrome and Michel's aplasia.

C a u s e s of C o n g e n i t a l D e a f n e s s

Conductive
• Meatai atresia
• Fixation of stapes footplate
• Fixation of malleus head
• Congential cholesteatoma
• Ossicular discontinuity
• Crouzons syndrome
• Aperts syndrome

mi
CHAPTER 19 Hearing Loss

nemonic
Sensorineural deafness
Assistant Aplasia
Branch Bartter's syndrome
Manager MELAS
W Waardenburg syndrome/ wildervanck syndrome
A Alport syndrome (SNHL develops by the age of 30 yrs)
R Refsum syndrome
K Klippel feil syndrome
U Ushers syndrome
T Treacher Collins syndrome
Just Jervell and Lange neilson syndrome
Loves Leopard syndrome
To Trisomy 13,15,21
Have Hyper pigmentation
Pineapple Pendred syndrome
And Albinism
Orange Onychodystrophy
Raita Renal tubular acidosis (Distal/Type I)

Stickler syndrome Treacher collins syndrome (current otolaryngology 2/e pg-700), vander hoeve syndrome, Pierre Robin syndrome can lead to both
SNHL or conductive hearing loss.

Aplasia - Michels aplasia: characterised by lack of development of inner ear. External ear and middle ear may be normally
functioning.
Other aplasias: M o n d i n i aplasia/scheibe aplasia / Alexandar aplasia.

Nail P a t e l l a S y n d r o m e

• An autosomal d o m i n a n t trait".
• Iliac horns develop on the pelvis
• Characterised by: multiple osseus abnormalities primarily affecting the elbows knees and nail.
• 5 0 % patients have clinically evident nephropathy.
• It is associated w i t h neural - sensory hearing impairment and Glaucoma. ... Harrison 17th/edp 1794
23. Ans. is c i.e. Perilabyrinthine fistula Ref. Dhingra 4th/ed p 46,5th/ed pg 52

Barotrauma /Sudden Sudden t i n intra cranial


Accidental dislocation
Stapedectomy pressure change in pressure as in coughing
of stapes
middle ear / Diving and weight lifting
t t t t

Leads to - perilabyrinthine/perilymph fistula •

1 -
Perilymph leaks into the middle ear through
oval or round window and causes

E 1
-
Fluctuating SNHL Intermittent vertigo Sometimes tinnitus


Derangement of Vestibular system is indicated by: BPPV (Benign Paroxysmal positional vertigo)
• Vertigo Lermoyez syndrome
• Nystagmus
[} N Y S T A G M U S
| VERTIGO
It is involuntary, rhythmical, oscillatory movements o f eyes away
It is hallucination of movement i.e. one feels as if a person is moving f r o m direction of gaze.

as compared t o his surroundings or vice versa. Nystagmus has 2 components


I
C a u s e s of v e r t i g o : ( V E R T IGO )
2 2 2
Quick (fast) component Slow component
• Vascular: (V)
Thromboembolic phenomenon
Vertebrobasilar insufficiency
Anaemia • Nystagmus is named after quick component.
Hyper/hypotension • It is eliminated under the effect of anaesthesia.

• Epilepsy: (E)
T e s t s for V e s t i b u l a r F u n c t i o n s
• Endocrinological disorders: (E)
Diabetes
Clinical tests Laboratory test
Hypothyroidism
• Remedial drugs: (R) Spontaenous nystagmus Caloric test
Antibiotics - streptomycin - Cold caloric tets with ice cold
water modified (Kobrak's test)
Sedatives
- Fitzgerald-Hallpike test
Antihypertensives
(Bithermal caloric test)
• Trauma-T - Temperature of water used
• Tumour -T is + 7°C from normal body
Acoustic neuroma temperature
• Infections -1 - Cold-air caloric test by Dundas-
Viral / Bacterial / Syphilitic labyrinthitis. Grant method. Done in case
of perforation of tympanic
• Glial disease: (G)
membrane.
Disseminated sclerosis
• Ocular disease: (O) Fistula test 0
Electronystagmography
High refractive error Romberg test Optokinetic test
Diplopia. Gait Rotation test
Past-pointing and falling Galvanic test0

• Others: (O)
0

Hallpike manaeuver Posturography


Menieres disease
(positional test)
Vestibular neuronitis
CHAPTER 20 Assessment of Vestibular Function

Fistula test is done by pressing the tragus and alternately releasing it or by compression of air by Siegle's speculum. Positive test is indicated
by vertigo and nystagmus and signifies persence of fistulous communication between middle ear and labyrinth. Negative test signifies
absence of fistula andfistulawith dead labyrinth.
Galvanic test is the only vestibular test which helps in differentiating an end organ lesion from that of vestibular nerve leison.
Hennebert's sign: This is positive fistula test in the absence of fistula. The causes include congenital syphilis (utricular adhesions to
stapes) and some cases of Meniere's disease.
Romberg's sign: It is indicative of not the cerebellum lesions but the dorsal column (somatosensory) lesions.
Frenzel glass: Nystagmus is best observed in the darkened room by illuminated Frenzel glass, which is nothing but a 20 diopters lens.
Causes of ipsilateral (same direction) nystagmus: Irrigation of ear with warm water and serous labyrinthitis.
Causes of contralateral (opposite direction) nystagmus: Purulent labyrinthitis, labyrinthectomy and irrigation of ear with cold
water.
Dix-Hallpike maneuver: This test is used in patients with episodic positional vertigo. On Dix-Hallpike testing, central nystagmus
appears immediately without a latent period as soon as head is in critical position.
Fitzgerald Hallpike Bithermal caloric test: The lateral (horizontal) semicircular canal (SCC) is stimulated (tested) by irrigating cold
(30°C) and warm (44°C) water in the external auditory canal. Cold water induces opposite side nystagmus while warm water results into
the same side nystagmus (COWS (Cold, opposite; Warm, same)). In a sitting position with head tilted 60° backward, lateral semicircular
canal is stimulated during caloric testing. To bring the lateral SCC in vertical position, patient's head is raised 30° forward if s/he is in
supine position but in a sitting position the head is tilted 60° backward.
. , ,—,




252? SECTION V Ear

QUESTIONS

1. Which of the following statement regarding Eustachian 12. On otological examination all of the following will have
tube dysfunction is wrong? [AP2000] positive fistula test except: [AI02]
a. Undistorted light image on t h e anterior quadrant of a. Dead ear
tympanic membrane b. Labyrinthine fistula
b. No m o v e m e n t of the tympanic membrane on siegel's c. Hypermobile stapes footplate
method d. Following fenestration surgery
c. Malleus is easily visible 13. A positive fistula test during Siegelisation indicates:
d. Lusterless tympanic membrane [2000]
2. Common cause of eustachian diseases is due: a. Ossicular discontinuity
a. Adenoids b. Siegle's b. Para-labyrinthitis due to erosion of lateral semi-circular canal
c. Otitis media d. Pharyngitis c. CSF leak through the ear
3. Al I are tests to check eustachian tube patency except: d. Fixation of stapes bone
[AIIMS] 14. False positive fistula test is associated with: (TN 2005)
a. Valsalva manuvere b. Fistula's test a. Perilymph fistula b. Malignant sclerosis
c. Frenzel's manuvere d. Tonybee's manuvere c. Congenital syphilis d. Cholesteatoma
4. Semicircular canal involved in Positive Romberg test with [DNB 2002]
15. Hallpike test is done for:
eyes closed detects defect in: [AIIMS may 09] a. Vestibular function b. Corneal test
a. Proprioceptive pathway b. Cerebellum d. Audiometry
c. Cochlear function
c. Spinothalmic tract d. Peripheral nerve 16. Fitzgerald's caloric test uses temperature at: [JIPMER 92]
5. Site of lesion in unilateral past pointing nystagmus is: a. 30°Cand44°C b. 34°Cand41°C
[AIIMS June 97] c. 33°Cand21°C d. 37°Cand41°C
a. Posterior semicircular canal
17. At what angle is Hallpike thermal caloric test done:
b. Superior semicircular canal
[APPGI06]
c. Flocculonodular node
a. 15° b. 30°
d. Cerebellar hemisphere
c. 45° d. 60°
6. Post traumatic vertigo is due to: [PGI June 06,03]
18. Cold caloric test stimulates: [AP2008]
a. Perilymphatic fistula
a. Cochlea
b. Vestibular neuritis
b. Lateral semi circular canal
c. Secondary endolymphatic hydrops
c. Posterior semicircular canal
d. Ossicular discontinuity
d. All
e. Benign Positional vertigo
19. In 'cold caloric stimulation t e s t l , the cold water, induces
7. Postitional vertigo is: [UP2001]
movement o f t h e eye ball in the following direction:
a. Lateral b. Superior
[AI99]
c. Inferior d. Posterior
a. Towards the opposite side
8. What is th treatment for Benign Positional vertigo?
b. Towards the same side
[APPG06]
c. Upwards
a. Vestibular exercises b. Vestibular sedatives
c. Anthistamines d. Diuretics d. Downwards
9. Latest treatment in BPPV is: [Kerala 03] 20. In Fitzgerald Hallpike differential caloric test, cold-water
a. Intralabrynthine streptomycin irrigation at 30 degrees centigrade in the left ear in a
b. Intralabrynthine steroids normal person will include: [2000]
c. Valsava manuvere a. Nystagmus to the right side
d. None b. Nystagmus to the left side
10. Vestibular function is tested by: [PGIDec.02] c. Direction changing nystagmus
a. Galvanic Stimulation test b. Acoustic reflex d. Positional nystagmus
c. Fistula test d. Impedance audiometry 21. Which of the following is not true of caloric test?
e. Cold caloric test [MH2005]
11. Fistula test stimulates: a. Induction of nystagmus by thermal stimulation
a. Lateral semicircular canal b. Normally, cold water induces nystatmus to opposite side
b. Posterior semicircular canal and warm water to same side.
c. Anterior semicircular canal c. In canal paresis the test is inconclusive
d. Cochlea d. None
CHAPTER 20 Assessment of Vestibular Function J 253

22. Caloric test has: [Delhi 96] 27. Vertigo is definied as: [FMGE2013]
a. Slow component only b. Fast component only a. Subjective sense of imbalance
c. Slow + fast component d. Fast component occasionally b. Objective sense of imbalance
23. Spontaneous vertical nystagmus is seen in t h e lesion c. Both of the above
of: [Kolkatta -2005] d. Round movement
a. Midbrain b. Labyrinth 28. Calorie test based on thermal stimulation stimulates of
c. Vestibule d. Cochlea which part of the semi circular canals: [FMGE 2013]
24. True about central nystagmus:
a. Posterior b. Anterior
a. Horizontal
c. Lateral d. All of the above
• Direction fixed
Direction changes
Not suppressed by visual fixation NEET PATTERN
Suppressed by visual fixation
25. Third window effect is seen in: [AIIMS Nov2012] 29. In cold caloric stimulation test, the cold water, induces
a. Perforated tympanum movement of th eye ball in the following direction:
b. Dehiscent superior semicircular canal [NEET Pattern]
c. Round w i n d o w a. Towards the opposite side
d Oval window b. Towards the same side
26. Features of superior canal dehiscence are: [PGI -2010] c. Upwards
a. Positive Romberg's sign d. Downwards
PositiveTullio's phenomenon 30. Epleys menouver: [NEET Pattern]
Positive Hennebert's sign a. Positional vertigo b. Otosclerosis
Oscillopsia c. ASOM d. CSOM
Positive Dix-Hallpike Maneuver

EXPLANATIONS AND REFERENCES


1. Ans. is a i.e. undistorted light image on the anterior quadrant of tympanic membrane
Ref. Dhingra 5th/ed pg 66,61,6th/ed p 55,57-59

Eustachian Tube Dysfunction

• Normally Eustachian t u b e (ET) is closed and opens intermittently during yawning, swallowing and sneezing t h r o u g h active
contraction of Tensor vili palatini muscle.
• It serves important functions like

£ Protection against
X
Ventilation and regulation • Middle ear clearance of
of middle ear pressure • Naso pharyngeal sound pressure secretions
• Reflux of nasopharyngeal secretions.

When ET is blocked it leads t o negative pressure in middle ear and retraction of Tympanic membrane

Symptoms

• Otalgia/ear p a i n
0

• Hearing loss 0

• Popping sensation 0

• Tinnitus
Disturbance of equilibrium or vertigo

O/E

Tympanic membrane is retracted 0


i.e. cone of light will be distorted obviously)
Congestion along the handle of malleus (i.e. malleus will be easily visible)
Transudate will be visible behind the tympanic membrane imparting it an amber colour (i.e. it will be lusterless)
In severe cases as in barotraumas, there may be visible haemorrhages / h e m o t y m p a n u m or even perforation of the tympanic
membrane.
254]_ SECTION V Ear

Friends here it is very important t o know features of Retracted Tympanic membrane:

1. It appears dull and lusterless


2. Cone o f light is absent or interrupted
3. Handle of malleus appears foreshortened
4. Lateral process of malleus becomes more prominent
5. Anterior and posterior malleal folds become sickle shaped.
So even if we d o not know anything about Eustachian tube blockage - then also, by just remembering the features of retracted
tympanic membrane, we can solve this one.
2. Ans. is a i.e Adenoids Ref. Dhingra 5th/ed pg 67,6th/ed p 60
Eustachian Tube dysfunction is commonly caused by:
• Adenoids/allergy
• Barotrauma
• Cleft palate • -

• Down syndrome
• Nasal condition like: - Polyps
- Sinusitis
- DNS
- Nasopharyngeal t u m o r / m a s s
3. Ans. is b i.e. Fistula test Ref. Dhingra 5th/ed pg 65-66, 6th/ed p 59
This question can be solved even if we d o n t know all tests for eustachian t u b e patency, because we know fistula test is for assessing
vestibular functions and n o t for Eustachian t u b e patency. Still it is w o r t h while knowing tests for eustachian tube patency.

T e s t s for E u s t a c h i a n T u b e P a t e n c y

Mnemonic: PMT Is So Very Furiously Complicated


P Politzertest 0

M Methylene blue test 0

T Toynbee test 0

Is Inflation, Deflation test 0

So Sonotubometry 0

Very Valsalva test 0

Furiously Frenzel manovuere 0

Complicated Catheterization 0

... Dhingra 5th/edpg 458

Ans. is a i.e. Proprioceptive pathway


Ref. Dhingra 5th/edpg
47; Neurological differential diagnosis by RoongrofBhidayasiri, Michael F. Waters,
Christopher C. Giza pg 94; Scotts Brown 7th/ed vol-3 pg 3731
• The Romberg is a test of proprioceptive function:
- "The Romberg test explores for imbalance due to proprioceptive sensory loss. The patient is able to stand with feet together and
eyes open but sways or falls with eyes closed; it is one of the earliest sings of posterior column disease."
- DeJong's the neurologic examination By William Wesley Campbell, Russell N. DeJong, Armin F. Haerer6/e p447
• Proprioceptive pathway:
- Proprioception is the ability t o sense the position of one's extremities w i t h o u t the aid of vision.
- The peripheral sense organs are located in the muscle, tendons, and joints. The first cell body is situated in the dorsal root
ganglion, going w i t h o u t a synapse t o the ipsilateral fasiculi cuneatus and gracilis (dorsalcolumn) t o the lower medulla where
the synapse occurs. Following a decussation o f t h e internal arcuate fibers, the impulses ascend in the medial lemniscus to
the thalmus, terminating in the parietal lobe, posterior t o those that convey touch.
• Patholophysioiogic basis of Romberg test:
Central postural control (equilibrium) is dependent on input from three peripheral modalities:
1. Joint position sense (proprioception), carried in the dorsal columns of the spinal cord;
2. Vision
3. Vestibular apparatus
• Disturbance in any one of these modalities can be compensated for (completely or incompletely) by i n p u t f r o m the other t w o
systems.
CHAPTER 20 Assessment of Vestibular Function J 255

Impaired proprioception can be over come by visual and vestibular feedback. However, reduced visual i n p u t in t h e dark
surroundings or due t o failing vision can seriously predispose such a patient t o severe incoordination (ataxial)
Asking the patient t o close his eyes during rombergs test helps uncover any disordered proprioception that may have been
masked by vision.
Conditions c o m m o n l y causing a positive Romberg test:
- Posterior column dysfunction
• Posterior cord compression
• Multiple sclerosis
• Subacute combined degeneration o f t h e spinal cord

• Tabes dorsalis
- Sensory polyneuropathy
• Idiopathic
• Diabetes mellitus
- Intracranial lesions
• Less c o m m o n

Romberg's test is not a test of cerebellar function. Patients with cerebellar ataxia will, generally, be unable to balance even with the eyes open.

5. Ans. is d i.e. Cerebellar hemisphere Ref. Ganong 22th/ed p 221,222; Dhingra 6th/ed p 46

C e r e b e l l u m is F u n c t i o n a l l y D i v i d e d I n t o

Flocculonodular lobe Spinocerebellar) Neo cerebellum.

I
Mainly concerned with equilibrium and learning
induced changes. Parts of cerebellar hemispheres I
Lesions cause:
T
Concerned with rest of the functions of cerebellum
- Truncal ataxia Lesions cause:
- Wide base gait - Scanning speech
- Falling in any direction - Dysmetria/past pointing (inability to control movements)
- Intention tremors
- Adiadochokinesia (Inability to perform rapidly
alternatingopposite movements).
- Decomposition of movement (i.e. difficulty in performing
actions which involve simultaneous motion at one or more
than one joints.
3

Nystagmus can occur in both midline or hemispheral disease, but past pointing indicates hemispheral lesion.
6. Ans. is a, c and e i.e. Perilymphatic fistula, Secondary endolymphatic hydrops and BPPV
Ref. Dhingra 6th/ed p 46; Current otolaryngology 2nd/edpg714
Post traumatic vertigo can be seen in:
• Severe trauma to parietal skull bone
• Longitudinal temporal bone # cause concussion of labyrinth or completely
• Whiplash injury disrupt bony labyrinth or cause injury of VIII
• Barotrauma . nerve or cause a perilymphatic fistula
• Severe acoustic trauma
In case of acoustic trauma vertigo can be due t o disturbance in the vestibular end organs i.e. otolitis
• Secondary endolymphatic hydrops (secondary Meniere's disease) is clinical presentation of Meniere's disease viz episodic
vertigo, fluctuating hear loss, tinnitus and ear fullness due t o conditions like head trauma or ear surgery, viral infection (measles/
mumps) syphilis and Logan's syndrome.
• Benign Paraxysmal positional vertigo: It is most c o m m o n type of peripheral vertigo which arises due t o collection o f debris
in posterior semicircular canal. 2 0 % patients of BPPV have an antecedant h/o head trauma.
7. Ans. is d i.e. Posterior
8. Ans. is a i.e. Vestibular exercises
9. Ans. is d i.e. None Ref. Dhingra 5th/edpg51,6th/edp45; Current Otolaryngology 2nd/edpg 713-714
256 T SECTION V Ear

Benign Paroxysmal Positional Vertigo

• Characterised by vertigo w h e n the head is placed in certain critical position


• Not associated w i t h hearing loss or any other symptom.
° Caused b y d i s o r d e r o f p o s t e r i o r semicircular canal." (generally debris is collected in it)
• Average age of presentation -5th decade
• History o f head trauma/ear infections may be present in 2 0 % cases.
• Vertigo is fatiguable
« Vertigo may be associated w i t h nausea
• Characteristic nystagamus (latent, geotropic, fatigable) w i t h Dix Hallipike test

Management

Vestibular exercises (Epley's manoeuvre) done t o reposition the debris in the utricle is the only current treatment of choice. In some
patients labyrinthine sedatives like prochlorperazine, promethazine may be given.

Role o f S u r g e r y in BPPV

Surgery is reserved only for those very rare patients w h o have no benefit f r o m vestibular exercises and have no intracranial pathol-
ogy on imaging studies.
S u r g e r y o f choice: Posterior semicircular canal occlusion
10
Ans. is a, c a n d e i.e. Galvanic t e s t , Fistula t e s t ; a n d Cold caloric test Ref. Dhingra Sth/ed pg 46-50,6th/ed p 43-44

Vestibular Function
Clinical tests Laboratory test

• Spontaneous nystagmus • Caloric test


- Modified (Kobrak's test)
- Fitzgerald-Hallpike test (Bithermal caloric test)
- Cold-air caloric test by Dundas-Grant method. Done in case of perforation of tympanic membrane.
• Fistula test" • Electronystagmography
• Romberg test • Optokinetic test
• Gait • Rotation test
• Past-pointing and falling 0
• Galvanic test 0

• Hallpike maneuver (positional test) • Posturography

1 1 . A n s . is a i.e. Lateral semicircular canal


12. Ans. is a i.e. Dead ear
13. Ans. is b i.e. Para l a b r y n t h i t i s d u e t o e r o s i o n o f lateral semi circular canal
Ref. Dhingra 5th/ed pg 46,6th/edp 41; Tuli 1/ed p 39
.

F i s t u l a t e s t is d o n e t o A s s e s s t h e V e s t i b u l a r F u n c t i o n

Basis: In case of'fistulous communication between middle ear and labyrinth


i
Any pressure change in
-
External auditory canal (produced by pressing tragus or by siegel's speculum
i
will stimulate lateral semicircular canal (Ans 11 - Ref Tuli 1/ep. 939)
I

Produce nystagmus/vertigo

F i s t u l a T e s t is -

Positive Negative False positive False negative


• (Means labyrinth is functioning and a (i.e. positive fistula test without (i.e. fistula is present but still fistula
fistulous communication is present the presence of fistula is called as test is negative)
between middle ear and labyrinth) Hennebert's Sign)

Contd..

-
CHAPTER 20 Assessment of Vestibular Function J 257
Contd..

Positive Negative False positive False negative


• In erosion of lateral semi circular • In norma individuals Seen in: Congenital syphilis and • When cholesteatomacovers the
canal as in cholesteatoma (Ans 13) meniere's disease site of fistula
• Surgically createdwindow in the • In dead labyrinth (Ans 12) • III fitting speculum.
horizontal v
• Abnormal opening in oval window -
post stapedectomy fistula.
• Abnormal opening in oval window
- innst
• Abnormal opening inround window
• Hypermobile stapes footplate
14. Ans. is c i.e. Congenital syphilis Ref. Dhingra 5th/ed pg 47

False Positive Fistula Test-Hennebert Sign

• Fistula test is positive w i t h o u t the presence of fistula.


• It is seen in case of congenital syphilis and Meniere's disease
• In congenital syphilis, stapes foot plate is hypermobile while in meniere's disease there is a fibrous band connecting utricular
macula t o t h e stapes footplate.
• In b o t h these conditions, movements of stapes result in stimulation of utricular macula w h i c h will cause nystagmus and vertigo
leading t o false positive fistula test.This is called as Hennebert sign.
15. Ans. is a i.e. Vestibular function Ref. Dhingra 5th/ed pg 47

Hallpike test/Positional Test

• It is a test for assessing vestibular function


• Particularly useful w h e n patients complain of vertigo in certain head positions.
. Helps t o differentiate peripheral and central lesions.

Method

Patient sits an a couch


Examiner holds the patients head, turns it 45° t o the right and t h e n places the patient in supine position so that his head hangs
30° below the horizontal.
Patients eyes are observed for nystagmus
The test is repeated w i t h head turned t o left

Four parameters of nystagmus are observed
a. latency b. duration
c. direction d. fatigability
16. Ans. is a i.e. 30°C and 44°C •

17. Ans. is b i.e. 30°


18. Ans. is b i.e. Lateral semicircular canal Ref. Dhingra 5th/ed pg 48,6th/edp 43; Maqbool 11 th/ed pg 43

Caloric Tests: Important points

• Principle: t o induce nystagmus by thermal stimulation of vestibular system.


• Lateral semicircular canal is c o m m o n l y tested by all these tests
0

• There are 3 methods of performing these tests:

Cold caloric test (modified Kobraktest) Fitzgerald: (Hallpike test (bithermal caloric Cold air caloric test
test)

Patient position: patient is seated with head Patient position: patient lies supine with head Done when there is perfora -tion of tympanic
tilted 60°C backwards (to place horizontal tilted 30°C m e m b r a n e (as i r r i g a t i o n w i t h w a t e r is
canal in vertical position) contraindicated in these cases)

Temperatuer of water used - ice cold water. Temperature of water = 30°C and 44°C Air cooled by ethylchloride is blown into the
ear by Dundas Grant tube

i
SECTION V Ear

19. Ans. is a i.e Towards the opposite side Ref. Dhingra 5th/ed pg 48,6th/ed p 43
20. Ans. is a i.e. nystagmus to the right side

In c a l o r i c t e s t : H a l l p i k e F i t z e g e r a l d t e s t

Patient lies supine with head tilled at 30°C (so that horizontal canal is vertical)
Ear is irrigated with water at 30°C and 44°C (body temperature + 7°C)
T
In normal individuals In case of dead labyrinth/canal paresis
| Response" T
Cold water r
Induces nystagmus to opposite side
Warm water induces
No response / slow response

nystagmus to same side


Mnemonic -Cows
Cold water: Opposite side
Warm water: Same side

In Q 20:
Since cold water is used to irrigate left side: Nystagmus will be towards opposite side i.e. right side
21. Ans. is c i.e. in canal paresis the test is inconclusive Ref. Scott Brown 7th/ed vol-3 pg 3727
As discussed in previous question:
• Nystagmus can be induced both by cold as well as thermal stimulation
• Cold stimulation causes nystagmus towards opposite side while thermal stimulation causes Nystagmus towards same side.
(COWS)
• In canal paresis either there is a reduced or absent response (causes of U/L canal paresis are-U/L vestibular Schwannoma or
vestibular neuritis).
• B/L absence of caloric nystagmus is seen in case ofamminoglycoside ototoxicity or postmeningitis
22. Ans. is c i.e. Slow +Fast component Ref. Dhingra 5th/edp48,6th/edp 43; Maqbool 11 th/edp 43
Caloric test is used to test vestibular function/labyrinthine function
So nystagmus induced by it is vestibular in origin.
Vestibular nystagmus has b o t h fast (of cerebral origin) and a slow component (of vestibular origin).
23. Ans. is a i.e. Midbrain Ref. Scotts Brown 7th/ed vol-3 Pg-3922
"Vertical nystagmus means vertical displacement o f t h e eye, not side to side nystagmus when a t t e m p t i n g upward or d o w n ward
gaze. As denned vertical nystagmus always indicates brainstem dysfunction". - Scott Brown 7th/ed vol 3 p. 3922
24. Ans. is a, c and d i.e. Horizontal, Direction changes and Not suppressed by visual fixation
Ref. PL Dhingra 5th/46,6th/edp 42; Harrison 17th/ed pg 144-45 www.jeffmann.net/NeuroCuidemaps/nystagmus.html; Maqbool lll/ed
pg-43; Scotts Brown 7th/ed vol 3 pg-3724
• Nystagmus is rhythmic oscillatory movement of eye and has t w o components slow and fast.
• It can be o f vestibular or ocular in origin
• Vestibular nystagmus is called peripheral w h e n it is due t o lesion of labyrinth o r V l l l t h n e r v e and central, w h e n lesion is in the
0

central neural pathways (Vestibular nuclei, brainstem and cerebellum)


0


Central Vestibular Nystagmus

Characteristics

• Central nystagmus may be horizontal, vertical, purely torsional or m i x e d while peripheral vestibular nystagmus is horizontal
0

in nature.
• Nystagmus may be bi-directional, and changes direction in different directions of gaze . 0

• Nystagmus is unaffected by visual f i x a t i o n 0

• Nystagmus is constant and does not wane w i t h t i m e (vestibular nystagmus is fatigueble)


• Impaired saccades and impaired smooth eye pursuit movements are c o m m o n l y present. It is coarse in nature.
• Brainstem and cerebellar signs are c o m m o n l y present
• Any associated vertigo and nausea/vomiting is mild (vestigo is a prominent symptoms of vestibular nystagmus)
• Any tendency t o fall is often multi- directional, and not unidirectional
• Hearing loss is rarely present
CHAPTER 20 Assessment of Vestibular Function J 259

Features Peripheral Central


Latency 2-20 seconds No latency
Duration Less than 1 minute More than 1 minute
Direction of nystagmus 0
Direction fixed towards the undermost ear Direction changing 0

Fatiguability Fatiguable Non fatiguable


Accompanying symptoms Severe vertigo None or slight

25. Ans. is b i.e. Dehiscent Superior Semicircular Canal. Ref. Current otolaryngology 3rd/ed p 737-738
26. Ans. is b, c a n d d i.e. Positive Tullio's phenomenon. Positive Hennebert's sign and Oscillopsia
Ref. Current otolaryngology 3rd/edpg 737-738
In 1998, Lloyd minor and colleagues described sound and/or presssure induced vertigo associated w i t h bony dehiscence o f t h e
superior semicircular canal.
• Third window effect takes place in case of dehiscent superior semicircular canal whereby the dehiscent part of semicircular
canal acts as a third window of inner ear. As a result, e n d o l y m p h w i t h i n the labyrinthine system continues t o move in relation
t o sound or pressure changes which causes activation o f t h e vestibular system.


Superior semicircular canal dehiscence •

T
Third window effect

Vertigo when exposed to Vertigo with pressure Oscillopsia - objects in


loud noise (tullio phenomenen) changes in ear (Hennebert sign) visual field appear to oscillate

Auditory symptoms
T •

• Increased sensitivity to bone conducted sound -

Low frequency conductive hearing loss


Presence of stapedial reflex

The presence of stapedius reflex with low-frequency conductive hearing loss should prompt radiological imaging of the inner ear to exclude the
possibility to dehiscence ofthe inner ear.

Patient profile

Age: A l t h o u g h dehiscence o f t h e supertior canal may be congenial symptoms and signs usually do not present early in life; the
youngest patients have been in their teen..Median age at diagnosis is 40 years.
• Sex: SCDS appears to affect males and females equally.
• Symptoms: Patients may complain of vestibular symptoms only, auditory and vestibular symptoms, or, less commonly, isolated
auditory symptoms.
- Patients report increased sensitivity t o bone-conducted sounds.
- Inner ear conductive hearing loss is c o m m o n .
- Stapedial reflex is present.
• Pathology: The dehiscent portion o f t h e superior canal acts as a third mobile w i n d o w allowing acoustic energy t o be dissipated
there. As a result, endolymph w i t h i n the Inbyrinthine system continue t o move in relation t o saound or pressure, w h i c h causes
an activation o f t h e vestibular system.
• Imaging studies of choice is high-resolution CT of the temporal bone.
• Audiologic testing demonstrates low-frequency conductive hearing loss w i t h the presence of stapedius reflex Differential
diagnosis for the condition is -Otosclerosis where although low frequency conductive hearing loss is seen but due t o fixation
o f t h e stapes footplate, the stapedial reflex is absent.

Also know

"Oscilopsia" is visual disturbance in which objects in the visual field appear t o oscillate. The severity o f t h e effect may range f r o m
a mild blurring t o rapid and periodic j u m p i n g . Oscillopsia may be caused by loss o f t h e vestibulo-ocular reflex, involuntary eye
260 {_ SECTION V Ear

movements such as nystagmus, or impaired coordination in the visual cortex (especially due t o toxins) and is one ofthe symptoms
of superior canal dehiscence syndrome. Sufferers may experience dizziness and nausea, Oscillopsia can also be used as a quantitative
test t o d o c u m e n t aminoglycoside t.ox\c\ty"-en.wikipedia.org/Oscillopsia.

Other causes leading to third window effect


1. Anatomical thrid w i n d o w
2. Diffuse third w i n d o w
A. Semicircular w i n d o w
- Superior canal dehiscence
- Posterior canal dehiscence
- Posterior canal dehiscence
- Lateral canal dehiscence
B. Vestibule
- Large vestibular aqueduct syndrome
- Inner ear malformation causing a dehiscence between internal auditory canal and dehiscence
C. Cohlea
- Dehiscence between carotid canal and scala vastibule
- Inner ear malformation causing a dehiscence between internal auditory canal and scala vestibule.

27. Ans. is a i.e. Subjective sense of imbalance Ref. Mohan Bansal Ist/edp227
Vertigo is a subjective sense of imbalance or false sense of m o t i o n felt by patient.

Vertigo can be

Peripheral (M/C = 85% cases) Central (15%)


• Involves vestibular end organs and their 1st order neurons (i.e. • Involves central nervous system after the entrance of vestibular
the vestibular nerve) nerve in the brainstem and involves vestibulo-ocular and
• Cause lies in the internal ear or Vlllth nerve. vestibulospinal pathways

28. Ans. is c i.e. Lateral Ref. Mohan Bansal Ist/ed p 236; Point 12
Fitzgeraled Hallpike Bethernalcaloric test: the lateral semicircular canal (SCC) is stimulated (tested) (horizontal) by irrigating cold
(30°C) and w a r m water (44°C) in the external avditoy canal w a r m ...MohanBansal Ist/edp236
29. Ans. is a i.e. Towards the opposite side Ref. Dhingra 6th/ed p 43
As discussed previously:
The mnemonic 'COWS' (cold-opposite; warm-same side) is very helpful t o remember the direction in which water induces nystagmus
in caloric test
30. Ans. is a i.e. Positional vertigo Ref. Dhingra 6th/ed p 45
Benign paroxysmal positional vertigo (BPPV) characterised by vertigo w h e n the head is placed in a certain critical postion, can be
treated by Epley's monoeuvre.
The principle of this manoeuvre is t o reposition the otoconial debris from the posterior semicircular canal back into the utricle.
After manoeuvre is complete, patient should maintain an upright posture for 48 hour. Eighty percent o f t h e patients will be cured
by a single manoeuvre.


CHAPTER -

>

Diseases of External Ear


N o r m a l C o m m e n s a l F l o r a o f t h e E x t e r n a l E a r1 • May have associated deafness if canal gets occluded due


t o edema
• Staphylococcus epidermidis
Signs:
• Corynebacterium species
• Tragal sign positive
• Staphylococcus aureus
• In severe cases:
• Streptococcus viridans
a. Retroauricular sulcus is obliterated
Inflammatory Conditions o f t h e External Ear b. Forward displacement o f t h e pinna
Treatment:
• L o c a l — 1 0 % ichthanmol glycerin pack
| OTITIS EXTERNA ° Oral antibiotics -» if local cellulitis is present
• Oral analgesics
Any inflammatory condition of the skin of the external auditory • Incision and drainage—> if abscess formation
canal is otitis externa • In recurrent furunculosis - Rule out diabetes mellitus

Classification B. D i f f u s e O t i t i s E x t e r n a : ( T r o p i c a l / S i n g a p o r e e a r )

a. Localized -furunculosis Most common Organisms: • Pseudomonas pyocyaneas


j b. Diffuse otitis externa •
• Bacillus proteus
Idiopathic • Staphylococcus aureus
Traumatic • E. coli
Diffuse otitis externa can be:
Irritant
i. Acute - Signs and symptoms similar t o furunculosis
Bacterial
ii. Chronic - Symptoms: • Irritation in the ear
Fungal
• Constant desire t o itch
Environmental
Signs
Part of generalized skin conditions-
• Scanty discharge in the external auditory canal
• Seborrheic dermatitis
• Dried crusts
• Allergic dermatitis
• Scaling and fissuring in the canal wall
• Atopic dermatitis
Treatment:
• Psoriasis
• Ear toileting: most important step
Malignant Necrotizing - otitis externa
• Medicated wicks (Antibiotic + steroids)
Other (keratosis obturans)
• Oral antibiotic: are indicated in case of Cellulitis and Lym-
phadenitis
• Analgesics for relief of pain
A. Furunculosis (Localized Acute Otitis Externa)

Most common organism: Staphylococcus aureus C . O t o m y c o s i s : It is s e e n in H o t a n d H u m i d C l i m a t e


Site: Hair bearing area of the cartilaginous part of the external Most common Organisms
auditory canal • Aspergillus niger - Black-headed filamentous g r o w t h
Symptoms: • Candida albicans - White and creamy deposit
• Discomfort and pain • A.fumigatus-Green/Blue g r o w t h
• Aggravated by jaw/pinna movement • Dermatophytes - Actinomyces


262 T SECTION V Ear

Sign: Wet blotting paper appearance


Symptoms:
Mastoid
A r~-_ Medial _ ,
L->
.... •Petrous apex
• Intense itching Lateral sinus
l

• Pain Thrombosis Stylomas!oid<3 Spread Anterior


Parotid gland
1

foramen — ^ > • Temperomandibular


• Watery discharge w i t h musty odor
| Inferior \f joint
Treatment:
• Ear toileting t o remove all discharge and epithelial debris VII Nn Skull hasp.
• Antifungal ear drops Fig. 21.1: Spread of malignant otitis externa
• Antibiotics: As they help to reduce edema and inflam- Spread of the infection: See Fig. 21.1
mation and thus permit better penetration of anti fungal
• Nerves commonly involved:
agents
Most c o m m o n nerve involved - VII.
Others - IX, X, XI, XII,
D. B u l l o u s M y r i n g i t i s : O t i t i s E x t e r n a H e m o r r h a g i c a
• Investigation:CTscan, gallium and technetium-99 scintigraphy
Oraganism: Viral or mycolplasma pneumoniae
Prognosis: • It has high mortality rate (So termed as malignant)
Features: Hemorrhagic blebs on the lateral surface o f t h e t y m -
• Death due t o intracranial c o m p l i c a t i o n s like
panic membrane and the skin o f t h e External auditory canal.
sigmoid sinus thrombosis
• It is painful c o n d i t i o n 0

Treatment: Treatment: Includes correction of immunosuppression (when


• Analgesics possible), local treatment o f t h e auditory canal, long-term systemic
• Antibiotic: Only in case of secondary ear infection antibiotic therapy, and in selected patients, surgery.
• Blebs NOT t o be incised • In all cases, the external ear canal is cleansed and a biopsy
specimen o f t h e granulation tissue sent for culture.
E. H e r p e t i c O t i t i s E x t e r n a : • IV antibiotics is directed against the offending organism.
• For Pseudomonas aeruginosa, the most c o m m o n pathogen,
Organism: H. simplex t h e r e g i m e n involves an a n t i p s e u d o m o n a l p e n i c i l l i n or
H. zoster cephalosporin (3rd generation piperacillin or ceftazidime) with
Features of H. zoster/Ramsay Hunt syndrome an aminoglycoside. A fluoroquinolone antibiotic can be used
• Site of affection: in place o f t h e aminoglycoside.
Geniculate ganglion o f t h e facial nerve • Ear d r o p s c o n t a i n i n g a n t i p s e u d o m o n a l a n t i b o t i c e.g.
May also involve the V and VIII nerves ciprofloxacin plus a glucocorticoid is also used.
Symptoms: • Early cases can be managed with oral and otic fluoroquinolones
Severe otalgia only.
Vesicular eruptions on pinna o f t h e affected ear. • Extensive surgical debridement once an important part o f t h e
- Facial nerve palsy (LMN type)
treatment is now rarely needed.
May show associated vesicular eruption in the buccal
mucosa, hard palate and hypopharynx.
| TUMORS OF THE EXTERNAL AUDITORY CANAL
Treatment:
Oral acyclovir
Benign: - Papilloma Ceruminoma
(to be started within 72 hours of the onset of rash)
- Adenoma-
- Fibroma Sebaceous adenoma
F. M a l i g n a n t O t i t i s E x t e r n a / N e c r o t i s i n g O t i t i s E x t e r n a
- Exostoses
Progressive debilitating and sometimes fatal infection o f t h e - Osteoma
external auditary canal, characterized by granulation tissue in Exostoses (Most common Osteoma
external auditary canal at the j u n c t i o n of bone and cartiliage. benign tumor of external
Most common organism: Pseudomonas aeruginosa auditory canal)
Others: - S. aureus • Multiple • Rounded, pedunculated
- S. epidermidis • Sessile hemispherical elevations • U/L condition
- Aspergillus • B/L condition • Arises at the junction of bony
- Actinomyces • Arises in the bony meatus and cartilaginous meatus
Pathologically: characterized by necrotizing vasculitis
Features: occurs c o m m o n l y in:
1 MISCELLANEOUS CONDITIONS OF EXTERNAL EAR
• Elderly diabetic
• Immunosuppressed patient/use o f immunosupressive Impacted Wax/cerumen
drugs • Secreted by c e r u m i n o u s a n d sebaceous g l a n d s in t h e
• Patients w h o have received radiotherapy t o skull base cartilaginous part of external canal.
CHAPTER 21 Diseases of External Ear J 263

• Clinical features: sense of blockage • Malignant otitis external is caused by pseudomonas and is seen
Itching in early diabetic patients.
I hearing • Herpes zoster oticus also called Ramsay Hunt syndrome is
Tinnitus caused by chickenpox virus, varicella and affects geniculate
Vertigo ganglion.
• T r e a t m e n t : If h a r d , s o f t e n it by w a x solvents like soda • Cholesteatoma of externa! a u d i t o r y meatus is also called
glycerin and removed by syringing w i t h sterile water at body Keratosis obturans and is characterized by hyperaemia and
temperature, or w i t h wax hook. irritability of canal skin.
• Singapore ear also known as Telephonist ear or Tropical ear is
a type of diffuse otitis externa due t o hot and humid climate
For syringing pinna is pulled upwardQ and backwardQ and a stream • Exostosis is the most c o m m o n benign t u m o r of the external
of water from the ear syringe is directed along the posterior superior auditory meatus.
wall ofthe meatus 0
• Osteomas are usually single and arise at bony and cartilaginous
junction of external auditory canal, while exostosis are multiple
Important Points bony outgrowths from bony meatus.
• Syringing is indicated in patients w i t h ear symptoms where • The M/c congenital anomaly of ear is Bat ear.
wax obstructs the view o f t h e tympanic membrane. • The M/c rarest congenital anomaly of ear is Polyotia.

-
:

-
:


264 T SECTION V Ear

QUESTIONS

Common causes of otitis externa: [PGI 08] 13. An elderly diabetic present with painful ear discharge
a. Aspergillus b. Mucor and e d e m a of the external auditory canal w i t h facial
c. Candida d. Pseudomonas palsy, not responding to antibiotics. An increased uptake
e. Klebsiella on technetium bone scan is noted. The most probable
External otitis is also known as: [DNB 2003] diagnosis is [AI12]
a. Glue ear b. Malignant otitis externa a. Malignant otitis externa
c. Telephonists ear d. ASOM b. Malignancy o f t h e middle ear
Causes of Otomycosis: [PGI-08] c. Infective disease o f t h e middle ear
a. Candida b. Aspergillus d. M a l i g n a n c y o f n a s o p h a r y n x w i t h Eustachian t u b e
c. Thermophilus d. Staphylococcus obstruction
4. Fungus causing otomycosis most commonly \s:[Delhi96] 14. An old diabetic male presented with rapidly spreading
a. Aspergillus fumigatus b. Candida infection ofthe external auditory canal with involvement
c. Mucor d. Penicillin
ofthe bone and presence of granulation tissue. The drug
5. Myringitis bullosa is caused by: [AI93]
of choice for this condition is: [AIIMS May. 08]
a. Virion b. Fungus
a. Ciprofloxacin
c. Bacteria d. Virus
b. Penicillin
6. In Ramsay Hunt syndrome, all nerves are involved except
c. Second generation cephalosporin
[RJ 2002]
d. Aminoglycosides
a. 5 b. 7
15. W h i c h of t h e f o l l o w i n g is not a t y p i c a l f e a t u r e of
c. 8 d. 9
7. Heamorrhagic external otitis media is caused by: malignant otitis externa? [AIIMS May 06]
[PGI Dec. 98] a. Caused by Pseudomonas aeruginosa
a. Influenza b. Proteus b. Patients are usually old
c. Staphylococcus d. Streptococcus c. Mitotic figures are high
8. A patient has come with furuncle of ear. What is the d. Patient is immunocompromised
commonest method of treatment ? 16. Facial nerve palsy is seen in: [Jipmer 03]
a. Ear pack with 1 0 % ichthammol in glycerin wick a. Seborrheic otitis externa b. Otomycosis
b. Antibiotic and rest c. Malignant otitis externa d. Eczematous otitis externa
[Orissa 99] 17. A female diabetic have severe ear pain, granulation tissue
c. Antibiotic and drainage in external ear with Facial palsy is due to: [Bihar2004]
d. Analgesic a. Malignant otitis externa b. Herpes zoster otitis
9. Malignant otitis externa is caused by: [AP 96; Corned 07] c. Otomycosis d. None
a. S. aureus b. S. albus 18. Keratosis obturans is [TN2007]
c. P. aeruginosa d. E. coli a. Foreign body in external auditory canal
10. True statement about malignant otitis externa is: b. Desquamated epithelial cell + Cholesterol
a. Not painful c. Cholesterol crystals surrounded by calcium
[PGI 96] d. Wax in external auditory canal
b. Common in diabetics and old age 19. Chondritis of aural cartilage is most commonly due to:
c. Caused by streptococcus
[NIMHANS 06]
d. All of the above
a. Staphylococcus
11. Malignant otitis externa is: [PGI Dec. 99]
b. Pseudomonas
a. Malignancy of external ear
c. Candida
b. Caused by hemophilus influenzae
d. Both staphylococcus & Pseudomonas
c. Blackish mass of aspergillus
20. Cauliflower ear is: [Manipal06]
d. Pseudomonas infection in diabetic patient
a. Keloid
12. Malignant otitis externa is characterized:
b. Perichondritis in Boxers
[PGI Dec. 03; June 06]
a. Caused by pseudomonas aeruginosa c. Squamous cell carcinoma
b. Malignancy of external auditory canal d. Anaplastic cell carcinoma
c. Granulation tissue is seen in the floor of extra auditory 21. Not true about auricular hematoma [PGI May2011]
canal a. All case should receive antibiotic
d. Radiotherapy can be given b. Commonly seen in rugby player
e. Gallium scan is helpful for monitoring treatment c. Resolve spontaneously
CHAPTER 21 Diseases of External Ear T 265

22. Direction of water jet while doing syringing of ear should a. < 1 year of age b. 5-7 years of age
be: [Mahara 02] c Puberty d. Adulthood
a Anterior b Posterior 2 4
- Features of moderately retracted tympanic membrane
c Anterosuperior d. Posteroinferior are all except: [MH2005]
a. Handle of malleus appearance foreshortened
23. A newborn presents with bilateral microtia and external
b. Cone of light is absent or interrupted
a u d i t o r y c a n a l atresia. Corrective s u r g e r y is usually
c. Lateral process of mallous becomes more prominent
performed is: [Al 07]
d. None

EXPLANATIONS AND REFERENCES

1. Ans. is a, c and d i.e. aspergillus, Candida and Pseudomonas Ref. Current otolaryngology 2nd/ed pg 629,630
• Otitis externa is an inflammatory and infectious process o f t h e external auditory canal which is seen in all ages and b o t h sexes.
• M/C organism causing otitis externa are
a. Pseudomonas aeruginosa 0

b. Staphylococcus aureus
• Less c o m m o n l y isolated organisms are -
a. Proteus species
b. Staphylococcus epidermidis
c. Diphtheroids
d. E.coli

Fungal Otitis Externa/Otomycosis

• It is responsible for ~ 1 0 % o f t h e cases of otitis externa


• In 8 0 % of cases organism is aspergillus 0

• 2 n d M/C organism is Candida


Other more rare fungal pathogens include
• Phycomycetes
• Rhizopus
• Actinomyces
• Penicillium
2. Ans. is i.e. Telephonists ear Ref. Internet search
Humidity and hot climate are one d f the predisposing factors for otitis externa. Hence - otitis externa is also k/a Singapore ear
(where climate is hot & humid) or Telephonist ear as talking on phone causes humidity around ear) or Swimmers ear.

Also know

Pseudomonas aeruginosa is a normal inhibitant o f external ear. Its numbers are kept in balance by the normal acidity of EAC. j
Prolonged s w i m m i n g or abusive use of cotton typed ear buds can alter the pH, producing a more basic environment in which
pseudomonas grows rapidly.

3. Ans. is a and b i.e. Candida and aspergillus


4. Ans. is a i.e. aspergillus niger.
Ref. Dhingra 5th/ed pg 58,6th/edp 52; Current otolaryngoly 2nd/edpg 630; Scotts Brown 7th/ed vol 3 pg 3355

Otomycosis

• It is fungal otitis externa


• Accounts for» 9 - 1 0 % cases of otitis externa
• In 8 0 % of these cases, the etiologic agent is aspergillus (A. niger > A. fungatus) whereas Candida is the next most frequently
isolated fungus.
• Other more rare fungal pathogens include
r,L. » r 3
- Phycomycetes
- Rhizopus
Actinomyces
Penicillium

#
266 J_ SECTION V Ear

ALSO KNOW
• Otomycosis is c o m m o n in hot and h u m i d climate.
• It also occurs in patients using topical antibiotics for treatment of otitis externa or middle ear suppuration
• Clinical feature:

- Intense itching
• •
- discomfort
- discharge w i t h musty odor.
• On examination:
- A niger appears as black-headed filamentous g r o w t h .
- A fumigatus: as pale blue or green g r o w t h
- Candida: as w h i t e / c r e a m y deposit.
Ans. is d i.e. Virus Ref. Turner 1 Oth/ed p 323; Dhingra 5th/ed pg 62,6th/ed p 55
Myringitis bullosa hemorrhagica is a painful condition.
Characterized by formation of hemorrhagic blebs on tympanic membrane and deep meatus. It is probably caused by virus or
mycoplasma pneumoniae (Dhingra 6th/edp 62) but according t o Turner 10th/edp 323
"Myringitis bullosa hemorrhagica occurs in presence of viral infection, usually influenzae."

Myringitis granulosa is associated with impacted wax, long standing foreign body or external ear infection.

External ear condition Most common organism

Furunclosis Staphylococcus
Otomycosis Aspergillus niger (M/c); Candida albicans (2 nd
M/c)
Otitis externa hemorrhagica Influenza virus
Myringitis bullosa Influenza virus
Less commonly
Mycoplasma pneumoniae
Malignant otitis externa Pseudomonas aeruginosa
Perichondritis Pseudomonas
Myringitis granulosa Impacted wax
Foreign body

6. A n s . i s d i . e . 9 Ref. Dhingra 5th/ed pg 107, 6th/ed p 52, 96; Scotts Brown 7th/ed vol 3,260,3379-3382
Herpes zoster oticus / Ramsay Hunt syndrome -
- It is herpetic vesicular rash on the cochlea, external auditory canal or pinna w i t h lower motor neuron palsy o f t h e ipsilateral
facial nerve.
- It is k/a Ramsay Hunt syndrome following the first description of 60 cases by John Ramsay hunt in 1907.
- It may be accompanied by anesthesia of face, giddiness and hearing impairment due t o involvement of Vth and Vlllth nerve.
7. Ans. is a i.e. Influenza Ref. Dhingra 5th/edpg 58,6th/edp 52
Hemorrhagic external otitis media: (Otitis externa hemorrhagia) is caused by influenza virus.
• Characterised by formation of haemorrhagic bullae on tympanic membrane.
• Clinical features: severe pain and blood stained discharge.
• Treatment: Analgesics + antibiotics.
8. Ans. is a i.e. Ear Pack with 1 0 % ichthammol in glycerin wick Ref. Dhingra Sth/edp 57,51; Turner 1 Oth/ed p 272
Furuncle (Boil) is due to staphylococcal infection ofthe hair follicle.

Management

• Local heat + sedatives


• Packs of 1 0 % ichthammol (acts as antiseptic) and glycerin (hygroscopic action decreases edema). It is the commonest treatment
and most o f t h e furuncles burst spontaneously by this treatment.
• Antibiotics (Flucloxacillin) is given for 5 days
• If abscess is f o r m e d : Incision and drainage is done
• In case of recurrent furunculosis - Rule out diabetes and staphylococcal skin infection.
CHAPTER 21 Diseases of External Ear

9. Ans. is c is P. aeruginosa
10. Ans. is b i.e. Common in diabetics or old age
11. Ans. is is d i.e. Pseudomonas infection in diabetic patient
12. Ans is a, c and e i.e. Caused by pseudomonas aeruginosa; Granulation tissue is seen in the floor of external auditory canal;
and Gallium scan helpful for monitoring treatment
Ref. Dhingra 5th/ed pg 58,6th/ed p 52 Scott's Brown 7th/ed vol 3 pg 3336-3339; Harrison 17th/edpg 208

M a l i g n a n t Otitis Externa: Brief s u m m a r y


• It is an inflammatory condition of external ear."
• Most commonly caused by Pseudomonas.
• Other organisms responsible are: S. aureus, S. epidermidis, Aspergillus, Actinomyces.
• Seen in elderly d i a b e t i c s " / i m m u n o compromised patients"/patients on immunosuppressive d r u g . "
• Called as malignant because it behaves like a t u m o r in causing destruction of tissues of canal and pre and post auricular region
by various enzymes."
• Characterised by presence of granulation tissue in external auditory canal", at the j u n c t i o n of bony and cartilaginous p a r t . "
• It is very painful
• It can spread t o base of skull and temporal bone (causing osteomyelitis of temporal bone).
• As the infection spreads t o temporal bone, and base of skull, it may involve cranial nerves (mostcommon being facial nerve).
• Gold standard for diagnosis: positive technetium 99 bone scan.
•Treatment: high dose IV antibiotics
DOC
• Cefepime / ceftazidine (3rd generation cephalosporin).
• Antipseudomonal penicillin w i t h an aminoglycoside / fluoroquinolone.
13. Ans. a i.e. Malignant otitis externa Ref. Dhingra 5th/edp 58,6th/edp 52 Scott's Brown 7th/ed vol 3 p 3336-3339

An elderly diabetic patient


+
Painful ear discharge
+ All are highly suggestive of malignant otitis externa
Facial N palsy
+
No Response t o treatment
+ •
•I'ed uptake on T e c h n e t i u m bone scan

• Gold standard for diagnosis of malignant otitis externa is technetium 99 scan


• In refractory cases of otitis externa if it is not responding to antibiotics even after 7-10 days of treatment always suspect nalignant otitis externa
• M/c organism causing malignant otitis exteura = Pseudomonas

14. Ans. is b i.e. Pencillin Ref. Harrison 17th/edp208


Rapidly spreading infection of external auditory canal, seen in diabetic patient with involvement of bone and presence of
granulation tissue point towards malignant otitis externa as the diagnosis.

Treatment

• Includes correction of immunosuppression (when possible), local treatment o f t h e auditory canal, long-term systemic antibiotic
therapy, and in selected patients, surgery:
• In all cases, the external ear canal is cleansed and a biopsy specimen o f t h e granulation tissue sent for culture.
• IV antibiotics is directed against the offending organism.
• For Pseudomonas aeruginosa, the most common pathogen, the regimen involves an antipseudomonal penicillin or cephalosporin
(3rd generation piperacillin or ceftazidime) w i t h an aminoglycoside. A fluoroquinolone antibiotic can be used in place o f t h e
aminoglycoside.
• Ear drops containing antipseudomonal antibotic e.g. ciproflaxacin plus a glucocorticoid is also used.
• Early cases can be managed w i t h oral and otic fluoroquinolones only.
• Extensive surgical debridement once an important part of the treatment is now rarely needed.
268 ]_ SECTION V Ear

15. Ans. is c i.e. Mitotic figures are high Ref. Dhingra5th/edpg52,6th/edp52;Harrison 17th/edp208
"Malignant otitis externa is a misnomer where the term malignant doesnot indicate malignant pathology".\t is an inflammatory condi-
t i o n caused by pseudomonas infection. (So high mitotic figures will not be seen).
16. Ans. is c i.e. malignant otitis externa
17. Ans. is a i.e. Malignant otitis externa Ref. Dhingra 5th/ed pg 58,6th/ed p 52
Malignant otitis externa - can cause destruction o f tissues of canal, pre and post auricular region by various enzymes like leci-
thinase and hemolysis. Infection can spread t o skull base and jugular foramen causing multiple cranial nerve palsies in which
most common is facial nerve palsy. 0

18. Ans. is b i.e. desquamated epithelial cell + cholesterol Ref. Scott's Brown 7th/edpg/ed vol-3 pg-3342 Dhingra 5th/edpg 61

Keratosis Obturans
-

• It is accumulation of a large plug of desquamated keratin in the external auditory meatus


• Seen between 5 and 20 yrs of age (i.e. younger age as compared to cholestealoma which is seen in middle age)
• May be U/L or Bilateral (occasionally)
• It may be associated w i t h bronchiectasis and chronic sinusitis.
Clinical Features

• Pain in t h e ear (severe olatagia)


. Hearing loss (of conductive type)
• Tinnitus

• Ear discharge - sometimes

O/E
• Pearly w h i t e mass of keratin is visible in the ear canal
• Tympanic membrane is thickened
• Ear canal is ballooned

Treatment

Removal by syringing / Instrumentation


• Periodic checkup should be done t o see reaccumulation
• If it recurs - keratolytic agent - 2 % salicylic acid in alcohol can be tried.

The answer to this question should have been 'desquamated epithelium'only but since it is not given in options - we are choosing the next
best option.

19. Ans. is b i.e. Pseudomonas Ref. Turner 10th/edp 263; Harrison 17th/ed p 207
"Perichondritis of auricle is most commonly caused by pseudomonas pyocyanea". ... Turner 10th/edp263
"It is most commonly caused by pseudomonas aeuroginosa and staphylococcus aureus. ... Harrison 17th/ed p 207
20. Ans. is b i.e. Perichondritis in boxers Ref. Dhingra 5th/ed pg 56,6/e, p50; Current Otolaryngology 2th/edpg 649

Blunt trauma in boxers |


j
Hematoma of auricle (collection of blood
between auricular cartilage and perichondrium)

I Deformity called as cauliflower ear

Infection occurs in hematoma

severe perichondritis may be seen.

21. Ans. is c i.e. Resolve spontaneously Ref. Current Otolaryngology 2nd/ed p 649; Dhingra 5th/edpg 54,6th/edp 49
Hematoma of auricle
- M/c seen in boxers, wrestlers and rug by players
- Accumalation of blood in subperichondrial space, secondary t o blunt trauma lifting the perichondrium away from cartilage
CHAPTER 21 Diseases of External Ear

- As cartilage lacks its o w n blood supply and relies on the vascularity o f t h e perichondrium
- It leads t o necrosis of cartilage and predisposing t o infection
- New cartilage may then form at the pericondrium creating a rather thick deformed, unattractive ear called as cauliflower ear
- Treatment is aspiration of hematoma under aseptic condition and carefully packing the auricle
- All cases should receive prophylactic antibiotics
22. Ans. is b i.e. Posterior or none Ref. Dhingra 5th/ed pg 60,6th/ed p 53
In syringing (done to remove impacted wax) pinna is pulled upwards and backwards and a stream o f water from the ear syringe is
directed along the posterosuperior wall o f t h e meatus.
So t h e ans is either the o p t i o n b - posterior wall or none
23. Ans. is b i.e. 5 - 7 years Ref. Current Diagnosis and Treatment in Otorhinology 2nd/edp 627
Microtia: Here the patient presents at birth w i t h obvious auricular malformations

Classification

Grade I Grade II Grade III


Mild deformity like All pinna structures are present but tissue Also k/a classic microtia or peanut ear.
deficiency and significant deformity exist
• Low set ear (i.e. interiorly angled auricular There is no recognizable landmark of auricle. It
cartilage) includes anotia also
• Cupped ear (Has a deep conchal bowl)

Treatment

Classical treatment involves auricular reconstruction in multiple stages. Patients undergo observation until the age of 5 years t o
allow for g r o w t h o f rib cartilage which is harvested for reconstruction. This approach offers the benefit of reconstruction w i t h
autogenous material which ultimately requires little or no maintainance.Typically reconstruction occurs in 4 stages.
Stages in reconstruction of Microtia

A. Stage I Auricular Reconstruction


Goal - Symmetry in position o f t h e reconstructed
cartilaginous ear framework w i t h normal ear
Note - Watch for pneumothorax postoperatively
(as Rib is used)
After 2 - 3 months
I
B. Stage II Lobule Transposition After 3 months

Goal -To align the lobule w i t h the reconstructed cartilage


framework

C. Stage III Post Auricular skin grafting


Goal - A post auricular sulcus is created t o allow t h e ear
to
project away from mastoid.
Note - skin for creation of sulcus is harvested f r o m
groin, lowerabdomen, buttocks, contralateral posterior
sulcus or back
After several months
I
D. Stage IV Tragal Reconstruction and soft tissue debulking

24. Ans. is d i.e. none Ref. Dhingra 5th/edpg 61-62,6th/edp 55

Retracted Tympanic Bembrane

It is the result of negative intratympanic pressure when Eustachian t u b e is blocked


270 [ SECTION V Ear

Characteristics

• It appears dull and lusterless 0


• Cone of light is absent or interrupted 0

Q
• Handel of malleus appears foreshortened Q

• Lateral process of malleus becomes more p r o m i n e n t


Q

• Anterior and posterior malleal folds become sickle shaped Q

• It is immobile or has limited mobility w h e n tested w i t h pneumatic otoscope or siegle's speculum.


-

Features of Normal Tympanic M e m b r a n e

• It is shiny and pearly grey in colour


• Has concavity on its lateral surface
• Cone of light seen in antero - inferior quadrant
• It's transparency varies
It is mobile w h e n tested w i t h pneumatic otoscope or siegle's speculum.


CHAPTER

Diseases of Middle Ear


-

1. Otitis media refers to an inflammatory process within the middle Signs


ear cleft. • Tympanic membrane appears red and bulging with loss of
Otitis media can be either acute or chronic. There is no absolute landmarks (cartwheel appearance seen) . 0

time limit, but in general, disease that persists for more than 3 • 85% ofthe tympanic membrane rupture occurs in the antero-
months should be considered as chronic. inferior quadrant.
2. Eustachian tube is central to the pathogenesis of all forms of OM • Closure ofthe perforation in 90% of cases occurs in one month.
(with the possible exception of cholesteatoma). Any anatomic or • Tuning fork tests show conductive deafness.
functional obstruction of Eustachian tube can cause otitis media. • Facial paralysis in A.S.O.M is rare.
3. The more acute angle of ET in children as compared t o adults
is responsible for more prevalence of OM in children Treatment
4. In patients of Down syndrome, ET is abnormally patent or short
and it loses its normal protective function against reflux of Watchful waiting
nasopharyngeal contents which results in more cases of OM The current practice guidelines advise on an initial watchful
in this population. w a i t i n g w i t h o u t antibiotic therapy for healthy 2 yr old or
older children w i t h non severe illness (mild otalgia and fever
< 39°c) because AOM symptoms improve in 1 -3 days. Watchful
| A C U T E S U P P U R A T I V E OTITIS MEDIA (ASOM)
waiting is not recommended for children < 2yrs even in case
of uncertain diagnosis.
Acute inflammatiion of middle ear cleft < 3 weeks, infective in origin.
Antibiotics: Penicillin group - Amoxicillin (80 mg/kg/d) given
in 3 divided doses x 10 day is the drug of choice
Organism Analgesics
• Streptococcus pneumoniae (Mostcommon) Aural toileting
• H. influenzae ( 2 most common)
nd
Myringotomy:
• Moraxella catarrhalis
• Viral • Synctial virus
• Influenza virus Indications of Myringotomy:
• Rhino and adeno virus a. Tympanic membrane bulging and there is acute pain.
• It is one o f t h e most c o m m o n infectious disease seen in children b. Incomplete resolution with antibiotics and patient
• Peak i n c i d e n c e - f i r s t 2 years of life complains of persistent deafness
c. Persistent effusion>12 wk
Stages

Stage Stage of Pre- Stage of Stages of Prognosis


of tubal suppuration Suppuration Resolution/
Most o f t h e cases resolve w i t h o u t any adverse outcome. Rarely it
Occlusion complication
may lead t o the following complications.
Symptoms: • Deafness • Excruciating • Earache is
-
Deafness, • Deafness pain relieved Complications
Earache • Fever • Tympanic
membrane Intratemporal Intracranial
bulges and Facial paralysis Extradural abscess
finally ruptures Labyrinthine infections Subduralabscess
• Fever Mastoiditis Lateral sinus thrombophlebitis
Petrositis Otitc hydrocephlus

ft
\
272 ]_ SECTION V Ear

Investigations
Recurrent AOM is defined as > 3 episodes of ASOM in a 6 month • Tuning fork test: conductive hearing loss (20 - 40db)
period or > 4 episodes in a 12 month period, with complete • Audiometry shows conductive hearing loss provides an
resolution of symptoms and signs in between the episodes. assessment of the severity of hearing loss and is
• Hence important in monitoring the progress of the condition
and providing useful information for management decision
| ACUTE NECROTISING OTITIS MEDIA • Impedance audiometry shows Type B curve. It is a very useful
investingation in children.
Variant of ASOM, often seen in children suffering from measles, • X-ray mastoid: clouding of air cells
scarlet fever or influenza
Treatment
Organism (3 hemolytic streptococcus 1 • Medical: - Topical decongestants
Age group Infants, young children • - Antiallergics
Predisposing factor Children acutely ill with scarlet fever, • - Antibiotics - effect is short lived
measles, pneumonia, influenzae • Surgical: - M y r i n g o t o m y and g r o m m e t insertion
Features Necrosis and sloughing of the (Treatment of choice)
tympanic membrane, ossicles and • - Surgical management of causative factor i.e.
mastoid air cells VII N palsy seen adenoidectomy / tonsillectomy.
Symptoms Profuse foul smelling discharge
(due to necrosis of the tympanic B. Aero-otitis media/Ottic Baro trauma
mucoperiosteum) Aetiology
Treatment I.V, penicillin
• Rapid descent during air flight
In fulminant cases: i.m. gamma globulin
• Under water diving
is given • Compression in pressure chamber
In resistant cases: If acute mastoiditis
supervenes cortical mastoidectomy Pathogenesis
is done Atmospheric pressure increases more than that of middle ear by
critical level of 90mm Hg.
| NONSUPPURATIVE OTITIS MEDIA 1
Eustachian tube blocked
I
A. Serous Otitis Media/ Secretory Otitis Media/Otitis Negative pressure in middle ear
Media with Effusion/Mucoid Otitis Media/Glue Ear/ •i-
Silent Otitis Media Retraction of tympanic membrane
i
• Characterised by accumulation of non purulent effusion
in middle ear cleft. <r Hyperemia and engorgement of vessels
• It is common in 2-6 years of age.
Transudation and haemorrhage
Symptoms Symptoms
• Painless condition. • Severe earache
M/c symptoms is - Deafness: mild and often detected only • Deafness
with audiogram (It is the most common cause of hearing loss • Tinnitus
in children in the developed world)
Delayed and defective speech. Signs
• Feeling of blocked ears Air bubbles or haemorrhagic effusion in middle ear.
Signs Treatment
• Tympanic membrane appears dull with thin leash of blood Medical: • Oral and topical decongestants
vessels at the periphery. • Antihistaminics
• It is Yellow/dull grey in colour Surgical: • Myringotomy
• It is light reflex is absent
, i , . . . . . Preventive Measures
• Retracted and its mobility is restricted
Avoid air travel in presence of upper respiratory infection.
• Fluid levels and air bubbles are seen through it
Do not sleep during descent.
NOTE Chewing gum exercises should be done during descent.
Autoinflation of eustachian tube by valsalva should be done.
In case of glue ear - fluid is sterile.
Use vasoconsrictor nasal spray and systemic decongestant half a*
hour before descent in case of previous history of similar episode.
CHAPTER 22 Diseases of Middle Ear J 273

P a t h o g e n e s i s of T u b o t y m p a n i c t y p e

Barotrauma cannot occur in who have perforation of tympanic Unadequately treated Acute otitis media
membrane
T
Perforation of para tensa /central perforation
X
X X
EXTRA EDGE Exposure to environment Ascending infections Allergy |
Via eustachian tube
Lighthouse sign and pulsating otorrhea are seen in ASOM and
acute mastoiditis following ASOM. X
Silent otitis media or otitis media with effusion (OME) shows Symptoms: Ear discharge Hearing loss (conductive type)
• Mucopurulent
fluid level and air bubbles with no perforation in TM with B type
• Non foul smelling
(flat) curve on impedance audiometry. • Painless
In chronic adhesive ottitis media, adhesions form between
Flow chat 22.1: Pathogenesis of Tubotympanic variety
drum and middle ear, while in atelactatic ear there is complete
collapse of thin drum on the promontory. Treatment
Best treatment of adhesive otitis media is hearing aid. Medical
Fluctuating deafness of conductive nature is seen in secretory otits Treatment of choice:
media, while fluctuating SNHL is a feature of Meniere's disease. » Aural toilet - It is an i m p o r t a n t step in treatment and should
Potbelly tympanic membrane is a feature of secretory not be missed
otitis media. • Topical and systemic antibiotic
Surgical: done at a later stage t o correct the hearing loss

| CHRONIC SUPPURATIVE OTITITS MEDIA Prerequisites


• Ear dry for 6 weeks w i t h o u t antibiotics
It is of 2 Types: A. Tubotympanic • Eustachian tube function normal
B. Atticoantral • Normal middle ear mucosa
Procedure of choice: myringoplasty - if ossicle chain is intact;
A. T u b o t y m p a n i c T y p e
tympanoplasty - ifossicularchainisdisrupted
• It is particularly prevalent in developing countries and is
most c o m m o n in low socio economic group B. A t t i c o a n t r a l T y p e : U n s a f e o r D a n g e r o u s T y p e
• Most c o m m o n organisms isolated are - P. aeruginosa, S. • Involves posterosuperior part of middle ear cleft i.e. attic,
aureus and proteus species. antrum and mastoid
• It is safe or benign type of CSOM and involves anteroinferior • Associated w i t h an attic or a marginal perforation.
p a r t o f m i d d l e ear c l e f t , i.e. E u s t a c h i a n t u b e a n d • Associated w i t h cholesteatoma:
mesotympanum and is associated w i t h central perforation. • Risk of complication is high
• There is no risk of serious complication. Cholesteatoma is the presence of keratinising stratified squamous
epithelium w i t h i n the middle ear cleft.
• M/C in children.
Pathogenesis: see flow chart 22.2.

Flow chart 22.2: Pathogenesis o f atticoantral type

Eustachian tube dysfunction

T
Acute necrotising otitis media
I Persistent negative middle ear pressure X

T
Marginal perforation
Retraction pocket (mostly in pars flaccida T.
but may occur sometimes in pars tensa) Epithelial migration in the
perforation
Primary acquired cholesteatoma X
Secondary acquired cholestatoma
1 1 i
Due to liberation By pressure ischaemia By enzymatic bone destruction
of chemicals leading to necrosis (most acceptable therory)

X
I Scanty foul smelling discharge (like dead mouse/rotten fish)

Osteitis

Granulations with red fleshy polyp

Ossicular necrosis - -

X
Hearing loss (mainly conductive type but in long standing cases can be of mixed type).

-
274 T SECTION V Ear

Clinical Features
Extra E d g e
Scanty foul smelling discharge.
Ciliated columnar e p i t h e l i u m lines the eustachian t u b e ,
Conductive type of hearing loss
anterior mesotympanum and inferior hypotympanum, while
Tinnitus may be present cuboidal epithelum lines the attic, mastoid and posterior
Bleeding: in case of polyps/granulation tissue mesotympanum.
It can lead t o facial nerve twitching, palsy or paralysis Simple patch test helps t o f i n d o u t t h e i n t e g r i t y o f
Signs ossicular chain, hence t o decide w h e t h e r myringoplasty
or tympanoplasty needs t o be done in case of safe CSOM.
Marginal posterosuperior or attic perforation w i t h granulation Hearing in CSOM is better w h e n the ear is discharging due
tissue or pearly white flakes of cholesteatoma t o shielding effect of round w i n d o w or discharge covering
Imaging study: CT scan is the investigation of choice the perforation.
Posterior perforation tends t o have more hearing loss due
Treatment
t o loss o f sound protection for round window. Larger the
• Surgical: It is the mainstay of t r e a t m e n t .
0
perforation, greater the loss of surface area on w h i c h sound
• Primary aim is removal of disease by mastoidectomy t o make pressure can act.
ear safe followed by reconstruction of hearing at a later stage. In safe CSOM perforation lies in pars tensa.
• Surgery of choice: Modified Radical mastiodectomy 0
In unsafe CSOM perforation lies in pars flaccida.

The newer classification of Chronic otitis media

Mucosal disease (i.e. Tubotympanic disease) Squamosal disease (i.e. Atticoantral disease)

£ r I.
Active Inactive Healed Inactive Active
I 7
i.e. there is a perforation of
partensa with inflammation
Permanent perforation
partensa is seen but
T
When tympanic
membrane has healed
Retraction pockets
present
Cholesteatoma present
It erodes bone, forms
of mucosa and mucopurulent middle ear mucosa is (in 2 layers) is atrophic No discharge granulation tissue and
discharge (chronic not inflammed and there and easily retracted if Also k/a has purulent offensive
suppurative otitis media) is no discharge there is a negative atelectatic ear discharge
pressure in middle ear.
There may be some
areas of tympanosclerosis
present

Discharge becomes profuse and increases in purulence. It may


COMPLICATIONS OF OTITIS MEDIA be pulsatile (light house effect)

Extra cranial complications Intracranial complications


• Mastoiditis • Meningitis
Any persistence of discharge beyond 3 weeks, in a case of acute otitis
• Petrositis/Gradenigo syndrome • Extradural abscess media points to mastoiditis.
• Facial paralysis • Subdural abscess
• Labyrinthitis • Otogenic Brain Abscess Signs
• Osteomyelitis of temporal bone • Lateral sinus
• Septicaemia or pyaemia thrombophlebitis • Tenderness over the mastoid antrum / suprameatal triangle
• Otogenic tetanus • Otitic hydrocephalus • Retroauricular swelling: Over the mostoid which pushes pinna
forwards and downwards and obleterates the retro auricular
sulcus
1 ACUTE MASTOIDITIS
• Posterosuperior canal wall sagging
• M/C extracranial intratemporal complication o f acute otitis • Tragal sign -ve
media. • Movement of pinna is not painful:
• Inflammation o f t h e mucosal lining o f t h e mastoid antrum and Ironed out appearance of skin over mastoid due t o thick
its air cell system. periosteum is the first sign of acute mastoiditis
• Organism (most common): streptococcus pneumoniae Condutive type of hearing loss present.

Symptoms X-ray M a s t o i d

• Recurrence of Pain behind the ear after ASOM. • Clouding of air cells due t o collection of ex udates in t h e m
• Fever (Its pesrsistence or recurrence inspite o f adequate • Bony partition between the cells becomes indistinct.
treatment).
CHAPTER 22 Diseases of Middle Ear

T y p e of A c u t e Mastoiditis

Acute mastoiditis can be staged as: If a subperiosteal abscess or an intracranial extension of disease is
1. Acute mastoiditis without periostitis/osteitis suspected, surgery in combination with high dose 1/v antibiotics
It is the extension o f t h e pathological process of acute should be 1 st line of therapy.
middle ear infection. No periostitis or osteitis of the
mastoid is present.
| PETROSITIS/GRADENIGO'S SYNDROME
2. Acute mastoiditis with periostitis
Infection w i t h i n the mastoid spreads t o periosteum
i.e. Infetion of ear/mastoid spreads to petrous part of temporal bone.
covering t h e mastoid process. The route of infection
from the mastoid cells t o the periosteum is by venous Classical presentation of petrositis is gradenigo's syndrome i.e.,
channels, m o t commonly the mastoid emissary vein. triad of (3D)
3. Acute mastoiditis with osteitis • Persistent ear Discharge: otorrhoea
Also called as acute coalescent mastoiditis or acute • Diplopia (due t o VI nerve involvement)
surgical mastoiditis. Basic pathology is osteitis, in which
• Deep seated orbital or retro-orbital pain (due t o Vth nerve
necrosis and demineralization o f t h e bony trabeculae
occur. From this stage onward disease progression involvement)
depends o n the direction in which the erosive process Sudden disappearance of s y m p t o m s in g r a d e n i g o s y n d r o m e
goes: suggests intracranial rupture.
• Most commonly, mastoid cortex is eroded and a
subperiosteal abscess develops.
• Medial progression causes petrositis and Grad-
Persistent ear discharge in cases of cortical or modified redical
enigo's syndrome.
mastoidectomy is due to Petrositis
Anterior progression can compromise the fallopi-
an canal or labyrinth causing facial palsy or vertigo.
• I n f e c t i o n in t h e c r a n i u m causes i n t r a c r a n i a l | FACIAL PARALYSIS
complications meningitis, abscess, lateral sinus
thrombophlebitis, otitisc hydrocephalus. For details see chapter: Facial Nerve and its disorders.
Classically, the term mastoiditis referred t o acute co-
alescent mastoiditis w i t h superiosteal abscess lateral

I
LATERAL SINUSTHROMBOPHLEBITIS/SIGMOID
t o the mastoid cortex occurring 2 weeks after onset
SINOUS THROMBOSIS
of ASOM.
Positive mastoid reservoir sign is seen in coalescent
May occur as a complication of:
mastoiditis in which there is rapid re-accumulation of
• Acute coalescent mastoiditis
discharge after cleaning up occurs i.e., pus fills up again
on mopping. • CSOM and cholesteatoma

A b s c e s s e s in R e l a t i o n t o M a s t o i d I n f e c t i o n Clinical Features

Patient presents w i t h :
Postauricular Abscess
• Picket fence type of fever w i t h rigors i.e. fever rises twice during
Zygomatic Abscess Due t o Infection o f the zygomatic air day reaching 104° or 105°F and comes t o normal.
cells
Fever coincides w i t h release of septic embolic into blood
Bezolds Abscess Pus passes t h r o u g h Pus passes t h r o u g h stream.
Sternocieoidomastoid sheath
Patient is alert w i t h sense of well-being in between bouts
Citellis Abscess Pus passes f r o m Mastoid inner table of fever.
Posterior belly of digastric/occipital • Headache
bone
Meatal (Luc's) Abscess Pus passes"between t h e a n t r u m and Signs
external osseous meatus Progressive anaemia and emaciation:
• Torticollis of neck
Treatment • Griesinger's sign: due t o thrombosis of mastoid emissary vein.
There is oedema over posterior part of mastoid.
• I.V. antibiotics
• Myringotomy: If pus under tension • Tobey - Ayer test (queckenstedt's test): compression o f jugular
• Cortical mastoidectomy: vein on thrombosed side does not produce any change in CSF
In case of intracranial / intratemporal complications pressure. Whereas compression of jugular vein on healthy side
If patient's condition deteriorates after 24 hours despite raises CSF pressure.
adequate treatment • Crow -Beck test
2761_ SECTION V Ear

Compression of jugular vein on healthy side Clinical Feature

• Temporal lobe abscess can present as:


Engorgement of retinal veins and supraorbital veins. If there is a Nominal aphasia
thrombosed sinus, no such change is seen. Homonymous hemianopia (earliest focal sign).
Contralateral motor paralysis
Tenderness along jugular vein.
- Epileptic fits
Contrast enhanced CT/MRI show a typical delta sign. It is a
Hallucinations of taste and smell
triangular area w i t h rim enhancement and central low density
Occulomotor palsy
area is seen in posterior cranial fossa on axial cut. • Cerebellar abscess:
Ipsilateral spontaneous nystagmus
INTRACRANIAL COMPLICATIONS Ipsilateral ataxia
- Past pointing
1 EXTRADURAL ABSCESS Intentional tremors
- Dysdiadochokinesia
It is collection of pus between bone and dura. It is called:
Treatment
Epidural abscess Perisinus abscess
i 1 Medical = high dose 1/v antibiotics + for raised ICT->
If abscess lies medial t o sigmoid sinus If abscess, encloses dexamethasone or mannitol.
Surgical
the sinus
Drainage of abscess
In the associated ear = Modified Radial mastoidectomy
| SUBDURAL ABSCESS
in CSOM w i t h cholesteatoma
Collection of pus between dura and arachnoid
| OTITIS H Y D R O C E P H A L U S
| MENINGITIS
Rare complication:
• It is the second most common complication of otitis media • Characterised by raised intracranial pressure w i t h normal CSF
findings.
Turner 10/e,p 311
• Mostcommonorganismresponsibleforoticmeningitisare -S. • Caused due t o thrombus extending t o superior sagittal sinus
pneumoniae and - H. influenza Type B which impedes the function of arachnoid villi t o absorb CSF
and therefore cause ICT.
• Positive kernig's sign i.e. painful extension of leg on flexed thigh
• Positive brudzinski's sign i.e. flexion of neck causes flexion o f
hip and knee.
j Potsitive babinski sign i.e., extension of big toe on stimulation Remember:
of lateral aspect of sole. MRI is the IOC in extradural, Bezold and cerebral abscess. CT is
the IOC in cases of coalescent mastoiditis.
-
| BRAIN ABSCESS

• It is the most c o m m o n complication of chronic otitis media. SURGICAL MANAGEMENT OF


... Turner 10/e,p 311
• Conversely ear infections are the most c o m m o n cause of brain
MIDDLE EAR SUPPURATION
abscess. ...Turner 10/e,p 311-312
| INCISIONS F O R EAR S U R G E R Y
• 5 0 % brain abscess in adults and 2 5 % brain abscess in children
are otogenic in origin Postaural (William Wilde's) and endaural (Lempert's) incisions
are used in mastoidectomy and tympanoplasty.
Brain abscess is of 2 types
Endomeata (Rosen's) incision is used in stapedectomy and in
Cerebral abscess Cerebellar abscess tympanoplasty.
(M/C temporal abscess)
| MYRINGOTOMY
Cerebral abscess is seen t w i c e as f r e q u e n t l y as cerebellar
abscess and M/C site of cerebral abscess is Temporal lobe
Incising the tympanic membrane t o drain the middle ear.
Cerebellar abscess can develop as direct extension t h r o u g h
Can be coupled w i t h insertion of ventilation t u b e (grommet)
Trautmann's triangle.
Microbilogy: G-ve organisma (proteus, E. Coli, P seudomon Indication
and arearobic bacteria along w i t h staphylococci) 1. Acute otitis media: Indications in AOM are:
• Severe pain (bulging red tympanic memberane)
CHAPTER 22 Diseases of Middle Ear

• AOM going in for complications Nasal allergy


• Unresolved AOM Otitis externa
• AOM occurring during antibiotic therapy Ingrowth of squamous epithelium into the middle ear
• AOM in immunoceficiency When the other ear is dead or not suitable for hearing aid
• Recurrent AOM (along w i t h g r o m m e t insertion): rehabilitation
More than 3 episodes of ASOM in 6-6 episodes in 12 months.
Children < 3 years
Patientshould be free of infection in between the episodes.
Predisposing causes include adenoid hypertrophy, nasal
allergy, chronic sinusitis, cleft palate, and other causes
j TYMPANOPLASTY
o f velopharyngeal insufficiency, craniofacial anomalies,
immunodeficiency, and GERD. Eradication of disease f r o m middle ear along w i t h repair, which
2. Otitic barotrauma for drainage and unblocking Eustachian
includes ossicular reconstruction w i t h or w i t h o u t myringoplasty.
tube.
Possibly it is the commonest surgery done in CSOM.
Myringotomy is coupled with grommet insertion in:
T y p e s of T y m p a n o p l a s t y
• Suppurative or serous otitis media
• Recurrent Acute otitis media Wullstein and zollner (1953) classified tympanoplasty into types:
• Adhesive otitis media • T y p e I: It d i f f e r s f r o m s i m p l e c l o s u r e o f p e r f o r a t i o n
• Meniere's disease (myringoplasty) in that here middle ear is also examined to
rule o u t any pathology.
P r e f e r r e d S i t e for M y r i n g o t o m y • Type II: It is done where there is disease in atticoantral region
w i t h mild erosion of malleus or incus Temporalis fascia graft is
^onditioii Site
placed on the incus or remnant of maleus.
Acute Suppurative
Otitis Media (ASOM)
Circumferential incision is made
in the posterio-inferior quadrant
• Type III: M/C type of tympanoplasty (Columellar type or
effect): It is done when malleus and incus are destroyed b u t
of tympanic membrane, midway
stapes is healthy. Graft is placed on the head of stapes. It is also
between handle of malleus and
tympanic annulus. called myringostapediopexy/columella effect.This columellar
effect is usually present in birds.
A small radial incision is given in
antero-inferior quadrant. • Type IV: All ossicles including stapes head are eroded. Graft
is placed in such a way that a small air-containing cavity w i t h
Serous Otitis Media ± Eustachian t u b e and round w i n d o w is created (cavum minor).

0
growmet insertion Footplate o f t h e stapes should be mobile and is left exposed
t o sound waves.
« Type V: It is also called fenestration operation. Here footplate of
stapes is fixed, but round w i n d o w is functioning. In such cases
another w i n d o w is created on horizontal semicircular canal.
• Commonest ossiculoplasty material is autograft incus (incus
transposition). Others are autograft tragal/septal cartilage,
homograft ossicle and prosthetic materials.
Prosthetic materials are made up of Teflon, ceramic, ititanium,
Myringotomy was first performed by astley cooper for serous gold.
otitis media
Myringotomy is contraindicated in case of suspected intratym-
CORTICAL MASTOIDECTOMY/SIMPLE MASTOIDEC-
paic glomus tumor- In such a case tympanotomy should be done.
TOMY/SCHWARTZ OPERATION

j MYRINGOPLASTY Simple mastoidectomy/Schwartz operation.


Repair of tympanic membrane defect (In Pars tensa) Involves exenteration of all accessible mastoid air cells w i t h o u t
• Commonest graft material used is temporalis fascia. taking d o w n the posterior meatal wall.
• Other materials include tragal perichondrium, Fat and vein
Indication
(autografts), or cadaveric dura and vein (homografts).
• Acute coalescent mastoiditis
Indication • Incompletely resolved otitis media w i t h reservoir sign
• Masked mastoiditis
Is a perforated tympanic membrane w i t h only mild conductive
• As an initial step t o perform:
hearing loss, which implies a normal ossicular chain.
Endolymphatic sac surgery
-
Decompression of facial nerve
Contraindications
Translabyrinthine or Retrolabyrinthine procedure, for
• Active discharge f r o m middle ear acoustic neuroma.
278^ SECTION V Ear

| RADICAL MASTOIDECTOMY all healthy mucosa, remnants of t y m p a n i c membrane a n d


ossicles are preserved t o facilitate tympanoplasty later o n .
Aims at exenteration a n d exteriorization. No reconstruction is • This is t h e treatment of choice for attico artrum disease and
attempted. resectable cholesteatoma of middle ear and mastoid including
The disease f r o m t h e m i d d l e ear a n d mastoid is exenterated, complications.
middle ear, attic, a n t r u m , and mastoid air cells are coverted into
a single cavity by taking d o w n the posterior canal wall and thus Measures to Avoid Injury to Facial Nerve d u r i n g Mas-
exteriorized. The w h o l e mucosa of the middle ear, remnants of toidectomy
t y m p a n i c membrane, and ossicles except stapes are removed. • Change t o higher power of microscope near facial nerve.
The middle ear is closed off by curetting the Eustachian t u b e and • Adequate irrigation t o avoid thermal injury.
plugging w i t h muscle. No attempts are made t o pressure hearing. • Avoid using cutting burr near t h e nerve (use d i a m o n d burr
instead).
Indications • Use the burr along the direction o f t h e nerve - never across.
• Malignancy o f t h e external ear and middle ear. • Never pull o u t granulations o n the nerve.
• Unresectable cholesteatoma, scarring, eustachian tube orifice,
and producing severe sensorineural hearing loss.
Mastoidectomy is one ofthe commonest causes of iatrogenic
• If previous attempts to eradicate cholosteatoma have failed facial palsy
Commonest site of injury to the facial nerve during
Modified Radical Mastoidectomy mastoidectomy is the 2nd genu).
• Here in a d d i t i o n t o e x e n t e r a t i o n a n d e x t e r i o r i z a t i o n , Focal Length of the Objective Lens of the Operating
reconstruction ofthe hearing mechanism is also attempted. So in Microscope Used for Ear Surgeries
200-250 mm.
addition to creating an open cavity as in radical mastoidectomy
Note: It is 300 mm for nasal surgeries and 400 mm for
microlaryngeal surgeries

Another way of classifying mastoidectomy is based on the approach to mastoid


Canal wall up procedure Canal wall down procedure
• Posterior canal wall is left intact Posterior canal wall is removed thereby exteriorizing the mastoid into the external ear. It can be
• Middle ear is approached through facial done as
recess in mastoid
• Includes posterior tympanotomy Modified radical mastoidectomy Radical mastoidectomy
and simple/cortical mastoidectomy Attempt is made to preserve as much hearing No attempts are made to preserve hearing
(Schwartz operation) Consists of complete as possible.
exenteration of all accessible mastoid air Steps: Steps:
cells and converting them into a single • Post meatal wall and lateral attic wall are • Post meatal wall is removed
cavity removed • TOE are all removed
• Middle er structures are not disturbed • TOE i.e.Tympanic membrane remnant, • Entire area of middle ear, attic, antrum and
Indications - (MAM) Ossicles and mastoid are converted to a single cavity
M Acute coalescent mastoiditis Eustachian,Tube functions are preserved.
A Acute otitis media with reservoir sign
M Masked mastoiditis
Drawback: Indications: Indications:
Associated with high incidence of residual/ • Cholesteatoma confined to attic and • When cholesteatoma can not be removed
recurrent cholesteatoma antrum safely or if previous attempts have failed
• Localised chronic otitis media

"A canal wall up mastoidectomy with ossicular reconstruction may be considered only in patients with chronic otitis media without any evidence ofevidence of
cholseteatoma".... Otology and Neurology, Inc, Vol. 2615, Sept. 05, p 1045-1051
More importantly canal up technique is the surgical approach for cochlear implant
-

| SUPRAMEATAL (MACEWEN'S) TRIANGLE | CITELLI'S ANGLE

It is b o u n d e d superiorly by the supra mastoid crest, anteriorly by Citelli's angle is sino dural angle (angle between the plate of bone
the posterosuperior canal wall and a trangential line f r o m here separating the sigmoid sinus from the mastoid cavity (sinus plate!)
t o t h e supremastoid crest completes t h e triangle. A n t r u m lies and the plate of bone separating middle cranial fossa dura from
approximately 1.5 c m deep t o the triangle in adults. It is the Surgical the mastoid cavity [dural plate]). This is a common site of residual/
Landmark for Mastoid A n t r u m during Mastoidectomy. recurrent disease after surgery.


CHAPTER 22 Diseases of Middle Ear J 279

QUESTIONS

ACUTE SUPPURATIVE OTITIS MEDIA 12. Secretory otitis media is diagnosed by:
a. Impedance audiometry
[PGI June 98]

b. Pure tone audiometry


Commonest cause of acute otitis media in children is:
c. X-ray
[AIIMS June 00; Delhi- 06; UP-03]
d. Otoscopy
a. H. inflenzae b. S-pneumoniae
13. Bluish tympanic membrane is seen in: [JIPMER 93]
c. S aureus d. Pseudomonas a. Early ASOM b. Glue ear
Commonest causative organism for ASOM in 2 years child c. Cholesteatoma d. Cholesterol granuloma
is: [AIIMS Dec. 95; 91] 14. Treatment of choice for glue ear is: [AIIMS May 07]
a. Pneumococcus b. H. influenzae a. Myringotomy with cold knife
c. Staphylococcus d. Streptococcus b. Myringotomy with diode laser
True statement about ASOM is:[A/ 99] c. Myringotomy with ventilation tube insertion
a. Most frequently it resolves without sequelae d. Conservative treatment with analgesics and antibiotics
b. Commonly follows painful parotitis 15. 6 year old child with recurrent URTI with mouth breathing
c. Radical mastoidectomy is required for treatment and failure to grow with high arched palate and impaired
d. Most c o m m o n oganism is pseudomonas hearing is: [AIIMS May 07]
Cart Wheel sign is seen in: [MP 2008] a. Tonsillectomy
a. ASOM b. AOM b. Grommet insertion
c. OME d. CSOM c. Myringotomy with grommet insertion
Acute suppurative otitis media is treated using all except: d. Adenoidectomy with grommet insertion
[AIIMS 91] 16. A child p r e s e n t i n g w i t h recurrent r e s p i r a t o r y tract
a. Erythromycin b. Penicillin infection, m o u t h b r e a t h i n g a n d d e c r e a s e d h e a r i n g
c. Streptomycin d. Cephalosporin Treatment is: [PGI- 08]
A child p r e s e n t s w i t h b a r o t r a u m a p a i n . T h e r e is no a. Tonsillectomy b. Adenoidectomy
inflammation of middle ear, management is: c. Grommet insertion d. Myringotomy
e. Myringoplasty
[Jharkhand 03]
17. Following statements are true about otitis media with
a. Antibiotics b. Paracetamol
effusion in a child: [PGI Dec. 03]
c. Suppurative d. Grommet tube insertion
a. Immediate myringotomy is done
Pulsatile otorrhoea seen in: [AP97]
b. Type B tympanogram

a. Glomus tumour b. CSF otorrhea


c. The effusion of middle ear is sterile
c. ASOM d. Fistula
d. Most c o m m o n cause of deafness in a child in day care
patients
NON SUPPURATIVE OTITIS MEDIA 18. In s e r o u s otitis m e d i a w h i c h o n e of t h e f o l l o w i n g
statements is true? [2000]
8. Aboy with ASOM undergoing treatment with penicillin
a. Sensorineural deafness occurs as a compli-cation in 8 0 % of
therapy for 7 days now presents with subsidence of pain
the cases
and persistence of deafness, diagnosis is:[Kolkatta 2003]
b. Intracranial spread o f t h e infection complicates the clinical
a. Ototoxicity b. Secretory otitis media
courses
c. Adhesive otiti media d. Tympanosclerosis
c. Tympanostomy tubes are'usually required for treatment
9. Cause of U/L secretory otitis media in an adult is:
d. Gram-positive organisms are grown routinely in culture in
[PGIDec.99/UP-04] the aspirate
a. CSOM b. Nasopharyngeal carcinoma 19. All except one are true in a case of secretory otitis media:
c. Mastoiditis d. Foreign body of external ear [MAHE07]
10. Acute non suppurative otitis media in adults is due to: BlueTM
[UP 2003] B Shaped tympanogram
a. Allergic rhinitis b. URTI Marginal perforation m o s t c o m m o n
c. Trauma d. Malignancy Rinne test-fve
11. Glue ear: [DNB 2003] 20. Medical treatments is NOT effective in which type of
a. Is painful suppurative media: [UP 07]
b. Is painless a. Tuberculous OM b. Secretory OM
c. Radical mastoidectomy is required c. Acute suppurative OM d. Chronic suppurative OM
d. NaF is useful
280 {_ SECTION V Ear

21. Which ofthe following is characteristic of T.B otitis media: c. Eardrops are best
[AIIMS May 95] d. Ottic hydrocephalus is a known complication
a. Marginal perforation b. Attic perforation e. Common in females than males
c. Large central perforation d. Multiple perforation 32. Levinson's criteria for diagnosing congenital cholesa-
22. Tuberculous otitis media is characterized by all except: teatoma includes: [PGI Nov. 2010]
[(AIIMS 1994) (AMU2000)(AP 1996) (Delhi 1985, 1991, 1992, a. Whitish mass behind intact TM
2003) (Kerala 1998) (PG11999 Dec, PG1 1996) b. Normal pars tensa and pars flaccida
(AP2004)] c. Recurrent attacks of otorrhea
a. Multiple perforations b. Pale granulations d. Prior otitis media is not an exclusion criteria
c. Pain d. Thin odourless fluid 33. Scanty, foul smelling, painless discharge from the ear is
characteristic feature of which o f t h e following lesions:
[AIIMS Nov. 00; 04]
CHRONIC SUPPURATIVE OTITIS MEDIA a. ASOM b. Cholesteatoma
23. Cholesteatoma is commonly caused by: [AI94] c. Central perforation d. Otitis externa
a. Attico-antral perforation 34. True about cholesteatoma is/are: [PGI Dec. 02; 06]
b. Tubotympanic disease a. It is a benign tumour
c. Central perforation of tympanic membrane b. Metastasizes to lymphnode
d. Meniere's disease c. Contains cholesterol
24. Cholestatoma is usually present at: [Delhi 01] d. Erodes the bone
a. Anterior quadrant of tympanic membrane . e. Malignant potential
b. Posteroinferior quadrant of tympanic membrane 35. Cholesteatoma commonly perforates: [PGI 00]
c. Attic region a. Lat. Semicircular canal b. Sup. semicircular canal
d. Central part c. Promontory d. Oval window
25. Cholesteatoma occurs in: [AIIMSMay 94] 36. Cholesteatoma (Atticoantral) true about: [PGI June 06]
a. CSOM with central perforation a. Scanty, malodorous discharge
b. Masked mastoiditis b. Otalgia
n
c. Coalescent mastoiditis ; c. Central perforation
d. Acute necrotizing otitis media d. Ossicular invovement
26. Cholesteotoma is seen in: [RJ 2006] e. Eustachian tube dysfunction
a. ASOM b. CSOM 37. The treatment of choice for atticoantral variety of chronic
c. Secretory ottitis media d. Osteosclerosis suppurative otitis media is: [AIIMS Nov. 02]
27. Most a c c e p t e d t h e o r y for t h e f o r m a t i o n of c h o l e s - a. Mastoidectomy b. Medical management
teatoma: [DNB 2001] c. Underlay myringoplasty d. Insertion of ventilation tube
a. Congenital 38. Treatment of choice for Perforation in pars flaccida of
b. Squamous metaplasia the tympanic membrane with cholesteatoma is: [A196]
c. Ingrowth of squamous epithelium a. Myringoplasty
d. Retraction pocket b. MRM -
28. Perforation of tympanic membrane with destruction of c. Antibiotics
tympanic annulus is called: [Bihar 2004] d. Radical mastoidectomy
a. Attic b. Marginal 39. T h e postero superior retraction pocket, if allowed to
c. Subtotal d. Total progress, will lead to: [Al 03]
29. What is true in case of perforation of pars flaccida: a. Sensori-neural hearing loss
[AIIMS May 93] b. Secondary cholesteatoma
a. CSOM is a rare cause c. Tympanoscelerosis
b. Associated with cholesteatoma d. Tertiary cholesteatoma
c. Usually due to trauma 40. Most difficult site to remove c h o l e s t e a t o m a in sinus
d. All of the above tympani is related with: [Kolkatta2 001]
30. Treatment of choice in central safe perforation is: a. Anterior facial ridge b. Posterior facial ridge
[Al 94] c. Epitympanum d. Hypotympanum
a. Modifed mastoidectomy 4 1 . A child presents with ear infection with foul smelling
b. Tympanoplasty discharge. On further exploration a small perforation is
c. Myringoplasty r found in the pars flaccida of the tympanic membrance.
d. Conservative management Most appropriate next step in the management would
31. True about C S O M : [PGI Dec. 00] be: [AIIMS Nov. 07]
a. Etiology is multiple bacteria a. Topical antibiotics and decongestants for 4 weeks
b. Oral antibiotics are not affected b. IV antibiotics and follow up after a month
CHAPTER 22 Diseases of Middle Ear J 281

c. Tympanoplasty 53. Essential radiological feature of acute mastoiditis is:


d. Tympano-mastoid exploration [UP-03]
42. A 5 year old boy has been diagnosed to have posterior a. Temporal bone pneumatisation
superior retraction pocket. All would constitute part of b. Clouding of air cells of mastoid
the management except: [Al 03] c. Rarefaction and tuning of petrous bone
a. Audiometry b. Mastoid exploration d. Thickening of temporal bone
c. Tympanoplasty d. Myringoplasty 54. In Mastoiditis tenderness is/are present at:
43. Ossicle M/C involved in CSOM: [Kolkatta 04] [PGI Nov. 2010]
a. Stapes Tragus
b. Long process of incus Concha
c. Head of malleus Mastoid tip
d. Handle of malleus Root of Zygoma
Mastoid antrum
55. Mastoid tip is involved in: [UP-06]
COMPLICATION OF OTITIS MEDIA
a. Bezold abscess b. Luc abscess
44. The most common complication of chronic supprative c. Citelli abscess d. P a r a p h a r y n g e a l abscess
otitis media is: [UPSC 05] 56. Bezolds abscess is located in: [AIIMS 92, DNB-07]
a. Meningitis b. Intracerebral abscess a. Submandibular region b. Sternomastoid muscle
c. Cholesteatoma d. Conductive deafness c. Digastric triangle d. Infratemporal region
4 5 . Commonest complication of CSOM is: 57. The diagnosis in a patient with 6th nerve palsy, retro
[Corned 08, DNB-07] orbital pain and persistent ear discharge is:
a. Sub periosteal abscess b. Mastoiditis [PGIJune 99]
c. Brain absess d. Meningitis a. Gradenigo's syndrome b. Sjogrens syndrome
46. Commonest complication of CSOM is: [Karn 96] c. Frey's syndrome d. Rendu osier weber disease
a. Conductive deafness b. Meningitis 58. All are true for Gradenigo's syndrome except: [Al 05]
c. Temporal lobe abscess d. Cholesteatoma a. It is associated with conductive hearing loss
47. Most c o m m o n complication of acute otitis media in b. It is caused by an abscess in the petrous apex
children: [SRMC02] c. It leads to involvement o f t h e Cranial nerves V a n d VI.
a. Deafness b. Chronic mastoiditis d. It is characterized by retro-orbital pain
c. Cholesteatoma d. Facial nerve palsy 59 Gradenigo's syndrome characterised by: [PGIDec. 02]
48. Extracranial complications of CSOM: [PGI Dec. 02] a. Retroorbital pain
a. Epidural abscess b. Profuse discharge from the ear
Facial nerve plasy c. VII nerve palsy
Hearing loss d. Diplopia
Labyrinthitis 60. Treatment of cholesteatoma with facial paresis in child
Sigmoid sinus thrombosis is: [AIIMS 93]
49. Extracranial complication(s) of CSOM: [PGIJuneOI] a. Antibiotics to dry ear and then mastoidectomy
a. Labyrinthitis b. Immediate mastoidectomy
Otitic hyrocephalus c. Observation
Bezold's abscess d. Only antibiotic ear drops
Facial nerve plasy 61. Treatment of choice for CSOM with vertigo a n d facial
Lateral sinus thrombophlebitis nerve palsy is: [Al 96]
50. Most common extra-cranial complication of ASOM is: a. Antibiotics and labyrinthine sedative
[UP 2001] b. Myringoplasty
-
a. Facial nerve paralysis b. Lateral sinus thrombosis c. Immediate mastoid exploration
c. Sub periosteal abscess d. Brain abscess d. Labyrinthectomy
51. Mastoid reservoir phenomenon is positive in: 62. Most potential route for transmission of Meningitis from
[PGIJune 99] CNS to Inner ear is: [AI-09]
a. CSOM b. Petrositis a. Cochlear Aqueduct
c. Coalescent otitis media d. Coalescent mastoiditis b. Endolymphatic sac
52. Acute mastoiditis is characterized by all except: [AP97] c. Vestibular Aqueduct
a. Clouding of air cells d. Hyrtle fissure
b. Obliteration of retroauricular sulcus 63. Commonest cause of brain abscess: [PGIJune 00]
c. Deafness a. CSOM b. Pyogenic meningitis
d. Outward and downward deviation o f t h e pinna c. Trauma d. Chr. sinusitis
282 {_ SECTION V Ear

64. True about otogenic brain abscess is are: 73. Procedure for serous otitis media is: [AP2002]
a. H. influenzae is most common causative organism a. Trmpanoplasty b. Mastoidectomy
b. C.S.O.M. with lat. sinus thrombosis inturn can cause brain c. Myringotomy d. Medical treatment
abscess 74. Grommet tube is used in: [TN2002]
c. M o s t c o m m o n complication of CSOM a. Secretory otitis media b. Mucoid otitis media
d. Temporal lobe abscess is associated w i t h personality c. Serous otitis media d. All o f t h e above
changes 75. For ASOM, myringotomy is done in which quadrant:
65. Patient is having scanty, foul smelling discharge from [AI95]
middle ear, develops fever, headache and neck rigidity. a. Antero-inferio b. Antero-superior
CT ofthe temporal lobe shows a localized ring enhancing c. Postero-superior d. Postero-inferior
lesion, which o f t h e following is least likely cause of this 76. Ideal site for myringotmy and grommet insertion:
condition: [AI2011] [CUPGEE02]
a. S. aureus b. Pseudomonas a. Anterior superior aquadrant
b. Anterior inferior quadrant
c. S.Pneumoniae d. H. influenza
c. Posterior superior
66. Lateral sinus thrombosis is associated with all except:
d. Posterior inferior
[AP2008]
77. Myringoplasty is plastic repair of: [PGI]
a. Greisingersign b. Gradenigo sign
a. Middle ear b. Internalear
c. Lily-Crowe sign d. Tobey Ayer test
c. Eustachian tube Tympanic membrane
67. Griesinger's sign is seen in: [TN 03]
78. Myringoplasty is done using : [PGI 97]
a. Lateral sinus thrombosis b. Meningitis
a. Temporalis fascia b. Dura mater
c. Brain abscess d. Cerebellar abscess
c. Perichondrium d. Mucous membrance
68. Light house sign is seen in: Columella effect is seen in:
79. [TN2005]
a. ASOM b. CSOM a. Tympanoplasty b. Septoplasty
c. Menieres disease d. Cholesteatoma c. Tracheostomy d. None o f t h e above
69. A child w a s treated for H. Influenza meningitis for 6 Surgery on ear drum is done using: [Kerala 91]
80.
month. Most important investigation to be done before a. Operative microscope b. Laser
discharging the patient is: [Al 99] c. Direct vision d. Blindly
a. MRI 81. Which focal length in the objective piece of microscope
b. Brainstem evoked auditory response is commonly used for ear surgery: [AIIMS May 05]
c. Growth screening test a. 100 m m b. 250
d. Psychotherapy c 450 d. 950
70. A patient of C S O M has c h o l e a s t a t o m a a n d presents 8 2 . Schwartz operation is also called as: [PGI 97]
vertigo with. Treatment of choice would be: [A198] a. Cortical mastoidectomy
a. Antibiotics and labyrinthine sedative b.
b. Myringoplasty c. Radial mastoidectomy
c. Immediate mastoid exploration d. Fenestration operation
d. Labyrinthectomy 83. Simple mastoidectomy is done in: [MP 2004]
a. Acute mastoiditis
b. Cholesteatoma
SURGICAL MANAGEMENT OF c. Coalescent mastoiditis
MIDDLE EAR SUPPURATION d. Localized chronic otitis media
84. Cortical mastoidectomy in indicated in: [AIIMS 93]
71. A-7 year child presenting with acute otitis media, does a. Cholesteatoma without complication
not respond to ampicillin. Examination reveals full and b. Coalescent mastoiditis
bulging tympanic membrane, the treatment of choice c. CSOM with brain abscess
is: [AI98] d. perforation in Pars flaccida
a. Sytemic steroid b. Ciprofloxacin 85. Radical mastoidectomy is done for: [DNB 2000]
c. Myringotomy d. Cortical mastiodectomy a. ASOM
72. A 3 year old child presents with fever and ear ache. O n •b. CSOM
examination there is congested tympanic membrane c. Atticoantral cholesteotoma
with slight bulge. The treatment of choice is: [Al 95] d. Acute mastoiditis
a. Myrigotomy with penicillin 86. All o f o f t h e f o l l o w i n g s t e p s a r e d o n e in r a d i c a l
b. Myringotomy with grommet mastoidectomy except: [Al 97]
c. Only antibiotics a. Lowering of facial ridge
d. Wait and watch b. Removal of middle ear mucosa and muscles
CHAPTER 22 Diseases of Middle Ear J 283

c. Removal of all ossicles of eustachiean tube plate 90 A -30-yead old male is having Attic cholesteatoma of
d. Maintainance of patency of eustachian tube left ear with lateral sinus thromboplebitis. Which of the
87. Radical mastoidectomy includes all except [AIIMS 00] following will be the operation of choice? [Al 06]
a. Closure of the auditory tube a. Intact canal will be the operation of choice
b. Simple mastoidectomy with Tympanoplasty
b. Ossicles removed
• c. Canal wall down mastoidectomy
c. Cochlea removed
d. Mastodidectomy with cavity obliteration
d. Exteriorisation of mastoid
91 Treatment of choice of cholesteatoma with sensorineural
88. Nerve d a m a g e d in radical mastiodectomy is: [MH2000] deafness is: [AIIMS Dec. 94]
a. Facial a.
b. Chochlear b. Radical mastoidectomy
c. Vestibular c. Myringoplasty
d. All d. Tympanoplasty
89. Modified redical mastoidectomy is indicated in all except: 92. All of t h e following t e c h n i q u e s a r e u s e d to control
[MP 2000] bleeding from bone during mastoid surgery except:
[AIIMS Nov. 04]
a. SafeSCOM
a. Cutting drill over the bleeding area
b. Unsafe CSOM with atticoantral disease
b. Diamond drill over the bleeding are •
c. Coalescent mastoiditis
c. Bipolar cautery over the bleeding area
d. Limited mastoid pathology
d. Bone wax

EXPLANATIONS AND REFERENCES

ACUTE SUPPURATIVE OTITIS MEDIA


1. Ans. is b i.e. Strepto-pneumoniae Ref. Harrison 17th/edp 208; Current Otolaryngology 2nd/ed pg 656
2. Ans. is a i.e. Pneumococcous
Most common cause of acute otitis media:
• Streptococcus pneumonia / pneumococcus (35-40% cases)
• H. influenza (25-30%)
• M. catarrhalis (10-20%) -

3 Ans. is a i.e. Most frequently it resolves without sequelae


Ref. Turner Wth/edp 424,428; Dhingra 5th/ed pg 69,70,6th/ed p 62,63; Current Otolaryngology 2nd/ed pg 656-658
ASOM is t h e most c o m m o n infectious disease seen in children. Peak incidence is 2 yrs of age.
• Most common predisposing factor for ASOM is; recurrent attacks o f c o m m o n cold, upper respiratory tract infections and
exanthematous fevers like measles, diphtheria, w h o o p i n g cough.
Others include: tonsilitis, adenoids, rhinitis, sinusitis, allergy, cleft palate, d o w n syndrome, Tumors of nasopharynx
Painful parotitis (mumps) most commonly leads t o orchitis, oophritis, aseptic meningtits, pancreatitis and not ASOM
... Harrison 17th/edp 1220
Most common organism for ASOM is: S. pneumonia (35-40%)
- H. influenza (25- 30%) and M. catarhalis (15%) are less c o m m o n .
Treatment o f ASOM is Essentially Medical (not surgical) and involves:
- Antibiotics - Penicillin group of antibiotics
- Analgesic and antipyretics
Ear toileting
. Some cases may require: Myringotomy
- Mastoidectomy is not done in case of ASOM
- As far as complications are concerned.
Turner 10/e, p 424,428 says "Prognosis of ASOM is good, most cases recover completely. Whether in infants or children."
Current otolaryngology 2/e pg-658 says "The vast majority of uncomplicated episodes of AOM resolves w i t h o u t any adverse outcome"
Ans. is a i.e. ASOM Ref. Dhingra 5th/ed pg 70; 6th/ed p62-63
Otoscopy Signs for ASOM:
• There is congestion of pars tensa
-
284 |_ SECTION V Ear

• Leash of b l o o d vessels appear along the handle of malleus and at the periphery giving it a cartwheel like appearance.
• Transluscency is reduced.
• Later tympanic membrane appears red and bulging w i t h loss of landmarks.
• Tympanic membrane is immobile n pneumatic ostoscopy
5. Ans. is c i.e. Streptomycin Ref. Turner 7 Oth/ed p 281, Dhingra 6th/edp 63
Medical management is the Treatment of choice in a case of ASOM
Antibiotics of choice are: Ampicillin or amoxicillin (DOC)
Other w h i c h can be used Cotrimoxazole
• Cefaclor
• Erthromycin
• Penicillin
Ans. is c i.e. Supportive Ref. Turner I Oth/ed p 349
Barotraumatic otitis media
"Treatment consists of teaching the patient valsalva manoeuvre. If this fails, politzerization or
Eustachian tube catheterization is carried out.
If fluid is present a myringotomy maybe necessary and occasionally in resistant cases, grommet insertion may be required until the middle
ear mucosa has returned to normal." ... Turner 1Oth/ed p 349
7. Ans. is c i.e. ASOM Ref. Tuli Ist/edp 53
ASOM - In stage of suppuration-pulsatile otorrhea is present.
Light house sign: Seen in ASOM w h e n pulsatile otorrhea reflects light intermittently on otoscopy.

NON SUPPURATIVE OTITIS MEDIA


Ans. is b i.e. Secretory otitis media Ref. Dhingra 5th/edpg 72,6th/ed p64
• Inadequate antibiotic treatment of acute suppurative otitis media may inactivate infection b u t fail t o resolve it completely.
• Low grade infection lingers on which acts as a stimulus for the mucosa t o secrete more mucus which leads t o development of
serous/secretory otitis media.

ALSO KNOW

• Secretory otitis media is characterized by accumulation of non- purulent effusion in the middle ear cleft.
• The fluid collected in serous otitis media is sterile Q

• It is most c o m m o n l y seen in children between 2 t o 5 years of age Q

• It is the most c o m m o n cause of hearing loss in children in developed world 0

Ans. is b i.e. Nasopharyngeal carcinoma Ref. Dhingra 5th/edpg 72,6th/ed p 251; Current Laryngology 2nd/edp 659
Unilateral serous otitis media in an adult should always raise the suspicion of a benign / malignant t u m o r of nasopharynx
"In adults presenting w i t h a unilateral middle ear effusion the possibility of a nasopharyngeal carcinoma should be considered".
- Current Otolaryngology 2nd/edpg 659
10. Ans. is d i.e. malignancy Ref. Scotts Brown 7/e vol 3 p 3389
"A high incidence of NPC (Nasopharyngeal Carcinoma) in Souct East Asia and Southern China correlates with the high incidence of OME
(Otitis Media with Effusion) in adults in these regions." -Scotts Brown 7th/ed vol 3 p. 3389
"Presence of unilateral serous otitis media in an adult should raise suspicion of nasopharyngeal growth". Ref. Dhingra 6/e p257
11 Ans. is b i.e. Is painless Ref. Dhingra 5th/ed pg 72,6th/edp 64; Current Otolaryngology 2nd/edpg 658

G l u e Ear/serous Otitis M e d i a

• Glue ear/serous otitis media is a painless condition patients are generally asymptomatic and the condition is detected on routine
audiologic screening. (\ it is also k/a silent otitis media)
• M/C s y m p t o m of Glue ear is hearing loss. Older children may complain of reduced hearing, but in many cases the hearing loss
is noticed by parents, teachers or caretakers
• There may be delayed speech development or child may have behavioral problems
• There may be a blocked feeling o f t h e ear which may cause infants and young children to pull at their ears
As far as other options are concerned:
• NaF is n o t used for its treatment -
For details on management see later questions
• Radical mastoidectomy is not done in glue ear. .
12. Ans. is a i.e Impedance audiometry Ref. Current Otolaryngology 2nd/edpg 659,3rd/edp676
• Pure tone audiometry: gives information about the quantity and quality of hearing loss.

ff
CHAPTER 22 Diseases of Middle Ear

• In secretory otitis media: conductive deafness of 20-40 dB is seen (which is not a specific finding as conductive deafness can
be seen in many other conditions). Therefore, pure tone audiometry is not diagnostic of serous otitis media but provides an
assessment o f t h e hearing loss and is therefore important in monitoring the progress o f t h e condition and provides information
useful for management decisions
• On otoscopy:Tympanic membrane appears dull, opaque w i t h loss of light reflex (which again is not diagnostic).
• X-ray mastoid: Shows clouding of air cells.
• Impedance audiometry is an accurate way of diagnosing serous otitis media. It shows type B tympanogram which is diagnostic
of fluid in ear .0

13. Ans. is b i.e. Glue ear Ref. Dhingra 5th/edp 72,6th/edp 64


In glue ear (serous otitis media) Tympanic membrane is dull opaque w i t h loss of light reflex and appears yellow / grey / blue in
colour.
• Normal colour of tympanic membrane is pearly grey.
• Congested membrane w i t h prominent blood vessels (cartwheel sign) is seen in early stages of acute otitis media.
• Bluish discoloration is seen in haemotympanum.
• Flamingo pink color is seen in otosclerosis.
14. Ans. is c i.e. Myringotomy with ventilation tube insertion
Ref. Logan Turner 10th/ed p 437; ENT by Tuli ist/ed p 75-76; Current Otolaryngology 2nd/ed pg 660; Dhingra 6th/edp 64,5th/ed p 73
Treatment of choice for glue ear is insertion of grommet (i.e., ventilation tube insertion).
Tympanotomy / cortical mastoidectomy has a very limited role, and is not done nowadays for serious otitis media.
Myringotomy and aspiration of middle ear effusion without ventilation tube insertion has a short lived benefit and is not recommended
- (Current otolaryngology 2nd/ed pg 660)
"From three trials, myringotomy with aspiration has not been shown to be effective in restoring the hearing levels in children with
OME" - Scott Brown 7th/ed pg 896
15. Ans. is d i.e. Adenoidectomy with grommet insertion
16. Ans. is b, c and d i.e. Adenoidectomy, Grommet insertion and myringotomy Ref. Scott Brown 7th/ed vol I p 896-904
Child is presenting w i t h m o u t h breathing. Palate is high arched.There is nasal obstruction and recurrent respiratory tract infections
along w i t h hearing impairment. All these features are suggestive of adenoid hyperplasia. In case of adenoid hyperplasia impairment
of hearing is due t o secretory otitis.
Thus the logical step in the management w o u l d be myringotomy w i t h g r o m m e t insertion (to treat SOM) and adenoidectomy (to
remove the causative factor). Now since in Q.16 all 3 are given in option, we are going for all three but in Q. 15 choice is between
myringotomy w i t h g r o m m e t insertion and adenoidectomy and g r o m m e t insertion better o p t i o n is adenoidectomy and g r o m m e t
insertion (as it is obvious gromet cannot be inserted in tympanic membrane w i t h o u t myringotomy).
17. Ans. is b, c, and d i.e. Type B tympanogram; The effusion of middle ear is sterile; and Most common cause of deafness in a
child in day care patients
Ref. Dhingra 5th/ed pg 71-73,6th/edp 64-65; Current Otolaryngology 2nd/ed pg 658-659; Ghai 6th/ed p 332

Otitis Media with Effusion/Secretory Otitis Media


• Mostcommon cause of deafness in children (specially school going children) ... Current otolaryngology
0
2/e pg-658.
• Characterised by accumulation of non purulent, sterile fluid in middle ear . 0

Clinical feature: • Conductive deafness - Presenting s y m p t o m 0

• Delayed and defective speech


Tympanogram:Type B c u r v e / f l a t or dome shaped curve .
0 0

Management: Grommet insertion is t h e T O C but should not be performed immediately.


0

According to G h a i
• "Since 50% of serous middle ear effusions resolve spontaneously within 3 months, newly diagnosed effusions should be observed for
this period in nearly all cases.
• Use of antihistaminics and decongestants has been abandoned based on adequate scientific data demonstrating lack of efficacy.
The benefit o f brief steroid administration is unproven.
• If effusion persists beyond 3 months, tympanostomy tube insertion may be considered for significant hearing loss (>25
dB). Other indications of tube placement are ear discomfort or pain, altered behavior, speech delay, recurrent acute
otitis media or impending cholesteatoma formation from tympanic membrane retraction".... Ghai 6/e, p 332

A c c o r d i n g to Current otolaryngology 2nd/edpg 659

"A large number of patients with OME (otitis media with effusion) require no treatment, particularly if the hearing impairment is mild.
Spontaneous resolution occurs in a significant proportion of patients. A period of watchful waiting for 3 months from the onset (if known)
or from the diagnosis if onset unknown), before considering intervention is advisable".
2861_ SECTION V Ear

ALSO KNOW

Indications for early insertion of Tympanostomy tube/grommet tube:


• Cases where spontaneous resolution is unlikely as predicted by season of presenting t o OPD (i.e. between July to December)
and a B/L hearing impairment of >30 dB
• It is causing significant delay in speech and language development
• OME is present in an only hearing ear.
• Recurrent otitis media.

18. Ans. is c i.e. tympanostomy tube's are usually required for treatment
Ref, Scott's Brown 7th/ed vol-l pg 879-893-896; Current Otolaryngology 2nd/edpg 658-662; Dhingra 5th/ed pg71-72,6th/edp 64-65
Lets see at t h e options one by one:
Serous otitis media'

Option a

Sensori neural deafness occurs as a complication in 8 0 % of cases


This is not correct because serous otitis media leads t o conductive type o f hearing loss.

Option b •

Intracranial spread o f t h e infection complicates the clinical course


This is not true as complications of serous otitis media are:
• Adhesive otitis media
• Atrophy of tympanic membrane
• [Tympanosclerosis (chalky white deposits seen on membrane)
Atelectasis of middle ear
Ossicular necrosis
Cholesteatoma due t o retraction pockets
Cholesterol granuloma due t o stasis of secretions

Option c

Tympanostomy tubes are usually required for treatment:


• This is quite correct as myringotomy and aspiration of middle ear effusion w i t h o u t ventilation tube/Tympanostomy t u b e /
g r o m m e t insertion has a short lived benefit and is not recommended. - Current otolaryngology 2nd/edpg 660
• Hence if otitis media w i t h effusion / serous OM is not resolved spontaneously, tympanostomy tube is inserted.

Option d

• Gram positive organism are grown routinely in culture in the aspirate


• Absolutely incorrect because fluid collection in serous otitis media is sterile —Dhingra - 5th/edpg 71
19. Ans. is d i.e. Rinnie test positive Ref. Dhingra 5th/ed p 72,26,6th/ed p 64,22
Secretory otitis media is associated w i t h conductive type of hearing loss. So, Rinnie's test will be negative.
20. Ans. is b i.e. Secretory otitis media Ref. Current Otolaryngology 2nd/edpg 662
Treatment summary for otitis media - Current Otolaryngology 2nd/ed pg 662

Acute otitis media Otitis media with effusion CSOM" j


Watchful waiting Upto 72 hours with analgesia/ For 3 months from onset or Nl
antipyretics if non severe and diagnosis
patient > 2 yrs old
Medical Therapy Antibiotic (amoxicillin) Nl Aural toileting and topical
antibiotics (Quinolones)
Surgical intervention Myringotomy for refractory AOM VT insertion if unresolved after 3 • Tympanoplasty
Cortical mastoid oitectomy in non months Aderoidectomy on second • Tympano mastoid surgery if
responding cases VT insertion refractory to medical therapy
Wtm .••.-.,<.•.>.••...,-.,. i . .

Nl = not indicated; VT: = ventilation t u b e

-
CHAPTER 22 Diseases of Middle Ear

21. Ans. is d i.e. Multiple perforation


22. Ans. is c, or d i.e. pain, or Thin odourless fluid Ref. Dhingra 5th/ed pg 83,6th/edp 74; Scott's Brown 7th/ed vol-3 pg 3447-3448

Tubercular Otitis Media

• Important points:
- Seen mainly in children and young a d u l t Q

- It is secondary t o pulmonary tuberculosis. 0

- Route o f spread - Mainly t h r o u g h eustachian t u b e (not blood b o r n e ) . 0

• Symptoms:
1. Patients often present w i t h chronic painless otorrhoea (usually foul smelling) which is resistant to antibiotic t r e a t m e n t 0

3. Severe conductive type hearing loss , (sometimes due t o involvement of labyrinth may be SNHL)
0

4. Facial nerve palsy may be the presenting symptom in c h i l d r e n 0

5. Cough; fever and night sweats may be present in patients w i t h tuberculous infection in other organ system.
• O/E
- Multiple perforations in tympanic membrane (This feature was once considered characteristic ofTB but now is seldom seen).
0

- Middle ear and mastoid are filled with pale granulation tissue (It is a characteristic of tuberculous otitis media)
0

• Complications: (Early onset of these symptoms is seen)


- Mastoiditis
- Osteomyelitis
- Postauricular fistula
- Facial nerve palsy
.

Coming to the Qs Now

Ans. 21 is obvious - i.e. multiple perforations.

As far as Ans 22 is concerned - there is confusion between o p t i o n c i.e. pain and option d i.e. thin odourless fluid

Pain
According to Scott's Brown 7th/ed vol-3 pg 3447"Otalgia may or may not be present but is usually m i l d "
According t o Dhingra 5th/ed p. 83,6th/ed p 74; Earache is characteriscally absent, b u t discharge is often foul smelling due t o u n -
derlying bone destruction. Then discharge is not present, it is thick.
Hence answer 22 can be aither c-i.e pain or d i.e then odourless fluid.

CHRONIC SUPPURATIVE OTITIS MEDIA


23. Ans. is a i.e. Attito-Antral Ref. Dhingra 5tth/edp 77,6th/ed p 69; Pediatric Otolaryngology 2nd/ed p 478
Tubotympanic type (safe or benign type)
• It is associated with cenfra/perforaf/on of tympanic membrane and involves the anterior inferior part of middle ear cleft (eustachian
t u b e and mesotympanum).
• There is no association with cholesteatoma.
• Tubotympanic type is also called safe or benign type as there is no risk of serious complicatiions.
Attico-antral type (unsafe or dangerous type)
• it is associated w i t h an attic or a marginal perforation of the tympanic membrane and involves posterosuperior part of middle
ear cleft (attic, antrum and mastoid).
• The attico-antral disease is characterized with cholesteatoma which erodes the bone.
• Risk of intracranial extension o f infection, and thus the risk of complication, is very high, therefore it is called dangerous or
unsafe ear.
24. Ans. c i.e. Attic region Ref. Dhingra 5th/ed p 77,6th/ed p 72
Atticoantral type or marginal or unsafe otitis media involves posterosuperior part o f middle ear cleft (attic, antrum, posterior
t y m p a n u m and mastoid) and is associated w i t h cholesteatoma.
25. Ans. is d i.e. Acute necrotising otitis media
26. Ans. is b i.e. C S O M Ref. Dhingra Sth/ed pg 81,6th/ed p 67-68
Cholesteatoma is presence of keratinising squamous epithelium in middle ear.
SECTION V Ear

Origin

T 1
Primary acquired Secondary acquired •

No H/O of previous otitis media or Occurs in pre existing perforation in pars tensa
pre existing perforation • Acute necrotizing otitis mediaQ
Most common cause is formation of • Attico antral/ Unsafe CSO
retraction pocket of pass flaccida in which I
keratin debris accumulates • These perforations result in squamous epithelial
migration from tympanic membrane
• It can also result from implantation of squamous

epithelium into the middle ear during surgery.

27. Ans. is d i.e. Retraction pocket Ref. Current Otolaryngology 2nd/edpg 666
Most common accepted theory for formation of cholesteatoma Is formation of a retraction pocket. According to this theory, chronic
negative middle ear pressure (which occurs due t o poor Eustachian t u b e function and chronic inflammation o f t h e middle ear) leads
to retractions ofthe structurally weakest area ofthe tympanic membrane, the pars flaccida. Once the retractions form, the normal migra-
tory pattern ofthe squamous epithelium is disrupted, resulting in the accumulation of keratin debris in the cholesteatoma sac.
28. Ans. is b i.e. marginal Ref: Dhingra 5/epg-78 fig-1 1.5
29. Ans. is b i.e. Associated with cholesteatoma Ref. Dhingra 6/e p59,5th/edpg-78
Tympanic membrane can be divided in 2 parts:
Parstensa : It forms most of the tympanic membrane. Its periphery is thickened t o form fibro cartilaginous ring called as
annulustympanicus.
Pars Falccida : It is situated above the lateral process of malleus between the notch of Rivinus and the anterior and posterior
malleolar fold.

P e r f o r a t i o n in t y m p a n i c m e m b r a n e c a n b e in

Parstensa Pars flacida

Central Marginal (Perforation destroys even the Attic perforation


annulus) theraby reaching sulcus tympanicus
• Occurs in tubotympanic type of CSOM
Most common isposterio superior marginal 0

- occur in attico antral type of CSOM


Not associated with cholesteatoma - associated with cholesteatoma
Considered safe - considered unsafe/dangerous

1 NOTE
Most common cause of perforation is chronic otitis media. ... Dhingra Ath/ed p 55

^f^nemonic
FAMOUS
F Perforation o f Pars Flaccida.
A Seen in Atticoantral/marginal perforation
M Associated w i t h CSOM (of atticoantral type) or acute
necrotizing otitis media
O Associated w i t h Cholesteatoma
U Unsafe type
S Surgery is TOC.
CHAPTER 22 Diseases of Middle Ear T 289

30. Ans. is d i.e. Conservative management Ref. Turner Wth/edp 285; Scott's Brown 7th/edd vol 3 pg 3421 and 3424
There are 2 schools of t h o u g h t as far as this question is concerned - Some believe that.
• TOC o f central perforation is myringoplasty.
• TOC of t u b o tympanic CSOM is conservative management.
But according t o Turner 10/e,p 285- central perforation/tubo tympanic CSOM are b o t h managed conservatively by antibiotics and
by keeping the ear dry.
"If there is recurring discharge or if there is deafness sufficient to cause disability, closure of the perforation by myringoplasty
should be considered."
According to Scott's Brown (7 /ed vol-3 pg-3421)>h

• Dry perforations that are symptom free do n o t require usually require closure.
• If the only s y m p t o m is a hearing impairment, the chances of improving hearing w i t h surgery should be considered carefully,
n o t just the hearing in the operated ear b u t the overall hearing ability o f t h e patient.
• In patients w i t h a H/O i n t e r m i t t e n t activity, surgery t o close t h e p e r f o r a t i o n is probably indicated t o m i n i m i z e f u t u r e
activity.
• So f r o m all above discussions it is clear that TOC for central safe perforation is conservative management.
31. Ans. is a i.e. Etiology is multiple bacteria Ref. Dhingra 5th/edp 77,78,80,6th/ed p 70
• CSOM is caused by multiple bacteria - both aerobic and anaerobic. - Dhingra 5/e p. 78 0

• Their is no sex predilection in CSOM - both sexes are affected equally. 0


Dhingra 5th/edp 77,6th/edp70
• Treatment of Tubotympanic type of CSOM is aural toileting and antibiotic ear drops. Dhingra 5th/ed p 80,6th/edp71
As far as oral antibiotics are concerned.
" T h e y are useful in acute exacerbotion of chronically infected ear, otherwise role of systemic antibiotics in the treatment
of CSOM is limited." ... Dhingra 5th/ed pg 80,6th/ed p 72
• Otitic hydrocephalus - is a rare complication of CSOM. ... Turner 10th/edp 309
32. Ans. is a, b and c i.e. Whitish mass behind intact TM, Normal pars tensa and pars flaccida, Recurrent attacks of otorrhea
Ref. Internet

L e v e n s o n C r i t e r i a for C o n g e n i t a l C h o l e s t e a t o m a

• White mass medial t o normal TM


• Normal pars flaccida and tensa
• No history o f otorrhea or perforations
• No prior otologic procedures
• Prior bouts o f otitis media (no ground for exclusion)
33. Ans. is b i.e. Cholesteatoma Ref. Dhingra 5th/edpg 81,6th/edp 72
Cholesteatoma / attico antral type of CSOM / marginal perforation is characterised by scanty foul smelling, painless discharge from
the ear. The foul smell is due t o saprophytic infection and osteitis
34. Ans. is d i.e. Erodes the bone Ref. Dhingra 5th/edp 75-76,6th/edp 72
• Normally middle ear is not lined by keratinising squamous epithelium. If keratinising squamous epithelium is present anywhere
in the middle ear or mastoid, it is called as cholesteatoma.
• The term cholesteatoma literally means - "Skin in the wrong place." It is a misnomer because neither it contains cholesterol
crystals nor it is a t u m o r . 0

Cholesteatoma has 2 parts:


• Matrix: made of keratinising squamous e p i t h e l i u m . -. 0

• Central white mass - made of keratin debris. J H e n c e a l s o


^ epidermosis or keratoma
- Cholesteatoma has the property t o destroy bones (due to the various enzymes released by it and not by pressure necrosis).
- Dhingra 5th/ed pg 76, 77,6th/ed p 68
35. Ans. is a i.e. Lateral semicircular canal Ref. Logan Turner 10th/edp 287
• Cholesteatoma has the property t o destroy the bone by virtue of the various enzymes released by it.
• Structures immediately at the risk of erosion are:
- Long process of incus. 0

- Fallopian canal containing facial nerve. 0

- Horizontal / lateral semicircular canal. 0

' v ^ k r > m - : : - : \ - - U::.ib.y .;'.=> OJ.H% •-; sr.: • . ••. • ...Tin

-
290 {_ SECTION V Ear

36. Ans. is a, d and e i.e. Scanty, malodoruos discharge; Ossicular involvement; and Eustachian tube dysfunction
Ref. Dhingra 5th/ed p 77,81,6th/edp 68,72; Current olotaryngology 2nd/ed pg 666,3rd/ed p 683-684; Mohan Bansalp211
• Cholestatoma is associated w i t h atticoantral type of CSOM / atticoantral or marginal perforation (and not central perforation).
• Cholesteatoma leads t o destruction of bones therefore there is scanty foul smelling discharge and ossicular necrosis.
• Hearing loss occurs if ossicles are involved.
• It is of conductive type but if complications like labyrinthitis intervene, SNHL may also be seen.
• Bleeding may occur from granulations or polyp.
Otalgia is not seen in case of cholesteatoma.
Etiopathogenesis of primary and secondary acquired cholesteatoma

Eustachian tube obstruction
T
Chronic ME -ve pressure

1
Mild ME effusion Retraction pocket
or subclinical infection (attic or posterior superior region)

Metaplasia of Proliferation of •

ME mucosa basal layer •

T
Primary acquired
cholesteatoma
(without pre-existing
perforation of TM)
1_ Metaplasia of
Secondary acquired ME mucosa
cholesteatoma Large canal perforation with
(with pre-existing) recurrent infections
perforation Epithelial migration
through perforation

Cholesteatoma can also cause facial nerve palsy and labyrinthitis.

37. Ans. is a i.e. Mastoidectomy Ref. Dhingra 5th/edpg 77,6th/ed p 73


38. Ans. is b i.e. Modified Radical Mastoidectomy [MRM] Ref. Dhingra Sth/ed p 82,6th/ed p 73; Logan Turner 10th/ed p29l

C S O M is o f T w o T y p e s

Tubotympanic or safe type Atticoantral or unsafe type or dangerous type


Discharge Profuse, mucoid, odourless Scanty, purulent, foul- Smelling 0

Perforation Central 0
Attic or posterosuperior marginal 0

Granulations Uncommon Common


Polyp Pale * Red and Fleshy
Cholesteatoma Absent Present 0

Complications Rare 0
Common 0

Audiogram Mild to moderate conductive deafness 0


Conductive or mixed deafness 0

TOC for atticoantral variety of CSOM is surgery i.e..


TOC for tubotympanic type of CSOM is mainly conservative in the form of aural toileting and systemic antibiotics and once the ear is
dry myringoplasty can be done.

• Perforation of pars flaccida leads to attic perforation which is considered dangerous and should be managed with modified radical mastoidectomy
CHAPTER 22 Diseases of Middle Ear

39. Ans. is c i.e. Tympanosclerosis Ref. Dhingra 4th/ed p 60,5th/ed p 76


In ventilation o f t h e middle ear cleft, air passes from eustachian t u b e t o mesotympanum, f r o m there to attic, aditus, antrum and
mastoid air cell system.
Any obstruction in the pathways o f ventilation can cause retraction pockets or atelectasis of tympanic membrane, e.g.:
i. Obstruction of eustachian t u b e Total atelectasis of tympanic membrane
8

ii. Obstruction in middle e a r Retraction pocket in posterior part of middle ear while anterior part is ventilated
8

iii. Obstruction of isthmi ® Attic retraction pocket


Depending on the location of pathologic process, other changes such as t h i n atrophic tympanic membrane, (due t o absorption of
middle fibrous layer), cholesteatoma, tympanosclerosis, and ossicular necrosis
A posterior superior retraction pocket - if allowed t o progress leads t o primary acquired cholesteatoma and not secondary choles-
teatoma.

I Primary acquired cholesteatoma I


i • 1 1

I Conductive deafness I I Erodes bony labyrinth I


T
labyrinthitis

ISensorineurafhearing loss!
i j

So, tympanosclerosis and sensorineural hearing loss are both correct but tympanosclerosis is a better option than SNHL (which
occurs very late when retraction pocket gives rise t o cholesteatoma which later causes labyrinthitis)

ALSO KNOW

Tympanosclerosis

• It is hyalinisation and later calcification in the fibrous layer o f tympanic membrane.


• Tympanic membrane appears as chalky white plaque.
• Mostly, it remains asymptomatic.
• It is frequently seen in cases of serous otitis media, as a complication of ventilation tube and in CSOM
• Tympanosclerosis mostly affects tympanic memebrane b u t may be seen involving ligaments, joints of ossicles, muscle tendons
and submucosal layer of middle ear cleft and interferes in the conduction of sound.
40. Ans. is b i.e. Posterior facial ridge Ref. Scott's Brown 7th/ed vol 3pg3112-3113
The sinus t y m p a n i (Posterior facial ridge) is the posterior extension o f t h e mesotympanum and lies deep t o both the p r o m o n t o r y
and facial nerve.
The medial wall of sinus tympani becomes continuous w i t h the posterior portion o f t h e medial wall o f t h e tympanic cavity. This is
t h e worst region for access because it is above pyramid, posterior t o intact stapes and medial t o facial nerve.
A retro facial approach via mastoid is not possible because the posterior semi circular canal blocks the access.
41. Ans. is d i.e. Tympano mastoid exploration Ref. Dhingra Sth/ed pg 82,6th/ed p 73
Child presenting with foul smelling discharge with perforation in the pars flaccida of the tympanic membrane suggests unsafe CSOM.
Conservative management is not of much help in these cases; surgery is the mainstay of treatment.
Tympano mastoid exploration is the ideal option in such cases.
• Tympano-mastiod exploration can be done t h r o u g h various procedures:
- Canal wall down procedures: Atticotomy, and rarely radical mastoidectomy.
- Canal wall up procedures: Cortical mastoidectomy
Preferred treatment w o u l d be

A i c n i/h.\r\\nr

Recounstruction of hearing mechanism by tympanoplasty is only the second priority.


42. Ans. is d i.e. Myringoplasty Ref. Dhingra Sth/edp 82,6th/edp 72-73; Logan turner 10th/edp 289
Myringoplasty consists of closing a 'central perforation' in the tympanic membrane in the 'tubotympanic type' or 'safe type' of chronic
suppurative otitis media. It is not indicated in unsafe or dangerous type of otitis media with posterosuperior otitic perforation.
• The patient in question is a case of dangerous or unsafe type of CSOM as signified by the presence of posterosuperior retraction pocket
cholesteatoma.
292 [_ SECTION V Ear

• The mainstay in treatment of this type of CSOM is surgery.


- Primary aim is t o remove the disease and render the ear safe.
Secondary aim is t o preserve or reconstruct hearing, but never at the cost o f t h e primary aim.
• (Mastoid exploration) is the operation of choice.
• Tympanoplasty: forms part of secondary aim t o reconstruct hearing after a primary mastoid exploration.
- Dangerous type CSOM is associated w i t h a a perforation in attic or posterosuperior region ofT.M. along w i t h variable extent
of destruction of ossicles and other middle ear contents. Reconstruction of hearing in this type of CSOM thus requires vari-
able extent of ossicular reconstruction besides closure of perforation.

Audiometry forms an i m p o r t a n t step in evaluation of disease process preoperatively.


Although myringoplasty also forms a type o f tympanoplasty its use is limited t o closure o f perforation in t h e parts tensa o f
tympanic membrane which is seen in safe type CSOM.
43. Ans. is b i.e. Long process of incus Ref. Scott's Brown 7th/ed vol 3 pg 342 7

COMPLICATION OF OTITIS MEDIA


44. Ans. is b i.e. Intracerebral abscess
45. Ans. is c i.e. Brain abscess Ref. Turner 10th/ed pg 311-312; Scotts Brown 7th/ed vol 3 pg 3435
There is no confusion as far as this question is concerned:
• According t o Turner 10th/ed pg 311-312
• "Most c o m m o n complication of CSOM is brain abscess (intracerebral)".
• According t o Scotts Brown 7th/ed vol3 pg 3435
• "In a large series of 268 patients with complications of COM, he incidence of extracranial complications was 32%, intracranial
complications was 56% and combined intracranial and extracranial was 12%"

Relative Incidence of Complications in Active Mucosal a n d S q u a m o u s C O M

Extracranial % Intracranial

Post auricular abscess 75 1


Subdural abscess 51
Facial palsy Brain abscess

Bezold abscess Extradural abscess 10,, * l t ? --.'j*V« A-:\0$.


f

Petrous apicitis Lateral sinus thrombosis r<H*-20 •


Meningitis 12

So from above text of Scotts Brown it is clear that Intracranial complications are more common than extracranial a n d
amongst intracranial as is clear from the table - M/C is Brain abscess
Hence there is no d o u b t regarding this answer.

• Most common complication of acute otitis media -» Acute mastoiditis


• Most common intracranial complication of acute otitis media -> Meningitis
• Most common extracranial complication of CSOM -> Post-auricular abscess
• Most common intracranial complication of CSOM -» Brain (intracerebral abscess)

46. Ans. is c i.e. Temporal lobe abscess Ref. Turner 10th/ed pg 311-312; Scott Brown 7th/ed pg 3435
• Brain abscess is the most c o m m o n complication of ear disease (CSOM) . Q

(as discussed in previous question)


• Also ear disease, is the most c o m m o n cause of brain abscess . 0

• Cerebral abscess is twice as c o m m o n as cerebellar abscess . 0

• Temporal lobe abscess is most c o m m o n type of cerebral abscess . 0

Hence our Answer is Temporal lobe abscess


4 7 . Ans. is b i.e. chronic mastoiditis Ref: Current Otolaryngology 2nd/ed pg 663,3th/ed p 679; Dhingra 5th/ed pg 85,6th/ed p 76/77
Firstly - Remember there is no such t e r m as chronic mastoiditis
CHAPTER 22 Diseases of Middle Ear
293
M a s t o i d i t i s is o f 2 T y p e s i •

Acute Masked / Latent mastoiditis


It typically presents as a complication of Acute otitis media in a child. It results from inadequate treatment of AOM and is less severe.
Treatment options
• Antibiotics TOC - Cortical mastoidectomy along with 1/V antibiotics
• Myringotomy
• Cortical mastoidectomy (required under special
circumstances)

48. Ans. is b and d i.e. Facial nerve palsy; and Labyrinthitis Ref. Dhingra 5th/ed p 85,6th/ed p 75-76; Tuli Ist/edp 66
4 9 . Ans. is a, c and d i.e. Labyrinthitis; Bezold's abscess and Facial nerve palsy
Extra cranial complications of CSOM are:
Petrositis (gradenigo syndrome)
Labyrinthitis I
Osteomyelitis of temporal bone
Septicemia / pyaemia
Otogenic Tetanus.
F. Facial nerve palsy
Acute Mastoiditis: - Postaural sub periosteal abscess
- Zygomatic abscess
- Luc's abscess
- Citelli abscess
- Bezold abscess
Mnemonic: Pakistan L O S T First Match

Remember: M/C Extra cranial complication - mastoiditis (postaural abscess)


Overall M/C complication - Brain abscess (Temporal lobe abscess).

Friends here it is i m p o r t a n t t o n o t e t h a t ' h e a r i n g loss'will not be include in the complications of CSOM. As it is a sequalae and not
complication of CSOM

S e q u a l a e of C S O M

These are the direct result of middle ear infection and should be differentiated from complications:
Perforation of tympanic membrane
Tympanosclerosis

Cholesteatoma formation
Ossicular erosion
Atelectasis and adhesive otitis media
Conductive hearing loss (d/t ossicular erosion/fixation)
SNHL
Speech impairment
Learning disabilities
Hence - hearing loss, cholesteatoma and conductive deafness are not included in the complications of otitis media.
50. Ans. is c i.e. Subperiosteal abscess Ref: Scott Brown 7/ed vol-3 pg-3435
As discussed earlier the relative incidence of various extracranial complications in a case of chronic otitis media are:

Extracranial complication Percentage :


• Post auricular abscess 75
• Facial palsy l%^9tstb9mniE auiesrn .e.i) note iiovis«sn jvijisoM *
• Bezold abscess f & f t e & o w IfiG Jositsq >o noijibno-5 ni egrtsrb o l * *
• Petrous apicitis (Petrositis) 0.2
• Meningitis 12

So M/C complication is post auricular abscess -

Now what is post auricular abscess -

There are many abscesses in relation t o mastoid • •

-
294 T SECTION V Ear

Post aural subperiosteal abscess Zygomatic abscess Bezold abscess


• It is the commonest abscess that forms over • Posterior root of zygoma is invoved • Passes through the tip of mastoid into sterno
the mastoid • Swelling lies in front of and above the pinna cleido mastoid muscle in the upper part of
• Pinna is displaced forward, out ward and • Associated oedema of upper eye lid neck. 0

downward • Pus lies superficial or deep to temporalis


• In infants and children abscess forms over muscle.
McEwan's triangle
Citelli abscess Lues abscess (Meatal abscess)
Pus passes through inner table of mastoid In this case, pus breaks through the bony wall Lateral sinus thrombosis
process into the digastric triangle 0
between the antrum and external osseus
meatus. Swelling is seen in deep part of
meatus. 0

So As is clear f r o m above explanation - M/C. Extra cranial complication is - Post Aural sub periosteal abscess: If this o p t i o n is not
given then t h e next best option w o u l d be Mastoiditis.
51. Ans. is d i.e. Coalescent mastoiditis Ref. SKDe, p 107, 98
• Mastoid reservoir phenomenon is the latent infection in the mastoid resulting from inadequate treatment of ASOM. (Also called
as masked I latent mastoiditis).
• It is a slow process of destruction of mastoid air cells w i t h o u t acute features.

Clinical features: Patient is a child, not entirely feeling well with persistence of hearing loss, and mild pain in mastoid region in a
treated case of ASOM.
On examination: • Tympanic membrane appears dull and thick with loss of translucency
• X-ray mastoid - clouding of air cells with loss of cell outline.
Management: • Cortical mastoidectomy with full doses of antibiotics.

52. Ans. is none •

53. Ans. is b Clouding of air cells of mastoid


54. Ans. is c, d and e i.e. Mastoid tip, root of zygoma and Mastoid atrum
Ref. Dhingra 5th/ed pg 86,6th/ed p 76; Current Otolaryngology 2nd/ed pg 663,3rd/ed, pg 679

Mastoiditis

• It is a complication of actue otitis media in a child.


• Pain and tenderness over the mastoid process are the initial indicators of mastoiditis.
• Signs: - Mastoid tenderness: Tenderness is present over mastoid process, mastoid, tip, posterior border or root ofzygome
- Mucopurulent / purulent pulsatile discharge (light house effect).
- Sagging of postero superior meatal w a l l . 0

- Perforation of pars tensa of tympanic membrane.


- Obliteration of retroauricular sulcus due t o swelling over m a s t o i d . 0

- Pinna is pushed forward and downward (antero inferior).


- Conductive type of hearing loss is always present. 0

• X-ray mastoids shows clouding of air cells. •

l O C i s C T scan
Mgt - Antibiotics, myringotomy (If pus is under tension) and cortical mastoidectomy

C o r t i c a l M a s t o i d e c t o m y I n d i c a t i o n s in C a s e o f M a s t o i d i t i s

• Sub periosteal abscess


• Sagging of postero superior meatal wall
• Positive reservoir sign (i.e. meatus immediately fills w i t h pus after it has been moped out).
• No change in condition of patient or it worsens inspite o f adequate medical treatment for 48 hours.
• Mastoiditis leading t o complications eg facial nerve palsy, labyrinthitis or intracranial complications.
55. Ans. is a i.e. Bezold abscess
56. Ans. is b i.e. Sternocleidomastoid Ref. Dhingra 5th/ed pg 87,6th/ed p 78-79; Tuli Ist/ed pg 56
Bezold abscess - Pus passes t h r o u g h the tip of mastoid into the sternocliedomastoid muscle in the upper part of neck.
Here in 55 some of you may confuse the answer w i t h cielli's abscess - this is becoz the language given in Dhingra is very confusing.
CHAPTER 22 Diseases of Middle Ear J 295
For all purposes

Remember: • In citelli's abscess - pus is seen in digastric triangle after passing t h r o u g h inner table of mastoid process.

57. Ans. is a i.e. Gradenigo syndrome


58. Ans. is a i.e. It is associated with conductive hearing loss
59. Ans. is c i.e. VII nerve palsy Ref. Dhingra Sth/ed p 89,6th/ed p 79

G r a d e n i g o ' s S y n d r o m e is t h e C l i n i c a l M a n i f e s t a t i o n o f P e t r o s i t i s

CSOM Acute coalesent mastoiditis Latent mastoiditis

T
Abscess at petrous apex
Petrositis
Clinically presents as
Gradenigo's syndrome which consists of triad of (3D's)

• Diplopia (due to VI nerve palsy)


• Deep seated ear / retro orbital pain (due to Vth nerve irritation)
• Persistent ear Discharge
.-. Answer 57 is obvious i.e. Gradenigo's syndrome.
• As far as Ans 58 is concerned, t h o u g h conductive deafness may also be seen in petrositis (as petrositis is a complication of CSOM);
but since we have t o exclude one o p t i o n , it is the answer of consensus here.
Management: - Antibacterial therapy
- Surgery - Cortical / Modified Radical / Radical mastoidectomy.
60. Ans. is b i.e. Immediate mastoidectomy Ref. Dhingra Sth/ed p 90,6th/ed p 80
61. Ans. is c i.e. Immediate mastoid exploration Ref. Dhingra Sth/edp 90,6th/ed p 80

Facial Palsy a n d C S O M

In CSOM, facial palsy may be due t o erosion of fallopian canal by cholesteatoma (which erodes fallopian canal) osteitis, or demin-
eralization.The treatment should be urgent mastoid exploration, w i t h decompression o f t h e facial nerve in the fallopain canal.

• However, the scenario is not the same in ASOM. An acute inflammatory process cannot effectively erode the bony falopian canal within the
short period of time. Hence, the only possibility in a patient with ASOM to develop facial palsy is the presence of a congenitally dehiscent
fallopian canal (facial nerve without a bony canal), which is the commonest congenital maliformation of temporal bone.
• Thus in this case the treatment is myringotomy to relieve pressure on the exposed nerve or sometimes cotical mastoidectomy.

62. Ans. is a i.e. Cochlear Aqueduct:


Ref: Dhingra Sth/edp 11,12,6th/edp9, 10 Current, Diagnosis and Treatment in Otorhinology 2nd/143
Cochler aqueduct is a bony canal that connects the cochlea t o the intracranial subrachnoid space. Perilymph w i t h i n the cochlear
aqueduct is in direct continuation w i t h the CSF and hence Cochlear Aqueduct is the most important route for meningitis t o spread
t o the inner ear.

Stapes Helicotrema

Scala media
(endolymph)

Subarachnoid
space
C.S.F

tf

296 T SECTION V Ear

Vestibular Aqueduct

• Vestibular Aqueduct is also a bony connection between the cerebral subarachnoid space and the inner ear.
• Vestibular Aqueduct contains the endolymphatic duct which contains the endolymph.The endolymph within the endolymphatic
duct does not however communicate freely w i t h the CSF as it forms a closed space and ends in a cul-de-sac.
• Because the endolymph does not directly communicate w i t h the CSF. Vestibular aqueduct is less i m p o r t a n t in allowing spread
of meningitis f r o m CSF t o inner ear than Cochlear Aqueduct.

Hyrtle's Fissure

• Hyrtle's fissure is an embryonic remnant that normally obliterates by 24 weeks.


• When persistent, Hyrtle's fissure provides a connection from the middle ear t o the subarachoid space
• Hyrtle's fissure does not directly communicate the internal ear w i t h CSF and usually obliterates early in life and hence is not an
important route of spread of infection f r o m CSF t o internal ear.


Endolymphatic Sac

Endolymphatic sac is a cul-de-sac containing endolymph that does not directly communicate w i t h CSF.

63. Ans. is a i.e. CSOM


64. Ans. is b and c i.e. CSOM with lateral sinus thrombosis inturn can cause brain abscess; and Most common complication of
CSOM Ref. Turner Wth/edp 311-312; Dhingra 5th/edp 92-93,6th/edp 82
• Commonest organisms in otogenic brain abscess include gram-negative (Proteus, E.coli, Pseudomonas) and anaerobic
bacteria along with Staphylococcus and Pneumococci.
• M/C complication of CSOM = Brain abscess.
• Lateral sinus thrombosis is usually preceded by a perisinus abscess, which may lead later on to cerebeller abscess.
• Temporal lobe abscess is usually associated w i t h hallucinations, visual field defects, and nominal aphasia, while personality
change is not a feature feature of temporal lobe abscess. (It is a feature of frontal lobe abscess.)
65. Ans. d i.e. H. influenzae Ref. Dhingra Sth/ed p 92-93,6th/ed p 82; Turner 19th/ed p 311-312
Commonest organisms in otogenic brain abscess include gram negative (Proteus, E coli, Pseudomonas) and anaerobic bacteria
along w i t h Staphylococcus and Pneumocci.
H. Influenza infection is a rare cause.
66. Ans. is b i.e. Gradenigo sign
67. Ans. is a i.e. Lateral sinus thrombosis Ref. Dhingra 5th/ed pg 95,6th/edp 84

Lateral Sinus Thrombophlepitis

• It is inflammation o f t h e walls of sigmoid sinus (lateral sinus) w i t h the formation of thrombus


Complication of acute coalescent mastoiditis or masked mastoiditis
Aetiology- •

Chronic SOM w i t h cholesteatoma


Many signs/ tests are associated with lateral sinus thrombophlebitis

J: T T 3
Greisengers sign Tobey Ayer test (Quickensteadts test) Crow beck test Delta sign
T I T T
Oedema over the posterior In this test CSF pressure is recorded Pressure on jugular vein
part of mastoid due to Seen in CT in
thrombosis of mastoid T
Internal jugular vein is pressed on
on one side produces
engorgement of retinal veins. case of lateral
emissary veins If there is thrombosed sinus, sinus
one side & again CSF pressure is measured
no such change is seen thrombosis

1_
If no increase in csf pressure If there is increase in pressure
T T
Means a thrombus is present i.e. no Lateral sinus thrombosis
which prevents - in csf pressure present
T
Lateral sinus thrombosis present
CHAPTER 22 Diseases of Middle Ear J 297
68. Ans. is a i.e. ASOM Ref. Tuli ist/edp 53; Dhingra 5th/edp 86,6th/edp 62
• Light house sign is seen in acute ASOM and in acute mastoiditis following ASOM.
• The is mucopurulent or purulent discharge, often pulsatile
• Onotoscopy examination of ear, this pulsatile discharge reflects light which called as light house effect
69. Ans. is b i.e. Brainstem evoked auditory response Ref. Ghai 6th/ed p 518; Dhingra 5th/ed p 132

H. I n f l u e n z a T y p e M e n i n i g t i s

"It is frequent in children between the ages of 3 and 12 months. Residual auditory deficit is a common
complication."
... Ghai 6th/edp 518
• Since, residual auditory deficit is a c o m m o n complication of H. influenza meningitis so audiological test to detect t h e deficit
should be performed before discharging any patient suffering f r o m H. influenza meningitis.
• In children best test to detect hearing loss is brainstem evoked auditory response.
"Auditory brainstem response is used both as screening test and as a definitive hearing assessment test in children".
... Dhingra 4th/ed p 117
70. Ans. is c i.e. Immediate mastoid exploration Ref. Turner Wth/edp 301; Scott's Brown 7th/ed Vol3p 3437
"Patient of CSOM with cholesteatoma when complains of vertigo - means cholesteatoma has given rise to labyrinthitis."
... Dhingra 5th/ed p 90-91,6th/ed p 80

Labyrinthitis can be

T X. •

Circumscribed Diffuse serous Diffuse purulent


• V common •Less common • Rare
• Less serious type •Serious type • V serious type as it leads to -
• Occurs due to erosion of bony •Generally follows permanent loss of vestibular
capsule of labyrinth usually circumscribed type and cochlea functions
lateral semicircular canal
(i.e. labyrinthine fistula) J
T
Management: Management
Bed rest and labrynthine sedatives
• Immediate mastoid exploration Myringotomy is done if labyrinthitis follows acute otitis media,
cortical mastoidectomy is done in acute mastoiditis or modified
radical mastoidectomy is done in cholesteatoma.
Since circumscribed labyrinthitis is the most common type, so immediate mastoid exploration will be the option of choice.

Since circumscribed labyrinthitis is the most c o m m o n type, so immediate mastoid exploration will be the o p t i o n of choice.
The answer is further supported by Scotts Brown T /ed vol-3 pg-3437 which says.
h

I "Chronic low grade imbalance w i t h or w i t h o u t detectable nystagmus, implies the development of a labyrinthine fistula"
- Scoffs Brown T /ed vol-3 pg-3436
h

Patients w i t h active squamous COM (i.e. Cholesteatoma present) and a suspected labyrinthine fistula should have early surgical
management t o prevent deterioration of inner ear function. Most surgeons recommend a canal wall d o w n mastoidectomy."
- Scoffs Brown 7 /ed vol-3 pg-3437
th

SURGICAL MANAGEMENT OF MIDDLE EAR SUPPURATION


71. Ans. is c i.e. Myringotomy Ref. Dhingra Sth/ed pg 7 l,6th/edp 63
Child presenting w i t h acute otitis media which is not relieved by antibiotics and bulging tympanic membrane is an indication for
myringotomy. .
Indications of myringotomy in acute otitis media:
• Drum is bulging + acute pain.
• Incomplete resolution despite antibiotics w h e n d r u m remains full with persistent conductive deafness.
• Persistent effusion beyond 12 weeks.
72. Ans. is a i.e. Myringotomy with Penicillin Ref. Dhingra 5th/ed pg 71,6th/edp63
Fever + earache + congested and bulging tympanic membrane in a four year old child points towards Acute suppurative otitis
media as the diagnosis.
• Antibiotics (Penicillin) f o r m the mainstay of treatment of acute otitis media and should be administered in a child w i t h Acute
otitis media and once tympanic membrane is bulging, my ringotomy should be done.
298 L SECTION V Ear

• Grommet insertion is not indicated in Acute suppurative otitis media. It may be used in cases of myringotomy for serous or
secretory otitis media.
73. Ans. is c i.e. Myringotomy Ref. Dhingra 5th/edp 407,6th/edp 65; Scotts Brown 7th/ed vol 1 pg. 896 and vol 3 pg 3392
Friends, all o f us know that in serous otitis media - myringotomy (i.e. incising the tympanic membrane in order t o drain the
suppurative effusion o f t h e middle ear) is done.

In C h i l d r e n

TOC of serous otitis media -»insertion of grommet/ventilation t u b e along w i t h adenoidectomy (if features of adenoid hyperplasia
are present)

In A d u l t s (Scotts Brown 7th ed vol 3 pg. 3392)

In case of serous otitis media without nasopharyngeal carcinoma.


M y r i n g o t o m y w i t h ventilation tube insertion is done (In adults ventilation t u b e improves hearing for a very short term < 1 yr)
If there is nasopharyngeal cancer along with serous otitis media
Then there are t w o treatment options: (i) Hearing aid (ii) Myringotomy w i t h o u t ventilation t u b e insertion
Recently, C 0 laser assisted tympanic membrane ventilation has been advocated for the treatment of adult OME.
2

74. Ans. is d i.e. all of the above Ref. Dhingra 5th/ed p71,73,6th/ed p 65,66; Scotts Brown 7th/ed vol/pg 896-897
As discussed in the text
Myringotomy is coupled with grommet tube insertion in:
1. Serous otitis media (also k/a mucoid otitis media/glue ear)
2. Adhesive otitis media
3. Recurrent acute otitis media
4. Meniere's disease
75. Ans. is d i.e. Postero Inferior

76. Ans. is b i.e. Anteroinferior quadrant Ref. Dhingra 5th/edp 71,73,6th/edp65,398;SIB 7/ed Vol, pg 896-897

Explanation
• As discussed- Ideal site for incision in ASOM is postero inferior quadrant
• For serious otitis media/Grommet insertion, ideal site is anterio inferior quadrant (though Dhingra says it can be posterior inferior
also) as is proven by the following lines from scotts brown:
Site of insertion of grommet
"Insertion ofthe ventilation tube posterosuperiorly is not recommended because ofthe potential for damaging the ossicular chain. It
makes no difference to the extrusion rate as to whether the tube is inserted through a radial or circumferential incision and whether
sited anterosuperiorly rather than antero-inferiorly.
• Placement antero-inferiorly compared with placement postero-inferiorly lengthens the time a ventilation tube is in situ.
To maximise the duration of potential tube function, the preferred insertion site is anteroinferior through a circmferential or radial
incision. Ref. Scott Brown 7th/ed vol/pg 896-897
77. Ans. is d i.e. tympanic membrane
78. Ans. is a and c Temporalis fascia and perichon drium Ref. Dhingra 5th/d, pg 409,416,6th/ed p 406-407
• Myrirgoplasty is repair of a perforation o f t h e tympanic membrane (the pars tensa).
• Tympanoplasty is ossicular reconstruction w i t h or w i t h o u t myringoplasty.
- Myringoplasty is done using the graft made of either o f t h e following materials.
• Temporalis Fascia (most common)
• Perichondrium from tragus
• Tragal cartilage
• Vein
79. Ans. is a i.e. tympanoplasty Ref. Dhingra 5th/edp 35,6th/edp 30; Tuli Ist/edp 491
As discussed- columella effect is seen in Type III tympanoplasty
Type III tympanoplasty is also the M/C type of tympanoplasty performed
80. Ans. is a i.e. Operative microscope Ref. Maqbool 11th/edp 62
M y r i n g o t o m y i:e surgery on Ear Drum is performed under the operating microscope under general anaesthesia.
81. Ans. is b i.e. 250 mn Ref. Temporal Bone Surgery by MS Taneja Ist/edp 16
Focal length of objective lens:
• For ear surgery-200 m m / 2 5 0 m m
• For Nose / Paranasal sinus surgery - 300 mn
• For Laryngeal surgery-400 m n

-
CHAPTER 22 Diseases of Middle Ear J 299

82. Ans. is a i.e. cortical mastoidectomy


83. Ans. is c i.e coalescent mastoiditis
84. Ans. is b i.e coalescent mastoiditis Ref. Dhingra 5th/ed pg411,6th/ed p401
Schwartz operation in another name for cortical/sample mastoidectomy
Explanation: See preceeding text for indications of cortical mastoidectomy
85. Ans. is c i.e. Atticoantral cholestotoma
86. Ans. is d i.e maintenance of patency of eustachian lube
87. Ans. is c i.e Cochlea removed Ref. Dhingra Sth/ed pg 413-414,6th/edp 405

Explanation

• Radical mastoidectomy is a procedure t o eradicate disease f r o m middle ear and mastoid w i t h o u t any a t t e m p t t o reconstruct
hearing.
• It is rarely done these days - Its only indications are:
- Malignancy of middle ear
- When cholesteatoma cannot be removed safely eg if it invades eustachian tube, round w i n d o w or perilabrynthine cells
- If previous attempts t o eradicate cholesteatoma have failed
Following steps are done in radical mastoidectomy:
1. Posterior meatal wall is removed and the entire area of middle ear, attic, antrum and mastoid is converted into a single cavity,
by removing the bridege and lowering the facial ridge.
2. All remnants of tympanic membrane, ossicles (except the stapes foot plate) and mucoperiosteal lining are removed (Not cohlea)
3. Eustachian tube is obliterated by a piece of muscle or cartilage
4. The diseased middle ear and mastoid are permanently exteriorised for inspection and cleaning.

Remember: Bridge is the most posterosuperior part of bony meatal wall lateral t o aditus and anturm, which overlies the Notch of
Rivinus while facial ridge lies lateral t o fallopian canal. Bridge is removed and ridge is lowered in radical or modified radical operation.

88. Ans. is a i.e. Facial nerve Ref. Scotts Brown 7th of vol 3 pg 3434
Complications of mastoid surgery- are actually very u n c o m m o n
1. Facial palsy (rare)
2. Total hearing loss/dead ear

The incidence of facial palsy is widely accepted to be rare in the hands of expert surgeons, however, total loss of hearing also occurs in the hands
of expert.

89. Ans. is c i.e. Coalascent mastoiditis Ref. Dhingra 5th/ed pg411,6th/ed p 400
Now, this is one of those questions where we can get the answer by exclusion.
Here we know- management of coalescent mastoiditis is cortical mastoidectomy so obviously is not done in this case.
Indications of are:
1. Cholesteatoma confined t o the attic and antrum
2. Localised chronic otitis media
90. Ans. is c i.e. Canal wall down mastoid ectomy
Ref. Dhingra 5th/edpg 82,6th/ed p 73; Turner 1 Oth/ed pg 304; Current Otolaryngology 2nd/ed pg 670 5th/B 7th/ed Vol 3, pg 3432-3433
As discussed in aitic cholesteatoma we do and if cholesteatoma invades eustachian tube or perilabyrynthine tissue t h e n manage-
ment is Radial Mastoidectomy. Now whether we perform radical mastoidectomy or modified radical mostoidectomy b o t h are canal
wall d o w n procedures.
91. Ans. is a i.e. - Ref. Scott's Brown 7th/ed vol-3 pg-3436
• CSOM / cholesteatoma generally presents w i t h conductive type of hearing loss.
• If cholesteatoma (CSOM) is presenting w i t h sensorineural hearing loss, it means it is associated w i t h some complications or it
can be due to the use of potentially ototoxic ear drops.
• Management w o u l d still remain .
92. Ans. is a i.e. Cutting drill over the bleeding area Ref. Internet
Here the answer is obvious as Cutting drill over the bleeding area will increase the bleeding instead of stopping it.
• Diamond drill over the bleeding area will produce heat and stop the bleeding.
• Bipolar cautery can be used t o control bleeding during mastoid surgery (Not monopolar cautery).
• Bone wax is also commonly used t o control bleeding during mastoid surgery(lt seals the bleeding site).
• ii
I
-

CHAPTER

23
-

Meniere's Disease
• •

| MENIERE'S DISEASE (ENDOLYMPHATIC HYDROPS) Clinical Featuers

1 st Symptom Vertigo: Onset sudden


It is a disease o f t h e membranous inner ear characterised by triad
Episodic in nature. It typically increases over a period of
of fluctuating deafness, episodic vertigo and fluctuating tinnitus.
minutes and then usually lasts for several hours.
The additional s y m p t o m of aural fullness has been added t o the
Associated w i t h nausea, vomiting, pallor, sweating, diar-
current definition.
rhoea and bradycardia
- No loss of conciousness.
Incidence

< Fluctuating Hearing loss:
• Males affected more than females. Fluctuant and progressive SNHL
• Age of onset: 35-60 years [Peak: 5-6th decade) Initially - low frequency losses
• Generally unilateral ( n o predilection for left or right side) Later both - high and low frequency are involved
• Diplacusis: sounds perceived as of abnormal high pitch.
Pathology
Intolerance t o loud sounds due t o recruitment p h e n o m -
M e m b r a n o u s labyrinth contains e n d o l y m p h and in Meniere's enon (therefore these patients are poor candidates for
disease the membranes containing this e n d o l y m p h are dilated hearing aids).
like a balloon due t o increase in pressure. This is called hydrops. '• Tinnitus: roaring type and fluctuating in nature.
So, t h e main p a t h o l o g y in Meniere's disease is distension o f 1
• Aural fullness.
endolymphatic system mainly affecting the cochlear duct (scala -

media) and the saccule, and t o a lesser extent t h e utricle and Other Features
semicircular cannals. Therefore, Meniere's disease is also called
• Tullios p h e n o m e n o n : seen in some cases o f Meniere's
endolymphatic hydrops.
disease: Subjective imbalance and nystagmus observed in
response t o loud, low frequency noise exposure.
Pathophysiology
• Henneberts sign: False positive fistula test seen in Meniere's
See f l o w chart 23.1: disease


Flow Chart 23.1: Pathophysiology meniere's disease.

Endolymph moves from cochlea (site of production) to


endolymphatic sac (site of absorption)

Any disturbance in production or absorption


Accumulation of endolymph in

I.
Cochlea Membranous labyrinth

X - •
Hearing loss Vertigo

CHAPTER 23 Meniere's Disease

Nystagmus:

Atypical attack of meniere's disease has three phases, each defined by the direction of spontaneous nystagmus

J: 3
1st phase 2nd phase 3rd phase
Irritative phase Paretic phase Recovery phase

Nystagmus - horizontal & beats towards j Nystagmus - horizontal & beats towards T T
Nystagmus again beats towards the
the affected ear the affected ear affected ear

Investigations
> Committee on Hearing and Equilibrium of the American Academy
• Tuning fork tests - show sensorineural hearing loss
of of Otolaryngology—Head and neck surgery (AA OHNS)
• P u r e t o n e a u d i o m e t r y : SNHL w i t h a f f e c t i o n o f l o w e r
classifed the diagnosis of Meniere's disease as follows
frequencies in early stages and the curve is o f rising type.
1. C e r t a i n : D e f i n i t e Meniere's disease c o n f i r m e d by
When higher frequencies are involved, curve becomes flat or
histopathology.
falling type.
2. Definite: Two or more definitive spontaneous episodes of
42% Flat audiogram
vertigo lasting 20 m m or longer.
32% Peaked pattern a. Audiometrically documented hearing loss on at least
19% Down ward sloping one occasion.
7% Rising pattern b. Tinnitus or aural fullness in the affected ear.
Speech audiometry - Discrimination score 55-85% c. All other causes excluded.
• Recruitment: present 3. Probable
• BERA: Shows reduced latency of wave V a. One definitive episode of vertigo.
• Electochochleography(ECoG):/Mostsens/tiVeflndd/ogiiosf/c. b. Audiometrically documented hearing loss on at least
Records the action potential and the summating potential of one occasion.
c. Tinnitus or aural fullness in the treated ear.
the cochlea t h r o u g h a recording electrode placed over the
d. Other causes excluded.
round w i n d o w area.
4. Possible
Normal w i d t h of summating potential / action potential
a. Episodic vertigo of Meniere's type without documented
= 1.2-1.8 msec. hearing loss (vestibular variant) or
Widening greater than 2 msec is usually significant b. Sensrineural hearing loss, fluctuating or fixed, w i t h dis-
- Summating potential (SP) / Action potential (Al) = 1:3 = 0.33 equilibrium but w i t h o u t definitive episodes (cochlear
(Normal) <30% variant).
In Menieres > 30-40% c. Other causes excluded.

The sensitivity of the test can be increased by giving the patient 4


Variants of Meniere's Disease
g of oral sodium chloride for 3 days prior t o electro cochleography
• Caloric tests: shows canal paresis (reduced response o n • There are some variants of Meniere's disease in w h i c h clinical
affected side) presentation is notthatclassical of Meniere's disease.
These variants are:
• Glycerol test:
. 1. Cochlear hydrops
Glycerol is given parenterally.
- Only the cochlear symptoms and signs of Meniere's
It produces a decreases in the intralabyrinthine pressure
disease are present. Vertigo is absent and it appears
and also improves the cochlear blood f l o w resulting in only after several years. There is block at ductus reu-
improvement of hearing loss or increase in discrimina- niens, therefore increased endolymphatic pressure is
t i o n score by 10. confined t o cochlea only.
• Reverse glycerol test: 2. Vestibular hydrops
Performed using acetazolamide - Patient gets typical episodes of vertigo while cochlear
Shows d e t e r i o r a t i o n in t h e pure t o n e thresholds and function remains normal. Typical picture of Meniere's
speech discriminaton scores. disease develops w i t h time.
3. Drop attacks (Tumarkin's otolithic crisis)
- There is sudden drop attack w i t h o u t loss of conscious-
ness.There is no vertigo or fluctuation in hearing loss.
• The diagnostic evaluation in menieres disease primarily includes (1)
Possible mechanism is d e f o r m a t i o n of t h e o t o l i t h
Audiometry (2) Fluoroscent treponemal antibody absorption (FTA
membrane o f t h e utricle or saccule due t o change in
- ABS) to rule out syphilis as syphilis can imitate Meniere's disease.
endolymphatic pressure.

302|_ SECTION V Ear

4. Lermoyez syndrome 5.
Other drugs
- Symptoms of Meniere's disease are seen in reverse - Propantheline bromide, phenobaritone and hyoscine
order. First there is progressive deterioration of hear- are effective alternatives.
ing, followed by an attack of vertigo, at which time 6. Avoid alcohol, smoking, excessive tea intake and coffee
hearing recovers. intake during treatment.
Newer Therapy:
| TREATMENT OF MENIERE'S DISEASE
lntratympanicGentamicinTherapy(CherrticalLabyrinthectomy)
I. Medical Management Gentamicin is mainly vestibultoxic. It has been used in daily or
II. Labyrinthine exercises biweekly injections into the middle ear. Drug is absorbed t h r o u g h
III. Surgical t h e r o u n d w i n d o w and causes d e s t r u c t i o n o f t h e vestibular
labyrinth. Total control o f vertigo spells has been reported in
I. M e d i c a l M a n a g e m e n t 6 0 - 8 0 % of patients.

• Initialtreatmentofmeniere'sdiseaseiswtth medical management. Microwick


• Medical treatment controls the condition in over t w o third of
It is a small wick made of polyvinly acetate and measures 1 m m x
patients.
9 m m . It is meant t o deliver drugs from external canal t o the inner
• Medical management includes:
ear and thus avoid repeated intratympanic injections. It requires a
1. Antihistamine labyrinthine sedatives (vestibular seda-
tympanostomy tube (grommet) t o be inserted into the tympanic
tives)
membrane and the wick is passed t h r o u g h it. When soaked w i t h
- Many cases can be controlled by vestibular sedatives
drug, wick delivers it t o the round w i n d o w to be absorbed into the
like prochlorperazine, promethazine, and cinnarzine.
inner ear. It has been used to deliver steroids in sudden deafness
2. Anxiolytic and transquillizers
and gentamicin t o destroy vestibular labyrinth in Meniere's disease.
- Many patients are anxious, therefore they may be
helped by anaxiolytic and transquillizers like diaz- II. L a b y r i n t h i n e E x e r c i s e s
epam.
Cooksey-Cawthorne exercise for adaptation of labyrinth:
3. Vasodilators
- Betahistine hydrochloride t h e most useful recent
III. S u r g i c a l M a n a g e m e n t
addition t o the medical management and is routinely
prescribed for most patients. It increases labyrinthine Surgery in meniere's disease can be conservative or destructive
(see table 23.1)
blood flow by releasing histamine.
- Other vasodilators employed include nicotinic acid,

I
thymoxamine, inhaled carbogen (5% C0 with 9 5 % S E C O N D A R Y E N D O L Y M P H A T I C H Y D R O P S O R D/D OF
2

0 ) , and histamine
2 drip. MENIERE DISEASE
- Vasodilators increase vascularity of endolymphatic
E n d o l y m p h a t i c hydrops is n o t u n i q u e t o Meniere's disease.
sac and its duct and thereby increases reabsorption
Meriere's disease also called as Primary Endolymphatic hydrops
of endolymphatic fluid.
as the cause of menieres disease is not known. Other conditions
4. Diuretics (furosemide) p r o d u c i n g e n d o l y m p h a t i c hydrops (secondary endolymphatic
- Diuretics w i t h fluid and salt restriction can help t o hydrops) are viral infection, syphilis, endocrine (hypothyroidism),
control recurrent attacks if not controlled by vestibular autoimmune, trauma, allergy, Paget's disease, acoustic neuroma,
sedatives or vasodilators. vertebrobasilar insufficiency and migrane, (CNS disease).

T a b l e 23.1 S u r g i c a l m a n a g e m e n t o f M e n i e r e ' s o f d i s e a s e

Conservative operation Destructive operations - rationale is to control vertigo


• Done where vertigo is present,.but hearing can be preserved • Endolymphatic hydrops causes fluid to accumalate which leads to
• Indications pressure in inner ear and that disturb vestibulate nerve leading to
- B/L disease vertigo. Destruction of inner ear/vestibular nerve prevents these
- Young patient abnormal signals.
• Decompression of endolymphatic sac. • Labyrinthectomy (hearing loss is permanent)
• Shunt operation, i.e. endolymphatic mastoid shunt, endo- lymphatic • Intermittent low pressure pulse therapy (Meniett device therapy)
mastoid shunt, endolymphatic subarchnoid shunt • Here first as grommet is placed through myringotomy and then the
• Sacculotomy (Pick's operation and Cody tack procedure) appratus is placed in ear canal which influences the round window
• Vestibular nerve section membrane pressure and thus bring down endolymphatic pressure
• Laser/ultrasonic cause partial destruction of labyrinth without hearing
loss
CHAPTER 23 Meniere's Disease J 303

QUESTIONS

Which ofthe following is not a typical feature of meniere's Carhart's Notch is a characteristic feature in puretone
disease? [AIIMS May 06] audiogram
a. Sensorineural deafness b. Pulsatile tinnitus Schwartz's sign is usually present in the tympanic membrane
c. Vertigo d. Fluctuating deafness Low frequency sensorineural deafness is often seen in pure
Menier's disease is characterised by all except: tone audiogram
[AIIMS Dec. 98] Decompression fallopian canal is the treatment of choice
a. Diplopia b. Tinnitus 13. Recruitment phenomenon is seen in?
c. Vertigo d. Fullness of pressure in ear [DNB 2007/Kolkatta 2002]
All are manifestations of Meniere's disease except: Otosclerosis
[AI97] Meniere's disease
a. Tinnitus b. Vertigo Acoustic nerve schwannoma
c. Sensorineural deafness d. Loss of consciousness Otitis media with effusion
Meniere's disease is manifested by all of the symptoms 14. Vasodilators in Menieres disease are useful because
except: [Delhi 96] they: [Kerala 94]
a. Tinnitus b. Vertigo a. Dilate lymphatic vessels
c. Deafness d. Otorrhoea b. Decrease endolymph secretion
Meniere's disease is characterized by: [AI04] c. Increase endolymph reabsorption
a. Conductive hearing loss and tinnitus d. Are of no use
b. Vertigo ear discharge tinnitus and headache 15. Vasodilators of internal ear is:
c. Vertigo, tinnitus hearing loss and headache a. Nicotinic acid
d. Vertigo, tinnitus and hearing loss b. Histamine
True about Meniere's disease: [PGIJune 03] c. Serotomin
a. Tinnitus d. Kinins
b. Episodic vertigo 16. Endolymphatic decompression is done in: [Delhi 2006]

c. Deafness a. Tinnitus b. Acoustic neuroma
d. Diarrhoea c. Meniere's disease d. Endolymphatic fistula
e. Vomiting 17. Destructive procedures for Meniere's disease are:
Meniere's disease is characterised by: [PGI Dec. 03] a. Fick's procedure
a. Fluctuating hearing loss Cody tack procedure
b. Also called endolymphatic hydrops Vestibular neuronectomy
c. Tinnitus and vertigo most common symtom Trans- labyrinthine neuronectomy
d. It is a disease of inner ear Labyrinthectomy
e. Endolymphatic decompression is done 18. Differential diagnosis of Meniere's disease are all expect:
The dilatation of Endolymphatic sac is seen in: [AI2011] a. Acoustic neuroma [UP 07]
a. Meniere's disease b. Otosclerosis b. CNS disease
c. Acoustuic neuroma d. CSOM c. Labyrinthitis
9. Meniere's disease is: [PGI June 99] . r .. ..... ,.
a. Perilymphatic hydrops b. Endolymphatic hydrops 19. d. Suppurative otitis media
c. Otospongiosis d. Coalescent mastoiditis A 55 year old female presents with tinnitus, dizziness and
10. True about Endolymphatic hydrops: [PGI June 06] n/o progressive deafness, which o f t h e following is not a
a. B/L Condition D/D: [AIIMS 2001]
b. Females more common a. Acoustic neuroma
c. 3 rd to 4 th decades b. Endolymphatic hydrops
d. Conductive deafness c. Meningitis
11. Glycerol test is done in: [API 995, TN 2000] 20. d. Histiocytosis'X'
a. Otosclerosis Initial mechanism of action of intra-tympanic gentamicin
b. Lateral sinus thrombosis microwick catherter inserted into inner ear in treatment
c. Meniere's disease of menier's disease: [AIIMS Nov. 2012]
d. None of the above a. Damage outer hair cell
12. In a classical case of Meniere's disease which one of the b. Binds to hair cell Na -K ATPAse channel
+ +

following statement is true: [Karn 01] c. Acts on melanoreceptors of outer hair cell
d Bind to Mg channel
2+
304 T SECTION V Ear

EXPLANATIONS AND REFERENCES


1. Ans. is b i.e. Pulsatile tinnitus
2. Ans. is a i.e. diplopia
3. Ans. is d i.e. Loss of consciousness
4. Ans. is d i.e. Otorrhoea
5. Ans is c i.e. Vertigo, tinnitus, hearing loss and headache
6. Ans. is a, b, c, d, e i.e. Tinnitus, Episodic vertigo; Deafness, Diarrhea and vomitting
Ref. Dhingra 5th/edpg 112,6th/edp 100,101; Tuli Ist/edp 127; Harrison 17/e, p 202, Turner 10/r, p 335

M i n i e r e ' s D i s e a s e is C h a r a c t e r i s e d b y

• Fluctuating tinnitus.
• Fluctuating deafness of sensorineural type
• Episodic vertigo (accompanied by nausea vomitting and vagal disturbances like abdominal cramps, diarrhea and bradycardia)
• Aural fullness.
Emotional disturbances, headache and anxiety.

• Pulsatile tinnitus is seen in glomus jugulare, AV shunts, aneurysms, stenotic arterial lesions. It may also occur in secretory otitis media.
• In the early stages of disease most patients are well in between the attack. As the disease progresses patients may have persistent hearing loss,
tinnitus and postural imbalance between the attacks of vertigo
• Some patients in the later stages develop drop attacks k/a Tumarkin or otolithic crisis due t o otolith dysfunction
• During this attack patient simply drops without a warning. There is no associated vertigo or loss of consciousness

7. Ans. is a, b, c, d and e i.e. Fluctuating hearing loss; Also called endolymphatic hydrops; Tinnitus and vertigo are most com-
mon symptoms. It is a disease of inner ear; Endolymphatic decompression is done Ref. Dhingra 6th/edp 100-101

Meniere's Disease

• It is a disease o f inner ear 0

• It is characterized by distension o f t h e endolymphatic system mainly affecting the cochlear duct (Scala media) and t h e saccule,
and t o a lesser extent the utricle and the semicircular canals
Hence it is also k/a Endolymphatic hydrops
• Because the pathology lies in the endolymphatic system so endolymphatic sac decompression can be used as a management
option
Endolymphatic sac surgery may result in a reduction in the frequency, duration and intensity of vertigo attacks. Although popular,
it is not always effective in stopping the vertigo attacks and has no benefits for the auditory symptoms
• Fluctuating hearing loss and tinnitus and vertigo are all seen in meniere's disease.
8. Ans. is a > c i.e. Meniere's disease > Acoustic neuroma Ref. Dhingra 5th/ed p 111,6th/edp 103
9. Ans. is is b i.e. Endolymphatic hydrops Ref. Dhingra 5th/edpg111,6th/edp 103
Meniere's disease, which is an idiopathic lesion, is a clinical diagnosis. The following conditions, which are included in Meniere's
syndrome or secondary Meniere's disease, can mimic the clinical features of Meniere's disease and should be kept in mind.
• Migraine and basilar migraine
• A u t o i m m u n e disease of inner ear and otosclerosis
• Syphilis and Cogan's syndrome
• Cardiogenic
. Vertebral basilar insufficiency
• Trauma: Head injury or ear surgery
• Acoustic neuroma

Also Know

• Lermoyez syndrome is a variant of Meniere's disease, where initially there is deafness and tinnitus, vertigo appears later w h e n
deafness improves.
CHAPTER 23 Meniere's Disease J 305

10. Ans. is c i.e. 3rd to 4th decade Ref. Dhingra Sth/edp 112,6th/ed p 100-101
• Meniere's disease lead to sensorineural hearing loss and not conductive type.
• Generally unilateral
• Age = Most common 35-60 years.
• It is more c o m m o n in males

Also remember: • Otosclerosis - Bilateral condition, more c o m m o n in females


• Bells palsy - Unilateral condition w i t h equal sex distribution
• Acoustic neuroma - Unilateral w i t h condition equal sex distribution
• Glomus t u m o r - More c o m m o n in females
11. Ans. is c i.e. Meniere's disease Ref. Dhingra Sth/ed p 113,6th/edp 102
Glycerol is a dehydrating agent. When given orally, it reduces endolymph pressure and causes improvement in hearing as evidenced
by an improvement of 10 dB or 1 0 % gain in discrimination score in menieres disease patients.
12. Ans. is c i.e. Low frequency sensores neural deafness is often seen in pure tone audiogram
Ref. Dhingra 5th/edp 113,6th/edp 101
• Carharts' Notch and Schwartz's sign are seen in otosclerosis - Dhingra 5/e pg-98-99,6/ep87
• Decompression of endolymphatic sac (and not fallopian canal) is done in Menieres disease. - Dhingra 5/e pg-116,6/e pi 04
• Decompression of Fallopian canal is done in traumatic facial nerve palsy.
• Meniere's disease is associated w i t h - SNHL which affects low frequencies first, followed by higher frequencies later.This is visible
o n pure tone audiogram.

ALSO KNOW

• Conductive deafness often involves all frequencies (high as well as low) whereas sensorineural hearing loss such as presbycusis
affects higher frequencies more than lower frequencies. Meniere's disease is an exception which affects lower frequencies more.
• Hennebert's sign: False positive fistula test is seen in Meniere's disease.
13. Ans. is b i.e. meniere's disease Ref. Dhingra 5th/edpg31,113,6th/edp 101

Recruitment Phenomenon

• It is a p h e n o m e n o n of abnormal growth of loudness0

• The ear which does not hear low intensity sound begins t o hear greater intensity sounds as loud or even louder than normal
hearing ear.
• Thus a loud sound which is tolerable in normal ear may grow t o abnormal levels of loudness in the recruiting ear and thus
become intolerable Q

• Recruitment is typically seen in lesions of c o c h l e a i.e. meniere's disease , presbycusis.


0 0 0

• Patients w i t h recruitment are poor candidates for hearing a i d s . 0

14. Ans. is c i.e. Increase endolymph reabsorption Ref. Dhingra 5/e, p 115,6/e pi 04
15. Ans. is a and b i.e. Nicotinic acid and histamine
Ischaemia of endolymphatic sac

I absorption of endolymph
4-
Endolymphatic hydrops/menieres disease
Vasodilators improve labyrinthine circulation, So, increase e n d o l y m p h reabsorption

Vasodilators Used

During acute attack:


. Carbogen OS /, 0 +5°/o C0 )
0
2 2

• Histamine (contraindicated in asthmatics)


During chronic attack:
• Nicotinic acid
• Betahistine
16. Ans. is c i.e. Meniere's disease Ref. Dhingra 5th/ed p116,6/ep104
As explained in Text:
• Decompression o f endolymphatic sac is done in menieres disease
17. Ans. is e i.e. Labyrinthectomy Ref. Dhingra 5th/edpg 116,6th/edp 104
306 L SECTION V Ear

S u r g i c a l P r o c e d u r e s for M e n i e r e ' s D i s e a s e

• Surgical therapy for Meniere's disease is reserved for medical treatment failures and otherwise contraindicated.
• Surgical procedures can be divided into main categories
X * ... - i i

- Destructive surgical procedures


- Nondestructive surgical procedures
• Destructive surgical procedures: rationale t o control vertigo Endolymphatic hydrops causes fluid pressure accumulation within
the inner ear, which causes temporary malfunction and misfiring of the vestibular nerve. These abnormal signals cause vertigo.
Destruction o f t h e inner ear and /or the vestibular nerve prevents these abnormal signals. The procedures performed are:
- Labyrinthectomy 0

- Intermittent low pressure pulse therapy (Meniett device therapy)


• Conservative surgical procedure : are used in cases where vertigo is disabling but hearing is still useful and needs t o be
preserved. They are:
- Decompression of endolymphatic sac
- Endolymphatic shunt operation"
- Sacculotomy (Fick's o p e r a t i o n and Cody tack procedure )
0 0

- Vestibular neuronectomy
- Ultrasonic destruction o f vestibular labyrinth t o preserve cochlear function.
18. Ans. is d i.e suppurative otitis media Ref. Logan Turner Wth/edp 334,337,338
Differential D i a g n o s i s of Vertigo + Tinnitus + S N H L D e a f n e s s - Includes:

Meniere's disease (Endolymphatic hydrops)


Syphilis
• Labyrinthitis .
Labyrinthine trauma due t o fracture of temporal bone, postoperatively after stapedectomy
Cogan syndrome
Labyrinthine hemorrhage due t o blood dyscrasia
. L . ' . , , , . . . .

Acoustic neuroma/meningioma (CP angle lesion),


Multiple sclerosis,
Hypothyroidism/Hyperlipidenia

In serous otitis media these symptoms may be seen but then hearing loss will be of conductive variety and not SNHL

19. Ans. is d. i.e. Histiocytosis'X' Ref. Current Otolaryngology 2nd/edp 616

Delayed Endolymphatic Hydrops


• Hydropes sometimes develop in patients w h o have lost their hearing in one or both ears previously. The causes of hearing loss
vary, f r o m head injury, meningitis or any other etiology. Patient subsequently develops attacks of vertigo similar t o that seen
in Meniere's disease in a delayed fashion.
• Histocytosis X belong t o the group of disorders collectively termed inflammatory reticuloendotheliosis chracterizee by multiple
osteolytic lesions involving skull, temporal bone, long bones, ribs, and vertebrae. There is generalized lymphadenopathy,
hepatosplenomegaly, and in severe cases involvement o f t h e bone marrow. Involvement of temporal bone leads t o features
mimicking complicated like otorrhea, mastoiditis, facial palsy, and labyrinthitis.
20. Ans. is b i.e. direct damage.to outer hair cells Ref. Dhingra 5/e pi 16,6/e p 104

Intratympanic Gentamicin Therapy

• Gentamicin is mainly vestibulotoxic.


• It has been used in daily or biweekly injection into the middle ear.
• Drug is absorbed t h r o u g h round w i n d o w and causes destruction of vestibular labyrinth.
• Total control of vertigo spells has been reported in 60-80% of patients w i t h some relief f r o m symptoms in others.
• Hearing loss, sometimes severe and profound, has been reported in 4 t o 3 0 % of patients treated w i t h mode of therapy.



CHAPTER
-

CDtoscIero;

§ ANATOMY PERTAINING TO LABYRINTH T y p e s of Otosclerosis

1. O t i c l a b y r i n t h : Also c a l l e d m e m b r a n o u s l a b y r i n t h or
Stapedial Cochlear Histological type
endolymphatic labyrinth. It consists of utricle, saccule, cochlea,
(Fenestral) (Fenestral)
semicircular ducts, endolymphatic duct and sac. It is filled w i t h
endolymph. Most common type Involves round Lesion detected only
2. Periotic labyrinth or perilymphatic labyrinth (or space). Most common site window on post-mortem
It surrounds the otic labyrinth and is filled w i t h perilymph. It is:- • •
includes vestibule, scale tympani, scale vestibuli, peri-lymphatic Fissula
space of semicircular canals a n d t h e p r i o t i c d u c t , w h i c h antefenestrum 0

surrounds the endolymphatic duct of otic labyrinth.


On histopathology "Blue mantles"are characteristic.
3. Otic capsule. It is the bony labyrinth. It has three layers.
a. EndosteahThe innermost layer. It lines the bony labyrinth.
Clincial Feature
b. Enchondral: Develops from the cartilage adn later ossifies
into bone. It is in this layer that some islands of cartilage Symptoms:
are left unossified that later give rise to otosclerosis.
• Deafness: Slowly progressive B/L c o n d u c t i v e deafness,
c. Periosteal: Covers the bony labyrinth.
in a female aggravated by pregnancy generally suggests
otosclerosis. U/L hearing loss may occur in 1 5 % cases
| OTOSCLEROSIS
• Paracusis willisi : patient hears better in noisy surrounding.
0

It is a hereditary localised disease o f t h e bony labyrinth (bony otic • T i n n i t u s : Indicates sensorineural h e a r i n g loss (cochlear
capsule) characterised by alternating phases of bone resorption otosclerosis).
and f o r m a t i o n . Here the normal dense enchondral layer of the • V o i c e of t h e p a t i e n t : Q u i e t v o i c e , l o w v o l u m e s p e e c h
bony otic capsule gets replaced by irregularly laid spongy bone. because they hear their o w n voices by bone conduction and
Etiology consequently talk quietly.
• Vertigo: Generally not seen.
• Autosomal d o m i n a n t . 5 0 % cases are hereditary
0

• In cochlear otosclerosis-there may be vertigo, tinnitus and


• Male: Female is 1:2 b u t in india it is 2:1
SNHL
• Age g r o u p affected
20-45 years (maximum between 20-30 years)
Signs
Puberty pregnancy and menopause, accelerate the condition
Races: • On otoscopic examination: tympanic membrane is normal 0

White > Negroes - More in the Caucasians and mobile in most o f t h e cases.

1
• In 7 0 - 8 5 % Bilateral.

In 10% cases flamingo-pink blush is seen through the tympanic


Site - Most common is fissula ante fenestrum (anterior to the membrane called as Schwartze's sign.
oval window) 0 Importance of Schwartz sign: It is indicative of active focus with
increased vascularity.
• ! 1
• Surgery is contraindicated in patients with Schwartz sign and is
Other sites:
Round w i n d o w area - Stapedial footplate an indication for sodium fluoride therapy.
— — • ! —: &
Internal auditory canal - Semicircular canal
308? SECTION V Ear

Tests
Classic audiometric findings in otosclerosis
• Tuning fork tests show conductive type of hearing loss. Low frequency conductive hearing loss. 0

- Rinne's: negative (first for 256 HZ and t h e n 512 HZ). Caharts n o t c h 0

Weber's: lateralised t o the ear with greater conductive loss.


Type A or As t y m p a r o g r a m 0

Absolute bone conduction: normal (can be decrease in


Diphasic or absent acoustic reflex . 0

cochlear otosclerosis).
Negative Rinne test
Gelles test: No change in the bone conduction threshold
w h e n air pressure is increased by Siegel's speculum.
| EXTRA EDGE
Tympanometry /Audiometry
Histological otosclerosis:The gold standard f o r t h e reporting
It is one o f t h e i m p o r t a n t tools in evaluating a patient suspected
of the incidence of histological otosclerosis is t h e study of
of otosclerosis
bilateral temporal b o n e . 0

A. Impedance audiometry: Patients w i t h early disease may


Imaging studies: I m a g i n g m o d a l i t y o f c h o i c e is h i g h
show Type A t y m p a n o g r a m (because middle ear aeration is
resolution CT s c a n . 0

not affected)
Progressive stapes fixation results in AS type curve.
Treatment
B. Acoustic reflex: It is one o f t h e earliest signs of otosclerosis
and precedes t h e development of an air bone gap Observation:
In the normal hearing ear: The configuration o f t h e acoustic • It is the least risky and least expensive option.
reflex pattern is one of a sustained decrease in compliance • Preferred for patients w i t h (i) unilateral disease (ii) Mild
o w i n g t o the contraction o f t h e stapedial muscle that lasts the conductive hearing loss
duration of stimulus • If the patient is not concerned about the hearing loss, then
In otosclerosis: In early stages no intervention is required.
A characteristic diphasic o n - o f f p a t t e r n is seen in w h i c h • Audiograms are obtained on yearly basis.
there is a brief increase in compliance at the onset and at • Hearing loss typically progresses slowly, ultimately requir-
the termination of stimulus occurs. This is pathognomic for ing intervention
otosclerosis. 0

Medial therapy:
In later stages:The Reflex is absent - Stapedial reflex is absent
(i) Sodium fluoride therapy

Pure Tone audiometry: Normally in otosclerosis - bone


• It is given in doses of 50 to 70 mg/day for 1 to 2 years.
conduction is normal but in some cases there is a dip in bone
conduction which is maximum at 2000 Hz and is called the Role
Cahart's notch.
• It reduces osteoclastic bone resorption and increases osteoblastic
Remember: Caharts notch disappears after successful
stapedectomy bone formation. (It hastens the maturity of active focus)
• It inhibits proteolytic enzymes that are cytotoxic to cochlea
(may lead to SNHL)
Frequency in Hertz
Dangers of Fluoride Therapy
125 250 500 1000 2000 4000 8000
• Fracture long bone and spine (due to fluorosis)
1 • So X-ray spine and X-ray of long bones are done as a routine for
0
p. j
"1 observing the thickening of trabeculae.
10 JJ-. ,
20 Y Indications
A
30 • Cochlear otosclerosis i.e. Malignant otosclerosis (rapidly
40 prorgressive cochlear otosclerosis)
• Radiologically active focus (New onset disease)
0

50 V • Patients with a positive Schwartz sign 0

c 60
\
/
• Adverse effect - most common Gl disturbances
CO
0 70 Contraindications
X
80
• Chronic nephritis
90 • Chronic rheumatoid arthritis
100 • Pregnant women / lactating women
110 • Children

Fig. 24.1: Caharts Notch


CHAPTER 24 Otosclerosis

(ii) Bisphosphonates (e.g. Alendronate, Etidronate) Other surgeries which can be done:
They are anti resorptive agents that are helpful for the Laser stapedotomy ( C 0 Argon and KTP)
2

prevention and treatment of osteoporosis and other Stapes mobilisation


conditions characterized by increased bone r e m o d - Lemperts fenestration operation: (outdated procedure).
eling. They are being tried for use in osteosclerosis.
They inhibit osteoclastic activity w i t h o u t affecting Contraindications for surgery
bone deposition Only hearing ear (Absolute CI)
C o m m o n bi sphosphonates used are Occupation
Main side effect - Gl symptoms like nausea and diarrhea Athletes, Working ~
(iii) Hearing Aids: Most patients w i t h otosclerosis have a normal Divers in noisy
cochlear function with excellent speech discrimination and Frequent surroundings
are therefore g o o d hearing aid candidates Air travelers
Accociated meniere's disease _
Surgical treatment:
As rapid change in middle ear pressure can lead to dislodgement
• Selection of patients for stapes surgery
of prosthesis and perilymph leak.
Air bone gap of at least 15dB (the larger the air bone
Pregnancy, young children
gap, the more there is t o gain by surgical intervention
Otitis externa/Otitis media
- Hearing threshold should be < 30 dB or worse.
Tympanic membrane perforation
- Rinnie Negative (both for 256 and 512 Hz.)
Inner ear malformation/exostosis _
Speech discrimination score > 6 0 % .
Medically unfit
- It is also indicated in patients w i t h profound hearing
Active/malignant otosclerosis (It is an indication for fluoride
loss but w i t h g o o d speech discrimination, score, so as
therapy)
t o enable t h e m t o use a hearing aid)

the TOC
Here the fixed otosclerotic stapes is removed and a prosthe-
sis inserted between the incus and oval window. Prosthesis Most important complication ofstapes surgery -hearing loss so second
can be of t e f l o n , stainless steel, platinum or titatinium.
0
operation is considered 6 months after surgery
Disadvantage-associated with high incidence of perilymph
leak and SNHL.
• New treatment of choice is stapedotomy : Here a hole is
0
Patients who refuse surgery or are unfit for surgery can
made in centre of footplate of stapes and a teflon prosthesis hearing aid.
inserted between incus and foot plate. — ^^mlmmmm^.. 1 1 1 1 1 1 1 1 J

• i
310|^ SECTION V Ear

QUESTIONS

Otospongiosis is inherited as: [Al 95] 14. Feature in otosclerosis includes: [AP2003]
a. Autosomal dominant b. Autosomal recessive a. Sounds not heard in noisy environment
c. X-linked dominant d. X-linked recessive b. Normal tympanum
True about otosclerosis: [PGIJune 03] c. More common in males
a. 5 0 % have family history d. Malleus is most commonly effected
b. Males are affected twice than female 15 Carharts notch in audiogram is deepest frequency of:
c. More c o m m o n in Negro's and African's [AI03;TN03]
d. Deafness occurs in 20 - 30 yrs but less in before 10 yrs and a. 0.5 KHz b. 2 KHz
after 40 yrs
c. 4 KHz d. 8 KHz
e. Pregnancy has bearing on it 16. Carhart's notch in audiometery is seen in: [MAHE 05]
Common age for otosclerosis is: [UP-06]
a. Ocular discontinuity b. Haemotympanum
a. 5 - 10 yrs b. 1 0 - 2 0 yrs
c. Otomycosis d. Otosclerosis
c. 2 0 - 3 0 yrs d. 3 0 - 4 5 yrs
17. Acoustic dip occurs at: [TN95]
Commonest site of otosclerosis is: [Corned 07]
a. 2000 Hz b. 4000 Hz
a. Round w i n d o w b. Oval window
c. 500 Hz d. 1500 Hz
c. Utricle d. Ossicles
18. Lady has B/L hearing loss since 4 years which worsened
The part most commonly involved in Otosclerosis is:
during pregnancy. Type of impedance audiometry graph
[PGIJune 99/Rohtak 98/UP-08]
will be: [AIIMS May 07]
a. Oval w i n d o w
b. Round w i n d o w a. Ad b. As
c. Tympanic membranes <«rH-=^&!bfl6bwo^oia;^
d. Malleus 19. All are true about otosclerosis except:
e. Ossicles [PGI June 06, June 05]
Most common site for the initiation of otosclerosis is: a. Increased incidence in female
[Karn. 06] b. Sensorineural deafness
a. Foot plate of stapes b. Margins of stapes c. Irreversible loss of hearing
c. Fissula antefenestrum d. Fissula postfenestrum d. Carhart's notch at 2000 Hz
Otospongiosis causes: [Al 96] e. Family history positive
a. U/L conductive deafness 20. Characteristic feature of Otosclerosis are all except:
b. B/L conductive deafness [AIIMS June 97]
c. U/L Sensorineural deafness a. Conductive deafness b. Positive Rinne's test
d. B/L sensorineural deafness c. Paracusis wiliisi d. Mobile eardrum
Paracusis willisii is feature of: 21. A 30- year old woman with family history of hearing
(MHPGMCET2002, JIPMER 2000 March, MH 2005) loss from her mother's side developed hearing problem
a. Tympanosclerosis b. Otosclerosis d u r i n g p r e g n a n c y . Hearing loss is b i l a t e r a l , slowly
c. Meniere's disease d. Presbycusis progressive, Pure tone audiometry bone conduction
A patient hears better in Noise. The diagnosis is: hearing loss with an apparent bone conduction hearing
[Karn. 95] loss at 2000 Hz. What is the most likely diagnosis?
a. Hyperacusis b. Hypoacusis [AIIMS May 06]
c. Presbycusis d. Paracusis a. Otosclerosis
10. In otosclerosis tinnitus is due to: [Bihar 2005] b. Acoustic neuroma
a. Cochlear otosclerosis t i Increased vascularity in lesion c. Otitis media with effusion
c. Conductive deafness d. All of the above d. Sigmoid sinus thrombosis
In majority o f t h e cases with otosclerosis the tympanic
11. 22. Medication which may prevent rapid progress of cochlear
membrane is: [Kerala 94] otosclerosis is: [Karn. 94]
a. Normal b. Flamingo pink a. Steroids b. Antibiotics
c. Blue d. Yellow c. Fluorides d. Vitamins
12. Schwartz sign seen i n : [MAHE05,PGI-98] 23. All are true statements regarding use of sodium fluoride
a. Glomus Jugulare b. Otosclerosis in the treatment of otosclerosis except: [AI2011]
c. Meniere's diseases d. Acoustic neuroma a. It inhibits osteoblastic activity
13. Gelle's test is for: [Bihar 2006] b. Used in active phase of otosclerosis when Schwartz sign is
a. Otosclerosis b. NIHL positive
c. Sensorineural deafness d. None c. Has proteolytic activity (bone enzymes)
d. Contraindicated in chronic nephritis
CHAPTER 24 Otosclerosis

24. A 31 y e a r old female patient c o m p l a i n s of bilateral a. Teflon biston •

impairment of hearing for the 5 year. On examination, b. Grommet


tympanic membrane is normal and audiogram shows a c. Total ossiculear replacement
bilateral conductive deafness. Impedance audiometry d. All of the above
shows As type of curve and acoustic reflexes are absent.
All constitute part of treatment, except:
a. Hearing aid b. Stapedectomy
NEET PATTERN QUESTIONS
c. Sodium fluoride d. Gentamicin
27. Schwarte sign seen in: [NEETPattern]
25. Following operations are done in case of otosclerosis:
a. Glomus jugulare b. Otosclerosis
[PCI Dec. 03]
c. Meniere's diseases d. Acoustic neuroma
a. Stapedectomy
28. A p u r e t o n e a u d i o g r a m w i t h a d i p a t 2 0 0 0 Hz is
b. Fenestration
characteristic of: [NEETPattern]
c. Stapedotomy
a. Presbyacusis
d. Sacculotomy
b. Ototoxicity
e. Mastoidectomy
c. Otosclerosis
26. In otosclerosis during stapes surgery prosthesis used d. Nose induced hearing loss
is: [UP 06] •

EXPLANATIONS AND REFERENCES

1. Ans. is a i.e. Autosomal dominant Ref. Dhingra 5th/edpg 97,6th/ed p 86


The exact etiology of otosclerosis is not known. In otosclerosis family history/heredity plays an important role. About 5 0 % of patients
of otosclerosis have a positive family history.
• It is an automosal dominant c o n d i t i o n 0

• Shows incomplete penetrance and variable expression


0 0

Also Remember: It may be associated w i t h Vander Hoeve syndrome —: ' '


osteogenesis imperfecia
Bluescelera / \ Otosclerosis

2. Ans. is a, d and e i.e. 5 0 % have family history; Deafness occurs in 20-30 years but less in before 10 years and after 4 0 years;
and Pregnancy has bearing on it
3. Ans. is c i.e. 20 to 30 yrs Ref. Dhingra 5th/ed pg 97,6th/edp 86
• 5 0 % of patients o f otosclerosis have positive family history.
• Females are more c o m m o n l y affected than males. (Note unless and until the Question says in India always mark female> male
as the correct option)
• Whites are affected more than negroes.
• Age = Most c o m m o n between 20-30 years (Ans. 3) and is rare before 10 and after 40 years.
• Deafness is increased by pregnancy, menopause, trauma and major operations.
• Viruses like measles virus have also been associated w i t h it.
4. Ans. is b i.e. Oval window
5. Ans. is a i.e. Oval window
6. Ans. is c i.e. Fissula antefenestrum. Ref. Dhingra 5th/ed pg 97-98,6th/edp 86-87

• Most c o m m o n type o f otosclerosis Stapedial otosclerosis


• Most c o m m o n site of otosclerosis Fissula ante fenestram (i.e. just in front of oval w i n d o w )
• Most c o m m o n site for stapedial otosclerosis Fissula ante fenestram (i.e. just in front of oval window)
• Most c o m m o n site for cochlear otosclerosis Round w i n d o w

7. Ans. is b i.e. Bilateral conductive deafness


8. Ans. is b i.e. Otosclerosis
9. Ans. is d i.e. Paracusis
10. Ans. is a i.e. Cochlear Otosclerosis Ref. Dhingra 5th/d, p 98,6th/ed p 87; Current Otolaryngology 3th/ed p 690
312|^ SECTION V Ear

S y m p t o m s of Otosclerosis

Hearing loss - It is the presenting symptom. Hearing loss is painless and has insidious onset. It is bilateral conductive type and
usually starts in twenties.
Paracusis willisi - Patient hears better in noisy than quiet surroundings

Tinnitus - More in cochlear otosclerosis i.e. it indicates sensorineural degeneration

Vertigo - Uncommon

Speech - Monotonous, well modulated soft speech

Paracusis Patient hears better in noise. Seen in otosclerosis.


Presbycusis SNHL associated with aging. Manifests at 65 years of age. (it is physiological)
Hyperacusis Sensation of discomfort/pain on exposure to loud noises. Seen in injury to nerve to stapedius.
Diplacusis Patient hears same tone as of different pitches in either ear (distortion of sound). Seen in Meniere's disease

EXTRA EDGE


Paracusis: Scotts Brown 7th/ed vol-3 pg-3596

Paracusis refers t o auditory dysfunction, in which the perception of volume, pitch, timbre or other quality of sound may be
altered.
• In majority o f cases, paracusis are attributed t o abnormalities at the auditory periphery (as in otosclerosis) However, they have
also been reported in CNS lesions including temporal lobe (This is because 'Timbre' of a sound is perceived by well defined
regions o f posterior Heschl's gyrus and superior temporal sulcus extending into the circular insular sulcus, of both left and right
hemisphere)
11. Ans. is a i.e. Normal

12. Ans. is b i.e. Otosclerosis Ref. Dhingra Sth/ed pg 98,6th/ed p 87; Current Otolaryngology 2nd/ed pg 674,3rd/ed p 690

In O t o s c l e r o s i s o n O t o s c o p y
• Tympanic membrane is normal in appearance mostly, middle ear space is well pneumatized and malleus moves w i t h pneumatic
otoscopy (i.e. mobility is normal)
• Sometimes a reddish hue / Flammingo pink may be seen o n t h e p r o m o n t o r y and oval w i n d o w niche o w i n g t o the prominent
vascularity associated w i t h an otospongiotic focus. This is k/a Schwartz sign.
13. Ans. is a i.e. Otosclerosis Ref. Dhingra Sth/ed pg 27,6th/edp 22
Gelle's Test

This test was earlier done t o confirm the presence of otospongiosis. In this test, BC (bone conduction) is tested and at the same time
Siegle's speculum compresses t h e air in the meatus. In normal individuals hearing is reduced after this; i.e. Gelles test is positive;
b u t in stapes fixation, sound is not affected, i.e Gelles test is negative.

Basis ofthe Test

In normal individuals In case of otosclerosis


t in air pressure in ear canal by siegel's speculum t air pressure in ear canal by seigels speculum
U
Push the tympanic membrane and ofsicles inward Push the tympanic membrane
u u
t Intralabyrinthine pressure But ossicles are fixed
Hence this t e d pressure is not transmitted further
Immobility of basilar membrane v
u Hence no 4- in hearing (i.e. test is negative)
I hearing
(i.e. test is positive)

Gelles test will also be negative in case of ossicular discontinuity


CHAPTER 7:4 Otosclerosis

14. Ans. is b i.e. Normal tympanum


Ref. Dhingra 5th/ed pg 97-98,6th/ed p 87; Current Otolaryngology 2nd/ed pg 673,674,3rd/ed p 689,90
Already explained
15. Ans. is b i.e. 2 kHz

16. Ans. is d i.e. Otosclerosis Ref. Dhingra 5th/ed p 98,6th/ed p 87; Scott's Brown 7th/ed vol-3 pg 3461-3462

Carharts notch
• Bone conduction is normal in otosclerosis.
• In some cases there is a dip in bone conduction curve which is m a x i m u m at 2000 Hz / 2 KHZ called as Carharts notch.
• Carharts notch is seen only in bone conduction curve
• It disappears after successful stapedectomy/stapedotomy
-

EXTRA EDGE
• The reason w h y it disappears after successful surgery is that w h e n the skull is vibrated by bone: conduction sound, the sound
is detected by the cochlea via 3 routes
• Route (a) - is by direct vibration w i t h i n the skull
• Route (b) is by vibration of the ossicular chain which is suspended w i t h i n the skull
• Route (c) - is by vibrations emanating into the external auditory canal as sound and being heard by the normal air-conduction
route
• I n a conduction type of hearing loss (as in otosclerosis) the latter t w o routes are deficient but regained by successful reconstruction
surgery. Hence bone conduction thresholds improve following surgery.

ALSO KNOW
• Dip in noise induced hearing loss is seen at 4 KHZ.
• In noise induced hearing loss - Dip is seen in both air and bone conduction curves.
. Trough shaped audiogram is seen in congenital SNHL.
• Flat audiogram w i t h moderate t o severes SNHL is characteristic of presbycusis.
17. Ans. is b i.e. 4000 Hz Ref. Dhingra Sth/ed pg 40,6th/edp 35; Tuli Ist/ed p 115
Acoustic dip is t h e dip seen in pure tone audiometry due t o noise trauma, which is seen typically at 4 kHz i.e. 4000 Hz.
18. Ans. is b i.e. As curve Ref. Dhingra 5th/ed pg 97,98,99,6th/ed p 87,88 Current Otolaryngology 2nd/edpg 677,3rd/edp 691
Lady presenting w i t h hearing loss

Bilateral in nature

+

Which worsens during pregnancy


Leaves no confusion - for otosclerosis being the diagnosis.
In otosclerosis - impedance audiometry shows As type of curve.

In the early disease since middle ear aeration is not affected patient shows Type A curve)
19. Ans. is b and c i.e. Sensorineural deafness; and Irreversible loss of hearing
Ref. Dhingra 5th/edpg 97-99,6th/edp 88-89; Current Otolaryngology 2nd/ed pg 673-674,3rd/edp689,690-91
• In otosclerosis - 5 0 % cases have positive family history.
• Females are affected more than males
• Bilateral conductive deafness seen in otosclerosis is not irreversible as it can be successfully treated by stapedectomy /
Stapedotomy
• Senserineural hearing loss occurs w h e n later in the course of time osteosclerotic focus reaches the cochlear endosteum but
actually most c o m m o n hearing loss seen is conductive t y p e . 0

• Cahartz notch is seen in bone conduction curve at 2000 Hz.


20. Ans. is b i.e. Positive rinne's test Ref. Dhingra 5th/ed pg 98-99,6th/ed p 87-88; Current Otolaryngology 2nd/ed pg 675-677
Otosclerosis causes conductive deafness, (i.e. o p t i o n a is correct)
ii
T u n i n g Fork Tests in Otosclerosis
• Rinnes test - negative (i.e. option b is incorrect)

... , . . . . . . , .,
• Webers test - lateralised t o ear w i t h greater conductive loss
314^ SECTION V Ear

• Absolute bone conduction - normal (It is decreased in cochlear otosclerosis)


• Pure tone audiometry -shows loss of air conduction more for lower frequency.
- Bone conduction is normal. In some cases there is a dip in bone conduction maximum at 2000 Hz (called as Caharts notch)
• Acoustic reflex - shows a biphasic peak or it is absent
• Speech audiometry - Normal discrimination score
• Impedance audiometry - As type of curve

Also Remember: • Tympanic membrane is normal and mobile in 9 0 % cases, (i.e. option d is correct).
• Schwartz sign - Flammingo cases pink colour of tympanic membrane is seen in 10% cases. It indicates active
focus w i t h increased vascularity.
• Stapes footplate - Shows a rice grain / biscuit type appearance
• Blue mantles are seen histopathologically.

21. Ans. is a i.e. Otosclerosis Ref. Dhingra 4th/ed p 86-87,6th/ed p 86-87,5th/edpg 97-98-99

30 years female

Bilateral slowly progressive hearing loss

Positive family history


Loss apparent during pregnancy
+
Cahart's notch at 2000 Hz

nAll
i l these
Li i leave
iv^ci v no
III/ d oUuUbIt about
U U ci u\J u I otosclerosis
U L U J U t i UJIJ being
w c i n y the
u i t diagnosis.
u i u y n w j u .

22. Ans. is c i.e. Fluorides


23. Ans. is a i.e. It inhibits osteoblastic activity
Ref. Current Otolaryngology 2nd/ed pg 677,3rd/ed p 693; Tuli 1 st/edpg81 and 82, Otosclerosis and

stapectomy, Diagnosis, Management and treatment by Casscocck 1 st/ed p61,62
The most useful medication which prevents rapid progression of cochlear otosclerosis is sodium fluoride

| SODIUM FLUORIDE THERAPY

Mechanism of Action

• It reduces osteoclastic bone resorption and increases osteoblastic bone formation, which promote recalcification and reduce bone
remodelling in actively expanding osteolytic lesion.
• It also inhibits proteolytic enzymes that are cytotoxic to cochlea and lead t o SNHL (Hence specially useful in cochlear otosclerosis).
"Fluoride therapy has been found to significantly arrest the progression of SNHL in the low and high frequencies"
- Current Otolaryngology 2nd/ed pg 678
"Sodium fluoride therapy has a role in helping maturity of active focus to arrest cochlear loss" - Tuli Ist/ed p 82
• Dose: Initially 50 m g daily followed by 25 mg daily for maintenance
• Duration of treatment = 1 - 2 years
• Indications for Sodium Fluoride Therapy:
- Patients w i t h progressive sensorineural deafness disproportionate w i t h age (whose audiometric pattern indicates the
possibility of cochlear-otosclerosis)
- Patients w i t h radiological evidence o f a demineralized focus in the cochlear capsule (demonstration of spongiotic changes
in the cochlear capsule by popytomography).
- Patients with positive Schwartz sign (indicates activity of otosclerotic focus).
- Patients w h o have otosclerosis and are diagnosed t o prevent w i t h secondary hydrops.
• Contraindications for Sodium Fluoride Therapy:
- Patien ts with chronic nephritis with nitrogen retention
- Patients w i t h chronic rheumatoid arthritis
- Pregnant or lactating w o m e n
- In children before skeletal g r o w t h has been achieved.
- Patients w i t h skeletal fluorosis
- Patients allergic t o fluoride.

-
CHAPTER 24 Otosclerosis

• Response is evidenced by:


- Reduced tinnitus
- Reduced dizziness
- Fading of Schwartz sign
- Radiological signs o f recalcification
• Side effects
- M/C Gastrointestinal side effects like nausea and v o m i t i n g
- It can lead t o skeletal fluorosis so X-ray spine and long bones should be done routinely
24. Ans. is d i.e. Gentamcin Ref. Dhingra 5th/ed p 99; Current Otolaryngology 2nd/ed pg 678-679
Bilateral conductive hearing loss 1

Normal tympanic membrane - Suggest otosclerosis as the diagnosis


+
As t y p e of impedance auditometry curve
Gentamicin is used t o treat Meniere's disease. Rest all options are managements for otosclerosis
25. Ans. is a, b and c i.e. Stapedectomy; Fenestration and stapedotomy
Ref: Scotts brown 7th/ed vol-3 pg-3468 onwards; Current olotaryngology 2/e pg-678-680
Role of surgery in a case of otosclerosis

Surgery forms t h e mainstay of management in a case of otosclerosis

Surgical Options
Stapedectomy / stapedotomy Lemperts fenestration procedure Stapes mobilization
I i
(Surgery of choice) Fenestration ofthe lateral semicircular It is done in those cases only in which there is
canal is done. It is reserved for cases where foot partial ankylosis of footplate of stapes although
plate cannot be mobilized during stapedectomy reankylosis tends t o develop a although
(Outdatednowadays) reankylosis tends to develop a

For mobilization procedure - a prerequisite is (a) lack of ankylosis at posterior stapediovestibular joint (b) otosclerosis limited to fissula ante fenestram

26. Ans. is a i.e. Teflon piston Ref: Current otolaryngology 2/e pg-679, Tuli 1/ed pg-82, Scotts; Brown 7th/ed vol-3 pg-3479
The currently used prosthesis in otosclerosis surgery are:

• Teflon (M/C used)



• Stainless steel
• Platinum - All are MRI compatible
• Gold
• Titanium
• The prosthesis is placed between the long process of incus and foot plate of stapes
27. Ans. is b i.e. Otospongiosis Ref. Dhingra 6th/edp 87
Schwartz sign is seen in otosclerosis, (already discussed)
28. Ans. is c i.e. Otosclerosis Ref. Dhingra 6th/ed p 87
Already explained •


CHAPTER •

Facial Nerve and its Lesions


-

| FACIAL NERVE the mastoid t i p forms and elongates during childhood, the
facial nerve assumes a more medial and protected position.
It is the nerve of second brachial arch. • In unilateral UMN (upper motor neuron) lesions of facial nerve,
It is a mixed nerve and has both Motor and sensory components. upper part of face is spared due t o B/L cortial representation
unlike. Lower m o t o r neuron lesion, where b o t h upper and
Motor component Sensory component lower faces are involved. Also there is a lack of emotional facial
• Supplies the muscles of • Secretomotor to submandibular, movements in UMN lesions.
facial expression (except sublingual, salivary and lacrimal
levator palpebral superioris) glands Carries taste fibres from Course
and muscles of the 2nd the anterior 2/3rd ofthe tongue • Intracranial part: From pons t o internal a u t i d o r y meatus
pharyngeal arch and palate General somatic (15-20 mm)
sensations from the retroauricular • Intrameatal Part/Labyrinthine segment: Part present in the
skin internal auditory meatus (8-10 mm).
• I n t r a t e m p o r a l p a r t : From i n t e r n a l a u d i t o r y m e a t u s t o
stylomastoid foramen. It has 3 subparts:
| ALSO KNOW • Extracranial part: From stylomastoid foramen t o its peripheral
branches.
At b i r t h facial nerve is located just beneath t h e skin near
• At t h e stylomastoid f o r a m e n , t h e facial nerve passes i n t o
t h e mastoid t i p as it emerges f r o m the temporal bone and is parotid gland as a single trunk and then divides into peripheral
vulnerable t o the post auricular incision in a young child. As branches.

Labyrinthine segment / first segment Intratemporal segment


(3 - 5 mm) Horizontal segment ( 1 0 - 1 2 mm) Vertical segment ( 9 - 1 6 mm)
Facial nerve pierces meatal foramen to enter From geniculate ganglion to oval window - From oval w i n d o w t o stylomastoid
labyrinthine segment This portion of facial N is 10 mm long foramen
It extends from internal auditory meatus to Landmarks for the nerve at this segment
the geniculate ganglion include the Cochleariform process which
Shortest and thinnest segment, therefore gives rise to tensor tympani and the 'cog'
any infection of this segment can lead to a small bony prominence projecting from
the roof of epitympanum.
temporary/ permanent paralysis of nerve.
Dehiscence's are more commonly seen
Hence this is the site of lesion in Bell's palsy.
in this segment
Since the very 1 st segment of facial nerve
is the site of pathogenesis all intratemporal
functions of facial nerve are impaired in bells
palsy.
No branches arise from this part.
CHAPTER 25 Facial Nerve and its Lesions J 317

Branches of Facial Nerve

In the Fallopian canal/l ntratemporal branches At its exit from stylomastoid foramen Communicating branches
Note: From the lateral end ofthe internal auditory
canal to its exit out the stylomastoid foramen,
the nerve travels ~3 cms within the fallopian tube.

I. Greater superficial petrosal nerve Posteriour auricular nerve: Terminal branches:


(It arises from geniculate ganglion) Supplies posterior auricular I. Temporal (innervate eye brows and
• It joins with deep petrosal nerve to form muscle, occipital belly of allows for voluntary raising of eye
vidian nerve (nerve of pterygoid canal) which
0
occipitofrontalis along with brows.
supplies: muscular branches to stylohyoid II. Zygomatic (innervates
Lacrimal gland and posterior belly of digastric orbicularis occuli muscles and is
Nasal gland critical for proper eye closure)
Palate gland III. Buccal (Innervate buccinator
- Pharynx gland and orbicularis oris allowing for
II. Nerves to stapedius - supplies the stapedius proper mouth closure)
muscle. Its injury leads to hyperacusis. IV. Mandibular (innervates platysma)
III. Chorda Tympani V. CervicalAII terminal
(It arises from the vertical /descending segment branches supply the muscle
ofthe facial nerve, 4-6 mm above the of face and neck.
sylomastoid foramen. As it arises from the
facial nerve it makes a 30° angle and delineates
a triangular space k/a facial recess.)
• It is the terminal branch of nerves intermedius.
• Enters the tympanic cavity through the
posterior canaliculus and exits through the
petrotympanic fissure (Canal of Huguier).
• Carries taste fibres from the Anterior 2/3
ofthe tongue and also supplies secreomotor
fibres to submandibular and sublingual glands.

Electrodiagnostic Tests to Predict Prognosis in


Facial Palsy - Loss of taste sensation from Anterior 2/3 of tongue: Due
to chorda tympani.
Electrophysiogical testing: Includes electroneuronography,
- Paralysis of muscle of facial expression: Due to terminal
maximal stimulation test, and electromyography.
(peripheral) branches.
These assess t h e p e r c e n t a g e o f n e r v e fibers t h a t have Now we can make out the site of injury:
undergone degeneration along w i t h signs of recovery. This - All the 5 symptoms (i to v) are present: Injury is at or
is diagnostic as well as prognostic, e.g. in Bell's palsy there is proximal to geniculate ganglion (as all the branches of facial
m a x i m u m degeneration w i t h i n the 1st 10 days after which nerve are involved).
there is recovery. Hence, if degeneration persists beyond 10 - There is no loss of lacrimation (greater superficial
days, Bell's palsy is unlikely and it carries a poor prognosis. petrosal nerve is spared) but symptoms (ii to v are
Hence, electrophysiological testing should be done in those seen): Injury is distal to second genu but proximal to origin of
cases o f suspected Bell's palsy not responding t o steroids. chorda tympani, i.e, Injury is between second genu and mid
This also predicts t h e feasibility of surgical decompression of portion of vertical segment.
facial nerve. - Only (vth) symptom is present: Injury is distal to the origin
of chorda tympani, which may be at the level of stylomastoid
foramen.

{Site o f I n j u r y o f F a c i a l N e r v e
P r e s e n t a t i o n of Facial Nerve Paralysis
We have read about the branches of facial nerve and their site
of origin. So we can easily make out the site of injury from Facial nerve paralysis produces following manifestations:
the symptoms of the patient. First see the major symptoms • Weakness o f t h e muscle of thefacial expression and eye closure,
of facial nerve palsy. which results in:
- Loss of lamination: Due to involvement of greater Absence of nasolabial fold
i.
superficial petrosal nerve. Epiphora
ii.
- Loss of stapedial reflex: Due to involvement of nerve to

iii. Wide palpable fissure


stapedius. Voluntary eye closure may not be possible and can pro
iv.
- Lack of salivation: Due to chordatymapani /
duce damage t o the conjunctiva and cornea.

-
318^ SECTION V Ear

v. Loss of wrinkles of forehead Was t h o u g h t t o be idiopathic, but there are recent evidences
vi. The face sags and is drawn across to the opposite side on indicating Herpes simplex virus as the causative agent.
smiling H/O viral prodromal symptoms
Rapidly progresses w i t h i n 1st 10 days put complete recovery
vii. Drooping of angle of m o u t h
is a rule.
T y p e s of Facial L e s i o n Facial muscles on one side are paralysed.
• a. Inability t o close eye.
A. Central or Upper Motor Neuron (UMN) Facial Paralysis b. On attempting t o close eye, eye ball turns up and out-Bells
• It causes paralysis of only the lower half of face on contralateral phenomenon.
side Ipsilateral loss of salivation and lacrimation.
• Forehead muscles are retained due t o bilateral innervation o f
Hyperacusis is present.
frontalis muscle.
Taste may be affected.
B. Peripheral or Lower Motor Neuron (LMN) Facial Paralysis Ear and other CNS functions are normal.
• All muscles ofthe face ate i n v o l v e d on t h e side o f lesion Recurrences both ipsilateral and contralateral occur in u p t o
(Ipsilateral side) 1 2 % patients.
• Site of lesion may be:
i. Supratemporal: Lesion is proximal t o the bony canal, Treatment
which may be:
Conservative:
a. At the level of nucleus: There is associated VI nerve • Steroids: Prednisolone (1 mg/kg/days x 10 days and t h e n
involvement taper for next 5 days)
b. At the cerebellopontine angle:There is associated
• Acyclovir: Adults: 200-400 m g five times/ day
vestibular and auditory defects and other cranial
• Care of the eye
nerve involvement Vth, IXth, Xth, Xlth.
• Physiotherapy
ii. Intratemporal: Lesion is in the bony canal, f r o m internal
• Vitamin B1, B6and B12 combinations.
acoustic meatus t o stylomastoid foramen.
• The side can be localized by topographic tests: Surgery (Nerve decompression): Done if medical therapy fails and
there is no recovery in 8-12 weeks.
Topographic Tests for Intratemporal Lesion

Schirmer's test (for lacrimation): Decreased lacrimation when


lesion is at or proximal to geniculate ganglion. Prerequisites for Surgery
Stapedial reflex: Lost if lesion is proximal to the nerve to stapedius. BAD syndrome:
Taste test: Impaired taste when lesion is proximal to chorda > Lack of Bell's phenomenon
tympani. > Corneal anesthesia
Submandibular salivary flow test: Impaired when lesion is Dry eyes
proximal to chorda tympani.

iii. Infratemporal: Lesion is outside the temporal bone in | HERPES ZOSTER OTICUS/RAMSAY HUNT S Y N D R O M E
the parotid area. Only the motor functions of nerve are
affected. • Reactivation of d o r m a n t herpes zoster virus t h e geniculate
Test for identifying whether the patient has upper motor neuron ganglion of facial nerve and spiral and vestibular ganglion
(UMN) or lower motor neuron (LMN) Vlllth nerve.
In a LMN lesion the patient cannot wrinkle their forehead, • It is characterized by vesicles around the external ear canal,
i.e. the final c o m m o n pathway t o the muscle is destroyed. pinna, and soft palate sensorineural hearing loss and vertigo
Lesion m u s t either in t h e pons, or outside brainstem due t o involvement of Vlllth nerve along w i t h facial palsy. This
(posterior fossa, bony canal, middle ear or outside skull). is called as Ramsay Hunt syndrome.
In an UMN lesion, the upper facial muscles are spared be- • In comparison t o Bell's palsy, progression begins by 11th t o
cause of alterntive pathways in the brainstem, i.e., t h e 14th day b u t prognosis is poor. Recovery is seen only in 4 0 %
patient can wrinkle their forehead (unless there is bilateral of patients.
lesion) and the sagging o f t h e face seen w i t h LMN palsies • Treatment is acyclovir 800 m g 5 times/day.
is not as prominent.
-

| TEMPORAL BONE FRACTURES


| BELL'S PALSY
8 0 % o f t h e temporal bone fracture is of longitudinal t y p e .
0

Commonset cause o f acute onset LMN facial palsy. 10 - 3 0 % are transverse fractures .0

Sudden in onset 40 - 5 0 % o f t h e transverse fractures cause facial nerve injury . 0

It is unilateral Facial nerve involvement is rare w i t h longitudinal fracture . 0


CHAPTER 25 Facial Nerve and its Lesions J 31

IMPORTANT CLINICAL CONCEPTS FOR NEET

- 7. Heerfordts syndrome : There is bilateral parotid enlargement


0

1. Total length of facial nerve is 60-70 m m . w i t h uveitis and transient facial palsy due t o sarcoidosis.
• Intracranial segment 15-20 m m
8. Bannwarth's syndrome / Lyme's disease: There is rash, fever,
• Meatal segment 8-10 m m
myalgias, arthralgia, pharyngitis and lymphadenopathy w i t h
• Labyrinthine segment 3-5 m m
facial nerve palsy. It is due to spirochaetes infection.
• Tympanic segment 8-10 m m
9. Genu of Facial Nerve: The sharp turns made by facial nerve is
• Mastoid segment 15-20 m m
called as genu. 1st genu is thickened t o f o r m the geniculate
• Extratemporal segment 15-20 m m
ganglion, surface landmark being processus cochleariformis.
2. Vidian nerve is f o r m e d by greater superficial petrosal nerve
0 Surface landmarkfor 2nd genu is horizontal semicircular canal.
j o i n i n g deep petrosal nerve (sympathetic) for supplying the Tympanomastoid suture line.is the l a n d m a r k f o r descending
lacrimal glands, mucous glands of nose, palate and pharynx . 0
p o r t i o n . These landmarks are used in mastoid surgery. 1st
genu is the commonest site of injury t o facial nerve in trauma,
3. M/C t u m o r of facial nerve is Schwannoma
while 2nd genu is the commonest site of injury to facial neve
4. Schirmer's test, taste sensation or salivation test give informa-
in mastoid surgery.
tion a b o u t the probable site of lesion in facial nerve injury.
5. Crocodile tears while eating are due t o misdirection of se-
0
10. Facial nerve palsy at stylomastoid foramen causes deviation of
cretomotor impulses meant for salivary gland and are treated angle of m o u t h t o opposite side (due t o paralyses o f muscles
by tympanic neurectomy. of facial expression) and absence of corneal reflex.
6. Melkersson's syndrome is characterised by recurrent facial nerve
0 11. Causes of B/L facial palsy: Guillian-Barre Syndrome, infectious
palsy, swelling of lips and furrowing of tongue. mononucleosis, amyloidosis, Sarcoidosis, Skull trauma, acute
porphyria, lyme's disease and botulism.


320 [_ SECTION V Ear

QUESTIONS

BRANCHES AND SITE OF LESION CLINICAL FEATURES


First branch of the facial nerve is: [UP. 2004] 10. Right upper motor neuron lesion of facial nerve causes:
a. Greater petrosal nerve b. Lesser petrosal nerve [AIIMS 95]
c. Chorda-tympani nerve d. Nerve to the stapedius a. Loss of taste sensation in right anterior part tongue
2. All t h e following muscle are innervated by t h e facial b. Loss of corneal reflex right side
nerve except: [AIIMS May 03] c. Loss of wrinkling of forehead left side
a. Occipito-frontalis b. Anterior belly of digastric d. Paralysis of lower facial muscles left side
• c. Risorius d. Procerus 11. Which one of the following statements is correct in facial
3. Lacrimation is affected w h e n facial nerve injury is at: paralysis? [MP 2009]
[Aim a. The naso labial fold is obliterated on same side
a. Geniculate ganglion b. The naso labial fold is obliterated on opposite side
b. In semicirculalr canal c. The face deviates to the same side
c. At sphenopalatine gangila d. The face deviates to the opposite side
d. At foramen spinosum 12. Which test can detect facial nerve palsy occurring d u e
A patient presents with hyperacusis, loss of lacrimation to lesion at the outlet of stylomastoid: [AIIMS Nov. 93]
and loss of taste sensation in the anterior 2/3rd of the a. Deviation of angle of mouth towards opposite side
t o n g u e . O e d e m a extends u p to w h i c h level of facial b. Loss of taste sensation in anterior 2/3 of tongue
nerve: [2001] c. Loss of sensation over right cheek
a. Vertical part d. Deviation of tongue towards opposite side
b. Vertical part beyond nerve to stapedius 13. Crocodile tears is due to: (Delhi2005)
c. Vertical part and beyond nerve to stapedius a. Cross innervation of facial nerve fibers
d. Proximal t o geniculate ganglion b. Cross innervation of trigeminal nerve fibers
Dryness of eye is caused by injury to facial nerve at: c. Improper regeneration of trigeminal nerve
[AI96] d. Improper regeneration of facial nerve
a. Chorda tympani
b. Cerebellopontine angle
c. Tympanic canal
CAUSES OF FACIAL PALSY
d. Geniculate ganglion 14. Iatrogenic traumatic facial nerve palsy is most commonly
Hyperacusis in Bell's palsy is due to the paralysis o f t h e caused during:
follwing muscle: [AIIMSMay06] a. Myringoplasty b. Stapedectomy
a. Tensor tympani b. Levator palatii c. Mastoidectomy d. Ossiculoplasty
c. Tensor veli palatii d. Stapedius 15. Which fracture ofthe petrous bone will cause facial nerve
Intratemporal lesion of chorda tympani nerve results palsy: [AI07]
in: [AIIMS Dec. 94] a. Longitudinal fractures
a. Loss of taste sensations from papilla of tongue b. Transverse fractures
b. Loss of taste sensations from anterior 2/3rd of tongue c. Mastoid
c. Loss of taste sensations from posterior 1/3rd of tongue d. Facial nerve injury is always complete
d. Loss of secretomotor fibres to the submandibular salivary 16. Facial nerve palsy is seen in this condition: [JIPMER03]
gland a. Seborrheic otitis externa b. Otomycosis
Dryness of mouth with facial nerve injury - site of lesson c. Malignant otitis externa d. Cerebellar abscess
is at: [UP 2008] 17. Which part of the facial nerve is commonly exposed
a. Chorda tympani N through natural dehiscence in the fallopian canal?
b. Cerebellopontine angle [2005]
c. Geniculate ganglion a. Horizontal part
d. Concussion ofTympanic membrane b. Upper half of the vertical part
Facial nerve palsy at sternomastoid canal can cause: c. Lower half of the vertical part
[AIIMS June 99] d. Labyrimthine part
a. Loss of corneal reflex at side of lesion 18. Most common cause of facial palsy:
b. Loss of corneal taste sensation anterior 2/3 of ipsilateral a. Postoperative b. Trauma
tongue c. Ramsay Hunt syndrome d. Bells palsy
c. Loss of lacrimation at side of lesion 19. Most common cause of lower motor neuron facial palsy
d. Hyperacusis is: [MP 2004]
CHAPTER 25 Facial Nerve and its Lesions J 321
a. Cholesteatoma b. Physiotherapy
b. Cerebellopontine angle tumours c. •!• Steroids dose
c. Bell's palsy d. Electrophysiological Nerve testing
d. Postoperative (ear surgery) 27. Treatment of choice for mastoid fracture with facial nerve
palsy is: [AIIMs June 99, Sept 96]
a. Nerve decompression
BELLS PALSY b. High dose of steroid
20. Bell's palsy is paralysis of: [Corned 07] c. Sling operation
a. UMN V nerve b. UMN VII nerve d. Repair the fracture and wait and watch
c. LMN V nerve d. LMN VII nerve 28. A patient presents with facial nerve palsy following head
21. Which of the following is not true about Bell's palsy? trauma with fracture of the mastoid: best intervention
[Delhi 2008] here is: [AI01]
Acute onset a. Immediate decompression
Always recurrent • •
b. Wait and watch
Spontaneous remission c. Facial sling
Increased predisposition in Diabetes Mellitus d. Steroids
22. Which one o f t h e following statements truly represents
Bell's paralysis: [AIIMS May 05; Al 04]
a. Hemiparesis and contralateral facial nerve paralysis
RAMSAY HUNT SYNDROME
b. Combined paralysis o f t h e facial, trigeminal, and abducens 29. A m a n presents with vesicles over external acoustic
nerves meatus with ipsilateral facial palsy of LMN type. T h e
c. Idiopathic ipsilateral paralysis o f t h e facial nerve cause is [AP2005]
d.' Facial nerve paralysis with a dry eye a. Herpes zoster b. Herpes simpex virus-l
23. All of the following are seen in bell's palsy except: c. Varicella d. None
[SGPGI05] 30. Ramsay hunt syndrome is caused by: [PGI Dec. 98]
Ipsilateral-facial palsy a. H. simplex b. H. Zoster
Ipsilateral-loss of taste sensation c. Influenza d. Adenovirus
Hyperacusis 31. Ramsay hunt syndrome all are true except: [SGPGI05]
Ipsilateral ptosis a. VII Nerve is involved
24. True about lower motor neuron palsy of Vllth nerve: b. Facial muscle are involved
[PGI Nov. 05] c. Facial vesicle is seen
Other motor cranial nerves also involves d. Herpes zoster is etiologic agent
Melkersson's syndrome cause recurrent paralysis 32 All of the following are true for Ramsay hunt syndrome,
Eye protection done except: [AI02]
Prognosis can be predicted by serial electrical studies a. It has viral etiology
Bell's palsy is commonest cause b. Involves Vllth nerve
2 5 . Bell's palsy patient comes on d a y 3. Treatment given c. May involve Vlllth nerve
would be: [AIIMsNov09,May2010] d. Results of spontaneous recovery are excellent
a. Intratympanic steroids b. Oral steroids + vitamin B 33. True about Ramsay-hunt syndrome except: [UP 2000]
c. Oral steroids + Acyclovir d. Vitamin B Vasodilator a. Involves VII nerve
26. A case of bells palsy on steroids, shows no improvement b. May involves VIII nerve
after two weeks. Next step in manangement is: [MP2000] c. Surgical removal gives excellent prognosis
a. Vasodilators and ACTH d. Causative agent is virus

EXPLANATIONS AND REFERENCES

1. Ans. is a i.e. greater petrosal nerve Ref. Dhingara 5th/ed pg102,6th/ed p 90; Current Olotaryngelogy2nd/edpg 836
• Greater superficial petrosal nerve: It is the first branch and arises from geniculate ganglion (i.e. first genu). It jouns the deep petrosal
nerve t o f o r m vidian nerve (nerve t o pterygoid canal) and carries secretomotor fibres t o the lacrimal gland, nasal gland, Palate
gland and pharyngeal gland.

2. Ans. is b i.e. Anterior belly of digastric Ref.BDCVol.HI4th/edp 139-140

Facial Nerve Supplies


Forget = Facial muscles except levator palpebrae Superioris (Which is supplied by 3rd nerve).
322 J_ SECTION V Ear

Pediatric (Pd) = Posterior belly of Digastric


Surgery = Stapedius
Always = Auricular muscles
O p t For - Occipto Frontalis i
Plastic - Platysma
Surgery - Stylohyoid
Mnemonic: Forget Pediatric Surgery Always Opt for Plastic Surgery.

Anterior belly of digastric is supplied by nerve to mylohyoid.


Procerus and Risorius are muscles of face.

3. Ans. is a i.e. Geniculate ganglion Ref. Dhingra Sth/ed pg 102,6th/edp 90-91


For lacrimation greater superficial petrosal nerve which is a branch of facial nerve is responsible
It arises f r o m the geniculate ganglion, /any lesion occurring at the level of geniculate ganglion will injure this branch and will lead
t o dryness o f eyes.
. ~ ~ ......... a i n s a s K i i s i y l y > J i-'T«sUi-
ALSO KNOW
For locating the site of injury of facial nerve:
• First see t h e major symptoms of facial nerve palsy:
i. Loss of lacrimation: Due t o involvement of greater superficial petrosal nerve.
ii. Loss of stapedial reflex: Due t o involvement of nerve t o stapedius.
iii. Lack of salivatiion: Due t o chorataympani.
iv. Loss of taste sensation from Anterior 2/5 of tongue: Due t o chordatympani.
v. Paralysis of muscle of facial expression: Due to terminal (peripheral) branches.
Now you can make out the site of injury:
- All the 5 symptoms (i to v) are present: Injury is at or proximal t o geniculate ganglion (as all the branches) of facial nerve
are involved)
- There is no loss of lacrimation (greater petrosal and nerve to stapedius are spread) but symptoms (ii) to (v) occur:
Injury is distal t o geniculate ganglion but proximal t o or at the level of second genu f r o m where the nerve to stapedius arises.
- Only symptoms (iii) to (v) are present (greater petrosal and nerve to stapedius are spread): Injury distal t o second genu
but proximal t o origin of chorda t y m p a n i , i.e., Injury is between Second genu and mid portion of vertical segment.
- Only (vth) symptoms is present: Injury is distal t o the origin of chorda tympani, which may be at the level of stylomastoid
foramen.
4. Ans. is d i.e. Proximal to geniculate ganglion Ref. Current Otolaryngology 2nd/edpg 836-838,3rd/edp 865-67
In the question patient is presenting w i t h
i. hyperacusis which means nerve t o stapedius is involved which arises f r o m the vertical / descending part of facial nerve.
ii. loss o f lacrimation - i.e. greater superficial petrosal Nerve which arises f r o m geniculate ganglion is involved.
iii. Loss of taste sensation in anterior 2/3 of tongue - i.e. chorda tympani nerve which arises f r o m vertical/descending part of
facial nerve is involved.

Remember:
• Any lesion will lead t o paralysis of all Nerves distal t o it and will spare proximal nerves
• Hence - we will have t o look for the most proximal lesion which in this case is geniculate ganglion
• So lesion is either at or proximal t o geniculate ganglion

5. Ans. is d i.e. Geniculate ganglion


6. Ans. is d i.e. Stapedius Ref. Tuli Ist/edp 87
Hyperacusis (Phonophobia) occurs due to undue sensitivity to loud sounds.
• Stapedius muscle dampens excessive vibrations o f t h e stapes caused by high pitched sounds in order t o protect the internal ear.
• If this protective reflex is not elicited it indicates stapedius paralysis and results in hyperacusis.

Level of lesion of facial nerve palsy


Schimers test of lacrimation {led on paralysed side) Geniculate ganglion
Hyperacusis/Phonophobia (undue sensitivity to loud sounds) Nerve to Stapedius involved
4- ed taste sensation Chorda tympani nerve involved
Salivation test (4- ed salivation on paralysed side) Terminal branches-Nerve to submandibular gland involved
CHAPTER 25 Facial Nerve and its Lesions J 323
7. Ans. is d i.e. Loss of secretomotor fibres to the submandibular salivary gland
8. A n s is a i.e. Chorda tympani nervde. Ref. BDC Vol. Ill 3rd/edp 113,127
-

Chorda Tympani Nerve Carries

X 1
Preganglionic secretomotor fibres to Taste fibres from anterior
submandibular ganglion 2/3rd of tongue

Responsible for salivary secretion from •

submandibular & sublingual gland

So, Ideally a lesion of chorda tympani should impair both these functions but - sensations f r o m ant 2/3rd of tongue are not impaired
as an alternate pathway passing t h r o u g h the nerve of pterygoid canal t o the otic ganglion exists (which doesnotpass through middle
ear) which is preserved in lesions of chorda t y m p a n i .
Any lesion of chorda t y m p a n i thus leads t o dryness of m o u t h
9. Ans. is a i.e. Loss of Corneal reflex at the side of lesion Ref. Dhingra 5th/ed p 102,6th/edp 90-91

Course of Facial Nerve



Below stylomastoid formen, facial nerve gives following branches: Posterior auricular branch, muscular branches (stylohyoid
and posterior belly of diagastric) and terminal (peripheral) branches.
Lesion at sternomastoid foramen
i. Will spare:

- Greater superficial petrosal nerve -> Lacrimation present.
- Nerve t o stapedius —> Normal stapedial reflex and no hyperacusis. 1

- Chorda t y m p a n i —> Normal salivation and taste sensation in anterior 2/3 of tongue.
ii Will involve:
Terminal (peripheral) branches -H> Paralysis of muscles of facial expression. Corneal reflex will also be lost because efferent
fibres o f corneal reflex are derived f r o m peripheral branches of facial nerve (it is a LMN type lesion).

Remember:
Corneal Reflex: Afficient:Trigeminal nerve
Efferent: Peripheral branches of facial nerve '

10. Ans. is d i.e. Paralysis of lower facial muscles at left side


Ref. Macleods clinical examination 12th/ed p 282, Dhingra 5th/d, p 105-106
It is a General Rule that: • UMN lesion cause • Contralateral paresis
• LMN lesion cause • Ipsilateral paresis
So, right upper m o t o r neuron lesion of facial nerve will lead t o paresis / deformity of left side. (Ruling o u t options V a n d "o")

In R i g h t U M N P a l s y

• Facial muscles of opposite side (left side) will be affected


• Upper facial (forehead) muscles will be spared
• So patient will have paralysis of lower facial muscles on contralateral (left) side.
11. Ans. is a i.e. nasolabial fold is obliterated on the same side Ref. Dhingra 5th/edpg 106
Always remember: Lower m o t o r neuron type of facial paralysis is much more c o m m o n than upper motor neuron type. If any
question is asked on facial paralysis unless and until it mentions 'UMN type, all paralysis should be taken as LMN type.
LMN type facial paralysis causes ipsilateral facial paralysis.
Following features are seen in Facial nerve paralysis:
• Loss o f wrinkles (on ipsilateral side in LMN type paralysis)
• Wide palpebral fissure (on ipsilateral side)
. Epiphora (on ipsilateral side)
• Absence o f nasolabial fold (on ipsilateral side)
Drooping o f angle of m o u t h (on ipsilateral side) bns
ii

In facial nerve paralysis - the peripheral branches supplying the facial muscles will be paralysed which will lead to, paralysis of facial muscles on the
ipsilateral side and angle of mouth will be deviated to opposite side (Not the whole face so option d is incorrect)
324 [_ SECTION V Ear

12. Ans. is a i.e. Deviation of angle of mouth towards opposite side Ref. Dhingra 5th/ed pg 102,6th/ed p 95
• Lesion occuring at the outlet of stylomastoid foramen means LMN palsy so face sags and is drawn across t o opposite side.
Chorda t y m p a n i nerve is spared at this level hence taste sensation over anterior 2/3 of tongue preserved
13. Ans. is d i.e. improper regeneration of facial nerve
Ref. Dhingra5th/edpg 110, Current Ololaryngology2nd/edpg839,3rd/edp870
Crocodile tears (gustatory lacrimation) There is unilateral lacrimation w i t h mastication
• It is due t o faulty regeneration of parasympathetic fibres which normally travel t h r o u g h chorda t y m p a n i b u t are misdirected
towards greater superficial petrosal nerve and instead of going t o salivary glands reach the lacrimal glands.
This results in unilateral lacrimation w i t h mastication
• Treatment - Sectioning the greater superficial petrosal nerve or tympanic neurectomy

ALSO KNOW

• Frey's syndrome (gustatory sweating) - There is sweating and flushing of skin over the parotid area during mastication.

Remember:
Irreversible axonal injury and aberrant patterns of regeneration are more c o m m o n f r o m grade III degree of sunderland classifica-
tion of facial nerve d e g e n e r a t i o n
0
«

14. Ans. is c i.e. Mastoidectomy Ref. Logan Turner 10th/ed p 359


"All ear operations run the risk of facial nerve damage, particularly if the nerve is exposed. In particular a mastoidectomy has a high risk
because a sharp cutting rotating burr is used in close proximity to the nerve."

O t h e r Operations w h e r e Facial Nerve m a y be D a m a g e d

- Stapedectomy •

- Removal of acoustic neuroma


15. Ans. is b i.e. Transverse fracture Ref. Dhingra Sth/ed p 108,6th/ed p 97
Fracture of petrous temporal bone can be longitudnal or transverse. Facial palsy is seen more often w i t h transverse fracture . 0

Longitudnal Transvers
Frequency Most common (80%) Less common (20%)
Bleeding from ear Present Absent (as tympanic membrane is intact)
Csf otorrhoea Present Absent
Structures injured Tegmen, ossicles and TympanicMembrane Labyrinth or CN VIII
Hearing loss Conductive SNHL
Facial paralysis Less common, (10% cases) More common (40-50%)
Onset of paralysis Delayed onset paralysis Immediate onset of paralisis
Part of facial nerve injured Distal to geniculate ganglion Proximal to geniculate ganglion
Vertigo Less often More often

In these cases it is important t o k n o w whether paralysis was of immediate or delayed onset -


Immediate onset paralysis is treated conservatively.
Delayed onset paralysis - requires surgery in the f o r m of decompression, reanastomosis of cut ends or cable nerve grafts.
16. Ans. is c i.e. Malignant otitis externa Ref. Dhingra 5th/edp 58,6th/ed p 52
Facial paralysis is seen in malignant otitis externa as discussed in previous chapters.
17. Ans. is a i.e. Horizontal part Ref. Scotts Brown 7th/ed ed vol-3 pg 3888; Current Otolaryngology 2nd/ed pg 837
Explanation:Tbe Horizonal/tympanic part of facial nerve
• Is most susceptible t o injury during surgery.
• Maximum bone dehiscence occur in this part adjacent t o oval window.
18. Ans. is d i.e. Bells Palsy
19. Ans. is c i.e. Bells Palsy Ref. Scotts Brown 7th/ed vol 3 pg 3891; Harrison 17th/ed pg 2585
"The commonest cause of facial palsy in adults is Bells palsy" -Scotts Brown 7th/ed vol-3 pg 3891
It is unilateral and intranuclear type of palsy. It is also the M/C cause of LMN facial palsy - [Harrison 17th/edpg 3891)
20. Ans. is d i.e. LMN VII nerve
2 1 . Ans. is b i.e. Always recurrent
Ref. Dhingra 5th/edpg 105, 106,6th/edp 95; Current Otolaryngology 2nd/epg 847,854,855;
Scotts Brown 7th/ed vol-3 pg 3885; Harrison 17th/ed pg 2584
CHAPTER 25 Facial Nerve and its Lesions

Bells Palsy
• It is the commonest cause of facial palsy in adults
• Lower motor neuron type paralysis"
• It is idiopathic in nature
0

• Bells palsy manifests as an acute, unilateral paresis or paralysis ofthe face.


0

• The onset and evolution are rapid - typically less than 48 hours."
• The incidence reaches a maximum between the ages 15 and 45 years. It has a predominance in women younger than 20years and a
slight predominance in men older than 40 years, although it is more or less equal."
• Recurrence rate of Bells palsy is 4.5-15% (i.e. It is not always recurrent)"
• Familial incidence = 4.1%
• Bells palsy is uncommon in pregnancy, however the prognosis is significantly worse in pregnant women with Bells palsy than among
non pregnant women with palsy.
• Several authors have also demonstrated, a correlation between diabetes mellitus and Bells palsy in developing countries.
• Infectious causes have also been implicated in the origin of Bells palsy - viz:
- HSV land HSV 2, human herpes
- Varicella zoster virus
x
- Influenza B
- Adenovirus
- Coxsackievirus
- Epstein Barr virus
Recently it has been demonstrated that an inactivated intranasal influenza vaccine increased the risk of Bells palsy.
• Normal function is usually regained within 3 months in about 2/3 rd of patients
• No further recovery is expected after a period of 6 months.
• Majority ofthe patients with Bell's palsy recover completely
22. Ans. is c i.e. Idiopathic, ipsilateral paralysis of the facial nerve
Ref. Dhingra 5th/ed pg 105,6th/edp 95; Harrison 17th/edp 2585; Scott's Brown 7th/ed vol-3 pg 3883,3885
• Bells paralysis is a LMN type of facial nerve palsy of unknown etiology i.e. idiopathic nature.
• Lower m o t o r neuron type of palsy causes ipsilateral paralysis therefore, bells palsy causes ipsilateral facial paralysis.
• Other neurological.examinations are normal in Bells palsy
23. Ans. is d i.e. Ipsilateral ptosis Ref. Harrison.l7th/ed p 2585; Dhingra 5th/ed pg 105-106,6th/ed p 95
Bell's palsy is an acute onset lower motor neuron type of palsy - their will be Ipsilateral loss of :
• Taste sensation, lacrimation and salivation
• Facial paralysis
• Noise intolerance (hyperacusis)
• Eye balls will t u r n up and out (Bells phenomenon) on attempting to close eyes but ptosis will n o t be seen.

In Bells palsy - Facial paralysis is usually preceeded by pain behind the ear.

24. Ans. is a, b, c, d and e i.e. Other cranial nerves also involved, Melkersson's syndrome cause recurrent paralysis. Eye protection
done. Prognosis can be predicted by serial electrical studies. Bell's palsy is commonest cause
Ref. Dhingra Sth/ed p 105-06,6th/edp 95,96; BDC 4th/ed vol lll/p 54; Current Otolaryngology 2nd/edp 847,3rd/edp. 876
• M o s t c o m m o n cause of lower motor neuron (LMN) type of facial palsy is Bell's palsy.
• Melkersson's syndrome consists of a triad of: (i) Facial paralysis, (ii) Swelling of lips, (iii) Fissured tongue, Paralysis may be
recurrent.
• As patient is unable to close»the eye, eye protection is required to protect cornea and conjunctiva.
• The prognosis in acute facial palsy can be accurately determined by serial electrical testing. The response t o electrical tests have
been found to be most useful in the first 5 days after the onset.
• As far as option 'a' is concerned-other cranial nerves also involved-current otolaryngology 3rd/ed p 876 says -
• "There may also be subtle but frequent associated dysfunction of cranial nerves V, VIII, IX and X in association with Bells palsy."
i.e option a is correct.
25. Ans. is c i.e. Oral steroids + Acyclovir
Ref. Current Olotaryngology 2nd/ed pg 856; Scott's Brown 7th/ed vol-3 pg 3886,3rd/edp 884-887

Management of Bells Palsy -

I 1
f ~l
A. Medical Management B. Physical management j C. Surgical management
326|_ SECTION V Ear

Medical Management -

I. Steroid therapy:
• Oral prednisolone has been used extensively t o treat patients w i t h Bells palsy.
• Proof of efficacy is however controversial.
• Steroids are considered useful because they have an anti inflammatory response.
• Because the cost of therapy is less and it has low risk of side effects, predinosolone is commonly started at the initial visit
• Initiation of therapy during the first 24 hours of symptom confers a higher likelihood of recovery
II. Antiviral therapy:
• It represents a newer adjunct in treating acute facial palsy of viral origin (Both Bells palsy and Ramsay hunt syndrome)
• Oral acyclovir is the DOC
• Some studies have shown that patients who receive prednisolone plus oral acyclovir have a higher recovery rate and reduced rates
of synkinesis in comparison to those who receive prednisolone alone.
• Based on above evidence most surgeons advocate combination of steroids and antiviral drugs.
The usual recommended regime is predisolone Img/kg/day for five days followed by a ten day taper and oral acyclovir (200-400 mg 5
times daily) for ten days.

Physical M a n a g e m e n t

Includes:
• Electrical stimulation: It is done t o maintain membrane conductivity and reduce muscle atrophy
• It is generally used in patients left w i t h partial defects
• Eye care: The cornea is vulnerable t o drying and foreign body irritation in acute facial palsy due to orbicularis oculi dysfunction.
So measures conferring corneal protection are recommended. Like:
- Artificial tears drops at daytime
- Ocular o i n t m e n t at night
- Use o f sunglasses etc
In long standing cases: Reducing the area of exposed cornea by implanting a gold weight in the upper lid (tarsorapphy) is done.

Surgical Management
eftqt»V\to 3 0 t - c S H a 9 \ ! u ? arorunG ;cS£\ qbs
x A: i s r c 11 5fl iteaHf tetstelteqt,»J b ii .spA ,££
Nerve decompression
• Principle used behind it - Axonal ischemia can be reduced by the decompression of nerve segments presumed t o be inflammed
and entrapped.
• Decompression o f t h e facial nerve is done in cases w h o have a poor prognosis for complete recovery w i t h medical treatment
alone.
26. Ans. is d i.e. Electro physiological nerve testing Ref. Current Otolaryngology 2nd/edpg 858,842,3rd/edp. 887,870,872
In a patient w h o has had no improvement in steroids after 2 weeks of use will not benefit f r o m an increase in dose of steroid
Also vasodilators and ACTH have no role in management of Bells palsy
Hence they are also ruled out.
So now we are left w i t h 2 options viz¬
i. Electrophysiological nerve testing
ii. Surgical decompression
Firstly Remember: Electro physiological nerve testing is not the same as Electrical stimulation

Facial Nerve Testing Involves 2 Types of Tests

I ' 1
Topodignostic tests (like schemiers test) Electrophysiological tests
Which help in determining the level of facial nerve injury Which help in determining the extent of injury & in turn help to
by testing peripheral nerve function. identify the subset of facial palsy patients who will not obtain
satisfactory, spontaneous recovery

Nerve decompression - Surgical management of acute facial nerve palsy is based on the principle that axonal ischemia can be
reduced by decompression of nerve segments presumed t o be inflamed and entrapped. Nerve decompression is n o t done in all
cases of acute facial palsy.
Prerequisite for Nerve decompression (Read very carefully)
To identify those patients w h o may benefit f r o m nerve decompression, electro physiological testing should be done prior t o it
(Current otolaryngology 3/e, p 887)

r
CHAPTER 25 Facial Nerve and its Lesions J 327
The test done is - Evoked electro myograpy (EEMG). Surgical treatment is offered when evoked response amplitudes are 1 0 % (or
less) o f t h e normal side.
So n o w after understanding all this lets see the question once again -
It says - a case o f bells palsy on steroids, shows no improvement after 2 weeks, next step in management w o u l d be -
Next step w o u l d obviously be electrophysiolgical testing for t w o reasons:
1. Bells palsy as a rule recovers after 10 days and responds after sterioid, the diagnosis has t o reviewed t o rule out other causes
like herpes zoster oticus (which can be indicated by the pattern of degeneration on electro physiological nerve testing)
2. If electro physiological testing predicts poor prognosis for recovery. It is an indication for nerve decomppression.
27 Ans. is a i.e. Nerve decompression Ref. Dhingra 5th/edp 107

Traumatic Injury to Facial nerve

Traumatic Injury t o Facial nerve Delayed onset paralysis


i i
Nerve decompression - Reanastomosis High dose steroids
of cut end/cable nerve graft
• '

ALSO KNOW

As a general rule management of facial nerve paralysis following trauma is generally deffered until the patient is both medically
and neurologically stable.
28. Ans. is a i.e. Immediate decompression Ref. Scotts Brown 7th/ed vol- 3 pg 3888
In case of Temporal bone trauma
"In case of acute complete paralysis, surgical exploration is warranted ifENOG shows > 90% denervation within 6 days ofthe onset of
parlysis and the patient is neurologically stable"
29. Ans. is a i.e. Herpes zoster
30. Ans is b i.e. H. zoster
Ref. Dhingra Sth/ed pg 107,6th/edp 96; Scotts Brown 7th/ed vol-3 pg 3886; Current Otolaryngology 2nd/ed pg 847,849
Ramsay Hunt syndrome/Herpes zoster oticus is a lower motor neuron type of facial palsy due t o varicella zoster (herpes zoster). It
is characterised by vesicles around the external canal, pinna and soft palate, SNHL and vertigo due t o involvements of Vlllth nerve
along w i t h facial nerve palsy.
31. Ans. is c i.e. Facial vesicle is seen
Ref. Dhingra Sth/ed pg 107,6th/ed p 96; Current Otolaryngology 2nd/ed pg 849,3rd/ed p 878
• Vessicles in Ramsay hunt syndrome are seen in the preauricular skin, the skin of ear canal the soft palate and not on facial skin
• All other options are correct and explained in the perceeding text.
32. Ans. is d i.e. Results of spontaneous recovery are excellent
33. Ans. is c i.e. Surgical removal gives excellent prognosis Ref. Scotts Brown 7th/ed vol-3 pg 241
We have already discussed Ramsay hunt syndrome is ipsilateral facial nerve palsy accompanied by an erythematous vesicular
rash on the ear or in m o u t h (soft palate).
• It is caused by Herpes zoster virus
• It may involve other cranial nerves viz-V, VIII, IX and X also, (current otolaryngology, 3rd/ed p 878)
• Ramsay H u n t syndrome can be differented f r o m Bells by characteristic cutaneous changes and a higher incidence o f
cochleosaccular dysfunction due to involvement of VIII nerve.
• The prognosis of Ramsay Hunt is worse than Bells palsy. Persistent weakness is observed in 30-50% of patients and only 1 0 %
recover completely after complete loss of function w i t h o u t treatment.
• Treatment recommended - is steroids (oral prednisolone) for 5 days followed by a ten day taper combined within or oral acyclovir
• It is seen that Ramsay hunt syndrome patients treated w i t h prednisone and acyclovir w i t h i n 3 days of onset showed statistically
significant improvement.
• Surgical decompression is not indicated in Ramsay hunt syndrome.

-
CHAPTER
Lesion of Cerebellopontine - •

itffigle and Acoustic Neuroma

CN VIII and VII CN III, IV and V It is benign, encapsulated, extremely slow growing tumor.
Mostcommon site of acoustic neuroma:
Inferior vestibular nuclei 0
> superior vestibular nuclei °>
Cochlear nuclei. (rare)
0

Bilateral vestibular schwanoma is diagnostic of Neurofi-


bromatosis 2.
It originates in the Schwann cells of the inferior or superior
vestibular nerves at the transition zone (Obersteiner Redlich
Zone) o f t h e central and peripheral myelin.

Clinical Features

CN IX, X and XI • Age most common in 40-60years , 0


(occurs in 20-30years of age
when the tumour is found in association with\
Neurofibromatosis type 2)
• Both sexes are affected equally . 0

Fig. 26.1: Cerebellopontine Angle


• Tumour is radioresistant . 0

• In 90% cases it is Unilateral, may be bilateral in Von


| ANATOMY OF CEREBELLOPONTINE ANGLE (CPA)
_ Recklinghausen disease/Neurofibromatosis. 0

• It is a triangular area b o u n d e d a n t e r o l a t e r a l ^ by petrous


Clinical S y m p t o m s
temporal bone, medially by pons and brainstem, posteriorly
by cerebellum and flocculus. • Earliest s y m p t o m : Cochleovestibular symptoms (deafness,
• Contents o f t h e angle are: Anterior Inferior cerebellar artery 0 tinnitus).
and VII, VIII cranial n e r v e0 • Most common symptom: Progressive unilateral sensorineural
• Immediately superior t o is the V cranial nerve and III, IV, VI are (retrocochlear) hearing loss (present in 95% patients) often 0

further up. accompanied by tinnitus (Present in 65% patients).


• Inferiorly lies IX, X, XI cranial nerve: Thus in lesions of CPA all
these nerves can be involved. .
3
• There is marked difficulty in understanding speech out of
cteristic
proportion to the pure tone hearing loss. This is characteristic
3 of acoustic neuroma . 0

Lesions of CP angle
• M/C Acoustic neuroma = 80% • May also present with sudden hearing loss, (in 20% cases)
• Meningoma = 10% • True vertigo is seldom seen °.
• Congenital cholesteatoma = 5%
• Others = 5% Signs
J • Ear: Otoscopic findings: normal
• Cranial nerve:
| VESTIBULAR SCHWANNOMA/ACOUSTIC NEUROMA
V nerve: • Earliest nerve to be involved (when t u m o r
0

• It is the most common intracranial schwannoma. grows 2.5 cms in size) Corneal reflex is i m -
• Constitutes 80% of all cerebellopontine angle t u m o r s and 0
paired (Earliest sign ) 0

1 0 % of all brain tumors • Motor functions are rarely affected


CHAPTER 26 Lesion of Cerebellopontine Angle and Acoustic Neuroma J 329
VII nerve: • Sensory fibers are affected first 0
BERA - Delay of > 0.2 msec in Wave V between the 2 sides.
• Hitselberger's sign: Loss of sensation in (Important)
the postero-superior aspect o f t h e external • Acoustic reflex: Shows stapedial reflex decay.
auditory canal . 0
• Vestibulartest: Caloric test usually show diminished or absent
Inability to bury eyelashes on the affected side.
response b u t may be normal if t u m o u r is very small.
0

• Motor fibres are affected later. Remember


• Investigation of choice = Gadolinium enhanced MRI = 1 0 0 %
- T h o u g h Vllth nerve is involved there is
diagnostic yield
never facial nerve palsy (If CP angle t u m o r
presents w i t h facial nerve palsy always rule
Treatment:
out more aggressive tumors like meningoma
rather than Acoustic neuroma which is a slow • Surgery is the treatment of choice . 0

growing tumors)
IX, X Nerves: Palatal, pharyngeal and laryngeal paralysis Surgical Approach Indication
• Eyes: Nystagmus may be seen Hearing preservation
• Features o f brainstem, cerebellar i n v o l v e m e n t and raised
• Retro sigmoidal approach - Patient has good hearing and
intracranial tension are also present viz-Past pointing, positive
tumor is large > 3 cm size
romberg test, ataxia, exaggerated tendon reflexes . 0

• Middle cranial fossa approach - Patient has good hearing and


• In terminal stages, there is:
tumor size is < 1.5 cm (i.e.
Herniation o f cerebellar tonsils
small tumors)
Failure o f t h e vital centers in the brainstem
• Retro labyrinthine - Small CPAtumornotextending
into lateral part of internal
Histopathology
auditory canal
• Histopathological examination shows t w o m o r p h o l o g i c a l
Hearing ablation
tissue patterns.
Translabyrinthine approach - Suitable for tumors < 3 cms but
Antoni A Pattern: It has closely packed cells w i t h small
disadvantage is SNHL
spindle shaped and densely stained nuclei called as
Verocay Body
Hearing rehabilitation following tumour excision:
Antoni B Pattern: It has loose cellular aggregation of
Cochlear i m p l a n t 0

vacuolated pleomorphic cells.


Auditory brainstem implant-In cases of bilateral acoustic
In any particular VS, one type of cellular pattern may predominate
neuroma . 0

or both types can be completely admixed.


S t e r e o t a c t i c r a d i o s u r g e r y / G a m m a k n i f e ( R a d i a t i o n is
Classification of V S A c c o r d i n g t o Size
d e r i v e d w i t h Co-60):
Sterotatic radiotherapy: Concentrates high dose radia-
Intra meatal Tm Extrameatal size 2n millimetres tion on the t u m o r so that its g r o w t h is arrested w i t h o u t
affecting surrounding tissue
Grade 1 Small 1-10 Used in patients w h o refuse surgery 0

Grade II Medium 11-20 Source of radiation = cobalt 60.


Grade III Moderately large 21-30 Adantages:
Grade IV Large 31-40 No morbidity of surgery.
- VII nerve functions preserved
Grade V Giant > 40
Hearing preserved.
Now due to its low morbidity gamma knife surgery or sterotac-
Investigations tic RT is taken as an alternative t o surgery in tumors less than
3 cms in size
• Audiological test: Show features of retrocochlear hearing loss.
• M o d i f i c a t i o n o f Gamma knife is X-knife w h e r e source o f
Recruitment negative 0
radiation is linear accelarator
Speech discrimination score p o o r (speech discrimination
0

score becomes worse at higher speech intensity and this Cyber knife:
phenomenon is k/a roll over phenomenon. • It is a type of - Robotic Surgery where the surgery is done by
computer controlled robotics
330 T SECTION V Ear

QUESTIONS

1. Most common cerebellopontine angle tumour is: a. Facial nerve may be involved unilateral deafness
[Kerala 91] b. Reduced corneal reflex
a. Acoustic neuroma b. Cholesteastoma c. Cerebellar signs
c. Meningioma d. All o f t h e above d. Acute episode of vertigo
2. Cerebellopontine angle tumor poroduces: [PCI 2005] 12. Earliest ocular finding in acoustic neuroma: [PGI 00]
a. Tinnitus b. Deafness a. Diplopia b. Ptosis
c. Absent corneal reflex d. Trigeminal neuralgia c. Loss of corneal sensation d. Papilledema
3. Schwannoma involves the: [Al 99] 13. Vestibular neuroma not correct: [AP2005]
a. Vestibular part of Vlllth nerve Nystagmus
b. Cochlear part of Vlllth nerve High frequency sensorineural deafness
c. Vagus nerve Absence of caloric response
d. Hypoglossal nerve Normal corneal reflex
4. Acoustic neuroma commonly arise from: 14. True about Acoustic neuroma: [PGI June 04]
[Al 11, Al 10, AIIMS Nov. 09] [AIIMS Dec. 98, JandK-05] a. Malignant tumor
a. Superior vestibular nerve b. Inferior vestibular nerve b. Arises form vestibular nerve
c. Cochlear nerve d. Facial nerve c. Upper pole displaces IX, X, XI nerves
5. In acoustic neuroma cranial nerve to be involved earliest d. Lower pole displaces trigeminal cranial nerve
is: [AI07,UP-08] 15. Progressive loss of h e a r i n g , tinnitus a n d a t a x i a are
a. 5 b. 7 commonly seen in a case of: [SGPGI05]
c. 10 d. 9 a. Otitis media b. Cerebral glioma
6. The earliest symptom of acoustic nerve tumor is: c. Acoustic neuroma d. Ependymoma
[Al 95, Delhi -05, Karnatak- 09] 16. Acoustic neuroma of 1 cm diameter, the investigation of
a. Sensorineuran hearing loss choice: [Kerala 97]
b. Tinnitus a. CTScan b. MRI Scan
c. Vertigo c. Plain X-ray Skull d. Air encephalography
d. Otorrhea 17. A patient is suspected to have vestibular schwannoma
7. Earliest sign seen in Acoustic neuroma is: [UPSC 05] the investigation of choice for its diagnosis is: [AIIMS 04]
a. Facial weakness b. Unilateral deafness a. Contrast enhanced CT scan
c. Reduced corneal reflex d. Cerebellar signs b. Gadolinium enhanced MRI
8. Acoustic neuroma causes: [PGIJune 99] c. SPECT
a. Cochlear deafness b. Retrocochlear deafness d. PET scan
c. Conductive deafness d. Any o f t h e above
9. Hitzelberger's sign is seen in: [Al 08]
a. Vestibular schwannoma b. Mastoiditis NEET PATTERN QUESTION
c. Bells palsy d. Cholesteatoma
18. Neurofibromatosis type 2 is associated with:
10. In acoustic neuroma all are seen except: [MP2000]
a. Loss of corneal reflex b. Tinnitus a. B/L acoustic neuroma
b. Cafe-au-lait spots
c. Facial palsy d. Diplopia
c. Chromosome 22
11. In a patient with acoustic neuroma all are seen except:
d. Lisch nodule
[SGPGI07]
e. Posterior subcapsular lenticular cataract
-


-


CHAPTER 26 Lesion of Cerebellopontine Angle and Acoustic Neuroma

EXPLANATIONS AND REFERENCES


1. Ans. is a i.e. acoustic neuroma
Ref. Current Otolaryngology 2nd/ed pg 765; 3rd/edp 791-92; Turner 1 Oth/ed pg- 39; Dhingra 5th/ed p. 124, Dhingra 6th/edp. 112

M/C CP angle tumour is Acoustic neuroma = 80'


2nd M/C CP angle tumor is meningoma > 10%

2. Ans. is a, b, c, d i.e. Tinnitus, Deafness, Absent corneal reflex and d. Trigeminal neuralgia
Ref: current otolaryngology 3/e p.792
The two most common CP angle tumors are:

Acoustic neuromas Meningomas

• M/C symptom = U/L Deafness M/C symptom = U/L Deafness (80%) followed by vertigo (75%) and
• 2nd M/C symptom Tinnitus tinnitus = 60%
• M/C nerve involved = Facial nerve absent corneal reflex is seen In meningiomas
Unlike Acoustic neuroma - Trigeminal neuralgias, facial paresis, lower
cranial nerve deficits and visual disturbances are more common.

Hence all the above features are seen in CP angle tumors.


Ans. is a i.e. Vestibular part of VIII nerve
th
Ref. Logan Turner Wth/edp 339
Ans. is a i.e. Inferior vestibular nerve Ref. Glasscock-Shambaugh, Surgery ofthe Ear, 6th/ed p 644
• Vestibular schwannomas are benign, well circumscribed, encapsulated tumors that arise f r o m Schwann cells o f t h e vestibular
nerve, hence the term vestibular schwannoma or vestibular neurilemmoma.
• Historically, the superior vestibular nerve sheath was thought to be the site of origin, giving rise to nearly two-thirds of
tumors. More recent reviews show the inferior vestibular nerve to be the predominant site of origin for these tumors.
• The nerve or origin is identifiable in 33 t o 7 4 % of cases, and w h e n clearly seen, shows the t u m o r originating f r o m the inferior
vestibular nerve over twice as often and in upto 9 4 % in some reports.
• Rarely the cochlear portion o f t h e eight cranial nerve/facial nerve is the site of schwannoma origin.
Ans. is a i.e. 5 nerve Ref. Dhingra 5th/edpg 124,6th/edp 112
Ans. is a i.e. Sensory neural hearing loss
Ans. is c i.e. Reduced corneal reflex. Ref. Dhingra 5th/ed pg 124; Turner 10th/ed pg 341

Remember:

• Most common nerve f r o m which vestibular schwannoma arises • Inferior vestibular nerve

• Earliest s y m p t o m • Progressive unilateral sensorineural hearing loss often


accompanied by tinnitus


Earliest cranial nerve t o be involved by acoustic neuroma • Vth nerve

• Earliest presentation of Vth nerve involvement/ Decreased corneal sensitivity


Earliest sign of Acoustic neuroma

• Significance of Vth nerve involvement • Implies that t u m o u r is atleast 2.5 cm in size and
occupies
cerebellopontine angle


2nd earliest cranial nerve t o be involved by acoustic neuroma • Facial nerve (VII nerve)

• Earliest presentation of VII nerve involvement Involvement of Sensory fibres leading to hyposthesia
of posterior meatal wall (Hitzelberger sign)

1 N0TE
^^^^^^I^^^^^^M^^S^^^^SffiS3I^^M^^tt^^^^H^^^S^9
Although facial nerve is involved facial nerve palsy is rarely seen
l^ififtTifwlfT*'. i
Ans. is b i.e. Retrocochlear deafness Ref. Tuli Ist/ed pg 114 •

• SNHL can be:


a. Cochlear SNHL - Hair cells are mainly damaged.
b. Retrocochlear SNHL - There is lesion of Vlllth nerve or its central connections.
Acoustic neuroma cause retrocochlear type of SNHL as it damages \ ' nerve.
Meniere's disease causes cochlear deafness
332 T SECTION V Ear

• Important features of Retro Cochlear hearing loss /Acoustic Neuroma:


- Sensorineural hearing loss more marked in high frequencies.
- Poor discrimination score (0-30%).
- Recruitment phenomenon absent and roll over phenomenon present i.e. discrimination score further decreases when loudness is increased
beyond a particular point.
- Short increment sensitivity index (SISl) test will show a score of 0-20% in 70-90% cases.
- Tone decay significant

9. Ans. is a i.e. Vestibular Schwannoma Ref. Dhingra 5th/edpg 124,6th/ed p112


Hitzelberger's sign is hypoaesthesia of posterior meatal wall seen in vestibular Schwanoma /acoustic neuroma due to involvement
of sensory fibres of Vllth nerve.

10. Ans. is c i.e. Facial palsy Ref. Scott's Brown 7th/ed vol-3 pg 3959; Dhingra 5th/ed pg 124-125,6th/ed p 112,113

In A c o u s t i c N e u r o m a
• Loss of corneal reflex is seen - due t o the involvement of Tringeminal nerve
• Tinnitus - due t o pressure on cochlear nerve
• Large tumors can cause diplopia Turner 1 Oth/ed pg 341
As far as facial nerve palsy is concerned - Scott Brown 7th/ed vol-3 pg3931
"Vestibular schwannomas, although inevitably grossly distort the Vllth nerve, very rarely present as a Vllth nerve palsy. If there is a clinical
evidence of a cerebellopontine angle lesion and if the Vllth nerve is involved, alternative pathology is more likely".
Hence although Acoustic neuroma may involve the 7 nerve b u t complete palsy is never seen
11. Ans. is d i.e. acute episode of vertigo Ref. Dhingra 5th/ed pg 124

Lets see Each Option Separately

• Option a - Facial nerve may be involved -


This is correct as we have discussed in previous Questions,, facial nerve may be involved b u t complete palsy doesnot occur
• Option b - Reduced corneal reflex -
This is correct reduced corneal reflex is the first sign of Trigeminal nerve involvement
• Option c - Cerebellar signs - This is correct
• Option d - Acute episodes of vertigo
"Vestibular symptoms seen in acoustic neuroma are imbalance or unsteadiness. True vertigo is seldom seen"
-Dhingra Sth/ed p 124,6th/edp 112
Acute episode of vertigo is a rare presenting feature in acoustic neuroma since it is a slow growing t u m o r so there is adequate
t i m e for compensation.
12. Ans. is c i.e. Loss of corneal sensation Ref. Dhingra 5th/edpg 124; Turner 10th/ed p 341
• Earliest nerve involved by acoustic neuroma - Vth nerve / trigeminal nerve.
• Earliest manifestation of Vth nerve involvement is decreased corneal sensitivity leading t o loss of corneal reflex.
13. Ans. is d i.e. Normal corneal reflex Ref. Dhingra 5th/ed pg 125
As far as the answer is concerned -1 am sure no one has any doubts about it because corneal reflex is absent in acoustic neuroma
But lets focus on o p t i o n c. i.e. Absence of caloric response -
In acoustic neuroma -
Caloric test will show diminished or absent response in 9 6 % patients due to vestibular involvement.
Hence option c i.e. correct

Also Know
-
Criteria of suspicion for Acoustic neuroma (Turner 10th/ed pg 341)
• Unilateral deafness of less than 10 years.
• Sudden deafness w i t h retrocochlear involvement which does not respond t o steroids
• Poor speech discrimination score in relation t o pure tone threshold
• Spontaneous nystagmus w i t h eyes closed on electronystagmography w i t h o u t a history of disequilibrium
• Absence of caloric response in case of normal hearing
• Hearing loss w i t h reduced corneal reflex
• Local P a ' "
•i

If hearing loss is the only symptom and it is of more than 10 years duration, an acoustic neuroma is most unlikely as a tumor which has been growing
for longer than this period because it will give features of other cranial nerve or brainstem involvement also.
CHAPTER 26 Lesion of Cerebellopontine Angle and Acoustic Neuroma J 333

14. Ans. is b i.e. Arises from vestibular nerve Ref. Dhingra 5th/edp 114,5th/edp 134

Explanation

Here Option a i.e. malignant t u m o r is incorrect as acoustic neuroma is a benign tumor.


Option b: It arises f r o m vestibular nerve is correct
Option c: Upper pole displaces IX, X and XI nerve-incorrect, as is evident f r o m the diagram given in the text: Upper pole displaces
III, IV and V nerve whereas lower pole displaces IX, X and XI nerve.
15. Ans. is c i.e. Acoustic neuroma Ref. Dhingra 5/e, p 124
Already explained
16. Ans. is b i.e. MRI scan

17. Ans. is b i.e. Gadolinium enhanced MRI scan Ref: Current Otolaryngology 2/e pg-767, Dhingra 5/ed pg-126

I n v e s t i g a t i o n s t o b e d o n e in C a s e o f A c o u s t i c N e u r o m a
Initial step in evaluation includes an audiology testing w i t h pure tone audiometry, speech discrimination score (S D S), acoustic
reflex threshold and acoustic reflex delay. If these tests suggest a retrocochlear lesion, then imaging o f t h e CPA is performed.

I m a g i n g T e s t s in C P A t u m o r / A c o u s t i c N e u r o m a

1. MRI - MRI with gadolinium contrast is the gold standard f o r t h e diagnosis or exclusion of vestibular Schwannoma
• It also allows for surgical planning
• MRI can detect intracanalicular t u m o r of even a few millimeters
2. CT scan - CT scan can diagnose CPA tumors which are larger than 1.5 cms or have atleast a 5 m m CPA components. It can miss
tumors that are intracanalicular unless there is bony expansion o f t h e internal auditory canal. it
• •

ALSO KNOW

Auditory Brainstem response / BERA:


• In patients w i t h vestibular Schwanoma or retrocochlear lesion - ABR is either fully or partially absent or there is a delay in the
latency of wave V in the affected ear
• CSF examination - Shows increased proteins in acoustic neuroma.

18. Ans. a, b, c, a n d e i.e. B/L acoustic neuroma, cafe au lait spot, chromsome 22 and posterior subcapsular cataract (Ref. Current
Otolaryngology 3/e pg. 8 0 1 , 8 0 2 )
B/L acoustic neuromas are a hallmark of Neurofibromatosis 2
• Neurofibromatosis Type 2 is an autosomal d o m i n a n t highly penetrant condition
• Gene for NF-2 is located on chromosome 22q.
• Patients w i t h NF2 present in second and third decade o f life, rarely after the age of 60.
• M/C symptom/Presenting symptom = Hearing loss
• Skin tumors are present in nearly t w o thirds of patients of NF-2
"Cafe au lait spots, which are a hallmark of NF-1, are also frequently found in patients with NF2. In contrast to patients with NF1, patients
with NF2 invariably have fewer than sixof these hyperpigmented lesions. Juvenile posterior sub capsular lenticular opacties are common
and have been reported in up to 51% of patients with NF2."-Current Otolaryngology 3/e, p 801 -802
So as is clear f r o m above lines-cafe an lait spots and posterior subcapsular lenticular opacity are seen in NF-2 also.


Remember: Diagnostic criteria for NF-2
I. Bilateral Acoustic neuroma
or
II. Family hisory of NF-2 and •
U/L Vestibular schwannoma/acoustic neuroma
or
III. Any two of the following:
Meningioma
Glioma
-

Neurofibroma
Schwannoma I
Posterior subcapsular leticular opacity

CHAPTER
Glomus Tumor and
- Other Tumors of the Ear

GLOMUS TUMOUR S p r e a d of T u m o r

Glomus t u m o r are the m o s t c o m m o n benign tumors of middle
1 Site of Spread Presentation
ear.
Resemble carotid body therefore also k/a chemodectoma • Tympanic membrane - Vascular polyp
Consists of paraganglionic cells derived f r o m neural crest • Labyrinth, petrous, pyramid and - Hearing Loss
(Paragangliomas) mastoid
It usually arises from dome of jugular bulb as glomus jugulare
• Jugularforamen and baseof skull - Cranial nerve palsies VII, VIII,
or f r o m p r o m o n t o r y along the course of tympanic branch of IX IX to XII
cranial nerve (Jacobson's nerve) and along the course of branch
• Eustachian tube - Mass on nasopharynx
X' cranial nerve (Arnold's nerve) as glomus tympanicum.
h

Sometimes it may be multicentric. • Intracranially spreads


Most c o m m o n site in middle ear: hypotympanum • Lung, liver lymph nodes

Features

Slow growing locally invasive, noncapsulated t u m o r w h i c h Note: M/C cranial nerve involved = Facial nerve followed
causes destruction o f t h e bone and facial nerve. by the last four cranial nerves.
Highly vascular-Main Blood supply: ascending pharyngeal artery
Commonly affect middle-aged females (typically in 4 or 5 t h th

Clinical Features
decade of life)
Malignant transformation and metastasis are rare
When tumor is intratympanic:
Some may show endocrine activity: secrete catecholamine (similar
1. Earliest symptoms are deafness (conductive) and tinnitus
to pheochromocytoma).
(pulsatile and of swishing character, synchronous with pulse
Pathologically and can be temporarily stopped by carotid pressure).
2. Otoscopy shows red reflex; rising sun appearance, tym-
They originate f r o m the'chief cell'which contains acetylcholine,
panic membrane appears bluish and bulging.
catecholamine and serotonin
3. Pulsation sign/Browne sign/Blanching sing is positive
• Classic findings are clusters of chief cells k/a Zellballen, w i t h
(when ear canal pressure is raised w i t h Siegel's speculum,
a rich vascular plexus t h r o u g h o u t the entire
t u m o r pulsates vigorously and then blanches; reverse hap-
Turn or. The refore, they are highly vascular and may bleed sub-
pens w i t h release of pressure).
stantially during surgical excision
4. Aquino sign - It is blanching of the mass w i t h manual
Bilateral tumors occur in 1 - 2 % cases
compression of ipsilateral carotid artery.
Can be hereditary also
Also associated w i t h pharamatoses (neurologic disease When tumor present as polyp:
w i t h cutaneous manifestations like von Recklinghausen 1. History of profuse bleeding f r o m the ear either spontane-
neurofibromatosis, sturge-weber syndrome, tuberous ously or on attempts to clear it.
sclerosis and von Hippel-Lindau disease) 2. Dizziness, vertigo, facial paralysis, earache otorrhea.
• Also associated w i t h MEN Type I syndrome. • Audible bruit: Heard by stethoscope over mastoid at all stages.
CHAPTER 27 Glomus Tumor and OtherTumors ofthe Ear J 335
• Some glomus t u m o r secrete catecholamines and produce For carotid artery: Carotid arteriography
headache, sweating flushing, etc. For jugular bulb: Jugular venography
• P a t i e n t may s h o w f e a t u r e s o f c r a n i a l n e r v e IX a n d X, For intracranial extension: Vertebral arteriography
involvement viz. dysphagia or hoarseness.
Treatment
Investigations
Surgery - Microsurgical total tumor removal is the treatment of choice
• Examination under microscope: Pulsatile mass seen. for most patients. Patients w i t h functionally secreting tumors need
t o be alphablocked w i t h phentolamine before and during surgery
t o prevent life threatening hypertension as the alpha adrenergic
• CT scan: investigation of choice. Helps to distinguish glomus hormones are released w i t h t u m o r manipulation.
jugulare from glomus tympanicum with the help of Phelp's sign:
absence of normal crest between the carotid canal and jugular
fossa on lateral tomography, in case of glomus jugulare
Other Conditions Causing Pulsatile Tinnitus
Arterial - Glomus tumor, AV malformation of temporal bone,
H R C T a n d gadolinium enhanced MRI is used t o delineate the aberrant internal carotid artery, cartoid/subclavian atherosclerosis
intracranial extent of tumor. Venous - High jugular bulb, benign ICT.
A combination o f CT scanning and contrast MRI is the imaging
regimen of choice for glomus jugular tumor. Embolization: Is the sole treatment in inoperable patients who
Audiogram will show conductive deafness if the middle ear have received radiation.
space is invaded w i t h tumor. If inner ear is invaded SNHL is seen Preoperative embolization is done t o decrease vascularity of
Angiography: It is necessary when CT scan shows involvement t u m o r before surgery
of j u g u l a r b u l b , c a r o t i d a r t e r y or intracranial e x t e n s i o n . Radiation: is reserved for inoperable lesions, old age and
Following procedures are done: unfit patients.


C o m p l i c a t i o n s : S e e F l o w C h a r t 27.1 •

Flow Chart 27.1: Complication of Glomus t u m o r

Complications of Glomus Tumor

T
r T 1
A- Hearing loss B- Hypoglossal C- Facial nerve palsy D- Ipsilateral E- Jugular foramen syndrome
nerve palsy Homer syndrome
T
Mainly conductive in
nature
causing - ptosis,
miosis and l/L facial Involves CNS T
flushing and IX— dysphagia & aspiration as
SNHL is uncommon sweating sensation to pharynx is decreased
but can occur if the X— It can also cause hoarseness
tumor erodes the due to vocal cord paralysis
dense optic capsule X I —weakness & atrophy of
and invades the inner sternocleidomastoid & trapezius
ear


336 J_ SECTION V Ear

QUESTIONS

1. The usual location of Glomus jugular tumor is: 8. The glomus tumor invasion of jugular bulb is diagnosed
[Delhi 90, UP-03] by [UP 05]
a. Epitympanum b. Hypotympanum a. Carotid angiography b. Vertebral venous venography
c. Mastoidal cell d. Promontory c. X-ray d. Jugular venography
2. Earliest symptom of glomus tumor is: [UP 06] 9. A patient presents w i t h b l e e d i n g from the ear pain
a. Pulsatile tinnitus b. Deafness tinnitus a n d progressive d e a f n e s s . O n e x a m i n a t i o n ,
c. Headache d. Vertigo t h e r e is a red s w e l l i n g b e h i n d t h e intact t y m p a n i c
3. Pulsatile tinnitus in ear is due to: [TN01] membrane which blanches on pressure with pneumatic
a. Malignant otitis media b. Osteoma speculum. Management includes all except:
c. Mastoid reservoirs d. Glomus jugulare tumor [AIIMS Nov. 01]
4. True about Glomus jugulare tumor: [PGI June 04] a. Radiotherapy b. Surgery
a. Most common in male c. Interferons d. Preoperative embolization
b. Arises from non-chromaffin cells 10. Which is the most pulsatile tumor found in external
c. Lymph node metastasis seen auditary meatus which bleeds on touch: [AIIMS 95]
d. Multicentric a. Squamous cell ca of pinna
e. Pulsatile tinnitus and conductive type of hearing loss seen b. Basal cell ca
5. All are true about glomus jugulare tumors except: c. Adenoma
d. Glomus tumor
[UP 03]
11. Mass in ear, on touch bleeding heavily, causes: [DNB 01]
a. Common in female
a. Glomus Jugulare b. Ca mastoid
b. Causes sensory neural deafness
c. Acoustic neuroma d. Angiofibroma
c. It is a disease of infancy
12. Most common bony tumour of middle ear is: [UP 07]
d. It invades labyrinth, petrous pyramid and mastoid
a. Adenocarcinoma b. Squamous cell carcinoma
Brown sign is seen in: [Al 07]
c. Glomous tumor d. Acoustic neuroma
a. Glomus tumor b. Meniere's desease
13. Treatment of middle ear malignancy includes: [Mahe07]
c. Acoustic neuroma d. Otoscleorsis
a. Excision of petrous part of temporal bone
Phelp's sign is seen in: [AllMSMay02]
b. Subcortical excision
a. Glomus jugulare b. Vestibular Schawannoma
c. Modified radical mastoidectomy
c. Maniere's disease d. Neurofibromatosis
d. None

EXPLANATIONS AND REFERENCES

1. Ans. is b i.e. Hypotympanum Ref. Dhingra 5th/ed pg 120,6th/edp 109


Glomus tumor is of 2 types:


Arises from: Arises from
• Dome of jugular bulb Promontory of middle ear
• Hypotympanum
Invades:
• Jugular foramen therefore involves cranial nerves IX to XII and
compresses jugular vein
Clinical features: Clinical features:
• Signs of compression of cranial nerves IX to XII • Aural symptoms sometimes with facial paralysis

Ans. is a i.e. Pulsatile tinnitus Ref. Dhingra 5th/ed pg 120,6th/ed p 109; Current Otolaryngology 2nd/edpg 799,3rd/ed p815
"The two most common presenting symptoms of paraganglioma of temporal bone (Glomus tumor) 0
are conductive hearing loss 0

and pulsatile tinnitus 0


" - Current Otolaryngology 3rd/ed p815
Hearing loss is conductive and slowly progressive Tinnitus is pulsatile and of swishing character , synchronous with pulse ,
0 0 0

and can be temporarily stopped by carotid pressure . 0


CHAPTER 27 Glomus Tumor and Other Tumors ofthe Ear J 337
Thus, both pulsatile tinnitus and deafness are seen in glomus tumor.
According t o Turnerl 0/e, p 214 •

"The earliest s y m p t o m of a glomus t u m o r is pulsatile tinnitus."


3. Ans. is d i.e. Glomus jugulare tumor Ref. Dhingra 5th/edpg120,6th/edp 109
Pulsatile tinnitus - Seen in Glomus t u m o r
Pulsatile otorrhea - Seen in ASOM
4. Ans. is b, d and e i.e Arises from non-chromaffin cells; Multicentric; and Fluctuating tinnitus and conductive type of hearing
loss seen Ref. Dhingra 5th/ed pg120,6th/ed p 109-110; Curren t Otolaryngology 2nd/ed pg794-800,3rd/ed pg 814,815,816

Explanation

• Glomus t u m o r is more c o m m o n in females.


• Glomus t u m o r is also referred t o as chemodectomy or nonchromaffin paraganglion.
• Glomus t u m o r is a benign tumor, therefore lymph node metastats is not present.
• Multicentric tumors are found in 3-10% of sporadic cases and in 25-50% of familial cases.
• Fluctuating (Pulsatile) tinnitus and conductive hearing loss are the earliest symptoms of glomus tumor.
Ans. is c i.e. It is a disease of infancy Ref. Dhingra 5th/edpg 120-121, 6th/edp 109-110
Let us see each o p t i o n separately
Option a - Common in females
It is correct as females are affected five times more than males.
Option b - Causes sensorineural deafness
This is partially correct as glomus t u m o r leads t o mainly conductive type hearing loss. Sensorineural hearing loss is u n c o m m o n but
can occur if the t u m o r erodes the dense otic capsule bone and invades the inner ear.
Option c - It is a disease of infancy
This is incorrect as Glomus t u m o r is seen in middle age (40-50 years)
Option d - It invades labyrinth, petrous pyramid and mastoid.
This is correct

Spread of Glomus Tumor

T T T
It can perforate the It can spread It can invade labyrinth Through Eustachian tube it
It can invade jugular
tympanic membrane intracranially to petrous pyramid can spread to nesopharynx
foramen & base of
posterior and and mastoid skull causing IXth
middle cranial
to XII nerve palsy
fossa

Ans. is a i.e. Glomus tumor Ref. Dhingra 5th/edpg 120,6th/edp 109


Ans is a i.e. Glomus tumor Ref. internet search
Phelp's sign
Rising sun sign are all seen in Glomus t u m o r •

Pulsation sign/brown sign •

Aquino sign

Phelp sign This sign is seen on CT scan


In CT - in case of glomus jugulare t u m o r the normal crest between the carotid canal and jugulare t u m o r is absent whereas it is not
so in case of glomus tympanicum
For details of other sign see the preceding text
Ans. is d i.e. Jugular venography Ref. Dhingra 5th/ed pg. 121,6th/edp 110
MRI - gives soft tissue extent of t u m o r ; Magnetic Resonance Angiography (MRA) shows compression o f t h e carotid artery whereas
magnetic resonance venography shows invasion of jugular bulb by the t u m o r (For more details on imaging techniques used in
case of Glomus t u m o r - see the preceding text)

Remember: Preoperative biopsy is never done in case of glomus t u m o r as it can lead t o bleeding.

338 {_ SECTION V Ear

9. Ans. is c i.e. Interferons Ref. Dhingra Sth/ed pg 121, 6th/ed p 110; Current Otolaryngology 2nd/ed pg 801,802
Patient presenting w i t h progressive deafness, tinnitus and bleeding f r o m ear
+
Red swelling behind the intact tympanic membrane (i.e. rising sun sign)
+
Swelling blanches on pressure w i t h pneumatic speculum (i.e. Brown's sign)
1
Indicate Glomus t u m o r as the diagnosis

Management options for Glomus tumor

Surgery Radiotherapy Embolization J


Microsurgical total tumor removal is the • Does not cure the tumor • Pre-operative embolization after digital
TOC • Reduces the vascularity of tumor and substraction angiography, reduces the
arrests its growth vascularity of the tumor prior to surgery
• Used for • Used as sole treatment in inoperable cases
- Inoperable tumor who have received prior radiotherapy
- Residual tumor
- Recurrences after surgery
- Elderly patients

10. Ans. is None or d i.e. Glomus tumor Ref.Turner lOth/ed p 215; Dhingra 5th/ed p 120,6th/edp 109
It is w o r t h n o t i n g here that t h o u g h the glomus t u m o r is the neoplasm of middle ear, it may perforate the tympanic membrane and
appears as a polypus in the external auditory meatus which bleeds profusely if touched.
11. Ans. is a i.e. Glomus Jugulare Ref. Dhingra Sth/ed pg 120,6th/ed p 109
The answer t o this question is quite obvious as Glomus tumors are highly vascular tumors and bleed on Touch.
12. Ans. is b i.e. Squamous cell carcinoma
13. Ans. is a and c i.e. Excision of petrous part of temporal bone; and Modified radical mastoidectomy
Ref. Dhingra Sth/ed pg 122-123,6th/ed p 110-111
Mostcommon malignant t u m o r of middle ear and mastoid is squamous cell carcinoma.

Clinical Features

• It affects age group 40-60 years


• Slightly more c o m m o n in females
• Most i m p o r t a n t predisposing cause is long standing CSOM
• Patient may present w i t h chronic foul smelling blood stained discharge
• Pain is severe and comes at night.
• Facial palsy may be seen
• O/E - Friable, hemorrhagic granulation or polyp are present.
• Diagnosis - made only on biopsy
CT and angiography are done t o see the extent of disease.
Metastasis occurs t o cervical lymph nodes later.
Treatment o f carcinoma of middle ear is combination of surgery followed by radiotherapy.
Surgery consists of radical mastoidectomy/subtotal or total petrosectomy depending on the extent of tumor.


CHAPTER

Rehabilitative Methods

| HEARING AIDS Contd...

Hearing aids are devices t o amplify sounds reaching t h e ear. In the ear type [ITE]
Suitable for patients w i t h conductive hearing loss. In SNHL, there - Larger than ITC
may be distortion of sound due t o recruitment. - Fills the bowl of the ear
- Used for wide variety of hearing impairment
Hearing Aid C o m p o n e n t s - Easier to use than CIC & ITC

Behind the ear [BTE] DBS


Microphone Amplifier Receiver
- Circuit & the microphone fit behind the ear
Collects the sound & Intensifies electrical Electrical impulses
- Used for wide range of hearing loss
transforms into impulses translated to
- Good for children
electric energy louder sounds
Indications
Types of Hearing Aid
• A b s o l u t e I n d i c a t i o n : C o n g e n i t a l deafness, f o r p r o p e r
• Conventional t y p e : Increases the v o l u m e of all i n c o m i n g development of speech and languge:
sounds w i t h minor adjustments. • Patient w h o has hearing problem which is not treatable by
• Programmable analogue: Programmed by computer, has medical or surgical methods.
some flexibility for a d j u s t m e n t based o n preferences and • Conductive deafness patients w h o do not w a n t surgery/unfit
listening environment. for surgery.
• Digital type: The software is programmed by an audiologist
t o allow dramatic flexibility in adjustments. Soft sounds are
distinguished f r o m loud sounds. Clarity is enhanced. In SNHL - results are not very good particularly in those with
recruitment positive.
Hearing Aid Styles
Disadvantages o f conventional hearing aids
Hearing Aid Styles • Cosmetically unaceptable due to visibility.
• Acoustic feedback.
Completely in the canal [CIC]
• Spectral distortion.
- Smallest type • Occlusion of external auditory canal.
- Used for mild to moderate hearing loss • Collection of wax in the canal and blockage of insert.
• Sensitivity of canal skin to earmoulds.
- Most difficult to
• Problem to use in discharging ears.
- Place & adjust
| B O N E A N C H O R E D HEARING AID (BAHA)
In the canal type [ITC]
- Larger than CIC Newer A d v a n c e d Hearing Aid

- Used for mild to moderate hearing loss It acts by directly stimulating cochlea, bypassing external and
middle ear since it is anchored t o bone.
- Easier to use
Contd...
340 {_ SECTION V Ear

Current cochlear devices are FDA approved for implantation in


Indications for BAHA
children 12 months and older, w i t h no upper age restrictions.
1. When air-conduction (AC) hearing aid cannot be used:
Furthermore, it has been shown that outcomes in adults > 65 yrs
• Canal atresia, congenital or acquired, not amenable to treatment.
• Chronic ear discharge, not amenable to treatment. are no better or no worse than those in young adults. The earlier
• Excessive feedback and discmfortfrom air-conduction hearing the implantation is done in children, the more favorable the results.
aid. .
2. Conductive or mixed hearing loss, e.g. otosclerosis and Prerequisites:
tympanosclerosis where surgery is contraindicated.
3. Single-sided hearing loss. Intact VIII nerve and higher auditary pathways
At least 1 year of age.

I
Postlingual deaf patients tend t o do better than prelingual
VIBRANT SOUND BRIDGE/IMPLANTABLE
deafs.
HEARING AID

It is an i m p l a n t a b l e hearing aid w h i c h directly stimulates t h e


Surgical approach to place the electrode:
ossicles, bypassing external ear and tympanic membrane.
M/C used is facial recess approach (posterior tympantomy).
The implanted part consists of transducer attached to incus.
Recently Vera technique is gaining popularity.
C a n d i d a t e profile:

Appropriate candidates for direct drive middle ear hearing devices § AUDITORY BRAINSTEM IMPLANTS
include adult aged 18 years and older w i t h moderate-to-severe
sensorineural hearing loss. Candidates should have experience • It is designed t o stimulate the cochlear nuclear complex in the
of using traditional hearing aids and should have a desire for an brainstem directly by placing the implant in the lateral recess
alternative hering system. of f o u r t h ventricle. Such an implant is needed w h e n CN VIII has
been severed in surgery of vestibular schwannoma. In these
Advantage cases, cochlear implants are of no use.

Better sound q u a l i t y and less wax related problems and less


feedback.
In unilateral acoustic neuroma, ABI is n o t necessary as
h e a r i n g is possible f r o m t h e contralateral side b u t in
| COCHLEAR IMPLANTS bilateral acoustic neuromas as neurofibromatosis type 2
rehabilitation is required by ABI.
Cochlear Implant
• Site of implant = Lateral recess of fourth ventricle
• It is an electronic device that converts the mechanical sound
energies into electrical signals that can be directly delivered
i n t o t h e a u d i t o r y nerve in severe or p r o f o u n d l y hearing
In some patients, where auditory brainstem implant is not possible due
impaired individuals, w h o cannot benefit f r o m hearing aids
to tumor induced damage to cochlear nucleus after acoustic neuroma
(i.e. it bypasses the cochlea)
surgery; Inferior colliculus of midbrain can be stimulated-This is called
as Auditory midbrain implant.
C o m p o n e n t s of I m p l a n t s :

External component Internal component


• Microphone • Receiver/stimulator (implanted under the skin
• Speech processor • Electrode is implanted in
• Transmitter " 4-
• It remains outside the body Scale tympani ofthe cochlea via cochleastomy and if is not possible it can be assessed through the round window
0

It may be placed at other locations like promontory or round


window but these are inferior locations than cochlea.


CHAPTER 28 Rehabilitative Methods

QUESTIONS

1. Which of the following would be the most appropriate 4. In cochlear implants electrodes are most c o m m o n l y
t r e a t m e n t f o r r e h a b i l i t a t i o n of a p a t i e n t , w h o h a s placed at:
b i l a t e r a l p r o f o u n d d e a f n e s s f o l l o w i n g s u r g e r y for a. Oval window
bilateral acoustic schwannoma: [AIIMS Nov 03] b. Round window
a. Bilateral high powered digital hearing aid c. Horizontal semicircular canal
b. Bilateral cochlear implant d. Cochlea
c. Unilateral cochlear implant 5. Cochlear implant is done in: [Bihar 05]
d. Brainstem implant a. Scala vestibuli b. Scala tympani
2. A child aged 3 years, presented with severe sensorineural c. Cochlear duct d. Endolymphatic duct
deafness was prescribed hearing aids, but showed no 6. Which of the following statement regarding cochlear
improvement. What is the next line of management: implant is true: [AIIMS Nov 10]
a. Fenestration surgery b. Stapes mobilisation a. Cochlear malformation is not a CI to its use
c. Cochlear implant d. Conservative b. Contraindicated in children < 5 yrs of age
3. 10-year-old boy Rajan is having sensorineural deafness, c. Indicated in mild-moderate hearing loss
not benefited by hearing aids. Next best management d. Approached through oval window
is: [AIIMS 01] 7. Absolute indication for cochlear implantation is:
a. Cochlear implant b. Stapes fixation [AIIMS Nov 12]
c. Stapedectomy d. Fenestration a. Outer hair cell b. Inner hair cell
c. Spiral ganglion cell d. Auditory nerve

EXPLANATIONS AND REFERENCES

1. Ans. is d i.e. Brainstem implant Ref. Harrison 17th/ed p204

Hearing loss Rehabilitative measure


• Conductive hearing loss • Corrective surgery / Hearing aids
• Mild/moderate SNHL • Hearing aids

• Bilateral severe to profound SNHL with word recognition score < 30% • Cochlear implants

• Bilateral damage to eight nerve by trauma /bilateral vestibular • Brainstem auditory implants (placed near cochlear nucleus)
schwannoma

2. Ans. is c i.e. Cochlear implants Ref. Dhingra 5th/ed p139,6th/ed p 125; Current Otolaryngology 2nd/ed pg 882
3. Ans. is a i.e. Cochlear implant

[ B / L severe or profound hearing loss not benefited by hearing aid and it is an indication for use of cochlear implants

Criteria for t h e u s e of C o c h l e a r implants

Prelingual and Postlingual Children


Bilateral severe to profound hearing loss (only profound hearing loss in children < 2 years of age)
Lack of auditory develop ment with a proper binaural hearing aid trial
Properly aided open-set word recognition scores < 20-30% in children capable of testing
Suitable auditory developmental education plan
Lack of medical contraindication
Postlingual Adults
• 18 years of age
• B/L severe to profound hearing loss
• Properly aided recognition scores <40%
• Lack of medical contraindication, with cochlea and auditory nerve present
342|_ SECTION V Ear

Prelingual A d u l t s
• 18 years of age
• Bilateral profound deafness
• Minimal benefit from properly fitted hearing aid
• Lack of medical contraindication, with cochlea & auditory nerve present

Postling ually deaf adults and children benefit most by implants, but it can be used in prelingually deaf patients also.

4. Ans. is d i.e. Cochlea Ref. Dhingra 5Xh/edpg 140,6th/ed p 126


5. Ans. is b i.e. Scala tympani
M/C site of electrode placement in cochlear implant is scala tympani of cochlea via cochleostomy (i.e. opening basal turn of cochlea).
This allows the electrodes t o be in close proximity t o the spiral ganglion cells and their dendrites.
Round w i n d o w is a route for approach t o cochlea in those cases where a separate cochleostomy cannot be drilled.

M/C Surgical approach for placing cochlea implant = Facial Recess approach (Posterior tympanotomy) which involves doing a cortical mastoidectomy.
Recently Veria technique (Non-Mastoidectomy technique) is gaining popularity for cochlear implantation. It uses transcanal approach.

A d v a n t a g e of Vera technique

• Simple
• Less chances o f injuring facial nerve
• Suitable in young children where mastoid has not developed fully.
• Minimal bone trauma .•. fast healing and less complication rate
6. Ans. is a i.e. Cochlear malformation is not a CI to its use Ref. Current Otolaryngology 3rd/edp 856; Dhingra Sth/ed p 139-140

Explanation

As discussed earlier Cochlear implants are useful in B/L severe t o profound hearing loss and not in mild-moderate hearing loss
.-. Option C is incorrect (Dhingra 5th/ed p 139), 6th/ed p 125
Cochlear implants can be implanted in children at 12 months of age, rather early implantation gives better results. (Dhingra 5 t h /
edp139),6th/edp125
"The timing of implantation is very important. Earlier implantation in children generally yields more favorable results and many
centers roultinely implant children under 12 months of age." -Current Otolaryngology 3rd/ed p 856.
So f r i e n d s — O p t i o n b—C/l in children < 5 years of age is incorrect.
Approach for cochlear implants is via facial recess, where a simple cortical mastoidectomy is done first and short process of
incus and lateral semicircular canal is identified.
The facial recess is operated by performing a posterior tymparotomy. A cochleostomy is then done inferior t o round w i n d o w
(Not oval w i n d o w ) w i t h the goal of affording access t o scale tympani (where the electrode has t o be placed).
Thus o p t i o n d i.e. it is approached t h r o u g h oval w i n d o w is incorrect.
So by exclusion are answer is a i.e. cochlear malformation is not a contradiction t o its use.
7. Ans. is d i.e. Auditory Nerve - Ref. Mohan Bansal Textbook of Diseases of Ear, Nose and Throat 1 st/ed p 178

Cochlear Implants
• "They are indicated for patTents of profound binaural SNHL (with non functional cochlear hair cells) who have intact auditory nerve
functions and show little or no benefit from hearing aids." ...Mohan Bansal Ist/ed p 1 78

• •


CHAPTER

Misci

Laser uvulopharyngopalatoplasty is the surgery done for which of the following?


a. Snoring
b. Recurrent pharyngotonsilitis
c. Cleft palate
d Stammering
1. Ans. is a i.e. snoring of diseases of Ear Ref: Mohan Bansal Textbook of Diseases ofEar,Nose and Throat 7/e, p 435
Snoring: Noisy breathing, a rough, rattling inspiratory noise produced by vibration of pendulous soft palate or occasionally of vocal
cords, during sleep.
• Snoring indicates some obstruction in upper airway and represents a continum o f t h e similar pathology as of Obstructive Sleep
Apnea (OSA), where snoring is on one end and OSA on the other.
• Management of snoring without Obstructive Sleep Apnea.
• Uvulopalatoplasty-Laser Assisted Uvulopalatoplasty (LAUP) or Bovie-Assisted Uvulopalatoplasty (BAVP). It can be performed
under Local A naesthesia in OPD. In this procedure uvula is amputated and 1 cm trenches are created in the soft palate on either
side o f uvula. The soft palate elevates and stiffens after healing.
• Uvulopalatopharyngoplasty- It is the M/C surgery performed for Obstructive Sleep Apnea. It is also very effective in treating
snoring.
Also Know
• OSA - Obstructive sleep apnea is a disorder characterised by loud, habitual snoring and repetitive obstruction o f t h e upper
airway during sleep, resulting in prolonged intervals of hypoxia and fragmented sleep.
• Gold standard test in evaluation of OSA - Polysomnography. It differentiates snoring w i t h o u t OSA f r o m snoring w i t h OSA and
also identifies the severity of apnea.
Mullers maneuver - This test is done before uvulopalatopharyngoplasty t o know whether the patient will benefit f r o m the

surgery or not.


Z44]_ SECTION V Ear

FLOW CHART OF MANAGEMENT OF SNORING BOTH WITH OR WITHOUT OSA.

Snoring without Palatal surgery [uvulopalatoplasty (UPP)]


Sleep disorder apnea - Radiofrequency ablation or
breathing - Laser assisted
Snoring with <10
P S G h Respiratory
apnea
disturbance index

(Polysomnography)

T TT

Removal
Adenotonsillar
hypertrophy Anatomic r Continuous Dental devise Night time
-
abnormality positive (Contraindicated oxygen
and potential airway pressure in children and
sources of -CPAP or TMJ disorders)
Midface airway abstruction -BPAP
deficiency Na sal (cephalometrics) (Bilevel positive
retrognathic obstr jction airway pressure)

Maxillo- Surgical Retrolingual


mandibular correction obstruction
advancement!

\ •

Tracheostomy

2. Citellis angle is:


a. Sold angle b. CP angle
c. Sinodural angle d. Part of MC Evans triangle
2. Ans. is c i.e. Sinodural angle Ref. Dhingra 6th/edp402; Fig. 79.5
As seen f r o m the figure Citellis's angle is Sinodural angle
Anterior
Up —j—Down
Posterior

Incus
CN VII

Trautmann's Donaldson's line


triangle
Site of
Sinodural
(Citelli1

endolymphatic sac
angle

Digastric ridge
Sigmoid sinus plate
CHAPTER 29 Miscellaneous _J 345

3. Which of the following is a features of tympanic membrane perforation (printed esophageal rupture in paper): [UP 00]
a. Tinnitus b. Vertigo
c. Conductive deafness d. Fullness in ear
3. Ans. is c i.e. Conductive deafness Ref. Dhingra Sth/ed pg 34; Turner 1 Oth/ed pg 284
Tympanic membrane perforation is associated w i t h a conductive hearing loss of 10-40 dB.
4. Which a m o n g the following is not a feature of retracted tympanic membrane: [PGI June 99]
a. Loss of cone of light
b. Shortening of handle of malleus
c. Draping of tympanic membrane over handle of malleus
d. Degeneration of head of malleus
4. Ans. is d i.e. Degeneration of head of malleus Ref. Dhingra 4th/edp 54,5th/edpg 61-62

Normal tympanic membrane Retracted tympanic membrane


• Shiny and pearly gray in color with a concavity on its lateral • Dull and lusterless
surface • Cone of light is absent or interrupted
• Cone of light is present in anteroinferior quadrant • Lateral process of malleus becomes very prominent
• It is mobile • Anterior and posterior malleal folds become sickle shaped

• A retracted tympanic membrane is the result of negative intratympanic pressure when Eustachian tube is blocked.
• Most important landmarkfor otoscopy - Lateral process of malleus.
• Cone of light is formed by handle of malleus.

5. Chalky white tympanic membrane is seen in: [RJ01]


a. ASOM b. Otosclerosis
c. Tympanosclerosis d. Cholesteotoma
5. Ans. is c i.e. Tympanosclerosis: Ref. Dhingra 5th/ed p62
Tympanosclerosis is hyalinization and later calcification in the fibrous layers of tympanic membrane. It appears as chalky w h i t e
plaque. It is seen in cases of serous otitis media as a complication of ventilation t u b e and is generally asymptomatic.
6. Direction of water jet while doing syringing of ear should be: [MH02]
a. Anterior " . * b. Posterior
c. Anterosuperior d. Posterosuperior
6. Ans. is d i.e. Posterosuperior Ref. Dhingra 4th/ed p 53; 5th/ed pg 60
"Pinna is pulled upward and backward and a stream of water from the ear syringe is directed along posterosuperior wall ofthe me-
atus." - Dhingra 5th/ed p 60
7. Use of Siegel's speculum during examination of the ear provides all except:
a. Magnification b. Assessment of movement o f t h e tympanic membrane
c. Removal of foreign body from the ear d. As applicator for the powdered antibiotic of ear
7. Ans. is c i.e. Removal of foreign body
Ref. Dhingra 5th/ed p 383; SR Mowson Disease of Ear 4th/ed p 93-94; Maqbool 11 th/ed p 34
.

U s e s of S i e g e l ' s P n e u m a t i c S p e c u l u m
Mnemonic: 3T-3M

Fistula test . -
3T's are: Gelle's test
Powder test

For magnification
3M's are: For instillation of medicines into middle ear •

To assess the mobility of tympanic membrane

Periphery of tympanic membrane has maximum mobility . 0


346^ SECTION V Ear

The focal length o f t h e mirror used in head lamp: [APPGI06]


a. 85 m m . b. 150 m m
c. 250 mm d. 400 rnm
8. Ans. is c i.e. 250 mm Ref. Dhingra 5th/edpg 379
• Head mirror is a concave mirror used t o reflect light f r o m the bull's eye lamp o n t o the part being examined
It has focal length of ~ 25 cm (= 10 inch) The examiner sees t h r o u g h the hole in the centre o f t h e mirror
Diameter of central hole = 19 m m (3/4 inch)
9. Focal length of head mirror used in ENT-OPD: [Bihar 2005]
a. 9 inch b. 0 inch (25 cm)
c. 11 inch d. 2 inch
9. Ans. is b i.e. 10 inch (25 cm) • Ref. Dhingra 4th/ed p 336,5th/ed p379
10. Diameter of head mirror in ENT is: [Bihar2005]
a. 20 cm b. 22 cm
c. 10 cm d. 26 cm
10. Ans. is c i.e. 10 cm Ref. Dhingra 4th/ed p 336,5th/ed p379
Diameter o f the head mirror used in ENT is 89 m m (3and half inch)
11. 4 0 db compared to 20 db is: [DNB 01]
a. Double b. 10 times
c 100 times d. 1000 times
11. Ans. is b i.e. 10 Times Ref. Dhingra 4th/ed p20,5th/ed pg 23
Decibel is 1/10th of bel.
In audiology, sound is measured as sound pressure level (SPL)
SPLofSI
Sounds in dB = 20 log
SPL of SO
S1 = Sound being described
SO = Reference sound
• If a sound has an SPL of 1000 i.e., 103 times the reference sound, it is expressed as 20 x 3 = 60 dB.
• Similarly a sound of 1000,000 i.e., (106) times the reference sound, it is described as 20 x 6 = 120 dB working on the same principle.
• 20 dB = will come. If sound has SPL of 10 times the reference sound (20 x 1).
• 40 dB will come if sound has SPL of 100 times the reference sound (20 x 2).
• So 40 dB is 10 times more man 20 dB.
12. Impedance denotes: [PGI 99]
a. Site of perforation b. Disease of cochlea
c. Ds. of ossicles d. Higher function disorder
12. Ans. is c i.e. Disease of Ossicles Ref. Tuli Ist/edp 35
"Impenance audiometry, measures the resistance of tympanic membrane and middle ear and also compliance of tympanic membrane
and ossicular chain t o sound pressure transmission. Tympanogram is the graphic representation of compliance and impendance
of tympanic osscular system w i t h air pressure changes."
From above text it is quite obvious impedance is for disease of ossicles.
13. During normal conversation sound heard at 1 meter distance is: [Bihar 2004]
.a. 80dB b. 60dB
c. 90dB d. 120dB
13. Ans. is b i.e. 60dB Ref. Dhingra 4th/ed p 20,5th/ed p23
Whisper . 30 Db
Normal conversation 60 Db
Shout 90 Db •

Discomfort o f t h e ear 120 Db


Pain in the ear 130dB
14. Normal conversation sound level: [Bihar2005]
a. 20-25dB 60-65dB
c. 80-85dB 90-1 OOdB
14. Ans. is is b i.e. 60-65dB Ref. Dhingra 4th/edp 20; 5th/edpg 23
15. Prolonged exposure to noise levels greater than the following can impair hearing permanently: [Karnat96]
a. 40 decibels . b. 85 decibels
c. 100 decibels d. 140 decibels
CHAPTER 29 Miscellaneous T 347

15. Ans. is c i.e. 100 decibels Ref. Park 19th/ed p 599


"Repeated or continuous exposure to noise around 100 decibels may result in a permanent hearing loss." ... Park 19th/edp 599
"A noise of 90 dB SPL, 8 hours a day for 5 days per week is the maximum safe limit as recommended by ministry of labour, govt
of India-rules under factories act." ... Dhingra 4th/ed p 35; 5th/ed pg 40

NOTE HHII^fl^^^^H
Impulse noise (single time exposure) of more than 140 dB is not permitted.

16. A man Rajan, age 70 yrs, presents with tinnitus. Most probable diagnosis is: [AIIMS Nov 00]
a. Acoustic neuroma b. ASOM
c. Labrynthitis d. Acoustic trauma -

16. Ans. is a i.e. Acoustic neuroma Ref. Dhingra - read below

Condition Points in favour Points against

Acoustic neuroma Presenting symptom Associated with SNHL hearing loss (which is not
- Tinnitus given in the question)
- Age of patient - No history of ear ache, fever and hearing
ASOM (Dhingra 4th/ed p61) Tinnitus may be seen in stage of presupperation loss
Labyrinthitis Tinnitus may be seen - Tinnitus is not the presenting symptom
- It is common in infants and children
Acoustic trauma - Tinnitus is not seen
- No history of trauma
- It is associated with varying degree of
hearing loss which is not given

Amongst the options given, acoustic neuroma is the best o p t i o n here. If presbycuses w o u l d have been given in the options, w e
w o u l d have chosen it
17. Gustatory sweating and flushing (Frey's syndrome) follows damage to the: [JIPMER 80; DNB 91]
a. Trigeminal nerve b. Facial nerve
c. Glossopharynegeal nerve d. Vagus nerve
e. Auriculotemporal nerve
17. Ans. is e i.e. Auriculotemporal nerve Ref. Maqbool 11 th/ed p 276; S. Das Short Cases of Clinical Surgery 3rd/ed p 82
Auriculotemporal syndrome (Syn. Frey's Syndrome)
Partial injury t o the auriculotemporal nerve gives rise to such syndrome. This type of injury:
• May be congenital, possibly due t o birth trauma.
• May be accidental injury.
• . May be caused by inadverent incision for drainage of parotid abscess.
• May occasionally follow superficial parotidectomy.
Clinical features: There is flushing and sweating o f t h e skin innervated by the auriculotemporal nerve particularly during meal
and presence o f cutaneous hyperaesthesia in front and above (the ear the area supplied by the auriculotemporal nerve.)

The Explanation of this syndrome is:


- The postganglionic parasympathetic fibres become united t o the sympathetic nerves from the superior cervical ganglion which
are concerned t o supply vessels and sweat glands of that region. This causes flushing and sweating o f t h e skin.
- Following injury t o the auriculotemporal nerve, postganglionic parasympathetic fibres from the otic ganglion grow d o w n the
sheaths o f t h e cutaneous filaments, so hyperaesthesia follows stimulation o f t h e secretomotor nerves.
Treatment: If t h e symptoms persist, the treatment is avulsion o f t h e auriculotemporal nerve in front o f t h e auricle where it lies just
posterior t o t h e superficial temporal vessels.
18. CSF otorrhoea is caused by: [AIIMS 92]
a. Rupture of tympanic membrane b. Fracture of cribriform plate
c. Fracture of parietal bone d. Fracture of petrous temporal bone
18. Ans. is d i.e. Fracture of Petrous temporal bone Ref. Logan and Turner 1 Oth/ed p 347
• Fracture of petrous temporal bone can lead t o CSF otorrhea.
• CSF otorrhea mostly ceases spontaneously.
• Systemic antibiotics which cross blood brain barrier should be used (like - cotrimoxazole).
• Rarely surgical closure o f t h e tear o f dura in middle fossa is required.
348|_ SECTION V Ear

19. The most common cause of cerebrospinal otorrhoea is: [UP 97]
a. Rupture of tympanic membrance b. Fracture or petrous ridge
c. Fracture of mastoid air cells d. Fracture of parietal bone
19. Ans. is b i.e. Fracture of Petrous ridge Ref. Logan Turner Wth/edp 347
20. A patient has bilateral conductive deafness, tinnitus with positive family history. The diagnosis: [AIIMS 93]
a. Otospongiosis b. Tympanosclerosis
c. Menitere's disease d. Bilateral otitis media
20. Ans. is a i.e. Otospongiosis Ref. Dhingra Sth/ed pg 97-98
21. Presbycusis is: [TN 2007,205]
a. Loss of accommodation power b. Hearing loss due t o aging
c. Noise induced hearing loss d. Congenital deafness
21. Ans. is b i.e. Hearing loss due to aging Ref. Dhingra 5th/edpg 41; Scott-Brown's Otolaryngology 7th/ed vol 3,Chap 238 p 3539
It is m i d to late adult onset, bilateral, progressive sensorineural hearing loss, where underlying causes have been excluded.
22. Second primary tumor of head and neck is most commonly seen in malignancy of: [AIIMS May 2012]
a. Oral cavity b. Larynx
c. Hypopharynx d. Paranasal sinuses
2 2 . Ans. is a i.e. oral cavity. Ref. internet search
• Patients w i t h head and neck squamous cell carcinoma (HNSCC) are at increased risk for the development of second primary
malignancies compared w i t h the general population.
• These second primary malignancies typically develop in the aerodigestive tract (lung, head and neck, esophagus).
• The most frequent second primary malignancy is lung cancer.
• The highest relative increase in risk is for a second head and neck cancer.
• The site o f t h e index cancer influences the most likely site of a second primary malignancy.
- In patients w i t h an index malignancy of the larynx, the second primary t u m o r was c o m m o n l y seen in lung, while
- In patients w i t h an index malignancy of the oral cavity, the second primary t u m o r was commonly seen in head and neck or
esophagus.
The criteria for classifying a tumor as a second primary malignancy are:
• Histologic confirmation of malignancy in both the index and secondary tumors.
• There should be at least 2 cm of normal mucosa between the tumors. If the tumors are in the same location, then they should
be separated in t i m e by at least five years.
• Metastatic t u m o r should be excluded.
23. In right handed person, direct laryngoscope is held by which hand? [AIIMS May 2012]
a. Left b. Right
c. Both d. Either of these
23. Ans. is a i.e. Left Ref. Dhingra 5th/ed p ss432
"Laryngoscope is held by the handle in the left hand. Right hand is used, to retract the lips and guide the laryngoscope and to handle
suction and instruments." —Dhingra

-

• . •


CHAPTER

30 Important Operative Procedures

| U P P E R AIRWAY O B S T R U C T I O N A N D T R A C H E O S T O M Y Management of Upper airway Obstruction

Diagnostic sign of upper airway obstruction is stridor See Table 30.1


• O t h e r s y m p t o m s can be restlessness, Hoarseness (as in
laryngeal pathology). Nostril flaring, suprasternal/intercostal
retraction, C o u g h i n g or wheezing (as in trachea bronchial Most definitive management of upper airway obstruction
pathology) Tracheostomy
• Investigation of choice in upper airway obstruction-Fiberoptic
endoscopy

Table 30.1: Management o f Upper airway Obstruction

Immediate maneuvers Medical Management Alternate airway


- Heimlich maneuver - 0 inhalation through laryngeal
2 - Oral airway
- Jaw Thrust - Mask/Nasal cannula - Nasopharyngeal airway
- Heliox (80% helium and - Endotracheal intubation (C/l in fracture of cervical spine,
- 20% oxygen facial/oral
- Principle - It converts the turbulent - trauma, laryngeal trauma)
- flow at the site of obstruction into - Laryngeal mask ventilation C/l = Large retropharyngeal
- laminar pattern tumors,
- Retropharyngeal abscess
- Hiatus hernia Pregnancy
- Cricothyrotomy (Figure 1B)
- Emergency procedure done by piercing the cricothyroid
membrane called as minitracheostomy

| TRACHEOSTOMY F e a t u r e s of T r a c h e o s t o m y T u b e s
Material: Silicon is the preferred material especialy in children
S i t e — 2 n d , 3rd a n d 4 t h tracheal rings w h i c h lie under t h e
since it is flexibile and it reduces risk of mucosal trauma and
isthmus of thyroid gland. skin injury around the stoma.
If t r a c h e o s t o m y is d o n e a b o v e this, i t is called as h i g h Metal tubes (made of german silver) and Portex tube also avail-
tracheostomy; it can lead to perichondritis of cricoids cartilage able. Portextube (PVCtube/Nonmetallic tubes) is the best tube
and subglottic stenosis. If it is made below isthmus, it is called during radiotherapy
low tracheostomy and may injure great vessels o f neck and Cuff: Inflatable cuffs prevent aspiration of blood or saliva and
the apical pleura especially in children. f o r m a seal t o prevent leakage of ventilating gases d u r i n g
Elective high tracheostomy is done in malignancy o f larynx anesthesia or p r o l o n g e d mechanical v e n t i l a t i o n . But cuffs
presenting w i t h stridor where a laryngectomy has t o be done can be associated w i t h t h e risk of subglottic stenosis. For this
reason Low P ressure Cuffs are preferred. In children, cuffed
later. This is because after laryngectomy, a new tracheostoma
tracheotomy tube should not be used.
has t o be created lower d o w n .
Inner Tube: It projects 2-3 m m beyond t h e main outer tube
Elective low tracheostomy is done in patients w i t h laryngeal and helps in periodic cleaning w i t h o u t disturbing t h e patency
trauma t o prevent aggravation o f t h e laryngeal injury and in of t h e main tracheostomy. So they are t h e best f o r home
laryngeal papillomatoses t o avoid implantation. tracheostomy care.
SECTION VI Operative Surgery

Hyoid bone

Infrahyoid
muscles

Thyroid
cartilage
Cricoid
cartilage
1st and
2nd Sternocleido-
tracheal mastoid
cartilages
Trachea

Fig. 30.1: Incisions for tracheostomy. (A) Surface landmarks for the midline skin vertical incision for tracheostomy;
(B) Horizontal skin incision for cricothyrotomy
Courtesy: Textbook of Diseases of Ear, Nose and Throat, Mohan Bansal. Jaypee Brothers, p51l.

• Fenestration: Allows air t o pass t h r o u g h the t u b e and aids FOREIGN BODIES OF UPPER AERODIGESTIVE TRACT
phonation, it is the t u b e of choice in children. Drawback—Oral
contents and stomach contents can enter the lungs t h r o u g h • Foreign body aspiration is more c o m m o n in children in <4 yrs
these fenestrations. (vegetable foreign bodies even peanuts are the M/C foreign
body)
Disadvantage: Patient w h o are at risk of aspiration and are on
• M/C Site for lodging of foreign body of upper digestive t r a c t —
IPPV should n o t be given fenestrated tube.
Cricopharynx—since it is the narrowest part.
• Other sites of foreign body impaction a r e — Tonsil, Vallecula
S t r u c t u r e s D a m a g e d in E m e r g e n c y T r a c c h e o s t o m y
and Pyriform sinus.
1. Isthmus 2. Left brachiocephalic vein. Jugular vein
3. Pleura 4. Thymus Presentation
5. Inferior ima artery 6. Esophagus
• In foreign body in cricopharynx - B/L pooling of saliva
• In foreign body in pyriform sinus - U/L pooling of saliva
Drawbacks

1. Post tracheostomy apnea-it is due to wash out of CO w i t h rapid | FOREIGN BODY OF LARYNX
improvement in oxygenation after tracheostomy. Treatment
is Carbogen inhalation which is a mixture of 9 5 % oxygen and 1. A smaller foreign body may present as hoarseness, stridor and
5%C0 . 2 cough.
Emphysema—In Immediate postoperative period surgical emphy- 2. But a large laryngeal foreign body is an emergency since it
sema is either due to tight skin closure or large opening on the leads t o total airway obstruction and patient may asphyxiate
trachea. Immediate management is t o release the skin sutures. t o death, if first-aid measures are not taken.
Bleeding—Anterior jugular vein and inferior thyroid veins are
the commonest sites of bleeding. Management

Difficult decannulation- Patients w h o are on tracheostomy • Emergency measure is Heimlich's maneuver. In c h i l d r e n


for long time, develop psychological dependence. This is the pounding the back after turning the patient head d o w n can
commonest long term complication in children. be tried.
• •
Types of Tracheostomy Tube

See Table 30.2
CHAPTER 30 Important Operative Procedures

Table 30.2: Classification of Tracheostomy Tube Other F e a t u r e s Which Point T o w a r d s F o r i e g n B o d y in


Bronchus are—unexplained or unilateral wheeze, or unexplained
On the basis of cuff cough or hemoptysis or obstructive emphysema (if it leads to partial
Uncuffed obstruction), or t o atelectasis which in turn can cause pneumonitis
Cuffed tubes (if it leads t o complete obstruction)
* Single cuff t u b e
• Double cuff t u b e
• Low pressure cuff t u b e Foreign bodies are more common on right side as right bronchus is
On the basis of fenestra at the upper curvature of the t u b e short, wide and more in line with the trachea.
Tubes w i t h o u t fenestra
Single fenestrated t u b e Management
Multiple fenestrated tube
Bronchoscopy
On the basis of length o f t h e t u b e
Standard length Rigid bronchoscopy Flexible fiberoptic
Extra length tracheostomy tube bronchoscopy
Done via mouth Done through nose
• Adjustable flange long t u b
Structures seen—Uvula, Structures s e e n — P o s t e r i o r
On t h e basis o f number of lumens (cannula)
Epiglottis, vocal cords, tracheal choana, pharynx, larynx, tracheal
Single lumen (cannula) t u b e - Nonmetallic rings,
rings, carina and segmental
Double lumen (cannula) t u b e - Jackson and Fuller bronchi carina, segmental bronchi and
Suction-aided tracheostomy tubes - metallic subsegmental bronchi
On t h e basis of the material Advantage—in removal of foreign Advantage—Better magnification
- Metallic body and in children due to and better vision, can be used in
• Jackson problem of ventilation and in conditions where rigid
• Fuller establishing emergency airway bronchoscopy is C/l like in
Nonmetallic cervical spine instability,
trismus, micrognathia, recent
• Polyvinyl chloride (pVc)
Ml, and it is useful in bedside
« Silicone
examination ofthe critically ill
• Siliconized pVc patients. It can be passed easily
• Silastic through endotracheal tube or
• Rubber t u b e tracheostomy opening.
Mixed
• Armored tubes

• When these measures fail, cricothyrotomy (Laryngotomy) is Flexible fibreoptic bronchoscopy is replacing rigid bronchoscopy but
d o n e t o gain rapid entry t o airway and is converted into a its utility limited in children because ofthe problems of venulation.
normal Tracheostomy once the patient is shifted t o a primary
care set-up since it can lead t o laryngeal stenosis later on. N a m e d I n c i s i o n s u s e d in N a s a l S u r g e r i e s :

Incision Surgery
| FOREIGN BODY OF BRONCHUS • Killian's incision • Submucous resection
• Weber ferguson incision • Total maxillectomy
Presentation
• Freer's incision • Septoplasty
° Initial choking, cyanosis followed by cough and wheeze. • Moure's incision • Lateral rhinotomy

Pituitary Surgeries

[ Routes to pituitary |

[Trans-sphenoid I T Transcranial

[ 1 Used in craniopharyngioma
I Via septum via ethmoids Transnasal endoscopic and meningioma and in large -
approach tumors not accessible through nose.
Approach of choice)
r
Primary pituitary tumors are most
commonly removed by transsphenoid route


354 ]_ SECTION VI Operative Surgery

QUESTIONS

1. Tracheostomy is indicated in all except: [AI91] c. Tracheal dilation


a. Tracheal stenosis d. Resection and end-to-end anastomosis
b. Bilateral vocal cord palsy 12. Topical Mitomycin C is useful in treatment of?
c. Foreign body larynx [Al 09, 10,12]
d. Uncomplicated bronchial asthma a. Angiofibroma b. Tracheal stenosis
2. Tracheostomy is indicated in-all except: [MP 97] c. Skull base Ostemyelitis d. Laryngeal carcinoma
a. Carcinoma larynx 13. Which o f t h e following statements regarding Heliox are
b. Uncomplicated bronchial asthma correct: [Al 09]
c. Diphtheria a. It is inert
d. Comatose patient b. Has low viscosity
3. The most common indication for tracheostomy is: c. Decreases airway resistance
[JIPMER 91] d. Safe in pulmonary HT
a. Laryngeal diphtheria b. Foreign body aspiration 14. The commonest site of aspiration of a foreign body in
c. Carcinoma d. Asthma the supine position is into the: [PGI 99]
4. Tracheostomy is not indicated in: [Rajasthan 97; TN 04] a. Right upper lobe apical b. Right lower lobe apical
a. Emphysema b. Bronchiectosis c. Left basal d. Right medial
c. Atelectasis d. Pneumothorax 15. "Gold s t a n d a r d " surgical procedure for prevention of
5. A high tracheostomy may be indicated in: [SGPGI05] aspiration is: [AIIMS Nov 03]
a. Scleroma of the larynx a. Thyroplasty
, T V
b. Multiple papillomatosis of larynx
b. Tracheostomy
c. Bilateral vocal cord paralysis
c. Tracheal division and permanent tracheostome
d. Carcinoma of larynx
d. Feeding gastrostomy/jejunostomy
6. True about tracheostomy tube are all except: [Al 99]
16. Best management for inhaled foreign body in an infant
a. Double tube
is: [AI97]
b. Made of titanium silver alloy
a. Bronchoscopy b. IPPV and intubation
c. Cuffed tube for IPPV
c. Steroid d. Tracheostomy
d. Has to be changed ideally in every 2 to 3 days
17. Openings of the tube of bronchoscope are known as:
7. All are true about tracheostomy tube except: [MP 2001]
[MH03]
a. Jackson's tube has 2 lumens
a. Holes b. Apertures
b. Removal of metallic tube in every 2-3 days
c. Vents d. Any of the above
c. Cuffed tube is used t o prevent aspiration of pharyngeal
18. In a one-year-old child intubation is done using:
secretion
[MP 2002]
d. Made up of titanium-silver alloy
a. Straight blade with uncuffed tube
8. Montgomery tube used in ENT procedure is a:
b. Curved blade with uncuffed tube
a Double barrel t u b b. Lobster tail tube
c. Straight blade with cuffed tube
c. Airway tube d. Silicone tube
d. Straight curved blade with cuffed tube
9. In e m e r g e n c y tracheostomy following structures are 19. A 2 year old child with intercostal retraction and
damaged except: [AIIMS Nov 07] increasing cyanosis was brought with a history of foreign
a. Isthmus of thyroid b. Inferior thyoid vein b o d y aspiration w h i c h might be a lifesaving in this
c. Inferior thyroid artery d. Thyoid ima situation: [AIIMS 99]
10. Most common complication of tracheostomy is: [PGI 97] a. Oxygen through face mask
a. Tracheoesophageal fistula b. Heimlich's manoeuvre
b. Tracheocutaenous fistula c. Extra cardiac massage
c. Surgical emphysema d. Intracaridiac adrenaline
d. Tracheal stenosis 20. Bronchoscopy visualizes all except: [AI2010]
11. A 30-year-old Ravi presented with gradually increasing a. Trachea
r e s p i r a t o r y d i s t r e s s since 4 d a y s . S h e gives history b. Vocal cords
of h o s p i t a l i z a t i o n a n d m e c h a n i c a l v e n t i l a t i o n w i t h c. First segmental subdivision of bronchi
orotracheal intubation for 2 weeks. Now she is diagnosed d. Subcarinal Lymph nodes
as having severe laryngotracheal stenosis. Next step in 21. Which of the following is not a contraindication for
the management is: bronchoscopy: [JIPMER 79, Delhi 83]
a. Laser excision and stent insertion a. Lesions of cervical spine b. Cardiac failure
b. Steroid c. Active bleeding d. Trismus


CHAPTER 30 Important Operative Procedures • J 355

22. A 2-year-old child develops acute respiratory distress. 2 4 . Rigid esophagoscopy is not done in: [PGI 01]
O/E breath sounds are decreased with wheeze on right a. Cervical spine rigidity
side. Chest X-ray shows diffuse opacity on right s i d e — b. Aortic aneurysm
Most probable diagnosis: c. Carcinoma esophagus
a. Pneumothorax b. Foreign body aspiration d. Esophageal web
c. Pleural effusion d. U/L emphysema. e. Lung abscess
23. A 5-year-old boy h a v i n g d i n n e r s u d d e n l y b e c o m e s 2 5 . Route of approach of glossopharyngeal neurectomy:
aphonic a n d is brought to casuality for the complaint [Kolkata 00]
of respiratory difficulty. What is the most appropriate a. Tonsillectomy approach
management? b. Transpalatal approach
a. Cricothyroidotomy b. Tracheostomy c. Transmandibular approach
c. Humdified 0 2 d. Heimlich maneuver d. Transpharyngeal approach

EXPLANATIONS AND REFERENCES


-

1. Ans. is d i.e Uncomplicated bronchial asthma


2. Ans. is b i.e. Uncomplicated bronchial asthma
Ref. Maqbool 11 th/ed p351- 352; Dhingra 5th/ed p 337,6th/ed p 317; Head and Neck Chris deSouza Vol2p1643

Indications for Tracheostomy

a. Respiratory obstruction:
• Infections:
- Acute laryngotracheobronchitis, acute epiglottitis, diphtheria
- Ludwig's angina, peritonsillar, retropharyngeal or parapharyngeal abscess, tongue abscess.
• Trauma:
- External injury to larynx and trachea
- Trauma due to endoscopies especially in infants and children
- Fractures of mandible or maxillofacial injuries
• Neoplasms: Benign and malignant neoplasms of larynx, pharynx, upper trachea, t o n g u e and thyroid.
• Foreign body in larynx
• Edema larynx due t o steam, irritant fumes or gases, allergy (angioneurotic or d r u g sensitivity), radiation.
• Bilateral abductor paralysis
• Congenital anomalies:
- Laryngeal web, cysts, tracheooesophageal fistula
- _ Bilateral choanal atresia.
b. Retained secretions:
• Inability t o cough:
- Coma of any cause, e.g. head injuries, cerebrovascular accidents, narcotic overdose.
- Paralysis of respiratory muscles, e.g. spinal injuries, polio, Guillain-Barre syndrome.
- Spasm of respiratory muscles, tetanus, eclampsia, strychinine poisoning.
• Painful cough : Chest injuries, multiple rib fractures, pneumonia.
• Aspiration of pharyngeal secretions: Bulbar polio polyneuritis, bilateral laryngeal paralysis.
c. Prolonged ventilation/Fat assisted ventilation (m/c indication these days)
Note: If IPPR is expected t o prolong beyond 12 hours, tracheostomy is preferred over endotracheal intubation.
d. Respiratory insufficiency - chronic lung conditions - viz emphysema, chronic Bronchitis, bronchiectasis, atelectasis
e. As a part of other surgeries
3. Ans. is b i.e. Foreign body aspiration / none
Ref. Head and Neck Surgery - Chris deSouzavol 2 pg. 1643; Mohan Bansal pSIO; Scotts Brown 7th efvol 2 pg. 2293
Friends - earlier - when this Question was framed -the answer was Foreign body aspiration but now in to days scenario -the
answer is... (Read for yourself)
"Historically, the main indication for a tracheostomy was to bypass upper airway obstruction caused by a foreign body or infection/\
particularly diphtheria. Nowadays upper airway obstruction is the least common indicator for tracheostomy. Almost two thirds of j
tracheostomies are currently performed on intubated intensive care patients, mainly to aid removal of secretions from the distal tra- j
cheobronchial tree and to facilitate weaning from distal tracheobronchial tree in acute respiratory failure and prolonged ventilation" i
- Head and Neck Surgery Chris De Souza 2nd/ed p 1643
356 T SECTION VI Operative Surgery

"Today, prolonged Intubation usually with mechanical ventilation is the most common indication for tracheosotmy formerly it was upper
respiratory obstruction." —Mohan Bansal p510
4. Ans. is d i.e. Pneumothorax Ref. Dhingra Sth/ed p 36,339
Friends, it is quite obvious that pneumothorax can be a complication of tracheostomy (if not performed properly) rather than an
indication.
5. Ans. is d i.e Carcinoma of larynx Ref. Dhingra Sth/ed p 337; Scott Brown 7th/ed vol 2 pg. 2295; Mohan Bansal p510
"It is i m p o r t a n t (in tracheostomy) t o refrain f r o m causing any damage in the region of cricoid cartilage.
An exception t o this rule is w h e n a patient has laryngeal malignancy and under these circumstances tracheostomy should be placed
high so as t o allow resection of tracheostomy site at the t i m e of laryngectomy". - Scott Brown 7th/ed vol 2 pg. 2295
" The high tracheostomy is generally avoided because ofthe postoperative risk of peri-chondritis ofthe cricoid cartilage and subglottic
stenosis. In cases of carcinoma larynx with stridor when total laryn-gectomy would be done, high tracheostomy is indicated."
—Mohan Bansal p510
6. Ans. is d i.e. Has to be changed ideally in every 2 to 3 days
7. Ans. is b i.e. Removal of metallic tube in every 2-3 days
Ref. Mohan Bansal pp 592,593; Maqbool 11 th/ed p 354; Turner 10th/ed 195; Head and Neck Surgery Chris DSouzavol 2 pg. 1647
A tracheostomy t u b e may be metallic or nonmetallic

Metallic Tracheostomy Tube

Metallic tubes are formed f r o m the alloy of silver, copper phosphorus (option b in Q 6 a n d o p t i o n d in Q7).
Has an inner and an outer tube.The inner tube is longer than the outer one so that secretions and crusts formed in it can be removed
and the tube reinserted after cleaning w i t h o u t difficulty. However, they do not have a cuff and cannot produce an airtight seal.

Nonmetallic Tracheostomy Tube


• Can be of cuffed or noncuffed variety, e.g. rubber and PVC tubes.

Cuffed Tracheostomy Tubes


• A cuff is a balloon-like device around the distal end o f t h e tracheostomy tube. Most cuffed tubes now available have low pressure
cuffs w i t h a high volume.This significantly reduces the possibility of pressure necrosis and potential stenosis formation. Pediatric
tubes do n o t have a cuff . Cuffed tubes are used in situation where positive pressure ventilation is required, or w h e n the airway
0

is at risk f r o m aspiration. (In unconscious patient or w h e n patient is on respiration).


The cuff should be deflated every 2 hours for 5 mins t o present pressure damage to the trachea.
Uncuffed Tracheostomy Tubes

As the name suggest, this t u b e does not have a cuff that can be inflated inside the trachea. It is suitable for a patient w h o has
returned to t h e ward f r o m a prolonged stay in intensive care and requires physiotherapy and suction via trachea. This type of tube
is not suitable for patients w h o are unable t o swallow due t o incompetent laryngeal reflexes, and aspiration of oral or gastric c o n -
tents is likely t o occur. An uncuffed tube is advantageous in t h a t it allows the patient t o breathe around it in the event o f t h e t u b e
becoming blocked. Patients can also speak w i t h an uncuffed tube.
"Jackson and Fuller tracheostomy tube have two lumens (see the box given in the text)."s —Mohan Bansal p 592
"Tracheostomy tubes should not be disturbed for the first 48-72 hours, but thereafter the tube is changed daily and cleaned at regular
intervals." ... Turner 10th/ed, p 195

A c c o r d i n g t o S/B 7 t h e d v o l 2 p g . 2 2 9 8

"The frequency with which the inner tube needs to be cleaned will vary. In the early post operative period. It may need cleaning every
couple of hours".
8. Ans. is d i.e Silicone tube Ref. Internet Search
M o n t g o m e r y tracheal tube is designed t o give the surgeon a complete program for creating a secondary airway- f r o m initial incision
t h r o u g h long-term tracheostomy care. It is a tracheal cannula system used in place of tracheostomy tubes. The system provides
long-term access t o the tracheal airway in situations that require an artificial airway or where access is needed for pulmonary
hygiene.
• It is so designed that the thin inner flange o f t h e cannula is shaped t o fit snugly against the contour of the inner anterior tracheal
wall. No t u b e projects into the tracheal lumen.
• All tracheal cannulas are made of flexible implant grade silicone t o assure patient comfort and safety while reducing complications.


CHAPTER 30 Important Operative Procedures J 357
9. Ans. is. ci.e Inferior thyroid artery Ref. Keith L Moore 5th/ed pi 100
Structures w h i c h lie below the midline viz. isthmus of thyroid and thyroid ima artery can be damaged in emergency tracheostomy.
Inferior thyroid veins emerge at the lower border o f t h e isthmus f o r m a plexus in front o f t h e trachea and drains into brachioce-
phalic vein can be damaged during tracheostomy b u t inferior thyroid artery, a branch of thyrocervical t r u n k of subclavian artery
lies laterally away f r o m midline and can thus escape injury.
10. Ans. is a, b and c i.e. a. Hemorrhage; Displacement of tube of obstruction; and Surgical emphysema
Ref. Dhingra 5th/ed p 339-340; Scotts Brown 7th/ed vol 2 p. 2300-2301; Current Otolaryngology 3rd/ed p. 542

C o m p l i c a t i o n s of T r a c h e o s t o m y

Most c o m m o n complication of tracheostomy is hemorrhage. The commonest cause of bleeding during tracheostomy is Anterior
jugular vein.
Other Immediate Complication of tracheostomy
• Air embolism • Apnea (due t o sudden release of retained C02)
• Cardiac arrest • Local damage t o structures
• Pneumothorax (d/t injury t o apical pleura)

INTERMEDIATE

During first few hours or days


• Dislodgement/Displacement of the tube •
• Surgical e m p h y s e m a :
- May occur as the air may leak into the cervical tissues.
- This is occasionally f o u n d in the immediate postoperative period.
- Presents as a swollen area around the root o f t h e neck and upper chest, which displays crepitus on palpation. It is due t o
overtight suturing o f t h e w o u n d and is not dangerous unless it leads to mediastinal emyphysema and cardiac tamponade.
• Pneumothorax/pneumomediastinum
• Tubal obstruction by Scabs/crusts
• Infection (tracheitis and tracheobronchitis, local w o u n d infection).
• Dysphagia:
- This is fairly c o m m o n in the first few days after tracheostomy.
- In normal swallowing a positive subglottic pressure is created by the closing o f t h e vocal cords - which is why one cannot
speak during swallowing.This is not possible w i t h a tracheostomy tube in place, and thus swallowing is incoordinate.
- Another reason for dysphagia is that if an inflatable cuff is blown up it will press on and obstruct the oesophagus.
Tracheal necrosis
! Tracheo arterial (Tracheal innominate artery fistula) /Tracheoeshophageal fistula
Recurrent laryngeal nerve injury. ii

LATE

Hemorrhage due t o erosion of major vessels


Stenosis o f the trachea (at the level of stoma)
Laryngeal stenosis due t o perichondritis of cricoid cartilage.
Difficulty w i t h decannulation
Tracheocutaneous fistula/scars.

According to Scott-Brown's 7th vol 2 p. 2301 - Tracheoarterial fistula / Tracheoesophageal fistula are intermediate complications and not late
complications like tracheocutaneous fistula.

11. Ans. is d ie Resection and end-to-end anastomosis.

Laryngeal Stenosis

M/C cause Endotracheal intubation followed by in tracheostomy


M/C site Subglottis at the level of cricoid cartilage

Mild stenosis (No cartilage involved) Repeated dilatation, removal of stenosis with C0 laser or intralesional steroid injection
2

Moderate stenosis Laryngotracheal reconstruction/self-expanding stents


Severe stenosis Partial cricotracheal resection + anastomosis
358 T SECTION VI Operative Surgery

12. Ans. is b i.e Tracheal stenosis Ref. Internet search: www.bcm.edu/oto/grand.htm


• Laryngotracheal stenosis treated by (serial dilatation using Jackson bronchoscopes/laser have tendency t o recur and hence
adjuvant methods are done t o decrease the rate of restenosis.
• Mitomycin C is an example of one of these adjunctive agents. It is a chemical derived from the Streptomyces caespitosus bacterium.
It is an alkylating agent which has both antineoplastic and antiproliferative properties and inhibits fibroblast production and
thus prevents restenosis.
13. Ans. is a, b and d i.e. Mixture of helium and oxygen; inert gas; and decreases airway resistance
Ref. Current Otolaryngology 3rd/ed p 538
- Heliox is a mixture of 8 0 % helium and 2 0 % oxygen
- It has low density and high viscosity which decreases airway resistance
- It converts turbulent flow at the site of obstruction t o a regular flow, which ensures better oxygen delivery to tissues and thus
serves as a temporary m e t h o d t o improve ventilation until definitive control of airway can be achieved.
14. Ans. is b i.e. Apical lobe of right lung
Foreign Body Aspiration
Supine position Erect or sitting position

• Right upper lobe posterior segment • Right posterior basilar segment of lower lobe
• Right lower lobe superior segment
• Left lower lobe superior segment
Site of aspiration and foreign body in lung depends upon position of patient due to anatomical elation of lung:
• If the patient has aspirated in upright or sitting position basilar segment of lower lobe is most likely to be involved
• In supine position either the posterior segment of upper (apical) lobe or superior segment of lower lobe is likely to be involved.
• In both cases right side is more likely to be involved due to straight and shorter course of right bronchi.

15. Ans. is c i.e Tracheal division and permanent tracheostome


Ref. Scotts Brown 7th/ed vol 1 pg. 1278; Internet search - www.bcn.edu/oto/grad
Aspiration is the passage of foreign material beyond the vocal cords:
• The larynx has 3 distinct functions - respiration, phonation and airway protection. Dysfunction of larynx can lead t o aspiration.
• The primary goal of treatment of aspiration is t o separate the upper digestive tract f r o m the upper respiratory tract for a short
period of t i m e or in some cases, permanently.
. There are 3 broad categories of treatment.
_ .... ._
Temporary/Adjunct Treatments

• Medical Therapy - in the f o r m of antibiotics is important t o prevent aspiration pneumonia.


• It is important t o make the patient NPO, t o avoid further aspiration and t o find an alternate feeding route t o maintain the patients
nutritional status. A nasogastric tube (feeding gastostomy/jejunostomy) is commonly placed, but this may actually increase the
aspiration reflux by making the lower esophageal and upper esophageal sphincters incompetent.

B u t Still

"Tubal feeding (either by nasogastric t u b e or gastrostomy) however is often unavoidable." - Scoffs Brown 7th/ed vol 1 pg. 1278
• Here it is important t o note that feeding Gastrostotomy/jejunostomy are not the gold standard methods of preventing aspiration
but rather are done t o maintain the nutritional status of patient and prevent further aspiration. In fact according t o most texts
- they are a c o m m o n cause of aspiration.
• Vocal cord medialization (by injecting Gel foam) is useful in unilateral paralysis.This is helpful but is rarely curative, if there is a
serious aspiration problem.
• Tracheostomy will often make aspiration worse by preventing laryngeal elevation on swallowing. It does however, allow easy
access t o the chest for suctioning. Even a cuffed tube doesn't prevent aspiration as secretions pool above the cuff and the seal
is never perfect" - Scoffs Brown 7th/ed vol 1 pg. 1278

Definite - Reversible Procedures

• Endolaryngeal stents: They function like a cork in the bottle. There j o b is t o seal the glottis and therefore thay need to be used
in conjunction w i t h a tracheostomy tube. But they are not often used as they are effective only as a short term solution, plus
there is risk of glottic stenosis.
• Laryngotracheal separation: The procedure involves transecting the cervical trachea and bringing o u t the lower end as a
permanent end stoma
• According to Scotts Brown and Internet sites: It is the procedure of choice as it is reversible. But it has disadvantage of sacrificing voice.
• Alternative procedure is Tracheoesophageal diversion b u t has higher complication rates.

-
CHAPTER 30 Important Operative Procedures

Definite - Irreversible Procedure

It includes: Narrow field laryngectomy: it was considered as a g o l d standard prior to 1970s, w h e n the irreversible procedures like
laryngo tracheal separation were not done.

Also Know

• Investigation of choice for diagnosing aspiration = Fibreoptic endoscopic evaluation of swallow (FESS)
• Videofluoroscopic modified Barium swallow (often called as ideofluoroscopy)
16. Ans. is a i.e. Bronchoscopy Ref. Scotts Brown 7th/ed vol 1 pg. 1188-1190; Dhingra 5th/ed p 344
• The peak incidence of inhaled foreign bodies is between the ages of one and three years w i t h a male t o female ratio of 2:1
• Only 1 2 % o f t h e inhaled bodies impact in the larynx while most pass t h r o u g h the cords into the tracheobronchial tree.
• In contrast t o adults, where objects tend t o lodge in the distal bronchi or right main bronchus, in children they tend to lie more
centrally w i t h i n the trachea (53%) or just distal t o the carina (47%)
• The treatment of choice for airway foreign bodies is p r o m p t endoscopic removal w i t h a Bronchoscope.
• "In children - The choice of either using a rigid or flexible endoscope remains controversial. Otolaryngologists traditionally
believe rigid endoscopes t o be the optimal instrument for tracheobronchial foreign bodies. However, there are certain objects
that may be more suitably removed w i t h flexible fiberoptic instruments or a combination of rigid and flexible techniques."
"The treatment of choice for airway foreign bodies is endoscopic removal w i t h a rigid instrument"-Nelson 18/ed pp 169,170
17. Ans. is c i.e. Vents Ref. Bronchology by Lukomsky, 40
Bronchoscope is similar t o esophagoscope, b u t has openings at the distal part o f t h e tube, called Vents which help in aeration of
the side bronchi.
18. Ans. is a i.e. Straight blade with uncuffed tube Ref. Scotts Brown 7th efvol 1 pg.511
Pediatric Airway Management - Equipment
• Tracheal intubation remains the standard for airway maintenance during many procedures.
• Generally, a tracheal t u b e of the largest possible internal diameter should be chosen t o minimize resistance to gas flow and
avoid an excessive leak around the tube. It is important, however, to avoid inserting too large tube, which may cause mucosal
damage.
The length ofthe tube is calculated as:

Length = + 12 cm For orotracheal intubation


Length = + 15 cm For nasotracheal intubation •

• Uncuffed tubes are used in children - as there is potential for mucosal damage w i t h the cuffed tubes (with high volume, low
pressure cuffs)
• In older children approaching puberty - Cuffed endotracheal tubes are used, reflecting the anatomical development o f t h e airway.
• Endotracheal tubes are available in a variety of materials although the use of PVC and silicone rubber is now almost universal.
• As far as blades are concerned - A huge range of laryngoscopes blades are available. Anatomical considerations and to some
extent personal choice, determine the most appropriate blade t o use. In general position o f t h e infant larynx and the long
epiglottis makes intubation easier w i t h a straight blade and are often used in children under 6 months of age.
So from above description, it is clear that in children straight blade with uncuffed tube is the best for intubation.
19. Ans. is b i.e. Heimlich's m a n e u v e r Ref. Dhingra 5/e, p 344, Scotts Brown 7th ed vol 1 p. 1188
• The child is presenting w i t h cyanosis and intercostal retraction which indicates that the foreign body is lodged in the larynx.
• Initial management for a foreign body lodged in trachea/larynx is Himlich's maneuver where a person stands behind the child
and places his arms around his lower chest and gives four abdominal thrust.
In infants, lying the child on its back on the adults knee and pressing firmly on the upper abdomen is the preferred maneuver.
If Heimlich's maneuvre fails, cricothyrotomy or emergency tracheostomy should be done.
Once acute respiratory'emergency is over foreign body can be removed by direct laryngoscopy or by laryngofissure, if it is
impacted.

• Tracheal and bronchial foreign bodies are removed by bronchoscopy with full preparation and under GA.
• Emergency removal of bronchial foreign bodies is not indicated.

20. Ans. is i.e. d i.e. Subcarinal lymph nodes Ref. Read below
• Carina - midline partition between the t w o bronchi is the first endobronchial landmark during bronchoscopy. Subcarinal
lymph nodes cannot be visualized on bronchoscopy but widening of carina is suggestive of subcarnial lymphadenopathy, and
pulsations o f t h e carina may be seen in aneurysm of arch of aorta
• Rest all structures viz. vocal cord, trachea and first segmental subdivision of bronchi can be visualized.
360 L SECTION VI Operative Surgery

Rigid bronchoscope visualises only up to segmental bronchus while it is possible to inspect the 2nd to 5th order subsegmental bronchi or beyond
using the flexible bronchoscope.

21. Ans. is c i.e. Active bleeding Ref. Tuli 1 st/ed p 529


Bronchoscopy is a procedure used for endoscopic examination of tracheobronchial tree.

C o n t r a i n d i c a t i o n s of B r o n c h o s c o p y

• Emergency bronchoscopy has no contraindication as it may be a lifesaving procedure.


• Elective bronchoscopy may have the following contraindications:
- General contraindications such as HT, DM, bleeding disorders, active infections.
- Trismus
- Aortic aneurysm
- Cervical spine problems
- Active recent massive hemoptysis
- Metastatic involvement of cervical spine
- Pulmonary hypertension.
Although, cardiac arrest has not been mentioned as one o f t h e contraindication but it is a very important complication of b r o n -
choscopy. Hence, in patients of cardiac arrest bronchoscopy should not be performed.

Bronchoscopy should always be preceded by laryngoscopy during which the subglottis should be examined.
22. A is b i.e. Foreign body aspiration
Foreign body aspiration is a very c o m m o n problem in pediatric age group (< 4 years). In the question, child is presenting w i t h
sudden onset respiratory distress and there is U/L decreased breath sounds + U/L wheezing and on chest X-ray a diffuse opacity
is seen on right side i.e. there is clinical and radiological evidence of bronchospasm and collapse suggestive of a foreign body in
bronchus
23. Ans. is d i.e. Heimlich's maneuver
Ref. Scotts Broun 7/e, p 1188-1191, Emergency medicine 6/e, p 68,69; Dhingra 5th/ed p344; Emergency medicine (American college of
Emergency Physicians) 6th/ed pp 68,69
• Aphonia (inability t o speak) and sudden respiratory distress in a young boy while having food, suggests obstruction o f t h e airway
w i t h a large bolus of food. Heimlich's maneuver is the recommended, initial procedure of choice for relieving airway obstruction
due t o solid objects.
• Cricothyroidotomy or tracheostomy should be performed if the Heimlich's maneuver fails
24. Ans. is a and b i.e. Cervical spine rigidity; and Aortic aneurysm Ref. Dhingra 5th/edp436
Contraindications of esophagoscopy (rigid type):
Trismus
• Aneurysm of aorta
• Receding mandible
• Advanced heart, liver, kidney diseases (relative contraindication).
• Diseases of cervical spine, e.g. cervical trauma, spondylitis, TB, osteophytes, kyphosis, etc.
25. Ans. is a i.e. Tonsillectomy approach Ref. Dhingra 5th/edp 438
Tonsillecotmy is done as a part o f t h e following operations:
• Palatopharyngoplasty wjnich is done for sleep apnea syndrome
• Glossopharyngal neurectomy—tonsil is removed first and t h e n IX nerve is severed in the bed of tonsil
• Removal of styloid process.
RECENT LATEST PAPER

PGI-Nov 2012
AIIMS - May 2013
PGI-May 2013
PGI-May 2012
-

Latest Paper

PGI-NOV 2012
1. W h i c h i n t e r v e n t i o n is best in patients o p e r a t e d for 3. What are the boundaries of Trauttmann's triangle:
bilateral acoustic neuroma for hearing rehabilitation: a. Bony labyrinth anteriorly
a. .Brainstem hearing implant b. Bony labyringh posteriorly
b. Bilateral cochlear implant c. Sigmoid sinus posteriorly
c. Unilateral cochlear implant d. Sigmoid sinus anteriorly
d. High power hearing aid e. Superior petrosal sinus superiorly
e. Myringoplasty 4. Perforation of palate is/are seen with:
2. All ofthe following constitute supraglottic cancer except: a. Minor aphthous ulcers
a. Vallecula b. Major aphthous ulcers
b. Lower border of the cricoids c. Tertiary syphilis
c. False vocal cords d. Cocaine abuse
d. Aryepiglottic fold
e. Posterior commissure

EXPLANATIONS AND REFERENCES


Ans. a i.e. Brainstem hearing implant Ref. Dhingra 6th/edp 127

Auditory brainstem implant (ABI)


"This implant is designed to stimulate the cochlear nuclear complex in the brainstem directly by placing the implant in the lateral recess
ofthe fourth ventricle. Such an implant is needed when CN VIII has been severed in surgery of vestibular schwannoma. In these cases,
cochlear implants are obviously of no use"
"In unilateral acoustic neuroma, auditory brainstem implant (ABI) is not necessary as hearing is possible from the contralateral side but
in bilateral acoustic neuroma as in neurotibromatosis-2, rehabilitation is required by ABI" Dhingra 6th/ed p 127
Note: Brainstem implant is currently used only in patients w i t h NF-2 and is always implanted simultaneously w i t h t u m o r removal
(usually during excision of the patient's second tumor). It is useful in patients who have had both cochleovestibular nerves sacrificed,
since this implant stimulates the cochlear nuclear complex in the brainstem directly.
Ans. a, b and c i.e. a. Vallecula; b. Lower border of the cricoids; c. Posterior commisure
"The supraglottic includes the laryngeal surface of the epiglottis, aryepiglottic fold, arytenoids, false cords and ventricle. The lingual
surface ofthe epiglottis and valleculae are in the oropharynx. The glottis comprises the vocal cords and the anterior and posterior com-
missures. Subglottis is a small area extending from the lower border ofthe cricoids to the under surface ofthe vocal cords"
—Logan Turner 10th (1st Indian edition/171)
364|_ SECTION VII Recent Latest Paper

According to Dhingra:—
Table: AJCC 2002 classification of carcinoma larynx [Dhingra 6th/ed p 307]

Subsite
Supraglottic • Suprahyoid epiglottis
• Infrahyoid epiglottis
• Aryepiglottic folds (laryngeal aspect only)
• Arytenoid
• Ventricular bands (or false cords)
Glottis True vocal cords including anterior and posterior commissure
Subglottis Subglottis up to lower border of cricoids cartilage
Ans. is a, c and e, i.e. a. Bony labyrinth anteriorly; c. Sigmoid sinus posteriorly; e. Superior petrosal sinus superiorly
Ref. Dhingra 6th/ed p 450 point 122
"Trautmann's triangle is bounded by the bony labyrinth anteriorly, sigmoid sinus posteriorly and the dura or superior petrosal sinus
superiorly" —PL Dhingra 6th/edp 450 point 122
Anterior
Up - j - D o w n
Posterior
II?// \ \ ^
iff / i ;
' n C U S
~~ 7/ / \V - CN VII

Trautmann's
triangle Site of
Sinodural
(Citelli's) endolymphatic sac
angle
Digastric ridge
Sigmoid sinus plate

Ans. is c and d i.e. c. Tertiary syphilis; d. Cocaine abuse Internet search

Causes of palatal perforation


• Developmental: During the sixth week o f prenatal period, palatal shelve coalesce t o f o r m the hard palate. Failure t o this
integration results in cleft palate. Some syndromes, maternal alcohol consumption and cigarette smoking, folic acid deficiency,
corticosteroid use and anticonvulsant therapy are some causative agents for this abnormality.
• Infectious: Leprosy, tertiary syphilis, tuberculosis, rhinoscleroderma, naso-oral blastomycosis, leishmaniasis, actinomycosis,
histoplasmosis, coccidiomycosis and diphtheria.
• Autoimmune: Lupus erythematosus, sarcoidosis, Crohn's disease and Wegener's granulomatosis.
• Neoplastic: Different tumors can extend f r o m maxillary sinus or nasal cavity and perforate the palate. Although these neoplasms
usually f o r m a mass, but in advanced cases perforation o f palate may occur in course of disease or following treatment.
• Drug related: Palatal perforation due t o cocaine abuse is a well-known situation. Other durgs (heroine, narcotics) can be
responsible for palatal perforation.
• Iatrogenic: Sometimes following a t o o t h extraction an oro-antral fistula remains. Other procedures such as t u m o r surgery
(maxillectomy), corrective surgeries (e.g. septoplasty) or intubation can cause palatal perforation.
• Rare causes: Rhinolith can result in palatal perforation. Patients w i t h psychologic problems may present w i t h a fictitious palatal
perforation.
Note: Apthous ulcers involve soft palate whereas spare the mucosa of hard palate and gingivae.

-
Latest Paper

AIIMS-MAY 2013
1. 75 year old diabetic patient with granulation tissue at a. Keratosis obturans
external auditory canal, diagnosis is? b. CSOM
a. Malignant otitis externa c. External otitis
b. Keratosis obturans d. Carcinoma of external auditory canal
c. Squamous cell carcinoma of ear canal Mr. R a m u p r e s e n t e d w i t h p e r s i s t e n t e a r p a i n a n d
d. Simple wax discharge, retro-orbital pain, modified radical
2. True About BAHA mastoidectomy was done to him. Patient comes back
a. Useful in canal atresia and microtia with persistent discharge, what is your diagnosis?
• b. Useful in bilateral severe SNHL a. Diffuse serous labyrinthitis
c. Useful after surgery in neurofibromatosis 2 for acoustic b. Purulent labyrinthitis
' neuroma c. Petrositis
d. It can bypass cochlea d. Latent mastoiditis
3. A 70 year old man presented with left sided conductive A child with features of upper respiratory infection,
h e a r i n g l o s s , o/e T M i n t a c t a n d T y p e B c u r v e o n on investigations is found to have ' t h u m b p r i n t sign,
tympanogram. Next step is: diagnosis is:
a. Myrinogotomy and grommet insertion a. Acute larynagotracheobronchitis
b. Conservative management

b. Acute epiglottitis
c. Type 3 tympanoplasty c. Acute laryngeal diphtheria
d. Endoscopic examination t o look for nasopharyngeal d. Laryngomalacia
causes True regarding Bell's palsy is all except
60 year old man presented with left sided ear discharge a. Steroids are used
for 7 y e a r s w i t h d u l l e a r a c h e . O/e intact t y m p a n i c b. U/L facial weakness
membrane on both sides, mass is seen in the posterior c. Role of herpes simplex in etiology
canal wall on left side. Diagnosis is? d. Immediate surgical decompression is required

EXPLANATIONS AND REFERENCES


1. Ans is a i.e. Malignant otitis externa Ref. Dhingra 6th/ed p 52
Granulation tissue in a diabetic patient in external auditory canal indicates malignant otitis externa.
For details see chapter - Diseases of External ear o f t h e guide.
2. Ans is a i.e. useful in canal atresia and microtia Ref. Dhingra 6th/ed p 122-123
BAHA: Bone anchored hearing aid is a type of hearing aid which is based on the principle of bone conduction. BAHA uses a surgi-
cally implanted a b u t m e n t t o transmit sound by direct conduction t h r o u g h bone t o cochlea, bypassing the external auditory canal
and middle ear.

Indications for BAHA


• When air-conduction (AC) hearing aid cannot be used:
- Canal atresia, congenital or acquired, not amenable to treatment.
- Chronic ear discharge, not amenable to treatment.
- Excessive feedback and discomfort from air-conduction hearing aid.
• Conductive or mixed hearing loss, e.g. otosclerosis and tympanosclerosis where surgery is contraindicated.
• Single-sided hearing loss.

Note: For severe bilateral SNHL—Cochlear implant is used.


For bilateral acoustic neuromas in Neurofibromatosis - auditory brainstem implant is used.
Ans is d i.e. endoscopic examination to look for nasopharyngeal causes Ref. Dhingra 6th/edp 251
A 70 year old man is presenting w i t h U/L conductive deafness and on O/E—Tympanic membrane is intact (i.e. any otology cause
for conductive deafness ruled out) andType B tymparogram (i.e. serous otitis media is present which has t o be due t o a cause other
t h a n ear because tympanic membrane is intact).
Always R e m e m b e r -
"Presence of a unilateral serous otitis media is an adult should raise suspicion of nasopharyngeal growth." Dhingra 6th/edp 251
Answer is a, i.e. Keratosis obturans Ref. Dhingra 6th/ed p 108,54
Explanation:
Presence of mass seen in the ear canal, w i t h a long history of symptoms rules o u t carcinoma of ext. auditory canal.
366 [_ SECTION VII Recent Latest Paper

Option D:—
• Carcinoma of external ear canal
Squamous carcinoma is the most frequent neoplasm in the external auditory canal (EAC), about four times more c o m m o n than
basal carcinomas.This ratio is reversed in the pinna.
Basal cell carcinoma, adenocarcinoma, ceruminoma, and malignant melanoma are the other types of cancers seen in external
auditory canal.
Most squamous cell carcinomas occur in the fifth and sixth decades of life. Foul smelling blood stained discharge is the
primary symptom, and there is severe otalgia, hearing loss, and bleeding.
These tumors have an aggressive nature and spread along preformed vascular and neural pathways, invading adjacent
structures like facial nerve labyrinthine, cranial nerves IX, X, XI and XII. Treatment usually combines surgery w i t h free margins
and radiotherapy.
Duration o f t h e symptoms being 7 years and these features not occurring, rules out this option.
• Option b: CSOM
Normal tympanic membrane and absence of deafness are against CSOM.
• Option c: Presence of mass does not support the diagnosis of chronic external otitis.
• Thus by exclusion our answer is keratosis obturans (option a).
• Keratosis Obturans: Also known as canal wall cholesteatoma.
It is seen c o m m o n l y in younger age groups, due t o defective epithelial migration f r o m the tympanic membrane to posterior
meatal wall, which results in collection of pearly white epithelial debris in deep meatus.
It can cause pressure effects, enlargement of bony canal and sometimes facial palsy.
Usually patients w i t h conductive deafness and earache.
Treatment is removal of entire keratotic mass.
Recurrence is c o m m o n .
5. Answer is c, i.e. petrositis Ref. Dhingra 6th/ed p 79
In the question patient is a case of CSOM, w i t h local spread of infection.
Dhingra clearly mentions in a patient w i t h CSOM, persistent ear discharge w i t h or w i t h o u t deep seated pain in spite of an
adequate cortical or modified mastoidectomy points towards petrositis.
Persistent ear discharge w i t h or w i t h o u t deep seated pain in spite of an adequate cortical or modified radical mastoidectomy
also points t o petrositis.
Petrositis: Important Points
• Spread o f infection f r o m middle ear and mastoid t o the petrous part of temporal bone is petrositis
• It can also involve adjacent 5 cranial nerve and 6 cranial nerve w h e n it produces classical triad of symptoms - 6 nerve palsy,
th th th

retro orbital pain (5 nerve) and persistent discharge f r o m the ear, known as Gradenigo's syndrome
th

Note: All the three classical components of Gradenigo's syndrome are not needed for diagnosing petrositis.
• Treatment
Adequate drainage is the mainstay of treatment along w i t h specific antibiotic therapy. Modified radical or radical mastoidectomy
is often required if not done already. The fistulous tract should be identified, curetted and enlarged t o provide free drainage.
6. Answer is b, i.e. a/c epiglottitis Ref. Dhingra 6th/edp 289-290
T h u m b p r i n t sign, is a classic radiologic sign found on a lateral cervical-spine radiograph suggestive of eniglottitis.This sign is caused
by the thickened free edge o f t h e epiglottis.
Therefore our answer is b, i.e. acute epiglottitis.
7. Answer is d, i.e. Immediate surgical decompression is required
Explanation:
Surgical decompression of facial nerve in Bells palsy is needed only in indicated cases, i.e.
• In case of complete paralysis and
• If electroneurography of facial nerve shows > 9 0 % o f degeneration. When done it should be w i t h i n 2 weeks of onset of palsy.
Rest all options are correct and have been discussed in detail earlier.

Surgical decompression of facial nerve


Approaches: Transmastoid, transcanal, via middle fossa
All indications of facial nerve decompression
• Complete paralysis ( > 9 0 % by ENG in case of Bell's palsy)
• Tumours of facial nerve
• Cholesteatoma causing facial palsy
• Traumatic facial palsy.
Latest Paper

S I
PGI-MAY 2013
Method of speech communications after laryngectomy 5. Most common cause of B/L recurrent laryngeal paralysis:
include: a. Thyroid surgery
a. Electrolarynx b. Cancer cervical oesophagus
b. Oesophageal speech c. Blow from nasal cavity
c. Tracheo-oesophageal speech d. Thyroid cancer
d. Tracheal speech e. Bronchogenic carcinoma
e. Transoral pneumatic device 6. True about otosclerosis:
2. Most common site of laryngeal papilloman in adult: a. Most common site is footplate of stapes
a. Anterior commissure b. More common in female
Posterior commissure c. Schwartz sign indicate active focus
Anterior half of vocal cord d. Autosomal recessive
Middle of vocal cord e. Corhort not becomes negative aftersuccessful stapedectomy
False vocal cords Most common location of nasal hemangioma:
Most common site of vocal nodule of larynx: a. Nasal Septum
a. Anterior part of epiglottis Inferior turbinate
False vocal folds Vestibule •
Anterior commisure Uncinate process
Posterior commisure Nasopharynx
On true vocal cord at junction of A 1/3 with P 2/3 8. Veins not involved in spreading infection to cavernous
True about benign paroxysmal positional vertigo sinus from danger area of face:
a. Hearing loss is often present a. Lingual vein
Most commonly seen in 2 decadend
Pterygoid plexus
Hallpike manuever is not helpful in diagnosis Facial vein
Epley maneuver is used for treatment Ophthalmic vein
Disorder of posterior semicircular canal Cephalic vein
•:

EXPLANATIONS AND REFERENCES


1. Ans. is a, b, c and e i.e. Electrolarynx, oesophageal speech, transoesophageal speech and transoral pneumatic device.
Ref. Dhingra 6th/edp 312

Methods of communication in laryngectomized patients


• Oesophageal speech
• Electrolarynx
• Transoral pneumatic device
• Tracheo-oesophageal speech
- Blom-Singer prosthesis
- Panje prosthesis

2. Answer is a and c i.e. anterior commissure and anterior half of vocal cord Ref. Dhingra 6th/ed p 306
A d u l t onset papilloma usually arise from t h e anterior half of t h e vocal cord or anterior commissure.
They are usually single, smalljn size, less aggressive and d o not recur after surgical removal.
M/C in males (2:1), in age g r o u p 30-50 year.
3. Ans is e i.e. or true vocal cord at function of anterior 113 with posterior 2/3. Ref. Dhingra 6th/ed p 303
Vocal nodules appear symmetrically on t h e free edge of vocal cord, at the j u n c t i o n of anterior one-third w i t h t h e posterior two-
thirds, as this is t h e area of maximum vibration o f t h e cord and thus subject t o maximum trauma.
4. Answer is d a n d e i.e. Epley maneuver is used for treatment and disorder of posterior semicircular canal.
BPPV:
• It is characterised by vertigo w h e n head is placed in a certain condition.
• There is no hearing loss or other neurologic symptoms.
• It occurs as a result of disorder of posterior semi-circular canal t h o u g h patients may have history of head trauma and ear infection.
SECTION VII Recent Latest Paper

• Within the labyrinth o f t h e inner ear lie collections of calcium crystals known as otoconia or otoliths. In patients w i t h BPPV, the
otoconia are dislodged f r o m their usual position w i t h i n the utricle and migrate over time into one o f t h e semicircular canals
(the posterior canal is most commonly affected due to its anatomical position).
• When the head is reoriented relative t o gravity, the gravity-dependent movement o f t h e heavier otoconial debris (colloquially
"ear rocks") within the affected semicircular canal causes abnormal (pathological) fluid endolymph displacement and a resultant
sensation of vertigo.
• A l t h o u g h BPPV can occur at any age, it is most often seen in people over the age of 60.
• The vertigo is fatigneable on assuming the same position repeatedly b u t can be induced again after a period of rest.
• Diagnosis is by-Typical history and by performing the Hallpike maneuver.
• Management is by Epley maneuver. The principle o f this maneuver is t o reposition the otoconial debris from the posterior
semicircular canal to utricle.
5. Ans is a, i.e. Thyroid surgery Ref. Dhingra 6th/edp 300
Bilateral Recurrent Laryngeal Paralysis:
"Neuritis or surgical trauma (thyroidectomy) are the most important causes of bilateral abductor paralysis or recurrent laryngeal nerve
paralysis.." Dhingra 6th/ed p 300
Other causes of B/L Recurrent laryngeal Nerve:
• Carciroma thyroid
• Cancer cervical oesophagus
• Cervical lymphadenopathy
6. Is b, c and e, i.e. more common in females, Schwartz sign indicates active focus and Carhart's notch becomes negative after
successful stapedectomy Ref. Dhingra 6th/edp 86-87
Otosclerosis:
• M/C site: Anterior to oval w i n d o w (fissula ante fenestrum). (thus option a is correct)
• M/C is females (option b is correct)
• 5 0 % cases have a positive family history. Genetic studies reveal it is autosomal dominant trait, (option d incorrect).
• In 1 0 % cases: Tympanic membrane may exhibit reddish hue on the promontory—"schwartze sign" which is indicative of active
focus w i t h increased vascularity, (option is correct).
• Generally in otosclerosis: Bone conduction is normal b u t in some cases there is a dip in bone conduction which is m a x i m u m
at 2000 Hz and is called Cahart's Notch.
Cahart's Notch disappears after successful stapedectomy (option e is correct).
7. Answer is a, i.e. septum Ref. Dhingra 6th/edp 203; Mohan Bansal Ist/edp 354
Nasal Hemagiomas:
• Capillary hemangioma: This bleeding polyp o f t h e anteroinferior part of nasal septum is a soft, dark red pedunculated or sessile
tumor.
- It presents w i t h recurrent epistaxis and nasal obstruction.
- This smooth g r o w t h may become ulcerated.
Treatment: It needs local excision w i t h a cuff of surrounding mucoperichondrium.
• Cavernous hemangioma: It arises f r o m the turbinates.
Treatment: It is treated by surgical excision w i t h preliminary cryotherapy. Extensive lesions may need combined radiotherapy
and surgical excision.
Now since the question says, M/C site: we will have t o choose between nasal septum (site for capillary hemangioma) and turbinate
(site for cavernous hemangioma).
Capillary hemangioma is more c o m m o n type oft nasal hemangioma hence M/C site is nasal septum as is indicated by the
following lines:—
• Haemangiomas are benign vascular tumours, which originate in the skin, mucosae and deep structures such as bones, muscles
and glands. They are of t w o major types, capillary and cavernous. When these neoplasms rarely arise in the nasal cavity, they
are predominantly capillary and are found attached to the nasal septum. Cavernous haemangiomas, on the other hand, are more
likely t o be f o u n d on the lateral wall ofthe nasal cavity.

8. A n s i s a i . e . lingual vein and e i.e. cephalic vein Ref. BDChaurasiap 62-63; Maqbool 11 th/edp 172

Dangerous area of face


Dangerous area of face includes upper lip and anteroinferior part of nose including the vestibule. This area freely communicates
w i t h the cavernous sinus t h r o u g h a set of valveless veins, anterior facial vein and superior ophthalmic vein. Any infection of this
area can thus travel intracranially leading t o meningitis and cavernous sinus thrombosis.
Latest Paper

Vein draining dangerous area M. Maqbool 11 th/edp 172


• Through facial veins communicating w i t h ophthalmic veins (both having no valve)
• Through the pterygoid plexus of veins which communicate w i t h facial vein on one hand and the cavernous sinus t h r o u g h
emissary vein on the other hand.
According t o B.D. Chaurasia Vol 3,5th/ed p 62
Deep connections of the facial vein include:
• A communication between the supraorbital and superior ophthalmic veins.
• Another w i t h the pterygoid plexus t h r o u g h the deep facial vein which passes backwards over the buccinator.
The facial vein communicates w i t h the cavernous sinus through these connections. Infections from the face can spread in a retrograde
direction and cause thrombosis of the cavernous sinus. This is specially likely t o occur in the presence of infection in the upper lip
and in the lower part o f t h e nose. This area is, therefore, called the dangerous area ofthe face.

-
SECTION VII Recent Latest Paper

PGI-MAY2012
All are true statement about tracheostomy and larynx in Posterior crioarytenoid
children except: Lateral cricoarytenoid
a. Omega shaped epiglottis Thyroarytenoid
b. Laryngeal cartilages are soft and collapsable Cricothyroid
c. Larynx is high in children Vocalis
d. Trachea can be easily palpated Otitis media with effusion is also known as:
e. Avoid too much extension of neck during positioning a. Serous otitis media
Which of the following procedure is helpful in diagnosis Suppurative otitis media
of choanal atresia: Mucoid otitis media
a. Anterior rhinoscopy Glue ear
b. Passing red rubber catheter Secretary otitis media
c. Breath sounds by stethoscope 5. Tests of balance include(s): •

d. Endoscopy of nose a. Dysdiadochokinesia


e. Acoustic rhinometry b. Romberg's sign
3. A patient met with recurrent laryngeal nerve palsy while c. Weber's test
u n d e r g o i n g thyroid surgery. Which of the following d. Unterberger's test
muscles of larynx is/are affected; e. Finger nose test

EXPLANATIONS AND REFERENCES


1. Ans is d i.e. Trachea can be easily palpated Ref. Dhindra 6th/edp285; Logan and Turner lOth/edp 396
Infant's larynx differs from adult in:
• It is situated high up (C2-C4). (in adults = C 3 - C 6 )
Q

• Of equal size in both sixes (in adults it is larger in males)


• Larynx is funnel shaped
• The narrowest part o f t h e infantile larynx is the j u n c t i o n of subglottic larynx w i t h trachea and this is because cricoid cartilage
is very small
• Epiglottis is omega shaped, soft, large and patulous.
• Laryngeal cartilages are soft and collapse easily
• Short trachea and short neck.
• Vocal cords are angled and lie at level of C4
• Trachea bifurcates at level of T2
• Thyroid cartilage is flat.The cricothyroid and thyrohyoid spaces are narrow.

T r a c h e o s t o m y in I n f a n t s a n d C h i l d r e n Dhingra 5th/edp338
"Trachea of infants and children is soft and compressible and its identification may become difficult and the surgeon may easily displace
it and go deep or lateral t o it injuring recurrent laryngeal nerve or even the carotid."
"During positioning, do not extend too much as this pulls structures f r o m chest into the neck and thus injury may occur t o pleura,
innominate vessels and thymus or the tracheostomy opening may be made t w o o low near suprasternal notch"

T r a c h e o s t o m y in I n f a n t s a n d C h i l d r e n Logan and Turner 10th/edp396


"The incision is a short transverse one, midway b/w lower border of thyroid cartilage and the suprasternal notch. The neck must be well
extended"
"A incision is made through two tracheal rings, preferably the third or fourth."
2. Ans is b, c, d and e i.e. Passing red rubber catheter, breath sounds by stethoscope, endoscopy of nose and acoustic rhinometry
"Structure normally seen on posterior rhinoscopy are choanal polyp or atresia" Dhingra 5th/edp38S
"Choanal atresia: Posterior rhinoscopy may be undertaken in older children and will show the occlusion."
Logan and Turner 10th/edp380
Thus posterior rhinoscopy and not anterior rhinoscopy are useful in the diagnosis of choanal atresia.
"Acoustic rhinometry is a new technique which evaluates nasal obstruction by analysing reflections of a sound pulse introduced via
the nostrils. The technique is rapid, reproducible, non-invasive and requires minimal cooperation f r o m the subject. A graph of nasal
cross-sectional area as a function of distance f r o m the nostril is produced, f r o m which several area and volume estimates o f t h e
nasal cavity can be derived. The role of acoustic rhinometry in diagnosis is somewhat limited compared t o nasal endoscopy, but it
Latest Paper

is useful for nasal challenge and for quantifying nasal obstruction. It is helpful in evaluating childhood nasal obstruction, as it is well
tolerated by children as young as 3 years old-a group of patients t o w h o m objective tests have hitherto been difficult t o apply."
—www.ncbi.nim.nih.gov/.JPMC 129

Choanal Atresia
It is due t o persistence of bucconasal membrane and may be unilateral or bilateral, complete or incomplete, bony (90%) or
membranous (10%). Unilateral atresia is more c o m m o n and may remain undiagnosed until adult life. Bilateral atresia presents
w i t h respiratory obstruction as the newborn, being a natural nose breather, does not breathe f r o m m o u t h . Diagnosis of
choanal atresia can be made by (i) presence of mucoid discharge in the nose, (ii) absence of air bubbles in the nasal discharge,
(iii) failure t o pass a catheter f r o m nose t o pharynx, (iv) p u t t i n g a few drops of a dye (methylene blue) into the nose and seeing
its passage into the pharynx, or (v) flexible nasal endoscopy, (vi) installing radio-opaque dye into the nose and taking a lateral
f i l m , and (vii) c o m p u t e d t o m o g r a p h y (CT) scan in axial plane is more useful.

3. Ans is a, b, c and e i.e. Posterior crioarytenoid, lateral cricoarytenoid and thyroarytenoid, Vocalis Dhingra6th/edp298
All muscles which move the vocal cord (abductors, adductors or tensors) are supplied by the recurrent laryngeal nerve except the
cricothyroid muscle which is supplied by external laryngeal nerve (a branch o f superior laryngeal nerve).
4. Ans is a, c, d and e i.e. Serous otitis media, mucoid otitis media, glue ear and secretary otitis media Dhingra 6th/edp 64
Otitis media w i t h effusion is also called as serous otitis media, secretory otitis media, mucoid otitis media and glue ear.
5. Ans is a, b, d and e i.e. Dysdiadochokinesia, romberg sign, unterberger test and finger nose test Ref. Internet
Weber test is for hearing and n o t for balance. All the tests given in the options are tests for balance.
Tests for balance
• Romberg test
• Unterberger test
• Positional test
- Tandem walking
- Finger nose test
- Finger t o finger test
- Dysdiadochokinesia
- Post-pointing and falling

The Unterberger test, also Unterberger's test and Unterberger's stepping test
• It is a test used in otolaryngology t o help assess whether a patient has a vestibular pathology. It is not useful for detecting
central (brain) disorders of balance.
• M e t h o d : Stepping on one spot w i t h the eyes closed.
• Result:
- Peripheral lesions: rotation o f t h e body axis t o the side o f t h e labyrinthine lesion.
- Central disorders: the deviation is irregular.
- Deviations of greater than 40 degree are significant.
• If the patient rotates to one side they may have a labyrinthine lesion on that side, but this test should not be used t o diagnose
lesions w i t h o u t the support of other tests.

-
Color Plates

-
IMPORTANT PICTURES FOR PICTORIAL QUESTIONS
EAR
1. Auricular cartilage: external features 4. Parts of middle ear cleft
Aditus ad antrum
External auditory meatus Attic
(Epitympanum)
Scaphoid fossa S
Triangular fossa ' Eustachian tube
Cymba conchae
Auricular (Darwin's) tubercle

Concha
Helix
Antihelix
Tragus
Intertragic notch 5. Parts of middle ear as in seen on coronal section
Tegmen tympani

Epitympanum
Antitragus (Attic)
Ossicles
Malleus
Lobule Incus
Tympanic Stapes
2. Nerve supply of Pinna membrane Oval window
between Promontory
external
Mesotympanum
and middle ear

Hypotympanum

6. Mc-Ewan triangle: Surface landmarkfor mastoid antrum

A. Lateral surface B. Medial surface

3. Tympanic membrane as seen on otoscopy


Tympanomastoid
Tympanic bone suture

Middle ear ossicles


-Short
Head 4 process
Malleus Incus
Neck ^Body
Processes
- Anterior-

- Round
window

Umbo Cone of light


Anterior crus
Posterior crus
Footplate

it
Section VIII Color Plates

8. Cochlea: Peri and endolymphatic systems: 11. Structure of Organ of Corti


relation Outer hair

Stapes covering ovai window Inner


cells
Perilymph in scala vestibuli

Endolymph in scaia media

Perilymph in scaia tympani

Deiter's^
Round •—Basilar c e l l s

window - Aqueduct of cochlea membrane


Nerve fibers
Helicotrema Cochlear nerve (unmyelinated)
fibers (myelinated)

12. Central auditory pathway

9. Medial wall of left bony labyrinth seen from lateral side Auditory
cortex in
after the removal of its lateral wall temporal
lobe
Spherical recess Elliptical recess for utricle Medial
for saccule Auditory radiations '
Posterior in sublentiform part geniculate body
Cochlea of internal capsule
Semicircular canals
Scala vestibuli
Superior Mid brain
Osseous spiral lamina
(anterior)
Scala Posterior
tympani Lateral
(horizontal)
Anterior Crus commune

Ampullated ends Pons


Fenestra
Opening for
Cochlear aqueduct cochleae endolymphatic duct

Cochlea
10. Structure of cochlear canal after its cut section
13. Vestibular pathway

CN I I I — V / Nucleus CN IV
Cerebellum ?S r ® -
i ®> Medial longitudinal
Stria vascularis CNVH i - bundle

Superior nucleus

Lateral nucleus • Vestibular


Medial nucleus nuclei

Inferior nucleus

Osseous spiral
lamina
Vestibulospinal tract
Vestibular ganglion
Color Plate

14. Aco ustic reflex pathway 16. Symbols used in audiogram charting
Cochlea Ear
Modality
Right Left
AC unmasked O X
AC masked A •
>
BC unmasked
<
BC masked •
No response P

17. Audiogram of left normal ear

Audiogram Left

0 7~
< ^

10 7! 7" 7
I 7
20
| 30

| 40
I 50
J2 60
to
% 70

80
! 90
100
110

.25k ,5k 1k 1.5k 2k 3k 4k 6k 8k


In normal persons, hearing threshold values w i t h both air and bone
remain between 0 and 10 dB
18. A u d i o g r a m o f left e a r w i t h c o n d u c t i v e hearing
loss
A. Normal w i t h normal latency Audiogram Left
0
\ _ \
•--> •-->
\.
10 V
7
— 20
CO.
1 30
1 40
2 50
< < )r
I 60
S 70
| 80
TO
o 90
100
110
.25k ,5k 1k 1.5k 2k 3k 4k 6k 8k
In this graph, bone-air gap is seen which means a patient can hear
B. BERA in severe h e a r i n g less. N o t e : N o peaks seen
by bone under 10-20 dB, while w i t h air hearing is much below,
depending on the severity, indicating conductive hearing loss.
Section VIII Color Plates

19. Audiogram of left ear with SNHL 20. Audiogram in Early case of noise-induced hearing loss.

Audiogram Left Frequency in Hertz


125 250 500 1000 2000 4000 8000
Of
10 0
20 10

| 30 20
A
—/•\ X
| 40
S 30

IT* .= 40 \>
I 50
8 50
° 60
(n I 60

I 0)
-7T
70 70
X
E 80

S 90 80
I
90
100
100
110 In acoustic
110 trauma, there is a sudden dip at 4000 Hz b o t h in air and
.25k .5k 1k 1.5k 2k 3k 4k 6k 8k bone conduction values

In SNHL, both bone and air conduction values are decreased and
may even overlap each other.

21. Alternate binaural loudness balance test

Right Left Right

90

(A) Nonrecruiting ear. The initial difference of 20 dB between the right and left ear is maintained at all intensity levels.
(B) Recruiting ear right side. At 80 dB loudness perceived by right ear is as good as left ear t h o u g h there was difference of 30 dB initially
Color Plate

2 2 . T y p e s of T y m p a n o g r a m : I m p e d e n e A u d i o m e t r y C u r v e s :

A5\
S
s.

-200 -100 0 +100 +200 -200 -100 0 +100 +200

Type A Type B
A- Normal B-flat or dome shaped audiogram
(middle ear fluid)

-200 -100 0 + 1 0 0 +200 -200 -100 0 + 1 0 0 +200


mmH,0 mmPU
Type C Type D

Types of curve Conditions seen in


A curve Normal

(Normal peak height and pressure Eustachian tube obstruction


Otosclerosis0

As curve 0

Tumors of middle ear


(It is also a variant of normal tympanogram Fixed malleus syndrome
but may be shallow Tympanosclerosis
Ad curve Ossicular discontinuity
Post stapedectomy
(Variant of normal with high peak) Monometric ear drum
Fluid on middle ear 0

B curve Secretory otitis media 0

Tym+anic membrane perforation 0

(Flat or dome-shaped curve) 0

Grommet in ear 0

Indicating lack of compliance


C curve Retracted tympanic membrane
(negataive peak pressure) Faulty function of Eustachian tube/ Eustachian tube obstruction
viii T Section VIII Color Plates

23. Incisions for myringotomy

In case of Acute Suppurative Otitis Media (ASOM) In case of Serous Otitis Media + grommet insertion

23. Different view of X-ray for diseases of the ear

(A) Towne's (Fronto-occipital): (B) Submenu

NOSE
1. Openings of paranasal sinus as in lateral wall of noseafter 3. Tripod fracture

2. Blood supply of nasal septum


Left zygoma (tripod) fracture showing three sites of fracture.
(1) Zygomaticofrontal: (2) Zygomaticotemporal; (3) Infraorbital
-

Color Plate J IX

4. Le fort classification of fracture of nasomaxillary complex 5. Ohngrens classification for malignant neoplasm of PNS

Le Fort classification o f fractures o f nasomaxillary c o m p l e x Ohngren's classification: Ohngren's line is an imaginary line
crossing nasal s e p t u m and p t e r y g o i d plates. (I) Transverse (OL), w h i c h extends between medial canthus and the angle of
(separating maxillary dentition); (II) Pyramidal (fracture of root of mandible, divides the maxilla into t w o regions anteroinferior (Al)
nose, medial wall and floor of orbit and maxilla), (III) Craniofacial and posterosuperior (PA). Al growths are easy t o manage and have
disjunction (separating face f r o m the cranium) better prognosis than PS tumors

6. Structures seen an posterior rhinoscopy 7. A radiopaque foreign body in the nose of a child
Adenoids
Posterior . i _ , Superior
free margin V1,rf^n^N turbinate
of septum

Superior
meatus

Middle - T . ^
meatus \ ^

Pharyngeal
opening of
eustachian
tube

8. X-ray: PNS, Water's view

Frontal intersinus

orbital wall j tBSsH


Sup. orbital rim,

Maxillary s i n u s
Roof
Nasoantrai wall
"Maxillary sinus

-
Section VIII Color Plates

9. View for the paranasal sinuses

A. B. C.
Radiology of nasal structures: (A) Occipitomental view: (B) Occipitofrontal view: (C) Submentovertical view

It is difficult t o examine all the paranasal sinuses on one canthus o f t h e eye towards the film.
projection, so the examination of individual sinus requires The maxillary, ethmoidal and frontal sinuses superimpose
many views. The few standard views that are taken, which each other but this film is useful for the following purposes:
give an adequate idea a b o u t t h e c o n d i t i o n o f paranasal To demonstrate the extent of pneumatization of the
sinuses are as follow: sphenoid and frontal sinuses.
Occipitomental view (Waters view): The X-ray is taken To demonstrate the position of a radiopaque foreign
in t h e nose-chin position w i t h an open m o u t h . The film
body in the nasal cavity or nasopharynx.
demonstrates mainly t h e maxillary sinuses, nasal cavity,
To demonstrate the thickness o f soft tissues of the
septum, frontal sinuses and few cells o f t h e ethmoids. The
nasopharynx w h i c h should n o t n o r m a l l y be more
view taken in the standing position may show fluid level in
than 5 m m .
the antrum (Fig. A)
To show the nasopharyngeal airway.
O c c i p i t o f r o n t a l v i e w ( C a l d w e l l v i e w ) : The p a t i e n t ' s
To demonstrate the adenoid mass or a t u m o r in the
forehead and t i p o f t h e nose are kept in contact w i t h the film.
nasopharynx.
This view is particularly useful for fontal sinuses. A portion of
the maxillary antrum and nasal cavity are also shown (Fig. B) Lateral oblique view for ethmoids: If the disease involves
X-ray, the base o f t h e skull (Submentovertical view): The the ethmoids, a special lateral oblique view provides an
neck and head are fully extended so that vertex faces the idea about the ethmoidal air cells, being relatively free of
f i l m and the rays are directed beneath the mandible. The superimposition by other structures.
view is useful for demonstrating sphenoid sinuses, ethmoids, On plain radiography, the normal sinuses appear as air filled
nasopharynx, petrous apex, posterior wall o f t h e maxillary translucent cavities. Opacity o f t h e sinuses can be caused by
sinus and fracture o f t h e zygomatic arch (Fig. C) fluid, thickened mucosa or tumors. Bony erosion can occur
Lateral view:The patient's head is placed in a lateral position because of tumors, osteomyelitis or mucoceles.
against the f i l m and the ray is directed behind the outer

PHARYNX
Waldeyers ring 2. Blood supply of tonsil
Externa! - Retropharyngeal nodes
f Adenoids Lateral Maxillary artery
pharyngeal band

Secondary crypt

Tonsillar artery
Palatine
tonsil
Lingual Facial artery
Jugular
chain of nodes tonsil
Submental nodes External
Submandibular nodes -Dorsal lingual branches of
carotid artery
lingual artery
Color Plate

IMPORTANT INCISIONS AND POSITION IN ENT SURGERY


1. A b b e estander flap 2. Rose Position

Used for lip reconstruction Used during I.Tonsillectomy II. Abenoidectomy III.Tracheostomy

INSTRUMENTS
1. Head mirror 3. Aural speculum

*
Section VIII Color Plates

6. Tuning fork 12. Mastoid retractor

17. Correct method of holding Thudicum's nasal speculum


Color Plate

18. St Clair-Thompson's nasal speculum 26. Antral burr

27. Antral wash cannula

19. Posterior rhinoscopy mirror

20. Nasal foreign body hook

21. Nasal packing, forceps

23. Antral cannula

^ - 'CO 31. Long-bladed nasal speculum

24. Antral perforator

25. Myle's nasoantral perforator


Color Plate XV

41. Anterior commissure larynogoscope 46. Esophageal speculum

47. Laryneal forceps

42. Negus bronchoscope


48. Crocodile punch biopsy foreps

43. Chevalier-Jackson bronchoscope


49. Boyle-Davis mouth gag

3<L

44. Chevalier-Jackson esophagoscope

50. Tongue plate with throat suction

45. Negus esophagoscope


XVI Section VIII Color Plates

51. Tonsil holding forceps 58. Fuller's tracheostomy tube

59. Jackson's tracheostomy tube


52. Tonsillar suction

53. Tonsil pillar retractor and dissector

54. Tonsillar snare

55. Guillotine
60. Blunt tracheal hook

r
61. Sharp tracheal hook

r
56. Adenoid curette with cage 62. Draffin bipod stand with plate

57. Peritonsillar abscess drainage forceps


ENT
SELF A S S E S S M E N T &. R E V I E W

The 5th edition of the book covers the entire ENT in a holistic yet focussed approach to
cater the needs of PG aspirants. After a high yield synopsis of topics in each chapter,
there are detailed explanations of the MCQ's from AIIMS (2000-2013), All India
(2000-2011) and PGI (2000-2013). The new edition of the book also includes
explanatory Questions from DNB and FMGE.
Keeping in mind the recent trend, NEET pattern questions and color plates with all
important figures and instruments are included, to increase the utility of the book.
Must Read for:
• Undergraduates
• Foreign medical graduates
• Interns
• All post graduate medical aspirants
• Any exam of ENT
Salient Features:
• Best selling book on ENT
• Contains lucid presentation of text in a new layout
• Includes recent AIIMS and PGI Questions (2013)
• Includes DNB, FMGE and NEET pattern questions.
• Hot Topic-Snoring and sleep apnea included
• Color plates with all important illustrations and instruments are given in a separate
section.

Sakshi Arora MBESS D G O is a consultant Obstetrician and Gynecologist in I §


New Delhi. She has done her post graduation from MLN Medical college, f
Allahabad. She has been a very popular and prominent faculty for various 9
mm \
coaching classes preparing students for PG entrance examinations. She is i j r *
presently working as Chief Development Editor for a prestigious publishing ^ ^
house. In her capacity as Chief Development Editor, she has helped
eminent authors in the revision of various textbooks.

Available at all medical book stores


or buy directly from Jaypee Brothers through online shopping
at www.jaypeebrothers.com

A JAYPEE BROTHERS
M Medical Publishers (P) Ltd.
JAYPEE www.jaypeebrothers.com

You might also like